Download as pdf or txt
Download as pdf or txt
You are on page 1of 655

INVENTORY (IAS 2)

Definition of inventory
Inventories are:
 Assets held for sale. For a retailer, these are items that the business sells – its stock-in trade.
For a manufacturer, assets held for sale are usually referred to as ‗finished goods‘
 Assets in the process of production for sale (‗work-in-progress‘ for a manufacturer)
 Assets in the form of materials or supplies to be used in the production process (‗raw
materials‘ in the case of a manufacturer).There are several rules in IAS 2 dealing with the
valuation (measurement) of inventory in the statement of financial position.

Inventories per IAS 2 comprise:


a) Merchandise
b) Production Supplies
c) Materials
d) Work in Progress
e) Finished Goods

In the case of a service provider, inventories include the costs of the service for which the entity
has not yet recognised the related revenue. These costs consist primarily of the labour and other
costs of personnel directly engaged in providing the service, including supervisory personnel
and attributable overheads. Labour and others costs relating to sales and general administrative
personnel are not included but are recognised as expenses in the period in which they are
incurred.

Valuation: lower of cost and net realisable value


IAS 2 Inventories states that inventories should be valued at the lower of:
 cost and
 net realisable value.
Cost consists of all the costs of purchase, plus the costs of conversion and other costs incurred in
bringing the inventories to their present location and condition.
 Costs of purchase include the cost of the item itself (less any trade discounts) plus import
duties, transport costs and other handling costs.
 Costs of conversion are the ‗internal costs‘ incurred in getting the inventory into its current
state, such as the internal costs incurred in producing finished goods.They include both
direct costs (such as labour and expenses) and a share of production overheads, where
production overhead absorption rates are based on normal levels of activity.

Allowable Costs per IAS 2


The cost of inventories shall comprise all costs of purchase, costs of conversion and other costs
incurred in bringing the inventories to their present location and condition.
a) Costs of purchase comprise purchase price, import duties and other taxes and transport,
handling and other costs directly attributable to the acquisition of finished goods, materials and
services, less trade discounts, rebates and other similar items
b) Costs of conversion include
i) Costs which are directly related to units of production i.e. direct labour, direct expenses and
sub contracted work.

1
ii) Systematic allocation of fixed and variable production overheads incurred in converting
materials into finished goods
c) Other costs can be included in the cost of inventories to the extent incurred in bringing the
inventories to their present location and condition i.e. non-production overheads of designing a
product for a specific customer.
IAS 2 outlines examples of costs which are excluded from the cost of inventories and instead
recognised as expenses in the period in which they are incurred i.e.
a) Abnormal amounts of wasted materials, labour or other production costs;
b) Storage costs unless these costs are necessary in the production process before a further
production stage;
c) Administrative overheads that do not contribute to bringing inventories to their present
location and condition; and
d) Selling costs.

Net realisable value (NRV) is the estimated selling price of the item minus:
 all the (estimated) costs to make the item ready for sale, and
 all the (estimated) costs necessary to make the sale.
NRV < Cost
The principal situations in which net realisable value is likely to be less than cost is where there
has been;
a) An increase in costs or a fall in selling price
b) Physical deterioration of inventories
c) Obsolescence of Products
d) A decision as part of a company‘s marketing strategy to manufacture and sell products at a
loss
e) Errors in production or purchasing

Example 1
Inventory at 31 December is GHC146,000. This includes obsolete inventory costing GHC4,240
which will be given away free to a local children‘s charity.Determine the value of the closing
inventory.
Example 2
Ramona limited‘s year-end inventory amounted to GHC142,800 valued at cost. Included in this
amount is some timber garden furniture which has been damaged by a forklift and is beyond
repair. The cost of this damaged inventory was GHC4,650. Ramona limited sold it to a local
wood chip company for GHC1,200 and incurred transport costs of GHC170. Determine the
value of the closing inventory.
Example 3-
Inventory at 31 December 2014 is GHC243,510. This includes GHC2,410 for items accidentally
destroyed on 3 January 2015 and GHC1,540 which relates to the cost of damaged inventory
which can be reworked at a cost of GHC200 and which can then be sold for GHC1,230.
Determine the value of the closing inventory.
Example 4
Tingling Limited‘s year-end inventory at the 31 December amounted to GHC164,300 valued at
cost. However, some inventory items were damaged prior to year-end and will require repair
work that will cost an estimated GHC1,280. When repaired, the items can be sold for 90% of

2
cost. The cost of these damaged goods was GHC2,400. Determine the value of the closing
inventory.
IAS 2 comments that the practice of writing down inventories below cost to their net realisable
value is consistent with the view that assets should not have a carrying value in the statement of
financial position that exceeds the amount expected to be realised from their sale or use.
Suppose a company has three items of inventories on hand at the year-end. Their costs and
NRVs are as follows:
Item Cost(GHC) NRV(GHC)
1 36 40
2 28 24
3 46 48
110 112
Calculate the closing value of inventory at the year-end?
Solution:
It would be incorrect to compare the total cost of GHC110 with total NRV of GHC112 and state
inventories as GHC110. The comparison should be made for each item of inventory and thus a
value of GHC106 would be attributed to inventories i.e .

Item Cost - GHC NRV - GHC Lower of Cost/NRV - GHC


1 36 40 36
2 28 24 24
3 46 48 46
110 112 106
Measuring the cost of inventory
IAS 2 allows three methods for measuring the cost of inventories.
 Actual cost
 First-in, first-out (FIFO)
 Weighted average cost (AVCO).
Actual cost is used where items can be individually traced. This is usual for high value items.
For example, cars for sale in a car dealer‘s showroom will normally be valued at actual cost in
the financial statements of the car dealer.
Where it is not possible to identify actual cost, a choice is allowed between FIFO and AVCO.
The chosen method should be applied consistently to all similar types of inventories.

Techniques for the Measurement of Cost


Estimation techniques may be used for convenience if the results approximate to actual costs.
Examples of potential estimation methods include:
a) Standard Cost: Cost is based on normal levels of materials and supplies, labour efficiency
and capacity utilisation. They are regularly reviewed and revised where necessary.
b) Retail Method: Cost is determined by reducing the sales value of the inventory by the
appropriate percentage gross margin. The percentage used takes into consideration inventory
that has been marked down to below its original selling price. This method is often used in the
retail industry for measuring inventories of rapidly changing items that have similar margins.
Example
Beauitful Tree (BT) is a new luxury retail company located in the principal streets of Kumasi
Central Market. Its accountant previously worked abroad and is not familiar with international

3
financial reporting standards and has asked you, the trainee accountant, to give advice on the
accounting treatment necessary for the following items;
a) One of BT‘s product lines is beauty products, particularly cosmetics such as lipsticks,
moisturizers and compact make-up kits. BT sells hundreds of different brands of these
products. Each product is quite similar, is purchased at similar prices and has a short lifecycle
before a new similar product is introduced. The point of sale and inventory system in BT is not
yet fully functioning in this department. The sales manager of the cosmetic department is
unsure of the cost of each product but is confident of the selling price and has reliably informed
you that BT, on average, make a gross margin of 65% on each line.

b) BT also sells handbags. BT manufactures their own handbags as they wish to be assured of
the quality and craftsmanship which goes into each handbag. The handbags are manufactured
in the UK in the head office factory which
has made handbags for the last fifty years. Normally, BT manufactures 100,000 handbags a year
in their handbag division which uses 15% of the space and overheads of the head office factory.
The division employs ten people and is seen as being an efficient division within the overall
company.
Required:
In accordance with IAS 2 - Inventories, explain how the items referred to in a) and b) should
be measured

Solution
The retail method can be used for measuring inventories of the beauty products. The cost of the
inventory is determined by taking the selling price of the cosmetics and reducing it by the gross
margin of 65% to arrive at the cost.
The handbags can be measured using standard cost especially if the results approximate cost.
Given that BT has the information reliably on hand in relation to direct materials, direct labour,
direct expenses and overheads, it would be the best method to use to arrive at the cost of
inventories.

Example 3
On 1st June 2015 a company held 400 units of finished goods valued at GHC22 each. During
June, the following transactions took place
Date Units Purchased Cost per Unit
10/06/15 300 GHC23
20/06/15 400 GHC24
25/06/15 500 GHC25
Goods sold out of inventories during December were as follows:
Date Units Sold Sales Price per Unit
14/06/15 600 GHC30.00
21/06/15 400 GHC31.00
28/06/15 100 GHC32.00
Required: Calculate the value of closing inventories using
a) FIFO
b) Weighted Average Cost

4
Disclosure requirements for inventory
IAS 2 requires the following disclosures in notes to the financial statements.
 The accounting policy adopted for measuring inventories, including the cost valuation
method used.
 The total carrying amount of inventories, classified appropriately. (For a manufacturer,
appropriate classifications will be raw materials, work-in progress and finished goods.)
 The amount of inventories carried at net realisable value or NRV.
 The amount of inventories written down in value, and so recognised as an expense
during the period.
 Details of any circumstances that have led to the write-down of inventories to NRV.

QUESTION 1
Purchases of raw materials
Purchases: 1,000 tonnes of raw materials per week
Price: GHC100 per tonne on 1 January, increasing to
GHC150 per tonne on 1 July
Import duties: GHC10 per tonne
Transport from docks to factory: GHC20 per tonne
Production costs
Production capacity: 1,500 tonnes per week
Variable costs: GHC25 per tonne
Fixed costs: GHC30,000 per week
Sales details
Selling price: GHC240 per tonne
Delivery costs to customers: GHC8 per tonne
Selling costs: GHC4 per tonne
Inventories at 31 December 2011
Raw materials: 2,000 tonnes
Finished goods: 2,000 tonnes
Required
Calculate and disclose the value of inventories on 31 December 2011 in accordance with IAS 2.

Solution:
GHC
Raw materials (2,000 × 173 (W2)) 346,000
Finished goods (2,000 × 228 (W2)) 456,000
–––––––
802,000
–––––––
Workings
(1) Cost per unit
GHC
Raw materials 150
Import duties 10
Transport costs (to present location and condition) 20
Total raw materials cost 180
Variable costs 25

5
Fixed costs (GHC30,000 ÷ 1,000) (based on normal level of activity) 30
––––
Total finished goods cost 235
––––

(2) Net realisable value Raw materials:


GHC GHC
Estimated selling price 240
Minus all costs to make item ready for sale:
Variable (W1) 25
Fixed (W2) 30
–––– (55)
Minus all costs necessary to make the sale
Delivery costs 8
Selling costs 4
–––– (12)
––––
NRV 173
––––
Finished goods:
GHC
Estimated selling price 240
Minus all costs necessary to make the sale (as above) (12)
––––
228
––––
QUESTION 2
Esinam Ltd has the following products in inventory at the end of 2016:
Units Cost per unit GH¢
Ahomka (completed) 5,400 22
Adonko (part complete) 2,800 26
Each product normally sells at GH¢34 per unit. Due to the difficult trading conditions,Esinam
Ltd intends to offer a discount of 15% per unit and expects to incur GH¢4 per unit in selling
costs. GH¢10 per unit is expected to be incurred to complete each unit of Adonko.
Required:
In accordance with IAS 2 Inventories, at what amount should inventory be stated in the
financial statements of Esinam Ltd as at 31 December 2016? (3 marks)
ICAG NOV.2017
Solution:
No. Cost NRV GH¢
Ahomka 5,400 22 34*0.85=28.9- 4 =24.9 118,800
Adonko 2,800 26 34*0.85=28.9-4-10 =14.9 41,720
160,520

6
QUESTION 3
Saboba Ltd (Saboba) manufactures plastic water tanks for the farming industry. On 31 May
2018, its closing inventory consisted of 950kg of plastic resin raw material, and also 250 finished
units (plastic water tanks). Further information is provided as follows:

i) Plastic: The purchase price of plastic resin was GH¢3 per kg throughout the year to 31 May
2018, plus an additional GH¢0.50 per kg of delivery cost. Saboba has a policy of always
keeping plenty of plastic resin in inventory, as its supply can be unreliable. However, close to
the year-end, the price of plastic resin reduced due to supply exceeding demand. The
purchase price of Saboba‟s raw material is now GH¢2.10 per kg plus the GH¢0.50 per kg
delivery charge. The existing inventory of plastic resin can be sold in the market for
GH¢1.80 per kg net of all costs.

ii) Tanks: Each tank requires 10 kg of plastic to manufacture, and each unit incurs GH¢25 in
conversion costs (labour and overhead). Saboba sells the tanks for GH¢100. It is expected
that this price will drop to GH¢90 as a result of the fall in the market price of plastic. All
completed units sold by Saboba incur a GH¢6 selling and distribution cost.
Required:
Calculate the value of closing inventory in the books of Saboba Ltd at 31 May 2018 applying
the principles ofIAS 2: Inventories. (5 marks)

Solution:
a) The inventory of Saboba Ltd should be valued as follows:
Finished goods:
Cost per unit: GH¢
Material – 10kg *GH¢3 30
Conversion 25
Total 55

Net realisable value:


Expected selling price 90
Less selling costs estimate (6)
Net realisable value 84

As the Net Realisable Value exceeds the cost, the finished goods are valued in the books
at cost. Hence a value of 250 *GH¢55 =GH¢13,750 will be entered into the books as
closing inventory of finished goods.

Raw Material: GH¢


Cost per unit:
Purchase price 3.00
Delivery costs 0.50
Total cost 3.50

7
Net realisable value:
Expected sale proceeds if sold as inventory 1.80
Expected sale proceeds if sold as finished units 90
Less selling cost (6)
Less completion costs: Conversion (25)
Net realisable value 59
NRV per kg of raw material GH¢5.90

The NRV of the raw material if sold as raw material is lower than the cost. However the
NRV if processed into finished units is higher than cost. Therefore the inventory should
not be written down, and should be recorded in the books at cost. Hence a value of 950
*GH¢3.50 =GH¢3,325 will be entered into the books as closing inventory. Total closing
inventory = 3,325 + 13,750 =GH¢17,075.

8
ACCOUNTING POLICIES, CHANGES IN
ACCOUNTING ESTIMATES AND ERRORS (IAS 8)
Objective
The objective of this Standard is to prescribe the criteria for selecting and changing accounting
policies,together with the accounting treatment and disclosure of changes in accounting
policies, changes in accounting estimates and corrections of errors. The Standard is intended to
enhance the relevance and reliability of an entity‘s financial statements, and the comparability
of those financial statements over time and with the financial statements of other entities.
Disclosure requirements for accounting policies, except those for changes in accounting policies,
are set out in IAS 1 Presentation of Financial Statements.

Scope
This Standard shall be applied in selecting and applying accounting policies, and accounting
for changes in accounting policies, changes in accounting estimates and corrections of prior
period errors.
The tax effects of corrections of prior period errors and of retrospective adjustments made to
apply changes in accounting policies are accounted for and disclosed in accordance with IAS 12
Income Taxes.

Accounting policies
Definition :
Accounting policies are defined in IAS 8 as the ‗specific principles, bases, conventions, rules and
practices applied by an entity in preparing and presenting financial statements‘.
The selection and application of accounting policies The selection and application of accounting
policies is covered by IAS 8 Accounting policies, changes in accounting estimates and errors.
IAS 8 includes requirements for the selection and application of accounting policies:
 When an item (for example, a transaction) is covered by an accounting standard, or by an
Interpretation of a standard, the accounting policy applied to the item should comply with
the requirements of the Standard or Interpretation.
 When an item is not covered by an accounting standard, or by an Interpretation of a
Standard, management should use judgement in selecting the accounting policy to apply.

The IASB Framework is not itself an accounting standard. If there is a conflict between an
accounting standard (or an Interpretation) and the Framework, management must follow the
standard.
When judgement is used to select an accounting policy, the financial information resulting from
the application of the accounting policy must be:
 relevant to the economic decision-making needs of the users of the financial statements, and
 reliable.
In making a judgement about the selection of an accounting policy, management should refer to
the following, in descending order of priority, and consider whether they are applicable: „
 the requirements of accounting standards or guidance from Interpretations that deal with
similar and related issues
 definitions, recognition criteria and measurement concepts for assets, liabilities, income and
expenses in the IASB Conceptual Framework.

9
In making the judgement described above, management may also consider the most recent
pronouncements of other standard-setting bodies that use a similar conceptual framework to
develop accounting standards, other accounting literature and accepted industry practices, to
the extent that these do not conflict with International accounting Standards (Interpretations),
or the IASB Conceptual Framework.
CONSISTENCY OF ACCOUNTING POLICIES
An entity should select and apply its accounting policies consistently for similar transactions,
events or conditions, unless an accounting standard requires or permits:
 some items to be categorised separately, and „
 a different accounting policy to be applied to this category.
CHANGES IN ACCOUNTING POLICIES
A change in accounting policy is permitted only under either of two circumstances
(IAS 8): „
 if a change in policy is required by an accounting standard or Interpretation, or
 if a change in accounting policy will result in reliable and more relevant financial
information.
A change in an accounting policy is dealt with retrospectively, that is the accounts of all
previous years presented are amended to show the financial information as if the entity had
always followed the new policy.
Users of financial statements need to be able to compare financial statements of an entity over
time, so that they can identify trends in its financial performance or financial position. Frequent
changes in accounting policies are therefore undesirable because they make comparisons with
previous periods more difficult. IAS 8 therefore states that a change in accounting policy is
permitted only in the following circumstances: „
 When a change in accounting policy is required by an IFRS or revised IAS (or an
Interpretation of an IFRS), or „
 If a change in accounting policy results in the financial statements providing reliable and
more relevant financial information.
When a change in accounting policy is required by a new Standard, the Standard will often
include specific ‘transitional provisions’. These explain how the change should be introduced.
In the absence of specific transitional provisions, a change in policy should be applied
retrospectively.
ACCOUNTING POLICIES AND COMPARABILITY
In order to be able to compare the financial statements of different entities and the financial
statements of the same entity over time, users need information about the accounting policies
used by an entity. They also need information about any changes to accounting policies and the
effects of those changes.
Frequent changes of accounting policies do not improve the information provided to users (and
are effectively prohibited by IAS 8). However, an entity should not continue to use the same
accounting policy for the sake of comparability if there is an alternative that would produce
more relevant or more reliable information.
DETERMINING WHEN THERE IS A CHANGE IN ACCOUNTING POLICY
How should an entity decide whether it would be introducing a change in accounting policy, if
it decided to account for a particular type of transaction in a particular way?
IAS 8 specifies that when a new accounting policy is applied for transactions or events that did
not occur previously, in earlier financial periods, this is not a change of accounting policy; it
would simply be the application of a suitable accounting policy to a new type of transaction.

10
A change in accounting policy can be established as follows. The accounting policies chosen by
an entity should reflect transactions and events through: „
 recognition (e.g. capitalising or writing off certain types of expenditure)
 measurement (e.g. measuring non-current assets at cost or valuation)
 presentation (e.g. classification of costs as cost of sales or administrative expenses)
If at least one of these criteria is changed, then there is a change in accounting policy.

Example
A company has previously written off borrowing costs as incurred in the income statement. It is
now proposed that any relevant finance costs should be capitalised as allowed by IAS 23.
This affects: „
 recognition – the interest cost is now recognised as an asset rather than an expense „
 presentation – the interest cost is now presented in the statement of financial position
rather than the statement of comprehensive income.
In this example, there has been no change to the measurement of the finance costs but this is
still a change in accounting policy due to a change in recognition and presentation.
RETROSPECTIVE APPLICATION OF A CHANGE IN ACCOUNTING POLICY
When a change in accounting policy is required, and there are no transitional provisions
relating to the introduction of a new accounting standard, the change in policy should be
applied retrospectively.
The entity should adjust the opening balance for each item of equity affected by the change, for
the earliest prior period presented, and the other comparative amounts for each prior period, as
if the new accounting policy had always been applied. For example, suppose that a change in
accounting policy is required from Year 3 onwards, and the earliest prior period presented in
the Year 3 financial statements is Year 2, for which comparative prior year figures are presented.
The change in accounting policy should be applied retrospectively, which means that the
change should be applied to the opening balances at the start of Year 2 (in the comparative prior
period information), as if the new policy had always been applied.
Similarly, any other comparative amounts in previous periods should be adjusted as if the new
accounting policy had always been applied.
If this is impracticable, retrospective application should be applied from the earliest date that is
practicable.
DISCLOSURE OF A CHANGE IN ACCOUNTING POLICY
When an entity makes a change in an accounting policy, a note to the financial statements
should disclose the following information about the change:
IF THE CHANGE IS DUE TO A NEW IF THE CHANGE IN POLICY IS
STANDARD OR A NEW VOLUNTARY
INTERPRETATION
 The title of the Standard or
Interpretation
 The nature of the change in accounting  The nature of the change in accounting
policy policy

 A description of any transitional  The reason why the new accounting


provisions policy provides reliable and more
relevant information

11
 For the current and previous period(s),  For the current and previous period(s),
to the extent practicable, the amount of to the extent practicable, the amount of
the adjustment to each item in the the adjustment to each item in the
financial statements and the financial statements and the
adjustment to the basic and fully adjustment to the basic and fully
diluted earnings per share diluted earnings per share

 To the extent practicable, the  To the extent practicable, the


adjustment relating to accounting adjustment relating to accounting
periods before those presented in the periods before those presented in the
financial statements financial statements

 If retrospective application is  If retrospective application is


impracticable, an explanation of how impracticable, an explanation of how
the accounting policy change has been the accounting policy change has been
applied applied

ACCOUNTING ESTIMATES
An accounting estimate is made for an item in the financial statements when the item cannot be
measured with precision, and there is some uncertainty about it. An estimate is therefore based,
to some extent, on management‘s judgement. Management estimates might be required, for
example, for the following items:
 bad debts
 inventory obsolescence
 the fair value of financial assets or liabilities
 the useful lives of non-current assets
 the most appropriate depreciation pattern (depreciation method) for a category of non-
current assets
 warranty obligations.

It is important to distinguish between an accounting policy and an accounting estimate.


As an example a company may have an accounting policy to depreciate plant and equipment
over its useful life. However whether the company uses the straight line method of depreciation
or the reducing balance method will be a choice of accounting estimate.
IAS 8 requires a change in an accounting policy to be accounted for retrospectively whereas a
change in an accounting estimate is normally recognised in the current period (and there is no
requirement for retrospective application).
CHANGES IN ACCOUNTING ESTIMATES
A change in accounting estimate may be needed if changes occur in the circumstances on which
the estimate was based, or if new information becomes available. A change in estimate is not the
result of discovering an error in the way an item has been accounted for in the past and it is not
a correction of an error.
The effect of a change in accounting estimate should be recognised prospectively, by including
it:
 in profit or loss for the period in which the change is made, if the change affects that
period only, or

12
 in profit or loss for the period of change and future periods, if the change affects both.
PROSPECTIVE APPLICATION OF A CHANGE IN ESTIMATE
A change in accounting estimate is not applied retrospectively. There is prospective application
of the change. This means that the effect of the change is recognised in the current period and
the future periods affected by the change.
To the extent that a change in estimate results in a change in assets and liabilities, it should be
recognised by adjusting the carrying amount of the affected assets or liabilities in the period of
change.
Example
A non-current asset was purchased for GHC200,000 two years ago, when its expected economic
life was ten years and its expected residual value was GHC0. The asset is being depreciated by
the straight-line method.
A review of the non-current assets at the end of year 2 revealed that due to technological
change, the useful life of the asset is only six years in total, and the asset therefore has a
remaining useful life of four years.
The original depreciation charge was GHC20,000 per year (GHC200,000/10 years) and at the
beginning of Year 2, its carrying value was GHC180,000 (GHC200,000 - GHC20,000).
The change in the estimate occurs in Year 2. The change in estimate should be applied
prospectively, for years 2 onwards (years 2 – 6). From the beginning of year 2, the asset has a
revised useful remaining life of five years.
The annual charge for depreciation for year 2 (the current year) and for the future years 3 – 6
will be changed from GHC20,000 to GHC36,000 (= GHC180,000/5 years).
Example
The carrying value of an item of inventory is GHC7,000. The inventory is no longer used due to
a change in materials specifications for the products that used to contain the inventory item.
The inventory has no scrap value and is now considered worthless.

A change in accounting estimate should reduce the value of the inventory to zero. The change
affects the current year only, and the write-off of GHC7,000 should be treated as an expense for
the year.
ERRORS
Errors might happen in preparing financial statements. If they are discovered quickly, they are
corrected before the finalised financial statements are published. When this happens, the
correction of the error is of no significance for the purpose of financial reporting.
A problem arises, however, when an error is discovered that relates to a prior accounting
period. For example, in preparing the financial statements for Year 3, an error may be
discovered affecting the financial statements for Year 2, or even Year 1.
Errors might be:
 the effect of a mathematical mistake
 a mistake in applying an accounting policy
 an oversight
 a misinterpretation of facts
 caused by fraud.
Prior period errors are defined in IAS 8 as: ‗omissions from, and misstatements in, the entity‘s
financial statements for one or more prior periods arising from a failure to use, or misuse of,
reliable information that:
 was available when financial statements for those periods were authorised for issue; and

13
 could reasonably be expected to have been obtained and taken into account in the
preparation and presentation of those financial statements.‘
THE CORRECTION OF PRIOR PERIOD ERRORS
IAS 8 states that all material prior period errors should be corrected retrospectively in the first
set of financial statements following the discovery of the error.
Comparative amounts for the previous period should be re-stated at their corrected amount.
If the error occurred before the previous year, the opening balances of assets, liabilities and
equity for the previous period should be re-stated at their corrected amount.
The correction of a prior period error is excluded from profit or loss in the period when the
error was discovered.
For example, suppose that an entity preparing its financial statements for Year 3 discovers an
error affecting the Year 2 financial statements. The error should be corrected in the Year 3
financial statements by re-stating the comparative figures for Year 2 at their correct amount. If
the error had occurred in Year 1, the comparative opening balances for the beginning of Year 2
should be re-stated at their correct amount. The reported profit for Year 3 is not affected.
Example
Didier Plc is preparing its financial statements for 2013. The draft profit for 2013 is GHC547,000
before tax. During 2013 Didier Plc:
(a) paid dividends of GHC100,000
(b) revalued a non-current asset, creating a revaluation reserve of GHC40,000
(c) raised GHC300,000 by issuing new shares (GHC200,000 nominal value and GHC100,000
share premium).
At the end of 2012 owners‘ equity totalled GHC890,000, consisting of share capital of
GHC500,000, share premium GHC50,000, revaluation reserve GHC100,000 and retained
earnings of GHC240,000.
(At the end of Year 1, owners‘ equity totalled GHC740,000; retained profits for the year were
GHC150,000 and there were no dividend payments, no share issues and no non- current asset
revaluations during 2011.)
DEF has now discovered an error in its inventory valuation. This has resulted in an
overstatement of inventory at 31 December 2013 of GHC67,000 and at 31 December 2012 of
GHC60,000.
The rate of tax on profits was 30% in both 2012 and 2013.
The error in 2012 should be corrected retrospectively, as follows.
Retrospective correction of the statement of comprehensive income
The error in the valuation of opening and closing inventory in 2013 is corrected before the
publication of the 2013 financial statements. As a result of this correction, the draft profit for
2013 before tax will be GHC547,000 - GHC67,000 + GHC60,000 = GHC540,000. After deducting
tax at 30%, the profit is GHC378,000.
The comparative figures for 2012 should also be changed, and because closing inventory at the
end of 2012 was overstated by GHC60,000, the profit before tax in 2012 was actually GHC60,000
lower. Profit after tax at 30% would therefore be GHC42,000 lower (and the tax liability
GHC18,000 lower).
In the statement of comprehensive income for 2013,
the comparative figures for 2012 should therefore be re-stated to show cost of sales higher by
GHC60,000, gross profit lower by GHC60,000, taxation lower by GHC18,000 and profit after tax
lower by GHC42,000.

14
Statement of changes in equity
The reduction in profit in 2012 affects some of the opening balances in the statement of financial
position in 2013. The tax liability will be GHC18,000 lower and retained profits will be
GHC42,000 lower. Restating the opening balance for the current year the opening shareholders‘
funds will now be GHC848,000 (GHC890,000 – GHC42,000).

The statement of changes in equity for the year would be shown as follows:
Share Share Revaluation Retained Total
capital premium reserve earnings
GHC000 GHC000 GHC000 GHC000 GHC000
Balance at 31
December,2011 500 50 100 90 740
Profit for the year
ended
31 December,2011,
as re-stated - - - 108 108
–––– –––– –––– –––– ––––
Balance at 31
December,2012 500 50 100 198 848
2013
Dividends (100) (100)
Issue of shares 200 100 300
Profit for the year 378 378
Gain on property
revaluation 40 40
–––– –––– –––– –––– ––––
Balance at
31 December2013 700 150 140 476 1,466
–––– –––– –––– –––– ––––
In this statement, the retained profit for the year has been re-stated as GHC108,000, to correct
the error in 2012, and the opening balance of retained profits is also re- stated. There is no re-
statement in 2013 because the error was corrected before the final financial statements were
published.
DISCLOSURE OF PRIOR PERIOD ERRORS
IAS 8 requires the following disclosures for material prior period errors: the nature of the prior
period error for each period presented in the financial statements, and to the extent practicable,
the amount of the correction for each financial statement item and the change to basic and fully
diluted earnings per share the amount of the correction at the beginning of the earliest prior
period in the statements (typically, at the start of the previous year) if retrospective re-
statement is not practicable for a prior period, an explanation of how and when the error has
been corrected.
IAS 8 therefore requires that a note to the financial statements should disclose details of the
prior year error, and the effect that the correction has had on ‗line items‘ in the Year 2 figures. In
the previous example, the effect of the error should be disclosed in a note to the financial
statements as:

15
Effect on 2012
GHC
(Increase) in cost of goods sold (60,000)
Decrease in tax 18,000
–––––––
(Decrease) in profit (42,000)
–––––––
(Decrease) in closing inventory (60,000
Decrease in tax payable 18,000
–––––––
(Decrease) in equity (42,000)
–––––––

REVIEW QUESTIONS
QUESTION 1
The following issue has arisen during the preparation of Sarpong‘s draft financial statements for
the year ended 31 March 2014:
From 1 April 2013, the directors have decided to reclassify research and amortised development
costs as administrative expenses rather than its previous classification as cost of sales. They
believe that the previous treatment unfairly distorted the company‘s gross profit margin.
Required:
Advise how the above item should be treated in Sarpong‘s financial statements for the year
ended 31 March 2014. (3 marks)
Solution:
Changing the classification of an item of expense is an example of a change in accounting
policy, in accordance with IAS 8 Accounting Policies, Changes in Accounting Estimates and
Errors.
Such a change should only be made where it is required by an IFRS or where it would lead to
the information in the financial statements being more reliable and relevant. It may be that this
change does represent an example of the latter, although it is arguable that amortised
development costs should continue to be included in cost of sales as amortisation only occurs
when the benefits from the related project(s) come on-stream.
If it is accepted that this change does constitute a change of accounting policy, then the
proposed treatment by the directors is acceptable; however, the comparative results for the year
ended 31 March 2013 must be restated as if the new policy had always been applied (known as
retrospective application).

QUESTION 2
In March 2012, Fabiano‘s internal audit department discovered a fraud committed by the
company‘s credit controller who did not return from a foreign business trip. The outcome of the
fraud is that GHC4 million of the company‘s trade receivables have been stolen by the credit
controller and are not recoverable. Of this amount, GHC1 million relates to the year ended 31
March 2011 and the remainder to the current year. Fabiano is not insured against this fraud.
Required:
Explain briefly how this issue should be treated in Fabiano‘s 2012 financial statement.

16
Solution:
The discovery of the fraud represents an error part of which is a prior period adjustment (GHC1
million) in accordance with IAS 8 Accounting policies, changes in accounting estimates and errors.
The following entries need to be passed:
 Credit Trade Receivables with the whole GHC4 million
 Debit Retained Earnings balance from 2011 with GHC1 million as a prior period
adjustment
 Debit administrative expenses with GHC3 million

QUESTION 3
According to IAS 8 Accounting Policies, Changes in Accounting Estimates and Errors, an entity
must select and apply its accounting policies consistently from one period to the next and
among various items in the financial statements. However, an entity may change its accounting
policies under certain conditions.
Required:
Identify the circumstances under which it may be appropriate to change accounting policy in
accordance with the guidance given in IAS 8 Accounting Policies, Changes in Accounting
Estimates and Errors. (2 marks)
ICAG NOV,2016
Solution:
Change of accounting policy Under IAS 8 it is appropriate to change accounting policy in two
situations.
If a new standard or interpretation requires a change form an existing policy, and
If a different policy would produce financial statements that are more relevant than the existing
policy. In other words, the proposed policy would result in financial statements that reflect the
financial position and performance of the entity more fairly.
QUESTION 4
a) Talensi, a company reporting under IFRS, is considering making the following changes to its
financial statements for the year ended 31 December 2017. Talensi presents one year of
comparative information.

i) Changing the method of depreciation of its plant from straight line depreciation over five
years (with a nil residual value) to reducing balance at 20% per annum with effect from1
January 2017. The plant originally cost GH¢100 million on 1 January 2015.

ii) Changing the basis of valuation of certain non-seasonal inventories from first-in first-out
(FIFO) to weighted average cost (WAC). Inventories were valued as follows under the two
different methods:
31 December 2015 31 December 2016 31 December 2017
GH¢ million GH¢ million GH¢ million
FIFO 64 66 71
WAC 62 63 67

iii) Changing the revenue recognition basis for certain seasonal goods that were first sold in 2015
such that revenue is recognised on delivery to the customer rather than on shipment. This has

17
arisen as a result of a change in delivery arrangements such that, with effect from 1 January
2017, risks are now borne by Talensi until delivery has been made to the customer.
2015 2016 2017
GH¢ million GH¢ million GH¢ million
Revenue based on shipment date 50 86 90
Revenue based on delivery date 46 84 88

The cost of the seasonal goods is consistently 80% of sales price.

Profit (calculated using existing policies and accounting estimates) was GH¢240 million for
the year ended 31 December 2017.

Required:
Calculate the adjustment to opening retained earnings in the statement of changes in equity
(including 2016 comparative figures) in the financial statements for the year ended 31
December 2017; and profit or loss for the year ended 31 December 2017. (6 marks)

Solution:

Opening retained earnings Profit (1 Jan 2016)


GH¢ million GH¢million
Profit using existing policies - 240

Depreciation method (change in estimate)


Old depreciation 100 / 5 years 20
Revised depreciation (100 x 3/5) x 20% (12)
Inventory valuation (change in policy)
Adj to opening inventories (62 – 64)/(63 – 66) (2) 3
Adj to closing inventories (67 – 71) (4)
Revenue (change in nature not policy, therefore
apply from 1 Jan 2017) (88 – 90) x 20% (0.4)
(2) 246.6

QUESTION 5
QUESTION 3
IAS 8 Accounting Policies, Changes in Accounting Estimates and Errors is applied in
selecting and applying accounting policies, accounting for changes in estimates and reflecting
corrections of prior period errors.
The standard requires compliance with any specific IFRS applying to a transaction, event or
condition, and provides guidance on developing accounting policies for other items that result in
relevant and reliable information.

Required:
i) Discuss the procedure for selecting accounting policies. (3 marks)

18
ii) Recommend how an entity should account for a change in accounting policy. (2 marks)

SOLUTION
b) i) Selection and application of accounting policies
When a Standard or an Interpretation specifically applies to a transaction, other event or
condition, the accounting policy or policies applied to that item must be determined by applying
the Standard or Interpretation and considering any relevant Implementation Guidance issued by
the IASB for the Standard or Interpretation. [IAS 8.7]

In the absence of a Standard or an Interpretation that specifically applies to a transaction, other


event or condition, management must use its judgement in developing and applying an
accounting policy that results in information that is relevant and reliable. [IAS 8.10]. In making
that judgement, management must refer to, and consider the applicability of, the following
sources in descending order:

the requirements and guidance in IASB standards and interpretations dealing with similar and
related issues; and the definitions, recognition criteria and measurement concepts for assets,
liabilities, income and expenses in the Framework. [IAS 8.11]

Management may also consider the most recent pronouncements of other standard-setting
bodies that use a similar conceptual framework to develop accounting standards, other
accounting literature and accepted industry practices, to the extent that these do not conflict with
the sources in paragraph 11. [IAS 8.12]

ii) How an entity should account for a change in accounting policy


If a change in accounting policy is required by a new IASB standard or interpretation, the
change is accounted for as required by that new pronouncement or, if the new pronouncement
does not include specific transition provisions, then the change in accounting policy is applied
retrospectively. [IAS 8.19]
Retrospective application means adjusting the opening balance of each affected component
of equity for the earliest prior period presented and the other comparative amounts disclosed for
each prior period presented as if the new accounting policy had always been applied. [IAS 8.22]
However, if it is impracticable to determine either the period-specific effects or the cumulative
effect of the change for one or more prior periods presented, the entity shall apply the new
accounting policy to the carrying amounts of assets and liabilities as at the beginning of the
earliest period for which retrospective application is practicable, which may be the current
period, and shall make a corresponding adjustment to the opening balance of each affected
component of equity for that period. [IAS 8.24] Also, if it is impracticable to determine the
cumulative effect, at the beginning of the current period, of applying a new accounting policy to
all prior periods, the entity shall adjust the comparative information to apply the new accounting
policy prospectively from the earliest date practicable. [IAS 8.2)

19
EVENTS AFTER THE REPORTING PERIOD
(IAS 10)
PURPOSE OF IAS 10
The financial statements are prepared as at the end of the reporting period, but often the
accounts are not authorised by the directors until some months later. During this time events
may take place within the entity that should be communicated to the shareholders.
IAS 10: Events after the reporting period has two main objectives:
 to specify when an entity should adjust its financial statements for events that occur or
become apparent after the reporting period, but before the financial statements are
authorised for issue, and
 to specify the disclosures that should be given about events that have occurred after the
reporting period but before the financial statements were authorised for issue.

IAS 10 also includes a requirement that the financial statements should disclose when the
statements were authorised for issue, and who gave the authorisation.
ADJUSTING AND NON-ADJUSTING EVENTS
IAS 10 sets out the following key definitions:
Events after the reporting period are events, favourable and unfavourable, that occur between
the end of the reporting period and the date the financial statements are authorised for issue.
There are two types of event after the reporting period:
 Adjusting events: These are events that provide evidence of conditions that already existed
at end of the reporting period.
 Non-adjusting events: These are events that have occurred due to conditions arising after
the end of the reporting period.

ACCOUNTING TREATMENT
Adjust financial statements for adjusting events
Do not adjust for non-adjusting events
If an entity declares dividends after the reporting period, the entity shall not recognize those
dividends as a liability at the end of the reporting period. That is a non-adjusting event.
An entity shall not prepare its financial statements on a going concern basis if management
determines after the reporting period either that it intends to liquidate the entity or to cease
trading.

ACCOUNTING FOR ADJUSTING EVENTS AFTER THE REPORTING PERIOD


IAS 10 states that if an entity obtains information about an adjusting event after the reporting
period, it should amend and update the financial statements to allow for this new information.

20
‗An entity shall adjust the amounts recognised in its financial statements to reflect adjusting
events after the reporting period.‘
This might seem common sense. If conditions existed at the end of the reporting period, it is
reasonable to expect that the financial statements should recognise those conditions, even
though the facts did not become known until later.

IAS 10 gives the following examples of adjusting events:


 The settlement of a court case after the end of the reporting period, confirming that the entity
had a present obligation as at the end of the reporting period as a consequence of the case.
 The receipt of information after the end of the reporting period indicating that an asset was
impaired as at the end of the reporting period. For example, information may be obtained
that indicates the need to make a provision for a bad debt against a trade receivable in the
year-end statement of financial position.
 Similarly, evidence might be obtained after the end of the reporting period indicating that as
at the end of the reporting period the net realisable value of some inventory was less than its
cost, and the inventory should therefore be written down in value in the statement of
financial position. The sale of inventory at less than cost soon after the end of the reporting
period would provide such evidence.
 The determination after the end of the reporting period of the purchase cost of an asset,
where the asset had already been purchased before the end of the reporting period, but the
purchase price had not been finally agreed or decided. Similarly, the determination after the
end of the reporting period of the sale price for a non-current asset, where the sale had been
made before the end of the reporting period but the sale price had not yet been finally
agreed.
 The discovery of fraud or errors showing that the financial statements are incorrect.
Example
At 31 December 2011, Graziano Ltd is involved in a court case. It is being sued by a supplier. On
15 April 2012, the court decided that Grazaino Ltd should pay the supplier GHC45,000 in
settlement of the dispute. The financial statements for Graziano Ltd for the year ended 31
December 2011 were authorised for issue on 17 May 2012.
The settlement of the court case is an adjusting event after the reporting period:
 It is an event that occurred between the end of the reporting period and the date the financial
statements were authorised for issue.
 It provided evidence of a condition that existed at the end of the reporting period. In this
example, the court decision provides evidence that the entity had an obligation to the
supplier at the end of the reporting period.
Since it is an adjusting event after the reporting period, the financial statements for 2011 must
be adjusted to include a provision for GHC45,000. The alteration to the financial statements
should be made before they are approved and authorised for issue.

DISCLOSURES FOR NON-ADJUSTING EVENTS AFTER THE REPORTING PERIOD


Non-adjusting events after the reporting period are treated differently. A non- adjusting event
relates to conditions that did not exist at the end of the reporting period; therefore the financial
statements must not be updated to include the effects of the event. IAS 10 states quite firmly:

21
‗An entity shall not adjust the amounts recognised in the financial statements to reflect non-
adjusting events after the reporting period‘.
However, IAS 10 goes on to say that if a non-adjusting event is material, a failure by the entity
to provide a disclosure about it could influence the economic decisions taken by users of the
financial statements. For material non-adjusting events IAS 10 therefore requires disclosure in a
note to the financial statements of:
 the nature of the event, and
 an estimate of its financial effect, or a statement that such an estimate cannot be made.
(Note: There are no disclosure requirements for adjusting events themselves as they have
already been reflected in the financial statements.)
IAS 10 gives the following examples of non-adjusting events:
 A fall in value of an asset, for example a large fall in the market value of some investments
owned by the entity, between the end of the reporting period and the date the financial
statements are authorised for issue. A fall in market value after the end of the reporting
period will normally reflect conditions that arise after the reporting period has ended, not
conditions already existing before then.
 The acquisition or disposal of a major subsidiary after the reporting period.
 The formal announcement after the reporting period of a plan to discontinue a major
operation.
 Announcing or commencing the implementation of a major restructuring.
 The destruction of a major plant by a fire after the reporting period. The ‗condition‘ is the
fire, not the plant, and the fire didn‘t exist at the end of the reporting period. The plant
should therefore be reported in the statement of financial position at its carrying amount as
at the end of the reporting period. The fire, and the financial consequences of the fire, should
be disclosed in a note to the financial statements.

DIVIDENDS AND THE GOING CONCERN ASSUMPTION


IAS 10 also contains specific provisions about proposed dividends and the going concern
presumption on which financial statements are normally based.
If equity dividends are declared after the end of the reporting period, they should not be
recognised as a liability in the statement of financial position, because they did not exist as an
obligation as at the end of the reporting period.
Dividends proposed after the end of the reporting period should be disclosed in a note to the
financial statements, in accordance with IAS 1.
There is one important exception to the normal rule that the financial statements reflect
conditions at the end of the reporting period. A deterioration in operating results and financial
position after the reporting period has ended might indicate that the going concern assumption
is no longer appropriate. If management decide after the reporting period has ended that it
intends to liquidate the entity or to cease trading, the financial statements for the year just
ended should be prepared on some other basis (for example, on a break-up basis).

PROBLEMS OF ACCOUNTING FOR EVENTS AFTER THE REPORTING PERIOD


An entity may enter into a transaction shortly before the year end and then reverse the
transaction soon after the year end. The aim is to improve the position shown in the year end
accounts. This is known as ‗window dressing‘.
For example, a company may write cheques to suppliers on the last day of the year and enter
the payments in the cash book. The cheques may not be sent out by post until the new financial

22
year has started, so that the entity has not been required to pay actual money from its bank
account. The effect of this may help to improve the reported liquidity position of the entity, as
measured by its current ratio (ratio of current assets to current liabilities). If this ratio is already
higher than 1.0, recording a payment to reduce cash and reduce trade payables will increase the
ratio further. A higher current ratio indicates better liquidity.

Example
An entity has current assets of GHC20,000 and current liabilities of GHC15,000. On the last day
of the year, the entity writes cheques to GHC5,000 of its trade payables but does not send the
cheques to the suppliers until two weeks later.
After recording the payment transactions, current liabilities will be GHC10,000 and current
assets will be GHC15,000, giving a current ratio of 1.5:1. Prior to the transaction, the current
ratio was 1.3: 1.
Therefore, the effect has been to improve the liquidity position of the entity, because the current
ratio has increased and it now appears that the entity has proportionally more assets to cover
their liabilities.
Some commentators have argued that where transactions are deliberately intended to
manipulate the reported financial position, disclosure should be required in the notes to the
financial statements, in order to alert users. Disclosure of the facts and amounts would then
allow the users to allow for the transaction when analysing the financial statements.
However, IAS 10 does not currently address this problem of ‗window dressing‘ the financial
statements and there is no requirement for such disclosures.

REVIEW QUESTIONS
QUESTION 1
(a)The objective of IAS 10 Events after the Reporting Period is to prescribe the treatment of
events that occur after an entity‘s reporting period has ended.
Required:
Define the period to which IAS 10 relates and distinguish between adjusting and non-adjusting
events. (5 marks)
(b) Iniesta‘s current year end is 31 March 2009. Its financial statements were authorised for issue
by its directors on 6 May 2009 and the AGM (annual general meeting) will be held on 3 June
2009. The following matters have been brought to your attention:

(i) On 12 April 2009 a fire completely destroyed the company‘s largest warehouse and the
inventory it contained. The carrying amounts of the warehouse and the inventory were GHC10
million and GHC6 million respectively. It appears that the company has not updated the value
of its insurance cover and only expects to be able to recover a maximum of GHC9 million from
its insurers. Iniesta‘s trading operations have been severely disrupted since the fire and it
expects large trading losses for some time to come. (4 marks)
(ii) A single class of inventory held at another warehouse was valued at its cost of GHC460,000
at 31 March 2009. In April 2009 70% of this inventory was sold for GHC280,000 on which
Iniestas‘ sales staff earned a commission of 15% of the selling price. (3 marks)
(iii) On 18 May 2009 the government announced tax changes which have the effect of increasing
Iniesta‘s deferred tax liability by GHC650,000 as at 31 March 2009.

23
(3 marks)
Required:
Explain the required treatment of the items (i) to (iii) by Iniesta in its financial statements for the
year ended 31 March 2009.
Note: assume all items are material and are independent of each other.
(10 marks as indicated)
(15 marks)

Solution:
(a)Events after the reporting period are defined by IAS 10 Events after the Reporting Period as
those events, both favourable and unfavourable, that occur between the end of the reporting
period and the date that the financial statements are authorised for issue (normally by the Board
of directors).
An adjusting event is one that provides further evidence of conditions that existed at the end of
the reporting period, including an event that indicates that the going concern assumption in
relation to the whole or part of the entity is not appropriate. Normally trading results occurring
after the end of the reporting period are a matter for the next reporting period, however, if there
is an event which would normally be treated as non-adjusting that causes a dramatic downturn
in trading (and profitability) such that it is likely that the entity will no longer be a going
concern, this should be treated as an adjusting event.
A non-adjusting event is an event after the end of the reporting period that is indicative of a
condition that arose after the end of the reporting period and, subject to the exception noted
above, the financial statements would not be adjusted to reflect such events.
The outcome (and values) of many items in the financial statements have a degree of
uncertainty at the end of the reporting period. IAS 10 effectively says that where events
occurring after the end of the reporting period help to determine what those values were at the
end of the reporting period, they should be taken in account (i.e. adjusted for) in preparing the
financial statements.
If non-adjusting events, whilst not affecting the financial statements of the current year, are of
such importance (i.e. material) that without disclosure of their nature and estimated financial
effect, users‘ ability to make proper evaluations and decisions about the future of the entity
would be affected, then they should be disclosed in the notes to the financial statements.
(b) (i) This is normally classified as a non-adjusting event as there was no reason to doubt that
the value of warehouse and the inventory it contained was worth less than its carrying amount
at 31 March 2009 (the last day of the reporting period). The total loss suffered as a result of the
fire is GHC16 million. The company expects that GHC9 million of this loss will be recovered
from an insurance policy. Recoveries from third parties should be assessed separately from the
related loss. As this event has caused serious disruption to trading, IAS 10 would require the
details of this non-adjusting event to be disclosed as a note to the financial statements for the
year ended 31 March 2009 as a total loss of GHC16 million and the effect of the insurance
recovery to be disclosed separately.
The severe disruption in Iniesta‘s trading operations since the fire, together with the expectation
of large trading losses for some time to come, may call in to question the going concern status of
the company. If it is judged that Iniesta is no longer a going concern, then the fire and its
consequences become an adjusting event requiring the financial statements for the year ended
31 March 2009 to be redrafted on the basis that the company is no longer a going concern (i.e.
they would be prepared on a liquidation basis).

24
(ii) 70% of the inventory amounts to GHC322,000 (460,000 x 70%) and this was sold for a net
amount of GHC238,000 (280,000 x 85%). Thus a large proportion of a class of inventory was sold
at a loss after the reporting period. This would appear to give evidence of conditions that
existed at 31 March 2009 i.e. that the net realisable value of that class of inventory was below its
cost. Inventory is required to be valued at the lower of cost and net realisable value, thus this is
an adjusting event. If it is assumed that the remaining inventory will be sold at similar prices
and terms as that already sold, the net realisable value of the whole of the class of inventory
would be calculated as:
GHC280,000/70% = GHC400,000, less commission of 15% = GHC340,000.
Thus the carrying amount of the inventory of GHC460,000 should be written down by
GHC120,000 to its net realisable value of GHC340,000.
In the unlikely event that the fall in the value of the inventory could be attributed to a specific
event that occurred after the date of the statement of financial position then this would be a
non-adjusting event.
(iii) The date of the government announcement of the tax change is beyond the period of
consideration in IAS 10. Thus this would be neither an adjusting nor a non-adjusting event. The
increase in the deferred tax liability will be provided for in the year to 31 March 2010. Had the
announcement been before 6 May 2009, it would have been treated as a non-adjusting event
requiring disclosure of the nature of the event and an estimate of its financial effect in the notes
to the financial statements.
QUESTION 2
Suame Ltd is a listed telecommunication company which prepares its financial statements for
the year ended 31 October, 2015 in accordance with IFRS. The financial statements are due to be
authorised for issue on 15 January 2016.
i) Suame Ltd holds an investment in the shares of a listed company, Asafo Ltd. During
November 2015 there was a material fall in the value of Asafo Ltd‘s shares. Analysts attribute
the fall in value principally to a fraud dating back to December 2014 that was discovered by
Asafo Ltd's management and announced publicly in November 2015.
ii) In December 2015, the directors of Suame Ltd publicly announced a plan to reduce the
workforce by 10% as a result of worsening economic conditions.
Required:
Discuss the effects of each of the above items on the financial statements of Suame Ltd for the
year ended 31 October 2015 in accordance with IAS 10 Events after the Reporting Period.
(4 marks)
ICAG NOV.2016
Solution:
i) The fall in value relates to conditions that arose after the end of the reporting period.
Therefore, the fall in value is a non-adjusting event after the reporting period. Whilst the effect
of the fraud may be an adjusting event in Asafo Ltd‘s own financial statements, it is not an
adjusting event for the value of the company's shares on the stock market as that market value
was based on all information available at that time (for example investors who purchased
shares on 31 December at the market price on that date would not be able to make a claim
against the previous owner when the fraud was discovered).
In accordance with IAS 10, which requires disclosure of material non-adjusting events after the
reporting period, disclosure will be made of:
 the nature of the event
 the amount of the financial effect, ie fall in value

25
(2 marks)
ii) The announcement of plans to restructure creates a constructive obligation to do a
restructuring. As a result, a restructuring provision will be recognised from that date, providing
the IAS 37 criteria are met. However, no legal or constructive obligation existed to restructure at
the 31 October 2015 year end and this is therefore a non-adjusting event after the reporting
period. In accordance with IAS 10, which requires disclosure of material non-adjusting events
after the reporting period, disclosure will be made of:
 The nature of the event
 The amount of the financial effect, ie the expected restructuring costs.
(2 marks)

QUESTION 3
The following events occurred after the year end, but before the financial statements were
authorised for issue:
i) Enactment by the government of a revised tax rate affecting the amount of the settlement of
the deferred tax liability included in the financial statements.
ii) A share split in respect of the earnings per share calculation.
iii) Criteria being met in order to classify non-current assets as held for sale.
iv) A material, but not fundamental, error arising in the comparative figures.

Required:
In accordance with IAS 10: Events after the reporting period, explain with justification whether
each of the above is an adjusting or a non-adjusting event after the reporting period.
(4 marks)

Solution:
i) Enactment by the government of a revised tax rate affecting the amount of the
settlement of the deferred tax liability included in the financial statements
Non-adjusting event (IAS 10 para 22(h)) – rates enacted after the year end do not
meet the definition of a liability at the year end.

ii) A share split in respect of the earnings per share calculation


Adjusting event in respect of the earnings per share calculation (IAS 33 para 64) –
although a non-adjusting event for the statement of financial position, if a share spit
occurs before the financial statements are issued to shareholders, earnings per share is
adjusted as it would otherwise be misleading as the shareholders' shareholdings have
been diluted before they receive the financial statements.

iii) Criteria being met in order to classify non-current assets as held for sale
Non-adjusting event (IAS 10 para 22(c)) – IFRS 5 requires the criteria to be met at any
given point in time so if the criteria are not met at the year end, the assets cannot be
classified as held for sale.

iv) A material, but not fundamental, error arising in the comparative figures
Adjusting event – all material errors are corrected retrospectively under IAS 8.

26
QUESTION 4
Nabdam Ltd (Nabdam) operates in the media and publications industry and reports under IFRS. The
2018 financial statements of Nabdam are still in draft form. The audit is ongoing, and the company
intends to authorise the financial statements in April 2019.
Nabdam rents a distribution warehouse in Korle, located beside the River Odorna. On 3 January
2019, the River Odorna burst its banks and GH¢650,000 of Nabdam‟s inventory was destroyed by
the flood. The inventory was not insured and Nabdam will not receive any compensation for the loss.
The company is not sure how to account for this event. The destroyed inventory is included in the
inventory figure that is disclosed on Nabdam‟s draft statement of financial position at 31 December
2018.
Required:
Explain with justification, the appropriate accounting treatment of the above transaction.
(4 marks)

SOLUTION 4

outlines that an ―event after the reporting period‖ is an event which occurs between the end of
the reporting period and the date that the financial statements are approved.

-adjusting events. Adjusting events


provide further evidence on a condition that existed at the reporting date. Adjusting events
must be adjusted in the financial statements.

-adjusting events are events that are indicative of conditions that arose after the reporting
date. No adjustments are made for non-adjusting events. The flood occurred on the 3rd January
2019. The condition (the flood and damage to the inventory) did not exist at the reporting date
of 31st December 2018. Therefore, the event is a non-adjusting event and Nabdam does not have
to adjust the 2018 financial statements for the GH¢650,000 inventory loss.

nature of the event and an estimate of its financial effect. Therefore, as the inventory loss is
material, Nabdam would have to make a disclosure describing the nature of the event (a flood
affecting a distribution warehouse) and an estimate of the financial effect of the event
(GH¢650,000 damage to inventory) in its 2018 financial statements.

QUESTION 5
(a)The objective of IAS 10 Events after the Reporting Period is to prescribe the treatment of
events that occur after an entity‟s reporting period has ended.
Required:
Define the period to which IAS 10 relates and distinguish between adjusting and non-adjusting
events. (5 marks)
(b) Iniesta‟s current year end is 31 March 2009. Its financial statements were authorised for issue
by its directors on 6 May 2009 and the AGM (annual general meeting) will be held on 3 June
2009. The following matters have been brought to your attention:

(i) On 12 April 2009 a fire completely destroyed the company‟s largest warehouse and the
inventory it contained. The carrying amounts of the warehouse and the inventory were GHC10

27
million and GHC6 million respectively. It appears that the company has not updated the value of
its insurance cover and only expects to be able to recover a maximum of GHC9 million from its
insurers. Iniesta‟s trading operations have been severely disrupted since the fire and it expects
large trading losses for some time to come.
(4 marks)
(ii) A single class of inventory held at another warehouse was valued at its cost of GHC460,000
at 31 March 2009. In April 2009 70% of this inventory was sold for GHC280,000 on which
Iniestas‟ sales staff earned a commission of 15% of the selling price.
(3 marks)
(iii) On 18 May 2009 the government announced tax changes which have the effect of increasing
Iniesta‟s deferred tax liability by GHC650,000 as at 31 March 2009.
(3 marks)

Required:
Explain the required treatment of the items (i) to (iii) by Iniesta in its financial statements for the
year ended 31 March 2009.
Note: assume all items are material and are independent of each other.
(10 marks as indicated)
(15 marks)

Solution:
(a)Events after the reporting period are defined by IAS 10 Events after the Reporting Period as
those events, both favourable and unfavourable, that occur between the end of the reporting
period and the date that the financial statements are authorised for issue (normally by the Board
of directors).
An adjusting event is one that provides further evidence of conditions that existed at the end of
the reporting period, including an event that indicates that the going concern assumption in
relation to the whole or part of the entity is not appropriate. Normally trading results occurring
after the end of the reporting period are a matter for the next reporting period, however, if there
is an event which would normally be treated as non-adjusting that causes a dramatic downturn in
trading (and profitability) such that it is likely that the entity will no longer be a going concern,
this should be treated as an adjusting event.
A non-adjusting event is an event after the end of the reporting period that is indicative of a
condition that arose after the end of the reporting period and, subject to the exception noted
above, the financial statements would not be adjusted to reflect such events.
The outcome (and values) of many items in the financial statements have a degree of uncertainty
at the end of the reporting period. IAS 10 effectively says that where events occurring after the
end of the reporting period help to determine what those values were at the end of the reporting
period, they should be taken in account (i.e. adjusted for) in preparing the financial statements.
If non-adjusting events, whilst not affecting the financial statements of the current year, are of
such importance (i.e. material) that without disclosure of their nature and estimated financial
effect, users‟ ability to make proper evaluations and decisions about the future of the entity
would be affected, then they should be disclosed in the notes to the financial statements.

(b) (i) This is normally classified as a non-adjusting event as there was no reason to doubt that
the value of warehouse and the inventory it contained was worth less than its carrying amount at

28
31 March 2009 (the last day of the reporting period). The total loss suffered as a result of the fire
is GHC16 million. The company expects that GHC9 million of this loss will be recovered from
an insurance policy. Recoveries from third parties should be assessed separately from the related
loss. As this event has caused serious disruption to trading, IAS 10 would require the details of
this non-adjusting event to be disclosed as a note to the financial statements for the year ended
31 March 2009 as a total loss of GHC16 million and the effect of the insurance recovery to be
disclosed separately.
The severe disruption in Iniesta‟s trading operations since the fire, together with the expectation
of large trading losses for some time to come, may call in to question the going concern status of
the company. If it is judged that Iniesta is no longer a going concern, then the fire and its
consequences become an adjusting event requiring the financial statements for the year ended 31
March 2009 to be redrafted on the basis that the company is no longer a going concern (i.e. they
would be prepared on a liquidation basis).

(ii) 70% of the inventory amounts to GHC322,000 (460,000 x 70%) and this was sold for a net
amount of GHC238,000 (280,000 x 85%). Thus a large proportion of a class of inventory was
sold at a loss after the reporting period. This would appear to give evidence of conditions that
existed at 31 March 2009 i.e. that the net realisable value of that class of inventory was below its
cost. Inventory is required to be valued at the lower of cost and net realisable value, thus this is
an adjusting event. If it is assumed that the remaining inventory will be sold at similar prices and
terms as that already sold, the net realisable value of the whole of the class of inventory would be
calculated as:
GHC280,000/70% = GHC400,000, less commission of 15% = GHC340,000.
Thus the carrying amount of the inventory of GHC460,000 should be written down by
GHC120,000 to its net realisable value of GHC340,000.
In the unlikely event that the fall in the value of the inventory could be attributed to a specific
event that occurred after the date of the statement of financial position then this would be a non-
adjusting event.

(iii) The date of the government announcement of the tax change is beyond the period of
consideration in IAS 10. Thus this would be neither an adjusting nor a non-adjusting event. The
increase in the deferred tax liability will be provided for in the year to 31 March 2010. Had the
announcement been before 6 May 2009, it would have been treated as a non-adjusting event
requiring disclosure of the nature of the event and an estimate of its financial effect in the notes
to the financial statements.

QUESTION 6
Suame Ltd is a listed telecommunication company which prepares its financial statements for the
year ended 31 October, 2015 in accordance with IFRS. The financial statements are due to be
authorised for issue on 15 January 2016.
i) Suame Ltd holds an investment in the shares of a listed company, Asafo Ltd. During
November 2015 there was a material fall in the value of Asafo Ltd‟s shares. Analysts attribute
the fall in value principally to a fraud dating back to December 2014 that was discovered by
Asafo Ltd's management and announced publicly in November 2015.
ii) In December 2015, the directors of Suame Ltd publicly announced a plan to reduce the
workforce by 10% as a result of worsening economic conditions.

29
Required:
Discuss the effects of each of the above items on the financial statements of Suame Ltd for the
year ended 31 October 2015 in accordance with IAS 10 Events after the Reporting Period.
(4 marks)
ICAG NOV.2016
Solution:
i) The fall in value relates to conditions that arose after the end of the reporting period.
Therefore, the fall in value is a non-adjusting event after the reporting period. Whilst the effect
of the fraud may be an adjusting event in Asafo Ltd‟s own financial statements, it is not an
adjusting event for the value of the company's shares on the stock market as that market value
was based on all information available at that time (for example investors who purchased shares
on 31 December at the market price on that date would not be able to make a claim against the
previous owner when the fraud was discovered).
In accordance with IAS 10, which requires disclosure of material non-adjusting events after the
reporting period, disclosure will be made of:
 the nature of the event
 the amount of the financial effect, ie fall in value
(2 marks)
ii) The announcement of plans to restructure creates a constructive obligation to do a
restructuring. As a result, a restructuring provision will be recognised from that date, providing
the IAS 37 criteria are met. However, no legal or constructive obligation existed to restructure at
the 31 October 2015 year end and this is therefore a non-adjusting event after the reporting
period. In accordance with IAS 10, which requires disclosure of material non-adjusting events
after the reporting period, disclosure will be made of:
 The nature of the event
 The amount of the financial effect, ie the expected restructuring costs.
(2 marks)

30
INCOME TAXES (IAS 12)
Income taxes are an expense which most businesses will incur as a result of their activities and
as such are to be reflected in the entity‘s operating results. Accounting for income taxes can
become confusing because, in most jurisdictions, the amounts of revenues and expenses
recognised in a given period for taxation purposes will not fully correspond to what is reported
in the financial statements.
IAS 12 seeks to address some of the fundamental issues dealt with in this standard for ‗Deferred
Tax‘ and ‗Income Tax‘.
IAS 12, Income Taxes, deals with taxes on income, both current tax and deferred tax. Income tax
accounting is complex, and preparers and users find some aspects difficult to understand and
apply. These difficulties arise from exceptions to the principles in the current standard, and
from areas where the accounting does not reflect the economics of the transactions.
The current tax expense for a period is based on the taxable and deductible amounts that will be
shown on the tax return for the current year. Current tax assets and liabilities for the current
and prior periods are measured at the amount expected to be paid to or recovered from the tax
authorities, using the tax rates and tax laws that have been enacted or substantively enacted by
the date of the financial statements.
A mismatch can occur because International Financial Reporting Standards (IFRS) recognition
criteria for items of income and expense are different from the treatment of items under tax law.
Deferred taxation accounting attempts to deal with this mismatch. The IAS 12 standard is based
on the temporary differences between the tax base of an asset or liability and its carrying
amount in the financial statements.
The tax base of an asset or liability is the amount attributed to it for tax purposes, based on the
expected manner of recovery. IAS 12 focuses on the future tax consequences of recovering an
asset only to the extent of its carrying amount at the date of the financial statements. Future
taxable amounts arising from recovery of the asset will be capped at the asset's carrying
amount.
For example, a property may be revalued upwards but not sold, creating a temporary difference
because the carrying amount of the asset in the financial statements is greater than the tax base
of the asset. The tax consequence is a deferred tax liability.
Deferred tax is provided in full for all temporary differences arising between the tax bases of
assets and liabilities and their carrying amounts in the financial statements. There are
exceptions where the temporary difference arises from:
 Initial recognition of goodwill.
 Initial recognition of an asset or liability in a transaction that is not a business
combination and that affects neither accounting profit nor taxable profit.
 Investments in subsidiaries, branches, associates and joint ventures where certain
criteria apply.
Deferred tax assets and liabilities are measured at the tax rates that are expected to apply to the
period when the asset is realised or the liability is settled and discounting of deferred tax assets
and liabilities is not permitted.
The measurement of deferred tax liabilities and deferred tax assets reflects the tax consequences
of the manner in which the entity expects to recover or settle the carrying amount of its assets
and liabilities. The expected manner of recovery for land with an unlimited life is always
through sale, but for other assets the manner in which management expects to recover the asset,
either through use or sale or both, should be considered at each date of the financial statements.

31
A deferred tax asset is recognised to the extent that it is probable that taxable profit will be
available against which the deductible temporary difference can be used. This also applies to
deferred tax assets for unused tax losses carried forward.
Current and deferred tax is recognised in profit or loss for the period, unless the tax arises from
a business combination or a transaction or event that is recognised outside profit or loss, either
in other comprehensive income or directly in equity in the same or different period.
For example, a change in tax rates or tax laws, a reassessment of the recoverability of deferred
tax assets or a change in the expected manner of recovery of an asset have tax consequences that
are recognised in profit or loss, except to the extent that they relate to items previously charged
or credited outside profit or loss.
That, at least, is the current position on current and deferred taxation under IFRS. The proposed
amendments to IAS 12 issued in March 2009 would have made significant changes. However,
after considering the unenthusiastic feedback, the International Accounting Standards Board
(IASB) has decided not to proceed with its proposals but to focus on practical issues with the
existing standard.
The IASB and the Financial Accounting Standards Board (FASB) will consider fundamentally
reviewing the accounting for income taxes sometime in the future. In the meantime, the IASB is
undertaking a limited-scope project to see which issues can be addressed in the shorter term.
The project aims to resolve problems without changing the fundamental approach under IAS
12. The project will cover the following:
 deferred tax arising from property remeasurement at fair value;
 uncertain tax positions;
 introduction of a step to consider whether the recovery of an asset or settlement of a
liability will affect taxable profit;
 recognition of a deferred tax asset in full and an offsetting valuation allowance to the
extent necessary;
 guidance on assessing the need for a valuation allowance;
 guidance on substantive enactment of tax laws; and
 allocation of current and deferred taxes within a group that files a consolidated tax
return.
In September 2010, an IASB exposure draft proposed an exception to the existing principle for
measuring deferred tax assets or liabilities arising on certain non-financial assets measured at
fair value. The exception applies to:
 investment property measured using the fair value model in IAS 40;
 property, plant and equipment or intangible assets measured using the revaluation
model in IAS 16 or IAS 38; and
 investment property, property, plant and equipment or intangible assets initially
measured at fair value in a business combination if the fair value or revaluation model
was used when the underlying asset was subsequently measured.
Deferred tax assets and liabilities are currently measured on the basis of:
 the expected manner of recovery (asset) or settlement (liability); and
 the tax rate expected to apply when the underlying asset (liability) is recovered (settled).
The expected manner of recovery or settlement may affect the calculation of the tax base or the
applicable tax rate or both. In such cases management's intentions are key in determining the
amount of deferred tax to recognise.
The issue is that it can be difficult and subjective to determine the expected manner of recovery.
The IASB's proposed exception to this measurement principle applies to investment property,

32
property, plant and equipment, and intangible assets measured using the fair value or
revaluation model in accordance with relevant IFRSs.
Under this exception, the measurement of deferred tax assets and liabilities reflects a rebuttable
presumption that the carrying amount of the underlying asset will be recovered entirely
through sale.
The presumption could be rebutted only when there is clear evidence that the underlying asset's
economic benefits will be consumed by the entity throughout the asset's economic life.The IASB
has proposed this exception to the measurement principle that disregards management's
intention unless there is clear evidence to support consumption through use.
It will affect entities holding investment property, property, plant and equipment or intangible
assets measured at fair value where the capital gains tax rate is different from the income tax
rate, and/or the tax base from sale is different from tax base from use. The deferred tax liability
will be reduced significantly where there is no capital gains tax. Judgment will be required to
determine whether clear evidence exists. SIC 21, Income Taxes - Recovery of Revalued Non-
Depreciable Assets, will be withdrawn by the amendment.
Assets measured at cost, or other assets measured at fair value such as financial instruments, are
not in the scope of the IASB's exposure draft. However, it is unclear why the same principles do
not apply to these assets. The result is that there will be a different approach to deferred tax
accounting by entities with identical assets and tax rates but different accounting policies.
The IASB's exposure draft requires full retrospective application, with early adoption permitted.
Complexities might therefore arise if the underlying assets were acquired in a business
combination.
In terms of deferred tax, you will come across the following terms which can be defined as
follows:
A deferred tax asset is the amount of income tax recoverable in future periods in respect of
deductible temporary differences. The most common form of deferred tax asset arises because
of unused tax losses which are carried forward by the reporting entity.
A deferred tax liability is the amount of income tax payable in future periods in respect of
taxable temporary differences.
Temporary differences are the differences between the tax and financial reporting bases of
assets and liabilities which will result in taxable or deductible amounts in future periods.
Permanent differences are the differences between the accounting profit and taxable profit as a
result of the treatment of different items within the financial statements to that which is
permitted under tax legislation. For example, depreciation is taken into account in arriving at
the accounting profit but is added back when calculating taxable profit. This is known as a
‗permanent‘ difference.
It is important to address the inherent nature of deferred tax. Deferred tax is essentially a
provision : in other words we are providing for the future tax consequences of present and/or past
transactions. As a result deferred tax is never paid : it is simply recognised and provided for but
you will never write a cheque out for ‗deferred tax‘. In reality, deferred tax provisions crystallise
on the sale or termination of a business.
The complexity in IAS 12 often arises because accountants have difficulty understanding the
nature of temporary differences. Temporary differences are essentially ‗timing differences‘ :
examples of which can be seen as follows:
 Revenue recognised for tax purposes prior to recognition in the financial statements.
These include certain types of income streams received such as prepaid rental income and

33
service contract revenue. Referred to as deductible temporary differences, these items give
rise to deferred tax assets.
 Expenses that are deductible for tax purposes prior to recognition in the financial
statements. This happens when depreciation is accelerated or when short useful lives are
used for tax purposes so the asset is written off more quickly in the capital allowances
computation than in the financial statements, resulting in the book values per the accounts
being higher than the book values for tax purposes. These items are taxable temporary
differences and give rise to deferred tax liabilities.
 Expenses that are reported in the financial statements prior to becoming deductible for tax
purposes. Items such as these are often estimated expenses, such as warranty costs or certain
fees in connection with a contingent litigation claim. Expenses such as these are not tax
deductible until the obligation becomes fixed. These are deductible temporary differences
and give rise to deferred tax assets.
 Certain business combinations accounted for by the acquisition method of accounting.
Under certain circumstances, the cost assignable to assets or liabilities acquired in the
purchase will differ from their tax bases. The usual scenario under which this arises is when
the acquirer must continue to report the predecessor‘s tax bases for tax purposes, although
the price paid was more or less than book value. Such differences may be either taxable or
deductible and, accordingly, may give rise to deferred tax liabilities or assets.
 Assets which are revalued for financial reporting purposes although the tax bases are not
affected. Under IAS 16 ‗Property plant and equipment‘ and IAS 40 ‗Investment properties‘
assets may be uplifted to their current market value. For tax purposes, these uplifts are
ignored until (and unless) the assets are disposed of. The discrepancies between the adjusted
book carrying values and the tax bases are temporary differences under IAS 12 and deferred
taxes are to be provided on these variations. This is required even if there is no intention to
dispose of the asset(s) in question, or if, under the salient tax laws, exchanges for other
similar assets (or reinvestment of proceeds of sales in similar assets) would affect a
postponement of the tax obligation.
Accountants are more familiar with the temporary differences that occur between book values
and tax written down values of fixed assets, but as you can see from the above there are a
number of reasons why temporary differences may arise.
INCOME TAX: CURRENT VS. DEFERRED
Income tax is type of direct tax levied by a government on businesses. Income tax due in a
period is calculated by applying the applicable tax percentage to the taxable income of the
business.
Taxable income is the net income calculated in accordance with the tax laws.
Taxable income = taxable revenues – tax-deductible expenses – tax exemptions
It is different than accounting income, which equals revenue recognized under GAAP minus
expenses allowed under GAAP.
Accounting income = revenues under GAAP – expenses under GAAP
Accounting for income tax is complex due to the fact that there is a difference in financial
accounting treatment of transactions and their tax accounting treatment. Let‘s take different
components of a business‘s tax accounting one by one.

Current income tax obligation


Determining the current income tax payable is the most straightforward, because it represents a
business‘s tax obligation related to the current period taxable income.

34
Current income tax obligation = taxable income × tax rate
Matching concept of accounting suggests that tax expense for revenue should be recognized in
the period in which the relevant revenue was recognized. Similarly, tax shield i.e. the reduction
in tax expense due tax deductibility of an expense should be recognized in the period in which
the relevant expense is recognized.
We determined that taxable income is different than accounting income. It means that the
current income tax payable is not a good representation of current tax expense. It should be
adjusted. For example:
 Current income tax payable related to revenue that would be recognized under GAAP in
future periods should be carried forward;
 Income tax payable related to revenue that is recognized today under GAAP but which shall
be taxed in future periods should be included in current period‘s tax expense;
 Tax benefit of expenses that shall be recognized under GAAP in future periods but which are
allowed as tax deduction in current period should be carried forward; and
 Tax benefit of expenses that are recognized under GAAP today but which shall be allowed as
tax deduction in future should be subtracted from current period income tax expense.
These adjustments result in the concept of deferred taxation.

Deferred taxation
Deferred taxation is the process of transferring tax expense between different periods in order
to better match revenues with expenses through a process called deferred taxation.
In order to better understand this process of deferred taxation and calculate current income tax,
we need to define a few income statement and balance sheet accounts.
Deferred tax liability
Deferred tax liability is amount of tax a business shall be required to pay in future related to (a)
revenues recognized in current period under GAAP but not under tax laws (i.e. not taxed in
current period), which shall be taxed in future periods, and (b) expenses (not recognized under
GAAP) that are allowed as tax deduction in current period but which shall not be allowed as
deduction in future periods.
In other words deferred tax liability represents tax effects of transactions that will result in
future increases in taxable income.
Deferred tax asset
Deferred tax asset is the amount of tax a business shall pay less in future due to the fact that (a)
revenues that are taxed today shall not be taxed in future (when they will be eventually
recognized under GAAP) and (b) expenses (that are recognized under GAAP in current period)
that are not deducted in calculating taxable income in current period but which shall be
deducted in future periods.
In other words deferred tax asset represents tax effects of transactions that will result in future
decreases in taxable income.
Deferred tax expense
Deferred tax expense is the sum of any increase in deferred tax liability over a period minus an
increase in deferred tax asset over the period. Deferred tax expense may be negative which
results in current tax expense being less than current income tax obligation.
Current tax expense
Current tax expense equals a business‘s current income tax obligation adjusted for the effect of
transfer of income tax between different periods i.e. deferred taxation.
Current tax expense = current income tax obligation + deferred tax expense

35
Where deferred tax expense is negative for a period, current tax expense is lower than current
income tax payable.
The expression above can be expanded as follows:
Current tax expense = current income tax obligation + closing deferred tax liability – opening
deferred tax liability – (closing deferred tax asset – opening deferred tax asset)
Measurement of deferred tax assets and liabilities
In order to arrive at a deferred tax asset or liability there is a procedure which can be adopted to
compute the gross deferred tax provision (i.e. before addressing whether a deferred tax asset is
potentially realisable and therefore recognisable):
 Identify all temporary differences existing as at the reporting date;
 Segregate the temporary differences into those that are (a) taxable and those that are (b)
deductible. You need to do this because under IAS 12 only those deferred tax assets which
are probable of being realised are recognised, whereas all deferred obligations are given full
recognition;
 Accumulate information about the deductible temporary differences, in particular the net
operating loss and carry forwards that have expiration dates or other types of limitations.
 Measure the tax effect of aggregate ‗taxable‘ temporary differences by applying the
appropriate expected tax rates.
 Measure the tax effects of deductible temporary differences, including net operating loss
carry forwards.
When accounting for deferred tax assets and liabilities, it is important to understand that whilst
all deferred tax liabilities should be accounted for, it can be argued that deferred tax assets
should be included in the balance sheet only if they are likely to be realised. In other words if
the entity is sure that it will make sufficient profits in the foreseeable future against which the
unused tax losses can be offset.

RECOGNITION OF DEFFERED TAX ASSET.


Recognition of a deferred tax asset can only, therefore, be recognised if it can be concluded
that the realisation of the deferred tax asset is probable.
Possible factors which may give rise to a deferred tax asset being recognised are:
 Evidence of sufficient future taxable income to offset against the tax losses carried forward.
 Evidence of sufficient future taxable income arising from the reversals of existing taxable
temporary differences (deferred tax liabilities) to realise the benefit of the tax asset.
 An excess of appreciated asset values over their tax bases, the excess of which would be
sufficient to realise the deferred tax asset.
 Evidence that the loss created at the reporting date was due to unusual factors which are not
going to be repeated in subsequent periods.
Those are some potential reasons why a company might recognise a deferred tax asset.
Conversely, a company who is sustaining repeated losses year on year may have to consider
whether, in the circumstances, there is a probability that the deferred tax asset will ever, in fact,
be realised.
Measurement of income tax expense in the financial statements
There are two component parts of the income tax expense; current tax expense and deferred tax
expense.
Current tax expense is straight forward. It is basically the tax expense on the current period’s
profit or loss.
Deferred tax expense arises as the tax effect of timing differences occurring in the period.

36
The provisions of IAS 12 require the use of the ‗liability method‘. Using the liability method, the
reporting entity‘s current period total income tax expense cannot be calculated directly, unless
the entity has no temporary differences. It should be calculated as the sum of the current tax
expense and the deferred tax expense. This total may not amount to that which would be
calculated by applying the tax rate prevailing at the reporting date to the profit before tax as the
deferred tax expense is defined as the change in the deferred tax asset and liability accounts
occurring in the current period. This change may encompass more than the mere effect of the
tax rate at the reporting date multiplied by the net temporary differences arising or being
reversed in the current reporting period.
The liability method takes the approach from the statement of financial position (the balance
sheet). Therefore its primary concern is to present the estimated actual taxes to be payable in the
current and subsequent periods as the income tax liability within the statement of financial
position.
In theory, the nature of the liability is quite straight forward. However, there are difficulties
which entities may need to consider such as:
 The tax effects of business combinations;
 Group income tax allocation;
 Treatment of losses carried forward;
 Deferred tax assets that may not be realised; and
 The nature of the actual temporary differences.

REVIEW QUESTIONS
QUESTION 1
White Angel Ltd purchases a non current asset costing GHC80,000 on 1 January,2000.The asset
has an estimated life of five years and a residual value of GHC20,000.The company‘s policy is to
depreciate this class of assets on a straight line basis over its estimated useful life.The asset is
sold at its residual value on 31 December,2004.Profit before depreciation and taxation for each
five years of the asset‘s life is GHC62,000.
The company pays corporation tax at a rate of 30% and is entitled to capital allowances of 25%
per annum an a reducing balance basis. Any remaining balance after disposal is treated as a
balance allowance or charge.
You are required to show the relevant extracts for both income statement and statement of
financial position assuming deferred tax is provided in full on all timing differences.

Solution:
Workings:
Year Carrying Value Tax Base Difference Deferred Tax(30%)
GHC GHC GHC GHC
2000 Cost 80,000 80,000
Depn/Capital Allo‘ce 12,000 20,000 (Timing Diff.) 8,000 2,400
Carrying Value 68,000 60,000(Temporary Diff) 8,000 2,400
2001Depn./Capital Allce 12,000 15,000 3,000 900
Carrying Amount 56,000 45,000 11,000 3,300
Depn./Cap. Allce 12,000 11,250 (750) (225)
2003 Carrying Value 44,000 33,750 10,250 3,075
Depn./Capital Allce 12,000 8,438 (3,562) (1,069)
Carrying Value 32,000 25,312 6,688 2,006

37
2004 Depn/Cap Allce 12,000 5,312 (6,688) (2,006)
RV/Disposal 20,000 20,000 - -

Deferred Tax Account

Year Year
GHC GHC
2000 2000
Dec 31 Balance c/d 2,400 Dec 31 Profit & Loss a/c 2,400
2001 2001
Jan 1 Balance b/f 2,400
Dec 31 Balance c/d 3,300 Dec 31 Profit & Loss a/c 900
3,300 3,300

2002 2002
Dec 31 Profit & Loss a/c 225 Jan 1 Balance b/f 3,300
Dec 31 Balance c/d 3,075
3,300 3,300
2003 2003
Dec 31 Profit & Loss a/c 1,069 Jan 1 Balance b/f 3,075
Dec 31 Balance c/d 2,006
3,075 3,075
2004 2004
Dec 31 Profit & Loss a/c 2,006 Jan 1 Balance b/f 2,006

Current Tax Computation


2000 2001 2002 2003 2004
GHC GHC GHC GHC GHC
Profit before Deprn. 62,000 62,000 62,000 62,000 62,000
Less Capital All‘ce 20,000 15,000 11,250 8,438 5,312
Taxable Profit 42,000 47,000 50,750 53,56 56,688
Current Tax (30%) 12,600 14,100 15,226 16,069 17,000

Income Statement
2000 2 2001 2002 2003 2004
GHC GHC GHC GHC GHC
Accounting Profit(62-12)50,000 50,000 50,000 50,000 50,000
Income Tax Expense:
Current Tax (12,600) (14,100) (15,225) (16,069) (17,006)
Deferred Tax (2,400) (900) 225 1,069 2,006
Profit after Tax 35,000 35,000 35,000 35,000 35,000

38
Statement of Financial Position as at 31st Dec……
2000 2001 2002 2003 2004
Non Current Liab: GHC GHC GHC GHC GHC
Deferred Tax 2,400 3,300 3,075 2,006 -
Current Liabilities:
Current Tax 12,600 14,100 15,225 16,009 17,006

QUESTION 2

During 2003,The Blues Ltd revalued some land from GHC 50 million to GHC100
million.Its profit before tax for the year ended 31 December,2003,was GHC40 million,on
which it pays tax at 30%.
The opening balance on the retained earnings reserves was GHC220 million,and there
were no deffered tax balances brought forward.
Prepare all relevant extracts from The Blues Ltd’s financial statements for the year
ended 31 December,2003.

Solution:
Statement of Comprehensive Income for the year ended 31st December,2003
GHC GHC
Profit before tax 40,000
Less:Income tax expense(30% x GHC40,000) (12,000)
Profit for the year 28,000
Other comprehensive Income:
Revaluation Surplus on Land(100-50) 50,000
Tax on Revalued Land(50% x GHC50,000) (15,000) 35,000
Total Comprehensive Income 63,000

Statement of Financial Position as at 31st Dec.,2003


Non Current Assets GHC‘000
Land as Revalued 100,000

Equity & Liabilities


Capital Surplus(50,000-15,000) 35,000
Retained Earnings (220,000+28,000) 248,000

Non-Current Liabilities
Deferred Tax 15,000

Current Liabilities
Current Tax 12,000

39
QUESTION 3
The following is a Trial Balance Extract of Barcelona as at 30 September,2009
DR CR
GHC000 GHC000
Deferred Tax 4,000
Current Tax 900
Note:
A provision for income tax for the year ended 30 September 2010 of GHC5·6 million is
required. The balance on current tax represents the under/over provision of the tax
liability for the year ended 30 September 2009. At 30 September 2010 the tax base of
Barcelona‘s net assets was GHC15 million less than their carrying amounts. The
movement on deferred tax should be taken to the income statement. The income tax rate
of Barcelona is 25%.
Required:
Compute the tax values to be shown in the Financial statements of Barcelona.

Solution:
Deferred Tax

GHC GHC
Profit & Loss 250 Balance b/d 4,000

Balance c/d(0.25 x 15,000) 3,750

4,000 4,000

Profit and Loss a/c

GHC
Income tax (5,600+900-250) 6,250

Statement of Financial Position


GHC
Non-Current Liabilities
Deferred Tax 3,750

Current Liabilities
Current Tax 5,600

40
QUESTION 4
The following trial balance extract relates to Hazard Ltd as at 30 September 2011
DR CR
GHC000 GHC000
Deferred Tax 2,700

Note:
A provision for income tax for the year ended 30 September 2011 of GHC24·3 million is
required. At 30 September 2011, the tax base of Hazard‘s net assets was GHC15 million less
than their carrying amounts. The income tax rate of Hazard is 30%.
Required:
Compute the tax values to be shown in the Financial Statements of Hazard Ltd.

Solution:

Deferred Tax

GHC GHC
Balance b/d 2,700

Balance c/d(0.3 x 15,000) 4,500 Profit & Loss 1,800

4,500 4,500

Profit and Loss a/c

GHC
Income tax (24,300 +1,800) 26,100

Statement of Financial Position


GHC
Non-Current Liabilities
Deferred Tax 4,500

Current Liabilities
Current Tax 24,300

QUESTION 5
On 30 June 2012 Francesca Company had a credit balance on its deferred tax account of
GHC1,340,600 all in respect of the difference between depreciation and capital allowances.
During the year ended 30 June 2013 the following transactions took place.
(1) GHC45 million was charged against profit in respect of depreciation. The tax computation
showed capital allowances of GHC50 million.

41
(2) Interest receivable of GHC50,000 was reflected in profit for the period. However, only
GHC45,000 of interest was actually received during the year. Interest is not taxed until it is
received.
(3) Interest payable of GHC32,000 was treated as an expense for the period. However,only
GHC28,000 of interest was actually paid during the year. Interest is not an allowable expense
for tax purposes until it is paid.
(4) During the year Francesca incurred development costs of GHC500,600, which it has
capitalised. Development costs are an allowable expense for tax purposes in the period in which
they are paid.
(5) Land and buildings with a net book value of GHC4,900,500 were revalued to GHC6 million.
The tax rate is 30%. Francesca has a right of offset between its deferred tax liabilities and its
deferred tax assets.
Required
Calculate the deferred tax liability on 30 June 2013. Show where the increase or decrease in the
liability in the year would be charged or credited.

Solution:
GHC GHC
Opening liability 1,340,600
Capital allowances during the year 50,000,000
Depreciation charged during the year (45,000,000)
–––––––––––
5,000,000 × 30% 1,500,000
–––––––––––
Interest receivable in income statement 50,000
Interest received in tax computation (45,000)
–––––––––––
Receivable in statement of financial position 5,000 × 30% 1,500
–––––––––––
Interest payable in income statement 32,000
Interest paid in tax computation (28,000)
–––––––––––
Payable in balance sheet 4,000 × 30% (1,200)
–––––––––––
Development costs as allowable expense 500,600 × 30% 150,180

Revaluation 6,000,000
Carrying value (4,900,500)
–––––––––––
Revaluation surplus 1,099,500 x 30% 329,850
––––––––––– ––––––––––
Closing liability 3,320,930
––––––––––

GHC
Charged to the revaluation reserve 329,850
Charged in the income statement (balancing figure) 1,650,480

42
––––––––––
Total movement on the provision of (3,320,930 – 1,340,600) 1,980,330
––––––––––

QUESTION 6
Julian recognised a deferred tax liability for the year end 31 December 2013 which related solely
to accelerated tax depreciation on property, plant and equipment at a rate 30%. The net book
value of the property, plant and equipment at that date was GHC310,000 and the tax written
down value was GHC230,000.
The following data relates to the year ended 31 December 2014:
(i) At the end of the year the carrying value of property, plant and equipment was GHC460,000
and their tax written down value was GHC270,000. During the year some items were revalued
by GHC90,000. No items had previously required revaluation. In the tax jurisdiction in which
Julian operates revaluations of assets do not affect the tax base of an asset or taxable profit.
Gains due to revaluations are taxable on sale.
(ii) Julian began development of a new product during the year and capitalised GHC60,000 in
accordance with IAS 38. The expenditure was deducted for tax purposes as it was incurred.
None of the expenditure had been amortised by the year end.
(iii) Julian's income statement showed interest income receivable of GHC55,000, but only
GHC45,000 of this had been received by the year end. Interest income is taxed on a receipts
basis.
(iv) During the year, Julian made a provision of GHC40,000 to cover an obligation to clean up
some damage caused by an environmental accident. None of the provision had been used by
the year end. The expenditure will be tax deductible when paid.
The corporate income tax rate recently enacted for the following year is 30% (unchanged from
the previous year).
The current tax charge was calculated for the year as GHC45,000.
Current tax is settled on a net basis with the national tax authority (GRA)

Required
(a) Prepare a table showing the carrying values, tax bases and temporary differences for each
for the items above at 31 December 2014.

(b) Prepare the income statement and statement of financial position notes to the financial
statements relating to deferred tax for the year ended 31 December 2014.

Solution:
(a)
Carrying Tax Temporary
Value Base Difference
GHC'000 GHC'000 GHC'000
Property, plant and equipment 460 270 190
Development expenditure 60 – 60
Interest receivable (55 – 45) 10 – 10
Provision (40) – (40)
220

43
(b) Notes to the statement of financial position
Deferred tax liability GHC'000
Accelerated depreciation for tax purposes [(190 – 90) × 30%] 30
Product development costs deducted from taxable profit (60 × 30%) 18
Interest income taxable when received (10 × 30%) 3
Provision for environmental costs deductible when paid (40 × 30%) (12)
Revaluations (90 × 30%) 27
66

GHC'000
At 1 January 2014 [(310 – 230) × 30%] 24
Amount charged to income statement (balancing figure) 15
Amount charged to equity (90 × 30%) 27
At 31 December 2014 (220 × 30%) 66

Note to the income statement Income tax expense GHC'000


Current tax 45
Deferred tax 15
60

QUESTION 7
(a) Explain, with examples, the nature and purpose of deferred taxation. (10 marks)
(b) The information below relates to Giovani Ltd for the year ended 31 March 2013.
The balance on the provision for deferred taxation account at 1 April 2012 was GHC35,000. This
represented taxation at 35% on cumulative timing differences of GHC100,000 at 1 April 2012.
Capital allowances (tax depreciation) and depreciation for the year ending 31 March 2013 are as
follows.
Capital allowances Depreciation
GHC'000 GHC'000
2013 (actual) 100 90
The income tax rate for 2013 is 30% and is expected to remain at this level for the for the
foreseeable future.
Required
State, with reasons, how to account for deferred tax in the year ended 31 March 2013. (5 marks)
(Total = 15 marks)
Solution:
(a) IAS 12 Income taxes prescribes the accounting treatment for income taxes including the
recognition of deferred tax assets and liabilities. These assets and liabilities arise due to
temporary differences between the tax base of an asset or liability and its carrying amount in the
statement of financial position. The tax base of an asset or liability is the amount attributed to
that asset or liability for tax purposes. Temporary differences may be either taxable or
deductible.

Taxable temporary differences will result in taxable amounts in determining taxable profit (loss)
of future periods when the carrying amount of the asset or liability is recovered or settled.

44
Deductible temporary differences will result in amounts that are deductible in determining
taxable profit (tax loss) of future periods when the carrying amount of the asset is recovered or
settled.

IAS 12 identifies the main categories in which temporary differences can occur, which include
the following:
(i) Interest revenue is included in accounting profit on a time proportion basis but may, in some
jurisdictions, be included in taxable profit when cash is collected. The tax base of any receivable
recognised in the statement of financial position with respect to such revenues is nil because the
revenues do not affect taxable profit until cash is collected;
(ii) Depreciation used in determining taxable profit (tax loss) may differ from that used in
determining accounting profit. The temporary difference is the difference between the carrying
amount of the asset and its tax base which is the original cost of the asset less all deductions in
respect of that asset permitted by the taxation authorities in determining taxable profit of the
current and prior periods. A taxable temporary difference arises, and results in a deferred tax
liability, when tax depreciation is accelerated (if tax depreciation is less rapid than accounting
depreciation, a deductible temporary difference arises, and results in a deferred tax asset);
(iii) Development costs may be capitalised and amortised over future periods in determining
accounting profit but deducted in determining taxable profit in the period in which they are
incurred. Such development costs have a tax base of nil as they have already been deducted
from taxable profit. The temporary difference is the difference between the carrying amount of
the development costs and their tax base of nil.
(iv) Retirement benefit costs may be deducted in determining accounting profit as service is
provided by the employees, but deducted in determining taxable profit either when
contributions are paid to a fund by the entity or when retirement benefits are paid by the entity.
A temporary difference exists between the carrying amount of the liability and its tax base; the
tax base of the liability is usually nil. Such a deductible temporary difference results in a
deferred tax asset as economic benefits will flow to the entity in the form of a deduction from
taxable profits when contributions or retirement benefits are paid.
(v) Research costs are recognised as an expense in determining accounting profit in the period
in which they are incurred but may not be permitted as a deduction in determining taxable
profit (tax loss) until a later period. The difference between the tax base of the research costs,
being the amount the taxation authorities will permit as a deduction in future periods, and the
carrying amount of nil is a deductible temporary difference that results in a deferred tax asset;
(vi) Certain assets may be carried at fair value, or may be revalued, without an equivalent
adjustment being made for tax purposes. A deductible temporary difference arises if the tax
base of the asset exceeds its carrying amount.

(b) Giovani Ltd


Timing difference at 31 March 2013:
GHC
Timing differences b/f at 1 April 2012 100
Arising in year (100 – 90) 10
110
Deferred tax liability will be 110 × 30% = GHC33,000
This figure will be included in the statement of financial position. The decrease in the provision
of (GHC35,000 – GHC33,000) = GHC2,000 will reduce the tax charge for the year.

45
QUESTION 8
(a) IAS 12 Income Taxes details the requirements relating to the accounting treatment of deferred
taxes.
Required
Explain why it is considered necessary to provide for deferred tax and briefly outline the
principles of accounting for deferred tax contained in IAS 12 Income taxes.
(4 marks)

(b) Boatemaa Plc purchased an item of plant for GHC2,000,000 on 1 October 2010. It had an
estimated life of eight years and an estimated residual value of GHC400,000. The plant is
depreciated on a straight-line basis. The tax authorities do not allow depreciation as a
deductible expense. Instead a tax expense of 40% of the cost of this type of asset can be claimed
against income tax in the year of purchase and 20% per annum (on a reducing balance basis) of
its tax base thereafter. The rate of income tax can be taken as 25%.
Required
In respect of the above item of plant, calculate the deferred tax charge/credit in Boatemaa's
income statement for the year to 30 September 2013 and the deferred tax balance in the
statement of financial position at that date. (6 marks)
Note. Work to the nearest GHC'000. (Total = 10 marks)

Solution:
(a) Principles of deferred tax
In many countries different rules are used for calculating accounting profit (as used by
investors) and taxable profit. This can give rise to temporary differences.
Temporary differences arise when income or expenditure is recognised in the financial
statements in one year, but is charged or allowed for tax in another. Deferred tax needs to be
provided for on these items.
The most important temporary difference is that between depreciation charged in the financial
statements and capital allowances in the tax computation. In practice capital allowances tend to
be higher than depreciation charges, resulting in accounting profits being higher than taxable
profits. This means that the actual tax charge (known as current tax) is too low in comparison
with accounting profits. However, these differences even out over the life of an asset, and so at
some point in the future the accounting profits will be lower than the taxable profits, resulting
in a relatively high current tax charge.
These differences are misleading for investors who value companies on the basis of their post
tax profits (by using EPS for example). Deferred tax adjusts the reported tax expense for these
differences. As a result the reported tax expense (the current tax for the period plus the deferred
tax) will be comparable to the reported profits, and in the statement of financial position a
provision is built up for the expected increase in the tax charge in the future.
There are many ways that deferred tax could be calculated. IAS 12 states that the liability
method should be used. This provides for the tax on the difference between the carrying value
of an asset (or liability) and its tax base. The tax base is the value given to an asset (or liability)
for tax purposes. The deferred tax charge (or credit) in the income statement is the increase (or
decrease) in the provision reported in the statement of financial position.
(b) Boatemaa Plc
The provision for deferred tax in Boatemaa's statement of financial position at 30 September
2013 will be the potential tax on the difference between the accounting carrying value of

46
GHC1,400,000 and the tax base of GHC768,000. The difference is GHC632,000 and the tax on the
difference is GHC158,000.
The charge (or credit) for deferred tax in profit or loss is the increase (or decrease) in the
provision during the year. The closing provision of GHC158,000 is less than the opening
provision of GHC160,000, so there is a credit for GHC2,000 in respect of this year.
Movement in the provision for deferred tax for the year-ending 30 September 2013
GHC
Opening provision 160,000
Credit released to the income statement (2,000)
Closing provision 158,000

Workings
Accounting Tax base Difference Tax @ 25%
Carrying value
Y/E 09/11 GHC GHC GHC GHC
Purchase Cost 2,000,000 2,000,000 – –
Depreciation W1 (200,000) W2 (800,000)
Balance 1,800,000 1,200,000 600,000 150,000
Y/E 09/12
Depreciation (200,000) W3 (240,000)
Balance 1,600,000 960,000 640,000 160,000
Y/E 09/13
Depreciation (200,000) W4 (192,000)
Balance 1,400,000 768,000 632,000 158,000

(W1) GHC2,000,000 cost - GHC400,000 residual value over 8 years.


(W2) GHC2,000,000 x 40%
(W3) GHC1,200,000 x 20%
(W4) GHC960,000 x 20%

QUESTION 9
The draft financial statements for the year ended 31 March 2015 for Kobby Ltd include
the following:
GH¢000
Statement of comprehensive income (extract)
Income tax expense 850
Notes to the accounts:
Over provision for the year to 31 March 2014 (50)
Estimate of tax due for the year to 31 March 2015 700
Increase in deferred tax provision for the year to 31 March 2015 200
850
Statement of cash flows (extract)
Tax paid in the year to 31 March 2015 600

47
Required:
i)Explain how deferred tax arises. (2 marks)
ii)Identify the most likely reason for the increase of GH¢200,000 in the deferred tax provision
for the year to 31 March 2015. (2 marks)
iii) Explain what the over provision of GH¢50,000 in the income statement represents.
(2 marks)
ICAG MAY,2017
Solution:
i)Deferred tax is the estimated future tax consequences of transactions and events that have
been recognized in the financial statements of the current and previous periods. Deferred tax
arises due to the temporary differences between the accounting profit and the taxable profit.
The temporary differences cause the carrying value of some items in the statement of financial
position to be different from their tax base (the amount recognized for tax calculation).

ii)Kobby Ltd‘s statement of comprehensive income shows an increase in deferred tax, this
suggests that temporary differences in the year to 31/3/2015. The main reason was probably an
increase in non-current assets causing the tax depreciation to increase more than the accounting
depreciation for the year to 31/3/2015, thus causing the increase of GH¢200,000 in deferred tax
provision.

iii)Current tax is the estimated amount of corporate income tax payable on the taxable profits of
the entity for the period. The amount of current tax is accrued in the financial statements and
carried forward as a current liability to the next accounting period when it will be paid. When
the tax is paid there will usually be a difference between the amount paid and the amount
accrued.
If the amount paid is less than the amount accrued there will be an over provision of income
tax. The amount over provided will be an adjustment to the income tax expense in the following
period. In Kobby Ltd, the current tax estimate for year to 31 March2014 was GH¢650,000, the
statement of cash flows shows that GH¢600,000 was paid in the following period leaving a
balance of GH¢50,000 over provided.

QUESTION 10
Daaho Ltd (Daaho) manufactures and distributes security equipment. Daaho prepares financial
statements in accordance with International Financial Reporting Standards (IFRS) up to 31
August each year.
On 31 August 2019, the taxation liability account in the books of Daaho Ltd showed a debit
balance of GH¢17,500 after paying the 2018 liability. The estimated liability for 2019 is
GH¢84,500 and no entry has yet been made to record this.
Required:
Explain the appropriate accounting treatment of the above transaction for the year end 31
August 2019. (3 marks)

SOLUTION 10
A debit balance on the tax account (GH¢17,500) represents under provision of tax for the
previous year (2018). This must be added to the income tax charge for the current year.

48
treat as income tax expense for 2019. Thus, GH¢102,000 (GH¢17,500+ GH¢84,500) to be
recorded as income tax expense for 2019.

statement of financial position.

TRY
Question 1
Simple has estimated its income tax liability for the year ended 31 December 2015 at
GH¢180,000. In the previous year the income tax liability had been estimated as GH¢150,000
Required:
Calculate the tax charge that will be recognised in the statement of profit or loss for the year
ended 31 December 2015 if the amount that was actually agreed and settled with the tax
authorities in respect of 2014 was:
(a) GH¢165,000
(b) GH¢140,000
State what the income tax liability would be at 31 December 2015 in each of the above
circumstances.

Question 2
Parker Ltd‟s statement of financial position includes a number of assets (and liabilities that give
rise to temporary differences as follows at the current reporting date:
Carrying value Tax base
GH¢‟000 GH¢‟000
Property, plant and equipment (1) 26,500 18,000
Held to maturity financial assets (2) 1,020 1,000
Trade receivables (3) 5,700 6,500
Warranty provision (4) (1,200) 0
Long-term borrowings (2) (22,200) (19,500)

1. Property, plant and equipment is depreciated in the financial statements on a straiGH¢t


line basis over the assets‟ useful lives. Tax depreciation is 30% on a reducing balance basis.
2. The held to maturity asset and long-term borrowings are measured using the amortised
cost method in the financial statements. Tax is payable/receivable on interest arid any
redemption premium on a cash received/paid basis and therefore a temporary difference arises.
3. Trade receivables have a gross receivables balance of GH¢65 million (however Parker
Limited has created a specific allowance against GH¢800,000 which is four months old at the
reporting date, the debts only become tax deductible after 12 months.
4. Parker Limited offers one year warranties on its products. Warranty costs are tax
deductible when warranty repairs are incurred. The corporate income tax rate is 25%.
The corporate income tax rate is 25%.
Required:
Calculate the deferred tax balance on the above assets and liabilities at the reporting date,
clearly stating for each item whether the deferred tax balance would be an asset or liability

49
Question 3
Cartier Ltd, a multinational operating in Ghana purchased a plant for GH¢600,000 on 1 January
2015. Cartier Ltd depreciates its plant using the straight-line method over 15 years, assuming a
residual value of 10% of original cost. Cartier Ltd claims all available tax depreciation
allowances. On 1 January 2016, Cartier Ltd revalued the plant and increased its carrying value
by GH¢50,000. The asset‟s useful life was not affected. Assume there were no other temporary
differences in the period.
Required:
i) Calculate the amount of Cartier Ltd‟s deferred tax balance at 31 December 2016 in accordance
with IAS 12 Income Taxes.
ii) Calculate the change in Cartier Ltd‟s deferred tax balance for the year ended 31 December
2016 and explain how the change would be treated in Cartier Ltd‟s statement of profit or loss for
the year to 31 December 2016.

(Note: Assume an applicable tax rate of 25% and capital allowance of 50% of carrying amount in
the first year and 25% in the second year).

Question 4
On 1 October 2016, Abudu Ltd decided to revalue its land for the first time. The land was
originally purchased six years ago for GH¢65,000 and was revalued to its current market value
of GH¢80,000 on 1 October 2016. The difference between Abudu Ltd‟s Net assets (including
revaluation of land) and the lower tax base at 30 September 2017 was GH¢27, 000. The opening
deferred tax liability at 1 October 2016 was GH¢2,600 and Abudu Ltd‟s tax rate is 25%

Required: Explain how to account for the above transaction in the financial statements of Abudu
Ltd for the year to 30 September 2017. (5 marks)
Solutions

revaluation surplus on land = (80,000-65,000)x 25% = 3,750.


-GH¢2,600 =GH¢4,150 OF
WHICH GH¢3,750 should be charged to OCI and the remainder charged to profit or loss
account.
Journal Entry: GH¢ GH¢
Dr income tax expense (SPL) 400
Dr Income tax on OCI (SOCI) 3,750
Cr deferred tax liability (SFP) 4,150

50
PROPERTY, PLANT & EQUIPMENT (IAS 16)
The objective of this Standard is to prescribe the accounting treatment for property, plant and
equipment so that users of the financial statements can discern information about an entity‘s
investment in its property, plant and equipment and the changes in such investment. The
principal issues in accounting for property, plant and equipment are the recognition of the
assets, the determination of their carrying amounts and the depreciation charges and
impairment losses to be recognised in relation to them.
Scope
IAS 16 applies to the accounting for property, plant and equipment, except where another
standards requires or permits differing accounting treatments, for example:
 Assets classified as held for sale in accordance with IFRS 5 Non-current Assets Held for
Sale and Discontinued Operations
 Biological assets related to agricultural activity accounted for under IAS 41 Agriculture
 Exploration and evaluation assets recognised in accordance with IFRS 6 that‘s Exploration
for and Evaluation of Mineral Resources mineral rights and mineral reserves such as oil,
natural gas and similar non-regenerative resource
Property, plant and equipment are tangible items that:
(a)are held for use in the production or supply of goods or services, for rental to others, or for
administrative purposes; and
(b)are expected to be used during more than one period.
Definitions
Cost
It is the amount of cash or cash equivalents paid or the fair value of the consideration
transferred to acquire, purchase or construct an asset.
Depreciation
Depreciation is the systematic allocation of the depreciable amount of an asset over its useful
life.
Carrying Value
It is the value at which a tangible asset will be presented in the statement of financial position,
at the end of the reporting period, and it is determined as Cost less Accumulated Depreciation and
Accumulated Impairment Loss.
An impairment loss is the amount by which the carrying amount of an asset or a cash-
generating unit exceeds its recoverable amount.
The recoverable amount of an asset or a cash-generating unit is the higher of its fair value less
costs to sell and its value in use

Depreciable Amount
It is the amount of an asset, which will be depreciated over its useful life and is determined as
the cost of an asset less its residual value.
Useful Life
It is the period of time for which asset will be used by the management.
Recognition:
The cost of an item of property, plant and equipment shall be recognised as an asset if, and only
if:

51
(a)it is probable that future economic benefits associated with the item will flow to the entity;
and
(b) the cost of the item can be measured reliably.

Measurement at recognition: An item of property, plant and equipment that qualifies for
recognition as an asset shall be measured at its cost. The cost of an item of property, plant and
equipment is the cash price equivalent at the recognition date. If payment is deferred beyond
normal credit terms, the difference between the cash price equivalent and the total payment is
recognised as interest over the period of credit unless such interest is capitalised in accordance
with IAS 23.

A tangible non-current asset should initially be measured at cost. Cost is not simply the
purchase price of the asset. The cost should include the following items:
 Purchase costs, including import duties and non-refundable purchase taxes (for
example, non-refundable value added tax: however, purchase taxes are not included in
cost if they are refundable to the entity).
Costs directly related to getting the asset ready for use, including:
− professional fees (for example, the fees of architects or surveyors, and legal costs)
− costs of site preparation (getting a site ready for installation; for example, the costs of levelling
a factory floor before new machinery is installed)
− installation costs
− costs of testing the asset (for example, the costs of safety test flights for a new aeroplane).
 Dismantling costs. The initial estimate of the costs of dismantling and removing the
asset at the end of its expected life, plus the costs of restoring the site after use when an
obligation to incur these future costs arises at the time the asset is acquired.
Where the asset has been constructed by the entity, rather than purchased from an external
supplier, the cost should also include the employee costs that directly relate to the construction
and production overhead costs, but not administration and general overhead costs.

Dismantling costs
When a company acquires an asset it may have an obligation to dismantle the asset and restore
the site once the asset reaches the end of its life. The obligation must be included in the cost of
the asset, at the time that the asset is first recognised.
For example, a company that builds an oil rig in the sea may have an obligation to restore any
damage to the sea-bed once it has finished drilling for oil. A cost for future site restoration work
should be included in the cost of the oil rig, when the rig is initially recognised as an asset: this
cost should be the present value of the estimated future costs. As a part of the cost of the asset, it
will then be depreciated over the asset‘s life.
This treatment ensures that the restoration costs are matched to the associated income earned
from the rig during the rig‘s useful life. (Unless the future restoration costs are capitalised and
depreciated, the cost would eventually have to be accounted for as a ‗one-off‘ item of expense at
the end of the asset‘s life. This would be inconsistent with the accruals concept of accounting.)

Example
Hazard PLC has incurred the following costs prior to bringing a machine into full production:
GHC
Purchase price before trade rebate and tax 12,200

52
Trade rebate 600
Refundable purchase taxes 1,200
Architects‘ fees 700
Installation costs 100
Operating losses prior to achieving planned performance 250
Required
Calculate the amount at which the machine should be stated in the accounting records of
Hazard PLC in accordance with IAS 16.

Answer
The cost of the machine should be stated in the statement of financial position as:
GHC
Purchase price less trade rebate (12,200 – 600) 11,600
Architects‘ fees 700
Installation costs 100
Total cost 12,400
For self-constructed assets (non-current assets that are constructed by the entity itself):
 internal profits and abnormal costs should be excluded from cost
 administrative expenses and other similar overheads should be excluded from cost
 interest costs incurred in the course of construction might be included, in accordance
with IAS 23 Borrowing costs (see below).

**The cost of an item of property, plant and equipment comprises:


(a)its purchase price, including import duties and non-refundable purchase taxes, after
deducting trade discounts and rebates.
(b)any costs directly attributable to bringing the asset to the location and condition necessary
for it to be capable of operating in the manner intended by management.
(c)the initial estimate of the costs of dismantling and removing the item and restoring the site
on which it is located, the obligation for which an entity incurs either when the item is
acquired or as a consequence of having used the item during a particular period for purposes
other than to produce inventories during that period.

Example:
Paul Pogba incurs the following costs in relation to the construction of a new factory and
the introduction of its products to the local market.
GHC‘000
Site preparation costs 240
Materials used 1,500
Labour costs, including GHC90,000
incurred during an industrial dispute. (No
construction occurred during the period of 3,190
the dispute).
Testing of various processes in factory 150
Consultancy fees re installation of 220
equipment
Relocation of staff to new factory 110
General overheads 500

53
Costs to dismantle the factory at end of its
useful life in 10 years time 100
Required: How much of the costs should be capitalised?
Solution
GHC’000
Site preparation costs 240
Materials used 1,500
Labour costs (3,190 - 90) 3,100
Testing of various processes in factory 150
Consultancy fees re installation of equipment 220
Relocation of staff to new factory –
General overheads –
Costs to dismantle the factory at end of its useful
life in 10 years time 100
5,310

NOTE:
There are a number of costs which arise subsequent to acquisition which may be capitalised
throughout the life of the asset. Enhancement costs which significantly enhance the economic
benefits by increasing the capacity, improving the quality of output, extending the economic life
of the asset or by reducing the operating costs of the assets can be capitalised. The replacement
costs of major components and overhaul costs which improve the economic benefit that can be
generated can also be capitalised. Where PPE consists of a number of assets of different
economic lives, it may be appropriate to recognise and account for each component separately
for depreciation and inclusion of subsequent expenses. The component approach is also applied
where regular major inspections of an asset are a condition of continuing to use it. The cost of
each inspection is treated as a separate item (replacement) of PPE, provided recognition criteria
are satisfied. Any remaining carrying amount in respect of the previous inspection is
derecognised.
Measurement after recognition: An entity shall choose either the cost model or the revaluation
model as its accounting policy and shall apply that policy to an entire class of property, plant
and equipment.

Cost model: After recognition as an asset, an item of property, plant and equipment shall be
carried at its cost less any accumulated depreciation and any accumulated impairment losses.

Revaluation model: After recognition as an asset, an item of property, plant and equipment
whose fair value can be measured reliably shall be carried at a revalued amount, being its fair
value at the date of the revaluation less any subsequent accumulated depreciation and
subsequent accumulated impairment losses.
Revaluations shall be made with sufficient regularity to ensure that the carrying amount does
not differ materially from that which would be determined using fair value at the end of the
reporting period.
If an asset‘s carrying amount is increased as a result of a revaluation,the increase shall be
recognised in other comprehensive income and accumulated in equity under the heading of
revaluation surplus. However, the increase shall be recognised in profit or loss to the extent that
it reverses a revaluation decrease of the same asset previously recognised in profit or loss. If an

54
asset‘s carrying amount is decreased as a result of a revaluation,the decrease shall be recognised
in profit or loss. However, the decrease shall be recognised in other comprehensive income to
the extent of any credit balance existing in the revaluation surplus in respect of that asset.
Illustration 1
ABC Company has an item of land carried in its books at GHC13,000.Two years ago a slump in
land values led to the company to reduce the carrying value from GHC15,000.This was taken as
an expense in the income statement. There has been a surge in land prices in the current
year,however,and the land is now worth GHC20,000.
Account for the revaluation in the current year.(ignore depreciation)

Solution:
Land – 2 years Ago
GHC
Balance b/f 15,000
Revalued Amount 13,000
Decrease/Loss(Revaluation Loss) 2,000

Dr Income Statement GHC2,000


Cr Land GHC2,000

Current Year
GHC
Carrying Value 13,000
Revalued Amount 20,000
Increase /Revaluation surplus 7,000

Dr Asset (Land) GHC 7,000


Cr Income Statement GHC2,000
Cr Revaluation Surplus GHC5,000

2.Let us swap round the example above.The original cost was GHC15,000,revalued upwards to
GHC20,000 two years ago.The value has fallen to GHC 13,000.
Account for the decrease in value.(Ignore depreciation)

Solution:
Land – 2 years ago
GHC
Cost 15,000
Revalued Amount 20,000
Revaluation Surplus 5,000

Dr Land GHC5,000
Cr Revaluation Surplus GHC5,000

Current Year
GHC
Carrying Value 20,000

55
Revalued Amount 13,000
Revaluation Loss 7,000

Dr Revaluation Surplus GHC5,000


Dr Income Statement GHC2,000
Cr Land GHC7,000

Depreciation:
Depreciation is the systematic allocation of the depreciable amount of an asset over its useful
life. Depreciable amount is the cost of an asset, or other amount substituted for cost, less its
residual value. Each part of an item of property, plant and equipment with a cost that is
significant in relation to the total cost of the item shall be depreciated separately. The
depreciation charge for each period shall be recognised in profit or loss unless it is included in
the carrying amount of another asset. The depreciation method used shall reflect the pattern in
which the asset‘s future economic benefits are expected to be consumed by the entity. The
residual value of an asset is the estimated amount that an entity would currently obtain from
disposal of the asset, after deducting the estimated costs of disposal, if the asset were already of
the age and in the condition expected at the end of its useful life.

Illustration 3
(a) Asset X was bought for GHC500,000 five years ago and has been depreciated at 10% on cost
per annum. It is now revalued at GHC800,000. There is no change to the useful life.

(b) An item of land originally cost GHC30,000. Two years ago it was revalued to GHC35,000.
The value of the land has now fallen to GHC29,000. Assume the profit for the year before
adjusting for the fall in value of the land was GHC60,000.

Required: Show how the above items should be treated in the financial statements.
Solution
(a) GHC GHC
Debit Asset X – cost/valuation (GHC800,000 - GHC500,000) 300,000
Debit Asset X – accumulated depreciation (GHC500,000 x 5 x10%) 250,000
Credit Revaluation surplus 550,000

Statement of financial position (extract) GHC


Property, plant and equipment 800,000

Revaluation surplus 550,000

The revaluation increase should be shown as other comprehensive income in the entity‘s SCI.
Note: you could also increase the carrying value to GHC800,000 by debiting GHC550,000 and
crediting revaluation surplus with same.

(b)
When the land was revalued two years ago, the revaluation surplus of GHC5,000 would have
been recognised as other comprehensive income and credited to the revaluation surplus as part
of equity. The asset has fallen by GHC6,000, of which GHC5,000 should reverse the previous

56
revaluation surplus via other comprehensive income and the remaining GHC1,000 should be
recognised in profit or loss.
GHC GHC
Debit Revaluation surplus 5,000
Debit SCI 1,000
Credit Asset value 6,000

Statement of comprehensive income (extract)


GHC
Profit for year (GHC60,000 - GHC1,000) 59,000
Other comprehensive income:
Loss on land revaluation (5,000)
Total comprehensive income for the year 54,000

Statement of changes in equity


Retained Revaluation Total
Earnings Surplus
GHC GHC GHC
Total comprehensive income for the year 59,000 (5,000) 54,000
Where an asset has been revalued, the deprecation charge is based on the revalued amount, less
any residual value, from the date of revaluation. The whole of the depreciation charge is
recognised in profit or loss via the SCI. None is recognised in other comprehensive income and
consequently set against the revaluation surplus.

However, IAS 16 permits, and it is considered best practice to make, a transfer between
reserves, of the ‗excess‘ depreciation arising as a result of the revaluation. The effect is that SCI
shows the economic benefit consumed, but distributable profits are not affected by extra
depreciation on revalued assets. The amount of transfer is actual depreciation charged less
equivalent charge based on original historical cost of assets (Debit revaluation surplus, Credit
retained earnings). The transfer is shown in the statement of changes in equity.

A CHANGE IN THE METHOD OF MEASURING DEPRECIATION


IAS 16 requires depreciation methods to be reviewed at least annually. It only allows a change
in the method of measuring depreciation where this would give a fairer presentation of the
entity‘s financial results and financial position.
Where there is a change in the depreciation method used, this is a change in accounting
estimate. A change of accounting estimate is applied from the time of the change, and is not
applied retrospectively. The carrying amount (cost minus accumulated depreciation) of the
asset at the date of the change is written off over the remaining useful life of the asset.

Example
Patrick Ltd owns a machine which originally cost GHC30,000 on 1 January,2011. It has no
residual value. It was being depreciated over its useful life of 10 years on a straight-line basis.
At the end of 2014, when preparing the financial statements for 2014,Patrick Ltd decided to
change the method of depreciation, from straight-line to the reducing balance method, using a
rate of 25%.
Required

57
Calculate the depreciation charge for 2014.

Solution:
The change in accounting estimate is made at the end of 2014, but is applied to the financial
statements from 1 January 2014. The reducing balance method of depreciation is applied to the
2014 statements.
GHC
Cost on 1 January 2011 30,000
Depreciation for 2011 to 2013 (30,000 × 3/10) (9,000)
–––––––
Carrying amount at end of 2013 21,000
–––––––
Depreciation for 2014 will therefore be GHC21,000 × 25% = GHC5,250.
A CHANGE IN THE ASSET’S USEFUL LIFE
IAS 16 requires useful lives to be reviewed at each year-end. Any change is a change in
accounting estimate. The carrying amount (cost minus accumulated depreciation) of the asset
at the date of change is written off over the (revised) remaining useful life of the asset.

Example
Maxwell Ltd owns a machine which originally cost GHC60,000 on 1 January,2011. It has no
residual value. It was being depreciated over its useful life of 10 years on a straight-line basis.
On 31 December,2014 Maxwell Ltd revised the total useful life for the machine to eight years.
Required
Calculate the depreciation charge for 2014 and subsequent years.

Solution:
The change in accounting estimate is made at the end of 2014 but may be applied to the
financial statements from 2014 onwards.
GHC
Cost on 1 January 2011 60,000
Depreciation for 2011 to 2013 (60,000 × 3/10) (18,000)
–––––––
Carrying amount at end of 2013 42,000
–––––––
Remaining useful life at the end of 2013 = 8 – 3 years = 5 years.
Depreciation for Year 4 and subsequent years will be GHC42,000 ÷ 5 years = GHC8,400.
Example
Boafo Bottling Ltd owns a bottling plant with an original cost of GHS200,000. It was acquired
in January 2007. On acquisition, management determined that the useful life was 10 years and
the residual value would be GHS20,000. The asset is now 8 years old , and during this time
there have been no revisions to the assessed residual value. At the end of year 8 (as at 31
December 2014), management has reviewed the useful life and residual value and has
determined that the useful life [from date of acquisition] can be extended to 12 years in view of
the maintenance program adopted by the company. As a result, the residual value will reduce
to GHS10,000.
Required

58
Determine how the asset will be accounted for in the 2015 financial statements ( Statement
of Profit and Loss for 2015 and Statement of financial position as at 31 December 2015)
[5 marks]
Solution:
Workings
Accumulated Depreciation: 200,000 – 20,000 x 8years
10 years
Carrying Value as at 31/12/14 (200,000-144,000) 56,000
The assets life and residual value were revised on 31/12/14 to 12 years and GHC10,000
Therefore remaining life will be 4 years instead of 2 years
Annual Depreciation for 2015 year = 56,000-10,000
4 years
= GHC11,500
Income statement Extract for 2015
GHC
Depreciation 11,500

SOFP Extract as at 31/12/15


GHC
Bottling Plant 200,000
Less Accumulated Depn.(144,000 + 11,500) 155,500
44,500

Question(Revaluation at different dates)


A company has an item of building that cost GHC10,000 on 1st January,2010 with useful
economic life of 50years.Assuming the asset is revalued for GHC9,900 at the following dates;
 At Start(on 1st January,2015)
 Mid-year (on 30th June,2015)
 End of the year (on 31st December,2015)

You are required to account for the impact of the revaluation on the financial
statements(Both Comprehensive Income statement and statement of financial position
extracts).

Solution:
Revaluation at Start (1st January,2015)
GHC
Cost 10,000
Less Acum Depn(10,000/50) x 5 1,000
Carrying Value 9,000
Revalued Amount 9,900
Surplus 900

Comprehensive Income Statement Extract for 2015


GHC
Annual Depreciation Charge(9900/45) 220
Other Comprehensive Income:

59
Revaluation surplus 900

Statement of Financial Position Extract for 2015


GHC
Non-Current Assets:
PPE at Revalued Amount 9900
Less Accum depn 220
9,680
Equity:
Revaluation Surplus 900

Revaluation at Mid-Year (30th June,2015)


GHC
Cost 10,000
Less Accum Depn(10,000/50 x 5.5) 1,100
Carrying Value 8,900
Revalued Amount 9,900
Surplus 1,000

Comprehensive Income Statement extract for 2015


GHC
Annual Depreciation charge :
1/1/15 – 30/06/15 ; (10,000/50 x 6/12) 100
1/07/15 – 31/12/15; (9,900/44.5 x 6/12) 111 211
Other Comprehensive Income:
Revaluation Surplus 1,000

Statement of Financial Position Extract for 2015


Non-Current Assets:
PPE @ Revalued Amount 9,900
Accum depn 111
9,789
Equity:
Revaluation Surplus 1,000

Revaluation at Year end (31st December,2015)


GHC
Cost 10,000

60
Less Accum depn (10,000/50 x 6) 1,200
Carrying Value 8,800
Revalued Amount 9,900
Revaluation Surplus 1,100

Comprehensive Income Statement Extract for 2015


GHC
Annual Depn charge(10,000/50) 200
Other Comprehensive Income:
Revaluation Surplus 1,100

Statement of Financial Position for 2015


Non-Current Assets: GHC
PPE at Revalued Amount 9,900
Accum Depn -
Carrying Value 9,900

Equity:
Revaluation surplus 1,100

Question 1 (Transfers to Retained Earnings)


A company revalued its buiding at the start of the year to GHC 6,000,000.00.The buildings cost
GHC 4,000,000.00 ten years prior to the revaluation.The total expected useful life of 50years is
unchanged.The company‘s policy is to make annual transfer of realized amounts to Income
Surplus/Income Statement.Show the effect of the above in the financial statements for the year.

Solution:
GHC‘000
Cost 4,000
Accum Depn (4,000/50 x 10) 800
Carrying Value 3,200
Revalued Amount 6,000
Surplus 2,800
Excess Depreciation:
GHC’000
Dpreciation Based on Cost(4,000/50) 80
Depreciation Based on Revalued Amount(6,000/40) 150
Excess Depreciation (transfer to income surplus) 70

Alternatively:
Depreciation effect = 2,800,000/40 =GHC70,000
Statement of Changes in Equity (Extract)
Stated Capital Capital Surplus Share Deals Income Surplus
GHC000 GHC000 GHC000 GHC000

61
Surplus 2,800
Transfer (70) 70

Question 2
An item of PPE was purchased for GHC900,000 on 1 January 2007. It is estimated to have a
useful life of 10 years and is depreciated on a straight line basis. On 1 January 2009, the asset is
revalued to GHC960,000. The useful life remains unchanged at ten years. The company‘s policy
is to make annual transfer of realized amounts to Income Surplus/Income Statement
GHC
Actual depreciation for 2009 based on revalued amount (960,000/8) 120,000
Depreciation for 2009 based on historical cost (900,000/10) (90,000)
Difference 30,000

In the SCI for 2009, a depreciation expense of GHC120,000 will be charged. A reserve transfer,
which will be shown in the statement of changes in equity, may be undertaken as follows:
Debit revaluation surplus 30,000
Credit retained earnings 30,000

The closing balance on the revaluation surplus on 31 December 2009 will therefore be as
follows:
Balance arising on revaluation (GHC960,000 - GHC720,000) 240,000
Transfer to retained earnings (30,000)
210,000
DERECOGNITION
An item of PP&E is removed from the balance sheet (that is, derecognized) when it is disposed
of or when no future economic benefits are expected from its use or disposal. The gain or loss
arising from derecognition is included in profit or loss when the item is derecognized;
moreover, gains are not to be classified as revenue. To determine whether an item of property,
plant and equipment is impaired, an entity applies IAS 36 Impairment of Assets.
The carrying amount of an item of property, plant and equipment shall be derecognised:
(a)on disposal; or
(b)when no future economic benefits are expected from its use or disposal.

Example:
A delivery van purchased by Willian Ltd on January 1,2005 was sold in January 2008 for GHC
80million and the proceeds credited to motor vehicles account.The van cost GHC150
million.Willian Ltd writes off such vehicles over 5 years; no depreciation charge in the year of
disposal.

Required;
Show how will treat this item in the company’s financial statements

Solution:
GH‘000
Cost 150,000
Accum depn (150/5 x 3) 90,000
Carrying Value 60,000

62
Disposal Value 80,000
Profit on disposal 20,000

Income Statement
GHC‘000
Depreciation (30,000)
Profit on disposal 20,000

SELF CONSTRUCTED PLANTS/ASSETS


The cost of a self-constructed asset is determined using the same principles as for an acquired
asset. If the asset is made available for sale by the entity in the normal course of business then
the cost of the asset is usually the same as the cost of producing the asset for sale. Thiscost
would usually be determined under the principles set out in IAS 2 Inventories.
The normal profit that an enterprise would make if selling the self-constructed asset would not
be recognised in ‗cost‘ if the asset were retained within the entity. Following similar principles,
where one group company constructs an asset that is used as PPE by another group company,
any profit on sale is eliminated in determining the initial carrying value of the asset in the
consolidated accounts (this will also clearly affect the calculation of depreciation).
If an item of PPE is exchanged in whole or in part for a dissimilar item of PPE then the cost of
such an item is the fair value of the asset received. This is equivalent to the fair value of the asset
given up, adjusted for any cash or cash equivalents transferred or received.

Example:
The following trial balance relates to Barcelona as at 30 September 2010:
GHC’000 GHC’000
Plant and equipment at cost (note (iii)) 67,400
Accumulated depreciation plant and equipment
– 30 September 2009 13,400

Additional Information:
Plant and equipment includes an item of plant bought for GHC10 million on 1 October 2009
that will have a 10-year life (using straight-line depreciation with no residual value). Production
using this plant involves toxic chemicals which will cause decontamination costs to be incurred
at the end of its life. The present value of these costs using a discount rate of 10% at 1 October
2009 was GHC4 million.Barcelona has not provided any amount for this future
decontamination cost. All other plant and equipment is depreciated at 12·5% per annum using
the reducing balance method.
Required:
Prepare Barcelona‘s Income Statement for the year ended 30th Septemeber,2010 and Statement
of Financial Position Extracts as at 30th September,2010.

Solution:
The new plant of GHC10 million should be grossed up by the provision for the present value of
the estimated future decontamination costs of GHC4 million to give a gross cost of GHC14
million. The ‗unwinding‘ of the provision will give rise to a finance cost in the current year of
GHC400,000 (4,000 x 10%) to give a closing provision of GHC4·4 million.

63
Depreciation of new plant (14,000/10 years) 1,400
Depreciation of existing plant and equipment
((67,400 – 10,000 – 13,400) x 12·5%) 5,500
The carrying amount of the plant and equipment will be:
New plant (14,000 – 1,400) 12,600
Existing plant and equipment (67,400 – 10,000 – 13,400 – 5,500) 38,500

Income Statement Extract


GHC
Depreciation:
New Plant 1,400
Existing Plant 5,500

Statement of Financial Position


GHC
Non Current Assets:
PPE(12,600+38,500) 51,100

CAPITALISATION OF BORROWING COSTS


Where an asset takes a substantial period of time to get ready for its intended use or sale then
the entity may incur significant borrowing costs in the preparation period. Under the accruals
basis of accounting there is an argument that such costs should be included as a directly
attributable cost of construction. IAS 23 Borrowing Costs was issued to deal with this issue.
IAS 23 states that borrowing costs that are directly attributable to the acquisition, construction
or production of a ‗qualifying asset‘ should be included in the cost of that asset. A ‗qualifying
asset‘ is one that necessarily takes a substantial period of time to get ready for its intended use
or sale.
Borrowing costs that would have been avoided if the expenditure on the qualifying asset had
not been undertaken are eligible for capitalisation under IAS 23. Where the funds are borrowed
specifically for the purpose of obtaining a qualifying asset then the borrowing costs that are
eligible for capitalisation are those incurred on the borrowing during the period less any
investment income on the temporary investment of those borrowings. Where the funds are
borrowed generally and used for the purpose of obtaining a qualifying asset then the entity
should use a capitalisation rate to determine the borrowing costs that may be capitalised. This
rate should be the weighted average of the borrowing costs applicable to the entity, other than
borrowings made specifically for the purpose of obtaining a qualifying asset. Capitalisation
should commence when:
● expenditures for the asset are being incurred;
● borrowing costs are being incurred;
● activities that are necessary to prepare the asset for its intended use or sale are in progress.
When substantially all the activities necessary to prepare the qualifying asset for its intended
use or sale are complete then capitalisation should cease.

Example:
On 1st January,2015, Antonio Conte Ltd began to construct a supermarket which had an
estimated useful life of 40 years. It purchased a leasehold interest in the site for
GHC250,000.The construction of the building cost GHC90,000 and the fixtures and

64
fittings cost GHC60,000.The construction of the supermarket was completed on 30th
September,2015 and it was brought into use on 1st January,2016.
Antonio Conte borrowed GHC400,000 on 1st January,2015 in order to finance this
project. The loan carried interest at 10% per annum. It was repaid on 30th June,2016.
Calculate the total amount to be included at cost in property, plant and equipment in
respect of the development at 31st December,2015.
Solution:
Property,Plant & Equipment
GHC‘000
Leasehold Site 250
Building 90
Fixtures and Fittings 60
Borrowing Costs (400x10%x9/12) 30
430

Presentation and disclosure


IAS 16 provides a long listing of disclosure requirements for property, plant, and equipment.
For each class of PPE, the financial statements must disclose the following:
•the measurement bases used for determining the gross carrying amount
•the depreciation methods and rates or useful lives
•the gross carrying amount and the accumulated depreciation (aggregated with accumulated
impairment losses) at the beginning and end of the period
•the detailed reconciliation of the carrying amount at the beginning and end of the period
(showing, for example, additions, depreciation, impairment losses, revaluation information,
foreign currency translation impacts, and so on)

The financial statements must also disclose restrictions and contractual commitments related to
PPE, as well as details of self-constructed assets, and any compensation received from third
parties for items of property, plant, and equipment that were impaired, lost, or given up. If
items of property, plant, and equipment are stated at revalued amounts, IAS 16 requires the
entity to disclose the following:
a)the effective date of the revaluation
b)whether an independent valuer was involved
c)the methods and significant assumptions applied in estimating the items’ fair values
d)the extent to which the items’ fair values were determined directly by reference to observable
prices in an active market or recent market transactions on arm’s-length terms or were
estimated using other valuation techniques
e)for each revalued class of property, plant, and equipment, the carrying amount that would
have been recognized had the assets been carried under the cost model
f)the revaluation surplus, indicating the change for the period and any restrictions on the
distribution of the balance to shareholders

QUESTION 3
RoyCo acquired a brand new property (land and buildings) on 1 January 2016 for GH¢40 million
(including GH¢15 million in respect of the land). The asset was revalued on 31 December 2017 to
GH¢43 million (including GH¢16.6 million in respect of the land). The buildings element was
depreciated over a 50-year useful life to a zero residual value. The useful life and residual value did

65
not subsequently need revision. On 31 December 2018 the property was revalued downwards to
GH¢35 million as a result of the recession (including GH¢14 million in respect of the land).
The company makes a transfer from revaluation surplus to retained earnings in respect of realised
profit.
Required:
Calculate the amounts recognised in profit or loss and in other comprehensive income for the years
ended 31 December 2017 and 31 December 2018. (6 marks)

SOLOTION 3

Revaluation of Property plant and equipment


Land Buildings Total
GH¢'000 GH¢'000 GH¢'000
Cost 1 January 2016 15000 25000 40,000
Accumulated Depreciation -1-1-16-31- 0 (1,000) (1,000)
12-17
(25,000/20yrs x 2 years)
Carrying amount 31-12-17 15,000 22,400 39,000
Revaluation gain (balance) 1,600 2,400 4,000
Revaluation amount 31-12-17 16,600 26,400 43,000
Depreciation charge (26,400 /48) ______ (550) (550)
16,600 25,850 42,450
Revaluation loss (balance) (2,600) (4,850) (7,450)
Revaluation amount 31-12-18 14,000 21,000 35,000

Transfer (excess depreciation) 24,000/48yrs =50


RoyCo
Statement of Profit or Loss and other Comprehensive income Extract for the years 31 st
December:
2018 2017
GH¢'000 GH¢'000
Expenses
Depreciation Charge 550 500 (25,000/20)
Revaluation loss (7,450-3,950) 3,500 0
Other comprehensive Income
Revaluation Surplus 0 4,000
Transfer (2400/48) (50)
(3,950) 3,950

66
EMPLOYEE BENEFITS (IAS 19)
Objective
The objective of this Standard is to prescribe the accounting and disclosure for employee
benefits.
Scope
Employee benefits include:
(a) Short-term employee benefits
(b) Post-employment benefits
(c) Other long-term employee benefits
(d) Termination benefits.
Definitions
Employee benefits are all forms of consideration given by an entity in exchange for service
rendered by employees or for the termination of employment.
Short-term employee benefits are employee benefits (other than termination benefits) that are
expected to be settled wholly before twelve months after the end of the annual reporting period
in which the employees render the related service.
Post-employment benefits are employee benefits (other than termination benefits and short-
term employee benefits) that are payable after the completion of employment.
Other long-term employee benefits are all employee benefits other than short-term employee
benefits, post-employment benefits and termination benefits.
Termination benefits are employee benefits provided in exchange for the termination of an
employee‗s employment as a result of either:
(a) an entity‗s decision to terminate an employee‗s employment before the normal retirement
date; or
(b) An employee‗s decision to accept an offer of benefits in exchange for the termination of
employment.

DEFINITIONS RELATING TO CLASSIFICATION OF PLANS


Post-employment benefit plans are formal or informal arrangements under which an entity
provides post-employment benefits for one or more employees.
Defined contribution plans are post-employment benefit plans under which an entity pays fixed
contributions into a separate entity (a fund) and will have no legal or constructive obligation to
pay further contributions if the fund does not hold sufficient assets to pay all employee benefits
relating to employee service in the current and prior periods.
Defined benefit plans are post-employment benefit plans other than defined contribution plans.
Multi-employer plans are defined contribution plans (other than state plans) or defined benefit
plans (other than state plans) that:
(a) pool the assets contributed by various entities that are not under common control; and
(b) use those assets to provide benefits to employees of more than one entity, on the basis that
contribution and benefit levels are determined without regard to the identity of the entity that
employs the employees.
DEFINITIONS RELATING TO THE NET DEFINED BENEFIT LIABILITY (ASSET)

67
The net defined benefit liability (asset) is the deficit or surplus, adjusted for any effect of
limiting a net defined benefit asset to the asset ceiling.
The deficit or surplus is:
(a) the present value of the defined benefit obligation less
(b) The fair value of plan assets (if any).
The asset ceiling is the present value of any economic benefits available in the form of refunds
from the plan or reductions in future contributions to the plan.
The present value of a defined benefit obligation is the present value, without deducting any
plan assets, of expected future payments required to settle the obligation resulting from
employee service in the current and prior periods.
Plan assets comprise:
(a) assets held by a long-term employee benefit fund; and
(b) Qualifying insurance policies.
Assets held by a long-term employee benefit fund are assets (other than non-transferable
financial instruments issued by the reporting entity) that:
(a) are held by an entity (a fund) that is legally separate from the reporting entity and exists
solely to pay or fund employee benefits; and
(b) are available to be used only to pay or fund employee benefits, are not available to the
reporting entity‗s own creditors (even in bankruptcy), and cannot be returned to the reporting
entity, unless either:
(i) the remaining assets of the fund are sufficient to meet all the related employee benefit
obligations of the plan or the reporting entity; or
(ii) the assets are returned to the reporting entity to reimburse it for employee benefits already
paid. A qualifying insurance policy is an insurance policy issued by an insurer that is not a
related party (as defined in IAS 24 Related Party Disclosures) of the reporting entity, if the
proceeds of the policy:
(a) can be used only to pay or fund employee benefits under a defined benefit plan; and
(b) are not available to the reporting entity‗s own creditors (even in bankruptcy) and cannot be
paid to the reporting entity, unless either:
(i) the proceeds represent surplus assets that are not needed for the policy to meet all the related
employee benefit obligations; or
(ii) the proceeds are returned to the reporting entity to reimburse it for employee benefits
already paid.
DEFINITIONS RELATING TO DEFINED BENEFIT COST
Service cost comprises:
(a) current service cost, which is the increase in the present value of the defined benefit
obligation resulting from employee service in the current period;
(b) past service cost, which is the change in the present value of the defined benefit obligation
for employee service in prior periods, resulting from a plan amendment (the introduction or
withdrawal of, or changes to, a defined benefit plan) or a curtailment (a significant reduction by
the entity in the number of employees covered by a plan); and
(c) any gain or loss on settlement.
Net interest on the net defined benefit liability (asset) is the change during the period in the net
defined benefit liability (asset) that arises from the passage of time.
Remeasurements of the net defined benefit liability (asset) comprise:
(a) actuarial gains and losses;

68
(b) the return on plan assets, excluding amounts included in net interest on the net defined
benefit liability (asset); and A qualifying insurance policy is not necessarily an insurance
contract, as defined in IFRS 4 Insurance Contracts.
(c) any change in the effect of the asset ceiling, excluding amounts included in net interest on
the net defined benefit liability (asset).
Actuarial gains and losses are changes in the present value of the defined benefit obligation
resulting from:
(a) experience adjustments (the effects of differences between the previous actuarial
assumptions and what has actually occurred); and
(b) the effects of changes in actuarial assumptions.
The return on plan assets is interest, dividends and other income derived from the plan assets,
together with realised and unrealised gains or losses on the plan assets, less:
(a) any costs of managing plan assets; and
(b) any tax payable by the plan itself, other than tax included in the actuarial assumptions used
to measure the present value of the defined benefit obligation.
A settlement is a transaction that eliminates all further legal or constructive obligations for part
or all of the benefits provided under a defined benefit plan, other than a payment of benefits to,
or on behalf of, employees that is set out in the terms of the plan and included in the actuarial
assumptions.
Short Term Employee Benefits
Short-term employee benefits include items such as:
(a) Wages, salaries and social security contributions;
(b) Short-term compensated absences (such as paid annual leave and paid sick leave)
(c) Profit-sharing and bonuses payable
(d) Non-monetary benefits (such as medical care, housing, cars and free or subsidized goods or
services) for current employees.
Recognition and Measurement: All Short Term Employee Benefits
The undiscounted amount of short-term employee benefits expected to be paid in exchange for
the service during the period is recognized as:
(a) as a liability (accrued expense), after deducting any amount already paid (As an asset if
prepaid expense)
(b) as an expense, unless another IFRS requires or permits the inclusion of the benefits in the
cost of an asset.
Short Term Compensated Absences
Short term compensated absences fall into two categories: Accumulating absences and Non
Accumulating absences.
Profit-sharing and bonus plans
An entity shall recognise the expected cost of profit-sharing and bonus payments when, and
only when:
(a) The entity has a present legal or constructive obligation to make such payments as a result of
past events; and
(b) A reliable estimate of the obligation can be made.
POST-EMPLOYMENT BENEFITS
Post-employment benefits include, for example:
(a) retirement benefits, such as pensions; and
(b) other post-employment benefits, such as post-employment life insurance and post-
employment medical care.

69
Defined Contribution plans
Characteristics of a defined contribution plan are:

plan, which in turn depends upon the performance of the pension fund investments.
Risk Associated with Defined Contribution Schemes
Contributions are usually paid into the plan by both the employer and the employee. The
expectation is that the investments made will grow through capital appreciation and the
reinvestment of returns and that on a member‗s retirement, the plan should have grown to be
sufficient to provide the anticipated benefits. If the investments have not performed as
anticipated, the size of the plan will be smaller than initially anticipated and therefore there will
be insufficient assets to meet the expected benefits. The insufficiency of assets is described as the
investment risk and is carried by the employee.
The other main risk with retirement plans is that a given amount of annual benefit will cost
more than expected if, for example, life expectancy has increased markedly by the time benefits
come to be drawn; this is described as the actuarial and, in the case of defined contribution
plans, this is also carried by the employee.
Definitions
Investment risk: This is defined as the risk that there will be insufficient funds in the plan to
meet the expected benefits.
Actuarial risk: This is the risk that the actuarial assumptions such as those on employee
turnover, life expectancy or future salaries vary significantly from that actually happens.
Recognition and Measurement
IAS 19 requires the following.
(a) Contributions to a defined contribution plan should be recognised as an expense in the
period they are payable (except to the extent that labour costs may be included within the cost
of assets). (b) Any liability for unpaid contributions that are due as at the end of the period
should be recognized as a liability (accrued expense).
(c) Any excess contributions paid should be recognised as an asset (prepaid expense), but only
to the extent that the prepayment will lead to, e.g. a reduction in future payments or a cash
refund.
In the (unusual) situation where contributions to a defined contribution plan do not fall due
entirely within 12 months after the end of the period in which the employees performed the
related service, then these should be discounted.
Defined Benefit Plans
These are defined by IAS 19 as all plans other than defined contribution plans.
Characteristics of defined benefit plan are:

service and salary levels (i.e. it is guaranteed)

performing in relation to the expected future obligation (i.e. if there is a shortfall, contribution
will increase and vice versa)
Contribution Levels
The actuary advises the company on contributions necessary to produce the defined benefits
(‗the funding plan‗). Contributions may be varied as a result.
Risk associated with defined benefit schemes

70
As the employer is obliged to make up any shortfall in the plan, it is effectively underwriting
the investment and actuarial risk associated with the plan. Thus in a defined benefit plan, the
employer carries both the investment and the actuarial risk.
Types of Defined Benefit Plans
There are two types of defined benefit plan:
Funded plans: These plans are set up as separate legal entities and are managed independently,
often by trustees. Contributions paid by the employer and employee are paid into the separate
legal entity. The assets held within the separate legal entities are effectively ring-fenced for the
payments of benefits.
Unfunded plans: These plans are held within employer legal entities and are managed by the
employers‗ management teams. Assets may be allocated towards the satisfaction of retirement
benefit obligations, although these assets are not ring-fenced for the payment of benefits and
remain the assets of the employer entity.
Recognition and Measurement
Accounting by an entity for defined benefit plans involves the following steps:
(a) Determining the deficit or surplus. This involves:
(i) Using an actuarial technique, the projected unit credit method is used to make a reliable
estimate of the ultimate cost to the entity of the benefit using estimates (actuarial assumptions)
about demographic variables (such as employee turnover and mortality) and financial variables
(such as future increases in salaries and medical costs).
(ii) Calculating the present value of the defined benefit obligation and the current service cost
(iii) Deducting the fair value of any plan assets from the present value of the defined benefit
obligation.
(b) Determining the amount of the net defined benefit liability (asset) as the amount of the
deficit or surplus determined in (a), adjusted for any effect of limiting a net defined benefit asset
to the asset ceiling.
(c) Determining amounts to be recognised in profit or loss:
(i) Current service cost.
(ii) Any past service cost and gain or loss on settlement.
(iii) Net interest on the net defined benefit liability (asset).
(d) Determining the remeasurements of the net defined benefit liability (asset), to be recognised
in other comprehensive income, comprising:
(i) Actuarial gains and losses;
(ii) Return on plan assets, excluding amounts included in net interest on the net defined benefit
liability (asset); and
(iii) Any change in the effect of the asset ceiling , excluding amounts included in net interest on
the net defined benefit liability (asset).
Accounting For the Constructive Obligation
An entity shall account for any constructive obligation also that arises from the entity‗s informal
practices.
The Statement of Financial Position
In the statement of financial position, the amount recognised as a defined benefit liability
(which may be a negative amount, i.e an asset) should be the following.
(a) The present value of the defined obligation at the year end, minus
(b) The fair value of the assets of the plan as at the yearend (if there are any) out of which the
future obligations to current and past employees will be directly settled.

71
Recognition and Measurement: Present Value of Defined Benefit Obligation and Current
Service Cost
The ultimate cost of a defined benefit plan may be influenced by many variables, such as final
salaries, employee turnover and mortality, employee contributions and medical cost trends. The
ultimate cost of the plan is uncertain and this uncertainty is likely to persist over a long period
of time. In order to measure the present value of the post-employment benefit obligations and
the related current service cost, it is necessary:
(a) To apply an actuarial valuation method;
(b) To attribute benefit to periods of service; and
(c) To make actuarial assumptions.

Attributing benefits to periods of service

19 requires that the benefits are attributed to periods of service on a straight line basis over the
period in which the benefit accrues.

Where this is the case, IAS 19 again requires benefits to be attributed to periods of service on a
straight line basis over the period in which the benefit accrues.

the amount of benefit attributable to each period of service is a constant proportion of the
estimated final salary.
The Statement of Profit or Loss and Other Comprehensive Income
All of the gains and losses that affect the plan obligation and plan asset must be recognised. The
components of defined benefit cost must be recognised as follows in the statement of profit or
loss and other comprehensive income:
Component Recognised in
(a) Service cost Profit or loss
(b) Net interest on the net defined benefit liability Profit or loss
(c) Remeasurements of the net defined benefit liability Other comprehensive income (not
reclassified to P/L
Unwinding of Interest
IAS 19 requires that the interest should be calculated on the net defined benefit liability (asset).
This means that the amount recognised in profit or loss is the net of the interest charge on the
obligation and the interest income recognised on the assets.
The calculation is as follows:
Net defined benefit liability/(asset) x Discount Rate
The net defined benefit liability/(asset) should be determined as at the start of the accounting
period, taking account of changes during the period as a result of contributions paid into the
scheme and benefits paid out.

Discount Rate
The discount rate adopted should be determined by reference to market yields on high quality
fixed- rate corporate bonds.
Dr. Interest cost Cr. PV of defined benefit obligation
Service Costs
These comprise:

72
(a) Current service cost, this is the increase in the present value of the defined benefit obligation
resulting from employee services during the period.
(b) Past service cost, is the change in the obligation relating to service in prior periods. This
results from amendments or curtailments to the pension plan, and
(c) Any gain or loss on settlement.
Current Service Cost
The amount of pension paid out by defined benefit schemes is often calculated based on the
number of years of service of an employee. Therefore, with each year that an employee remains
in employment, the pension liability will increase.
The current service cost is accounted for by
Dr. Current service cost expense Cr. PV of Defined Benefit Obligation
Past Service Costs
Past service cost is the change in the present value of the defined benefit obligation resulting
from a plan amendment or curtailment.
An entity shall recognise past service cost as an expense at the earlier of the following dates:
(a) When the plan amendment or curtailment occurs; and
(b) When the entity recognises related restructuring costs (see IAS 37) or termination benefits.
A plan amendment occurs when an entity introduces, or withdraws, a defined benefit plan or
changes the benefits payable under an existing defined benefit plan.
A curtailment occurs when an entity significantly reduces the number of employees covered by
a plan. A curtailment may arise from an isolated event, such as the closing of a plant,
discontinuance of an operation or termination or suspension of a plan.
Where an entity reduces benefits payable under an existing defined benefit plan and, at the
same time, increases other benefits payable under the plan for the same employees, the entity
treats the change as a single net change.
Gains and Losses on Settlement
The gain or loss on a settlement is the difference between:
(a) The present value of the defined benefit obligation being settled, as determined on the date
of settlement; and
(b) The settlement price, including any plan assets transferred and any payments made directly
by the entity in connection with the settlement.
An entity shall recognise a gain or loss on the settlement of a defined benefit plan when the
settlement occurs.
Recognition and Measurement: Plan Assets Fair Value of Plan Assets
Plan assets are:
(a) Assets such as stocks and shares, held by a fund that is legally separate from the reporting
entity, which exists solely to pay employee benefits.
(b) Insurance policies, issued by an insurer that is not a related party, the proceeds of which can
only be used to pay employee benefits.
Points to Remember
1. The fair value of any plan assets is deducted from the present value of the defined benefit
obligation in determining the deficit or surplus.
2. When no market price is available, the fair value of plan assets is estimated, for example, by
discounting expected future cash flows using a discount rate that reflects both the risk
associated with the plan assets and the maturity or expected disposal date of those assets (or, if
they have no maturity, the expected period until the settlement of the related obligation).

73
3. Plan assets exclude unpaid contributions due from the reporting entity to the fund, as well as
any non-transferable financial instruments issued by the entity and held by the fund.
4. Plan assets are reduced by any liabilities of the fund that do not relate to employee benefits,
for example, trade and other payables and liabilities resulting from derivative financial
instruments.

Retirement Benefits Paid Out


During an accounting year, some of the plan assets will be paid out to retirees, thus discharging
part of the benefit obligation. This is accounted for by:
Dr PV of defined benefit obligation X CR FV of plan assets X
There is no cash entry as the pension plan itself rather than the sponsoring employer pays
money out.
Contributions Paid Into Plan
Contributions will be made into the plan as advised by the actuary. This is accounted for by:
DR Fv of plan assets X CR Cash X
Remeasurements of the net defined benefit liability (asset)
Remeasurements of the net defined benefit liability (asset) comprise:
(a) Actuarial gains and losses;
(b) The return on plan assets, excluding amounts included in net interest on the net defined
benefit liability (asset); and
(c) Any change in the effect of the asset ceiling, excluding amounts included in net interest on
the net defined benefit liability (asset).
Return on Plan Assets
In determining the return on plan assets, an entity deducts the costs of managing the plan assets
(except other administration costs) and any tax payable by the plan itself
Exam Focus
The opening and closing obligation and plan assets can be reconciled as follows:
PV of defined benefit obligation FV of plan assets
GHC GHC
B/f at start of year (advised by actuary) (X) X
Retirement benefits paid out X (X)
Contributions paid into plan X
Expected return on plan assets X
Unwinding of interest (X)
Current service cost (X)
(X) X
Remeasurement gains / losses
(Balancing figure) X(X) X(X)
C/f at end of year (advised by actuary) (X) X

Asset Ceiling Test


When we looked at the recognition of the net defined benefit liability/(asset) in the statement of
financial position stated above, the term ‗asset ceiling‗ was mentioned.
This term relates to a threshold established by IAS 19 to ensure that any defined benefit asset
(i.e. a pension surplus) is carried at no more than its recoverable amount. In simple terms, this

74
means that any net asset is restricted to the amount of cash savings that will be available to the
entity in future.
Net Defined Benefit Assets
A net defined benefit asset may arise if the plan has been overfunded or if actuarial gains have
arisen. This meets the definition of an asset ( as stated in the Framework) because all of the
following apply.
(a) The entity controls a resource (the ability to use the surplus to generate future benefits).
(b) That control is the result of past events (contributions paid by the entity and service
rendered by the employee).
(c) Future benefits are available to the entity in the form of a reduction in future contributions or
a cash refund, either directly or indirectly to another plan in deficit.
The asset ceiling is the present value of those future benefits. The discount rate used is the same
as that used to calculate the net interest on the net defined benefit liability/(asset). The net
defined benefit asset would be reduced to the asset ceiling threshold.
Any related write down would be treated as a remeasurement and recognised in other
comprehensive income.
If the asset ceiling adjustment was needed in a subsequent year, the changes in its value would
be treated as follows:
(a) Interest (as it is a discounted amount) recognised in profit or loss as prat of the net interst
amount
(b) Other changes recognised in profit or loss.
Other Long Term Benefits
Other long-term employee benefits include, for example:
(a) long-term compensated absences such as long-service or sabbatical leave;
(b) jubilee or other long-service benefits;
(c) long-term disability benefits;
(d) profit-sharing and bonuses payable twelve months or more after the end of the period in
which the employees render the related service; and
(e) deferred compensation paid twelve months or more after the end of the period in which it is
earned.
Recognition and Measurement: Other Long Term Benefits
There are many similarities between these types of benefits and defined benefit pensions. For
example, in a long-term bonus scheme, the employees may provide service over a number of
periods to earn their entitlement to a payment at a later date. In some case, the entity may put
cash aside, or invest it in some way (perhaps by taking out an insurance policy) to meet the
liabilities when they arise.
As there is normally far less uncertainty relating to the measurement of these benefits, IAS 19
requires a simpler method of accounting for them. Unlike the accounting method for post-
employment benefits, this method does not recognise remeasurements in other comprehensive
income.
The entity should recognise all of the following in profit or loss.
(a) Service cost
(b) Net interest on the defined benefit liability (asset)
(c) Remeasurement of the defined benefit liability (asset)

REVIEW QUESTIONS
QUESTION 1

75
The following information is given about a funded defined benefit plan. To keep the
computations simple, all transactions are assumed to occur at the year-end. The present value
of the obligation and the market value of the plan assets were both GHC1,000 at 1 January 2011.
Actuarial gains and losses are to be recognised as they occur outside the profit and loss in other
comprehensive income.
2011 2012 2013
Discount rate at start of year 10% 9% 8%
Expected rate of return on plan assets at start of year 12% 11% 10%
Current service cost(GHC 130 140 150
Benefits paid(GHC) 150 180 190
Contributions paid (GHC) 90 100 110
Present value of obligations at 31 December (GHC) 1,100 1,380 1,408
Market value of plan assets at 31 December (GHC) 1,190 1,372 1,188
Required:
Prepare the Financial statement effects for all three years.
Solution:
On assets 2011 2012 2013
Market value at start of the year 1,000 1,190 1,372
Expected return on the assets 120 131 137
Contributions 90 100 110
Benefits paid (150) (180) (190)
Actuarial gain (loss) –bal fig 130 131 (241)
Market value at end of the year 1,190 1,372 1,188

On obligations 2011 2012 2013


Obligation at start of the year 1,000 1,100 1,380
Interest 100 99 110
Current service cost 130 140 150
Benefits paid (150) (180) (190)
Actuarial (gain) loss –bal. fig 20 22 (42)
Obligation at end of the year 1,100 1,380 1,408

The statement of financial position


Pension assets 1,190 1,372 1,188
Pension obligations (1,100) (1,380) (1,408)
Pension asset (liability) 90 (8) (220)
The income statement
Operating
Current service cost 130 140 150
Finance Interest cost 100 99 110
Expected return on assets (120) (131) (137)
Other comprehensive Income
Actuarial gain (loss) on assets 130 131 (241)
Actuarial gain (loss) on obligations (20) (221) 42

QUESTION 2

76
(a) Explain the following as used in IAS 19 Employee Benefits:
(i) The term ‗defined benefit pension plan‘
(ii) The basis to be adopted in measuring scheme assets
(iii) The basis to be adopted in measuring scheme liabilities
(iv) Actuarial gains and losses.

(b) Universal Solutions operates a defined benefit pension scheme on behalf of its employees.
The company conducts an annual review of funding in conjunction with their actuaries who
have supplied the following information:
At 31 Dec 2013 At 31 Dec 2014
GHC GHC
Present value of pension fund obligations 1,000 1,200
Market value of pension fund assets 1,000 1,150
Information relevant to the actuarial valuation:
Expected return on plan assets 8%
Discount rate used to determine pension fund liabilities 12%
Current service cost GHC100
Contributions to the pension fund GHC140
Benefits paid out amounted to GHC95
Required
(i) Calculate the charge to profit or loss for the year to 31 December 2014.
(ii) Compute the amount of the pension surplus or deficit in the statement of financial position
as at 31 December 2014
(iii) Compute any actuarial gain or loss as at 31 December 2014 and explain the available
accounting treatments of such a gain or loss.
(iv) Describe proposals by the IASB to change the permitted methods of accounting for actuarial
gains or losses.

Solution:
a) (i) Defined benefit pension scheme
The employees of a defined benefit scheme will be guaranteed a pension based on their final
salary and their number of years of service. Accordingly, the higher paid the employee is on
retirement, and the longer the length of service: „
 the greater the employee‘s pension entitlement and „
 the greater the liability of the pension fund.
An actuary will advise the company of the cash contributions to be paid into the plan each year
in order to provide the promised pensions. This is a complicated calculation involving many
estimates such as employee mortality, future increases in salary and expected future investment
returns.
The employer has an open-ended liability to make additional contributions should there be a
deficit in the defined benefit pension fund. A deficit may arise, for example, if salary levels rise
more than expected or staff turnover reduces, increasing service years. It will be necessary for
the actuary to regularly re-value the pension fund‘s assets and liabilities to assess the surplus or
deficit position and revise the company‘s contributions.

(ii) The basis to be adopted in measuring scheme assets

77
Assets should be measured at their fair value. For quoted securities, for example, this means
their mid-market price.
(iii) The basis to be adopted in measuring scheme liabilities
Liabilities should be measured on an actuarial basis (i.e. discounted cash flow), using the
projected unit credit method. The projected unit method is an accrued benefits valuation
method in which the scheme‘s liabilities reflect projected future earnings. To derive the scheme
liabilities, the expected future pension payments should be discounted at a rate that reflects the
time value of money, for example, using an AA (high quality) corporate bond rate.
(iv) Actuarial gains and losses
Actuarial gains and losses are deficits or surpluses that arise because: „
 events have not coincided with the actuarial assumptions made at the last valuation
(experience gains and losses) or „
 the actuarial assumptions have changed.
For example, if the actuary forecast that investment returns were going to be 7% in a year, but in
fact the return actually achieved was only 5%, this would give rise to an actuarial deficit.
(b) Workings
Pension fund Company position
Assets Liabilities Expense
Statement of
financial position
GHC GHC GHC GHC
Openingbalance1January2014 1,000 1,000 0
Interest cost (12%×1,000) 120 120
Current service cost 100 100
Expected return (8%×1,000) 80 (80)
Net expense to company (140)
Contributions to the pension fund 140 140
Benefits paid out (95) (95)
Amounts recorded by company 1,125 1,125 0
Actuarial difference (balance) 25 75 (50)
Closing balance 31Dec2014 1,150 1,200 (50)
Answers:
(i) The charge to profit or loss for the year to 31December 2014 is GHC140.
(ii) There is no net pension surplus or deficit recorded by the company as at 31 December 2014.
(iii) There is an actuarial loss at 31 December 201 4 of GHC50. Under IAS 19 there are three
possible accounting treatments of this:
1. Immediate recognition in profit or loss
2. Immediate recognition in the statement of changes in equity
3. Deferred recognition under the corridor approach. This would involve recognition, starting in
the following period of any amount of the actuarial loss falling outside the corridor (the greater
of 10% of opening assets liabilities). The recognition may be spread over the estimated service
lives of the employees.
QUESTION 3
The following information relates to the pension scheme of Patience Pass All Limited for the
year ended 30 April 2016:
GH¢000
Plan assets at 1 May 2015 96,000

78
Defined benefit obligation at 1 May 100,000
2015
Service cost for year ended 30 April 8,000
2016
Discount rate at 1 May 2015 10%
Re-measurement loss in year ended
30 April 2016 4,000
Past service cost 1 May 2015 6,000
The pension costs have not been accounted for in the total comprehensive income as the
Accountant of the company is not qualified yet and lacks sufficient knowledge on the
provisions of IAS 19 employee Benefits.
Required:
Demonstrate how the above transaction would be accounted for under the provisions of IAS 19
Employee Benefits including relevant extracts to the financial statements for the year ended 30
April 2016.
(5 marks)
Solution:
Pension cost recognised for the year would be:
GHȻ000
Current service cost 8,000
Net interest cost (10% of GHȻ100m –GHȻ96m) 400
Past service cost 6,000
Net service cost recognised in profit or loss 14,400
Remeasurements in OCI 4,000
Net cost for year recognised in total comprehensive income 18,400
IAS 19 does not actually specify where service cost and net interest cost should be presented. As
a result, it is acceptable to include the net interest cost in finance costs.
IAS 19 states that past service cost should be recognised immediately and the past service cost
will be included in the defined benefit obligation at 1 May 2015. Thus there is no need to
calculate an interest cost on the past service cost.
Financial statement extracts
Statement of profit or loss
GHȻ000
Net service cost recognisedin profit or loss (14,400)
OCI
Remeasurements in OCI (4,000)
QUESTION 4
The following information is given about a funded defined benefit plan. To keep the
computations simple, all transactions are assumed to occur at the year-end.
Present value of obligations at year start GHC400m
Market value of plan assets at year start GHC390m
Discount rate at start of year 10%
Expected rate of return on plan assets at start of year 12%
Current service cost GHC14m
Benefits paid GHC26m
Contributions paid GHC34m
Present value of obligations at year end GHC530m

79
Market value of plan assets at year end GHC370m
There was a variation in the benefit terms during the year, which resulted in a past service cost
of GHC100m.
Required
(a) Show the Financial statement effects for the year.
(b) Explain the policies available for the recognition of the actuarial loss.
(c) Calculate the net unrecognised actuarial loss carried forward assuming that the Company
has applied the ―corridor method and has an opening unrecognised actuarial loss of GHC100m
and a remaining average employee working life of 10 years.
Solution:
On assets
Market value at start of the year 390
Expected return on the assets 47
Contributions 34
Benefits paid (26)
Actuarial gain (loss) –bal fig (75)
Market value at end of the year 370

On obligations
Obligation at start of the year 400
Interest 40
Service cost (14 + 100) 114
Benefits paid (26)
Actuarial (gain) loss –bal. fig 2
Obligation at end of the year 530

The statement of financial position


Pension assets 370
Pension obligations (530)
Net pension asset (liability) (160)

The income statement


OperatingService cost (114)
FinanceInterest cost (40)
Investment income 47
Other Comprehensive Income
Actuarial (loss) on assets (77)
Actuarial (loss) on obligations (2)
Net Actuarial (loss) (77)

(b)Policies
There are essentially three policies available as regards the recognition of the above net actuarial
loss:-
(1)Recognised in the OCI (Other Comprehensive Income, sometimes known as the SORIE,
Statement of Recognised Income and Expense)
(2)Recognised in the P&L (Income Statement)
(3)Unrecognisedin the B/S (Position Statement)

80
The last policy is widely criticised for its bizarre lack of logic as it recognises the loss on the
balance sheet as if it were an asset. The IASB are in the process of removing this option from the
IAS via a development project. However, in the meantime it is permissible to hold the entire
current loss in the balance sheet as an asset on top of the actual pension asset. This is subject to a
tiny release of the opening loss to the p&l. This releaseis even more bizarre than the
capitalisation of a loss. The opening unrecognised actuarial loss is compared to the opening
―corridor to generate an ―excess which is then divided by the employee working lives and that
resultant fractional loss is released to the p&l. The ―corridor is defined as 10% of the higher of
the pension assetor pension liability. Do not look for any sense here, because there is none.
(c)Closing Unrecognised Loss
Opening unrecognised loss 100
Release to P&L (6)
Current unrecognised loss 77
Closing unrecognised loss 171
Opening unrecognised loss 100
Opening corridor 10%(400 the higher) (40)
Excess 60
Life 10 years
Release to P&L 6

QUESTION 5
Zumah Ltd operates a defined benefit pension plan for its employees. At 1 April 2015 the fair
value of the pension plan assets was GH¢8,200,000 and the present value of the pension plan
liabilities was GH¢8,500,000. The Actuary estimated that,the service cost for the year to 31
March 2016 was GH¢2,100,000. The pension plan paid GH¢500,000 to retired members and
Zumah Ltd paid GH¢1,900,000 in contributions to the pension plan in the year to 31 March
2016. The Actuary estimated that,the relevant discount rate for the year to 31 March 2016 was
6%.

On 31 March 2016, Zumah Ltd announced improvements to the benefits offered by the pension
plan to all of its members. The Actuary estimated that,the past service cost associated with these
improvements was GH¢2 million. At 31 March 2016 the fair value of the pension plan assets
was GH¢10,200,000 and the present value of the pension plan liabilities (including the past
service costs) was GH¢12,500,000.

Required:
In accordance with IAS 19 Employee Benefits:
i) Calculate the net actuarial gain or loss that will be included in Zumah Ltd‘s other
comprehensive income for the year ended 31 March 2016. (3 marks)
ii) Calculate the net pension asset or liability that will be included in Zumah Ltd‘s statement of
financial position as at 31 March 2016. (2marks)

Solution:
a)Pension plan of Zumah Ltd
i) Actuarial gains and losses:
FV of plan assets PV of plan liabilities
GH¢000 GH¢000

81
Opening balance 8,200 8,500
Service cost 2,100
Interest cost (6% x opening balances) 492 510
Benefits paid (500) (500)
Contributions 1,900
Past service cost 2,000
10,092 12,610
Actuarial gain on plan assets 108
Actuarial gain on plan liabilities 110
Closing balance 10,200 12,500
The net actuarial gain in OCI is GH¢218,000 for the year.

ii) Statement of financial position:


GH¢000
Present value of pension plan liabilities at 31 March 2016 (12,500)
Fair value of pension plan assets at 31 March 2016 10,200
Net pension liability (2,300)

Question 5
The following information relates to the group pension plan of Accra Ltd:
1 April 2017 31 March 2018
GH¢‟m GH¢‟m
Fair value of plan assets 14 14.5
Actuarial value of defined benefit obligation 15 17.5

The contributions for the period received by the fund were GH¢1million and the employee
benefits paid in the year amounted to GH¢1.5million. The discount rate to be used in any
calculation is 5%. The current service cost for the period based on actuarial calculations is
GH¢0.5million.
Required:
Demonstrate how the above transaction would be accounted for under the provisions of IAS 19
Employee Benefits including relevant extracts to the financial statements for the year ended 31
March 2018.

Question 6
Nzema prepares its financial statements in accordance with International Financial Reporting
Standards (IFRS) with financial year end 31 December 2018. On 1 January 2018, Nzema
commenced a defined benefit pension plan for a number of head office employees. Under the
pension scheme, Nzema has an obligation to provide these staff with agreed post-employment
benefits. Nzema carries the actuarial and investment risk associated with the pension scheme.

The following information has been compiled from workings by Nzema‟s accounting staff and
actuarial reports for the 2018 financial year:
GH¢
Interest Income on plan assets 16,500
Employer contributions to plan 550,000

82
Current service cost 600,000
Interest on plan liability 18,000
Fair value of plan assets at 31/12/2018 580,000
Present value of plan obligation at 31/12/2018 620,000

The Accountant was not sure which accounting standard to apply when accounting for the
pension scheme. The only adjustment made to account for the scheme was to expense the
company‟s contributions of GH¢550,000 for the 2018 financial year in the Statement of Profit or
Loss and Other Comprehensive Income and to credit the „Cash‟ account.
Required:
Recommend, with appropriate calculations the necessary accounting treatment for this
accounting issue. (6 marks)

Solution
The applicable accounting standard is IAS 19 (Employee Benefits). Nzema‟s pension plan is a
defined benefit plan as Nzema has an obligation to provide agreed post-employment benefits and
the entity carries the actuarial and investment risk associated with the pension scheme.
The accountant has accounted for the employer contributions to the scheme incorrectly by
simply recognising the employer contributions in the profit or loss. IAS 19 requires a defined
benefit liability (asset) to be recognised on the balance sheet at the new amount of the present
value of the obligation less the fair value of plan assets at reporting date.
A defined benefit plan may give rise to current service cost and net interest expense, which
should be recognised through the profit or loss. In addition, any actuarial gains and/or losses
from re-measurement of the defined pension liability (asset) should be recognised through other
comprehensive income.

Workings – Calculation of Actuarial Gain/Loss: GH¢


Pension Asset
Opening balance 0
Return on assets 16,500
Employer contributions 550,000
Remeasurement –Actuarial Gain 13,500
Closing Balance 580,000

Pension Liability
Opening balance 0
Interest Cost 18,000
Current Service Cost 600,000
Remeasurement – Actuarial Loss 2,000
Closing Balance 620,000

Net Actuarial Gain: GH¢13,500 - GH¢2,000 = GH¢11,500


Journals: DR Net Interest Expense (P/L) 1,500
CR Pension Liability 1,500
Net interest expense (18,000 – 16,500)
DR Current Service Cost (P+L) 600,000

83
CR Pension Liability 600,000
Current service cost

DR Net Pension Liability 11,500


CR Remeasurement – Actuarial Gain (OCI) 11,500

Recognition of net actuarial loss in OCI


Finally, the accountant‟s previous accounting treatment for the employer‟s contributions
should be corrected: DR Net Pension Liability (SOFP) 550,000
CR Pension Contribution Expense (P&L) 550,000
Contributions paid into defined benefit scheme – correcting previous accounting treatment of
expensing contributions through P&L.

84
ACCOUNTING FOR GOVERNMENT GRANTS
AND DISCLOSURE OF GOVERNMENT
ASSISTANCE (IAS 20)
Introduction and definitions
In many countries the government provides financial assistance to industry. The most common
form of such assistance is a grant of cash from local or national government. Such grants are
defined by IAS 20 as:
 grants related to assets, or
 grants related to income.

Grants related to assets are for the purchase or construction of long-term assets. Conditions
may be attached to a grant, specifying the type of assets that should be purchased with the
grant, or the location of the assets, or the period in which they are to be acquired or held.

Grants related to income are any other government grants.

ACCOUNTING TREATMENT OF GOVERNMENT GRANTS


IAS 20 states that grants should not be recognised until there is reasonable assurance that:
 the entity will comply with any conditions attaching to the grant, and
 the grant will be received.
Once these recognition criteria are met, the grants should be recognised in profit or loss over
the periods necessary to match them with their related costs.

Neither type of grant should be credited directly to shareholders‘ interests in the statement of
financial position. They must be reported on a systematic basis through the income statement
(profit or loss).

Grants related to income


For grants related to income, IAS 20 states that an ‗income approach‘ should be used, and the
grant should be taken to income over the periods necessary to match the grant with the costs
that the grant is intended to compensate.

IAS 20 allows two methods of doing this:


Method 1: Include the grant for the period as ‗other income‘ for inclusion on profit or loss for
the period
Method 2: Deduct the grant for the period from the related expense.

For example, suppose that a company receives a cash grant of GHC30,000 on 1 January,2011.
The grant is towards the cost of training young apprentices, and the training programme is
expected to last for 18 months from 1 January,2011. Actual costs of the training were GHC50,000
in 2011 and GHC25,000 in 2012.

The grant should be recognised partly in profit for 2011 (GHC20,000, which is 12 minths/18
months × GHC30,000) and partly in 2012 (the remaining GHC10,000). The grant should be
taken to income either by:

85
 recording the grant as other income (GHC20,000 and GHC10,000 in 2011 and 2012
respectively), or
 deducting GHC20,000 from the training costs in 2011 and GHC10,000 from the training
costs in 2012.

At the end of 2011, there is a liability of GHC10,000 in the statement of financial position for the
grant received but not yet recognised in income.

Example:
Stacy Ltd has received a grant of GHC30,000 in relation to training costs of GHC100,000.
Required
Show how the costs and the grant could be presented in the financial statements in accordance
with IAS 20.

Answer
Statement of comprehensive income (extracts)
Method 1
GHC
Training costs (100,000)
Government grant received 30,000

Method 2
Training costs (100,000 – 30,000) (70,000)

Grants related to assets


For grants related to assets, IAS 20 allows two methods of doing this:
Method 1: Deduct the grant from the cost of the related asset. The asset is included in the
statement of financial position at cost minus the grant. Depreciate the net amount over the
useful life of the asset.
Method 2: Treat the grant as deferred income and recognise it as income on a systematic basis
over the useful life of the asset.

For example, suppose that a company receives a grant of GHC400,000 from the government
towards the cost of an asset that cost GHC1,000,000 and which has an estimated useful life of 10
years and no residual value. „
 One way of accounting for the grant would be to record the asset at cost net of grant
(GHC600,000) and depreciate this amount over ten years. „
 The other method is to record the asset at cost (GHC1,000,000) and depreciate over ten
years, and also to account for the grant as deferred income (a liability in the statement of
financial position) and ‗release‘ the grant as income over the next ten years, possibly at
the rate of GHC60,000 per year. These two methods achieve the same effective result.

Example:
In January 2011 Oscar Ltd purchased a non-current asset with a cost of GHC500,000 and
received a grant of GHC100,000 in relation to that asset. The asset is being depreciated on a
straight-line basis over five years.

86
Required
Show how these amounts could be reflected in the financial statements prepared at the end of
2011 in accordance with IAS 20.

Answer
Method 1:
Statement of financial position
Property, plant and equipment
GHC
Cost (500,000 – 100,000) 400,000
Accumulated depreciation (80,000)
––––––––
Carrying amount 320,000
––––––––
Included in statement of comprehensive income
GHC
Depreciation charge (GHC400,000/5 years) 80,000

Method 2:
Statement of financial position
Property, plant and equipment GHC
Cost 500,000
Accumulated depreciation (100,000)
––––––––
Carrying amount 400,000
––––––––
Deferred income (= liability)
Government grant (100,000 – 20,000) 80,000

Included in statement of comprehensive income


GHC
Depreciation charge (GHC500,000/5 years) (100,000) = expense
Release of deferred government grant (GHC100,000/5 years) 20,000 = income

DISCLOSURE REQUIREMENTS
IAS 20 requires the following disclosures in the notes to the financial statements.

 The accounting policy adopted for government grants, including the method of
presentation in the financial statements
 The nature and extent of government grants recognised in the financial statements and
an indication of other forms of government assistance from which the entity has directly
benefited
 Unfulfilled conditions and other contingencies attaching to government assistance (if
this assistance has been recognised in the financial statements).

87
REVIEW QUESTIONS
QUESTION 1
A company received two separate grants during the year from the United Kingdom
government. The first grant, an amount of GHC500,000 was given to acquire an Electricity
Plant, whilst the second grant of GHC50,000 was to support the training of electrical engineers
on the Electrical Plant.
The Plant was estimated to have an economic life of 25 years.
Required:
Detail out the treatment of the above events in accordance with IAS 20 – Accounting for
Government Grant and disclosure of Government assistance.
(4 marks)
ICAG November,2012
Solution:
GOVERNMENT GRANT – There are two types of Government Grants
The first is a Government Grant related to Assets, while the second is a grant Related to Income.
Grant Related to Assets: - are government grants whose primary conditions is that an entity
qualifying for them should purchase, construct, or otherwise, acquire long-term assets.
Grant related to Assets, shall be presented in the statement of financial position either
(i) by setting up the grant as deferred income and amortized over the useful life of the Assets or
(ii) by deducting the grant in arriving at the carrying amount of the asset.
Of the above treatment the first (i) is preferred. Therefore the Grant of GHS500,000 will be
amortized at the annual rate GHC20,000.

Grant related to Income are Government Grants other than those related to Assets.
Grant related to income are:
(i) sometimes presented as a credit in the statement of Comprehensive Income as other
separately or
(ii) they are deducted in exporting the related expense

Of the two methods of treating grant related to Income the first (i) method is preferred. Thus
the grant GHC50,000 will be credited to the statement of comprehensive income as a gain.

QUESTION 2
During the year ended 30 June 2012, Kante received three grants, the details of which are set out
below.
(1) On 1 September, a grant of GHC40,000 from local government. This grant was is respect of
training costs of GHC70,000 which Kante had incurred.
(2) On 1 November Kante bought a machine for GHC350,000. A grant of GHC100,000 was
received from central government in respect of this purchase. The machine,which has a residual
value of GHC50,000, is depreciated on a straight-line basis over its useful life of five years.
(3) On 1 June a grant of GHC100,000 from local government. This grant was in respect of
relocation costs that Kante had incurred moving part of its business from outside the local area.
The grant is repayable in full unless Kante recruits ten employees locally by the end of 2012.
Kante is finding it difficult to recruit as the local skill base does not match the needs of this part
of the business.

88
Required
Show how the above transactions should be reflected in the financial statements of Kante for the
year ended 30 June 2012. Where any accounting standards allow a choice you should show all
possible options.

Solution:
Option 1 – Net grants off related expenditure
Statement of financial position as at 30 June,2012 (extracts)
GHC
Non-current assets
Property, plant and equipment 223,333
Current liabilities
Other current liabilities 100,000
Notes to the financial statements for the year ended 30 June,2012 (extracts) Property, plant and
equipment
GHC
Cost (350,000 – 100,000) 250,000
Accumulated depreciation ((250,000 – 50,000) ÷ 5 × 8/12) (26,667)
––––––––
Carrying amount 223,333
––––––––
Included in income statement for the year ended 30 June,2012
GHC
Depreciation charge 26,667
Training costs (70,000 – 40,000) 30,000
Option 2 – Show grants separately from related expenditure
Statement of financial position as at 30 June,2012 (extracts)
GHC
Non-current assets
Property, plant and equipment 310,000
Current liabilities
Other current liabilities 186,667
Notes to the financial statements for the year ended 30 June,2012 (extracts)
GHC
Property, plant and equipment
Cost 350,000
Accumulated depreciation ((350,000 – 50,000) ÷ 5 × 8/12) (40,000)
––––––––
Carrying amount 310,000
––––––––
Other current liabilities
Deferred income relating to
government grants (100,000 - (100,000 ÷ 5 × 8/12)) 86,667
Government grant repayable 100,000
––––––––
186,667
––––––––

89
Included in income statement for the year ended 30 June,2012
GHC
Depreciation charge 40,000
Training costs 70,000
Government grant received (40,000)
Release of deferred government grant (13,333)

Tutorial note
The GHC100,000 grant in (3) has conditions attached to it. In such a situation, IAS20 states that
grants should not be recognised until there is reasonable assurance that the entity will comply
with any conditions attaching to the grant. Since Katie is struggling to recruit, and there is only
one month left for recruitment to meet these conditions, then it does not seem that there is
‗reasonable assurance‘. Hence the grant should not be recognised as such, but should be held in
current liabilities, pending repayment.
QUESTION 3
Wisdom Ltd received a Government grant of GHS150 million in January 2015 to install and
run a solar energy system in an economically backward area. Wisdom Ltd has estimated that
such a system would cost GHS250 million to construct. The secondary condition attached to the
grant is that the entity should hire labour in the local market (i.e., from the economically
backward area where the windmill is located) instead of employing workers from other parts of
the country. It should maintain a ratio of 1:1 local workers to workers from outside in its labor
force for the next 5 years. The windmill is to be depreciated using the straight-line method over
a period of 10 years. The management of Wisdom has adopted the policy of treating unearned
government grant on a deferral basis.
Required
Advise Wisdom Ltd on the treatment of this grant in the 2015 financial statements
(Statement of Profit and Loss for 2015 and Statement of financial position as at 31 December
2015) in accordance with IAS 20. [5 marks]

Solution:
Deferred Income Approach
Income Statement Extract
GHC000
Depreciaition(250,000 / 10 years) (25,000)
Income(grant) (150,000/10) 15,000
SOFP Extract
Solar Energy System 250,000
Less Accum Depn (25,000)
225,000
Non Current Liab
Deferred Grant 120,000
Current
Deffered Grant 15,000
135,000
QUESTION 4
Dodowa Ltd is a large textile manufacturing company. Wherever possible, it structures its
operations to take advantage of any financial assistance available from national and regional

90
authorities.
During the year, a heavy duty equipment was purchased for Dodowa Ltd's main
manufacturing operation for GH¢12 million on 1 April 2015. The equipment was expected to be
used for 10 years, with a zero residual value. Dodowa Ltd pre-applied for a government grant
on 1 January 2015, meeting all necessary criteria for awarding of the grant. On 1 February 2015,
the grant was awarded for 40% of the equipment's cost and the cash was received on 1 July
2015.
Conditions relating to maintaining employment are attached to the grant and if they are not
satisfied, then the grant becomes repayable, or partly repayable.
Dodowa Ltd expected to meet these conditions when the grant was applied for. However, due
to worsening economic conditions, redundancies for some staff on 31 December 2016 resulted
in a repayment of 10% of the original grant becoming due. The repayment was made on 1
February 2017. Dodowa Ltd accounted for the grant as a reduction in the carrying amount of
the asset.

Required:
Explain, with suitable calculations, the financial reporting treatment of the above in the
financial statements of Dodowa Ltd for the year ended 31 December 2016 in accordance with
IAS 20: Accounting for Government Grants and Disclosure of Government Assistance.
(6 marks)
Solution:
The grant was recognised in the financial statements on 1 February 2015 as deferred income on
that date. It is credited to the asset's value on 1 April 2015, the date of the asset's initial
recognition.
The carrying amount of the asset is calculated as follows:
GH¢
Asset value on initial recognition (1 April 2015) 12,000,000
Grant (GH¢12m x 40%) – credited to asset on 1 April 2015 (4,800,000)
7,200,000
Depreciation 2015 (GH¢7.2m/10 years x 9/12) (540,000)
31 December 2015 6,660,000
Depreciation 2016 (GH¢7.2m/10 years) (720,000)
5,940,000
Grant repayment (GH¢4.8m x 10%) 480,000
Additional depreciation (480,000 x 1¾/10 years) (84,000)
31 December 2016 6,336,000

The double entry is:


GH¢
DR Asset (480,000 –84,000) 396,000
DR Profit or loss 84,000
CR Grant repayable liability 480,000

QUESTION 5
Dambai Ltd (Dambai) is a large manufacturing company. Wherever possible, it structures its
operations to take advantage of any financial assistance available from national and regional
authorities.

91
During the year, Dambai decided to relocate some of its other operations to a regional development
area, which offers attractive labour costs and tax incentives. The regional government agreed to
contribute GH¢200,000 as a result of Dambai setting up in the regional development area. There are
no particular conditions as to what the money should be spent on. The cash was received on 1 August
2019.
Required:
In accordance with IAS 20: Accounting for Government Grants and Disclosure of Government
Assistance explain the financial reporting treatment of the above in the financial statements of
Dambai for the year ended 31 December 2019. (4 marks)
SOLUTION
The contribution of GH¢200,000 by the regional government is not dependent on any
particular activities of the entity.

specific expenses.
Recognition of the GH¢200,000 Government Grant - 2 marks
Treatment of the GH¢200,000 in Profit or Loss - 2 marks

QUESTION 6
On 1 January 2018, Asankragua Ltd (Asankragua) applied to a government agency for a
grant to assist with the construction of a factory in Enchi. The proposed construction cost of
the factory was GH¢52 million and the company projected that 350 people would be
employed after completion. The land was already owned by Asankragua.
On 1 March 2018, the government agency offered to grant a sum amounting to 25% of the
factory‘s construction cost to a maximum of GH¢13 million. The grant aid was to be
advanced on completion, and would be repayable on demand if total employment at the
factory fell below 300 people within 5 years of completion.
At the financial year end, 31 March 2018, Asankragua had accepted the offer of grant aid,
and had signed contracts for the construction of the factory at a total cost of GH¢52 million.
Construction work was due to commence on 1 April 2018.
By 31 March 2019, the factory had been completed on budget, 400 people were employed
ready to commence manufacturing activities, and the government agency agreed that the
conditions necessary for the drawdown of the grant had been met.
On 1 April 2019, the factory was brought into use. It was estimated that it would have a ten-
year useful economic life. On 1 June 2019, the government agency paid over the agreed
GH¢13 million. In addition, the company sought and was paid an employment grant of
GH¢1.2 million as employment exceeded original projections. This is expected to be payable
annually for 5 years in total, at a rate of GH¢12,000 per additional person employed over
300 in each year. There are no repayment provisions attached to the employment grant.
The directors of Asankragua expect employment levels to exceed 350 people for at least 4
further years from 31 March 2020.
Required:
Demonstrate, showing calculations and relevant entries, how Asankragua Ltd should
record the above transactions and events in its financial statements for years ended 31
March 2018, 2019 and 2020. (9 marks)

92
SOLUTION 6
Year ended 31 March 2018:
No accounting entry is made in this financial year, as no transaction has yet been entered into.
A capital commitment exists, and should be disclosed in the notes. The grant approval should
be disclosed also.
Year ended 31 March 2019:
At this date, the factory should be recorded at its cost of GH¢52 million. As all conditions for the
payment of the grant have been met, recognition should be made of this amount receivable also.
As the factory has not yet been brought into use, no depreciation will be charged for the year.
Similarly, no amortisation of the grant will take place in the period.
Recognition of factory:
Dr Property, plant & equipment GH¢52 million
Cr Cash GH¢52 million
(New factory constructed as a cost of GH¢52 million)
Recognition of grant:
Option 1
Dr Government grant receivable (current asset) GH¢13 million
Cr Property, plant & equipment GH¢13 million
(Government grant approved, not received yet) Page 18 of 28

Option 2
Dr Government grant receivable (current asset) GH¢13 million
Cr Deferred income – current liability GH¢1.3 million
Cr Deferred income – non-current liability GH¢11.7 million
(Government grant approved, not received yet)
Assuming the factory has a useful life of 10 years, as stated, 10% of the amount will be
recognised as income within the next financial year. This amount should be treated as a current
liability.
Year end 31 March 2020:
There are a number of transactions to record based on the new factory. These are (1)
depreciation and (2) amortisation of the grant. In addition, the cash was received from the
government agency.
Receipt of grant:
Dr Cash GH¢13 million
Cr Government grant receivable GH¢13 million
(Receipt of cash grant from government agency)
Option 1
Depreciation of factory:
Dr Profit or loss GH¢3.9 million
Cr Accumulated Depreciation – PPE GH¢3.9 million
(Depreciation of cost of factory net of grant over 10 years)
Option 2
Depreciation of factory:
Dr Profit or loss GH¢5.2 million
Cr Accumulated Depreciation – PPE GH¢5.2 million
(Depreciation of gross factory cost over 10 years)

93
Amortisation of grant:
Dr Deferred income GH¢1.3 million
Cr Profit or loss GH¢1.3 million
(Amortisation of grant over 10 years, reflecting the proportional expensing of the factory to
which the grant relates)
The employment grant relates entirely to the cost of employing staff in that year. Hence it
should be entirely recognised as income in the year ended 31 March 2019.
Recognition of employment grant:
Dr Cash GH¢1.2 million
Cr profit or loss 1.2 million
(recognition of employment grant as income as received)

94
THE EFFECTS OF CHANGES IN FOREIGN
EXCHANGE RATES (IAS 21)
Objective
The objective of this Standard is to prescribe how to include foreign currency transactions and
foreign operations in the financial statements of an entity and how to translate financial
statements into a presentation currency.

Scope
This Standard shall be applied:
(a) in accounting for transactions and balances in foreign currencies, except for those derivative
transactions and balances that are within the scope of IFRS 9 Financial Instruments;
(b) in translating the results and financial position of foreign operations that are included in the
financial statements of the entity by consolidation, proportionate consolidation or the equity
method; and
(c) in translating an entity‗s results and financial position into a presentation currency.

Definitions
The following terms are used in this Standard with the meanings specified:
Closing rate is the spot exchange rate at the end of the reporting period.
Exchange difference is the difference resulting from translating a given number of units of one
currency into another currency at different exchange rates.
Exchange rate is the ratio of exchange for two currencies.
Fair value is the price that would be received to sell an asset or paid to transfer a liability in an
orderly transaction between market participants at the measurement date.
Foreign currency is a currency other than the functional currency of the entity.
Foreign operation is an entity that is a subsidiary, associate, joint venture or branch of a
reporting entity, the activities of which are based or conducted in a country or currency other
than those of the reporting entity.
Functional currency is the currency of the primary economic environment in which the entity
operates.
A group is a parent and all its subsidiaries.
Monetary items are units of currency held and assets and liabilities to be received or paid in a
fixed or determinable number of units of currency.
Net investment in a foreign operation is the amount of the reporting entity‗s interest in the net
assets of that operation.
Presentation currency is the currency in which the financial statements are presented.
Spot exchange rate is the exchange rate for immediate delivery.

INDICATORS OF FUNCTIONAL CURRENCY


Primary Economic Indicators
The primary economic environment in which an entity operates is normally the one in which it
primarily generates and expends cash. An entity considers the following factors in determining
its functional currency:
(a)the currency:
(i) that mainly influences sales prices for goods and services and

95
(ii) of the country whose competitive forces and regulations mainly determine the sales
prices of its goods and services.
(b)the currency that mainly influences labour, material and other costs of providing goods or
services

Secondary Indicators
The following factors may also provide evidence of an entity‗s functional currency (judgements
from management if functional currency is not immediately):
(a)the currency in which funds from financing activities (i.e. issuing debt and equity
instruments) are generated.
(b)the currency in which receipts from operating activities are usually retained.

FUNCTIONAL CURRENCY OF FOREIGN OPERATIONS


The following additional factors are considered in determining the functional currency of a
foreign operation:
(a) whether the activities of the foreign operation are carried out as an extension of the reporting
entity, rather than being carried out with a significant degree of autonomy.
(b) whether transactions with the reporting entity are a high or a low proportion of the foreign
operation‗s activities.
(c)whether cash flows from the activities of the foreign operation directly affect the cash flows
of the reporting entity and are readily available for remittance to it.
(d)whether cash flows from the activities of the foreign operation are sufficient to service
existing and normally expected debt obligations without funds being made available by the
reporting entity

Change in Functional Currency


Once determined, the functional currency is not changed unless there is a change in those
underlying transactions, events and conditions.

Currency of Hyperinflationary Economies


If the functional currency is the currency of a hyperinflationary economy, the entity‗s financial
statements are restated in accordance with IAS 29 Financial Reporting in Hyperinflationary
Economies.

Reporting Foreign Currency Transactions in the Functional Currency


Initial recognition
A foreign currency transaction is a transaction that is denominated or requires settlement in a
foreign currency, including transactions arising when an entity:
(a)buys or sells goods or services whose price is denominated in a foreign currency;
(b)borrows or lends funds when the amounts payable or receivable are denominated in a
foreign currency; or
(c)otherwise acquires or disposes of assets, or incurs or settles liabilities, denominated in a
foreign currency.

A foreign currency transaction shall be recorded, on initial recognition in the functional


currency, by applying to the foreign currency amount the spot exchange rate between the
functional currency and the foreign currency at the date of the transaction.

96
Subsequent measurement
A foreign currency transaction may give rise to assets or liabilities that are denominated in a
foreign currency. These assets and liabilities will need to be translated into the entity's
functional currency at each reporting date. How they will be translated depends on whether the
assets or liabilities are monetary or non-monetary items.

Monetary items
The essential feature of a monetary item, as the definition implies, is the right to receive (or an
obligation to deliver) a fixed or determinable number of units of currency. Examples of
monetary assets include:
Cash and bank balances
Trade receivables and payables
Loan receivables and payables
Foreign currency bonds held as available for sale
Foreign currency bonds held to maturity
Pensions and other employee benefits to be paid in cash
Provisions that are to be settled in cash
Cash dividends that are recognised as a liability
A contract to receive (or deliver) a variable number of the entity's own equity instruments or a
variable amount of assets in which the fair value to be received (or delivered) equals a fixed or
determinable number of units of currency

Foreign currency monetary items outstanding at the end of the reporting date shall be
translated using the closing rate. The difference between this amount and the previous carrying
amount in functional currency is an exchange gain or loss.

Non-monetary items
A non-monetary item does not give the right to receive or create the obligation to deliver a fixed
or determinable number of units of currency. Examples of non-monetary items include:
Amounts prepaid for goods and services (e.g. prepaid rent)
Goodwill
Intangible assets
Inventories
Property, plant and equipment
Provisions to be settled by the delivery of a non-monetary asset
Equity instruments that are held as available for sale financial assets
Equity investments in subsidiaries, associates or joint ventures

Non-monetary items carried at historic cost are translated using the exchange rate at the date of
the transaction when the asset arose (historical rate). They are not subsequently retranslated in
the individual financial statements of the entity.

Issues in the measurement of non-monetary assets


Subsequent depreciation should be translated on the same basis as the asset to which it relates,
so the rate at the date of acquisition for assets carried at cost and at the rate at the last valuation

97
date for assets carried at revalued amounts. Application of the depreciation method to the
translated amount will achieve this.
The carrying amount of inventories is the lower of cost and net realisable value in accordance
with IAS 2 Inventories. The carrying amount in the functional currency is determined by
comparing:
The cost, translated at the exchange rate at the date when that amount was determined.
The net realisable value, translated at the exchange rate at the date when that value was
determined (e.g. the closing date at the reporting date).

Impairment testing of foreign currency non-monetary assets


Similarly in accordance with IAS 36 Impairment of Assets, the carrying amount of an asset for
which there is an indication of impairment, is the lower of:
The carrying amount, translated at the exchange rate at the date when that amount was
determined.
The recoverable amount, translated at the exchange rate at the date when that value was
determined (e.g. the closing rate at the reporting date).

The effect of this comparison may be that an impairment loss is recognised in the functional
currency but would not be recognised in the foreign currency or vice versa.

The Rules
The rules in IAS 21 for reporting assets and liabilities at the end of a subsequent reporting
period make a distinction between:
monetary items, such as trade payables and trade receivables, and
non-monetary items, such as non-current assets and inventory.

The rules are as follows, for entities preparing their individual financial statements:

Asset or liability Accounting treatment for the


statement of financial position:
Monetary items Re-translate at the closing rate.
Non-monetary items carried at cost No re-translation. The transaction is left at the
original spot rate.
Non-monetary items carried at Re-translate at the exchange rate ruling at the
fair value date of the fair value adjustment.

Recognition of Exchange Differences


Exchange differences arising on the settlement of monetary items or on translating monetary
items at rates different from those at which they were translated on initial recognition during
the period or in previous financial statements shall be recognised in profit or loss in the period
in which they arise, except as described below

There are two situations to consider.


(a)The transaction is settled in the same period as that in which it occurred: all the exchange
difference is recognised in that period.

98
(b)The transaction is settled in a subsequent accounting period: the exchange difference
recognised in each intervening period up to the period of settlement is determined by the
change in exchange rates during that period.
When a gain or loss on a non-monetary item is recognised in other comprehensive income, any
exchange component of that gain or loss shall be recognised in other comprehensive income.
Conversely, when a gain or loss on a non-monetary item is recognised in profit or loss, any
exchange component of that gain or loss shall be recognised in profit or loss.

Exchange differences arising on a monetary item that forms part of a reporting entity‗s net
investment in a foreign operation shall be recognised in profit or loss in the separate financial
statements of the reporting entity or the individual financial statements of the foreign operation,
as appropriate.

In the financial statements that include the foreign operation and the reporting entity (e.g.
consolidated financial statements when the foreign operation is a subsidiary), such exchange
differences shall be recognised initially in other comprehensive income and reclassified from
equity to profit or loss on disposal of the net investment.

Change in functional currency


When there is a change in an entity‗s functional currency, the entity shall apply the translation
procedures applicable to the new functional currency prospectively from the date of the change.

The foreign operation: accounting rules


Three stages in the consolidation process:
The translation stage
The consolidation stage

The Foreign Operation: Accounting Rules


Three stages in the consolidation process
If a company has a foreign operation (such as a foreign subsidiary) that prepares its accounts in
a functional currency that is different from the group's presentation currency, there are three
stages in the accounting process, for the purpose of preparing consolidated financial statements
(or including the foreign associate or joint venture in the financial statements of the reporting
entity).

Stage Description
Adjust and update Ensure that the individual financial statements of the
foreign entity is correct and up-to-date.
If any adjustments are required to correct the financial
statements of the foreign entity, these should be made in
the statements of the foreign entity and in its own
functional currency.

Translate The assets and liabilities of the foreign entity should be


translated into the presentation currency of the parent
company. (As explained earlier, the presentation currency
of the parent company might be the same or might be

99
different from its functional currency.)
The rules for translation are explained below.

Consolidate After translation, all the financial statements are now in


the same currency.
Normal group accounting principles are now used to
prepare the consolidated accounts of the group.

The translation stage


The rules set out below apply where the functional currency of the foreign entity is not a
currency suffering from hyperinflation. The normal rules for translation, contained in IAS 21,
are as follows:
(1)The statement of financial position
The assets and liabilities of the foreign operation, for inclusion in the consolidated statement of
financial position, are translated at the closing rate. (Comparative figures for the previous year
are translated at the same rate.)
For foreign subsidiaries, this rule also applies to purchased goodwill arising on the acquisition
of the subsidiary.

(2)The statement of profit or loss and other comprehensive income


Income and expenses for inclusion in the consolidated statement of profit or loss and other
comprehensive income are translated at the spot rates at the dates of each of the transactions.
For practical reasons, average rates for a period may be used, if they provide a reasonable
approximation of the spot rates when the transactions took place.

(3)Exchange differences
All resulting exchange differences are recognised in other comprehensive income for the
period and are credited (gain) or debited (loss) to a separate reserve within the equity section of
the consolidated statement of financial position, and this reserve is maintained within equity
until the foreign operation is eventually disposed of.
Gains or losses are therefore reported as gains or losses in other comprehensive income and
movements in the separate reserve, and not as a gain or loss in profit or loss and an increase or
reduction in retained earnings.

The gain or loss on translation


The exchange differences on translation result in a gain or loss. These gains or losses arise from
a combination of two factors:
Income and expense items are translated at the exchange rates ruling during the period, but
assets and liabilities are translated at closing rates. The profit is therefore calculated at the actual
exchange rates, but the accumulated profit in the consolidated statement of financial position is
re-translated at the closing rate.
The net assets of the subsidiary were translated at last year's closing rate at the end of the
previous financial year. These net assets have now been retranslated and included in this year's
statement of financial position at this year's closing rate.

IAS 21 states that these differences on translation are not recognised in profit or loss because
changes in the exchange rates for these items have little or no effect on cash flows from

100
operations. It would therefore be misleading to include them in profit or loss. However the
actual treatment is that the exchange loss should be recognised in other comprehensive income
for the year and taken to a separate reserve within equity in the consolidated statement of
financial position.

The consolidation stage


After the translation stage, the financial statements of the overseas entity are in the presentation
currency of the parent company.
The basic rule is that normal consolidation techniques can now be used. However, foreign
exchange reserve must be included in the consolidated statement of financial position for the
cumulative exchange differences.
It is also necessary to comply with the requirements of IAS 21 for purchased goodwill and
foreign subsidiaries.

Purchased goodwill and foreign subsidiaries


IAS 21 requires that goodwill and any fair value adjustments arising on the acquisition of a
foreign subsidiary are to be treated as part of the assets and liabilities of the foreign subsidiary.
The rules already described apply to these items.

This means that:


Goodwill arising on the purchase of the foreign subsidiary (and also any fair value
adjustments to the value of assets of the subsidiary) should be stated in the functional currency
of the foreign subsidiary.
The goodwill and fair value adjustments will therefore be translated each year at the closing
exchange rate.

A gain or loss on translation will therefore arise (as described above for other assets and
liabilities).

The effect of this rule is that goodwill and the acquisition of a foreign operation is re-stated over
time because it is re-translated every year at the new closing exchange rate.

Exchange differences in other comprehensive income


Using the method of creating the consolidated statement of financial position shown in the
previous example, you do not need to worry about exchange differences. By translating every
balance in the subsidiary's statement of financial position at the closing rate, the exchange
differences are automatically included in reserves.
However, you may be asked to calculate exchange differences arising for reporting in other
comprehensive income and a separate equity reserve.

The easiest way to work out the exchange differences (excluding the gain or loss on re-
translation of goodwill) is to create the accounting equation for the foreign subsidiary in its own
currency. Once this is translated into the parent's currency it will not balance, and the exchange
differences are the balancing figure.
These are the exchange differences arising from:
re-translating the opening net assets of the subsidiary at the closing rate, and
re-translating the subsidiary's post-acquisition profit at the closing rate.

101
Disposal of a foreign subsidiary
Most of the accounting rules for the disposal of a foreign subsidiary, or for the partial disposal
of a foreign subsidiary, are set out in IAS 27 (revised). Disposals are explained in Level 3
Corporate Reporting.

However IAS 27 does not deal with the accounting treatment of the balance on the separate
equity reserve account when a foreign subsidiary is disposed of. This matter is dealt with by
IAS 21.
When the entire investment in a foreign subsidiary is disposed of, the cumulative balance in
the separate equity reserve (which represents amounts previously recognised in other
comprehensive income) should now be reclassified from equity to profit and loss.
If there was a non-controlling interest in the subsidiary, the NCI is derecognised in the
consolidated statement of financial position. Amounts previously recognised in other
comprehensive income and attributed to NCI must not be reclassified and recognised in profit
or loss of the reporting entity.
When a proportion of an investment in a foreign subsidiary is disposed of, a proportionate
share of the amounts previously recognised in other comprehensive income (the cumulative
balance in the separate equity reserve) should now be reclassified from equity to profit or
loss.When income previously recognised as other comprehensive income is reclassified as a
gain or loss to profit or loss as a re-classification adjustment, there must be an offsetting loss or
gain in other comprehensive income, to avoid double-counting of the gain (or loss).In other
comprehensive income, negative income of GHC2 million should be recognised, to avoid
double counting of the income previously recognised as other comprehensive income but now
reclassified in profit or loss.
Disclosure
The amount of exchange differences recognised in profit or loss (excluding differences arising
on financial instruments measured at fair value through profit or loss in accordance with IAS
39).
Net exchange differences recognised in other comprehensive income and accumulated in a
separate component of equity,and a reconciliation of the amount of such exchange differences at
the beginning and end of the period.
When the presentation currency is different from the functional currency, disclose that fact
together with the functional currency and the reason for using a different presentation currency.
A change in the functional currency of either the reporting entity or a significant foreign
operation and the reason therefore.When an entity presents its financial statements in a
currency that is different from its functional currency, it may describe those financial statements
as complying with IFRS only if they comply with all the requirements of each applicable
Standard (including IAS 21) and each applicable interpretation.
QUESTION 1
Aspire, a public limited company, operates many of its activities overseas. The directors have
asked for advice on the correct accounting treatment of several aspects of Aspire‘s overseas
operations. Aspire‘s functional currency is the dollar.
(a)Aspire has created a new subsidiary, which is incorporated in the same country as Aspire.
The subsidiary has issued 2 million dinars of equity capital to Aspire, which paid for these
shares in dinars. The subsidiary has also raised 100,000 dinars of equity capital from external
sources and has deposited the whole of the capital with a bank in an overseas country whose
currency is the dinar. The capital is to be invested in dinar denominated bonds. The subsidiary

102
has a small number of staff and its operating expenses, which are low, are incurred in dollars.
The profits are under the control of Aspire. Any income from the investment is either passed on
to Aspire in the form of a dividend or reinvested under instruction from Aspire. The subsidiary
does not make any decisions as to where to place the investments.
Aspire would like advice on how to determine the functional currency of the subsidiary.
(7 marks)
(b)Aspire has a foreign branch which has the same functional currency as Aspire. The branch‘s
taxable profits are determined in dinars. On 1 May 2013, the branch acquired a property for 6
million dinars. The property had an expected useful life of 12 years with a zero residual value.
The asset is written off for tax purposes over eight years. The tax rate in Aspire‘s jurisdiction is
30% and in the branch‘s jurisdiction is 20%. The foreign branch uses the cost model for valuing
its property and measures the tax base at the exchange rate at the reporting date.
Aspire would like an explanation (including a calculation) as to why a deferred tax charge
relating to the asset arises in the group financial statements for the year ended 30 April 2014
and the impact on the financial statements if the tax base had been translated at the historical
rate.
(6 marks)
(c)On 1 May 2013, Aspire purchased 70% of a multi-national group whose functional currency
was the dinar. The purchase consideration was $200 million. At acquisition, the net assets at
cost were 1,000 million dinars. The fair values of the net assets were 1,100 million dinars and the
fair value of the non-controlling interest was 250 million dinars. Aspire uses the full goodwill
method.
Aspire wishes to know how to deal with goodwill arising on the above acquisition in the group
financial statements for the year ended 30 April 2014. (5 marks)
(d)Aspire took out a foreign currency loan of 5 million dinars at a fixed interest rate of 8% on 1
May 2013. The interest is paid at the end of each year. The loan will be repaid after two years on
30 April 2015. The interest rate is the current market rate for similar two-year fixed interest
loans.
Aspire requires advice on how to account for the loan and interest in the financial statements
for the year ended30 April 2014. (5 marks)
Aspire has a financial statement year end of 30 April 2014 and the average currency exchange
rate for the year is not materially different from the actual rate.
Exchange rates $1 = dinars
1 May 2013 5
30 April 2014 6
Average exchange rate for year ended 30 April 2014 5·6
Required:
Advise the directors of Aspire on their various requests above, showing suitable calculations
where necessary.
Note: The mark allocation is shown against each of the four issues above.
Professional marks will be awarded in question 2 for clarity and quality of presentation. (2 marks)
(25 marks)

Solution:
(a)The functional currency is the currency of the primary economic environment in which the
entity operates, which is normally the one in which it primarily generates and expends cash. An

103
entity‘s management considers the following primary indicators in determining its functional
currency:
(i) the currency which mainly influences sales prices for goods and services;
(ii) the currency of the country whose competitive forces and regulations mainly determine the
sales prices of goods and services; and
(iii) the currency which mainly influences labour, material and other costs of providing goods
and services.

Further secondary indicators which may also provide evidence of an entity‘s functional
currency are the currency in which funds from financing activities are generated and in which
receipts from operating activities are retained.

Additional factors are considered in determining the functional currency of a foreign operation
and whether its functional currency is the same as that of the reporting entity. These are:
(a) the autonomy of a foreign operation from the reporting entity;
(b) the level of transactions between the two;
(c) whether the foreign operation generates sufficient cash flows to meet its cash needs; and
(d) whether its cash flows directly affect those of the reporting entity.

When the functional currency is not obvious, management uses its judgement to determine the
functional currency which most faithfully represents the economic effects of the underlying
transactions, events and conditions.
In the case of Aspire, the subsidiary does not make any decisions as to the investment of funds,
and consideration of the currency which influences sales and costs is not relevant. Although the
costs are incurred in dollars, they are not material to any decision as to the functional currency.
Therefore it is important to look at other factors to determine the functional currency. The
subsidiary has issued 2 million dinars of equity capital to Aspire, which is a different currency
to that of Aspire,but the proceeds have been invested in dinar denominated bonds at the
request of Aspire.
The subsidiary has also raised100,000 dinars of equity capital from external sources but this
amount is insignificant compared to the equity issued to Aspire. The income from investments
is either remitted to Aspire or reinvested on instruction from Aspire. The subsidiary has a
minimum number of staff and does not have any independent management. The subsidiary is
simply a vehicle for the parent entity to invest in dinar related investments. Aspire may have set
up the entity so that any exposure to the dinar/dollar exchange rate will be reported in other
comprehensive income through the translation of the net investment in the subsidiary.There
does not seem to be any degree of autonomy as the subsidiary is merely an extension of
Aspire‘s activities.
Therefore the functional currency would appear to be the dollar.

In contrast, the dinar represents the currency in which the economic activities of the subsidiary
are primarily carried out as is the case regarding the financing of operations and retention of
any income not remitted. However, the investment of funds could have been carried out
directly by Aspire and therefore the parent‘s functional currency should determine that of the
subsidiary.
(b)Where a foreign branch‘s taxable profit is determined in a foreign currency, changes in
exchange rates may give rise to temporary differences. This can arise where the carrying

104
amounts of the non-monetary assets are translated at historical rates and the tax base of those
assets are translated at the rate at the reporting date. An entity may translate the tax base at the
year-end rate as this rate gives the best measure of the amount which will be deductible in
future periods. The resulting deferred tax is charged or credited to profit or loss.
Property Dinars (000) Exchange rate Dollars (000)
Cost 6,000 5 1,200
Depreciation for year (500) (100)
–––––– ––––––
Net book amount 5,500 1,100
Tax base Cost 6,000
Tax depreciation (750)
––––––
5,250 6 875
Temporary difference 225
Deferred tax at 20% 45
The deferred tax arising will be calculated using the tax rate in the overseas country. The
deferred tax arising is therefore $45,000, which will increase the tax charge in profit or loss. If
the historical rate had been used, the tax base would have been $1·05 million (5·25m/5) which
would have led to a temporary difference of $50,000 and a deferred tax liability of$10,000,
which is significantly lower than when the closing rate is used.
(c)The goodwill arising when a parent acquires a multinational operation with several
currencies is allocated to each level of functional currency. Goodwill arising on acquisition of
foreign operations and any fair value adjustments are both treated as the foreign operation‘s
assets and liabilities. They are expressed in the foreign operation‘s functional currency and
translated at the closing rate. Exchange differences arising on the retranslation of foreign
entities‘ financial statements are recognised in other comprehensive income and accumulated as
a separate component of equity
Exchange rate at 1 May 2013 $1= 5 dinars
Exchange rate at 30 April 2014 $1 =6 dinars
Net assets at fair value 1,100m dinars
Translated at 1 May 2013 $220m
Purchase consideration $200m
NCI (250m dinars/5) $50m
Goodwill $30m
Goodwill treated as foreign currency asset at 1 May 2013 ($30m x 5) 150m dinars
Goodwill translated at closing rate at 30 April 2014 (150m dinars/6) $25m
Translation adjustment for goodwill in equity ($5m)
An exchange loss of $5 million will be charged in other comprehensive income together with
any gain or loss on the retranslation of the net assets of the operations.
(d)The loan balance, as a monetary item, is translated at the spot exchange rate at the year-end
date. Interest is translated at the average rate because it approximates to the actual rate. Because
the interest is at a market rate for a similar two-year loan, Aspire measures the loan on initial
recognition at the transaction price translated into the functional currency. Because there are no
transaction costs, the effective interest rate is 8%.
On 1 May 2013, the loan is recorded on initial recognition as follows:
Dr Cash $1 million
Cr Loan payable – financial liability $1 million

105
Year ended 30 April 2014
Aspire records the interest expense as follows:
Dr Profit or loss – interest expense $71,429
Cr Loan payable – financial liability $71,429
To recognise interest payable for the year ended 30 April 2014 (0·4 million dinars/5·6).
On 30 April 2014 the interest is paid and the following entry is made:
Dr Loan payable – financial liability $66,666
Cr Cash $66,666
To recognise the payment of 2014 interest on financial liability (0·4 million dinars/6).
At 30 April 2014 the loan is recorded at 5 million dinars/6, i.e. $833,333, which gives rise to an
exchange gain of$166,667. In addition to this, a further exchange gain of $4,763 arises on the
translation of the interest paid ($71,429 –$66,666). The total exchange gain is therefore $171,430.

QUESTION 2
On 1 August 2018 Charlie Ltd, whose functional currency is the cedi, bought a property in
Morocco for DH40 million. The property had a 20-year useful economic life with no residual
value estimated. On 31 July 2019, the property was revalued to DH45 million.
Exchange rates were:
1 January 2018 GH¢1 = DH 1.32
1 August 2018 GH¢1 = DH 1.25
31 July 2019 GH¢1 = DH 1.125
Required:
In accordance with IAS 21: The Effects of Changes in Foreign Exchange Rates and IAS 16:
Property, Plant & Equipment how much should be recognised in Statement of Profit or Loss
and Other Comprehensive Income for year ended 31 July 2019? (5 marks)
ICAG NOV.2019
SOLUTION 2
Under IAS 21 an asset purchased in foreign currency is translated into the functional currency
of the entity at the date of purchase or revaluation and not restated otherwise. (1 mark)
Hence there are two movements here:
Cost of property: DH40 million / 1.25 = GH¢32 million (1 mark)
Depreciation for year: GH¢32 million / 20 years = GH¢1.6 million
(charged to profit or loss) (1 mark)
Carrying value at year end: GH¢30.4 million
Revalued amount: DH¢45 million / 1.125 = GH¢40 million (1 mark)
Revaluation gain: (40m – 30.4m) = GH¢9.6 million (credited to OCI) (1 mark)

QUESTION 3
Discuss what is meant by the concept of an entity‘s functional currency and how it may be
determined in accordance with IAS 21: The Effects of Changes in Foreign Exchange Rates.
(5 marks)
ICAG MAY,2020
SOLUTION 3
The functional currency of an entity can be understood literally as the currency in which the
entity functions. The choice of functional currency is a judgment which must be made under
IAS 21. The judgment involves assessing the facts, and deciding the currency on which the

106
entity is most dependent economically. For most entities, the functional currency is a clear
judgment, in that most entities operate primarily within a single economy or currency zone.
However IAS 21 does offer some guidance should the judgment prove difficult. This can
happen if more than one currency is important to the entity and it is not clear which is the most
significant.
IAS 21 requires that the entity consider:

ncurred.

QUESTION 4
Medina Power Ltd has carried out certain transactions denominated in foreign currency
during its financial year ended 31 October 2019 and has also conducted foreign operations
through a foreign entity. Medina Power Ltd.‘s functional and presentation currency is the
cedi.
On 31 July 2019 Medina Power Ltd purchased goods from a foreign supplier for 16 million
dinars. At 31 October 2019, the supplier had not yet been paid and the goods were still held
in inventory by Medina Power Ltd.
On 31 July Medina Power Ltd sold goods to a foreign customer for 8 million dinars and it
received payment for the goods in dinars on 31 October 2019.
Medina Power Ltd had also purchased an investment property on 1 November 2018 for 56
million dinars. At 31 October 2019, the investment property had a fair value of 48 million
dinars. The company uses the fair value model in accounting for investment properties.
Medina Power Ltd wants advice on how to treat these transactions in the financial
statements for the year ended 31 October 2019.
Exchange rate (direct quotes) GH¢/1 Average rate for year to:
1 November 2017 GH¢0.9091/1 dinar
31 October 2018 GH¢0.7143/1 dinar 1 GH¢0.8333/1 dinar
1 November 2018 GH¢0.7143/1 dinar
31 July 2019 GH¢0.6250/1 dinar
31 October 2019 GH¢0.7692/1 dinar GH¢0.6667/1 dinar
Required
Discuss the accounting treatment of the above transaction in accordance with the advice
required by the directors. (You should show detailed workings as well as a discussion of
the accounting.
SOLUTION 4
Inventory, sale of goods and investment property
Inventory and payable
The inventory and trade payable would be recorded initially at GH¢10 million (16 million
dinars X GH¢0.6250).
At the year-end on 31 October 2019, the amount payable is still outstanding. It should be re-
translated at the closing rate to. GH¢12.3 million (16 million dinars X GH¢0.7692).
This creates an exchange loss of GH¢2.3 million (12.3 – 10) which should be recognised in
profit or loss.

107
Unless it has been impaired, the inventory (a non-monetary asset) should be recorded at
GH¢10 million at the year end.
Sale of goods
The sale of goods should be recorded at GH¢5 million (8 million dinars X GH¢0.6250)
million as revenue and as a trade receivable.
Payment in dinars was received on 31 October 2019 and the actual cedi value of the dinars
received was GH¢6.2 million (8 million dinars X GH¢0.7692). This creates a gain on
exchange of GH¢1.2 million (6.2 – 5) which should be recognised in profit or loss.
Investment property
The investment property should be recognised on 1 November 2018 at GH¢40 million (56
million dinars X GH¢0.7143). At the year-end on 31 October 2019 the property should be
recognised at its fair value of GH¢36.9 million (48 million dinars X GH¢0.7692). The fall in
fair value (40 – 36.9 = 3.1) should be recognised in profit and loss as a loss on investment
property. The property is a non-monetary asset and when a gain or loss on a non-monetary
item is recognised in profit or loss, the element of the gain or loss relating to exchange rates
is also recognised in profit or loss.
QUESTION 5
Nyinahini Ltd (Nyinahini) is a company reporting under IFRS. Nyinahini normally operates only
within the country where its buildings are physically located. Recently it entered into a contract to
supply its products to a new client based in South Africa. All the work was completed in the period
October to November 2018. The (fixed) contract price of 100 million Rand has been agreed as
denominated in South African Rand. The full amount was invoiced on 1 December 2018 when the
exchange rate was GH¢1 = 10.1889 Rand. The new client paid 50 million Rand in advance on 1
November 2018 when the exchange rate was GH¢1 = 9.9783 Rand. The balance will be paid in two
equal instalments on 31 March 2019 and 30 June 2019. The exchange rate at 31 December 2018 was
GH¢1 = 10.5037 Rand.
Nyinahini decided to eliminate exchange rate differences on the final two payments and entered into
two forward rate agreements on 1 December 2018 to sell the appropriate amount of Rand on 31
March 2019 and 30 June 2019, and set up the relevant documentation to treat them as fair value
hedges of the recognised receivables. At 31 December 2018, the two contracts for settlement on 31
March 2019 and 30 June 2019 were valued at GH¢148,000 collectively, as an asset from Nyinahini's
point of view.
Required:
Set out and discuss the accounting treatment of the above items, including relevant calculations, as
the information provided permits, in the financial statements of Nyinahini for the year ended 31
December 2018. (6 marks)
SOLUTION
IAS 21: The Effects of Changes in Foreign Exchange Rates governs the treatment of
transactions in foreign currency. The invoice should be recorded as a sale and a receivable
on 1 December 2018 at the spot exchange rate at the date, i.e. at GH¢9,814,602
(100m/10.1889).

(50m/9.9783). On 1 December 2018 an exchange gain of GH¢103,573 (GH¢5,010,874 –


GH¢4,907,301 (50m/10.1889)) is recognised in profit or loss when 50 million rand of the
translated 100 million rand receivable is derecognised.

108
GH¢4,760,227 (50m/10.5037) generating an exchange loss of GH¢147,074 (GH¢4,907,301
(50m/10.1889) – GH¢4,760,227).

asset as the rand has depreciated in value) in the statement of financial position.
0 is a credit to
profit or loss, eliminating the exchange loss recognised on the receivables.

109
BORROWING COSTS (IAS 23)
Objective
This standard prescribes the accounting treatment of borrowing cost, the circumstance in which
the borrowing cost will be capitalized and when it will be recognized as expense.
Scope
The requirements of this Standard are applicable to deal with the accounting treatment of
borrowing cost. However this standard does not applies to the actual or imputed cost related to
the equity instruments.
Definitions
1. Borrowing Cost:
It is interest cost and any other cost which arises, in order to borrow the funds. It includes:
 Interest
 Discount on issuance of loan note or debenture
 Premium on redemption of loan note debenture
 Any interest cost included in finance lease
 Interest on overdraft
 Any Issuance cost on loan instruments
Borrowing cost does not include actual or imputed cost of equity capital, including any
preferred capital not classified as a liability pursuant to IAS 32
2. Qualifying Asset:
An asset, that essentially takes a long or substantial time period to get ready for sale or intended
use by the entity. For example Inventory, Investment property, or any self constructed asset
which takes a long time period to get complete.
 Qualifying asset does not include assets which are ready for sale or use, at the time
when these are acquired and the assets which are completed in the short interval.
Recognition of Borrowing Cost
The borrowing cost which is incurred for the construction or acquisition of a Qualifying Asset will
be capitalized as part of cost of such asset. Any other borrowing cost will be treated as expense
and will be charged to the statement of profit and loss.
 The borrowing cost which relates to a qualifying asset is called ‘Eligible Borrowing Cost’ as it
becomes eligible to be capitalized in the cost of asset.
 The borrowing cost related to qualifying asset, which becomes eligible to be capitalized, is
that borrowing cost that can be avoided if that asset is not produced or constructed.
 The cost of qualifying asset including the capitalized borrowing cost should not exceed the
Recoverable value of the asset, if exceeded then the asset will be written down to its
recoverable value as per the requirements of IAS 36.
Accounting Treatment
 The benchmark treatment is that all borrowing costs should be expensed in the period in
which they are incurred. The allowed alternative treatment is that borrowing costs in relation
to the acquisition, construction and production of a qualifying asset should be treated as part
of the cost of the relevant asset.
 Where the allowed alternative is adopted, that treatment should be applied consistently to all
borrowing costs incurred for the acquisition, construction and production of qualifying
assets.

110
Measurement of Eligible Borrowing Cost to be Capitalized
The measurement of the borrowing cost related to the qualifying asset which is capitalize as
part of the cost of such asset, depends upon:
1. Specific Loan/Fund:
The loan which is specifically borrowed for the construction or acquisition of a qualifying asset
only is called specific loan. In such situation the borrowing cost eligible for capitalization will be
calculated as, actual borrowing cost incurred on the asset less any income from temporary investment of
funds during the period of construction.
2. General Loan/Funds:
The loan which is borrowed for the qualifying asset and general use in business both is called
general loan. In such situation the borrowing cost eligible for capitalization will be calculated as,
the expenditure on the qualifying asset during the accounting period will be multiplied with weighted
average borrowing cost percentage of the entity in respect of the loans which were outstanding
during the accounting period.
Commencement of Capitalization:
The capitalization of borrowing cost will start right from the date when entity will meet all the
following conditions:
 The expenditure on the asset has been started;
 Borrowing cost is being incurred;
 The activities necessary to complete the asset are in progress.
 The expenditure on a qualifying asset includes the expenditure in the form of payments for
the material, associated labor cost and related overheads.
 The activities necessary to complete the asset includes not only the physical construction of
the asset, it also encompasses any technical working, administrative work and taking
planning permission from related authorities before the start of physical construction work.
 The borrowing cost can only be capitalized, during the period when activities necessary to
complete the asset are in progress. And the borrowing cost during the period when activities
necessary to complete the asset are interrupted will not be capitalized and such borrowing
cost will be charged to the statement of profit & loss as an expense.
Suspension of Capitalization of Borrowing Cost:
If during the period of construction, the activities necessary to complete the asset are
interrupted or suspended due to particular reasons, the borrowing cost of such period will be
accounted for as follows:
 If the period of interruption is material, the borrowing cost of such period will not be
capitalized and will be charged to statement of profit and loss as an expense.
 If the period of interruption is immaterial or temporary such as (Material shortage or labor
strikes), then entity may continue to capitalize the borrowing cost during such period.
Cessation of Capitalization:
 The capitalization of the borrowing cost will cease, when activities necessary to complete the
asset are finished i.e. the completion of the physical structure of the qualifying asset,
although some administrative or decorative work may still continue.
 If a qualifying asset contains different component parts such as (Industrial plant which has
several processes) and the entity will complete the construction of such qualifying asset by
constructing each part or component on individual basis in a sequence, where each part or
component can be used individually while the construction work is in progress on other
parts or components, the capitalization of the borrowing cost will cease when activities
necessary to complete that part for its intended use or sale has been finished.

111
Disclosure
The standard requires the entity to disclose the following:
 The accounting policy adopted
 Borrowing cost capitalized during the accounting period;
 The weighted average borrowing cost rate or percentage used to determine the
borrowing costs eligible for capitalization.(capitalization rate used)

REVIEW QUESTIONS
QUESTION 1
On 1st January,2015, Antonio Conte Ltd began to construct a supermarket which had an
estimated useful life of 40 years. It purchased a leasehold interest in the site for GHC250,000.The
construction of the building cost GHC90,000 and the fixtures and fittings cost GHC60,000.The
construction of the supermarket was completed on 30th September,2015 and it was brought into
use on 1st January,2016.
Antonio Conte borrowed GHC400,000 on 1st January,2015 in order to finance this project. The
loan carried interest at 10% per annum. It was repaid on 30th June,2016.
Calculate the total amount to be included at cost in property, plant and equipment in respect of
the development at 31st December,2015.
Solution:
GHC’000
Leasehold Site 250
Building 90
Fixtures and Fittings 60
Borrowing Costs (400x10%x9/12) 30
430
QUESTION 2
On 1 January,2006 Richest Club Ltd borrowed GHC1,500,000 to finance the production of two
assets,both of which were expected to take a year to build.Work started during 2006.The loan
facility was drawn down and incurred on 1 January,2006,and was utilized as follows, with the
remaining funds invested temporarily.

Asset A Asset B
GHC’000 GHC’000
1 January,2006 250 500
1 July,2006 250 500

The loan rate was 9% and Richest Club Ltd can invest surplus funds at 7%.
Required:
Ignoring compound interest,calculate the borrowing costs which may be capitalized for each of
the assets and consequently the cost of each asset as at 31 December,2006.

Solution:
Asset A Asset B
GHC GHC
Borrowing Cost to 31st Dec:
(500,000/1,000,000) x 9% 45,000 90,000

112
Less Investment Income:
(250,000/500,000) x 7% x 6/12) (8,750) (17,500)
36,250 72,500
Cost of Assets;
Expenditure Incurred 500,000 1,000,000
Borrowing Costs 36,250 72,500
Total Cost of Asset 536,250 1,072,500

QUESTION 3
Soccer University Ltd had the following loans in place at the beginning and end of 2006.
1 January,2006 31 December,2006
GHC’000 GHC’000
10% Bank Loan repayable 2008 120,000 120,000
9.5% Bank loan repayable 2009 80,000 80,000
8.9% Debenture repayable 2007 - 150,000

The 8.9% debenture was issued to fund the construction of a qualifying asset (a piece of mining
equipment), construction of which began on 1 July,2006.
On 1 January, 2006,Soccer University Ltd began construction of a qualifying asset, a piece of
machinery for a hydroelectric plant, using existing borrowings. Expenditure drawn down for
the construction was GHC30,000,000 on 1 January,2006,GHC 20,000,000 on 1 October,2006.

Required:
Calculate the borrowing costs that can be capitalized for the hydroelectric plant.

Solution:
Weighted Average Capitalisation Rate:
10% x 120,000,000 = 12,000,000
9.5% x 80,000,000 = 7,600,000
200,000,000 19,600,000
Weighted Average rate = 19,600,000 x 100
200,000,000
= 9.8%
Borrowing Costs:
30,000,000 x 9.8% = 2,940,000
20,000,000 x 9.8% = 490,000
3,430,000
QUESTION 4
(i) Draxler is a publicly listed supermarket chain. During the current year it started the building
of a new store. The directors are aware that in accordance with IAS 23 Borrowing costs certain
borrowing costs have to be capitalised.
Required:
Explain the circumstances when, and the amount at which, borrowing costs should be
capitalised in accordance with IAS 23. (3 marks)
(ii) Details relating to construction of Draxler‘s new store:

113
Draxler issued a GHC10 million unsecured loan with a coupon (nominal) interest rate of 6% on
1 April 2009. The loan is redeemable at a premium which means the loan has an effective
finance cost of 7·5% per annum. The loan was specifically issued to finance the building of the
new store which meets the definition of a qualifying asset in IAS 23. Construction of the store
commenced on 1 May 2009 and it was completed and ready for use on 28 February 2010, but
did not open for trading until 1 April 2010. During the year trading at Draxler‘s other stores was
below expectations so Draxler suspended the construction of the new store for a two-month
period during July and August 2009. The proceeds of the loan were temporarily invested for the
month of April 2009 and earned interest of GHC40,000.
Required:
Calculate the net borrowing cost that should be capitalised as part of the cost of the new store
and the finance cost that should be reported in the income statement for the year ended 31
March 2010. (4 marks)
Solution:
(i) Where borrowing costs are directly incurred on a ‗qualifying asset‘, they must be capitalised
as part of the cost of that asset. A qualifying asset may be a tangible or an intangible asset that
takes a substantial period of time to get ready for its intended use or eventual sale. Property
construction would be a typical example, but it can also be applied to intangible assets during
their development period. Borrowing costs include interest based on its effective rate (which
incorporates the amortisation of discounts, premiums and certain expenses) on overdrafts, loans
and (some) other fi nancial instruments and finance charges on finance leased assets. They may
be based on specifically borrowed funds or on the weighted average cost of a pool of funds.
Any income earned from the temporary investment of specifically borrowed funds would
normally be deducted from the amount to be capitalised.
-Capitalisation should commence when expenditure is being incurred on the asset, which is not
necessarily from the date funds are borrowed.
-Capitalisation should cease when the asset is ready for its intended use, even though the funds
may still be incurring borrowing costs.
-Also capitalisation should be suspended if there is a suspension of active development of the
asset.
Any borrowing costs that are not eligible for capitalisation must be expensed. Borrowing costs
cannot be capitalised for assets measured at fair value.
(3 marks)
(ii) The finance cost of the loan must be calculated using the effective rate of 7·5%, so the total
finance cost for the year ended 31 March 2010 is GHC750,000 (GHC10 million x 7·5%). As the
loan relates to a qualifying asset, the finance cost (or part of it in this case) can be capitalised
under IAS 23.
The Standard says that capitalisation commences from when expenditure is being incurred (1
May 2009) and must cease when the asset is ready for its intended use (28 February 2010); in
this case a 10-month period. However, interest cannot be capitalised during a period where
development activity is suspended; in this case the two months of July and August 2009. Thus
only eight months of the year‘s finance cost can be capitalised = GHC500,000 (GHC750,000 x
8/12). The remaining four-months finance costs of GHC250,000 must be expensed. IAS 23 also
says that interest earned from the temporary investment of specific loans should be deducted
from the amount of finance costs that can be capitalised. However, in this case, the interest was
earned during a period in which the finance costs were NOT being capitalised, thus the interest

114
received of GHC40,000 would be credited to the income statement and not to the capitalised
finance costs.
In summary:
Income statement for the year ended 31 March 2010
GHC
Finance cost (debit) (250,000)
Investment income (credit) 40,000

Statement of financial position as at 31 March 2010


GHC
Property, plant and equipment (finance cost element only) 500,000

QUESTION 5
On January 1,2015,Roman Abromovich Ltd contracted Antonio Conte Ltd to construct a new
office building in London for GHC20milllion on land owned by Roman Abromovich Ltd. The
payments made to Antonio Conte Ltd:
Date Amount
GHC’million
01/01/15 4,800
30/09/15 15,200
The construction was completed and the building was ready for occupancy by 31/12/15.Roman
Abromovich Ltd had no new borrowings directly associated with London Office building, but
had the following debt outstanding at 31/12/15:
10 year 30% Debentures GHC8 billion
8 year 22.5% Loan GHC16 billion
The loans were substantially used for the construction of the building. The accounts officer
debited the GHC20 billion paid to Antonio Conte Ltd to capital WIP and included it under fixed
assets in the statement of financial position without depreciation .The policy of the company
(Roman Abromovich) is to capitalize borrowing cost and to depreciate the office building at 2%
p.a.
Required:
Show all the necessary computations and determine the amount to be capitalize and
expensed(ignore depreciation).

SOLUTION:
Weighted average rate:
30% x 8,000 = 2,400
22.5% x 16,000 = 3,690
24,000 6,090
Rate = 6,090 x 100
24,000
= 25.28%
Borrowing Costs;
25.38% x 4,800 x 12/12 = 1,218.24
25.38% x 15,200 x 3/12 = 964.44
Borrowing Cost to be capitalised 2,182.68
Borrowing Cost to be Expensed:

115
Actual Finance Cost (25.38% x 24,000) = 6,091.20
Capitalised Borrowing costs = 2,182.68
Borrowing cost to be expensed(P & L) = 3,908.52

QUESTION 6
On 1 May, 2014, Fabregas Ltd borrowed GHC100 million at an interest rate of 35% p.a to
finance construction of a hotel. In addition, on 1 September, 2014, the company borrowed a loan
of GHC50 million 36% Debentures to finance other activities of the company.
Construction of the hotel is expected to be completed in the year 2016.Expenditure on the
project is as follows:
GHC’million
1/08/14 20
1/09/14 30
1/12/14 20
1/03/15 20
The company prepares account to 31 March each year.

Required:
By applying the appropriate accounting standard on capitalization of borrowing costs;
calculate the interest that Fabregas Ltd should capitalized.
Solution :
Eligible Borrowing Costs: GHC
1/08/14 35%x20,000,000x 8/12 4,666,666.67
1/09/14 35%x30,000,000x 7/12 6,125,000
1/12/14 35%x20,000,000x 4/12 2,333,333.33
1/03/15 35%x20,000,000x 1/12 583,333.33
13,708,333.33
QUESTION 7
Kantanka Ltd is constructing a warehouse that will take about 18 months to complete. It began
construction on 1st January 2014. The following payments were made during 2014:
GH¢’000
31st January 200
31st March 450
30th June 100
31st October 200
30th November 250
The first payment on 31st January was funded from the entity‘s pool of debt. However, the
entity succeeded in raising a medium-term loan for an amount of GH¢800,000 on 31st March,
2014, with simple interest of 9 percent per annum, calculated and payable monthly in arrears.
These funds were specifically used for this construction. Excess funds were temporarily
invested at 6 percent per annum monthly in arrears and payable in cash. The pool of debt was
again used to an amount of GH¢200,000 for the payment on 30th November, which could not be
funded from the medium-term loan. The construction project was temporarily halted for 3
weeks in May when substantial technical and administrative work was carried out.
Kantanka Ltd adopted the accounting policy of capitalizing borrowing costs. The following
amounts of debt were outstanding at the balance sheet date, 31st December 2014:

116
GH¢’000
Medium-term loan (see description above) 800
Bank overdraft 1,200
(The weighted average amount outstanding during the year was GH¢750,000 and total interest
charged by the bank amounted to GH¢33,800 for the year)
A 10%, 7-year note dated 31st October 2018
with simple interest payable annually at 31st December 9,000
Required: Calculate the borrowing costs to be capitalized
Solution:
Specific Loan:
GHC
GH¢800,000 x 9 % x 9/12 = 54,000
Interest earned on unused portion of loan available during the year:
April 1 to June 30 [(800,000—450,000) x 3/12 x 6%] (5,250)
July 1 to October 31 [(800,000—550,000) x 4/12 x6%] (5,000)
1 November to November 30 [(800,000—750,000) x 1/12 x 6%] (250)
43,500
General pool of funds
GH¢
Capitalization rate is 9.58 % (w 1)
Paid on January 31 (200,000 x 11/12 x 9.58%) 17,563
Paid on November 30 (200,000 x 1/12 x 9.58%) 1,597
19,160
Total amount to be capitalized 62,660
Workings
1) Capitalization rate for a pool of debt
GH¢
Total interest payable on these borrowings:
Bank overdraft 33,800
7 year note (9,000,000 x 10%) 900,000
933,800
Weighted average total borrowings
Bank overdraft 750,000
7 year note (9,000,000 x 10%) 9,000,000
9,750,000
Capitalization rate 933,800 ÷ 9,750,000 = 9.58%

QUESTION 8
IAS 23 ―Borrowing Costs‖ regulates the extent to which entities are allowed to capitalize
borrowing costs incurred on money borrowed to finance the acquisition of certain assets.
Biakoye Ltd is a retail supermarket chain which constructs its own malls.

On 1 January, 2010, it started the construction of a supermarket. It acquired 50-year leasehold


interest in the site for GHS3 million. The construction of the building cost GHS9 million and
fixtures and fittings cost GHS6 million.
Fixtures and fittings would have an estimated economic useful life of ten years. The
construction was completed on 30th September, 2010 and was put to use immediately. The

117
building is expected to have a useful life of 50 years. Biakoye Ltd borrowed GHS18 million on
1st January, 2010 to finance the project. The loan carried an interest rate of 10% p.a and was
repaid on 30th June, 2011.
Required:
i. State the conditions to be met for:

(a) Capitalization of borrowing costs to commence. (3 marks)


(b) Capitalization of borrowing costs to cease. (3 marks)

ii. Assuming that borrowing costs are capitalized where appropriate, calculate

(a) the carrying amount to be included in non-current assets in respect of the Shopping mall at
31st December 2010. (3 marks)
(b) the total amount to be charged to the income statement in respect of the interest expense and
depreciation for the year to 31st December, 2010. (3 marks)

Total 12 marks
ICAG November,2011
Solution:
(a) Commencement of capitalisation
The capitalisation of borrowing costs as part of the cost of a qualifying asset shall commence
when:
(i) expenditures for the asset are being incurred;
(ii) borrowing costs are being incurred; and
(iii) activities that are necessary to prepare the asset for its intended use or sale are in progress.
(b)Cessation of capitalisation
Capitalisation of borrowing costs shall cease when substantially all the activities necessary to
prepare the qualifying asset for its intended use or sale are complete.
An asset is normally ready for its intended use or sale when the physical construction of the
asset is complete even though routine administrative work might still continue. If minor
modifications, such as the decoration of a property to the purchaser‘s or user‘s specification, are
all that are outstanding, this indicates that substantially all the activities are complete.
When the construction of a qualifying asset is completed in parts and each part is capable of
being used while construction continues on other parts, capitalisation of borrowing costs shall
cease when substantially all the activities necessary to prepare that part for its intended use or
sale are completed.
ii. Income of financial position for the year ended 31 December 2010 (extract)
GHC000
Depreciation charge(10% of GHS18m x 3/12) 272,500
Interest expenses 450,000 ++
Statement of financial position for the year ended 31 December 2010 (extract)
GHC000
Shopping mall 19,350,000
Depreciation 272,550
19,077,450

118
Workings
Initial recognised cost GHC
Cost :
Leasehold land 3,000,000
Building 9,000,000
Fixtures and fittings 6,000,000
18,000,000
Capitalised borrowing cost:
10% of GHS18m x 9/12 1,350,000
19,350,000
Depreciation calculation for 2010
Land GHS3,000,000/50 years 60,000
Building (9,810,000/50 years x 3/12)+ 49,050
Fixtures and fittings 6,450,000 x 3/12+ 163,500
272,550

Notes:
+ The capitalised borrowing cost is apportioned between the building and the furniture and fittings for
the purpose of determining the depreciation value of the different components of the complex item of PPE.
++ The interest expense relating to 1 October 2010 to 31 December 2010, which cannot be capitalised
(because the mall was completed and available for use) has to be expensed.

QUESTION 9
Fenerbache, a public limited company, is a sports organisation which owns several football and
basketball teams. It has a financial year end of 31 May 2016. Fenerbache needs a new stadium to
host sporting events which will be included as part of Fenerbache‘s property, plant and
equipment. Fenerbache therefore commenced construction on a new stadium on 1 February
2016, and this continued until its completion which was after the year end of 31 May 2016. The
direct costs were GHC20 million in February 2016 and then GHC50 million in each month until
the year end. Fenerbache has not taken out any specific borrowings to finance the construction
of the stadium, but it has incurred finance costs on its general borrowings during the period,
which could have been avoided if the stadium had not been constructed. Fenerbache has
calculated that the weighted average cost of borrowings for the period 1 February–31 May 2016
on an annualised basis amounted to 9% per annum.
Required:
Fenerbache needs advice on how to treat the borrowing costs in its financial statements for the
year ending 31 May 2016. (6 marks)
Solution:
The cost of an item of property, plant and equipment may include borrowing costs incurred for
the purpose of acquiring or constructing it. IAS 23 Borrowing Costs requires such borrowing
costs to be capitalised if the asset takes a substantial period of time to be prepared for its
intended use or sale. The definition of borrowing costs includes interest expense calculated by
the effective interest method, finance charges on finance leases and exchange differences arising
from foreign currency borrowings relating to interest costs. Borrowing costs should be
capitalised during construction and include the costs of funds borrowed for the purpose of
financing the construction of the asset, and general borrowings which would have been avoided

119
if the expenditure on the asset had occurred. The general borrowing costs are determined by
applying a capitalisation rate to the expenditure on that asset. The capitalisation rate will be the
weighted average of the borrowing costs applicable to the general pool.

The weighted-average carrying amount of the stadium during the period is


GHC(20 + 70 + 120 + 170) million/4, that is GHC95 million.

The capitalisation rate of the borrowings of Fenerbache during the period of construction is 9%
per annum, therefore the total amount of borrowing costs to be capitalised is the weighted-
average carrying amount of the stadium multiplied by the capitalisation rate.
That is (GHC95 million x 9% x 4/12) GHC2·85 million
.
QUESTION 10
b) Dompoase Ltd incurred the following borrowing costs during the financial year 2018:
GH¢'000
Overdraft interest 12
Foreign currency loan interest (correctly translated into GH¢) 84
Foreign currency loan exchange differences on capital 140
In addition a three-year fixed rate GH¢2 million loan was taken out on 1 January 2018 at 6.5%. A
loan set-up fee was charged at GH¢20,000. This increased the effective interest rate on the loan
to 6.88%.
Required:
Determine the maximum amount that could potentially be capitalised as borrowing costs
during the period (assuming an asset was being financed using all available finance). (5 marks)
SOLUTION 10
Maximum amount to capitalise (IAS 23 para 6):
GH¢‘000
Overdraft 12
Foreign currency loan interest 84
Foreign currency loan exchange differences on capital -
Effective interest on loan ((2,000 – 20) x 6.88%) 136.2
232.2

QUESTION 11
On 1 April 2018, Chofi Ltd (Chofi) borrowed GH¢15 million in order to partly fund the
construction of a new building. The interest rate was 6% payable annually in arrears. On 1 July
2018, construction commenced. On 1 October 2018, GH¢10 million was paid to the contractor as
the first stage payment. On 1 December 2018, a further GH¢10 million was paid to the
contractor. Construction was still in progress at the reporting date of 31 March 2019.
Required:
In accordance with IAS 23: Borrowing Costs, show the appropriate calculations, the amount that
should be capitalised in the financial statements of Chofi for the year end 31 March 2019.
(4 marks)

120
SOLUTION
IAS 23 requires that three conditions be met before capitalization of finance costs takes place.
These are

On this basis all three are met only on the payment of the first GH¢10 million to the contractor
on 1 October 2018. The finance cost on this amount from 1 October 2018 to 31 March 2019
should be capitalised (GH¢10m * 6% * 6/12), amounting to GH¢300,000.
(2 marks)
On 1 December 2018 a further GH¢10 million was paid to the contractor. However only GH¢5
million of this was borrowed. Hence (GH¢5m * 6% * 4/12) GH¢100,000 should be capitalised.
(2 marks)
Determination of finance costs to be capitalized for 6 months from 01/10/2018 to 31/03/2019
Determination of further finance costs to be capitalized for 4 months from 01/12/2018 to
31/03/2019.

SHARE BASED PAYMENTS


What you need to know •
IFRS 2 Share-based Payment requires an entity to measure and recognise share-based payment
awards – to employees or other parties - in its financial statements.
• IFRS 2 sets out measurement principles and specific requirements for three types of share-
based payment transactions: equity-settled, cash-settled, and when there is a choice of either cash
or equity-settled.

121
• Share-based payment awards are measured at the fair value of the goods and services received.
Where the fair value1 of goods and services cannot be measured reliably, the share based
payment is measured by reference to the fair value of the equity instruments granted.
• A share-based payment award generally vests upon meeting specified conditions, which can
either be service conditions or performance conditions.
• Awards are expensed as vesting conditions, if any, are satisfied.

Overview and background


Share-based payment awards (such as share options and shares) are common features of
employee remuneration for directors, senior executives and other employees. Some entities also
issue shares or share options to pay suppliers, such as providers of professional services. Prior to
the issuance of IFRS 2 Share-based Payment (IFRS 2 or the standard), there was no IFRS
covering the recognition and measurement of these types of transactions. This became a key
issue for executives, entrepreneurs, employees and directors, given the increasing prevalence of
share-based payment awards in many countries. As share-based payment awards became a larger
component of employee and executive compensation (e.g., in the Silicon Valley technology
companies in the late 1990s), standard-setters came to believe that share-based payment awards
are an integral component of a total compensation package. As such, they concluded that an
entity should recognise an expense for share-based payments, just as it does for cash
compensation. IFRS 2 was issued in February 2004 and prescribes the measurement and
recognition principles for all share-based payment awards within scope of the standard. IFRS 2
applies to share-based payment transactions with employees and third parties, whether settled in
cash, equity instruments or other less common assets (e.g., gold). The standard has been
amended several times since it was issued. Most recently, it was amended as part of the Annual
Improvements to IFRSs 2010-2012 Cycle with respect to vesting conditions; these amendments
became effective on 1 July 2014. The application of IFRS 2 involves difficult classification and
complex valuation issues and, as described below, is sometimes counter-intuitive. The general
principle of IFRS 2 is that an entity recognises an expense or asset for goods or services, with the
credit entry recognised either in equity or as a liability (depending on how the share-based
payment award is required to be settled). The definitions of „equity‟ and „liability‟ in IFRS 2 are
different from those used in IAS 32 Financial Instruments: Presentation and IAS 39Financial
Instruments: Recognition and Measurement (or IFRS 9Financial Instruments). IFRS 2 also uses
the term „fair value‟ in a way that differs in some respects from the definition of fair value in
IFRS 13 Fair Value Measurement. Therefore, in accounting for share-based payment
transactions an entity measures fair value in accordance with IFRS 2, not IFRS 13. After much
debate, the IASB settled on a grant date model to measure share-based payment awards to
employees (see sections 4 and 6). Under the grant date model, an entity measures the fair value
of a share-based payment award issued to an employee on the grant date. For equity-settled
share-based payments, the entity does not adjust the grant date fair value afterwards (even if it
becomes more or less valuable or does not ultimately vest), unless the award is modified.
Frequently, this results in an entity recognising an expense even if the employee receives no
monetary benefit from the award. Although this situation seems counter-intuitive, we believe this
model is preferable to entities not recognising any expense for share-based payment transactions.

Scope of IFRS 2
IFRS 2 encompasses three types of transactions:

122
• Equity-settled share-based payment transactions in which the entity receives goods or services
as consideration for its own equity instruments or those of another entity in the same group or a
shareholder of any group entity

• Cash-settled share-based payment transactions, also referred to as „liability awards‟, in which


the entity receives goods or services and incurs a liability based on the value of the entity‟s
shares or other equity instruments of the entity or another group entity (e.g., the grant of share
appreciation rights to employees, which entitle the employees to future cash payments based on
the increase in the entity‟s share price)

• Share-based payment transactions with cash alternatives in which the entity receives goods or
services and either the entity (or another group entity) or the supplier of the goods or services
(the counterparty) has a choice of the entity settling the transaction in cash, other assets, or by
issuing equity instruments IFRS 2 provides requirements on group share-based payment plans,
which is discussed further in see section 9. „Group‟ is defined in IFRS 2 as a parent and its
subsidiaries from the perspective of the reporting entity‟s ultimate parent.
Goods and services referred to above can be received from external suppliers or employees. For
example, if an external supplier of goods or services is paid in shares, share options or cash
based on the price (or value) of shares or other equity instruments of the entity, IFRS 2 must be
applied. Goods do not include financial assets, but do include inventories, consumables,
property, plant and equipment, intangibles, and other non-financial assets. Likewise, an
employee may receive equity instruments as remuneration for services rendered.
Even if an entity cannot specifically determine the goods or services it receives in return for its
shares, it must apply IFRS 2. For example, if an entity grants shares to a charity for no
identifiable benefit, that transaction is within the scope of IFRS 2.

IFRS 2 does not cover the following transactions:


• Transactions with shareholders that are acting solely in their capacity as shareholders
• Goods and services received by the entity that are settled by entities or shareholders not within
the group
• Transactions within the scope of IAS 32 and IAS 39 (or IFRS 9)
• Share-based payment transactions to acquire goods as part of a business combination to which
IFRS 3Business Combinations applies, in a combination of entities or businesses under common
control, or the contribution of a business on the formation of a joint venture, as defined by IFRS
11 Joint Arrangements
• Transfers of assets in certain group restructuring arrangements Awards granted to employees
of an acquiree in their capacity as employees (e.g., in return for continued service) are within the
scope of IFRS 2, as are the cancellation, replacement and modification of share-based payment
awards as a result of a business combination or other equity restructuring.

Basic principles
When an entity enters into a share-based payment arrangement, it needs to determine:
1. The classification of the share-based payment i.e. whether it is equity-settled or cash-settled
2. The grant date
3. Vesting conditions, if any, and whether they are market or non-market related
4. The period over which the award vests

123
5. The fair value at grant date

At each subsequent reporting date until vesting, the entity calculates a best estimate of the
cumulative charge to profit or loss at that date, being the product of:
• The grant date fair value of the award
• The current best estimate of the number of awards that will vest
• The expired portion of the vesting period
The charge (or credit) to profit or loss for the period is the cumulative amount calculated above
less the amounts already charged in previous periods.
• An expense (or an asset if the goods and/or services received meet the criteria for recognising
an asset)
• A corresponding increase in equity (for transactions settled in equity instruments) or in
liabilities (for cash-settled transactions)

IFRS 2 distinguishes between the accounting treatment for share-based payment transactions of
equity-settled versus cash-settled. A transaction is treated as equity-settled when an entity
receives goods or services as consideration for its own equity instruments (including shares or
share options), or it receives goods or services but has no obligation to settle the transaction with
the supplier. A transaction is considered to be cash-settled when the entity acquires goods or
services by incurring a liability to transfer cash or other assets to the supplier of those goods or
services for amounts that are based on the price (or value) of equity instruments (including
shares or share options) of the entity or another group entity.

For a share-based payment transaction in which the terms of the arrangement provide an entity
with the choice of whether to settle in cash or by issuing equity instruments, the entity must
determine whether it has a present obligation to settle in cash and account for the share-based
payment transaction accordingly. The entity has a present obligation to settle in cash if the choice
of settlement in equity instruments has no commercial substance (e.g. because the entity is
legally prohibited from issuing shares), or the entity has a past practice or a stated policy of
settling in cash, or generally settles in cash whenever the counterparty asks for cash settlement. If
no such obligation exists, the entity must account for the transaction in accordance with the
requirements that apply to equity-settled share-based payment transactions.
It is important that the share-based payment transaction is correctly classified as this has a
consequential impact on the recognition and measurement. How and when the entity measures
the award, and whether the entity must remeasure the award, all depend on whether the award is
equity-settled, cash-settled, or there is a choice of settlement, and we explore this further in
sections 4 and 5 of this publication.
Equity-settled awards
Measurement principle
For equity-settled awards (such as share options), the general principle in IFRS 2 is that an entity
measures the fair value of goods obtained or services received, and recognises a corresponding
increase in equity. But, if an entity cannot reliably estimate the fair value of the goods obtained
or services received, it must measure their value indirectly using the fair value of the equity
instruments granted.
However, IFRS 2 requires that:

124
• For awards to employees, an entity must use the fair value of the equity instruments, measured
at the grant date
• For awards tonon-employees, there is a rebuttable presumption that the fair value of the goods
or services is more reliably determinable, which is measured when the goods or services are
received or obtained.
The IASB‟s decision to require entities to measure the equity instruments issued to employees
based on their fair value is practical rather than theoretical, in that entities might have difficulty
establishing which services relate to which component of an employee‟s compensation package.
Furthermore, if a share-based payment award serves as a bonus, the entity pays additional
compensation to receive additional services, but it may be difficult to determine the value of such
services.

As there are often no quoted market prices for share-based payment awards, IFRS 2 requires
entities to estimate the grant date fair value of their share-based payment awards using option-
pricing models, which we will discuss in more detail in section 4.7.
Diagram 1 illustrates the measurement principle for equity-settled awards:

Diagram 1: Measurement of share-based payments


Counterparty Measurement basis Measurement date Recognition date

Employee Fair value of equity Grant date Date services received


instruments awarded

Non-employee Fair value of goods or Date goods or Date goods or


services received services received services received

Determination of grant date


The determination of the grant date is critical to the measurement of equity-settled share-based
payment transactions with employees, since the grant date is the date at which the entity
measures such transactions. The grant date is defined as the date on which the reporting entity
and the employee have a shared understanding of the terms of the arrangement, based on a
legally binding agreement. In practice, the following issues need to be considered:
• How precise the shared understanding of the terms of the award must be
• What level of communication between the reporting entity and the counterparty is sufficient to
ensure the appropriate degree of shared understanding
The determination of the grant date is often difficult in practice. Entities need to consider the
following circumstances when determining the grant date:
• Services are rendered in advance of grant date or before any communication of awards to
employees
• Exercise price depends on a formula or a future share price
• Exercise price is paid in shares
• A fixed-monetary-amount award of equity instruments
• Awards over a fixed pool of shares (including 'last man standing' arrangements)
• Awards with multiple service periods
• Modification or discretionary re-assessment by the entity after the original grant date

125
• Mandatory or discretionary awards to 'good leavers'
• Special purpose acquisition companies (which are typically established for the purposes of
flotation or sale)

The implementation guidance for IFRS 2 indicates that the grant date occurs when there is both a
mutual understanding of the terms and a legally enforceable arrangement. Thus, if an award
requires board or shareholder approval to be legally binding on the reporting entity, under IFRS
2 the grant date is not until such approval has been given, even if the terms of the award are fully
understood at an earlier date. However, if the employee is rendering services for the award
beginning on a date earlier than the grant date, the entity estimates the cost of the award and
recognises such cost over a period starting with that earlier date. The entity adjusts the fair value
estimate to the grant date when approval is given.

How we see it
The determination of whether there is a shared understanding may require the exercise of
significant judgement. This may be the case, for example, when: the formula for determining the
number of awards to employees is not clearly defined; final substantive approvals are required;
or the number of shares ultimately received will not be known until the vesting date. It is
important for entities to evaluate all terms of the award and the specific facts and circumstances
in making the assessment as to whether there is a shared understanding between the entity and
counterparty.

Vesting conditions
Under IFRS 2, the point at which a cost is recognised for goods or services depends on the
vesting conditions. A vesting condition determines whether an entity receives the services that
entitle the counterparty to receive the share-based payment award. A share-based payment award
generally vests upon meeting specified conditions. Vesting conditions are either:
• Service conditions, which require the counterparty to complete a specified period of service
during which the services are provided to the entity
Or
• Performance conditions, which require the counterparty to complete a specified period of
service (i.e., a service condition)and involves specified performance targets to be met while the
counterparty is rendering the required service

A performance target can be defined by reference to the entity‟s own operations or activities,
such as achieving a specified EBITDA target or the price (or value) of its equity instruments. A
performance target can relate either to the performance of the entity as a whole, or to some part
of the entity (i.e., a division) or an individual employee. If the period of achieving the specific
target extends beyond the employee‟s service period, the performance target is a non-vesting
condition (discussed below).
A performance condition is further defined as either a market condition or a non-market
condition.
A market condition is a performance condition (i.e., requires specified targets to be met) and the
performance conditions are related to the market price (or value) of the entity‟s equity
instruments, such as: attaining a specified share price or achieving a specified target that is based

126
on the market price (or value) of the entity‟s equity instruments relative to an index of market
prices of equity instruments of other entities.
A condition linked to a purely internal financial performance measure, such as profit or earnings
per share, is not a market condition. Such measures will affect the share price, but are not
directly linked to it, and hence are not market conditions.
In order for a market condition to be treated as a performance vesting condition rather than a
non-vesting condition, there must also be an implicit or explicit service condition.

Non-vesting conditions
IFRS 2 does not specifically define a non-vesting condition, but uses the term to describe a
condition that is neither a service condition nor a performance condition. A performance
condition is distinguished from a non-vesting condition in that it has an explicit or implicit
service requirement whereas a non-vesting condition does not. This means that, if an employee is
entitled to an award on the grant date and is not required to provide any future services to the
entity, such a condition is not regarded as a vesting condition for the purpose of IFRS 2. Instead,
it is referred to as a non-vesting condition. Examples of non-vesting conditions include a non-
compete clause, a target based on a commodity index or the employees paying contributions
towards the exercise price of a share-based payment award.

Impact of conditions on measuring share-based payments


Under IFRS 2, the nature of the condition (i.e., vesting or non-vesting, service, performance,
market or non-market) affects the timing of when the expense is recognised and, in some cases,
the measurement of the expense. In addition, if a condition is not met, whether or not the entity
may reverse the previously recognised compensation expense depends on the nature of the
condition that was not met. Therefore, the classification of a condition is a critical step in
accounting for share-based payments transactions.
Market conditions are only taken into account when estimating the fair value of the award at the
grant date.
Non-market vesting conditions are not taken into account when estimating the fair value of the
shares or share options at the grant date. Instead, these vesting conditions are taken into account
by adjusting the number of equity instruments included in the measurement of the transaction
amount so as to reflect the number of awards that are expected to vest. Such non-market vesting
conditions include a service condition.
Performance conditions can either be market conditions or non-market conditions.
Accounting for share-based payments with conditions

Illustration 1a: Award with only service conditions


An entity grants 100 share options to each of its 500 employees. Each grant is conditional upon
the employee working for the entity over the next three years. The entity estimates that the fair
value of each share option is GH¢15. The entity estimates that 20% of the employees will leave
during the three-year period and, therefore, forfeit their rights to the share options. Application of
the requirements:
Yr Calculation Expense for period Cumulative expense
1 50,000 options x 80% x GH¢15 x 1/3 years 200,000 200,000
2 (50,000 options x 80% x GH¢15 x 2/3 years) – GH¢200,000 200,000 400,000
3 (50,000 options x 80% x GH¢15 x 3/3 years) – GH¢400,000 200,000 600,000

127
Under IFRS 2, an entity only recognises compensation expense for options with non-market
performance conditions if such awards ultimately vest. Therefore, if an entity grants options to a
large number of employees on one grant date, the entity would need to estimate the number of
employees that will terminate employment prior to meeting the non-market performance
conditions, i.e., the number of employees that will forfeit awards. The entity adjusts its estimate
of awards that will vest at each reporting date so that, on the vesting date, the expense recognised
equals the grant date fair value of the options that have vested. This is shown in Illustration 1b.

Illustration 1b: Award with only service conditions – change in estimates


Following from illustration 1a: In year 1, 20 employees leave, The entity revises its estimate of
total employee departures over the three-year period from 20% (100 employees) to 15% (75
employees). In year 2, a further 22 employees leave. The entity revises its estimate of total
employees over the three-year period from 15% to 12 % (60 employees). In year 3, a further 15
employees leave. Therefore, a total of 57 employees forfeited their rights to the share options
during the three-year period and a total of 44,300 share options (443 employees x 100 options
each) vested at the end of year 3.

Yr Calculation Expense for period Cumulative expense


GH¢ GH¢
1 50,000 options x 85% x GH¢15 x 1/3 years 212,500 200,000
2 (50,000 options x 88% x GH¢15 x 2/3 years)– GH¢212,500 227,500 440,000
3 (44,300 options x GH¢15)– GH¢440,000 224,500 664,500

Illustration 2: Award with market conditions


An entity grants 10,000 share options to a director on the condition that the director remaines in
employment for three years and the market price of the related shares increased from GH¢50 at
the beginning of year 1 to above GH¢65 at the end of year 3. The entity determines the fair value
at grant that takes into account the possibility that the share price will exceed GH¢65 at the end
of year 3 (and therefore become exercisable) and the possibility that the share price will not
exceed GH¢65 at the end of three years (and hence the options will be forfeited). It estimates the
fair value of the share options with this market condition to be GH¢24 per option.
The possibility that the share price target might not be achieved is already taken into account
when estimating the fair value of the options at grant date. Therefore, if the entity expects the
director to complete the three-year service period and the director does so, the entity recognises
the following amounts in years 1, 2 and 3:

Yr Calculation Expense for period Cumulative expense


GH¢ GH¢
1 10,000 options x GH¢24 x 1/3 years 80,000 80,000
2 (10,000 options x GH¢24 x 2/3 years) – CU80,000 80,000 160,000
3 (10,000 options x GH¢24 x 3/3 years) – CU160,000 80,000 240,000

These amounts are recognised irrespective of the outcome of the market condition. However, if
the director leaves during year 2 (or year 3), the amount recognised during year 1 (and year 2)
will be reversed in year 2 (or year 3). This is because the service condition, in contrast to the

128
market condition, was not taken into account when estimating the fair value of the share options
at grant date. Instead, the service condition is taken into account by adjusting the transaction
amount to be based on the number of equity instruments that ultimately vest.

Illustration 3
on 1 January 2014 an entity grants 100 share options to each of its 500 employees. Each grant is
conditional upon the employee working for the entity until 31 December 2016. At the grant date
the fair value of each share option is GH¢15.
during 2014 20 employees leave and the entity estimates that a total of 20% of the 500
employees will leave during the three- year period. During 2015, a further 20 employees leave
and the entity now estimates that only a total of 15% of its 500 employees will leave during the
three-year period. During 2016, a further 10 employees leave.
Required :
Calculate the remuneration expense that will be recognised in respect of the share-based payment
transaction for each of the three years ended 31 December 2016.

Solutions
Yr Calculation Expense for period Cumulative expense
GH¢ GH¢
2014 (100 x 500 x 80% x 15 x 1/3years) 200,000 200,000
2015 (100 x 500 x 85% x 15 x 2/3years) - 200,000 225,000 425,000
2016 (45000 x 15) - 425,000 250,000 675,000

The Financial statements will include the following amounts:

Income Statement 2014 2015 2016


GH¢ GH¢ GH¢
Staff Costs 20,000 225,000 250,000

Statement of Financial Position 2014 2015 2016


GH¢ GH¢ GH¢
included with equity 200,000 425,000 675,000

Illustration 4
On the 1 January 2014, 400 staff receive 100 share options each. They must work for the
company for the next three years and the options become exercisable on 31 December 2016. The
fair value at the time of granting is GH¢20 per option.
In the year ending 31 December 2014, 10 staff leave and it is thought that during the three-year
vesting period, the total amount leaving will be 15%.
In 2015, a further 15 leave but the estimate of total leaving is now reduced to 10%. In the final
year 12 staff leave.
Required:
Show how this will impact on the financial statements of the years 2014, 2015 and 2016.

129
2014 2015 2016
GH¢ GH¢ GH¢
share options 40,000 40,000 40,000
Expected to vest 85% 90% (3,700)
34,000 36,000 36,300

Fair value at grant date GH¢20 GH¢20 GH¢20


Total cost GH¢680,000 GH¢720,000 GH¢725,000

Proportion of vesting period passed 1/3 2/3 3/3


Equity GH¢226,667 GH¢480,000 GH¢726,000

Cost charged to statement of P&L (diff in equity) GH¢226,667 GH¢253,333 GH¢246,000

Illustration 5
Adu has set up an employee option scheme to motivate its sales team often key sales people.
Each sales persons was offered 1 million options, conditional upon the employee remaining with
the entity during the vesting period of 5years. The fair value of each option at the grant date was
GH¢20.
At the of year one, two sales people were expected to leave over the vesting period.
one of these two people left in year two, however the other recommitted to the entity and
therefore, at the end of year two, the entity expected the remaining nine sales people to remain
with the entity for the rest of the vesting period.
Required:
Calculate the charge to Adu statement of profit and loss for the year two in respect of the share
options and prepare the journal entry to record this.

Answers
At the end of year two the amount recognised in equity should be GH¢720,000
(1m x 9 x 20 x 2/5)
At the end of year one the amount recognised equity would have been GH¢320,000
(1m x 20 x 1/5)
the charge to the statement of profit and loss for the year two is therefore
GH¢400,000 (720,000 - 320,000)
The journal entry to record this is:
Dr P & L- Staff costs GH¢400,000
Cr Other reserves (equity) GH¢400,000

Illustration 6
JP granted share options to its 300 employees on 1 January 2013. Each employee will receive
1,000share options provided they continue to work for JP for 3 years from the grant date. The
fair value of each option at the grant date was GH¢1.22
The actual and expected staff movement over the 3years to 31 December 2015 is provided
below:
In 2013 25 employees left and another 40 were expected to leave over the next two years.

130
In 2014 a further 15 employees left and another 20 were expected to leave the following year/
Required:
Calculate the charge to JP statement of profit and loss for the year ended 31 December 2013 in
respect of the share options and prepare the journal entry to record this.

Answer
2013
(300 - 25 - 40 ) x 1,000 x GH¢1.22 = GH¢286,700 over 3years GH¢95,567 charge for 2013

2014
(300 - 25 - 20 ) x 1,000 x GH¢1.22 = GH¢292,800 x 2/3 = GH¢195,200 recognizable to date

Less amount recognised in 2013 = GH¢195,200 - 95,567 = GH¢99,633 charge for 2014

charge for 2014 of GH¢99,633 will be recorded as


Dr P&L - staff costs GH¢99,633
Cr Other reserves (equity) GH¢99,633
Being the charge for share-based payment for the year ended 31 December 2014

Cash-settled awards
For cash-settled awards (such as a share appreciation right), the general principle in IFRS 2 is
that an entity measures the fair value of the goods or services received based on the fair value of
the liability. However, unlike the grant date model for equity-settled awards for employees, an
entity remeasures the fair value of the award at each reporting date and on settlement. The
ultimate cost of a cash-settled award is the cash paid to the counterparty, which is the fair value
at settlement date. Until the award is settled, an entity presents the cash-settled award as a
liability and not within equity. Thus, changes in the measurement of the liability are reflected in
the statement of profit or loss and other comprehensive income. IFRS 2 does not specifically
address the impact of vesting conditions within the context of cash-settled share-based payment
transactions. However, at its November 2013 meeting, the IFRS Interpretations Committee
discussed proposed amendments to IFRS 2 and recommended that the IASB amend IFRS 2 to
make clear that:
• The effect of a market condition or a non-vesting condition must be reflected in the estimation
of the fair value of the cash-settled share-based payments both at the grant date and
subsequently.

• Vesting conditions (other than market conditions) must not be taken into account when
estimating the fair value of cash-settled share-based payments. Instead, vesting conditions (other
than market conditions) must be taken into account in the measurement of the liability incurred
by adjusting the number of awards that are expected to vest. Such an estimate must be revised
when the liability is remeasured at each reporting date and until the vesting date.

• On a cumulative basis, no amount is recognised for goods or services received if the awards
granted do not vest because of failure to satisfy a vesting condition or a non-vesting condition.

131
These amendments are included as part of the collection of narrow-scope amendments to IFRS 2
and an exposure draft was released on 11 November 2014, with a comment period of 120 days.

For cash-settled share-based payments, the entity recognises the services received and the
liability for those services as the employees render them. If an employee is not required to
provide a service, as is the case for some share appreciation rights, the entity recognises the
expense and liability immediately upon grant date. If the employee is required to provide
services over for a specified period in order to vest in the cash-settled award, the entity
recognises the expense and the liability over the vesting period, while reconsidering the
likelihood of achieving vesting conditions and remeasuring the fair value of the liability at the
end of each reporting period.

Illustration 1: Award that is cash-settled


An entity grants 100 cash-settled awards to each of its 500 employees on the condition that the
employees remain in its employment for the next three years. Cash is payable at the end of three
years based on the share price of the entity‟s shares on such date.
During year 1, 35 employees leave. The entity estimates that 60 additional employees will leave
during years 2 and 3 (i.e., the award will vest for 405 employees). The share price at year-end is
GH¢14.40.
During year 2, 40 employees leave and the entity estimates that 25 additional employees will
leave during year 3 (i.e., the award will vest for 400 employees). The share price at year-end is
GH¢15.50.
During year 3, 22 employees leave, so that the award vests for 403 employees. The share price at
year-end is GH¢18.20.

Answer
The entity recognises the cost of this award, as follows:

Yr Calculation Cumulative expense Expense for period


1 405 employees x 100 awards x GH¢14.40 x 1/3 194,400 194,400
2 400 employees x 100 awards x GH¢15.50 x 2/3 413,333 218,933
3 403 employees x 100 awards x GH¢18.20 x 3/3 733,460 320,127

Illustration 2
On 1 January 2011 an entity grants 100 cast share appreciation rights (SAR) to each of its 300
employees, on condition that they continue to work for the entity until 31December 2013.

During 2011 20 employees leave. The entity estimates that a further 40 will leave during 2012
and 2013.
During 2012 10 employees leave. The entity estimates that a further 20 will leave during 20 2013
During 2013 10 employees leave.
the fair value of one SAR at each year end are shown below.
Fair value
GH¢
2011 10.00

132
2012 12.00
2013 15.00

Required:
Calculate the amount to be recognised as an expenses for each of the three years ended 31
December 2013 and the liability to be recognised in the statement of financial position at 31
December for each of the three years.

Answer
Yr Calculation Liability at year-end Expense for period
GH¢ GH¢
2011 ((300 - 20 - 40) x 100 x GH¢10 x 1/3) 80 80
2012 ((300 - 20 - 10 - 20) x 100 x GH¢12 x 2/3) 200 120
2013 ((300 - 20 - 10 - 10) x 100 x GH¢15) 390 190

Illustration 3
On 1 January 2011 kindly sets up a cash based payment to each of its 100 employees, on
condition that they continue to work for the entity until 31 December 2013. Each employee has
been allocated 100 shares and will receive a payment in cash if the share price exceeds GH¢10
on 31 December 2013. of the amount by which it exceeds GH¢10.
During 2011, 5 employees leave. The entity estimates that a further 15 will leave during 2013.
During 2012, 10 employees leave. The entity estimates that a further 12 will leave during 2013.
During 2013, 18 employees leave.
The share prices at the reporting date in each year are shown below
Fair value
GH¢
2011 11.00
2012 12.00
2013 14.00

Required
Calculate the amount to be recognised as an expense for the two years ended 31 December 2013
and the liability to be recognised in the statement of financial position at 31 December for both
years.

Answer

Yr Calculation Liability at year-end Expense for period


GH¢ GH¢
2011 ((100 - 5 - 12) x 100 x (11 - 10) x 1/3) 2,767 2,767
2012 ((100 - 5 - 10 - 15) x 100 x (12 - 10) x 2/3) 9,333 9,566

133
2013 ((100 - 5 - 10 - 18) x 100 x (14 - 10) 26,800 17,467

Illustration 4
Power grants 100 share appreciation rights (SARs) to its 500 employees on 1 January 2013 on
the condition that the employees stay with the entity for the next two years. The SARs must be
exercised at the start of 2015.
During 2013 `15 staff leave and another 20 are expected to leave in 2014.
During 2014 25 staff leave
The fair value of the SAR is GH¢10 at 31 December 2013 and GH¢13 at 31 December 2014.
Required:
calculate the amount to be recognised as an expense for the two years ended 31 December 2013
and 2014 and the liability to be recognised in the statement of financial position at 31 December
for both years.

Answer

Yr Calculation Liability at year-end Expense for period


GH¢ GH¢
2011 ((100 - 15 - 20) x 100 x 10 x 1/2) 232,500 232,500
2012 ((100 - 15 - 10 - 25) x 100 x 13 598,000 365,500

Illustration 5
Abi's granted 1,000 share appreciation rights (SARs) to its 300 employees on 1 January 2009. To
be eligible, employees must remain employed for 3years from the date of issue and the rights
must be exercised in January 2012, with settlement due in cash.

In the year to 31 December 2009, 32 staff left and a further 35 were expected to leave over the
following two years.
In the year to 31December 2010, 28 staff left and a further 10 were expected to leave in the
following year.
No actual figures are available as yet for 2011.
The fair value of each SAR was GH¢8 at 31 December 2009 and GH¢12 at 31 December 2010.
Required:
prepare the accounting entry to record the expense associated with the SARs for the year to 31
December 2010 in accordance with IFRS 2 Share - based payments

Answer
2009
Eligible employees (300 - 32 - 35 ) =233
Equivalent cost of SARs = 233 employees x 1000 rights x FV GH¢8 = GH¢1,864,000

Allocate over 3 years vesting period GH¢1,864,000/3= GH¢621,333 equivalent charge to the
statement of profit and loss in the first year.

2010
Eligible employees (300 - 32 - 28 - 10 ) = 230

134
Equivalent cost of SARs = 230 employees x 1000 rights x FV GH¢12 = GH¢2,760,000
Cumulative amount to be recognised as a liability = GH¢2,760,000 x 2/3 years = GH¢1,840,000
less amount previously recognised = GH¢1,840,000 - GH¢621,333 = GH¢1,218,667

The expense will be recorded as:


Dr P&L - staff cost GH¢1,218,667
Cr Liability GH¢1,218,667
. Disclosures
Among other things, IFRS 2 requires entities to disclose the following:
• The type and scope of agreements existing during the reporting period
• Descriptions of each type of arrangement, including general terms and conditions of the
arrangement (e.g., settlement methods, vesting conditions)
• The number and weighted-average exercise price of share options (outstanding at the beginning
of the reporting period and at the end of the reporting period, granted, vested, exercised, expired
and forfeited during the period)
• The average share price of exercised options
• The range of exercise prices and weighted average remaining contractual life of options
outstanding at the end of the reporting period
• The valuation method used to estimate the fair value of the awards (model and input values,
etc.)
• The impact on the income statement (i.e., total expense) and the financial position (e.g.,
carrying amount of liabilities) of share-based payment awards

Illustration 5
On 1 October 2016, HO made an award of 4,000 share options to each of its seven directors. The
condition attached to the award was that the directors must remain employed by HO for three
years. The fair value of each option at the grant date was GH¢100 and the fair value of each
option at 30 September 2018 was GH¢110. At 30 September 2017, it was estimated that three
directors would leave before the end of three years. Due to an economic downturn, the estimate
of directors who were going to leave was revised to one director at 30 September 2018. The
expense for the year as regards the share options had not been included in profit or loss for the
current year and no director had left by 30 September 2018.
Required
Calculate the amount to be recognised as an expense for 30 September 2018 and amount to be
recognised in the statement of financial position at 30 September 2018.

RELATED PARTY TRANSACTIONS (IAS 24)

135
Objective
The objective of this Standard is to ensure that an entity‗s financial statements contain the
disclosures necessary to draw attention to the possibility that its financial position and profit or
loss may have been affected by the existence of related parties and by transactions and
outstanding balances, including commitments, with such parties.
Scope
This Standard shall be applied in:
(a) identifying related party relationships and transactions;
(b) identifying outstanding balances, including commitments, between an entity and its related
parties;
(c) identifying the circumstances in which disclosure of the items in (a) and (b) is required; and
(d) determining the disclosures to be made about those items.

Definitions
The following terms are used in this Standard with the meanings specified:
A related party is a person or entity that is related to the entity that is preparing its financial
statements (in this Standard referred to as the = reporting entity‗).
(a) A person or a close member of that person‗s family is related to a reporting entity if that
person:
(i) has control or joint control over the reporting entity;
(ii) has significant influence over the reporting entity; or
(iii) is a member of the key management personnel of the reporting entity or of a parent of the
reporting entity.
(b) An entity is related to a reporting entity if any of the following conditions applies:
(i) The entity and the reporting entity are members of the same group (which means that each
parent, subsidiary and fellow subsidiary is related to the others).
(ii) One entity is an associate or joint venture of the other entity (or an associate or joint venture
of a member of a group of which the other entity is a member).
(iii) Both entities are joint ventures of the same third party.
(iv) One entity is a joint venture of a third entity and the other entity is an associate of the third
entity.
(v) The entity is a post-employment benefit plan for the benefit of employees of either the
reporting entity or an entity related to the reporting entity. If the reporting entity is itself such a
plan, the sponsoring employers are also related to the reporting entity.
(vi) The entity is controlled or jointly controlled by a person identified in (a).
(vii) A person identified in (a)(i) has significant influence over the entity or is a member of the
key management personnel of the entity (or of a parent of the entity).

A related party transaction is a transfer of resources, services or obligations between a


reporting entity and a related party, regardless of whether a price is charged.

Close members of the family of a person are those family members who may be expected to
influence, or be influenced by, that person in their dealings with the entity and include:
(a) that person‗s children and spouse or domestic partner;
(b) children of that person‗s spouse or domestic partner; and
(c) dependants of that person or that person‗s spouse or domestic partner.

136
Compensation includes all employee benefits (as defined in IAS 19 Employee Benefits) including
employee benefits to which IFRS 2 Share-based Payment applies.
Employee benefits are all forms of consideration paid, payable or provided by the entity, or on
behalf of the entity, in exchange for services rendered to the entity.
It also includes such consideration paid on behalf of a parent of the entity in respect of the
entity.

Compensation includes:
(a) short-term employee benefits, such as wages, salaries and social security contributions, paid
annual leave and paid sick leave, profit-sharing and bonuses (if payable within twelve months
of the end of the period) and non-monetary benefits (such as medical care, housing, cars and
free or subsidised goods or services) for current employees;
(b) post-employment benefits such as pensions, other retirement benefits, post-employment life
insurance and post-employment medical care;
(c) other long-term employee benefits, including long-service leave or sabbatical leave, jubilee
or other long-service benefits, long-term disability benefits and, if they are not payable wholly
within twelve months after the end of the period, profit-sharing, bonuses and deferred
compensation;
(d) termination benefits; and
(e) share-based payment.

Control is the power to govern the financial and operating policies of an entity so as to obtain
benefits from its activities.

Joint control is the contractually agreed sharing of control over an economic activity.

Key management personnel are those persons having authority and responsibility for planning,
directing and controlling the activities of the entity, directly or indirectly, including any director
(whether executive or otherwise) of that entity.

Significant influence is the power to participate in the financial and operating policy decisions
of an entity, but is not control over those policies. Significant influence may be gained by share
ownership, statute or agreement.

Government refers to government, government agencies and similar bodies whether local,
national or international.

A government-related entity is an entity that is controlled, jointly controlled or significantly


influenced by a government.

Disclosure
Relationships between parents and subsidiaries. Regardless of whether there have been
transactions between a parent and a subsidiary, an entity must disclose the name of its parent
and, if different, the ultimate controlling party.
Management compensation. Disclose key management personnel compensation in total and
for each of the following categories:

137
 short-term employee benefits
 post-employment benefits
 other long-term benefits
 termination benefits
 share-based payment benefits

Key management personnel are those persons having authority and responsibility for planning,
directing, and controlling the activities of the entity, directly or indirectly, including any
directors (whether executive or otherwise) of the entity.

Related party transactions. If there have been transactions between related parties, disclose the
nature of the related party relationship as well as information about the transactions and
outstanding balances necessary for an understanding of the potential effect of the relationship
on the financial statements. These disclosures would be made separately for each category of
related parties and would include:
the amount of the transactions
the amount of outstanding balances, including terms and conditions and guarantees
provisions for doubtful debts related to the amount of outstanding balances
expense recognised during the period in respect of bad or doubtful debts due from related
parties

The disclosures shall be made separately for each of the following categories:
(a) the parent;
(b) entities with joint control or significant influence over the entity;
(c) subsidiaries;
(d) associates;
(e) joint ventures in which the entity is a venturer;
(f) key management personnel of the entity or its parent; and
(g) Other related parties.

Items of a similar nature may be disclosed in aggregate except when separate disclosure is
necessary for an understanding of the effects of related party transactions on the financial
statements of the entity
A reporting entity is exempt from the disclosure requirements in relation to related party
transactions and outstanding balances, including commitments, with:
(a) a government that has control, joint control or significant influence over the reporting entity;
and
(b) Another entity that is a related party because the same government has control, joint control
or significant influence over both the reporting entity and the other entity.

If a reporting entity applies the exemption, it shall disclose the following about the transactions
and related outstanding balances:
(a) the name of the government and the nature of its relationship with the reporting entity (i.e.
control, joint control or significant influence);
(b) the following information in sufficient detail to enable users of the entity‗s financial
statements to understand the effect of related party transactions on its financial statements:

138
(i) the nature and amount of each individually significant transaction; and
(ii) For other transactions that are collectively, but not individually, significant, a qualitative or
quantitative indication of their extent.

QUESTION 1
Mane is an entity specializing in importing a wide range of non-food items and selling them to
retailers. Aqeel is Mane‘s CEO and founder and owns 40% of Mane‘s equity shares:
i) Mane‘s largest customer, Zico accounts for 35% of Mane‘s revenue. Zico has just completed
negotiations with Mane for a special 5% discount on all sales.

ii) During the accounting period, Aqeel purchased a property from Mane for GH¢500,000.
Mane had previously declared the property surplus to its requirements and had valued it at
GH¢750,000.

iii) Aqeel‘s son, Sherif is a director in a financial institution, Cheap Capital. During the
accounting period, Cheap Capital advanced GH¢2 million to Mane as an unsecured loan at a
favourable rate of interest.

Required:
Explain, with reasons, the extent to which each of the above transactions should be classified
and disclosed in accordance with IAS 24 Related Party Disclosures in Mane‘s financial
statements for the period. (4 marks)
Solution:
According to IAS 24 Related Party Disclosures,a customer with whom an entity transacts a
significant volume of business is not a related party merely by virtue of the resulting economic
dependence.
Zico is not a related party and the negotiated discount does not need to be disclosed.
A party is related to an entity if it has an interest in the entity that gives it significant influence
over the entity. The party is related to an entity if they are a member of the key management
personnel of the entity. As founder member and major shareholder holding 40% of the equity,
Aqeel is able to exert significant influence and is a related party of Mane. Aqeel is also a related
party as he is Mane‘s president. He is a member of the key management personnel of Mane. The
sale of the property for GHȻ500,000 will need to be disclosed, along with its valuation as a
related party transaction.
Providers of finance are not related parties simply because of their normal dealings with the
entity. However, if a party is a close member of the family of any individual categorized as a
related party, they are also a related party. As Sherif is Aqeel‘s son and Aqeel is a related party,
Sherif is also a related party. The loan from Cheap Capital will need to be disclosed along with
the details of Sherif and his involvement in the arrangements.

QUESTION 2
Engina, a foreign company has approached a partner in your firm to assist in obtaining local
stock exchange listing (or stock market registration) for the company. Engina is registered in a

139
country where transactions between related parties are considered to be normal but where such
transactions are not disclosed.
The directors of Engina are reluctant to disclose the nature of their related party transactions as
they feel that although they are a normal feature of business in their part of the world, it could
cause significant problems politically and culturally to disclose such transactions.
The partner in your firm has requested a list of all transactions with parties connected with the
company and the directors of Engina have produced the following summary:

(a) Every month, Engina sells GHC50,000 of goods per month to Mr Satay, the financial director.
The financial director has set up a small retailing business for his son and the goods are
purchased at cost price for him. The annual turnover of Engina is GHC300 million.
Additionally, Mr Satay has purchased his company car from the company for GHC45,000
(market value GHC80,000). The director, Mr Satay, earns a salary of GHC500,000 a year, and
has a personal fortune of many millions of Cedis.

(b) A hotel property had been sold to a brother of Mr Soy, the Managing Director of Engina, for
GHC4 million (net of selling cost of GHC0.2 million). The market value of the property was
GHC4.3 million but prices have been falling rapidly. The carrying value of the hotel was GHC5
million and its value in use was GHC3.6 million. There was an over-supply of hotel
accommodation due to government subsidies in an attempt to encourage hotel development
and the tourist industry.

(c) Mr Satay owns several companies and the structure of the group is outlined below. Engina
earns 60% of its profits from transactions with Car and 40% of its profits from transactions with
Wheel. All of the above companies are incorporated in the same country.

Required
Write a report to the directors of Engina setting out the reasons why it is important to disclose
related party transactions and the nature of any disclosure required for the above transactions
under IAS 24 Related Party Disclosures.

Solution:
Engina
Report to: The Board of Directors of Engina
From: XXXXXXXX
Date: Subject: Related party transactions

140
Related party transactions
This report addresses the disclosure requirements of IAS 24 Related Party Disclosures with
regard to Engina. IAS 24 requires that all entities, listed or otherwise, provide disclosure of such
transactions as they may affect the assessments made by users of an entity‘s operations, risks
and opportunities.

It is understood that Engina is reluctant to disclose related party transactions because they are
believed to be both politically and culturally sensitive, however the following advice must be
followed in order to secure a listing/stock exchange registration.

IAS 24: Scope and purpose


IAS 24 does not provide any exclusion from its scope, and so disclosure must be made. Related
party transactions are a normal feature of business, but an entity‘s ability to succeed in business
is often affected by the strength of its relationship with other entities and individuals. The
results of the entity may be affected if these relationships were to be terminated. For example,
the ability of an entity to trade in a particular country may only be possible because of the
presence of its subsidiary in that local market. Similarly, prices and terms of trade may be
preferential because of the strength of the relationship. Therefore IAS 24 requires knowledge of
these transactions to be provided to the reader of the financial statements.

The results of an entity may be affected even if the related party transactions do not occur. A
parent may cease trading with a business partner upon acquisition of a subsidiary that can
supply similar products.

Disclosure must be given irrespective of whether the transactions took place at an arm‘s length
value, as such transactions may still be lost if the relationship is terminated. Hence the
knowledge of such transactions provides valuable information to investors and regulators.

Disclosure requirements
IAS 24 requires that, at a minimum, the following disclosures must be given: „
 The amount of the transaction „
 The amount of any outstanding balance and the terms, conditions and guarantees
attached „
 Allowance for any irrecoverable debts or amounts written off in the period „
 Disclosure that transactions were at an arm‘s length value can only be given if this
information can be substantiated.

Disclosures relevant to Engina


The following outlines the related party disclosure requirements for the three transactions you
have specifically requested comment on. It is your responsibility to bring any further related
party transactions to our attention in order that they can also be incorporated into your financial
statement disclosures.

(a) Sale of goods to directors


The sale of goods and a company car to Mr Satay both constitute related party transactions, due
to Mr Satay‘s position as a director of Engina. IAS 24 requires disclosure of all related party

141
transactions with key management personnel. However, accounting standards only apply to
material transactions. An item is considered material where knowledge of that transaction
might influence the decisions of a user of the financial statements. Materiality is not just a
matter of size, as small transactions with a director may still be of relevance to an investor if the
transaction is material to the director, despite not being material to the entity.

In the situation described, the transactions amount to GHC600,000 of sales and the sale of a
company car for GHC45,000 (market value GHC80,000). In terms of value these transactions
appear not to be material to Engina and neither do they appear to be material in value to Mr
Satay. However, given the sensitive nature of transactions with directors, and especially senior
directors like Mr Satay, the transactions should be disclosed in the financial statements in
accordance with good corporate governance practice. Significant contracts with directors, such
as these with Mr Satay, may also require disclosure by the local Stock Exchange.

(b) Hotel property


The sale of the hotel to the brother of Mr Soy, constitutes a related party transaction because of
Mr Soy‘s status as Managing Director. The property seems to have been sold at below market
price and IAS 24 requires disclosure of any information surrounding a transaction which will
allow the reader to understand its impact on the financial statements.

The hotel had a carrying value of GHC5m, but given the fall in market values it should have
been written down to its recoverable amount in accordance with IAS 36 Impairment.
Recoverable amount is measured at the higher of value in use (GHC3.6m) and fair value minus
costs of sale (GHC4.3 - 0.2m). Hence the property should have been recorded in the statement of
financial position at GHC4.1m. As the property was sold at GHC100,000 less than this impaired
value, disclosure of this fact should be made, together with any other information relevant to
the reader, such as the reason for the sale in light of the expected decline in prices in the future.

(c) Mr Satay
Mr Satay has investments in 100% of the equity of Car and 80% of the equity of Wheel. In turn,
Wheel owns 100% of Engina. Engina and Wheel are related because of their parent-subsidiary
relationship. In addition, because all three entities are under the common control of Mr Satay,
IAS 24 also considers Engina and Car to be related. Therefore, the transactions between Engina
and both Wheel and Car are related party transactions.
The transactions will need to be disclosed in the individual financial statements of all three
entities. In the group accounts, all intra-group transactions are cancelled on consolidation, and
so disclosure need not be made at this level.
Further disclosure requirements of director‘s interests in the equity of Engina may be necessary
under local Companies Code requirements and Stock Exchange rules.

QUESTION 3
Related party relationships are a common feature of commercial life. The objective of IAS 24:
Related Party Disclosures is to ensure that financial statements contain the necessary
disclosures to make users aware of the possibility that financial statements may have been
affected by the existence of related parties.
Required:

142
Explain TWO importance of disclosing related party relationships and transactions in financial
statements. (4 marks)
Solution:
IMPORTANCE OF RELATED PARTY DISCLOSURES
Investors invest in a business on the assumption that it aims to maximize its own profits for
the benefit of its own shareholders. This means that all transactions have been negotiated at
arm's length between willing and informed parties. The existence of related parties may
encourage directors to make decisions for the benefit of another entity at the expense of their
own shareholders. This can be done actively by selling goods and services cheaply to related
parties, or by buying in goods and services at an above market price. It can also happen when
directors chose not to compete with a related party, or offer guarantees or collateral for other
party's loans.
Disclosure is particularly important when a business is being sold. It may receive a lot of
custom, supplies, services or general help and advice from family or group companies. When
the company is sold these benefits may be withdrawn.
Related party transactions are not illegal, nor are they necessarily a bad thing. However
shareholders and potential investors need to be informed of material related party transactions
in order to make informed investment and stewardship decisions.

Question 4
Bongo Designs is the parent company of a small group. Its shares are stock market quoted, with
many shareholders. Only one shareholder, Akwasi Boakye has a holding over 5%. Akwasi
Boakye holds 20% of the shares and was the founder of the company. He still retains a seat on
the board which is made up of four executive directors (including himself) and two non-
executive directors.
Akwasi Boakye's domestic live-in partner of ten years, Abena Lamptey, recently set up a
company, Gushegu Ltd, in the textile industry with a friend, Akosua Pokuaa. Abena Lamptey
and Akosua Pokuaa each own 50% of the shares of Gushegu Ltd, and decisions are made jointly
under a contract that both parties signed.
Bongo Designs has two subsidiaries, Zabzugu Fabrics which is 100% owned and Binduri
Textiles which is 60% owned. The other 40% of Binduri Textiles is owned by a single
shareholder, Innovative Sissala, which has two seats on Binduri Textiles‟s five-member board.
Yaw Abdulai is the Finance Director of Zabzugu Fabrics. He is also the person responsible for
finance at the group level, but is not a member of the group‟s board.
Required: In accordance with IAS 24: Related Party Disclosures, identify the related parties of
Bongo Designs in the above scenario, explaining why each is a related party. (5 marks)
SOLU
Related Parties of Bongo Design

member of key management.

related party.
tey has invested in is jointly controlled. It is considered a related
party as there is joint control by a close family of key management of Bongo Designs.

same group as Bongo Designs.

143
group level.

at a group level).

PRACTICE QUESTIONS
QUESTION 1
During the year ended June 30, 2019. Ohene Limited [OL] has carried out several transactions
with the following individuals /entities.
a) AK Associates provides information technology services to OL. One of the directors of
OL is also the partner in AK Associates.
b) SS Bank Limited is the main lender. By virtue of an agreement it has appointed a
nominee director on the Board of OL.
c) Mr. Zee who supplies raw materials to OL, is the brother of the Chief Executive Officer
of the company.
d) JB Limited is the distributor of OL‘S products and have exclusive distribution rights for
the province of Punjab.
e) Mr. Tee is the General Manager-Marketing of OL, and is responsible for all major
decisions made in respect of sales prices and discounts.
f) OL‘s gratuity fund is administered by the Trustee appointed by the company.
g) MM Limited is the leading supplier of OL and supplies 60% of OL‘s raw materials.
h) Ms. Vee who conducted various training programmes for the employees of the
company, is the wife of OL‘s Chief Executive Officer.
Required:
Comment as to whether the above individuals/entities are ‗related parties‘ of the company of
not. Support your arguments with references from International Accounting Standards.
[15 marks]

QUESTION 2
The following related party transactions were carried out by Hernandez Limited [HL] during
the first year of its operation i.e. year ended December 31, 2019
a) Inventory costing GH¢15 million was sold for GH¢18 million to Chloride Limited [CL]
which owns 60% shares in HL. It is HL‘s policy to add 30% margin on cost. Outstanding
liability at year end, in respect of these purchases was GH¢6.5 million.
b) CL provided administrative services to HL. The cost of these services, if billed in the
open market, would have amounted to GH¢350,000. No entries were made to record
these transactions, as it was agreed that the services would be provided free of charge.
c) A property was sold to Bromide Limited [BL], an associated company, at its fair market
value of GH¢10 million. 50% of the amount was settled prior to year end. HL
reimbursed GH¢500,000 to BL on account of transfer and other incidental charges
related to this property.
d) An interest free loan of GH¢2 million was granted to an executive director of the
company under the terms of employment. During the year, GH¢200,000 were repaid by
the executive director.

144
e) On July 1, 2019 HL obtained a short term loan of GH¢25 million from one of its major
shareholders, at the prevailing annual interest rate of 12%. Both the principal and the
accrued interest were outstanding at the close of the year.
Required:
Prepare a note on related party transactions for inclusion in HL‘s financial statements for the
year ended December 31, 2019 showing disclosures as required by IAS 24 Related Party
Disclosures. [15 marks]

QUESTION 3
On 1 July 2018, Inert Gases Limited [IGL] acquired 80% shareholdings in Helium Limited [HL],
90% shareholding in Neon Limited [NL] and 55% shareholdings in Argon Limited [AL]. The
following transactions took place among these companies during the period up to 30 June 2019.
i. On 1 May 2018, IGL sold a machine to HL at 20% above the carrying amount of GH¢16
million. HL paid the entire amount on 15 July 2018. The useful life of the machine is 10
years.
ii. On 1 July 2018, NL awarded a contract of GH¢15 million to Gas extractors Ltd [GEL] for
the extension of its existing factory. One of the directors of IGL is also a partner in GEL.
iii. Since the date of acquisition, IGL has been providing management services to HL and
NL, IGL did not charge management fee for its services during the first year. However,
with effect from 1 July 2018, management fees has been charged from each company at
the rate of GH¢0.5 million per month. Payment is made on the 10th day of the next
month.
iv. On 1 January 2019, IGL sold goods amounting to GH¢10 million to Gold Limited [GL].
The wife of chief financial officer of NL is major shareholder in GL.
Required:
Prepare a note on related party disclosure including comparative figures, for inclusion in the
individual financial statements of IGL, HL, NL and AL for the year ended 30 June 2019.

FINANCIAL INSTRUMENTS
(IFRS 7 & IFRS 9)
Introduction to Financial Instruments
1. IAS 32 Financial Instruments: Presentation
2. IFRS 7 Financial Instruments: Disclosures
3. IFRS 9 Financial Instruments
IAS 32 FINANCIAL INSTRUMENTS: PRESENTATION
Objective
The stated objective of IAS 32 is to establish principles for presenting financial instruments as
liabilities or equity and for offsetting financial assets and liabilities.
IAS 32 addresses this in a number of ways:
clarifying the classification of a financial instrument issued by an entity as a liability or as
equity
prescribing the accounting for prescribing strict treasury shares (an entity's own repurchased
shares) conditions under which assets and liabilities may be offset in the balance sheet
Scope

145
IAS 32 applies in presenting and disclosing information about all types of financial instruments
with the exceptions of items under IAS 27, IAS 28, IFRS 11, IAS 19 and IFRS 2 and non- financial
items which will be settled through physical delivery.
However, IAS 32 applies to:
The contracts to buy or sell a non-financial item that can be settled net in cash or another
financial instrument
Definitions
The following terms are used in this Standard with the meanings specified:
A Financial Instrument is any contract that gives rise to a financial asset of one entity and a
financial liability or equity instrument of another entity.
A Financial Asset is any asset that is:
(a) Cash;
(b) An equity instrument of another entity;
(c) A contractual right:
(i) To receive cash or another financial asset from another entity; or (ii) To exchange financial
assets or financial liabilities with another entity under conditions that are potentially favourable
to the entity; or
(d) A contract that will or may be settled in the entity‘s own equity instruments and is: (i) A
non-derivative for which the entity is or may be obliged to receive a variable number of the
entity‘s own equity instruments; or (ii) A derivative that will or may be settled other than by the
exchange of a fixed amount of cash or another financial asset for a fixed number of the entity‘s
own equity instruments.
A Financial Liability is any liability that is:
(a) A contractual obligation:
(i) To deliver cash or another financial asset to another entity; or
(ii) To exchange financial assets or financial liabilities with another entity under conditions that
are potentially unfavourable to the entity; or
(b) a contract that will or may be settled in the entity‗s own equity instruments and is: (i) a non-
derivative for which the entity is or may be obliged to deliver a variable number of the entity‗s
own equity instruments; or
(ii) a derivative that will or may be settled other than by the exchange of a fixed amount of cash
or another financial asset for a fixed number of the entity‗s own equity instruments.
An Equity Instrument is any contract that evidences a residual interest in the assets of an entity
after deducting all of its liabilities.
A Puttable Instrument is a financial instrument that gives the holder the right to put the
instrument back to the issuer for cash or another financial asset or is automatically put back to
the issuer on the occurrence of an uncertain future event or the death or retirement of the
instrument holder.
Examples of financial assets include Trade receivables, Options, Shares (when used as an
investment).
Examples of financial liabilities include Trade payables, Debenture loans payable, Redeemable
preference (non-equity) shares, Forward contracts standing at a loss.
Financial instruments include both of the following.
(a) Primary instruments: e.g. receivables, payables and equity securities
(b) Derivative instruments: e.g. financial options, futures and forwards, interest rate swaps and
currency swaps, whether recognized or unrecognized
IAS 32 makes it clear that the following items are not financial instruments.

146
Physical assets, e.g. inventories, property, plant and equipment, leased assets and intangible
assets (patents, trademarks etc).
Prepaid Expenses, deferred revenue and most warranty obligations.
Liabilities or assets that is not contractual in nature
Contractual rights/obligations that do not involve transfer of a financial asset, e.g. commodity
futures contracts, operating leases
Compound Financial Instruments
The issuer of a non-derivative financial instrument shall evaluate the terms of the financial
instrument to determine whether it contains both a liability and an equity component. Such
components shall be classified separately as financial liabilities, financial assets or equity
instruments.
An entity recognises separately the components of a financial instrument that:
(a) creates a financial liability of the entity and
(b) grants an option to the holder of the instrument to convert it into an equity instrument of the
entity.
Treasury Shares
If an entity reacquires its own equity instruments, those instruments (‗treasury shares‗) shall be
deducted from equity. No gain or loss shall be recognized in profit or loss on the purchase, sale,
issue or cancellation of an entity‗s own equity instruments. Consideration paid or received shall
be recognized directly in equity.
Interest, Dividends, Losses and Gains
Interest, dividends, gains, and losses relating to an instrument classified as a liability should be
reported in profit or loss. This means that dividend payments on preferred shares classified as
liabilities are treated as expenses. On the other hand, distributions (such as dividends) to
holders of a financial instrument classified as equity should be charged directly against equity,
not against earnings.
Transaction costs of an equity transaction are deducted from equity. Transaction costs related
to an issue of a compound financial instrument are allocated to the liability and equity
components in proportion to the allocation of proceeds.

Offsetting Financial Assets and Liabilities


IAS 32 also prescribes rules for the offsetting of financial assets and financial liabilities. It
specifies that a financial asset and a financial liability should be offset and the net amount
reported when, and only when, an entity:
 has a legally enforceable right to set off the amounts; and
 intends either to settle on a net basis, or to realise the asset and settle the liability
simultaneously.
Disclosures
Financial instruments disclosures are in IFRS 7 Financial Instruments: Disclosures, and no
longer in IAS 32.
IFRS 9 Financial Instruments (Replacement of IAS 39)
Objective
The objective of this IFRS is to establish principles for the financial reporting of financial assets
and financial liabilities that will present relevant and useful information to users of financial
statements for their assessment of the amounts, timing and uncertainty of an entity‗s future cash
flows.
Recognition and Derecognition

147
Initial Recognition
Financial asset or financial liability should be recognized by an entity in its statement of
financial position when the entity becomes party to the contractual provisions of the financial
asset or financial liability, even at nil cost.
Effective interest rate method
Total finance cost:
GHC
Interest paid xx
Issuance cost xx
Discount xx
Premium xx
Total finance cost (Allocate over loan
term using effective interest rate) xx

Years Opening Finance cost Interest paid Rollover Closing


balance charged balance
Y1 xx Issue cost, discount Nominal value discount, xx
& premium x par value issue cost
& premium

Calculation of fair value of financial instrument


Fair value = Present value of future cash flows at current market interest for similar financial
instruments
 Use market rate of relevant year or date
 Future cash flows will be used.
CLASSIFICATION OF FINANCIAL ASSETS
An entity shall classify financial assets as subsequently measured at:
 Amortised cost or
 Fair value through profit or loss (FVTPL)
 Fair value through other comprehensive income (FVTOCI)
A financial asset (debt instrument only) shall be measured at amortised cost if both of the
following conditions are met:
(a) The asset is held within a business model whose objective is to hold assets in order to collect
contractual cash flows (Solely principal amount and interest).
(b) The contractual terms of the financial asset give rise on specified dates to cash flows that are
solely payments of principal and interest on the principal amount outstanding. Interest is
compensation for time value of money and credit risk.
Important points
 Assess business management model on portfolio level not on individual investment
 Irregular sales of investments do not impact business management model
 Businesses may have different business management models for different investments
Fair Value Designation of Financial Assets
Even if an instrument meets the two amortised cost tests, IFRS 9 contains an option to designate
a financial asset as measured at Fair Value Through Profit and Loss if doing so eliminates or
significantly reduces a measurement or recognition inconsistency.
Fair value through other comprehensive income
The criteria are as follows:

148
For investments in equity For investment in debt
Investment in equity not held for trading Business management model is to hold
investment for collection of contractual cash
flows and to sell
Entity has made an irrevocable election to Investment can generate contractual cash flow
recognize gain/ (loss) in other comprehensive at specified time, Principal and Interest, where
income interest is compensation of time value of
money & credit risk

Fair value through profit or loss


The criteria are as follows:
 It is the default category (After not meeting any other category‗s criteria)
 Investments not categorized at amortized cost and FVTOCI will be classified here
Examples
Investment in perpetuity bond At FVTPL
Investment in convertible bond At FVTPL
Investment in fixed rate bond Could be at amortised cost
Investment in variable interest rate bond Could be at amortised cost, market
interest rates are reset periodically
therefore it accounts for time value
of money and credit risk of the
company
Investment in bond of GHC10,000 at
6% variable with:
Gold price At FVTPL
Stock market At FVTPL
Inflation (Credit risk & Time value of money) Could be at amortised cost

Measurement of Financial Assets


1) At Amortised cost:
Initial measurement: Fair value (Cash paid) + Transaction cost
Subsequent measurement: At amortised cost using effective interest rate method
2) At FVTOCI:
Initial measurement: Fair value (Cash paid) + Transaction cost
Subsequent measurement: At fair value and gain/loss will be charged to OCI
3) At FVTPL:
Initial measurement: Fair value (Cash paid) Transaction cost is charged to P&L Subsequent
measurement: At fair value and gain/loss will be charged to P&L
Derecognition of Financial Assets
Derecognition is the removal of a previously recognised financial instrument from an entity‗s
statement of financial position.
An entity should derecognize a financial asset when:
(a) The contractual rights to the cash flows from the financial asset expire e.g., cash received
from receivable, Option exercised, option expired etc. , OR
(b) The entity transfers substantially all the risks and rewards of ownership of the financial asset
to another party.

149
Accounting Treatment
On derecognition the difference between the carrying amount and consideration received
should be recognised in P&L. Reclassify in P&L any accumulated gains or losses already
recognised in OCI
Assess risk and rewards of financial asset immediately before and after transfer
a) Substantial risk and rewards transferred
Derecognize:
Dr. Cash/Asset xx
Dr/Cr. (loss)/Profit xx
Cr. Financial asset xx
Examples:
 Unconditional sale of financial asset
 Factoring of receivables (without recourse)
 Sales of asset on repurchase terms where repurchase will be at market price
 Sale of asset with call or put option and option is deep out of money
b) Substantial risks and rewards retained Continue to recognize the asset Any cash received would be
secured loan
Dr. Cash xx
Cr. Loan xx
Examples:
 Factoring of receivables with recourse
 Sales of asset on repurchase term where repurchase price is already decided
 Sale of asset with call or put option and option is deep in the money
 Sale of asset with return swap contract
 Remember always to apply the principle of substance over form.

Question 1
On 1 January,2011 Action Ltd purchased a debt instrument for its fair value of GHC 1,000.The
debt instrument is due to mature on 31 December,2015.The instrument has a principal amount
of GHC 1,250 and the instrument carries fixed interest of 4.72% that is paid annually.The
effective interest rate is 10%.
How should Action Ltd account for the debt instrument over its five year term
Question 2
On 2 January 2009, a company buys GHC100,000 of 6% loan stock for GHC93,930. Interest will
be received on 31 December each year and the stock will be redeemed at par on 31 December
2013. The company intends to hold the stock until maturity and calculates the effective interest
rate to be 7.5% per annum. Financial statements are prepared to 31 December each year.
Required:
How should be accounted for?
Solution:
Year Net Proceeds Finance Income(7.5%) Interest Rec’d (6%) Bal.c/d
GHC000 GHC000 GHC000 GHC000
2009 93,930 7,045 (6,000) 94,975
2010 94,975 7,123 (6,000) 96,098
2011 96,098 7,207 (6,000) 97,305
2012 97,305 7,298 (6,000) 98,603
2013 98,603 7,397 (106,000) -

150
36,070
Total Cost: GHC000
Annual Interest (6%x100,000 X 5 years) 30,000
Redemption Premium (100,000 – 93,930 ) 3,070
36,070

Income Statement Extract for the year ended……


Year 2009 2010 2011 2012 2013
GHC000 GHC000 GHC000 GHC000 GHC000
Finance Cost 7,045 7,123 7,207 7,298 7,397
Statement of Financial Position as at ……………..
Non-Current Liabilities : 2009 2010 2011 2012 2013
GHC GHC GHC GHC GHC
6% Loan Stock 94,975 96,098 97,305 - -
Current Liabilities
6% Loan Stock - - - 98,603 -

Question 3
Harry plc purchases a bond for GHC441,014 on 1 Jan 2012. It will be redeemed on 31 December
2015 for GHC600,000. Harry intends to hold the bond to maturity and it carries no interest
coupon. The final payment of GHC600,000 consists entirely of compensation for principal
advanced by Harry and accrued interest thereon. The effective rate of return is 8%.
Required:
Discuss how this bond will be recognised on purchase, and subsequently accounted for over its
life as an investment of Harry plc. Assume the reporting date is 31 December each year. Show
appropriate journal entries.
Solution:
The bond will be classified as an amortised cost financial asset. This is because the payment due
consists entirely of principal and interest, and Harry plc has the intention to hold the
investment to collect the contractual cash flows. Therefore it is accounted for under the
―amortised cost‖ method. This method allocates the effective interest income to the accounting
periods the investment is held using the effective interest rate method. Accounting treatment
and journal entries are as follows:
1 January 2012: Initial recognition is at fair value. This is deemed to be the amount paid for the
financial asset. If there were costs incurred on the purchase these would be capitalised also.
Dr Financial asset GHC441,014
Cr Cash GHC441,014
31 December 2012: Interest for the year is calculated at 8% on the balance outstanding for the
year 2012. This is credited to income as a gain, and increases the carrying value of the financial
asset. Amount = 441,014 * 8% = 35,281. If the interest were paid, the debit would be to cash. If
the interest were partially paid, the unpaid amount would increase the carrying value of the
financial asset.
Dr Financial asset GHC35,281
Cr Profit or loss - Finance income GHC35,281
31 December 2013: Interest for year is again calculated at 8% of the amount outstanding.
However, the amount outstanding is deemed to include the accrued but unpaid interest

151
recognised in prior periods. Hence the interest income for 2013 is (441,014 + 35,281) * 8% =
38,104.
Dr Financial asset GHC38,104
Cr Profit or loss - Finance income GHC38,104
31 December 2014: The same principle applies to 2014 and 2015 as applied to 2013. Interest for
year = (441,014 + 35,281 + 38,104) * 8% = 41,152.
Dr Financial asset GHC41,152
Cr Profit or loss - Finance income GHC41,152
31 December 2015: Interest for year = (441,014 + 35,281 + 38,104 + 41,152) * 8% = 44,444
Dr Financial asset GHC44,444
Cr Profit or loss - Finance income GHC44,444
31 December 2015: The repayable amount is GHC600,000. As the carrying amount is slightly
different due to rounding errors, the difference of GHC5 is taken to profit or loss at the date of
disposal of the investment. The investment is derecognised as the entity no longer has an
interest in the bond.
Dr Cash GHC600,000
Cr Financial asset (sum of entries to date) GHC599,995
Cr Profit or loss - Finance income GHC5
Question 4
Louise plc purchased 10,000 shares in another company during 2011 as a long term investment
for a cost of GHC4.50 per share, incurring transaction costs of GHC1,450. She made an
irrevocable election immediately after purchase to take all remeasurement gains and losses
through ―other comprehensive income‖ as allowed by IFRS 9. The market value of the shares at
31 December 2011 was GHC5.04 per share. It would cost GHC1,500 in transaction costs to sell
the shares at that date. In the event, due to liquidity problems, Louise plc had to sell the shares
in 2012 for GHC5.45 per share, incurring transaction costs of GHC1,510 on the deal.
Required:
Discuss the accounting treatment of the financial asset in 2011 and 2012. Assume the financial
year ends on 31 December each year. Show appropriate journal entries.

Solution:
The shares are classified as a fair value financial asset. This is because they do not meet the
criteria for classification under amortised cost. As the election was made under IFRS 9 to take
gains and losses on remeasurement to other comprehensive income, they are not accounted for
as fair value through profit or loss. This means the transaction costs are included in the amount
capitalised. Therefore they are recognised initially at fair value plus transaction costs
[GHC46,450 (10,000 shares * GHC4.50) + GHC1,450].
Initial recognition during 2011:
Dr Financial assets GHC46,450
Cr Cash GHC46,450
At each period end, the investment is revalued to fair value. This is the quoted market price at
the end of the reporting period. In this case this is GHC5.04 per share giving a total value of
GHC50,400. No allowance is made for transaction costs, as no transaction has occurred. This
represents a gain of GHC3,950 (50,400 – 46,450). Any gain or loss is recognised in the statement
of comprehensive income through ―other comprehensive income‖ in accordance with the
election made by Louise.

152
31 December 2011 (remeasurement):
Dr Financial assets (50,400-46,450) GHC3,950
Cr Other comprehensive income GHC3,950
On disposal, the asset is derecognised, and any difference between the carrying value and the
net proceeds is recognised within profit or loss. The net disposal proceeds are GHC5.45 per
share less transaction costs of GHC1,510 [(10,000 * 5.45) – 1,510] or GHC52,990. The carrying
value is GHC50,400, leading to a gain on disposal of GHC2,590.
Disposal during 2012:
Dr Cash GHC52,990
Cr Financial assets GHC50,400
Cr Profit or loss – gain on disposal of financial assets GHC2,590
Question 5
Costa bought an investment for GHC 40,000 plus associated transaction costs of GHC 1,000.The
asset was designated upon initial recognition as fair value through other comprehensive
income. At reporting date the fair value of the financial asset has risen to GHC 60,000.Shortly
after the reporting date the financial asset was sold for GHC 70,000 .
Required:
(a) How should this be accounted for?
(b) How would the answer have been different if the investment had been classified as at
fair value through profit and loss?
Solution:
(a) Initial Recognition of Financial Assets
GHC
Dr Asset(40,000 +1,000) 41,000
Cr Cash 41,000
Increase in Value transferred to other Components of Equity
Dr Asset 19,000
Cr Other Comprehensive Income(other components of Equity) 19,000
Dr Cash 70,000
Cr Asset 60,000
Cr P&L 10,000
Financial Asset Account
Cash 41,000
Other Components of Equity (OCI) 19,000 Balance c/d 60,000
60,000 60,000
Balance b/d 60,000 Cash 70,000
P&L (Profit) 10,000
70,000 70,000
(b) Initial Measurement of Financial Assets
GHC
Dr Asset 40,000
Cr Cash 40,000
Transaction Cash Expensed
GHC
Dr Expenses 1,000
Cr Cash 1,000
Re-measurement of Financial Assets at the end of the year:

153
GHC
Dr Asset 20,000
Cr P&L 20,000
De-recognition of Financial Assets
Dr Cash 70,000
Cr Asset 60,000
Cr P&L 10,000

Financial Asset Account


Cash 40,000
Profit&Loss 20,000 Balance c/d 60,000
60,000 60,000
Balance b/d 60,000 Cash 70,000
P&L (Profit) 10,000
70,000 70,000

Question 6
An investment is purchased for GHC30,000 plus 1% transaction costs on 1 April 2016. It is
classified as available for sale. At the end of the financial year, which is 31 March 2017, the
investment is re-valued to its fair value. This is GHC40,000. On 11 December 2017 it is sold for
GHC50,000.
Required:
Show how the transaction should be treated.

Solution:
Initially, the investment will be recorded at cost at GHC30,300. This is the cost plus the
capitalised transaction costs. At the end of the financial year (31 March 2017) the investment is
re-valued to its fair valued of GHC40,000. There is a gain of GHC9,700 (GHC40,000 –
GHC30,300). This gain of GHC9,700 is included as other comprehensive income in the
statement of comprehensive income for the year to 31 March 2017, and is taken directly to an
‗Available for sale reserve‘ in equity.
The company will need to keep a record of what each financial instrument contributes to this
reserve, so that when it is eventually sold, the appropriate amount can be recycled.
The journal entry to record the disposal for the above example is as follows:
GHC GHC
DR Cash 50,000
CR Investment 40,000
DR Available for sale reserve 9,700
CR Profit or loss 19,700

The profit of GHC19,700 represents the gain on disposal of GHC10,000 (GHC50,000 –


GHC40,000) plus the recycled profit from the available sale reserve of GHC9,700.
The GHC9,700 gain is reclassified from other comprehensive income in the previous year, and
in the year to 31 March 2018, profit or loss will include the realised gain of GHC9,700 (as shown
above), but there should also be an offsetting reduction of GHC9,700 in other comprehensive
income for the year, to prevent double-counting of the GHC9,700 in income.
CLASSIFICATION OF FINANCIAL LIABILITIES

154
Financial liabilities are either classified as:
 Financial liabilities at amortised cost; or
 Financial liabilities as at fair value through profit or loss (FVTPL).
Financial liabilities are measured at amortised cost unless either:
 The financial liability is held for trading and is therefore required to be measured at
FVTPL (e.g. derivatives not designated in a hedging relationship), or
 The entity elects to measure the financial liability at FVTPL (using the fair value option).
Measurement of Financial Liabilities
1) At Amortised cost:
Initial measurement: Fair value (Cash received) - Issue cost
Subsequent measurement: At amortised cost using effective interest rate method
Examples of items at amortised cost are
 Trade payables
 Loan payables
 Bank borrowings
2) At Fair value option:
Fair value Gain/Loss will be split
 Gain/Loss due to own credit risk will be charged to OCI
 Gain/Loss due to other credit risk will be charged to P&L
3) At FVTPL:
Initial measurement: Fair value (Cash received) Transaction cost is charged to P&L
Subsequent measurement:
 Fair value will be calculated by PV(FCF) by using current market interest rate
 Any Gain or loss will be charged to P&L

Examples of items at FVTPL are


 Held for trading liability
 Derivatives standing at loss
 Contingent liability arising at business combination
Reclassification of Financial Instruments
For financial assets, reclassification is required between FVTPL and amortised cost, or vice
versa, if and only if the entity's business model objective for its financial assets changes so its
previous model assessment would no longer apply.
If reclassification is appropriate, it must be done prospectively from the reclassification date. An
entity does not restate any previously recognised gains, losses, or interest.
IFRS 9 does not allow reclassification where:
 financial assets have been classified as equity instruments, or
 the fair value option has been exercised in any circumstance for a financial assets or
financial liability.
Derecognition of Financial Liabilities
An entity shall remove a financial liability (or a part of a financial liability) from its statement of
financial position when, and only when, it is extinguished - i.e. when the obligation specified in
the contract is discharged or cancelled or expires.
The difference between the carrying amount of a financial liability (or part of a financial
liability) extinguished or transferred to another party and the consideration paid, including any
non-cash assets transferred or liabilities assumed, shall be recognised in profit or loss.

155
Question 1
Quaresma Ltd issued 6% Debt for GHC 2,000.The debt is redeemable at GHC 2,400 in three
years time. The effective rate of the debt is 12% per annum.
How is the debt valued and recognized?
Solution:
Year Net Proceeds Finance Cost(12%) Interest Paid(6%) Bal.c/d
GHC GHC GHC GHC
1 2,000 240 (120) 2,120
2 2,120 254 (120) 2,254
3 2,254 266 (2520) -
760
Total Cost: GHC
Annual Interest (GHC240 X 3 years) 360
Redemption Premium (2,400-2,000) 400*
760

Income Statement Extract for the year ended……


Year 1 2 3
GHC GHC GHC
Finance Cost 240 254 266

Statement of Financial Position as at ……………..


Non-Current Liabilities : 1 2 3
GHC GHC GHC
6% Debt 2,120 - -
Current Liabilities
6% Debt - 2,254 -
Question 2
Luiz Henrique Ltd issued GHC 1,000,000 of 10% (coupon rate) loan stock repayable at a
premium of 2% after 5 years. Issue costs are GHC 10,000 and the effective interest rate is 10.6%.
How is the debt valued and recognized.
Solution:
Year Net Proceeds Finance Cost(10.6%) Interest Paid(10%) Bal.c/d
GHC000 GHC000 GHC000 GHC000
1 990 105 (100) 995
2 995 105 (100) 1,000
3 1,000 106 (100) 1,006
4 1,006 107 (100) 1,013
5 1,013 107 (1,120) -
530
Net Proceeds: GHC000
NominalValue/Principal Amount 1,000
Less Issue Cost 10
990
Total Cost: GHC000
Issue Cost 10
Annual Interest (10%x1,000 X 5 years) 500
Redemption Premium (1,020-1,000) 20

156
530
Income Statement Extract for the year ended……
Year 1 2 3 4 5
GHC000 GHC000 GHC000 GHC000 GHC000
Finance Cost 105 105 106 107 107

Statement of Financial Position as at ……………..


Non-Current Liabilities : 1 2 3 4 5
GHC GHC GHC GHC GH
10% Loan Stock 995 1,000 1,006 - -
Current Liabilities
10% Loan Stock - - - 1,013 -
Question 3
Catalan Masters issues GHC 360,000 of a redeemable 2% debenture at a discount of 14% on 1
January, 2005.Issue cost were GHC 5,265.The debentures will be redeemed on 31
December,2007 at par. Interest is paid annually in arrears and the effective interest rate is 8%.
Show the effect of the transaction on the statement of financial position and income statement
for the three year term of the debenture.
Solution:
Year Net Proceeds Finance Cost(8%) Interest Paid(2%) Bal.c/d
GHC GHC GHC GHC
2005 304,335 24347 (7200) 321,482
2006 321,482 25,718 (7,200) 340,000
2007 340,000 27,200 (367,200) -
77,265
Net Proceeds:
GHC
Nominal Value 360,000
Less Discount (14%x 360,000) (50,400)
Less Issue Cost (5,265)
304,335
Total Cost: GHC
Annual Interest (GHC360,000 X2% X 3 years) 21,600
Discount (14% x 360,000) 50,400
Issue Cost 5,265
77,265
Income Statement Extract for the year ended……
Year 2005 2006 2007
GHC GHC GHC
Finance Cost 24,347 25,718 27,200

Statement of Financial Position as at ……………..


Non-Current Liabilities : 2005 2006 2007
GHC GHC GHC
2% Debentures 321,482 - -

Current Liabilities
2% Debentures - 340,000 -

157
COMPOUND FINANCIAL INSTRUMENTS
The issuer of a non-derivative financial instrument shall evaluate the terms of the financial
instrument to determine whether it contains both a liability and an equity component. Such
components shall be classified separately as financial liabilities, financial assets or equity
instruments.
An entity recognises separately the components of a financial instrument that:
(a) creates a financial liability of the entity and
(b) grants an option to the holder of the instrument to convert it into an equity instrument of the
entity.
Question 1
On 1 April 2015 Pedro Ltd issued an 8% GHC5 million convertible loan at par. The loan is
convertible in three years time to ordinary shares or redeemable at par in cash. The directors
decided to issue a convertible loan because a non-convertible loan would have required an
interest rate of 10%. The directors intend to show the loan at GHC5 million under non-current
liabilities. The following discount rates are available:
8% 10%
Year 1 0·93 0·91
Year 2 0·86 0·83
Year 3 0·79 0·75
Required
Describe (and quantify where possible) how Pedro should treat the loan in its financial
statements for the year ended 31 March 2016 commenting on the directors' views where
appropriate.
(4 marks)

Solution:
As this is a convertible loan, it must be apportioned between debt and equity. Per IAS 32 and
IAS 39 this is calculated as follows:
GHC'000
Present value of the principal to be repaid: GHC5 million × 0.75 3,750
Present value of interest: GHC0.4 million × 2.49 (0.91 + 0.83 + 0.75) 996
Debt element 4,746
Equity element(Difference) 254
Proceeds of issue 5,000

The income statement and statement of financial position amounts will be as follows:
GHC000 GHC000
Debt element of loan 4,746
Interest at 10% (income statement) 475
Interest paid (400)
Balance due 75
Balance of loan at 31 March 2016
(statement of financial position) 4,821

Question 2

158
Bertrand issued GHC10 million convertible loan notes on 1 October 2010 that carry a nominal
interest (coupon) rate of 5% per annum. They are redeemable on 30 September 2013 at par for
cash or can be exchanged for equity shares in Bertrand on the basis of 20 shares for each
GHC100 of loan. A similar loan note, without the conversion option, would have required
Bertrand to pay an interest rate of 8%.
When preparing the draft financial statements for the year ended 30 September 2011, the
directors are proposing to show the loan note within equity in the statement of financial
position, as they believe all the loan note holders will choose the equity option when the loan
note is due for redemption. They further intend to charge a finance cost of GHC500,000 (GHC10
million x 5%) in the income statement for each year up to the date of redemption.

The present value of GHC1 receivable at the end of each year, based on discount rates of 5%
and 8%, can be taken as:
5% 8%
End of year 1 0·95 0·93
2 0·91 0·86
3 0·86 0·79

Required:
(a) (i) Explain why the nominal interest rate on the convertible loan notes is 5%, but for non-
convertible loan notes it would be 8%. (2 marks)
(ii) Briefly comment on the impact of the directors’ proposed treatment of the loan notes on
the financial statements and the acceptability of this treatment.
(3 marks)
(b) Prepare extracts to show how the loan notes and the finance charge should be treated by
Bertrand in its financial statements for the year ended 30 September 2011.
(5 marks)
(10 marks)
Solution:
(a) (i) The interest rate (5%) for the convertible loan notes is lower because of the potential value
of the conversion option.The cost of equivalent loan notes without the option is 8%, the
difference is mainly due to the market expectation of the higher worth of Bertrand‘s equity
shares (compared to the cash alternative) when the loan notes are due for redemption.
From the entity‘s viewpoint, the conversion option means lower payments of interest (to help
cash flow), but it will eventually cause a dilution of earnings.
(ii) If the directors‘ treatment were acceptable, the use of the conversion option (compared to
issuing non-convertible loans) would improve profit and earnings per share because of lower
interest rates (and hence interest charges) and the company‘s gearing would be lower as the
loan notes would not be shown as debt. However, this proposed treatment is not acceptable. A
convertible loan note is a complex (hybrid) financial instrument and IFRS requires that the
proceeds of the issue should be allocated between equity (the value of the option) and debt and
the finance charge should be based on that of an equivalent non-convertible loan (8% in this
case).

(b) Extracts from the financial statements of Bertrand

Income statement for the year ended 30 September 2011

159
GHC‘000
Finance costs (9,190 x 8%) 735

Statement of financial position as at 30 September 2011


Equity
Equity option 810
Non-current liabilities
8% convertible loan notes ((9,190 x 1·08) – 500) 9,425
Working
Year ended Cash flow Discount rate Discounted cash flows
30 September GHC’000 at 8% GHC’000
2011 500 0·93 465
2012 500 0·86 430
2013 10,500 0·79 8,295
value of debt component 9,190
value of equity option component (= balance) 810
total proceeds 10,000
IMPAIRMENT OF FINANCIAL INSTRUMENTS
Scope
The impairment requirements are applied to:
 Financial assets measured at amortised cost (incl. trade receivables)
 Financial assets measured at fair value through OCI
 Loan commitments and financial guarantees contracts where losses are currently
accounted for under IAS 37 Provisions, Contingent Liabilities and Contingent Assets
 Lease receivables.
The impairment model follows a three-stage approach based on changes in expected credit
losses of a financial instrument that determine
 the recognition of impairment, and
 the recognition of interest revenue.
Initial recognition
At initial recognition of the financial asset an entity recognises a loss allowance equal to 12
months expected credit losses which consist of expected credit losses from default events
possible within 12 months from the entity‗s reporting date. An exception is purchased or
originated credit impaired financial assets.
Subsequent measurement
Stage 1 2 3
Impairment 12 month expected Lifetime expected credit loss
credit loss

Interest Effective interest on the gross carrying amount Effective interest on


(before deducting expected losses) the net (carrying)
amount

THREE STAGE APPROACH


Stage 1
12 month expected credit losses (gross interest)

160
 Applicable when no significant increase in credit risk
 Entities continue to recognise 12 month expected losses that are updated at each
reporting date
 Presentation of interest on gross basis
Stage 2
Lifetime expected credit losses (gross interest)
 Applicable in case of significant increase in credit risk
 Recognition of lifetime expected losses
 Presentation of interest on gross basis
Stage 3
Lifetime expected credit losses (net interest)
 Applicable in case of credit impairment
 Recognition of lifetime expected losses
 Presentation of interest on a net basis
PRACTICAL EXPEDIENTS
30 days past due rebuttable presumption
 Rebuttable presumption that credit risk has increased significantly when contractual
payments are more than 30 days past due
 When payments are 30 days past due, a financial asset is considered to be in stage 2 and
lifetime expected credit losses would be recognised
 An entity can rebut this presumption when it has reasonable and supportable
information available that demonstrates that even if payments are 30 days or more past
due, it does not represent a significant increase in the credit risk of a financial
instrument.

Low credit risk instruments


 Instruments that have a low risk of default and the counterparties have a strong capacity
to repay (e.g. financial instruments that are of investment grade)
 Instruments would remain in stage 1, and only 12 month expected credit losses would
be provided.
SIMPLIFIED APPROACH
Short term trade receivables
 Recognition of only ‗lifetime expected credit losses‗ (i.e. stage 2)
 Expected credit losses on trade receivables can be calculated using provision matrix (e.g.
geographical region, product type, customer rating, collateral or trade credit insurance,
or type of customer)
 Entities will need to adjust the historical provision rates to reflect relevant information
about current conditions and reasonable and supportable forecasts about future
expectations.
Long term trade receivables and lease receivables
Entities have a choice to either apply:
 the three-stage expected credit loss model; or
 the ‗simplified approach‗ where only lifetime expected credit losses are recognised.
LOAN COMMITMENTS AND FINANCIAL GUARANTEES
 The three-stage expected credit loss model also applies to these off balance sheet
financial commitments

161
 An entity considers the expected portion of a loan commitment that will be drawn down
within the next 12 months when estimating 12 month expected credit losses (stage 1),
and the expected portion of the loan commitment that will be drawn down over the
remaining life the loan commitment (stage 2)
 For loan commitments that are managed on a collective basis an entity estimates
expected credit losses over the period until the entity has the practical ability to
withdraw the loan commitment.
Question 1
On 1 Febraury,2006,Mourinho makes a four year loan of GHC 10,000 to Guardiola.The coupon
rate on the loan is 6%,the same as effective rate of ineterst.Interest is received at the end of each
year.
During February,2009,it becomes clear that Guardiola is in financial difficulties.This is the
necessary objective evidence of impairment.
At this time the current market rate of interest is 8%.
It is estimated that the future remaining cash flows from the loan will be only
GHC6,000,instead of GHC 10,600 (the GHC 10,000 principal plus interest for the fourth year of
GHC 600).
Required:
Calculate the impairment loss how it will be recognized.
Solution:
Computation of Impairment of Financial Assets
GHC
Carrying Value 10,000
Less PV of future Cash flow at original rate (Recoverable Amount):
(GHC6,000 x 1/1.06)-Reversing 1 year back (Balance Sheet) 5,660
Impairment Loss (P&L) 4,340
Income Statement Extract
Expenses: GHC
Impairment Loss 4,340
Statement of Financial Position Extract
NCA: GHC
Financial Asset 5,660

Question 2
On 1 January,2002,Anchelloti makes a five year loan of GHC 10,000 with an interest rate of 10%
(the same as effective rate of interest),with the principal being repaid at the end of the five
years.
During January,the borrower is in financial difficulty and it is estimated that the future cash
flows will be GHC 4,000 rather than GHC 11,000 (GHC 10,000 principal plus GHC 1,000
interest).At this date the current market rate of interest is 9%.
Required:
How should this be accounted for?
Solution:
Computation of Impairment of Financial Assets
GHC
Carrying Value 10,000
Less PV of future Cash flow at original rate (Recoverable Amount):

162
(GHC4,000 x 1/1.06)-Reversing 1 year back (Balance Sheet) 3,774
Impairment Loss (P&L) 6,226
Income Statement Extract
Expenses: GHC
Impairment Loss 6,224
Statement of Financial Position Extract
NCA: GHC
Financial Asset 3,774

HEDGING
Hedging, for accounting purposes, means designating one or more hedging instruments so that
their change in fair value is an offset, in whole or in part, to the change in fair value or cash
flows of a hedged item.
Hedging instruments: A hedging instrument is a designated derivative or (in limited
circumstances) another financial asset or liability whose fair value or cash flows are expected to
offset changes in the fair value or cash flows of a designated hedged item.
Hedged item
A hedged item is an asset, liability, firm commitment, or forecasted future transaction that:
(a) exposes the entity to risk of changes in fair value or changes in future cash flows, and that
(b) is designated as being hedged
Hedge accounting
Hedge accounting recognises the offsetting effects on profit or loss of changes in the fair values
of the hedging instrument and the hedged item.
Hedging relationships are of three types:
(a) fair value hedge: a hedge of the exposure to changes in fair value of a recognised asset or
liability or an unrecognised firm commitment, or an identified portion of such an asset, liability
or firm commitment, that is attributable to a particular risk and could affect profit or loss.
(b) cash flow hedge: a hedge of the exposure to variability in cash flows that (i) is attributable to
a particular risk associated with a recognised asset or liability (such as all or some future
interest
payments on variable rate debt) or a highly probable forecast transaction and (ii) could affect
profit or loss.
(c) hedge of a net investment in a foreign operation as defined in IAS 21.
Designation and Documentation
Must be formalised at the inception of the hedging relationship:
 The hedging relationship
 Risk management strategy and objective for undertaking the hedge
 The hedged item and hedging instrument
 How hedge effectiveness will be assessed.
All three hedge effectiveness requirements met
(a) An economic relationship exists between the hedged item and hedging instrument
(b) Credit risk does not dominate changes in value
(c) The hedge ratio is the is the same for both the:
 Hedging relationship
 Quantity of the hedged item actually hedged, and the quantity of the hedging
instrument used to hedge it.

163
Types of Hedging
1. Fair value hedge
It is the exposure to changes in fair value of recognized asset or liabilities
Accounting treatment
Hedge Instrument: Hedge instrument gain/loss will be charged to P&L (unless hedge item is
equity instrument measured at FVTOCI).
Hedge Item: Hedge item gain/loss will be charged to P&L (unless hedge item is equity
instrument at FVTOCI, then recognize in OCI)
2. Cash flow hedge
It is a hedge of the exposure to variability in cash flows that:
• Attributable to particular risk associated with recognized asset or liability or a high probable
forecast transaction
• Could affect profit or loss Accounting treatment Hedge Instrument:
• Gain or loss on hedge instrument that‗s determined to be an effective hedge must be
recognized in OCI (will become spate reserve in SOCIE) and ineffective portion will be charged
to P&L , and effective portion (separate reserve) will be reclassified to P&L when cash flows
expected to effect P&L
• If non financial asset recognized due to hedge item then reserve can be adjusted in initial cost
of non financial asset
Derivatives
Definition:
Any contract , that have three features: 1. Its initial cost is zero or nominal as compared to other
contracts that has similar response to changes in market value 2. It will be settled in future
3. Its value is dependent on certain underlying item
Derivative contracts
Types of derivative contract are as follows:
 Future contract
 Forward contract
 Options
 Swap contract
Accounting treatment:
Fair value through profit and loss
Initial measurement: Fair Value
Subsequent measurement: Fair value and gain/(loss) will be charged to P&L
Derivative Standing at gain; Financial Asset
Derivative Standing at loss; Financial Liability
Embedded Derivatives
An embedded derivative is a component of a hybrid contract that also includes a non-derivative host—
with the effect that some of the cash flows of the combined instrument vary in a way similar to a stand-
alone derivative.
Host contracts are the contracts in which derivative contracts are embedded.
Examples include:
(a) A lease
(b) A debt or equity instrument
(c) An insurance contract
(d) A sale or purchase contract

164
(e) A construction contract
A derivative that is attached to a financial instrument but is contractually transferable
independently of that instrument, or has a different counterparty, is not an embedded
derivative, but a separate financial instrument.
Measurement
Hybrid contracts with financial asset hosts
If a hybrid contract contains a host that is an asset within the scope of this IFRS, an entity shall
apply the measurement rules to the entire hybrid contract (recognized at FVTPL). E.g.
Investment in convertible loan, Investment in bond where interest rate vary with gold prices
Other hybrid contracts If host contact is a financial liability or non financial contracts E.g. Issue
of convertible loan, lease/construction contract in foreign currency
Criteria: (if met separation required)
• Economic characteristic should be different (host contract and derivative)
• Host contract should not be measured at FVTPL
• Embedded derivatives should meet the definition of ―stand alone derivative‖
IFRS 7 Financial instrument Disclosures
Objective
IFRS 7 requires entities to provide disclosures in their financial statements that enable users to
evaluate:
 The significance of financial instruments for the entity‗s financial position and
performance; and
 The nature and extent of risks arising from financial instruments to which the entity is
exposed during the period and at the reporting date, and how the entity manages those
risks.
Classes of financial instruments and level of disclosures
IFRS 7 requires that certain disclosures should be given by class of financial instruments. The
classes of financial instruments that will be disclosed should be appropriate to the nature of the
information disclosed and should take into account the characteristics of those financial
instruments. An entity should provide sufficient information to permit reconciliation to the line
items presented in the statement of financial position.
Statement of financial position:
 Financial assets measured at fair value through profit and loss, showing separately those
held for trading and those designated at initial recognition
 special disclosures about financial assets and financial liabilities designated to be
Measured at fair value through profit and loss, including disclosures about credit risk
and market risk, changes in fair values attributable to these risks and the methods of
measurement.
 reclassifications of financial instruments from one category to another (e.g. from fair
value to amortised cost or vice versa)
 information about financial assets pledged as collateral and about financial or non-
financial assets held as collateral
 reconciliation of the allowance account for credit losses (bad debts) by class of financial
Assets.
 information about compound financial instruments with multiple embedded derivatives
 breaches of terms of loan agreements Statement of profit or loss and other
comprehensive income

165
 Items of income, expense, gains, and losses, with separate disclosure of gains and losses
from financial assets measured at fair value through profit and loss, showing separately
those held for trading and those designated at initial recognition
 total interest income and total interest expense for those financial instruments that are
not measured at fair value through profit and loss
 fee income and expense
 amount of impairment losses by class of financial assets
 interest income on impaired financial assets

REVIEW QUESTIONS
QUESTION 1

On 1 January 2010, Johny issued GHC15m of 7% convertible loan notes at par. The loan notes
are convertible into equity shares in the company, at the option of the note holders, five years
after the date of issue (31 December 2014) on the basis of 25 shares for each GHC100 of loan
stock. Alternatively, the loan notes will be redeemed at par.
Johny has been advised by Fabiano Factors that, had the company issued similar loan notes
without the conversion rights, then it would have had to pay interest of 10%; the rate is thus
lower because the conversion rights are favourable.
Fabiano Factors also suggest that, as some of the loan note holders will choose to convert, the
loan notes are, in substance, equity and should be treated as such on Johny's statement of
financial position. Thus, as well as a reduced finance cost being achieved to boost profitability,
Johny's gearing has been improved compared to a straight issue of debt.
The present value of GHC1 receivable at the end of each year, based on discount rates of 7%
and 10% can be taken as:
End of year 7% 10%
1 0.93 0.91
2 0.87 0.83
3 0.82 0.75
4 0.76 0.68
5 0.71 0.62
Required
In relation to the 7% convertible loan notes, calculate the finance cost to be shown in the income
statement and the statement of financial position extracts for the year to 31 December 2010 for
Johny and comment on the advice from Fabiano Factors.
(5 marks)

Solution:
IAS 32 Financial instruments: presentation requires the issuer of a hybrid or compound
instrument of this nature – containing elements that are characteristic of both debt and equity –
to separate out the components of the instrument and classify them separately. Fabianio Factors
are thus wrong in their advice that such instruments should be recorded and shown as debt.
The proceeds of issue should be split between the amounts attributable to the conversion rights,
which are classed as equity, and the remainder which must be classed as a liability. Although
there are several methods that might be used, the question only gives sufficient information to
allow the amounts of debt liability to be calculated, leaving the equity element as the residual.

166
Year Cash flows Factor at 10% Present value
GHC'000 GHC'000
1 Interest (GHC15m × 7%) 1,050 0.91 955.5
2 1,050 0.83 871.5
3 1,050 0.75 787.5
4 1,050 0.68 714.0
5 Interest + capital 16,050 0.62 9,951.0
Total debt component 13,279.5
Proceeds of issue 15,000.0
Equity component (residual) 1,720.5

INCOME STATEMENT (EXTRACTS)


GHC'000
Interest paid ((7% × GHC15m) + 278 (W1)) 1,328

Working
((10% × GHC13.2795m) – GHC1.05m) (rounded) 278

STATEMENT OF FINANCIAL POSITION (EXTRACTS)


GHC'000
Non-current liabilities
7% convertible loan notes (13,279.5 + 278) 13,557.5
Equity Option to convert to equity 1,720.5

QUESTION 2
Pentagon issued a GHC10 million 3% convertible loan note at par on 1 April 2007 with interest
payable annually in arrears. Three years later, on 31 March 2010, the loan note is convertible
into equity shares on the basis of GHC100 of loan note for 25 equity shares or it may be
redeemed at par in cash at the option of the loan note holder. One of the company's financial
assistants observed that the use of a convertible loan note was preferable to a non-convertible
loan note as the latter would have required an interest rate of 8% in order to make it attractive
to investors. The assistant has also commented that the use of a convertible loan note will
improve the profit as a result of lower interest costs and, as it is likely that the loan note holders
will choose the equity option, the loan note can be classified as equity which will improve the
company's high gearing position.
The present value of GHC1 receivable at the end of the year, based on discount rates of 3% and
8% can be taken as:
3% 8%

End of year 1 0·97 0·93


2 0·94 0·86
3 0·92 0·79
Required
Comment on the financial assistant's observations and show how the convertible loan note
should be accounted for in Pentagon's income statement for the year ended 31 March 2008 and
statement of financial position as at that date. (10 marks)

167
Solution:
This convertible loan note is a compound financial instrument. It contains both a liability and an
equity component and IAS 32 Financial Instruments: Presentation requires these components to
be separately recognised. Interest costs on the liability element will be based on the non-
convertible rate of 8%, so the charge to the income statement will not be significantly lower than
if a non-convertible instrument were issued. The liability element will also add to gearing. So
the financial assistant's observations are incorrect.

Income statement
GHC
Finance costs (8,674,000 × 8%) 693,920

Statement of financial position


GHC
Equity – option to convert (W1) 1,326,000
Non-current liabilities
3% convertible loan note (W2) 9,067,920

Workings

1 Equity and liability elements GHC


Proceeds of loan note 10,000,000
3 years interest (10,000 × 3% × (0.93 + 0.86 + 0.79)) (774,000)
Redemption (10,000 × 0.79) (7,900,000)
Equity element of loan note 1,326,000
Liability element (10,000 – 1,326) 8,674,000

2 Loan note balance


GHC
Liability element 8,674,000
Interest at 8% 693,920
Less interest paid (300,000)
Carrying value at 31 March 2008 9,067,920

QUESTION 3
A company holds the following financial instruments. Explain how they should be accounted
for in accordance with IAS 32 and IAS 39 in the financial statements to 31 December,2013.
(i) A 3% bond was purchased on 1 January,2013 for GHC250,000. The nominal value is
GHC300,000 and redemption will be at par on 31 December,2016. The coupon is received
annually in arrears. The effective interest rate on the bond is 9.7%. The company has classified
the bond as ―held to maturity‖. The market value of the bond at 31 December,2013 is
GHC275,000.

(ii) An investment was made in the equity shares of XYZ. 3,000 shares were purchased (a 1%
stake) at a cost of GHC10 per share on 1 April,2011. A transaction fee of GHC300 was charged
on the purchase. The entity intended to sell the shares within three months and so classed the

168
investment as ―fair value through profit and loss‖. The market value of the investment
continued to rise and so the company decided not to sell in the near term. The market value of
the shares over the three years has been as follows:
GHC000
31 December Year 1 32
31 December Year 2 34
31 December Year 3 35

(iii) The company issued a convertible bond at par on 31 December,2013, raising GHC500,000.
The couponon the bond is 4%. The rate on an equivalent redeemable bond is 7%. The bond can
be redeemed at par on 31 December,2016 or converted into equity shares at a rate of five shares
per GHC100. The bond has not been classed as fair value through profit and loss.

Solution:

(i) 3% Bond The bond must initially be recorded at its purchase price of GHC250,000. If
classified as ‗held to maturity‘, it must be re-measured at the end of the reporting period to its
amortised cost. The market value is not relevant.
Interest will be credited to profit or loss using the effective interest rate, resulting in finance
income of GHC24,250 (9.7% × 250,000). The effective rate reflects the total return received by the
investor over the duration of the bond – being the coupon + GHC50,000 premium on
redemption.
The coupon recorded in the statement of cash flows is GHC9,000 (3% × 300,000). The difference
between the effective interest and the actual coupon is added to the investment to give an
amortised cost at the end of 2013 of GHC265,250 (250,000 + 24,250 – 9,000).

(ii) Equity shares in XYZ The shares will initially be recorded at their cost of GHC30,000. As
they have been classed as ‗fair value through profit or loss‘ the transaction costs must be
expensed to profit or loss immediately. At the end of each reporting period, the shares must be
re-measured to their market value, with the resulting gain or loss being taken to profit or loss.
At 1 January 2013, the investment has a carrying value of GHC34,000. By the 31 December 2013
this value is now GHC35,000. A GHC1,000 gain will therefore be recognised in profit or loss for
the year.
Even though the investment is no longer intended for sale in the short term, it must remain
classified as fair value through profit or loss as IAS 39 does not permit reclassification into or
out of this category.

(iii) Convertible bond


A convertible bond represents a compound instrument. In essence, issuing a convertible bond is
equivalent to issuing a non-convertible bond plus a call option on the entity‘s shares. Therefore,
the investment should be divided into a liability portion and an equity portion.
To establish the liability (debt) element, the future cash flows from the bond are discounted at
the normal market rate to establish the value of an equivalent but redeemable bond.
Using a rate of 7% this gives a net present value of:
( 20,000 /1.07)+ (20,000 /1.072)+(520,000/ 1.073) =GHC460,635

169
As the bond was issued for GHC500,000, it implies that the call option embedded within the
bond was sold for GHC39,365 (GHC500,000 – 460,635). Therefore, the proceeds of GHC500,000
will be shown in the statement of cash flows as a financing cash flow, and the credit will be split
into non- current liabilities GHC360,635 and equity GHC39,365.

QUESTION 4
Asamankese Ltd (Asamankese) purchased a 6% GH¢50 million bond on 1 August 2018 at a 10%
discount to par value. Expenses of purchase were GH¢500,000. The bond is due for redemption on
31 July 2028 at par. The effective annual interest rate to maturity is 7.3%. Asamankese intends to
hold the bond until its maturity date.
Required:
In accordance with IFRS 9: Financial Instruments, how much should be recognised in Asamankese
financial statements in respect of the above transaction for the year ended 31 July 2019 (to two
decimal places)? (5 marks)
SOLUTION 5
The initial carrying value of the bond will be as follows:
GH¢m
Purchase price (90% of GH¢50) 45
Add: Purchase costs 0.5
Total asset cost recognized 45.5
Finance income will be recognized @ 7.3% of the opening carrying value 3.32

Financial assets measured at amortized costs


Asset @ start Finance Income Interest Received Asset @ end
(7.3%) (6%)
GH¢ GH¢ GH¢ GH¢
2018/2019 45.5 3.322 (3) 45.8
Profit or Loss (Extract) for the year ended 31/7/2019
GH¢
Finance Income 3.322
Statement of Financial Position as at 31/7/2019 extract
GH¢
Financial assets 45.8
This bond meets the criteria for classifying it as Amortized Cost. These are

t to draw the contractual cash flows.

Hence the amortized cost method is appropriate. Fair value is irrelevant.


QUESTION 5
On 1 January 2018, Kaduna issued 10,000 bond instruments with a face value of GH¢100 at a market
price of GH¢95. Bond brokers charged fees totalling GH¢18,000 in relation to the bond issue. The
bonds carry a coupon rate of 5% and are redeemable in 3 years at face value.
Kaduna wishes to account for the bonds using IFRS 9: Financial Instruments amortised cost
method. However, there was some confusion about how the bonds should be accounted for.
Currently, the cash received from the bond issue of GH¢950,000 has been recognised as a non-
current liability. The broker fees of GH¢18,000 were deducted from the non-current liability carrying

170
amount, the coupon payment of GH¢50,000 has been expensed in arriving at profit before tax and the
effective rate of interest is 7.62%.
Required:
Justify the necessary accounting treatment of the above transaction relating to Kaduna Ltd for the
year ended 31 December 2018. (5 marks)
SOLUTIONS

Under IFRS 9‘s - amortised cost model, the financial liabilities should initially be recognised at
the fair value of the cash received. Any directly attributable costs to the bond issue should be
offset against the carrying amount of the liability.
Therefore, at initial recognition the bonds should be measured at:
GH¢
Fair value of consideration received 950,000
Less: transaction costs (18,000)
Carrying amount 932,000
The initial recognition and measurement of the financial liabilities has been carried out correctly
in Kaduna‘s financial statements.
The journal entires are
Dr Cash 950,0000
Cr Financial Liability 932,000
Cr Broker Fees Payable (Transaction costs) 18,000
However, IFRS 9‘s amortised cost method requires the bonds to be amortised over the bond
term using the effective interest rate.
The instrument cash flows are:
GH¢
CF0 932,000
CF1 (50,000)
CF2 (50,000)
CF3 (1,050,000)
The effective interest rate which discounts back the future cash flows to the instrument‘s
present value is 7.62% (rounded to 2 decimal places).
The debt amortisation schedule for the debt, using the effective interest rate, is shown below:
Year Opening Coupon Finance cost Difference: Closing
capital Payment charged @ to capital capital
balance 7.62% sum balance
0 932,000
1 932,000 (50,000) 71,018 21,018 953,018
2 953,018 (50,000) 72,620 22,620 975,638
3 975,638 (1,050,000) 74,344 24,344 (18)
rounding
Kaduna has failed to amortise the bond using the effective interest rate, instead expensing
the coupon payment in the profit or loss. Therefore, the following correcting journal entry is
required:
Dr Interest expense (P&L) GH¢21,018
Cr Non-current liability (SOFP) GH¢21,018

QUESTION 6

171
Duakwanta is a listed company which manufactures personal computers (PCs). It is preparing its
financial statements for the year ended 31 May 2019 and would like to seek advise on the following
accounting issue:
During the year, Duakwanta issued a debt finance to the financial markets to fund its expansion
plans. This was a very significant debt issue for Duakwanta. After the issue, the market price of each
block of debt on the market fell by approximately 10%. The financial press has stated that the reason
for the fall is due to an increase in the company's credit risk, as the market players are worried by the
size of the interest payments on Duakwanta's operating cash flows.
Required:
Advise the directors as to the financial reporting issues arising from the above scenario and explain
the appropriate treatment in Duakwanta‟s financial statements. (4 marks)
SOLUTION
The debt issue must be accounted for under IFRS 9 Financial Instruments. Initial recognition is
at fair value. However this would ordinarily be the amount of cash received.

ordinarily be held at amortised cost.

adjusted in Duakwanta's financial statements. However, it will be disclosed under IFRS 7


Financial Instruments: Disclosures.

QUESTION 7
Plyway Ltd (Plyway) was founded in 2001 and operates in the logistics and supply chain
management sector. It prepares financial statements in accordance with International Financial
Reporting Standards (IFRSs) up to 31 December each year.

On 1 January 2017, Plyway purchased a GH¢50 million social bond from a renowned finance
house. The bond pays a respectable 6% annual interest, which is also the effective rate of
interest payable on 31 December. Plyway has classified the bond at fair value through profit or
loss.

At 31 December 2019, the carrying value of the bond is GH¢50 million but there are reliable
reports within the financial sector that the finance house has financial difficulties. The market
interest is now 8%. The cash flows in respect of the bond are as follows:

Year Amount GH¢m


31 December 2019 4 31
December 2020 3.5
31 December 2021 42.5
Interest received for the accounting year ended 31 December 2019 has already been correctly
accounted for.

Required: In accordance with IFRS 9: Financial Instruments, show with appropriate


calculations, the accounting entries required to record the transaction above in the financial
statements of Plyway for the year ended 31 December 2019. (6 marks)
SOLUTION

172
As the bond is classified at fair value through the profit and loss, the fair value can be calculated
by discounting expected future cash flows using the current market interest rate. Any change in
fair value is taken to the profit and loss accounting for the current year.

Date flow 8% GH¢‘000


GH¢‘000
31/12/19 4,000 1.0000 4,000
31/12/20 3,500 0.9259 3,240
31/12/21 42,500 0.8573 36,435
43,675
Fall in FV charged to profit and loss 6,325
Carrying value of financial asset 50,000

GH¢million GH¢million
Cr Financial Assets (Statement of Financial Position) 6.325
Dr Retained earnings (P & L) 6.325

TRY
Illustration 1
on 1 January 2001 Obuasi Co. purchase a debt instrument for its fair value of
GH¢1,000. The debt instrument is due to mature on 31December 2005. The instrument
has principal amount of GH¢1,250 and the instrument carries fixed interest at 4.72%
that is paid annually. The effective interest rate is 10%.
Obuasi has classified the instrument as held-to-maturity.
How should Obuasi Co. account for the debt instrument over its five-year term?

Illustration 2
A company issues 3% bonds with a nominal value of GH¢150,000. The loan notes are
issued at a discount of 10% and issue cost of GH¢11,455 are incurred. The loan notes
will be repayable at a premium of GH¢10,000 after 4years. The effective rate of interest
is 10%
Required
what amount will be recorded as financial liability when the loan note is issued.

Question 3 (Nov 2017)


DanKay Ltd bought a ten-year bond on 1 August 2016 at a cost of GH¢45 million. The
bond carries an interest coupon of GH¢5 million paid annually in arrears, and its
effective yield to maturity was 12% at the date of purchase. Dankay Ltd is holding the
bond as a speculative investment, expecting its value to increase, and hopes to sell the
bond at a profit in the short to medium term. On 31 July 2017, its reporting date, the fair
value of the bond had declined to GH¢43 million. The interest payment was received as
scheduled. 


Required: 


173
Advise DanKay Ltd on the treatment of the above in the financial statements for the
year ended 31 July 2017 in accordance with IFRS 9: Financial Instruments. (4 marks) 

Solu
As the bond is not to be held to maturity it fails the “Business Model” test set out 
by
IFRS 9 Financial Instruments. 

This means the amortised cost method cannot be used, and the fair value method

must be used instead. 

This results in a fair value loss of GH¢2 million, and a carrying value of GH¢43

million. 

The interest received of GH¢5 million is recognised as a gain in profit or loss. This
results in a net gain of GH¢3 million to profit or loss.

Question 4
Abiba Limited is a company operating in Northern Ghana and provides loans to
customers and funds the loans by selling bonds in the market. The financial liability is
designated as fair value through profit or loss. The bonds have a fair value increase of
GH¢100 million in the year to 31 December 2015 of which GH¢5 million relates to the
reduction in Abiba‟s creditworthiness. The directors of Abiba Ltd have contacted your
consultancy firm for your advice on how to account for this movement.
Required:
Discuss, with appropriate computations where necessary, the accounting treatment of the above
transactions in the financial statements of Abiba Ltd for the year ended 31 December 2015. (Nov
2016)
Solu
IFRS 9 requires gains and losses on financial liabilities designated as at fair value through profit
or loss such as Abiba‟s Ltd to be split into the amount of change in the fair value which is
attributable to changes in the credit risk of the liability, which is shown in other comprehensive
income, and the remaining amount of the change in the fair value of the financial liability which
is shown in the statement of profit or loss.
IFRS 9 allows the recognition of the full amount of
change in the fair value in the statement of profit or loss only if the recognition of changes in the
liability‟s credit risk in other comprehensive income would create an accounting mismatch in
profit or loss.
This is determined at initial recognition and is not reassessed. Amounts presented in other
comprehensive income are not subsequently transferred to profit or loss, and the entity may only
transfer the cumulative gain or loss within equity. As a result, Abiba Ltd should charge GHȻ5
million to OCI and GHȻ95 million to statement of profit or loss.

Question 4
Kantanka Ltd holds a portfolio of equity investments the value of which was correctly
recorded at GH¢12 million on 1 April 2016. During the year ended 31 March 2017, the
company received dividends of GH¢0.75 million. Further equity investments were
purchased at a cost of GH¢1.6 million. Shares were disposed of during the year for

174
proceeds of GH¢1.1 million. These shares had cost GH¢0.4 million a number of years
earlier but had been valued at GH¢0.9 million on 1 April 2016. The fair value of the
financial assets held on 31 March 2017 was GH¢14 million. (5 marks)
Required:
Advise Kantanka Ltd on how to account for the above transactions in accordance with relevant
accounting standards. (may 2017)
(ii) Dividends received are recognised as income regardless of the treatment of the financial
assets.
Journal entry to record dividends received:
Dr Cash
 GH¢m 0.75
Cr Profit or loss GH¢m 0.75
Journal entry to record purchase of investments:
Dr Financial assets
 GH¢m 1.6
Cr Cash GH¢m 1.6
Remeasurements are treated in accordance with the policy of the entity. We must assume that the
irrevocable election required by IFRS 9 was made as this is the policy of Kantanka Ltd.
Journal entry to record remeasurement and disposal:
Dr Financial assets (1.1 – 0.9)
 GH¢m 0.2
Cr Other comprehensive income GH¢m 0.2

Dr Cash
 GH¢m 1.1


Cr Financial assets GH¢m 1.1

The assets held at the period end must be remeasured to GH¢14 million. These are already
carried at GH¢12.7 million (12.0 – 0.9 + 1.6). The original carrying value included GH¢0.9
relating to the investments sold, so these are no longer there. In addition, new assets costing
GH¢1.6 million were purchased.

The fair value of these remaining assets on 31 March 2017 was 14 million, hence a gain of
GH¢1.3 million (14 – 12.7) must be recognised.
Journal entry to record remeasurement at 31 March 2017:
Dr Financial assets
 GH¢m 1.3
Cr Other comprehensive income GH¢m 1.3

Question 5 (Nov 2015 / may 2018)


Alfa Limited issued a GH¢5,000,000 18% convertible loan note at par on 1 July 2015
with interest payable annually in arrears. Three years later, on 30 June 2018, the loan
note becomes convertible into equity shares on the basis of GH¢100 of loan note for 50
equity shares or it may be redeemed at par in cash at the option of the loan note holder.
The Financial Accountant of Alfa Limited has observed that the use of a convertible loan
note was preferable to a non-convertible loan note as the latter would have required an
interest rate of 24% in order to make it attractive to investors. 


175
The present value of GH¢1 receivable at the end of the year, based on discount rates of
18% and 24% can be taken as:

Year 18% 24%


1 0.847 0.806
2 0.718 0.650
3 0.609 0.524

Required:
(i) Show the accounting treatments for the convertible loan note in Alfa Limited‟s: i)
income statement for the years ended 30 June 2016, 2017 and 2018; and (3
marks)
ii) the statement of financial position as at 30 June 2016, 2017 and 2018. (4 marks)
(Note: Assume that the share option is taken at the end of June 30, 2018.)

Question 6
Stev Limited issued a GH¢80,000,000 15% convertible loan note at par on 1 January
2015 with interest payable annually in arrears. Three years later, on 31 December 2017,
the loan note becomes convertible into equity shares on the basis of GH¢100 of loan
note for 50 equity shares or it may be redeemed at par in cash at the option of the loan
note holder. The financial accountant of Stev Limited has observed that the use of a
convertible loan note was preferable to a non-convertible loan note as the latter would
have required an interest rate of 20% in order to make it attractive to investors. The
accountant has also commented that the use of a convertible loan note will improve the
profit as a result of lower interest costs and, as it is likely that the loan note holders will
choose the equity option, the loan note can be classified as equity which improve the
company high gearing position.
The present value of GH¢1 receivable at the end of the year, based on discount rates of
18% and 24% can be taken as:
Year 15% 20%
1 0.870 0.833
2 0.756 0.694
3 0.658 0.579

Required:
1. Discuss the validity or otherwise of the financial accountant comment (2marks)
2. Show how the convertible loan note should be accounted for in Stev Ltd income
statement for the year ended 31 December 2017 and statement of financial position as
at 31 December 2017

Practice question: A company issued a convertible bond for GH¢2,000,000 on 1 January 20X5.
The bond is to be redeemed on 31 December 20X7 (3 years after issue).
The bond holders can take cash or shares with a nominal value of GH¢1,200,000 on this date.
The bond pays interest at 5% but the market rate of interest for similar risk bonds without the

176
conversion feature was 9% at the date of issue.
a) Calculate the liability and equity components of the bond on initial recognition.
b) Construct the necessary journal on initial recognition.
c) Construct an amortisation table to show how the liability component would be measured over
the life of the bond.
d) Construct the journal to reflect the possible conversion of the bonds to shares on 31 December
20X7.

Question 7 nov 2018


Garu-Tempane Ltd had the following transaction during the year ended 31 December 2018:
The entity entered into a contractual commitment to make a variable rate loan to a customer
beginning on 1 January 2019 for a fixed period at 1% less than the rate at which the entity (not
the customer) can borrow money.
Required:
Advise the directors of Garu-Tempane Ltd of the accounting treatment of the above transaction
under IFRS 9: Financial Instruments for the year ended 31 December 2018. (4 marks)

SOLUTION
Commitment to provide a loan at less than market interest rates
The commitment to provide a loan to the customer appears to have been entered into at
less than market interest rates, given that the rate is lower than that at which the entity
can borrow money on the market. A commitment to provide a loan at less than market
interest rates represents a financial liability. This is because the entity has an obligation
to pay more interest on financing the loan than it will receive from the customer.
(2 marks)
The commitment was entered into during 2019 and is initially measured at its fair value.
Interest income is recognised on this amount using the effective interest method.
(0.5 mark)
At the year end, the liability is increased to its IAS 37 provision valuation ('the best
estimate of the expenditure required to settle the present obligation') if this amount
exceeds the amount of the initial fair value recognised less amounts already amortised to
profit or loss using the effective interest rate. (1 mark)
Any difference is charged to profit or loss. (0.5 mark)(4 marks)

MAY 2019
Chereponi Ltd (Chereponi) is a listed manufacturing company. Chereponi granted a
loan of GH¢25 million to a homeless charity for the building of a community centre. The
loan was granted on 1 January 2017 and is repayable on maturity in four years‟ time.
Interest, which is subsidised, is to be charged one year in arrears at 4%, but Chereponi
assesses that a normal rate for such a loan would have been 8%. Chereponi recorded a
financial asset at GH¢25 million and reduced this by the interest received each year.
Required:
In accordance with IFRS 9: Financial Instruments, recommend with justification the required
accounting treatment for the issue of the loan to the homeless charity in the financial statements
of Chereponi for the year ended 31 December 2017. (6 marks)

177
Charity Bond (IFRS 9)
The fair value of the bond is determined by calculating the present value of all future
cash receipts using the prevailing market interest rate for a similar financial instrument.
This will result in a lower figure than the amount advance. The difference is recognised
in profit or loss.
Cash flows Discount factor PV
GH¢m 8% GH¢m
2017 1 0.93 0.93
2018 1 0.86 0.86
2019 1 0.79 0.79
2020 26 0.74 19.24
21.82
The fair value of the loan is calculated by scheduling the cash flows due to take place
over the life of the loan and discounting them to present value at the unsubsidised rate
of interest of 8%. The making of the loan should have been accounted for as:
GH¢m GH¢m
Dr Financial assets 21.8
Dr Profit or loss 3.2
Cr Cash 25
The asset is then held at amortised cost.
1 January 2017 Interest rate Cash received 31
December2017
GH¢m (8%) GH¢m GH¢m
21.8 1.7 (1) 22.5
Correcting entries:
GH¢m GH¢m
Dr SPLOCI 3.2
Cr Financial asset 3.2
Dr Financial asset 1.7
Cr SPLOCI 1.7

Q10
On 1 January 2018, Kaduna issued 10,000 bond instruments with a face value of
GH¢100 at a market price of GH¢95. Bond brokers charged fees totalling GH¢18,000 in
relation to the bond issue. The bonds carry a coupon rate of 5% and are redeemable in
3 years at face value.

Kaduna wishes to account for the bonds using IFRS 9: Financial Instruments amortised
cost method. However, there was some confusion about how the bonds should be
accounted for. Currently, the cash received from the bond issue of GH¢950,000 has
been recognised as a non-current liability. The broker fees of GH¢18,000 were
deducted from the noncurrent liability carrying amount, the coupon payment of

178
GH¢50,000 has been expensed in arriving at profit before tax and the effective rate of
interest is 7.62%.

Required: Justify the necessary accounting treatment of the above transaction relating
to Kaduna Ltd for the year ended 31 December 2018.

Solu
Under IFRS 9‟s - amortised cost model, the financial liabilities should initially be recognised at
the fair value of the cash received. Any directly attributable costs to the bond issue should be
offset against the carrying amount of the liability.

Therefore, at initial recognition the bonds should be measured at:


GH¢
Fair value of consideration received 950,000
Less: transaction costs (18,000)
Carrying amount 932,000

The initial recognition and measurement of the financial liabilities has been carried out correctly
in Kaduna‟s financial statements.

The journal entires are

Dr Cash 950,0000
Cr Financial Liability 932,000
Cr Broker Fees Payable (Transaction costs) 18,000

However, IFRS 9‟s amortised cost method requires the bonds to be amortised over the bond term
using the effective interest rate.

The instrument cash flows are:


GH¢
CF0 932,000
CF1 (50,000)
CF2 (50,000)
CF3 (1,050,000)
The effective interest rate which discounts back the future cash flows to the instrument‟s present
value is 7.62% (rounded to 2 decimal places).

The debt amortisation schedule for the debt, using the effective interest rate, is shown below:
Year Opening Coupon Finance cost Difference: Closing
capital Payment charged @ 7.62% to capital capital balance
balance
0 932,000
1 932,000 (50,000) 71,018 21,018 953,018
2 953,018 (50,000) 72,620 22,620 975,638
3 975,638 (1,050,000) 74,344 24,344 (18) rounding

179
Kaduna has failed to amortise the bond using the effective interest rate, instead expensing the
coupon payment in the profit or loss. Therefore, the following correcting journal entry is
required:

Dr Interest expense (P&L) GH¢21,018


Cr Non-current liability (SOFP) GH¢21,018

Q11
Asamankese Ltd (Asamankese) purchased a 6% GH¢50 million bond on 1 August 2018 at a 10%
discount to par value. Expenses of purchase were GH¢500,000. The bond is due for redemption
on 31 July 2028 at par. The effective annual interest rate to maturity is 7.3%. Asamankese
intends to hold the bond until its maturity date.

Required: In accordance with IFRS 9: Financial Instruments, how much should be recognised in
Asamankese financial statements in respect of the above transaction for the year ended 31 July
2019 (to two decimal places)? (5 marks)
SOLUTION
The initial carrying value of the bond will be as follows:
GH¢m
Purchase price (90% of GH¢50) 45
Add: Purchase costs 0.5
Total asset cost recognized 45.5

Finance income will be recognized @ 7.3% of the opening carrying value 3.32

Financial assets measured at amortized costs


Asset @ start Finance Income (7.3%) Interest Received (6%) Asset @ end
GH¢ GH¢ GH¢ GH¢
2018/2019 45.5 3.322 (3) 45.8

Profit or Loss (Extract) for the year ended 31/7/2019 GH¢


Finance Income 3.322

Statement of Financial Position as at 31/7/2019 extract GH¢


Financial assets 45.8

This bond meets the criteria for classifying it as Amortized Cost. These are

amortized cost method is appropriate. Fair value is irrelevant.

180
EARNINGS PER SHARE (IAS 33)
Earnings are profits available for equity (ordinary shareholders). Earnings per share (EPS) is a
measure of the amount of earnings in a financial period for each equity share. As its name
implies, EPS is calculated as reported earnings divided by the number of ordinary shares in
issue.
EPS = Total earnings/Number of ordinary shares
EPS is used by investors as a measure of the performance of companies in which they invest –
or might possibly invest. Investors are usually interested in changes in a company‘s EPS over
time – trends – and also in the size of EPS relative to the current market price of the company‘s
shares.

The price/earnings ratio


The price/earnings ratio (P/E ratio) is a key stock market ratio. It is a measure of the company‘s
current share price (market price) in relation to the EPS. The P/E ratio is calculated as follows.
P/E ratio = Market value of share
EPS
The P/E ratio can be used by investors to assess whether the shares of a company appear
expensive or cheap. A high P/E ratio usually indicates that the stock market expects strong
performance from the company in the future and investors are therefore prepared to pay a high
multiple of historical earnings to buy the shares.

EPS and IAS 33


Stock market investors monitor P/E ratios and EPS very closely. They compare the P/E ratios of
different companies, and growth or decline in their EPS over time.
Their investment decisions may be made on the basis of these comparisons.
EPS should therefore be calculated by all companies in a standard way, so that investors can
obtain a reliable comparison between the EPS and P/E ratios of different companies. The rules
for calculating EPS are set out in IAS 33 Earnings per share.
The concept of EPS is quite simple. It is simply the amount of profit earned in the previous
financial year for each ordinary share in issue. However, there are some complexities with the
calculation of EPS, particularly when:
 new shares were issued during the year, or
 there is the possibility of a ‗dilution‘ in the EPS in the future, due to the existence of
‗potential ordinary shares‘.

Preference shares and EPS

181
Preference shares are not ordinary shares. Since EPS is a measure of earnings per ordinary share
in a financial year, preference shares are excluded from the number of shares.
The dividends paid to preference shareholders must therefore be excluded from the total
earnings for the period. A broad definition of ‗earnings‘ is therefore profit after tax less
preference dividends paid.

Objective and scope of IAS 33 and definitions


Objective of IAS 33
The objective of IAS 33 is to set out principles for:
 the calculation of EPS, and
 the presentation of EPS in the financial statements.
The purpose of standardising the calculation and presentation of EPS is to make it easier for the
users of financial statements to compare the performance of:
 different entities in the same reporting period, and
 the same entity for different reporting periods over time.

Scope of IAS 33
IAS 33 applies only to publicly-traded entities or those which are about to be publicly traded.
A publicly-traded entity is an entity whose shares are traded by the investing public, for
example on a stock exchange.
Most publicly-traded entities prepare consolidated financial statements as well as individual
financial statements. When this is the case, IAS 33 requires disclosure only of EPS based on the
figures in the consolidated financial statements.

Definitions
An ordinary share is an equity instrument that is subordinate to all other classes of equity
instruments.
The ordinary shares used in the EPS calculation are those entitled to the residual profits of the
entity, after dividends relating to all other shares have been paid. As stated earlier, if you are
given an examination question on this topic, preference shares are not ordinary shares because
they give more rights to their holders than ordinary shares.
A potential ordinary share is a financial instrument or other contract that may entitle its holder
to ordinary shares at some time in the future. The following examples of potential ordinary
shares are given by IAS 33:
 financial liabilities or equity instruments that are convertible into new ordinary shares at
some time in the future (convertible debentures, convertible preference shares)
 share options and warrants. Options and warrants are financial instruments that give
the holder the right (but not the obligation) to purchase new ordinary shares at some
time in the future, at a fixed price
 shares that will be issued if certain contractual conditions are met, such as contractual
conditions relating to the purchase of a business.

Basic and diluted earnings per share


IAS 33 requires entities to calculate:
 the basic earnings per share on its continuing operations
 the diluted earnings per share on its continuing operations.

182
Additional requirements apply to earnings relating to discontinued operations.
Diluted EPS and basic EPS will usually differ when there are potential ordinary shares in
existence.

EARNINGS PER SHARE AS A PERFORMANCE MEASURE


Earnings per share and trends
Investors and their advisers pay close attention to an entity‘s net profit for the period. However,
profit for the period can include large and unusual items and also the results of discontinued
operations. This may make it volatile: liable to fluctuate rapidly up and down. Users can then
find it difficult to assess trends in the profit figure or to use the current year‘s profit to predict
an entity‘s performance in future years.
The trend (improvement or deterioration) in an entity‘s published EPS figure can sometimes be
a more reliable indicator of future performance. There are a number of reasons for this.
 The standard version of both basic and diluted EPS is based on profit from continuing
operations. This means that the results of discontinued operations (which may distort
total profit) are excluded.
 An entity may also choose to present one or more alternative versions of EPS.
These normally exclude large or unusual items so that EPS is based on ‗normal‘ recurring
earnings.
 EPS measures an entity‘s performance from the viewpoint of investors. It shows the
amount of earnings available to each ordinary shareholder. This means that EPS takes
the effect of preference dividends (if any) into account. It also takes share issues into
account.
 Diluted EPS can provide an ‗early warning‘ of any changes to an investor‘s potential
return on their investment due to future share issues.

LIMITATIONS OF EARNINGS PER SHARE


EPS is probably the single most important indicator of an entity‘s performance. It is a very
useful measure when it is used as the starting point for a more detailed analysis of an entity‘s
performance.
However, EPS can have serious limitations:
 Not all entities use the same accounting policies. It may not always be possible to make
meaningful comparisons between the EPS of different entities.
 EPS does not take account of inflation, so that growth in EPS over time might be
misleading.
 EPS measures an entity‘s profitability, but this is only part of an entity‘s overall
performance. An entity‘s cash flow can be just as important as its profit (and more
essential to its immediate survival). Changes in the value of assets (holding gains) can
also be an important part of performance for some entities.
 Diluted EPS is often described as an ‗early warning‘ to investors that the return on their
investment may fall sometime in the future. However, diluted EPS is based on current
earnings, not forecast earnings. This means that it may not be a reliable predictor of
future EPS.
One of the main problems with EPS can be the way that it is used by investors and others. Users
often rely on EPS as the main or only measure of an entity‘s performance. Management know
this and try to make EPS appear as high as possible. They may attempt to manipulate the figure
by using ‗creative accounting‘.

183
They may also make decisions which increase EPS in the short term but which damage the
entity in the longer term

CALCULATING BASIC EPS


The calculation: total earnings
Basic earnings per share is calculated by dividing the profit or loss on continuing operations by
the weighted average number of ordinary shares in issue during the period.
The calculation of the basic EPS is therefore as follows.
EPS = Net profit (or loss) attributable to ordinary shareholders during a period
Weighted average number of ordinary shares in issue during the period
The net profit (or loss) attributable to ordinary shareholders during a period is commonly
referred to as ‗total earnings‘. Total earnings are calculated as:
 the profit or loss from continuing operations
 after deducting tax and preference dividends (and in the case of consolidated financial
statements, after excluding the earnings attributable to non-controlling interests or
minority interests).
Total earnings include any income from associates (i.e. any share of profits of associates).
Where preference dividends are cumulative, they should be deducted from total earnings
whether the dividend has actually been paid or not.
When there is a net loss, total earnings and the EPS are negative.
Earnings from discontinued operations are dealt with separately. An EPS from any
discontinued operations must also be disclosed, but this does not have to be disclosed on the
face of the statement of comprehensive income. Instead, it may be shown in a note to the
financial statements.

Example
In the year ended 31 December 2011, Godfred Ltd made profit after tax of GHC350,000. Of this,
GHC300,000 was from continuing operations and GHC50,000 from discontinued operations. It
paid ordinary dividends of GHC150,000 and preference dividends of GHC65,000. Throughout
the year it had 1 million ordinary shares in issue.
Required
Calculate the basic EPS for Godfred Ltd for the year ended 31 December 2011.
Answer:
EPS=Net profit (loss) attributable to ordinary shareholders during a period
Weighted average number of ordinary shares in issue during the period
= GHC300,000 - GHC65,000
1,000,000
= GHC0.235 or 23.5pesewas
Number of shares: issue of shares at full market price
The calculation of the basic EPS is more complex when new shares have been issued during
the financial year.
Shares issued during a financial period should be included in the calculation of the weighted
average number of shares. They should be included from the date that the consideration
(payment from the shareholder) is received for the shares. This is because it is only from that
date that the new shares are able to contribute to earnings (through investing the cash or other
assets obtained as the proceeds from
the share issue).

184
The starting point for the weighted average number of shares is the number of shares in issue at
the beginning of the period. This is then adjusted for any shares issued during the period, to
which a time weighting factor must be applied.
For example, if an entity with a financial year ending 31 December issues new shares on 1
October (= 9 months into the year), the number of new shares should be adjusted by a factor of
× 3/12.

Example
Hernandez has a financial year ending 31 December. On 1 January 2011 there were 6,000,000
ordinary shares in issue. On 1 April, it issued 1,000,000 new shares at full market price. Total
earnings in 2011 were GHC2,700,000.
Required
What was the EPS in 2011?
Answer
There are two methods of calculating the weighted average number of shares.
Method 1
Weighted average number
Shares at 1 January 2011 6,000,000
Shares issued on 1 April 1,000,000 × 9/12 750,000
––––––––
6,750,000
––––––––
EPS = GHC2,700,000/6,750,000 shares = GHC0.40 or 40pesewas.
Method 2
With the following method, a weighted average number is calculated from each new issue of
shares. A time factor is applied to the total number of shares after the new share issue. The time
factor to apply is either:
 for the remaining number of months to the year end, or
 for the number of months to the next share issue, if this is sooner.
This method produces exactly the same result as Method 1.
Date Detail Number of Shares Time factor Weighted
Average shares
1 January Brought forward 6,000,000 × 3/12 1,500,000
1 April Issue at full market price 1,000,000
––––––––
31 December Carried forward 7,000,000 × 9/12 5,250,000
–––––––– ––––––––
6,750,000
––––––––
EPS = GHC2,700,000/6,750,000 shares = GHC0.40 or 40pesewas.
Example
Jeremiah Plc has a financial year ending 31 December. On 1 January 2013, there were 9,000,000
ordinary shares in issue. On 1 May, Jeremiah Plc issued 1,200,000 new shares at full market
price. On 1 October, it issued a further 1,800,000 shares, also at full market price. Total earnings
in 2013 were GHC3,690,000.

185
Required
Calculate the EPS for the year to 31 December 2013.

Answer
Date Detail Number of shares Time factor Weighted
average number
1 January Brought forward 9,000,000 × 4/12 3,000,000
1 May Issue at full market price 1,200,000
–––––––––
Shares after the new issue 10,200,000 × 5/12 4,250,000
1 October Issue at full market price 1,800,000
–––––––––
31 Dec Carried forward 12,000,000 × 3/12 3,000,000
––––––––– –––––––––
10,250,000
–––––––––
Notes
(1) The first new share issue is in May, after 4 months. Therefore the number of shares at the
beginning of the year is given a time factor of × 4/12.
(2) There are 5 months between the two share issues, therefore the time factor to apply to the
number of shares after the first issue is × 5/12.
(3) The total number of shares in issue from 1 October to the end of the year (three months) is
12,000,000. These are given a time weighting of × 3/12.
EPS = GHC3,690,000/10,250,000 = GHC0.36.

BONUS ISSUES OF SHARES


A bonus issue of shares is also called a scrip issue or a capitalisation issue. It is an issue of new
shares to existing shareholders, in proportion to their existing shareholding, for no
consideration. In other words, the new shares are issued ‗free of charge‘ to existing
shareholders. The new shares are created by converting equity reserves in statement of financial
position, often some or all of the share premium account, into ordinary share capital.
When there is a bonus issue of shares, the situation is different from a new issue of shares at full
market price. With a bonus issue of shares, no cash is raised from the issue because the new
shares are issued ‗free‘. Therefore the new shares do nothing to generate additional
profits/earnings.
Unless a suitable adjustment is made to the EPS calculation, the comparison of EPS in the
current year (after the bonus issue) with EPS in the previous year (before the bonus issue) will
be misleading.
In order to ensure that the EPS in the year of the bonus issue is comparable with the previous
year‘s EPS, IAS 33 requires that the weighted average number of shares should be calculated as
if the bonus shares had always been in issue. IAS 33 states that: ‗The number of ordinary shares
outstanding before the [bonus issue] is adjusted for the proportionate number of shares
outstanding as if the [bonus issue] has occurred at the beginning of the earliest period
presented..‘ This means that:
 the current period‘s shares are adjusted as if the bonus shares were issued on the first
day of the year

186
 the comparative EPS for the previous year is restated on the same basis.
Example
Hazard Plc made a 1 for 4 bonus issue on 1 July 2015. Its financial year ends on 31 December.
The financial results for Hazard Plc in 2014 and 2015 were as follows.

Year 5 Year 4
Total earnings GHC2,100,000 GHC2,000,000
Number of shares in issue at 31 December 5,000,000 4,000,000

Answer:
The basic EPS could be calculated for the 2015 financial statements as follows, by taking as the
number of shares for the current period and the previous period the total number of shares after
the bonus issue.
Year 5 Year 4
Total earnings GHC2,100,000 GHC2,000,000
Number of shares 5,000,000 5,000,000
= GHC0.42 = GHC0.40

Another way of calculating the weighted average number of shares in the current year would
be:
Date Number of Bonus Time Weighted
shares fraction factor average number
1 Jan Bfwd 4,000,000 × 5/4 × 6/12 2,500,000
1 July Bonus issue 1,000,000
Shares after the
new issue 5,000,000 × 6/12 2,500,000
5,000,000

Comparative EPS for previous year


In the 2014 financial statements, the EPS would have been reported as GHC2,000,000 /4,000,000
= GHC0.50, because there were only 4 million shares in issue in 2014
In 2015, we can calculate the comparative EPS for 2014 (for the purpose of the 2015 financial
statements) by adjusting the original 2014 EPS by the following factor:

EPS reported last year × Number of shares before the bonus issue
Number of shares after the bonus issue

In this example, for every 4 shares in issue before the bonus issue, there are 5 in issue after the
bonus issue. The comparative EPS for 2014, for inclusion in the financial statements for 2015,
would therefore be:
GHC0.50 × 4/5 = GHC0.40
The calculation of the weighted average number of shares can be complicated when there is
another issue of shares for cash during the year, in addition to a bonus issue. In such cases, it is
recommended that you use the Method 2 described above, as follows.

Example
Johnson Ltd had 2,000,000 ordinary shares in issue on 1 January 2012.

187
On 1 April 2012, it issued 500,000 ordinary shares, at full market price.
On 1 July 2012, there was a 1 for 2 bonus issue (= one new bonus share for every two shares
held).
In 2011, the EPS had been calculated as GHC0.25.
The financial year ends on 31 December. In 2011, the EPS had been calculated as GHC0.25. In
2012, total earnings were GHC855,000.
Required
Calculate the EPS for the year to 31 December 2012, and the comparative EPS figure for 2011.

Answer:
The weighted average number of shares in 2012 is calculated as follows.
In every line in the table before the bonus issue, the number of shares is increased by a factor (a
‗bonus fraction‘) to allow for the bonus issue.

The factor to apply is:


× Number of shares after the bonus issue
Number of shares before the bonus issue

With a 1 for 2 bonus issue, the bonus issue adjustment fraction is × 3/2. This is because for
every 2 shares in issue before the bonus issue, there are 3 shares after the bonus issue.

Date Detail Number of Time Bonus Weighted


Shares factor fraction average no. of shares
1 Jan Brought forward 2,000,000 × 3/12 × 3/2 750,000
1 April Issue at market
price 500,000
––––––––
2,500,000 × 3/12 × 3/2 937,500
1 July Bonus issue (1 for 2)1,250,000
31 Dec Carried forward 3,750,000 × 6/12 1,875,000
3,562,500

The weighted average number of shares for the EPS calculation for Year 2 is therefore 3,562,500.
EPS in 2012 = GHC855,000/3,562,500 = GHC0.24.
To restate the comparative EPS figure for 2011, the original EPS figure of GHC0.25 is adjusted
by a factor × 2/3, to allow for the 1 for 2 bonus issue.

The 2011 EPS is therefore re-stated as: GHC0.25 × 2/3 = GHC0.1667.

RIGHTS ISSUES OF SHARES


A rights issue of shares is an issue of new shares for cash, where the new shares are offered
initially to current shareholders in proportion to their existing shareholdings. The issue price of
the new shares in a rights issue is always below the current market price for the shares already
in issue.
Because rights issues are usually at less than full market price they include a bonus element.
Since there is a bonus element in the issue price, an adjustment is needed for the EPS
calculation, to ensure a fair comparison of the current year EPS with the previous year EPS.

188
Current year EPS
To adjust for the bonus element in the rights issue, the number of shares in the current financial
year must be adjusted. You should multiply the number shares in issue before the rights issue
by a factor (a ‗rights factor) of:
Actual cum rights price
Theoretical ex rights price

Theoretical ex-rights price is explained and illustrated later.


Previous year EPS
In addition, to provide a fair comparison of the current year EPS with the previous year‘s EPS,
the comparative EPS for the previous year is obtained by adjusting the EPS actually reported
last year. The previous year‘s EPS is reduced by the following factor:
Theoretical ex rights price
Actual cum rights price

The actual cum-rights price is the market price of the shares before the rights issue.
The theoretical ex-rights price is the price that the shares ought to be, in theory,after the rights
issue. It is a weighted average price of the shares before the rights issue and the new shares in
the rights issue. If you are not sure how to calculate the theoretical ex-rights price, study the
following example carefully.

Example
Lazio Plc had 36,000,000 shares in issue on 1 January 2012. It made a 1 for 4 rights issue on 1
June 2012, at a price of GHC4 per share. The share price just before the rights issue was GHC5.
Total earnings in the financial year to 31 December 2012 were GHC25,125,000. The reported EPS
in 2011 was GHC0.64.
Required
Calculate the EPS for the year to 31 December 2012, and the adjusted EPS for 2011 for
comparative purposes.

Answer
After the rights issue, there will be 1 new share for every 4 shares previously in issue
GHC
4 existing shares have a ‗cum rights‘ value of (4 × GHC5) 20.00
1 new share is issued for 4.00
5 shares after the issue have a theoretical value of 24.00
Theoretical ex-rights price = GHC24.00/5 = GHC4.80.

Date Number of Time factor Rights Weighted


Shares fraction average shares
1 Jan Brought forward 36,000,000 × 5/12 × 5.00/4.80 15,625,000
1 June Rights issue (1 for 4) 9,000,000
–––––––––
31 Dec Carried forward 45,000,000 × 7/12 26,250,000
–––––––––
41,875,000

189
–––––––––
EPS 2012 = GHC25,125,000/41,875,000 = GHC0.60, or 60pesewas
Comparative EPS in 2011 = GHC0.64 × (GHC4.80/GHC5.00) = GHC0.6144 or 61.44p

Example
Maxwell Plc had 3 million ordinary shares in issue on 1 January 2017. On 1 April 2017, it made a
1 for 2 rights issue of 1,500,000 ordinary shares at GHC2 per share. The market price of the
shares prior to the rights issue was GHC5.
An issue of 400,000 shares at full market price was then made on 1 August 2017.
In the year to 31 December 2017, total earnings were GHC1,746,875. In 2016 EPS had been
reported as GHC0.35.
Required
Calculate the EPS for the year to 31 December 2017, and the adjusted EPS for 2016 for
comparative purposes.

Answer
After the rights issue, there will be 1 new share for every 2 shares previously in issue
GHC
2 existing shares have a ‗cum rights‘ value of (2 × GHC5) 10.00
1 new share is issued for 2.00
–––––
3 shares after the issue have a theoretical value of 12.00
–––––
Theoretical ex-rights price = GHC12/3 = GHC4.

Date Number of Time factor Rights Weighted


Shares fraction average shares
1 Jan Brought forward 3,000,000 × 3/12 × 5/4 937,500
1 April Rights issue (1 for 2) 1,500,000
––––––––
4,500,000 × 4/12 1,500,000
1 August Issue at market price 400,000
––––––––
4,900,000 × 5/12 2,041,667
––––––––
4,479,167
––––––––
EPS 2017 = GHC1,746,875/4,479,167 = GHC0.39, or 39pesewas
EPS 2016 = GHC0.35 × 4.00/5.00 = GHC0.28 or 28pesewas.

DILUTED EPS
The meaning of dilution
An entity might have some potential ordinary shares. When there are potential ordinary
shares, there is a possibility that they will become actual ordinary shares at some time in the
future.
For example, if an entity has issued some convertible bonds or convertible preference shares,
these might be converted into ordinary shares at some time in the future. Similarly, holders of

190
share options or warrants might exercise their right at a future date to subscribe for new shares
at a fixed price.
If potential shares become actual ordinary shares, there is a possibility – perhaps a probability –
that the EPS will be reduced because there will be a larger number of ordinary shares in issue.
Dilution will occur if the EPS is reduced as a consequence of potential ordinary shares
becoming actual ordinary shares. (Note: ‗Dilution‘ means ‗watering down‘ or ‗reduction in
strength‘.)

IAS 33 and diluted EPS


IAS 33 requires publicly-traded companies to calculate a diluted EPS in addition to their basic
EPS for the current year (with a comparative diluted EPS for the previous year), allowing for the
effects of all dilutive potential ordinary shares.
Note: potential ordinary shares are ‗dilutive‘ when there might have been a reduction or
‗dilution‘ in EPS if they had been actual ordinary shares during the financial period.
Potential ordinary shares might not dilute the EPS. The diluted EPS should allow only for
potential ordinary shares that would be dilutive.
Earnings should be adjusted to allow for the dividends or interest that have been recognised
during the year for the potential ordinary shares, and for any other income or expense that
would alter as a result of the conversion of the potential ordinary shares into actual ordinary
shares.
The main items of dividend or interest to adjust for are dividends on convertible preference
shares and interest on convertible debentures (convertible bonds). The dividend or interest
reduces total earnings. However, if they had already been converted into ordinary shares (and
the calculation of diluted EPS is based on this assumption) the dividends or interest would not
have been payable. Total earnings would therefore have been higher. To calculate the diluted
EPS, total earnings are adjusted to allow for this.
The weighted average number of shares must also be adjusted. The method of making this
adjustment is different for:
 convertible bonds or convertible preference shares and
 share options or warrants.

Diluted EPS: convertible preference shares and convertible bonds


When there are convertible bonds or convertible preference shares, diluted EPS is calculated as
follows, by making adjustments to total earnings and the number of shares in issue.

Basic assumption in calculating diluted EPS


It is assumed that all the convertible securities were converted into ordinary shares on the first
day of the financial period, and on the most favourable terms available for the holders of the
convertibles.

Total earnings
Total earnings must be adjusted. This is because the entity would not have to pay the dividend
or interest on the convertible securities.
 For convertible preference shares, add back the preference dividend paid in the year. If
the preference shares are converted, this dividend will no longer be paid.
Total earnings will be increased by the preference dividend saved.

191
Adjusted total earnings = Actual total earnings + Convertible preference dividend
 For convertible bonds, add back the interest charge on the bonds in the year minus the
tax relief relating to that interest. If the bonds are converted, this interest will no longer
be paid. Total profits will increase by the interest saved,but total earnings will increase
only by the interest saved less tax.
Adjusted total earnings = Actual total earnings + (Convertible bond interest – Tax on the
interest).

Number of shares
The weighted average number of shares should be increased, by adding the maximum number
of new shares that would be created if all the potential ordinary shares were converted into
actual ordinary shares. The additional number of shares should normally be calculated on the
assumption that they were in issue at the beginning of the year.

Example
A company has 12,000,000 ordinary shares in issue and GHC4 million of 5% convertible bonds.
As at 31 December 2012, there have been no new issues of shares or bonds for several years. The
bonds are convertible into ordinary shares in 2013 or 2014,at the following rates:
 At 31 December 2013: 30 shares for every GHC100 of bonds
 At 31 December 2014: 25 shares for every GHC100 of bonds.
Total earnings for the year to 31 December 2012 were GHC3,600,000. Total earnings for the
previous year (2011) were GHC3,300,000. Tax is payable at a rate of 30% on profits.
Required
Calculate the basic EPS and diluted EPS for 2012, and the comparative figures for 2011 (for
reporting in the 2012 financial statements).

Answer
Basic EPS:
(1) Year to 31 December 2012: GHC3,600,000/12 million = GHC0.30
(2) Year to 31 December 2011: GHC3,300,000/12 million = GHC0.275.

Diluted EPS:
2012 2011
GHC GHC
Actual earnings 3,600,000 3,300,000
Add back convertible bond interest (5% × GHC4 million) 200,000 200,000 Minus tax at 30%
(60,000) (60,000)
–––––––– ––––––––
Adjusted total earnings 3,740,000 3,440,000
–––––––– ––––––––
Number of shares:
Actual 12,000,000 12,000,000
Potential ( 4 million × 30/100) 1,200,000 1,200,000
––––––––– –––––––––
13,200,000 13,200,000
––––––––– –––––––––

192
Diluted EPS GHC0.2833 GHC0.2606
Note: The number of potential shares is calculated using the conversion rate of 30 shares for
every GHC100 of bonds, because this conversion rate produces more new shares than the other
conversion rate, 25 shares for every $100 of bonds.
New issue of convertibles in the year
If new convertibles are issued during the course of the year, the additional number of shares
should be calculated only from the time that the convertibles were issued, and a time factor
should be applied to calculate a weighted average number of shares for the year.

Example
Neymar Plc has 10,000,000 ordinary shares in issue. There has been no new issue of shares for
several years. However, the entity issued GHC2,000,000 of convertible 6% bonds on 1 April
2015. These are convertible into ordinary shares at the following rates:
On 31 March 2020 25 shares for every GHC100 of bonds
On 31 March 2021 20 shares for every GHC100 of bonds
On 31 March 2022 15 shares for every GHC100 of bonds
On 31 March 2023 10 shares for every GHC100 of bonds
Tax is at the rate of 30%.
In the financial year to 31 December 2016, total earnings were GHC4,536,000. In the previous
financial year to 31 December 2015, total earnings were GHC4,087,000.
Required
Calculate the figures that should be reported in the financial statements for the year to 31
December
2016:
(a) the EPS for Year 6 and the comparative EPS figure for 2015
(b) the diluted EPS for 2016 and the comparative diluted EPS figure for 2015.

Answer
(a) Basic EPS
2016 = GHC4,536,000/10,000,000 = GHC0.4536
2015 = GHC4,087,000/10,000,000 = GHC0.4087.
(b) Diluted EPS
Adjusted total earnings 2016 2015
GHC GHC GHC GHC
Reported earnings 4,536,000 4,087,000
Add back interest saved
(GHC2,000,000 × 6%)
:(9/12 × GHC2,000,000 × 6%): 120,000 90,000
Minus tax at 30% (36,000) (27,000)
84,000 63,000
Adjusted total earnings 4,620,000 4,150,000
Number of shares
The maximum number of new ordinary shares that might be created if the potential ordinary
shares are converted is 25 new shares for every GHC100 bonds. In a full year, this represents
GHC2,000,000 × 25/100 = 500,000.
However, the bonds were issued during 2016; therefore a time factor should be applied to
calculate the ‗diluted‘ number of shares in 2016.

193
2017 Number of shares
1 January Brought forward 10,000,000
Dilution 500,000
–––––––––
31 December 10,500,000
–––––––––
2016
Number Time factor Weighted average
of shares number of shares
1 Jan Brought forward 10,000,000 × 3/12 2,500,000
1 April Bond issue: dilution 500,000
–––––––––
10,500,000 × 9/12 7,875,000
––––––––– –––––––––
10,375,000
–––––––––
Diluted EPS, 2017 = GHC4,620,000/10,500,000 = GHC0.44
Diluted EPS, 2016 = GHC4,150,000/10,375,000 = GHC0.40.
DILUTED EPS: OPTIONS AND WARRANTS
A different situation applies with share options and share warrants. If these are exercised, the
holders of the options or warrants will pay cash to obtain new ordinary shares.
 If the options or warrants are exercised, the entity will receive cash that it can invest to
increase earnings. However, since the options or warrants have not yet been exercised,
it is impossible to predict how total earnings will be affected when the cash is
eventually received.
 The exercise price for the options or warrants will be less than the full market price for
the shares. This means that there will be a bonus element in the issue.
These factors complicate the calculation of diluted EPS, but IAS 33 has provided a solution, as
follows.
 Calculate the number of shares that will be issued if all the options or warrants are
exercised.
 Reduce this figure by the number of shares that could be purchased at full market price
with the cash received from the exercise of the options or warrants.
The market price of the shares should be the average market price during the period (the
financial year).
 This net figure is then added to the existing number of shares in issue, to obtain the total
shares for calculating the diluted EPS.

Example
Practical Plc had total earnings during 2013 of GHC1,030,000.
It has 5,000,000 ordinary shares in issue. There are outstanding share options on 400,000 shares,
which can be exercised at a future date, at an exercise price of GHC2.50 per share. The average
market price of shares in Practical Plc during 2013 was GHC4.
Required
Calculate the diluted EPS for 2013.
Answer
Cash receivable on exercise of all the options = 400,000 × GHC2.50 = GHC1,000,000.

194
Number of shares this would buy at full market price in
2013 = GHC1,000,000/GHC4 = 250,000 shares.
Shares
Options 400,000
Minus number of shares at fair value (250,000)
––––––––
Net dilution 150,000
Existing shares in issue 5,000,000
––––––––
Total shares 5,150,000
––––––––
Diluted EPS = GHC1,030,000/5,150,000 = GHC0.20 or 20pesewas.

POTENTIAL ORDINARY SHARES THAT ARE NOT DILUTIVE


If potential ordinary shares are not dilutive, they should be excluded from the calculation of the
diluted EPS.
When there are several types of potential ordinary share in issue, they should be ranked in
order of ‗dilution, with the most dilutive potential ordinary shares ranked first.
A diluted EPS should then be calculated in stages, taking in one potential ordinary share at a
time, to establish whether any of them are not dilutive. An example might help to illustrate the
technique to use.

Example
The following information relates to Quincy Plc for the year ended 31 December 2015.
Number of ordinary shares in issue 10,000,000
Reported earnings in the year GHC1,500,000
Average market price of shares during the year GHC8

Potential ordinary shares:


Options
600,000 options, with an exercise price of GHC6

4% convertible bond:
GHC5,000,000 Each bond is convertible in 2020 into ordinary shares at the rate of 40 new shares
for every GHC100 of bonds

1,000,000 7% convertible preference shares of GHC1 each


Each preference share is convertible in 2019 into ordinary shares at the rate of 1 ordinary share
for every 2 preference shares

Tax rate = 30%


Required
Calculate the diluted EPS for the year to 31 December 2015.
Answer
If all the options are exercised, the cash received will be 600,000 × GHC6 = GHC3,600,000. This
would purchase 450,000 shares (GHC3,600,000/GHC8) at the average market price in 2015. The
dilutive increase in the number of shares would therefore be (600,000 – 450,000) = 150,000.

195
Increase in Increase in Earnings Ranking
earnings number of per extra in order of
ordinary shares new share dilutiveness
GHC GHC
Options 01 50,000 0.00 1st
Convertible bonds 140,000 2,000,000 0.07 2nd
(4% × GHC5,000,000)
less tax 30%
Preference shares 70,000 500,000 0.14 3rd
(7% × GHC1,000,000)
Taking these three dilutive potential ordinary shares in order of their ranking, we need to
calculate the diluted EPS as follows.
Earnings Number of EPS
shares
GHC GHC
As reported, basic EPS 1,500,000 10,000,000 0.1500
Options 0 150,000
–––––––– –––––––––
Diluted EPS, options only 1,500,000 10,150,000 0.1478 Dilutive
Convertible bonds 140,000 2,000,000
Diluted EPS, options
and convertible bonds 1,640,000 12,150,000 0.1350 Dilutive
Convertible preference
shares 70,000 500,000
–––––––– –––––––––

Diluted EPS, options


and all convertibles 1,710,000 12,650,000 0.1352 Not dilutive
The convertible preference shares are not dilutive, and the reported diluted EPS should be GHC0.1350
(and not GHC0.1352)

IAS 33: PRESENTATION AND DISCLOSURE REQUIREMENTS


IAS 33: Presentation requirements
An entity should present in the statement of comprehensive income:
 the basic EPS and
 the diluted EPS
 for the profit or loss from continuing operations.
For consolidated accounts, this is the EPS and diluted EPS attributable the owners of the parent
company.
The basic EPS and diluted EPS should be presented with equal prominence for all the periods
presented (the current year and the previous year). These figures are presented at the end of the
statement of comprehensive income.
If the entity presents a separate statement for the components of profit or loss (i.e. a separate
income statement):
 the EPS and diluted EPS should be shown in this statement, and
 not in the statement of comprehensive income.

196
If there is a discontinued operation, the basic EPS and diluted EPS from discontinued operation
should be shown either on the face of the statement of comprehensive income or in a note to the
financial statements.
The basic and the diluted EPS should be presented, even if it is a negative figure (= even if it is a
loss per share).

IAS 33: Disclosure requirements


IAS 33 also requires disclosure in a note to the financial statements of the following:
 The total amounts used as the numerators (= total earnings figures) to calculate the basic
EPS and diluted EPS, and a reconciliation of these numerator figures to the profit or loss
for the period
 The total amounts used in the denominators (= weighted average number of shares) to
calculate the basic EPS and diluted EPS, and a reconciliation of these two denominator
figures to each other.
Alternative measures of earnings per share
IAS 33 allows an entity to disclose an alternative measure of EPS in addition to the EPS
calculated in accordance with IAS 33. For example, EPS could be calculated after adjusting
earnings for large and unusual items.
If an alternative EPS figure is presented, IAS 33 states that:
 a reconciliation must be shown between the earnings figure used in the alternative
measure and the amounts shown in the statement of comprehensive income
 the alternative EPS must use the same weighted average number of shares as the

IAS 33 calculation
 basic and diluted EPS should both be disclosed with equal prominence, and
 the alternative figure must only be shown in the notes, not on the face of the statement
of comprehensive income.

REVIEW QUESTIONS
QUESTION 1
On 1 January 2015, Mary had 5 million ordinary shares in issue. The following transactions in
shares took place during the next year.
1 February A 1 for 5 bonus issue
1 April A 1 for 2 rights issue at GHC1 per share. The market price of the
shares prior to the rights issue was GHC4.
1 June An issue at full market price of 800,000 shares.
In 2015 Mary made a profit before tax of GHC3,362,000. It paid ordinary dividends of
GHC1,200,000 and preference dividends of GHC800,000. Tax was GHC600,500. The reported
EPS for 2014 was GHC0.32.
Required
Calculate the EPS for 2015, and the adjusted EPS for 2014 for comparative purposes.

Solution:
GHC
2 existing shares have a cum rights value of (2 × GHC4) 8
1 new share is issued for 1

197
––
3 new shares have a theoretical value of 9
––
Theoretical ex-rights prices = GHC9/3 = GHC3

Date Number Time Bonus Rights Weighted


average
Shares factor fraction fraction no. of shares
1 January Brought forward 5,000,000 1/12 6/5 4/3 666,667
1 February Bonus issue
(1 for 5) 1,000,000
–––––––––
6,000,000 2/12 4/3 1,333,333
1 April Rights issue (1 for 2) 3,000,000
–––––––––
9,000,000 2/12 1,500,000
1 June Issue at full market
price 800,000
–––––––––
31 December Carried forward 9,800,000 7/12 5,716,667
––––––––––
9,216,667
Earnings for 2015 are (3,362,000 – 600,500 – 800,000) GHC1,961,500

EPS 2015 = 1,961,500/9,216,667 = GHC0.21 or 21pesewas


EPS 2014 (adjusted) = GHC0.32 × 3/4 × 5/6 = GHC0.20 or 20pesewas

QUESTION 2
Magdalene has had 5 million shares in issue for many years. Earnings for the year ended 31
December 2014 were GHC2,579,000. Earnings for the year ended 31 December 2013 were
GHC1,979,000. Tax is at the rate of 30%.
Outstanding share options on 500,000 shares have also existed for a number of years. These can
be exercised at a future date at a price of GHC3 per share. The average market price of shares in
2013 was GHC4 and in 2014 was GHC5.
On 1 April Year 3 Magdalene issued GHC1,000,000 convertible 7% bonds. These are convertible
into ordinary shares at the following rates.
On 31 December 2016 30 shares for every GHC100 of bonds
On 31 December 2017 25 shares for every GHC100 of bonds
On 31 December 2018 20 shares for every GHC100 of bonds
Required
Calculate the diluted EPS for 2014 and the comparative diluted EPS for 2013.

Solution:
Adjusted total earnings
2014 2013
GHC GHC GHC GHC

198
Reported earnings 2,579,000 1,979,000
Add back interest saved
(1,000,000 × 7%)
(1,000,000 × 7% × 9/12) 70,000 52,500
Minus tax at 30% (21,000) (15,750)
––––––– 49,000 ––––––– 36,750
Adjusted total earnings 2,628,000 2,015,750
–––––––– ––––––––
Number of shares
2014 Number of shares
1 January Brought forward 5,000,000
Dilutions: Share options (W) 200,000
Convertible shares (1,000,000 ÷ 100 × 30) 300,000
––––––––––
31 December 5,500,000
––––––––––
2013
Date Number of Time Weighted average
shares factor number of shares
1 January Brought forward 5,000,000
Share options: dilution (W) 125,000
–––––––––
5,125,000 3/12 1,281,250
1 April Convertibles: dilution 300,000
–––––––––
5,425,000 9/12 4,068,750
––––––––– ––––––––––
5,350,000
––––––––––
Diluted EPS
2014 = 2,628,000/5,500,000 = GHC0.48 or 48pesewas
201 3 = 2,015,750/5,350,000 = GHC0.38 or 38pesewas

Working
Cash receivable on exercise of all the options = 500,000 × GHC3 = GHC1,500,000

2014
Number of shares this would buy at full market price in 2014 = GHC1,500,000/5 = 300,000
shares

Shares
Options 500,000
Minus number of shares at fair value (300,000)
––––––––
Net dilution 200,000
––––––––
2013

199
Number of shares this would buy at full market price in 2013 = GHC1,500,000/4 = 375,000
shares
Shares
Options 500,000
Minus number of shares at fair value (375,000)
––––––––
Net dilution 125,000
––––––––
QUESTION 3
(a) Fenton had 5,000,000 ordinary shares in issue on 1 January 2011.
On 31 January 2011, the company made a rights issue of 1 for 4 at GHC1.75. The cum rights
price was GHC2 per share.
On 30 June 2011, the company made an issue at full market price of 125,000 shares.
Finally, on 30 November 2011, the company made a 1 for 10 bonus issue.
Profit for the year was GHC2,900,000.
The reported EPS for year ended 31 December 2010 was 46.4pesewas.
Required
What was the earnings per share figure for year ended 31 December 2011 and the restated EPS
for year ended 31 December 2010?

(b) Sinbad had the same 10 million ordinary shares in issue on both 1 January 2011 and 31
December 2011.On 1 January 2011 the company issued 1,200,000 GHC1 units of 5% convertible
loan stock. Each unit of stock is convertible into 4 ordinary shares on 1 January 2019 at the
option of the holder. The following is an extract from Sinbad's income statement for the year
ended 31 December 2011:
GHC'000
Profit before interest and tax 980
Interest payable on 5% convertible loan stock (60)
Profit before tax 920
Income tax expense (at 30%) (276)
Profit for the year 644
Required
What was the basic and diluted earnings per share for the year ended 31 December 2011?

(c) Talbot has in issue 5,000,000 50pesewas ordinary shares throughout 2013.
During 2011 the company had given certain senior executives options over 400,000 shares
exercisable at GHC1.10 at any time after 31 May 2014. None were exercised during 2013. The
average market value of one ordinary share during the period was GHC1.60. Talbot had made a
profit after tax of GHC540,000 in 2013.

Required
What is the basic and diluted earnings per share for the year ended 31 December 2013?

Solution:
(a)
Date Narrative Shares Time Rights Bonus Weighted
factor fraction fraction average

200
1,1/11 b/d 5,000,000 × 1/12 × 2.00/1.95 × 11/10 470,085
31/1/11Rights issue +1,250,000
6,250,000 × 5/12 × 11/10 2,864,583
30/6/11 FMP + 125,000
6,375,000 × 5/12 × 11/10 2,921,875
30/11/11Bonus issue +637,500
7,012,500 × 1/12 584,375
6,840,918

TERP :
4 @ 2 = 8.00
1 @ 1.75 = 1.75
5 9.75
∴ 1.95

EPS for y/e 31/12/11 = GHC2,900,000


6,840,918
= GHC0.424

Restated EPS for y/e 31/12/10 = 0.464 × 1.95× 10/11


2.00
=GHC 0.411
(b) Sinbad
Basic EPS = GHC644,000
10,000,000
= 6.44

Earnings GHC
Profit for the year 644,000
Interest saving (1,200,000 @ 5% × 70%) 42,000
686,000
Number of shares
Basic 10,000,000
On conversion 4,800,000
14,800,000

Diluted EPS =GHC686,000


14,800,000
=GHC0.464

(c) Talbot
Basic EPS = 540,000
5,000,000
=GHC 0.108

Diluted EPS:
Consideration on exercise 400,000 × GHC1.10 = GHC440,000

201
Shares acquired at FV GHC440,000/GHC1.60 = 275,000

∴shares issued for no consideration (400,000 – 275,000) = 125,000


EPS = 540,000
5,000,000 + 125,000
= GHC0.105

QUESTION 4
(a) The issued share capital of Savior, a publicly listed company, at 31 March 2013 was GHC10
million. Its shares are denominated at 25 pesewas each. Savior‘s earnings attributable to its
ordinary shareholders for the year ended 31 March 2013 were also GHC10 million, giving an
earnings per share of 25 pesewas.
Year ended 31 March 2014
On 1 July 2013 Savior issued eight million ordinary shares at full market value. On 1 January
2014 a bonus issue of one new ordinary share for every four ordinary shares held was made.
Earnings attributable to ordinary shareholders for the year ended 31 March 2014 were
GHC13,800,000.
Year ended 31 March 2015
On 1 October 2014 Savior made a rights issue of shares of two new ordinary shares at a price of
GHC1·00 each for every five ordinary shares held. The offer was fully subscribed. The market
price of Savior's ordinary shares immediately prior to the offer was GHC2·40 each. Earnings
attributable to ordinary shareholders for the year ended 31 March 2015 were GHC19,500,000.
Required
Calculate Savior's earnings per share for the years ended 31 March 2014 and 2015 including
comparative figures. (9 marks)
(b) On 1 April 2015 Savoir issued GHC20 million 8% convertible loan stock at par. The terms of
conversion (on 1 April 2018) are that for every GHC100 of loan stock, 50 ordinary shares will be
issued at the option of loan stockholders. Alternatively the loan stock will be redeemed at par
for cash. Also on 1 April 2015 the directors of Savior were awarded share options on 12 million
ordinary shares exercisable from 1 April 2018 at GHC1·50 per share. The average market value
of Savior's ordinary shares for the year ended 31 March 2016 was GHC2·50 each. The income
tax rate is 25%. Earnings attributable to ordinary shareholders for the year ended 31 March 2016
were GHC25,200,000. The share options have been correctly recorded in the income statement.
Required
Calculate Savior's basic and diluted earnings per share for the year ended 31 March 2016
(comparative figures are not required).
You may assume that both the convertible loan stock and the directors' options are dilutive. (4
marks)
(Total = 13 marks)

Solution:
(a) Year ended 31 March 2014
Bonus Weighted

202
Date Narrative Shares Time fraction average
1/4/13 Opening 40,000,000 × 3/12 × 5/4 12,500,000
1/7/13 Full market price 8,000,000
48,000,000 x 6/12 × 5/4 30,000,000
1/1/14 Bonus issue (1/4) 12,000,000
60,000,000 x 3/12 15,000,000
57,500,000

Earnings GHC13.8m, therefore EPS = 13.8/57.5 = GHC0.24

Comparative
The EPS for 2013 would be restated to allow for the dilutive effect of the bonus issue as follows:
0.25 × 48/60* = GHC0.20
* Existing shares + new issue = 48 Existing shares + new issue + bonus issue = 60

Note that the 8m shares issued at full price are non-dilutive and are therefore added to both
sides of the fraction.
Year ended 31 March 2015
'2 for 5' rights issue takes place halfway through the year and results in 24m additional shares.

Weighted average number of shares calculated as follows:


Bonus Weighted
Date Narrative Shares Time factor fraction average
1/4/14 Opening 60,000,000 x 6/12 2.4/2 (W) 36,000,000
30/9/14 Rights issue (2/5) 24,000,000
84,000,000 x 6/12 42,000,000
78,000,000

Earnings GHC19.5m, therefore EPS = 19.5/78 = GHC0.25

Comparative
The EPS for 2014 is now restated following the rights issue in October 2014 as follows:
GHC0.24× Theoretical ex-rights price (W)/Market price = 0.24 × 2/2.40 =GHC 0.20

Working GHC
Theoretical ex-rights price
5 shares at market price (5 × 2.4) 5 @ GHC2.4 12
2 shares at GHC1 2 @ GHC1 2
7 14
∴ Theoretical ex-rights price = GHC14/7 = GHC2

(b) Basic EPS = GHC25.2m/84m =GHC0. 30

Diluted EPS:
Shares Earnings
m GHCm

203
Existing 84.0 25.2
Loan stock 10.0 1.2 (W1)
Share options 4.8 (W2) –
98.8 26.4
EPS = 26.4/98.8 = GHC0.267

Workings
1. Loan stock GHCm
When conversion takes place there will be a saving of:
Interest (20m × 8%) 1.6
Less tax (1.6 × 25%) (0.4)
1.2
2. Share options
Shares issued will be 12m @ GHC1.50 = GHC18m.
At market price of GHC2.50 the value would be GHC30m.
The shortfall is GHC12m, which is equivalent to 4.8m shares at market price
PRACTICE QUESTIONS
Activity 1
Kalba Ltd.‟s earning for the year ended 31 December 2014 are GH¢2,208,000. On 1 January
2014, the issued share capital of Kalba Ltd was 9,200,000 6% preference shares of GH¢1 each
and 8,280,000 ordinary shares of GH¢1 each. The company issued 3,312,000 shares at full
market value on 30 June 2014.
Calculate the EPS for Kalba Ltd. for 2014.
Activity 2
Alif had four million ordinary shares in issue and ranking for dividend at 1 January 2015. On 30
September, one million further shares were issued. Earnings for the year ended 31 December
2015 were GH¢500,000.
Calculate the earnings per share.

Activity 3
Balara has four million ordinary shares in issue at 1 January 2015. On 30 September the entity
made a bonus issue of 1 for 4. Earnings for the year ended 31 December 2015 were
GH¢500,000. The EPS for 2014 was 9 pesewas per share.
Calculate the basic earnings per share and comparative figure for 2014.
Activity 4
A company issued a one new share for every two existing shares held by way of rights shares
held by the way of right issue at GH¢1.50 per share on 1 July 2016. Pre-issue market price was 3
per share.
Relevant information
2016 2015
Profit after tax 550,000 460,000
Number of ordinary shares issue at 31st December 1,200,000 800,000
Show the calculation of basic EPS to be presented in the financial statements for the year ended
31 December 2016 (including the comparative figure).
Activity 5

204
On 31 December 2015, the issued share capital consisted of 4,000,000 ordinary shares of no par
value issued at 25Gp each, and the shares were quoted at GH¢1. On 1 July 2016 the company
made a right issue at a proportion of 1 for 4 shares at 50Gp per share. Its trading results for the
last two years were as follows:
Year ended 31 December
2015 2016
Profit after tax 320,000 425,000
Show the calculation of basic EPS to be presented in the financial statements for the year ended
31 December 2016 (including the comparative figure).
Activity 6
Sagar Ltd. Had basic earnings of 105Gp based on earnings of GH¢ 105,000 and 100,000
ordinary shares. It also had in issue, GH¢40,000 15% convertible loan stock which is convertible
in two years‟ time at a rate of four ordinary shares for every GH¢5 of stock. The rate of tax is
30%. In 2007 profit before tax of GH¢150,000 was recorded.
Required: Calculate the diluted earnings per share.

Activity 7
Saamanbor Ltd has 5,000,000 ordinary shares in issue and also had issue in 2014: GH¢1,000,000
of 14% convertible loan stock convertible in three years‟ time at the rate of 2 shares of every
GH¢10 of stock.GH¢2,000,000 of 10% convertible loan stock convertible in one year‟s time at
the rate of 3 shares for every GH¢5 of stock. The total earning in 2014 was GH¢1,750,000. The
rate of income tax is 35%.
Calculate the diluted earnings per share.

QUESTION 8 . NOV 2019 TRY


A parent company had 6 million, GH¢1 fully paid ordinary shares outstanding on 1 January
2018. On 1 April 2018 the company made a consolidation of existing shares in issue (i.e. a
reverse share split) at nominal value, on a 2 for 3 basis. There was no special dividend, share
repurchase or other outflow of resources.

Having completed the consolidation of shares, a new share issue for 600,000 shares was made
through an offer for sale at the market price of GH¢1.55 per share. The allotment was made on 1
September 2018 and the proceeds were due on 1 October 2018.
The company's (summarised) statement of profit or loss for the year ended 31 December 2018 as
published showed:
GH¢'000
Revenue 15,740
Cost of sales and expenses (16,060)
Loss before interest and tax (320)
Finance costs 68
Taxation 60
Loss for the year (448)

Profit/(loss) attributable to:


Owners of the parent (478)
Non-controlling interests 30

205
(448)

The company also had in issue GH¢500,000 of 5% cumulative redeemable preferences shares
throughout the year ended 31 December 2018.

Required:
In accordance with IAS 33: Earnings per Share calculate the basic earnings per share figure for
the year ended 31 December 2018.

SIGNIFICANCE OF EARNINGS PER SHARE (EPS)


EPS gives a way to measure a company‟s profits relative to the number of shares 
in issue. It
is argued that as owners hold equity shares, it is more relevant to them to know how much profit
each share has earned than to know the overall profit figure. 

EPS feeds into the price / earnings ratio, one of the most important stock market measures of
value. This gives an estimate of the number of years it would take for an investment in an equity
share to return itself in earnings terms, assuming current performance continues into the future. 

It is essential that such an important measure of performance have clear guidelines regarding
its calculation. IAS 33 Earnings per Share gives us a standardised method of calculating both
earnings, and the number of shares. 

Many investors feel that other measures are more appropriate, and that the IAS 33 definition
of EPS is too conservative. IAS 33 allows alternative measures of EPS to be published, as long
as the IAS 33 figure gets equal or greater prominence. 

There is a danger in relying on a single measure of performance, as no single measure can
encapsulate all aspects of an entity‟s performance. 

Also, there is a danger that EPS may be seen by unsophisticated investors as a definite exact
number, when in reality it is subject to all the accounting estimates and judgments that are
necessary in preparing a set of financial statements. 

Despite these fears, it is generally agreed that IAS 33 gives a very fair method of calculating
EPS, and that the consistency it offers is of value to the investor and analyst. 


LIMITATIONS OF EARNINGS PER SHARE


EPS is probably the single most important indicator of an entity‟s performance. It is a very useful
measure when it is used as the starting point for a more detailed analysis of an entity‟s
performance.
However, EPS can have serious limitations:
 Not all entities use the same accounting policies. It may not always be possible to make
meaningful comparisons between the EPS of different entities.
 EPS does not take account of inflation, so that growth in EPS over time might be misleading.
 EPS measures an entity‟s profitability, but this is only part of an entity‟s overall performance. An
entity‟s cash flow can be just as important as its profit (and more essential to its immediate
survival). Changes in the value of assets (holding gains) can also be an important part of
performance for some entities.
 Diluted EPS is often described as an „early warning‟ to investors that the return on their
investment may fall sometime in the future. However, diluted EPS is based on current earnings,
not forecast earnings. This means that it may not be a reliable predictor of future EPS.

206
One of the main problems with EPS can be the way that it is used by investors and others. Users
often rely on EPS as the main or only measure of an entity‟s performance. Management know
this and try to make EPS appear as high as possible. They may attempt to manipulate the figure
by using „creative accounting‟.
They may also make decisions which increase EPS in the short term but which damage the entity
in the longer term

IMPAIRMENT OF ASSETS (IAS 36)


OBJECTIVE
The objective of this IAS is to set rules to ensure that the assets of an enterprise are carried at no
more than their recoverable amount.

DEFINITIONS
Recoverable amount is the higher of an asset‗s net selling price and its value in use.
Value in use is the present value of estimated future cash flows expected to arise from the
continuing use of an asset and from its disposal at the end of its useful life.

Net selling price the amount obtainable from the sale of an asset in an arm‗s length
transaction between knowledgeable, willing parties, less the costs of disposal.

An impairment loss is the amount by which the carrying amount of an asset exceeds its
recoverable amount.

207
A cash-generating unit is the smallest identifiable group of assets that generates cash inflows
from continuing use that are largely independent of the cash inflows from other assets or
groups of assets.

Corporate assets are assets other than goodwill that contribute to the future cash flows of
both the cash-generating unit under review and other cash-generating units.

IMPAIRMENT ASSESSMENT
An enterprise should assess at each reporting date: -
a) Whether there is any indication that an asset may be impaired;
b) Irrespective of any indication of impairment, an entity shall also: -
 Test in case of intangible assets having indefinite life or under development; and
 Test goodwill acquired in business combination for impairment annually

External sources of information include:


 Decline in asset‗s market value
 Adverse effect in the technological, market, economic or legal environment in which the
enterprise operates;
 Increase in market interest rates

Internal sources of information include:


 Obsolescence or physical damage
 Significant changes with an adverse effect in the extent to which, or manner in which, an
asset is used or is expected to be used
 Economic performance and expected net cash flows of an asset are worse than expected

MEASURING RECOVERABLE AMOUNT


Net selling price is the price in Binding Sale Agreement less Disposal Cost

Value in Use is the present value of estimated the future cash inflows and outflows to be
derived from continuing use of the asset and from its ultimate disposal
 The discount rate should be a pre-tax rate
 Cash flows projections of maximum five years
 Future expense of capital nature notunless a longer period can be justified allowed
 Only future revenue nature expenditures allowed

RECOGNITION AND MEASUREMENT OF IMPAIRMENT LOSS


An impairment loss should be recognized as an expense in the statement of profit or loss
immediately, unless the asset is carried at revalued amount (Recognized directly against any
revaluation surplus. Any over and above amount as expense in P&L)

Example
A non-current asset had a carrying amount of GHC80,000 in the statement of financial position
of an entity at the beginning of the financial year. The asset had previously been revalued, and
there was a revaluation surplus of GHC5,000 relating to it in the revaluation reserve.

208
At the end of the financial year, the entity suspected that the asset had been impaired. It
therefore estimated the recoverable amount of the asset and found this to be GHC60,000. The
depreciation charge for the asset for the year would be GHC8,000.
There is an impairment loss of GHC20,000 (= GHC80,000 - GHC60,000). Of this, GHC5,000 may
be offset against the revaluation reserve (the remaining revaluation surplus for the asset) and
reported as a negative amount in ‗other comprehensive income‘ for the year. The remaining
GHC15,000 must be written off as an expense for the period, and the asset should be re-stated at
its recoverable amount (GHC60,000) in the statement of financial position.
In the next year, depreciation should be based on the new carrying value of the asset
(GHC60,000) less any expected residual value.

CASH GENERATING UNIT


A cash generating unit (CGU) is the smallest identifiable group of assets for which independent
cash flows can be identified and measured. For example, for a restaurant chain a CGU might be
each individual restaurant.
 As goodwill acquired in a business combination does not generate cash flows
independently of other assets, it must be allocated to each of the acquirer‗s cash
generating units
 A CGU to which goodwill has been allocated is tested for impairment annually. The
carrying amount of the CGU including the goodwill is compared with its recoverable
amount.
The impairment loss on a CGU is allocated in the following order:
 first to any asset that is impaired (e.g. if an asset was specifically damaged)
 second, to goodwill in the cash generating unit
 third, to all other assets in the CGU on a pro rata basis based on carrying value
When allocating an impairment loss the carrying amount of an asset should not be reduced
below the higher of its fair value less costs to sell, value in use or zero.
Example
An entity engaged in mining operations owns a customized private railway to be used for its
mining activities and operations. There is an indication that the private railway is impaired.
However, the private railway does not generate cash inflows independently from the cash
inflows of the other assets of the mine, and is dependent upon other assets of the mine to
generate economic benefits or alternatively it could be sold only for scrap value.
The recoverable of the private railway is not determinable because its value in use cannot be determined
on individual basis, as it does not generate cash inflows independently from other assets and is probably
different from its scrap value. Therefore, the entity needs to determine the recoverable value of the cash
generating unit as whole to which the private railway belongs, i.e. the mine as a whole.
 If an active market is available for the output produced by an asset or group of assets,
that asset or group of assets shall be identified as a cash-generating unit even if some or
all of its output is used internally and in such circumstances, the cash flows of the cash
generating unit should reflect the management‘s best estimate of future price(s) that can
be obtained in an arm‘s length transaction.
 The entity is required to identify the cash generating units consistently over the
accounting periods using the same asset or types of assets, unless a change in assets of
cash generating unit can be justified.

REVERSAL OF IMPAIRMENT LOSS

209
 Under cost model, the increased carrying amount of an asset due to a reversal on
impairment loss should not exceed the carrying amount that would have determined
had no impairment loss been recognized for the asset in prior years.
 It is recognized as income immediately in statement of profit or loss
 Under revaluation model, any reversal of an impairment loss on a revalued asset should
be treated as a revaluation increase under that other International Accounting Standard.
Considerations on reversal of an Impairment Loss for a cash generating unit:
 First, asset other than goodwill on a pro-rata basis based on the carrying amount of each
asset in the unit; and
 An impairment loss recognized for goodwill shall not be reversed in a subsequent
period.
Disclosures
 The impairment loss recognized in the current reporting period
 The line item in which loss is presented in the statement of profit or loss
 The amount of impairment loss recognized, related to asset under revaluation model in
the other comprehensive income statement and any reversal related to such assets
 The amount of reversal of impairment loss recognized in the current period and the line
item in the statement of profit or loss in which such reversal is presented
 The entity is required to disclose the following in respect of individual asset, cash
generating unit and goodwill for which impairment loss is recognized in the current
period:
a) The description of individual asset
b) The description of cash generating unit
c) The circumstances reflecting impairment loss
d) Any change in the assets of the cash generating unit as compared to the previous accounting
period
 How the entity has determined the recoverable value
 Basis to determine the fair value less cost to sell and value in use
 The entity‘s estimates of future cash flows, related supportable assumptions and
discount rates for determination of value in use
 Amount of goodwill allocated to the cash generating unit
 Description of the assets forming cash generating unit

Worked Examples
Example 1:
The following information relates to three assets;
A B C
GHC GHC GHC
Carrying Value 200 300 240
Net Realisable Value 220 250 200
Value in Use 240 260 180

Required:

(i) What is the recoverable amount of each asset


(ii) Calculate the impairment loss for each asset

210
Solution:
A B C
GHC GHC GHC
(i)Recoverable Amount 240 260 200

(ii) Impairment loss - 40 40

Example 2:
An entity owns a plant which was originally purchased for GHC 600,000.The plant has revalued
to GHC 1,000,000 with the revaluation gain of GHC 400,000 being recognized in the revaluation
surplus(capital surplus).The plant has a carrying value of GHC 920,000 but the recoverable
amount of the plant is estimated at only GHC400,000.
What is the amount of the impairment loss and how should it be treated in the financial
statement.
Answer:
GHC’000
Carrying Value 920
Recoverable Amount 400
Impairment loss 520
Revaluation Gain 400
Impairment Loss(Income statement) 120
Example 3
The financial controller of CR7 Ltd has identified a matter below which may indicate an
impairment loss:
CR7 Ltd operates a plant which has a cost of GHC1,280,000 and accumulated depreciation of
GHC800,000 at 1 January 2013. It is being depreciated at 12.5% on cost using straight line
method. On 1 July 2013 in the mid of the current year, the plant was damaged because of
collusion with a factory vehicle. On this date, the entity estimated that present value of the plant
in use is GHC300,000 and it has a current disposal value of GHC40,000.
Required
Calculate impairment loss on the company‘s plant if any, and prepare the extracts of financial
statements for the year ended 31 December 2013 for CR7 Ltd.

Solution:
Statement of Profit or Loss 31/12/13
GHC‘000
Depreciation Expense (80 + 60) (W1) (140)
Impairment Loss(W1) (100)

Statement of Financial Position 31/12/13


GHC‘000
Assets:
Non-Current Asset:
Plant(W1) 240
(W1)

211
Plant 31/12/13
GHC‘000
Cost 1,280
- Accumulated Depreciation (800)
Carrying Value at 01/01/13 480
- Current Year Dep. (6 months) (1,280 × 12.5%) × 6/12 (80)
Carrying Value at 30/06/13 400
Impairment loss (100)
Recoverable value(W2) 300
- Current Year Dep. (6 months) (300 / 2.5 years) × 6/12 (60)
Carrying Value at 31/12/13 240
(W2) Recoverable Value:
It is determined as the higher of,
 Value in Use C300
 Fair Value less costs to sell C40
Example 4
Aquilani Ltd has acquired 100% share capital of Advent on I January 2011, which is engaged in
the supply of basic foods. The entity was generating healthy profits, but has now started
reporting operating losses from the last few months because of bad reputation resulting from
numerous customers becoming ill, because of the supply of sub-standard foods in May 2011.
The carrying values of Advent's assets at 31 December 2011 are as follows:
C'000
Goodwill 14,000
Factory Building 24,000
Purifying plant 16,000
Inventories 10,000
Total 64,000
Based on the estimated future cash flows, the directors have estimated that the value in use of
Advent as a cash generating unit at 31 December 2011is C40 million. There is no reliable
estimate of the fair value less costs to sell of Advent.
Required
Calculate the carrying values of Advent‘s assets at which these will be presented in the
consolidated statement of financial position of Aquilani Ltd, for the year ended 31 December
2011.
Solution:
(W1) Impairment loss at 31 December 2013
C‘000
Carrying value of cash generating unit 64,000
Impairment Loss (24,000)
Recoverable value 40,000

(W2) Allocation of Impairment loss to cash generating unit


C.V at Imp. Loss at New C.V at
31/12/13 31/12/13 31/12/13

212
C‘000 C‘000 C‘000
Goodwill 14,000 (14,000) -
Factory Building 24,000 (6,000) 18,000
Purifying plant 16,000 (4,000) 12,000
Inventory 10,000 - 10,000
64000 24,000 40,000
Notes:
 The impairment loss of C14,000 out of 24,000 will be first allocated to goodwill.
 Then the remaining loss of C10,000(C24,000 - C14,000) will be allocated to other assets of
the cash generating unit i.e. factory building and purifying plant on the basis of their
respective carrying value as follow:
Factor Building = C10,000 × C24,000 / (C24,000 + C16,000) = C6,000
Purifying plant = C10,000 × C16,000 / (C24,000 + C16,000) = C4,000
 The inventory will not be charged any impairment loss under IAS 36 as it is covered
under IAS 2.
REVIEW QUESTIONS
QUESTION 1
A company runs a unit that suffers a drastic drop in income due to the failure of its technology
on 1 January,2000.The following carrying values were recorded in the books immediately prior
to their impairment:
GHC
Goodwill 100,000
Technology 25,000
Brands 50,000
Land 250,000
Buildings 150,000
Other net assets 200,000
The recoverable value of the unit is estimated at GHC 425,000.The technology is
worthless,following its complete failure.
The other net assets include inventory,receivables and payables and it is considered that the
book values of other net assets is a reasonable representation of its net realizable value.

Show how the impairment of the CGU will be treated in the books.

Answer:
Carrying Value Impairment Loss Carrying Value
before Impairment after Impairment
GHC GHC GHC
Goodwill 100,000 (100,000) -
Technology 25,000 (25,000) -
Brands 50,000 (50/450x225) (25,000) 25,000
Land 250,000 (250/450x225) (125,000) 125,000
Buildings 150,000 (150/450x225) (75,000) 75,000
Other Net Assets 200,000 - 200,000
775,000 (350,000) 425,000

213
GHC
Carrying Amount 775,000
Less Recoverable Amount 425,000
Impairment Loss 350,000

QUESTION 2
There was an explosion in a factory.The carrying amounts of its assets were as follows:
GHC’000
Goodwill 100
Patents 200
Machines 300
Computers 500
Buildings 1,500
2,600
The factory operates as a cash generating unit.An impairment review reveals a net selling price
of GHC 1.2m for the factory and value in use of GHC 1.95m.Half of the machines have been
blown to pieces but the other half can be sold for at least their book value.The patents have been
superseded and are now considered worthless.
Show the effect of the explosion on the asset values.

Answer
Carrying Value Impairment Loss Carrying Value
before Impairment after Impairment
GHC GHC GHC
Goodwill 100,000 (100,000) -
Patents 200,000 (200,000) -
Machines 300,000 (150,000) 150,000
Computers 500,000 (5/20 x 200) (50,000) 450,000
Buildings 1,500,000 (15/20 x200) (150,000) 1,350,000
2,600,000 (650,000) 1,950,000

GHC000 GHC000
Carrying Amount 2,600
Less Recoverable Amount :
Net Selling Price 1,200
Value in Use 1,950 1,950
Impairment Loss 650

QUESTION 3
The following is relevant to three tangible non-current assets held by Charlotte.
Machine 1 was purchased on 1 January2011 for GHC420,000. It had an estimated residual value
of GHC50,000 and a useful life of ten years and was being depreciated on a straight-line basis.
On 1 January 2016 Charlotte revalued this machine to GHC275,000 and reassessed its total

214
useful life as fifteen years. On 1 January 2017 an impairment review showed machine 1‘s
recoverable amount to be GHC100,000 and its remaining useful life to be five years.
Machine 2 was purchased on 1 January 2011 for GHC500,000. It had an estimated residual
value of GHC60,000 and a useful life of ten years and was being depreciated on a straight-line
basis. On 1 January 2017 this machine was classified as held for sale, at which time its fair value
was estimated at GHC200,000 and costs to sell at GHC5,000. On 31 March 2017 the machine was
sold for GHC210,000.
Machine 3 was purchased on 1 January 2011 for GHC600,000. In 2011 depreciation of
GHC30,000 was charged. On 1 January 2012 this machine was revalued to GHC800,000 and its
remaining useful life assessed as eight years. On 1 January 2017 this machine was classified as
held for sale, at which time, its fair value was estimated at GHC550,000 and costs to sell at
GHC5,000. On 31 March 2017 the machine was sold for GHC550,000.
Tax is at the rate of 30%.

Required
For each machine show the effect of the above on profit or loss, other comprehensive income
and revaluation reserve of Charlotte in 2017. You should also show the brought forward
balance on the revaluation reserve (at 1 January 2017) in respect of machines 1 and 3.

Answer:
Effect on 2017 profit or loss
Machine 1 Machine 2 Machine 3
GHC GHC GHC
Impairment loss (107,500) (W1) (41,000) (W2) (5,000)
Depreciation charge (100,000 ÷ 5) (20,000) - -
Gain on disposal (210,000 - 195,000 (W2)) - 15,000

Effect on 2017 other comprehensive income


Machine 1 Machine 2 Machine 3
GHC GHC GHC
Revaluation loss (40,000) (W1) (41,000) (W2) (5,000)
Less: Associated taxation 12,000

Effect on 2017 revaluation reserve


Machine 1 Machine 3
GHC GHC
On 1 January,2017:
Machine 1 (W1): Machine 3
GHC230,000 less 30% 28,000 161,000
Revaluation to fair value on
classification as held for sale
Impairment loss, 31 December,2017 (W1) (28,000)
Transferred to retained earnings on disposal (161,000)
On 31 December 2017 - -
Workings
(1) Machine 1 GHC
1 January 2011 Cost 420,000

215
Depreciation to 1 January,2016 (((420,000 – 50,000) ÷ 10) × 5) (185,000)
Carrying amount on 1 January 2016 235,000
Revalued to 275,000
Revaluation gain before tax 40,000
Carrying amount on 1 January 2016 275,000
Depreciation to 1 January 2017 (275,000 ÷ (15 – 5)) (27,500)
Carrying amount at 1 January 2017 247,500
Recoverable amount (100,000)
Impairment loss 147,500
On 31 December 2016, the asset is revalued upwards by GHC40,000. Of this, GHC28,000 is
taken to the revaluation reserve and GHC12,000 (GHC40,000 × 30%) to deferred tax as a
liability.
In the year to 31 December,2017, the impairment loss is GHC147,500. Of this, GHC40,000
reverses the gain in the previous year. The revaluation reserve is reduced by GHC28,000 and
the deferred tax liability by GHC12,000. The remaining impairment loss of GHC107,500 is
written off as a loss in 2017.
(2) Machine 2 GHC
1 January 2011
Cost 500,000
Depreciation to 1 January 2017 (((500,000 – 60,000) ÷ 10) × 6) (264,000)
Carrying amount on 1 January 2017 236,000
Fair value minus cost to sell (200,000 – 5,000) (195,000)
Impairment loss 41,000
(3) Machine 3 GHC
1 January,2011 Cost 600,000
Depreciation to 1 January,2012 (30,000)
Carrying amount on 1 January,2012 570,000
Revalued to 800,000
Taken to revaluation reserve/deferred tax (70%/30%) 230,000
Carrying amount on1 January,2012 800,000
Depreciation to 1 January,2017 ((800,000 ÷ 8) × 5) (500,000)
Carrying amount on 1 January,2017 300,000
Fair value on classification as held for sale 550,000
Value at lower of carrying amount and fair value less costs to sell: therefore 300,000
QUESTION 4
A division of Imperial has the following non-current assets, which are stated at their carrying
values at 31 December 2014:
GHCm GHCm
Goodwill 70
Property, plant and equipment:
Land and buildings 320
Plant and machinery 110 430
500
Because these assets are used to produce a specific product, it is possible to identify the cash
flows arising from their use. The management of Imperial believes that the value of these assets
may have become impaired, because a major competitor has developed a superior version of
the same product and, as a result, sales are expected to fall.

216
The following additional information is relevant:
Forecast cash inflows arising from the use of the assets are as follows:
Year ended 31 December:
GHCm
2015 185
2016 160
2017 130
(i) The directors are of the opinion that the market would expect a pre-tax return of 12% on an
investment in an entity that manufactures a product of this type.
(ii) The land and buildings are carried at valuation. The surplus relating to the revaluation of
the land and buildings that remains in the revaluation reserve at 31 December 2014 is GHC65
million. All other non-current assets are carried at historical cost.
(iii) The goodwill does not have a market value. It is estimated that the land and buildings
could be sold for GHC270 million and the plant and machinery could be sold for GHC50
million, net of direct selling costs.
Required
(a) Calculate the impairment loss that will be recognised in the accounts of Imperial.
(b) Explain how this loss will be treated in the financial statements for the year ended 31
December 2014.

Solution:
(a) Impairment loss GHCm
Carrying value 500
Recoverable amount (385)
Impairment loss 115
Recoverable amount is value in use (Working 1) as this is higher than net realisable value
(Working 2).
Workings
(1) Value in use:
Forecast cash flows discounted at 12%: GHCm
2015 (185 × 0.893) 165.2
2016 (160 × 0.797) 127.5
2017 (130 × 0.712) 92.6
Total 385.3
(2) Net realisable value: GHCm
Goodwill 0
Freehold 270
Freehold land and buildings 50
320
(b) Treatment of impairment loss IAS 36 requires the impairment loss to be allocated to the
various non-current assets in the following order: firstly, goodwill, secondly, to other assets,
either pro-rata or on another more appropriate basis.
Before impairment Impairment loss (W1) After impairment
GHCm GHCm GHCm
Goodwill 70 (70) ‐
Land and buildings 320 (33) 287
Plant and machinery 110 (12) 98

217
500 (115) 385
Because the land and buildings have been re-valued, the impairment is treated as a revaluation
decrease until the carrying amount of the asset reaches its depreciated historical cost. The
revaluation reserve relating to the asset is GHC65 million and so is adequate to cover the full
impairment of GHC33m.
The impairment must be separately disclosed and the notes to the accounts must specify by
class of asset the impairment recognised directly to equity. The impairment loss on the goodwill
and plant (GHC66 million) must be recognised in profit or loss for the year. The notes to the
accounts must specify the line item in which the impairment loss has been included.
Where the impairment write-down is material, information must also be provided as to the
events and circumstances that led to the loss, the nature of the assets affected, the segment to
which the asset belongs, that recoverable amount was based on value in use and the discount
rate used to calculate this.
Workings
Loss on the various non-current assets After the impairment loss has been recognised on the
goodwill there is still 115 - 70 = 45 loss to be allocated to the other non current assets, on a pro-
rata basis.
Loss on land and buildings:
320 x 45 =33
320+110
Loss on plant and machinery:
110 x 45 =12
320+110

QUESTION 5
(i) Define an impairment loss explaining the relevance of fair value less costs to sell and value in
use; and state how frequently assets should be tested for impairment; (6 marks)
Note: your answer should NOT describe the possible indicators of an impairment.
(ii) Explain how an impairment loss is accounted for after it has been calculated. (5 marks)
(b) The assistant financial controller of the Wilderness group, a public listed company, has
identified the matters below which she believes may indicate an impairment to one or more
assets:
(i) Wilderness owns and operates an item of plant that cost GHC640,000 and had accumulated
depreciation of GHC400,000 at 1 October 2014. It is being depreciated at 121% on cost. On 1
April 2015 (exactly half way through the year) the plant was damaged when a factory vehicle
collided into it. Due to the unavailability of replacement parts, it is not possible to repair the
plant, but it still operates, albeit at a reduced capacity. Also it is expected that as a result of the
damage the remaining life of the plant from the date of the damage will be only two years.
Based on its reduced capacity, the estimated present value of the plant in use if GHC150,000.
The plant has a current disposal value of GHC20,000 (which will be nil in two years' time), but
Wilderness has been offered a trade–in value of GHC180,000 against a replacement machine
which has a cost of GHC1 million (there would be no disposal costs for the replaced
plant).Wilderness is reluctant to replace the plant as it is worried about the long–term demand
for the produce produced by the plant. The trade–in value is only available if the plant is
replaced.
Required

218
Prepare extracts from the statement of financial position and income statement of Wilderness in
respect of the plant for the year ended 30 September 2015. Your answer should explain how you
arrived at your figures. (7 marks)

(ii) On 1 April 2014 Wilderness acquired 100% of the share capital of Mossel, whose only
activity is the extraction and sale of spa water. Mossel had been profitable since its acquisition,
but bad publicity resulting from several consumers becoming ill due to a contamination of the
spa water supply in April 2015 has led to unexpected losses in the last six months. The carrying
amounts of Mossel's assets at 30 September 2015 are:
GHC'000
Brand (Quencher – see below) 7,000
Land containing spa 12,000
Purifying and bottling plant 8,000
Inventories 5,000
32,000
The source of the contamination was found and it has now ceased.
The company originally sold the bottled water under the brand name of 'Quencher', but because
of the contamination it has rebranded its bottled water as 'Phoenix'. After a large advertising
campaign,sales are now starting to recover and are approaching previous levels. The value of
the brand in the statement of financial position is the depreciated amount of the original brand
name of 'Quencher'.
The directors have acknowledged that GHC1.5 million will have to be spent in the first three
months of the next accounting period to upgrade the purifying and bottling plant.
Inventories contain some old 'Quencher' bottled water at a cost of GHC2 million; the remaining
inventories are labeled with the new brand 'Phoenix'. Samples of all the bottled water have been
tested by the health authority and have been passed as fit to sell. The old bottled water will
have to be re-labelled at a cost of GHC250,000, but is then expected to be sold at the normal
selling price of (normal) cost plus 50%.Based on the estimated future cash flows, the directors
have estimated that the value in use of Mossel at 30 September 2015, calculated according to the
guidance in IAS 36, is GHC20 million. There is no reliable estimate of the fair value less costs to
sell of Mossel.

Required
Calculate the amounts at which the assets of Mossel should appear in the consolidated
statement of financial position of Wilderness at 30 September 2015. Your answer should explain
how you arrive at your figures. (7 marks)
(Total = 25 marks)

Answer:
(a) (i)An impairment occurs when the carrying value of an asset exceeds its recoverable amount.
Recoverable amount represents the amount of cash that an asset will generate either through
use (value in use) or through disposal (fair value less costs to sell).
The value in use is the present value of all future cash flows derived from an asset, including
any disposal proceeds at the end of the asset's life. The present value of future cash flows will be
affected by the timing, volatility and uncertainty of the cash flows. This can be reflected in the
forecasted cash flows or the discount rate used.

219
Very few business assets generate their own cash flows, and so assets are often grouped
together into cash generating units for impairment purposes. A cash generating is the smallest
group of assets generating independent cash flows
Fair value less costs to sell is the amount obtainable for an asset in an arm's length transaction
between knowledgeable, willing parties, less the cost of disposal. The fair value of used assets
with no active market will have to be estimated. Valuations are based on willing parties, and so
a 'forced sale' value would not normally be used.

Impairment reviews
• At each reporting date an entity shall assess whether there are any indications that an
impairment has occurred; if there are such indications then the recoverable amount of the asset
must be estimated.
• Intangible assets with indefinite lives (and those not ready for use) should be reviewed for
impairment annually. The review should take place at the same time each year.
• Cash generating units that include goodwill should be reviewed for impairment annually.
(ii) Accounting for an impairment loss Impairment losses should be recognised immediately.
They will normally be charged to the income statement alongside depreciation, but the
impairment of a revalued asset should be taken directly to the revaluation surplus (until the
balance on the revaluation surplus is reduced to zero). In the statement of financial position the
impairment will normally be included within accumulated depreciation, although it could be
disclosed separately if material. Future depreciation charges will be based on the impaired
value and the remaining useful life at the date of the impairment.
Impairments of cash generating units must be apportioned to the individual assets within that
unit. The impairment is firstly allocated to goodwill, and then it is apportioned to all other
assets (both tangible and intangible) on a pro rata basis. However, individual assets are not
impaired below their own realisable value; any unused impairment being re-apportioned to the
other assets.

(b) (i) Summary


INCOME STATEMENT
GHC
Depreciation First six months 40,000
Second six months 37,500
Impairment 50,000
127,500
STATEMENT OF FINANCIAL POSITION 30 SEPTEMBER 2015
GHC
Cost 640,000
Accumulated depreciation and impairment
(400,000 + 40,000 + 50,000 + 37,500) (527,500)
Carrying value 112,500
At 1 April 2015 the asset should be restated at the lower of carrying value and recoverable
amount. Recoverable amount is the higher of value in use and fair value less costs to sell.
QUESTION 6

220
On 1 December 2006, Ananse opened a school at a cost of GHC5 million. The estimated useful
life of the school was 25 years. On 30 November 2012, the school was closed because numbers
using the school declined unexpectedly due to a population shift caused by the closure of a
major employer in the area. The school is to be converted for use as a library, and there is no
expectation that numbers using the school will increase in the future and thus the building will
not be reopened for use as a school. The current replacement cost for a library of equivalent size
to the school is GHC2·1 million. Because of the nature of the non-current asset, value-in-use and
net selling price are unrealistic estimates of the value of the school. The change in use would
have no effect on the estimated life of the building.
Required:
Discuss how the above events should be accounted for in the financial statements of Ananse.
(6 marks)
Solution:
An asset is carried at more than its recoverable amount if its carrying amount exceeds the
amount to be recovered through use or sale of the asset. If this is the case, the asset is described
as impaired and IAS 36 Impairment of Assets requires the recognition of an impairment loss. At
the end of each reporting period, an assessment should take place as to whether there is any
indication that an asset may be impaired. If any indication exists, the recoverable amount
should be estimated taking into account the concept of materiality in identifying whether the
recoverable amount of an asset needs to be estimated. If no indication of an impairment loss is
present, IAS 36 does not require a formal estimate of the recoverable amount, with the
exception of intangible assets.
Impairment in this case is indicated because the purpose for which the building is used has
changed significantly from a place for educating students to a library and this is not anticipated
to change for the foreseeable future. There is insufficient information to determine value in use
and net selling price (fair value less selling costs); as such, depreciated replacement cost should
be used as an approximation of the recoverable amount. An impairment loss using a
depreciated replacement cost approach would be determined as follows:

Asset Cost/replacement Accumulated Carrying amount/


cost GHC000 depreciation replacement cost GHC000
GHC000 – 6/25 30 November 2012
School 5,000 (1,200) 3,800
Library 2,100 (504) (1,596)
––––––
Impairment loss 2,204
––––––
Thus Ananse would record the impairment loss of GHC2·204m.

QUESTION 7
Due to a change in Pusiga Ltd‟s production plans, an item of machinery with a carrying value of
GH¢11 million at 31 December 2017 (after adjusting for depreciation for the year) may be
impaired due to a change in use. An impairment test conducted on 31 December 2017, revealed
its fair value less cost of disposal to be GH¢5 million. The machine is now expected to generate
an annual net income of GH¢2 million for the next three years at which point the asset would be

221
sold for GH¢2.4 million. An appropriate discount rate is 10%. Pusiga charges depreciation at
20% on reducing balance method on machinery.

Note:
 The present value of ordinary annuity of GH¢1 at 10% for one year, two years and three
years are 0.909,1.736 and 2.487 respectively.
 The present value of GH¢1 at 10% for one year, two years and three years 0.909, 0.826 and
0.751 respectively

Required:
In accordance with IAS 36: Impairment of Assets, explain with justification the required
accounting treatment in the financial statements of Pusiga Ltd for the year ended 31 December
2017. (3 marks)

Solution:
Under IAS 36 Impairment, The machinery needs to be tested for impairment.
GH¢m
Carrying value 11
Recoverable amount (GH¢2 x 2.4868) + (GH¢2.4 x 0.7513) (6.8)
Impairment 4.2

Recoverable amount is the higher of value in use (GH¢6.8m) and fair value less costs of
disposal (GH¢5m)

GH¢m GH¢m
Cr (PPE) 4.2
Dr SPLOCI - Serox 4.2

QUESTION 8
Cantona acquired a cash-generating unit (CGU) several years ago but, at 28 February 2017, the
directors of Cantona were concerned that the value of the CGU had declined because of a
reduction in sales due to new competitors entering the market. At 28 February 2017, the carrying
amounts of the assets in the CGU before any impairment testing were:

GHCm
Goodwill 3
Property, plant and equipment 10
Other assets 19
–––
Total 32
–––
The fair values of the property, plant and equipment and the other assets at 28 February 2017
were GHC10 million and GHC17 million respectively and their costs to sell were GHC100,000
and GHC300,000 respectively.

222
The CGU‟s cash flow forecasts for the next five years are as follows:
Date year ended Pre-tax cash flow Post-tax cash flow
GHCm GHCm
28 February 2018 8 5
28 February 2019 7 5
28 February 2020 5 3
28 February 2021 3 1·5
28 February 2022 13 10
The pre-tax discount rate for the CGU is 8% and the post-tax discount rate is 6%. Cantona has no
plans to expand the capacity of the CGU and believes that a reorganisation would bring cost
savings but, as yet, no plan has been approved.
The directors of Cantona need advice as to whether the CGU‟s value is impaired.
The following extract from a table of present value factors has been provided.
Year Discount rate 6% Discount rate8%
1 0·9434 0·9259
2 0·8900 0·8573
3 0·8396 0·7938
4 0·7921 0·7350
5 0·7473 0·6806

Required:
Show how the above transactions should be dealt with in its financial statements with
reference to relevant International Financial Reporting Standards. (8 marks)
Solution:
IAS 36 Impairment of Assets requires that assets be carried at no more than their carrying
amount. Therefore entities should test all assets within the scope of the standard if there is
potential impairment when indicators of impairment exist. If fair value less costs of disposal or
value in use is more than carrying amount, the asset is not impaired. It further says that in
measuring value in use, the discount rate used should be the pre-tax rate which reflects current
market assessments of the time value of money and the risks specific to the asset. The discount
rate should not reflect risks for which future cash flows have been adjusted and should equal
the rate of return which investors would require if they were to choose an investment which
would generate cash flows equivalent to those expected from the asset. Therefore pre-tax cash
flows and pre-tax discount rates should be used to calculate value in use. Discounting post-tax
cash flows with a post-tax discount rate could give the same result in an entity were it not for
any temporary differences and/or tax losses which might exist.
Date: year ended Pre-tax cash flow Discounted cash flows
(GHCm) (GHCm) at 8%
28 February 2018 8 7·41
28 February 2019 7 6·00
28 February 2020 5 3·97
28 February 2021 3 2·21
28 February 2022 13 8·85
––––––
Total 28·44
––––––

223
The CGU is impaired by the amount by which the carrying amount of the cash-generating unit
exceeds its recoverable amount which is the higher of an asset‘s fair value less costs of disposal
and its value in use. The fair value less costs to sell (GHC26·6 million) is lower than the value in
use (GHC28·44 million). The recoverable amount is therefore GHC28·44 million. The carrying
amount is GHC32 million and therefore the impairment is GHC3·56 million.
Cantona will allocate the impairment loss first to the goodwill and then to other assets of the
unit pro rata on the basis of the carrying amount of each asset in the cash-generating unit.
Consequently, the entity will allocate GHC3 million to goodwill and then allocate GHC0·56
million on a pro rata basis to PPE (0·56 x 10/29 = GHC0·19 million) and other assets (0·56 x
19/29 =GHC0·37 million). This would mean that the carrying amounts would be GHC9·81
million and GHC18·63 million respectively.
However, when allocating the impairment loss, the carrying amount of an asset cannot be
reduced below its fair value less costs to sell. The fair value less costs to sell of the CGU‘s assets
is GHC9·9 million (PPE) and GHC16·7 million (other assets).Therefore the carrying amounts of
the assets of the CGU after impairment will be PPE GHC9·9 million and other assets GHC18·54
million as the excess impairment of GHC0·09 million on PPE will be allocated to other assets.

QUESTION 9
In accordance with IAS 36: Impairment of Assets, explain which of the following would require
an impairment test to be performed during the year ended 31 December 2019 on assets or cash-
generating units or at group level in respect of Timtim Ltd (Timtim):
i) A fall in the benchmark interest rate used by the lender to calculate interest due on Timtim's
loans payable, its main form of finance.
ii) Acquisition by Timtim of 100% of another entity in 2018 at a price substantially above the fair
value of the entity's net assets. The entity acquired was not sold in 2019.
iii) Sanctions put in place restricting exports by Timtim to a major client.
iv) Increase in the tax rate applied to Timtim.

(Note: You should consider each event independently). (4 marks)

SOLUTION
i) Fall in benchmark interest:
No impairment test is required.
However a rise in interest rates would trigger an impairment test.

ii) Acquisition of entity last year:


Yes, impairment test is necessary because the acquisition would result in goodwill and must be
tested annually for impairment.

iii) Sanctions restricting exports:


Yes, this impairment indicator alters cash flow of the unit.

iv) Increase in tax rate:


No, impairment losses are calculated on a pre-tax basis.

Question 10

224
Stev acquired a plant which cost of GH¢3,000,000 as at 1 January 2011. The machine is not
producing as it was originally planned. Consequently, Stev has to reduce production by 40%.
Cash flow forecast for six years included in the budget submitted for management approval in
January 2012 shows the following as at 31st December:
Year Cash flows (GH¢)
2012 600,000
2013 700,000
2014 660,000
2015 500,000
2016 450,000
2017 800,000
The cash flow forecast for 2017 Includes expected proceeds from disposal of the plant. The cash
flow projections also ignore the effects general upwards movement in prices.
It is estimated if the plant is sold on 1st January 2013, it would realize proceeds of
GH¢1,500,000 and pay sales commission of 2.5% on proceeds. Stev Ltd depreciates all
Machinery at 15% per annum. The cost of capital for Stev is 20% (ignoring inflationary effect)
Required:
Calculate the recoverable amount of the plant at 1st January 2013 and impairment loss (if any)

Question 11
A cash generating unit comprising a factory, plant and equipment etc and assorted purchased
goodwill becomes impaired because the product it makes is overtaken by a technologically more
advanced model produced by a competitor. The recoverable amount of the cash generating unit
falls to GH¢60m, resulting in an impairment of GH¢80m, allocate as follows:
Carrying amount carrying amount Before impairment after impairment
GH¢m GH¢m
Goodwill 40 _
Patent (with no market value) 20 -
Tangible non-current assets (market value GH¢60m)80 60
Total 140 60
After three years, the entity makes a technological breakthrough of its own, and the recoverable
amount of the cash generating unit increases to GH¢90m. The carrying amount of the tangible
non-current assets had the impairment not occurred would have been GH¢70m.
Required
Calculate the reversal of the impairment loss.

Question 12
AT Group Ltd has an item of earth moving plant, which is rented out to companies on short-term
contracts. Its carrying value, based on depreciated historical cost is GH¢400,000. The estimated
selling price of this asset is only GH¢250,000, with associated selling expenses of GH¢5,000. A
recent review of its value in use based on its forecast future cash flows was estimated at
GH¢500,000. Since this review was undertaken, there has been a dramatic increase in interest
rates that has significantly increased the cost of capital used by AT Group Ltd to discount the
future cash flows of the plant.

225
Question 12
Afoko Ltd acquired a car taxi business on 1 January 2015 for GH¢230,000. The value of the
assets of the business at that date based on net selling price were as follows:
GH¢000
Vehicles 120
Intangible assets 30
Trade receivables 10
Cash 50
Trade payables (20)
190
On 1 February 2015, the taxi business had three (3) of its vehicles stolen. The net selling values of these
vehicles was GH¢30,000 and because of non-disclosure of certain risk to the insurance company, the
business was uninsured. As a result of this event, Afoko Ltd wishes to recognise an impairment loss of
GH¢45,000 inclusive of the loss of the stolen vehicles due to the decline in value of the stolen income
generating unit, that is the taxi business. On 1 March 2015, a rival taxi company commenced business in
the same area. It is anticipated that the business revenue of Afoko Ltd would be reduced by 25% leading
to a decline in the present value in use of the business which is calculated at GH¢150,000. The net selling
value of the taxi license has fallen to GH¢25,000 as a result of the rival taxi operator. The net selling
values of the other assets have remained the same as at 1 January 2015.

Required: Recommend how Afoko Ltd should account for the above transaction in its financial
statements in accordance with IAS 36 Impairment of Assets.

226
PROVISIONS, CONTINGENT LIABILITIES AND
CONTINGENT ASSETS (IAS 37).
The objective of IAS 37
The objective of IAS 37 was to eliminate the misleading use of provisions. It aims to ensure that
appropriate conditions are applied to the recognition of provisions (and contingencies), and that
suitable bases are used to measure the amount of provisions. In addition it requires that
sufficient information should be disclosed in the notes to allow users to understand the various
types of provision, their amounts
and the timing of their creation and release.
As a result of IAS 37 provisions can still be made, but only where there is some degree of
obligation (a real liability) on the part of the entity. A mere ‗management intention‘ is no longer
sufficient.
IAS37 also deals with contingent liabilities and contingent assets. IAS 37 does not apply to
provisions or contingencies covered by other accounting standards:
 construction contracts (IAS 11)
 income taxes (IAS 12)
 leases (IAS 17), except for onerous operating leases
 employee benefits (IAS 19)
 insurance contracts (IFRS 4), and
 contingent liabilities acquired as part of a business combinations (IFRS 3).
The notes in this section on provisions and in the next section on contingencies are based on the
current version on IAS 37. An exposure draft was issued in 2005 proposing major changes to
IAS 37. These are considered in a later section.

Recognition of provisions
A provision is a liability of uncertain timing or amount. As liabilities, provisions are included in
the statement of financial position and increases or reductions in provisions are reported in
profit and loss.
A liability should only be recognised when all of three conditions are met:
 The entity has a present obligation (legal or constructive) as a result of a past event.
 It is probable that an outflow of resources will be required to settle the obligation.
 A reliable estimate can be made of the amount of the provision.

Present obligation
There must be an obligation already in existence. Mere intention is not sufficient.
The obligation may be legal or constructive.
 A legal obligation is one arising from a contract, or some other aspect of the law. For
example, a company may sell goods with a two-year warranty if any defect arises after
the date of sale. The company now has legal obligation (a contractual obligation) to
make good by repair or replacement any defects that arise.
Where details of a proposed new law have yet to be finalised, an obligation only arises when
the legislation is virtually certain to be introduced (‗enacted‘).

227
 A constructive obligation is one arising from the entity‘s actions, whereby
− through established past practice, published policies, or a specific current statement, the entity
has indicated to other parties that it will accept certain responsibilities, and
− as a result, the entity has created a valid expectation that it will discharge those
responsibilities.
For example, a retail store may have a policy of refunding purchases made by dissatisfied
customers, even though it is only legally obliged to issue a refund if the goods are faulty. Its
policy of making refunds is widely known. As a result of its conduct, the company has created a
valid expectation amongst its customers that it will refund purchases. A constructive obligation
therefore exists.
Past event
The event leading to the present obligation must be past, and must have occurred before the
reporting date (end of the reporting period) when the provision is first recognised. No
provision is made for costs that may be incurred in the future but where no obligation yet
exists. For example, if an entity is planning a reorganization but does not yet have an obligation
(legal or constructive) to undertake the reorganisation, it cannot create a provision for
reorganisation costs.
For an event to be an ‗obligating event‘, it must be shown that the entity had no realistic
alternative to settling the obligation. This is the case only where the settlement can be enforced
by law or, in the case of a constructive obligation, that a valid expectation has been created.

Example
Under new legislation an entity is required to fit smoke filters to its factories by 30 June 2012.
The entity has not fitted the smoke filters. In its accounts to 31 December 2011, there is no
present obligation as the legislation is not yet effective, and so an obligating event has not yet
occurred. No provision for the cost of the smoke filters can be recognised in the accounts for
2011.

Probable outflow of economic benefits


For a provision to exist, the outflow of benefits must be probable. ‗Probable‘ is defined by IAS
37 as ‗more likely than not‘ (more than 50% probability). For example, an entity may have
given a guarantee but may not expect to have to honour it. In such a situation, it cannot create a
provision for the cost of expenses that it may have to incur under the terms of the guarantee.
This is because a payment under the guarantee is not probable.

Reliable estimate of the obligation


IAS 37 also requires that a provision should not be recognised unless a reliable estimate of the
amount of the provision can be made. A reasonable estimate can be based on a range of possible
outcomes. In the extremely rare case where no estimate can be made, a provision should not be
made and the liability should be disclosed instead as a contingent liability, in a note to the
financial statements.

Example
In each of the following situations, consider whether a provision should be recognised,
assuming that the reporting period ends on 31 December in each case:
(1) An entity in the oil industry cleans up contamination caused by its operations when required
to do so under the laws of the particular country in which it operates. One country in which it

228
operates has no legislation requiring clean up, and the entity has been contaminating the land
for several years. At 31
December 2014 it is virtually certain that a draft law requiring a clean-up of land already
contaminated will be enacted shortly after the year end.
(2) The government has introduced a number of changes to the regulation of the financial
services industry. As a result of these changes, entities will need to re-train a large proportion of
their employees in order to ensure compliance with the financial services regulations. However,
re-training is not a statutory requirement. One entity affected by the new regulations had not
done any retraining before the end of the reporting period.
(3) On 18 December 2016 the board of an entity decides to close down a division.However, this
decision was not communicated to any of those affected and no further steps were taken to
implement the decision before the end of the reporting period.
(4) On 18 December 2016 the board of an entity decides to close down a division.On 23
December a detailed plan for closing down the division was prepared by the board and
announcements were made to employees concerning redundancies. In addition, letters were
sent to customers advising them of the proposal and recommending alternative suppliers.

Answer
(1) By contaminating the land, the entity has created an obligation. There is virtual certainty that
the legislation will be introduced, giving a legal obligation to the entity to clean up the land.
This will result in a probable outflow of cash and resources to perform the clean-up and so a
provision should be made for the best estimate of the clean-up operation.
(2) No provision should be created by the entity, because as at the end of the reporting period
there has not been a past event giving rise to a present obligation. An obligation only arises
where the settlement can be enforced by law or the entity has created a valid expectation in
other parties. Neither applies in this situation.
(3) As there has not been an obligation created as at the end of the reporting period, no
provision should be created. For a constructive obligation to arise, the entity must have
communicated the plans to those affected (i.e. employees,customers and suppliers) and
implementation must be planned to start shortly.
(4) In contrast to situation 3, an obligating event exists as at the end of the reporting period,
because the entity has communicated the plans to its employees and customers. This gives rise
to a constructive obligation, as the entity has created an expectation which will lead to a
probable outflow of cash to settle redundancy payments and contract clauses. Therefore a
provision should be made for the best estimate of the costs of closure.

Measurement of provisions
The amount recognised as a provision should be the best estimate as at the end of the reporting
period of the expenditure required to settle the obligation. This estimate will be determined by
the judgment of management, together with experience of similar transactions or reports by
independent experts.
Risks and uncertainties should be taken into account in reaching the best estimate.Events after
the reporting period will provide useful evidence.

However, entities should:

229
 avoid creating excessive provisions (which could be used as a way of manipulating
profits between financial years), or
 avoid underestimating provisions.
The following guidelines apply to measuring a provision:
 In measuring a single obligation, the most likely outcome may be the best estimate of
the liability.
 However, if there are other possibilities which are mostly higher or mostly lower than
the most likely outcome, then the best estimate will be a higher or lower amount.
 When there is a large population of potential obligations (for example, a provision for
multiple claims under guarantees) the obligation should be estimated by calculating an
expected value. This is done by weighting all possible outcomes by their associated
probabilities.
 If there is a considerable time lag before the settlement of the provision then the amount
provided should be discounted to its present value.
Provisions should be reviewed at the end of each reporting period and adjusted to reflect the
current best estimate. If it is no longer believed probable that the provision is needed, it should
be reversed (reduced to zero) or reduced to a smaller amount.

Example
Griezman Ltd has a financial year ending 31 December. On 15 December 2011 an employee was
injured in the workplace and has sued Griezman Ltd for compensation under current health
and safety legislation. Griezman‘s solicitors believe that the employee‘s claim has a 60% chance
of success. The solicitors estimate that, if successful, the claim will be settled at GHC60,000.
Required
Consider whether or not Griezman Ltd should provide for the claim at 31 December 2011 and, if
so, at what amount.

Answer
If a provision is to be made:
 there must be a present obligation as a result of a past event
 it must be probable that an outflow of economic benefits will be required to settle the
obligation, and
 it must be possible to make a reliable estimate of the amount of the obligation.
Applying this to the facts in the example:
(1) Griezman has a legal obligation under the health and safety legislation. The obligation has
arisen from a past event, which is the accident on 15 December 2011.
(2) It is probable that Griezman will have to pay the employee. A ‗60% chance of success‘ means
that success is ‗more likely than not‘. (A probability above 50% meets the IAS 37 definition of
‗probable‘.)
(3) A reliable estimate can be made. This is GHC60,000.
Conclusion: A provision of GHC60,000 should be made at 31 December 2011.

Example
An entity sells high-definition televisions with a 12-month warranty under which customers are
covered for the cost of repairs or replacement of manufacturing defects found during the period
after purchase. If every product sold required minor repairs, the annual cost would be GHC0.5
million. If every product was replaced, the annual cost would be GHC3 million.

230
Based on trading history, it is believed that 80% of all goods sold will have no defects, 15% will
have minor defects and 5% will need replacing.
The warranty period covers just 12 months, and a provision should therefore be created for the
expected value of GHC225,000 [(80% × GHC0 nil) + (15% × GHC0.5 million) + (5% × GHC3
million)].

Discounting
Where the provision will not be settled for some years and so the effect of the time value of
money is material, the provision should be measured at its present value.
The discount rate used should be the pre-tax market rate that reflects the risks specific to the
liability.

Reimbursements
Where some or all of the costs to settle the provision will be reimbursed by another party, the
reimbursement should only be recognised when it is virtually certain that the reimbursement
will be received.
 The reimbursement should be treated as a separate asset in the statement of financial
position.
 The expense reported in profit or loss may be presented net of the reimbursement.

Accounting for specific provisions


The standard provides guidance on specific applications of the recognition and measurement
rules for provisions.

Future operating losses


An entity may forecast that it will make a substantial operating loss in the next year or several
years. If so, its directors might want to ‗take all the bad news‘ immediately, and create a
provision for the future losses.

Provisions cannot be made for future operating losses. This is because they arise from future
events, not past events.However, the expectation of future losses may indicate the need for an
impairment loss to be recognised for some of the entity‘s assets.

Onerous contracts
An onerous contract is a contract where the costs of fulfilling/completing the contract now
exceed the benefits to be received (the contract revenue).
A provision should be made for the additional costs of an onerous contract. (The ‗additional
costs‘ are the amount by which the costs are expected to exceed the benefits.)

Example
On 31 December 2016 Hazard Ltd is half way through an eight-year operating lease on its
factory when it moves to a new factory. Annual lease payments are GHC60,000. It cannot cancel
the lease or sub-let the factory.
Required
Consider whether or not the conditions for making a provision are met and at what amount, if
any, a provision should be made.

231
Answer
A present legal obligation exists as a result of a past event (the signing of the lease).
An outflow of resources is probable. (These are the rentals for the remainder of the term of the
lease.)
The amount can be measured reliably (GHC60,000 × 4 years, discounted to a present value).
The discounted value of the future lease payments for four years may therefore be recognised as
a provision.

Restructuring
An entity may plan to restructure a significant part of its operations. It expects the costs of
restructuring to be high. A provision may be made for the future restructuring costs only if a
present obligation exists.
A constructive obligation is likely to occur where, prior to the end of the reporting period:
 a detailed formal plan exists for the reconstruction
 and this plan has raised a valid expectation, in the mind of those people affected by the
restructuring, that the reconstruction will occur.
This may mean, for example, that the board of directors has drawn up a formal plan of
restructuring, and has announced that plan to the workforce or has made a public
announcement about it.
Any provision for restructuring costs that is created:
 should include only the expected direct expenses arising from the restructuring,and
 should not include ongoing expenses such as:
− the costs of retraining or relocating continuing staff
− the cost of marketing
− the cost of investing in new systems.

Environmental provisions and similar provisions


An entity may be required to ‗clean up‘ a location where it has been working when production
ceases. For example, an entity that operates an oil rig may have to repair the damage it has
caused to the sea bed once the oil has all been extracted.
An entity does not automatically have to recognise a provision if it has caused environmental
damage, even if it intends to clean up the site. The normal rules apply: an entity recognises a
provision only where it has an obligation to rectify environmental damage as a result of a past
event.
An entity has an obligation to ‗clean-up‘ a site if:
 it is required to do so by law (a legal obligation); or
 its actions have created a constructive obligation to do so.
A constructive obligation might exist if (for example) an entity has actually promised to
decontaminate a site or if it has adopted environmentally friendly policies and has made the
public aware of this.
A provision can only be recognised if the environmental damage has already happened (a past
event).

Example
An entity is about to begin to operate a coal mine. At the end of the reporting period the mine
shaft has been prepared and all the necessary equipment has been constructed and is in place,
but no coal has yet been extracted. Under local law, the entity is obliged to rectify all damage to

232
the site once the mining operation has been completed (this is expected to be several years from
now). Management estimates that 20% of the eventual costs of performing this work will relate
to removing the equipment and various buildings and the remaining 80% will relate to
restoring the damage caused by the actual extraction of coal.
Should a provision be recognised for the cost of restoring the damage?

Answer
The entity has a legal obligation to rectify the environmental damage caused by the actual
digging of the mine shaft and construction of the site. An outflow of economic benefits is
probable.
Therefore the entity should recognise a provision for the best estimate of removing the
equipment and rectifying other damage which has occurred to date. This is expected to be about
20% of the total cost of restoring the site.
Because no coal has yet been extracted, the entity has no obligation to rectify any damage
caused by mining. No provision can be recognised for this part of the expenditure (estimated at
about 80% of the total).

Accounting for a provision for environmental costs


When an entity has an obligation to clean up environmental damage, it normally has to
recognise a provision for expenditure which will take place many years in the future.
The estimated full cost of the expenditure should be recognised as soon as an obligation arises.
However, the provision is discounted to its net present value if the time cost of money is
material. (An examination question will make this clear and will give you the discount rate to
use.)
As well as recognising a liability for future expenditure, an entity normally recognises an asset.
Usually the entity cannot carry out its operations without agreeing to incur the expense of
cleaning up any damage that it causes. This expenditure meets the definition of an asset,
because it gives the entity access to
future economic benefits in the form of sales revenue.
The double entry is Dr Asset; Cr Provision (rather than Dr Expense; Cr Provision).
The asset is depreciated over its useful life in the same way as other non-current assets.
Example
Zlatan Ltd has constructed an oil rig which is due to become operational on 1st January 2011.
The entity has promised that when the oil rig is eventually decommissioned it will restore the
sea bed and clean up any contamination that it has caused. It is estimated that the cost of
decommissioning the oil rig will be GHC8 million and that decommissioning will take place in
ten years‘ time. The risk free cost of capital for the company is 10%. The oil rig is depreciated on
a straight line basis over its economic life of ten years.
Required:
Explain how the cost of decommissioning the oil rig should be treated in the financial
statements for the year ended 31st December 2011.

Answer
Zlatan has a constructive obligation to repair environmental damage as the result of a past
event (the construction of the oil rig). Therefore it must recognise a provision for the
decommissioning costs.

233
IAS 16 Property, plant and equipment states that the cost of an asset should include the
estimated costs of dismantling and restoring a site, if the entity has an obligation to incur these
costs. The decommissioning costs give Zlatan the right to future economic benefits in the form
of revenues from extracting and selling oil. Therefore the decommissioning costs are also
recognised as part of the cost of constructing the oil rig.
Because the costs will not be incurred for ten years, the estimated cost of GHC8 million is
discounted to its present value. Discount factors can be found from tables. If you have to
discount a provision in the examination, you will be given the discount factors that you need. In
this example:
 At 1 January 2011 the present value of GHC8 million is GHC1,912,000 (8 million × 0.239).
 At 31 December 2011 the present value of GHC8 million is GHC2,104,000 (8 million ×
0.263).
 The increase in the liability during 2011 (called the unwinding of the discount) is
recognised as a finance cost (similar to interest).
At 1 January 2011 Zlatan recognises:
 a provision of GHC1,912,000
 an asset for the same amount.
The statement of comprehensive income for 2011 includes in profit or loss:
 a finance cost of GHC192,000 (10% × 1,912,000) which is the unwinding of the discount;
and
 depreciation of GHC192,000 (1,912,000 / 10).

In the statement of financial position at 31 December 2011:


 the provision is measured at GHC2,104,000
 the asset is measured at its net book value of GHC1,720,000 (1,912,000 – 192,000).

Future repairs to assets


Some assets need to be repaired or to have parts replaced at intervals during their lives.
For example, suppose that a furnace has a lining that has to be replaced every five years. If the
lining is not replaced, the furnace will break down.
Before IAS 37 was issued, entities would often recognise provisions for the cost of future repairs
or replacement parts. These might be built up in installments over the life of the asset or the
relevant part of the asset.IAS 37 effectively prohibits this treatment. The reasoning behind this is
that an entity almost always has an alternative to incurring the expenditure, even if it is
required by law (for example, for safety reasons). For example, the entity which has to replace
the lining of its furnace could sell the furnace or stop using it, although this is unlikely in
practice.
IAS 37 states that a provision cannot be recognised for the cost of future repairs or replacement
parts unless the entity has an obligation to incur the expenditure. This is unusual. The
obligating event is normally the actual repair or purchase of the replacement part.
Instead of recognising a provision, an entity should capitalise the expenditure and depreciate it
over its useful life. This is the period until the next repair is required or the part needs to be
replaced again. For example, the cost of the furnace lining would be capitalised and depreciated
over five years. (IAS 16 Property, plant and equipment states that where an asset has two or
more parts with different useful lives, each part should be depreciated separately.)

Provisions: disclosure requirements

234
Prior to the introduction of IAS 37, only outline information was given on provisions. It was not
possible for the users of the financial statements to identify the creation, use and release of
provisions, because only the overall change for the year was disclosed.
IAS 37 introduced stringent disclosure requirements to allow users to fully understand the
impact of provisions on the period‘s financial statements.
For each class of provision (warranty, restructuring, refunds etc), an entity must disclose:
 the provision balance at the start and end of the period
 additional provisions made during the period
 amount of the created provision that was used in the period
 unused amounts reversed during the period, and
 the movement due to the unwinding of any discounting.

The entity must also give:


 a brief description of the need for the provision and the expected timing of its use
 an indication of any uncertainties surrounding the amount or timing of the outflows,
and
 the amount of any expected reimbursement.

CONTINGENT LIABILITIES AND CONTINGENT ASSETS


Definitions
Contingent liability
A contingent liability is either of the following:
A contingent liability is:
 a possible obligation
 arising from past events
 whose existence will be confirmed only by the occurrence or non-occurrence of one or
more uncertain future events.
A contingent liability is:
 a present obligation
 arising from past events
 which is not recognised as an actual liability (or provision) because
− an outflow of economic benefits is not probable, or
− the amount of the obligation cannot be estimated reliably.
A contingent liability arises when some, but not all, of the criteria for recognising a provision
are met. For example, a contingent liability exists, but not a provision or an actual liability if:
 a reliable estimate cannot be made, or
 no legal obligation or constructive obligation exists: there is merely a possible
obligation. Possible can be interpreted to mean less than 50% probability.
Example
Guardiola Ltd is involved in a legal dispute with a customer, who is making a claim against
Guardiola Ltd for losses it has suffered as a consequence of a breach of contract. If Guardiola‘s
solicitors believe that the likelihood of the claim succeeding is possible rather than probable,
and they estimate the possibility at about 30% to 40%. The claim should be treated as a
contingent liability and not as a provision.

Contingent asset
A contingent asset is:

235
 a possible asset
 arising from past events
 whose existence will be confirmed only by the occurrence or non-occurrence of one or
more uncertain future events.
An example of a contingent asset might be a probable gain arising from a current legal action
that has been taken against a third party, where the likelihood of success is considered to be
quite high. The existence of the asset (the money receivable) will only be confirmed by the
outcome of the legal dispute.

Recognising contingent liabilities or contingent assets


Unlike provisions, contingent liabilities and contingent assets:
 are not recognised in the financial statements, and
 are not recorded in the ledger accounts of an entity.
In some circumstances the existence of a contingent asset or a contingent liability is disclosed in
the notes to the financial statements
 Contingent liabilities are possible future liabilities (likelihood less than 50%) and should
be disclosed in a note to the financial statements unless the possibility of any outflow in
settlement is remote (the meaning of ‗remote‘ is not defined in IAS 37).
 Contingent assets are potential future assets and should be disclosed where an inflow
in settlement is probable. ‘Probable‘ is defined by IAS 37 as ‗more likely than not‘.
(However if the probability is very high, the item should be reported as an actual asset
and included in the statement of financial position.)

Disclosures about contingent liabilities and contingent assets


Where disclosure of a contingent liability or a contingent asset is appropriate, IAS 37 requires
the following disclosures in notes to the financial statements:
 A brief description of the nature of the contingent liability/asset
 Where practicable:
− an estimate of its financial effect
− an indication of the uncertainties.
 For contingent liabilities, the possibility of any reimbursement

CONTINGENCY TABLE

Event to be occured % of occurrence Contingent Liability Contingent Asset


Virtually certain 95% > Provide for Asset
Probable 50%-95% Provide for Disclose to notes
Possible 5%-50% Disclose to notes Do nothing
Remote 5%< Do nothing Do nothing

236
DECISION TREE FOR IAS 37.

TYPES OF PROVISIONS

Recognise a provision?
Circumstance
Restructuring by sale of Only when the entity is committed to a sale, i.e. there is a binding sale
an operation agreement.

Only when a detailed form plan is in place and the entity has started
Restructuring, to implement the plan, or announced its main features to those

237
Closure Or affected. A Board decision is insufficient.
reorganisation

Warranty When an obligating event occurs (sale of product with a warranty


and probable warranty claims will be made)

Land contamination A provision is recognised as contamination occurs for any legal


obligations of clean up, or for constructive obligations if the
company's published policy is to clean up even if there is no legal
requirement to do so (past event is the contamination and public
expectation created by the company's policy)

Recognise a provision if the entity's established policy is to give


Customer refunds
refunds (past event is the sale of the product together with the
customer's expectation, at time of purchase, that a refund would be
available)

Offshore oil rig Recognise a provision for removal costs arising from the construction
must be removed of the the oil rig as it is constructed, and add to the cost of the asset.
and sea bed restored Obligations arising from the production of oil are recognised as the
production occurs.

Abandoned leasehold,
A provision is recognised for the unavoidable lease payments
four years to run, no re-
letting possible

Firm must offer


staff No provision is recognised (there is no obligation to provide the
training forrecentchanges training, recognise a liability if and when the retraining occurs)
in tax law

No provision is recognised (no obligation)


Major overhaul or repairs

Onerous (loss-making) Recognise a provision


contract

238
Future operating losses No provision is recognised (no liability)

ILLUSTRATIONS WITH ANSWERS.


Illustration 1: Warranties
A manufacturer gives warranties at the time of sale to purchasers of its product. Under the
terms of the contract for sale the manufacturer undertakes to make good, by repair or
replacement, manufacturing defects that become apparent within three years from the date of
sale. On past experience, it is probable (i.e. more likely than not) that there will be some claims
under the warranties.

Solution
Present obligation as a result of a past obligating event - The obligating event is the sale of the
product with a warranty, which gives rise to an obligation.
An outflow of resources embodying economic benefits in settlement -
Probable for the warranties as a whole (see paragraph 23).
Conclusion - A provision is recognised for the best estimate of the costs of making good under
the warranty products sold before the balance sheet date

Illustration 2: Contaminated Land - Legislation Virtually Certain to be Enacted


An enterprise in the oil industry causes contamination but does not clean up because there is no
legislation requiring cleaning up, and the enterprise has been contaminating land for several
years. At 31 March 2005 it is virtually certain that a law requiring a clean-up of land already
contaminated will be enacted shortly after the year end.

Solution
Present obligation as a result of a past obligating event - The obligating event is the
contamination of the land because of the virtual certainty of legislation requiring cleaning up.-
An outflow of resources embodying economic benefits in settlement – Probable.
Conclusion - A provision is recognised for the best estimate of the costs of the clean-up.

Illustration 3: Offshore Oilfield


An enterprise operates an offshore oilfield where its licensing agreement requires it to remove
the oil rig at the end of production and restore the seabed. Ninety per cent of the eventual costs
relate to the removal of the oil
rig and restoration of damage caused by building it, and ten per cent arise through the
extraction of oil. At the balance sheet date, the rig has been constructed but no oil has been
extracted.

Solution;
Present obligation as a result of a past obligating event - The construction of the oil rig creates
an obligation under the terms of the licence to remove the rig and restore the seabed and is thus
an obligating event. At the balance sheet date, however, there is no obligation to rectify the
damage that will be
caused by extraction of the oil.
An outflow of resources embodying economic benefits in settlement – Probable.
Conclusion - A provision is recognised for the best estimate of ninety per cent of the eventual
costs that relate to the removal of the oil rig and restoration of damage caused by building it .

239
These costs are included as part of the cost of the oil rig. The ten per cent of costs that arise
through the extraction of oil are recognised as a liability when the oil is extracted.
Illustration 4: Refunds Policy
A retail store has a policy of refunding purchases by dissatisfied customers,even though it is
under no legal obligation to do so. Its policy of making refunds is generally known.

Solution;
Present obligation as a result of a past obligating event - The obligating event is the sale of the
product, which gives rise to an obligation because
obligations also arise from normal business practice, custom and a desire to maintain good
business relations or act in an equitable manner.
An outflow of resources embodying economic benefits in settlement -Probable, a proportion
of goods are returned for refund.
Conclusion - A provision is recognised for the best estimate of the costs of refunds.

Illustration 5: Legal Requirement to Fit Smoke Filters


Under new legislation, an enterprise is required to fit smoke filters to its factories by 30
September 2005. The enterprise has not fitted the smoke filters. (a) At the balance sheet date of
31 March 2005
Solution
Present obligation as a result of a past obligating event - There is no obligation because there
is no obligating event either for the costs of fitting smoke filters or for fines under the
legislation.
Conclusion - No provision is recognised for the cost of fitting the smoke filters.
(b) At the balance sheet date of 31 March 2006

Solution
Present obligation as a result of a past obligating event - There is still no obligation for the
costs of fitting smoke filters because no obligating event has occurred (the fitting of the filters).
However, an obligation might arise to pay fines or penalties under the legislation because the
obligating event has occurred (the non-compliant operation of the factory).

An outflow of resources embodying economic benefits in settlement - Assessment of


probability of incurring fines and penalties by non-compliant operation depends on the details
of the legislation and the stringency of the enforcement regime.
Conclusion - No provision is recognised for the costs of fitting smoke filters.However, a
provision is recognised for the best estimate of any fines and penalties that are more likely than
not to be imposed

Illustration 6: Staff Retraining as a Result of Changes in the Income Tax System


The government introduces a number of changes to the income tax system.As a result of these
changes, an enterprise in the financial services sector will need to retrain a large proportion of
its administrative and sales workforce in order to ensure continued compliance with financial
services regulation.At the balance sheet date, no retraining of staff has taken place.

Solution

240
Present obligation as a result of a past obligating event - There is no obligation because no
obligating event (retraining) has taken place.
Conclusion - No provision is recognised.
Illustration 7: A Single Guarantee
During 2004-05, Enterprise A gives a guarantee of certain borrowings of Enterprise B, whose
financial condition at that time is sound. During 2005-06, the financial condition of Enterprise B
deteriorates and at 30 September 2005 Enterprise B goes into liquidation.
(a) At 31 March 2005

Solution
Present obligation as a result of a past obligating event - The obligating event is the giving of
the guarantee, which gives rise to an obligation.
An outflow of resources embodying economic benefits in settlement -No outflow of benefits
is probable at 31 March 2005.
Conclusion - No provision is recognised. The guarantee is disclosed as a contingent liability
unless the probability of any outflow is regarded as remote.

(b) At 31 March 2006


Solution
Present obligation as a result of a past obligating event - The obligating event is the giving of
the guarantee, which gives rise to a legal obligation.
An outflow of resources embodying economic benefits in settlement – At 31 March 2006, it is
probable that an outflow of resources embodying economic benefits will be required to settle
the obligation.
Conclusion - A provision is recognised for the best estimate of the obligation.
Note: This example deals with a single guarantee. If an enterprise has a portfolio of similar
guarantees, it will assess that portfolio as a whole in determining whether an outflow of
resources embodying economic benefit is probable. Where an enterprise gives guarantees in
exchange for a fee, revenue is recognised under IAS 18 and IFRS 15, Revenue Recognition.

Illustration 8: A Court Case


After a wedding in 2004-05, ten people died, possibly as a result of food poisoning from
products sold by the enterprise. Legal proceedings are started seeking damages from the
enterprise but it disputes liability. Up to the date of approval of the financial statements for the
year 31 March 2005, the enterprise‘s lawyers advise that it is probable that the enterprise will
not be found liable. However, when the enterprise prepares the financial statements for the year
31 March 2006, its lawyers advise that, owing to developments in the case, it is probable that the
enterprise will be found liable.

(a) At 31 March 2005


Solution
Present obligation as a result of a past obligating event - On the basis of the evidence available
when the financial statements were approved, there is no present obligation as a result of past
events.
Conclusion - No provision is recognised. The matter is disclosed as a contingent liability unless
the probability of any outflow is regarded as remote.
(b) At 31 March 2006

241
Solution
Present obligation as a result of a past obligating event - On the basis of the evidence
available, there is a present obligation.
An outflow of resources embodying economic benefits in settlement -Probable.
Conclusion - A provision is recognised for the best estimate of the amount to settle the
obligation.

Illustration 9A: Refurbishment Costs - No Legislative Requirement


A furnace has a lining that needs to be replaced every five years for technical reasons. At the
balance sheet date, the lining has been in use for three years.

Solution
Present obligation as a result of a past obligating event - There is no present obligation.
Conclusion - No provision is recognised.
The cost of replacing the lining is not recognised because, at the balance sheet date, no
obligation to replace the lining exists independently of the company‘s future actions - even the
intention to incur the expenditure depends on the company deciding to continue operating the
furnace or to replace the lining.

Illustration 9B: Refurbishment Costs - Legislative Requirement


An airline is required by law to overhaul its aircraft once every three years.

Solution
Present obligation as a result of a past obligating event - There is no present obligation.
Conclusion - No provision is recognised.
The costs of overhauling aircraft are not recognised as a provision for the same reasons as the
cost of replacing the lining is not recognised as a provision in illustration 9A. Even a legal
requirement to overhaul does not make the costs of overhaul a liability, because no obligation
exists to overhaul the aircraft independently of the enterprise‘s future actions - the enterprise
could avoid the future expenditure by its future actions, for example by selling the aircraft.

Illustration 10: An onerous contract


An enterprise operates profitably from a factory that it has leased under an operating lease.
During December 2005 the enterprise relocates its operations to a new factory. The lease on the
old factory continues for the next four years, it cannot be cancelled and the factory cannot be re-
let to another user.

Solution
Present obligation as a result of a past obligating event - The obligating event occurs when the
lease contract becomes binding on the enterprise, which gives rise to a legal obligation.
An outflow of resources embodying economic benefits in settlement -When the lease
becomes onerous, an outflow of resources embodying economic benefits is probable. (Until the
lease becomes onerous, the enterprise accounts for the lease under IAS 17, Leases).
Conclusion - A provision is recognised for the best estimate of the unavoidable lease payments.

242
REVIEW QUESTIONS
QUESTION 1
At a board meeting on 1 July 2012, Pulsar‘s directors made the decision to close down one of its
factories on 31 March 2013. The factory and its related plant would then be sold.
A formal plan was formulated and the factory‘s 250 employees were given three months‘ notice
of redundancy on 1 January 2013. Customers and suppliers were also informed of the closure at
this date.
The directors of Pulsar have provided the following information:
Fifty of the employees would be retrained and deployed to other subsidiaries within the group
at a cost of GHC125,000; the remainder will accept redundancy and be paid an average of
GHC5,000 each.
Factory plant has a carrying amount of GHC2·2 million, but is only expected to sell for
GHC500,000 incurring GHC50,000 of selling costs; however, the factory itself is expected to sell
for a profit of GHC1·2 million.
The company rents a number of machines under operating leases which have an average of
three years to run after 31 March 2013. The present value of these future lease payments
(rentals) at 31 March 2013 was GHC1 million; however, the lessor has said they will accept
GHC850,000 which would be due for payment on 30 April 2013 for their cancellation as at 31
March 2013.
Penalty payments due to non-completion of supply contracts are estimated at GHC200,000.

Required:
Explain and quantify how the closure of the factory should be treated in Pulsar’s financial
statements for the year ended 31 March 2013.
Note: The closure of the factory does not meet the criteria of a discontinued operation. (6 marks)
(15 marks)
Solution:
On its own, a board decision to close the factory is not sufficient to justify the creation of a
provision under IAS 37 Provisions,Contingent Liabilities and Contingent Assets. However, by
formulating a plan and informing interested parties (employees,customers and suppliers), this
is likely to constitute a constructive obligation for a restructuring provision by raising a valid
expectation of the closure.
The amounts that should be provided for at 31 March 2013 are:
(workings in brackets are in GHC‘000)
GHC‘000
– redundancy (200 employees x 5) 1,000
– impairment loss on plant (2,200 – (500 – 50)) 1,750
(may be shown as a separate provision)
– onerous contract (lower amount) 850
– penalty payments 200
––––––
3,800
––––––

243
The GHC3·8 million should be charged to the statement of profit or loss for the year ended 31
March 2013 and the same amount reported in the statement of financial position as at 31 March
2013 as a current liability/plant impairment (assuming all parts of the factory closure will be
completed within the next 12 months).
The factory and the plant would be disclosed in the statement of financial position as non-
current assets held for sale at the lower of their carrying amount (the factory) or fair value less
cost to sell (the plant).
The GHC125,000 retraining costs cannot be provided for as they are part of future activities and
the anticipated GHC1·2 million profit on the disposal of the factory cannot be recognised until it
is realised.

QUESTION 2
Georgina Company is preparing its financial statements for the year ended 30 September 2017.
The following matters are all outstanding at the year end.
(a) Georgina is facing litigation for damages from a customer for the supply of faulty goods on 1
September 2017. The claim, which is for GHC500,000, was received on 15 October 2017.
Georgina‘s legal advisors consider that Georgina is liable and that it is likely that this claim will
succeed. On 25 October 2017 Georgina sent a counter-claim to its suppliers for
GHC400,000.Georgina‘s legal advisors are unsure whether or not this claim will succeed.
(b) Georgina‘s sales director, who was dismissed on 15 September, has lodged a claim for
GHC100,000 for unfair dismissal. Georgina‘s legal advisors believe that there is no case to
answer and therefore think it is unlikely that this claim will succeed.
(c) Although Georgina has no legal obligation to do so, it has habitually operated a policy of
allowing customers to return goods within 28 days, even where those goods are not faulty.
Georgina estimates that such returns usually amount to 1% of sales. Sales in September 2017
were GHC400,000. By the end of October 2017, prior to the drafting of the financial statements,
goods sold in September for GHC3,500 had been returned.
(d) On 15 September 2017 Georgina announced in the press that it is to close one of its divisions
in January 2018. A detailed closure plan is in place and the costs of closure are reliably
estimated at GHC300,000, including GHC50,000 for staff relocation.
Required
State, with reasons, how the above should be treated in Georgina‘s financial statements for the
year ended 30 September 2017.

Solution:
(a) Litigation for damages
Under IAS37, a provision should only be recognised when: „
 an entity has a present obligation as a result of a past event „
 it is probable that an outflow of economic benefits will be required to settle the
obligation „
 a reliable estimate can be made of the amount of the obligation.
Applying this to the facts given: „
 Georgina‘s legal advisors have confirmed that there is a legal obligation. This arose from
the past event of the sale, on 1 September 2017 (i.e. before the year end). „
 Probable is defined as ‗more likely than not‘. The legal advisors have confirmed that it is
likely that the claim will succeed. „
 A reliable estimate of GHC500,000 has been made.

244
Therefore a provision of GHC500,000 should be made.

Counter-claim
IAS37 requires that such a reimbursement should only be recognised where receipt is ‗virtually
certain‘. Since the legal advisors are unsure whether this claim will succeed no asset should be
recognised in respect of this claim.

(2) Claim for unfair dismissal


In this case, the legal advisers believe that success is unlikely (i.e. possible rather than probable).
Therefore this claim meets the IAS37 definition of a contingent liability: „
 a possible obligation
 arising from past events
 whose existence will be confirmed only by the occurrence or non- occurrence of one or
more uncertain future events.
The liability is a possible one, which will be determined by a future court case or tribunal. It did
arise from past events (the dismissal had taken place by the year end).
This contingent liability should be disclosed in the financial statements (unless the legal
advisors believe that the possibility of success is in fact remote, and then no disclosure is
necessary).

(3) Returns
Applying the IAS37 conditions in (1) to the facts given:
 Although there is no legal obligation, a constructive obligation arises from Georgina‘s
past actions. Georgina has created an expectation in its customers that such refunds will
be given.
 As at the year end, based on past experience, an outflow of economic benefits is
probable.
 A reliable estimate can be made. This could be 1% × 400,000 but since the returns are
now all in the actual figure of GHC3,500 can be used. Therefore a provision of GHC3,500
should be made.

(4) Closure of division


Applying the above IAS37 conditions in (1) to the facts given:
 A present obligation exists because at the year end there is a detailed plan in place and
the closure has been announced in the press.
 An outflow of economic benefits is probable.
 A reliable estimate of GHC300,000 has been made.
However, IAS37 specifically states in respect of restructuring that any provision should include
only direct expenses, not ongoing expenses such as staff relocation or retraining. Therefore a
provision of GHC250,000 (300,000 – 50,000) should be made.

QUESTION 3
IAS 37 Provisions, contingent liabilities and contingent assets sets out the principles of
accounting for these items and clarifies when provisions should and should not be made. Prior

245
to its issue, the inappropriate use of provisions had been an area where companies had been
accused of manipulating the financial statements and of creative accounting.
Required
(a) Describe the nature of provisions and the accounting requirements for them contained in
IAS 37. (6 marks)
(b) Explain why there is a need for an accounting standard in this area. Illustrate your answer
with three practical examples of how the standard addresses controversial issues.
(6 marks)
(c) Bodyline sells sports goods and clothing through a chain of retail outlets. It offers customers
a full refund facility for any goods returned within 28 days of their purchase provided they are
unused and in their original packaging. In addition, all goods carry a warranty against
manufacturing defects for 12 months from their date of purchase. For most goods the
manufacturer underwrites this warranty such that Bodyline is credited with the cost of the
goods that are returned as faulty. Goods purchased from one manufacturer, Header, are sold to
Bodyline at a negotiated discount which is designed to compensate Bodyline for manufacturing
defects. No refunds are given by Header, thus Bodyline has to bear the cost of any
manufacturing faults of these goods.
Bodyline makes a uniform mark up on cost of 25% on all goods it sells, except for those
supplied from Header on which it makes a mark up on cost of 40%. Sales of goods
manufactured by Header consistently account for 20% of all Bodyline's sales.
Sales in the last 28 days of the trading year to 30 September 2013 were GHC1,750,000. Past
trends reliably indicate that 10% of all goods are returned under the 28-day return facility.
These are not faulty goods. Of these 70% are later resold at the normal selling price and the
remaining 30% are sold as 'sale' items at half the normal retail price.
In addition to the above expected returns, an estimated GHC160,000 (at selling price) of the
goods sold during the year will have manufacturing defects and have yet to be returned by
customers. Goods returned as faulty have no resale value.
Required
Describe the nature of the above warranty/return facilities and calculate the provision Bodyline
is required to make at 30 September 2013:
(i) For goods subject to the 28 day returns policy
(ii) For goods that are likely to be faulty (8 marks)
(d) Rockbuster has recently purchased an item of earth moving plant at a total cost of GHC24
million. The plant has an estimated life of 10 years with no residual value, however its engine
will need replacing after every 5,000 hours of use at an estimated cost of GHC7.5 million. The
directors of Rockbuster intend to depreciate the plant at GHC2.4 million (GHC24 million/10
years) per annum and make a provision of GHC1,500 (GHC7.5 million/5,000 hours) per hour of
use for the replacement of the engine.
Required
Explain how the plant should be treated in accordance with International Financial Reporting
Standards and comment on the Directors' proposed treatment. (5 marks)
(Total = 25 marks)
Solution:
(a) Provisions and IAS 37 Provisions are liabilities of uncertain timing or amount. Because they
are liabilities they must meet the recognition criteria for liabilities - that is there must be:
• A present obligation arising from past transactions or events
• The transfer of economic benefits to settle the obligation must be probable

246
• A reliable estimate can be made of the amount of the obligation
The obligation giving rise to a provision can be legal or constructive. A constructive obligation
can arise when the actions or statements made by an entity create an expectation that they will
meet certain obligations, even if there is no legal requirement for them to do so; for example
they may have a well- known policy of replacing goods beyond the normal warranty period.
Provisions are recognised in full as soon as an entity is aware of them, but long term provisions
are recognised at present value. As time goes by the discount unwinds, increasing the
provision. The increase in the provision is charged to the income statement as a finance cost.
If an obligation depends upon a future event, then it is a contingent liability, not a provision.
Also, if the amount of an obligation cannot be measured reliably, then it is a contingent liability.
Contingent liabilities are not recognised in the financial statements, although they need to be
disclosed unless the possibility of an outflow of economic benefits is remote.
(In the past the term provision has been used to describe the reduction in the carrying value of
an asset; for example the provision for depreciation. This use of the word provision does not
meet the criteria of IAS 37, and so the term allowance is used instead; for example the allowance
for receivables.)

(b) The need for a standard


Although provisions have been a key area of financial reporting for many years, IAS 37 was the
first standard to address this issue. Before IAS 37 there were no rules governing the
• Definition
• Recognition
• Measurement
• Use, and
• Presentation of provisions.
IAS 37 definition of a provision as a liability of uncertain timing or amount means that
provisions must meet the recognition for liabilities. This means that provisions cannot be
created to suit management needs. In the past provisions were often created and released in
order to smooth profits, rather than to provide for a specific liability. These were sometimes
called 'big bath provisions' because they could be used for any and every purpose.
A specific example of this was the creation of provisions for reorganisation or restructuring. The
charge to set these provisions up could be explained away by management to their investors as
one-off exceptional items, but the release of the provision in the future would boost profits.
Under IAS 37 provisions for restructuring can only be recognised if the restructuring has begun
or if the restructuring has been announced publicly.
The measurement rules have standardised practice in an area where there were genuine
differences of opinion. For example there are at least three ways in which the cost of cleaning up
an industrial site after it is closed down (restoration costs) can be accounted for:
• ignore the costs until the site is abandoned,
• accrue the costs evenly over the productive life of the site, or
• provide for the costs in full immediately.
IAS 37 states that these costs should be provided for in full immediately, but at their present
value.
Under IAS 37 provisions can only be used for the purpose that they were created for; if a
provision is no longer needed it must be released. In the past provisions were sometimes
created for one purpose and then used to cover the costs of another.

247
IAS 37 includes detailed disclosure requirements, including the movement on provisions
during the year and an explanation of what each provision has been created for. This ensures
that the rules set out above have been complied with.

(c) This provision can be reliably measured on the basis of past experience. Although Bodyline
does not know which items will be returned or develop faults, it can make an estimate of the
total value of returns and faults that there will probably be.
The question does not make it clear whether the 28 day refund is part of the sales contract (in
which case it is a legal obligation) or whether it is just a well-known and established part of
Bodyline's trading practices (in which case it is a constructive obligation). Either way, an
obligation exists that needs to be provided for.
The provision itself can be reliably measured on the basis of past experience. Although Bodyline
does not know which items will be returned or develop faults, it can make an estimate of the
total value of returns and faults that there will probably be.

The returns provision is GHC52,850, calculated as follows:


• Of the 10% of sales that are returned under this policy, 70% are resold at the full price.
Therefore only the profit element is provided against on these items.
• A further 30% are sold at half the normal sales price, so the provision required will be half of
the sale proceeds.
GHC
70% resold at full price
Goods from Header (1.75m × 20% × 10% × 70% × 40/140) 7,000
Other goods (1.75m × 80% × 10% × 70% × 25/125 19,600
26,600
30% resold at half price (1.75m × 10% × 30% × ½ ) 26,250
52,850

The faulty goods provision is GHC57,600, calculated as follows:


• 20% of the goods returned will have been supplied by Header; Bodyline will suffer the loss in
full on these items.
• Bodyline reclaims the cost of the other 80% returned. Only the profit element is provided for.
GHC
Goods from Header GHC160,000 × 20% 32,000
Other goods GHC160,000 × 80% × 25/125 25,600
57,600

(d) No obligation exists to replace the engine and so it is wrong to create a provision for its
replacement. Rockbuster may decide to trade in the earthmover rather than replace the engine.
Also, the GHC2.4m depreciation charge includes an element in respect of the engine, so to make
a provision as well is double- counting.
Instead IAS 16 states that the earth-mover should be treated as an asset with two separate
components (the engine and the rest) with different useful lives. The engine (cost GHC7.5m)
will be depreciated on a machine hours basis over 5,000 hours, while the rest of the machine
(cost GHC16.5m) will be depreciated over ten years.

Cost Depreciation charge

248
GHC'000
Engine 7,500 Depreciated on a machine hours basis over 5,000hours. The charge is
GHC1,500 per hour
The rest 16,500 Depreciated on a straight line basis over its ten year useful life. The charge
is GHC1,650,000 per annum.
Total 24,000

When the engine is replaced the cost and accumulated depreciation on the existing engine will be retired
and the cost of the new engine will be capitalised and depreciated over its working life.

QUESTION 4
After the end of the reporting period, prior to authorising for issue the financial statements of
Tentacle for the year ended 31 March 2017, the following material information has arisen.
(i) The notification of the bankruptcy of a customer. The balance of the trade receivable due
from the customer at 31 March 2017 was GHC23,000 and at the date of the notification it was
GHC25,000. No payment is expected from the bankruptcy proceedings.
(3 marks)
(ii) Sales of some items of product W32 were made at a price of GHC5·40 each in April and May
2017. Sales staff receive a commission of 15% of the sales price on this product. At 31 March
2017 Tentacle had 12,000 units of product W32 in inventory included at cost of GHC6 each.
(4 marks)
(iii) Tentacle is being sued by an employee who lost a limb in an accident while at work on 15
March 2017. The company is contesting the claim as the employee was not following the safety
procedures that he had been instructed to use. Accordingly the financial statements include a
note of a contingent liability of GHC500,000 for personal injury damages. In a recently decided
case where a similar injury was sustained, a settlement figure of GHC750,000 was awarded by
the court. Although the injury was similar, the circumstances of the accident in the decided case
are different from those of Tentacle's case. (4 marks)
(iv) Tentacle is involved in the construction of a residential apartment building. It is being
accounted for using the percentage of completion basis in IAS 11 Construction contracts. The
recognised profit at 31 March 2017 was GHC1·2 million based on costs to date of GHC3 million
as a percentage of the total estimated costs of GHC6 million. Early in May 2017 Tentacle was
informed that due to very recent industry shortages, building materials will cost GHC1·5
million more than the estimate of total cost used in the calculation of the percentage of
completion. Tentacle cannot pass on any additional costs to the customer.
(4 marks)
Required
State and quantify how items (i) to (iv) above should be treated when finalising the financial
statements of Tentacle for the year ended 31 March 2017.
Note. The mark allocation is shown against each of the four items above.
(Total = 15 marks)
Solution:
(i) This is an adjusting event after the reporting period within the terms of IAS 10. GHC23,000
should be written off to irrecoverable debts at the year end and the trade receivables balance
correspondingly reduced.

249
(ii) In this case sales after the reporting period have demonstrated that the NRV of inventory
item W32 is below cost. In accordance with IAS 2, inventories of W32 should now be written
down to NRV as follows:
GHC
Cost (12,000 × 6) 72,000
NRV (12,000 × (5.4 × 85%)) (55,080)
Write off to income statement 16,920

(iii) As it is not yet known whether the employee's legal action will be successful, Tentacle is
correct to show it as a contingent liability. However, on the basis of the settlement in the other
case, the contingent liability should be increased to GHC750,000. If the case is settled before the
financial statements are authorised for issue, this will be an adjusting event requiring a
provision for damages if Tentacle is found liable.

(iv) IAS 11 requires that the profit on the contract be recalculated to take into account these
additional costs. Profit is based on costs to date as a percentage of total costs. As costs to date
are GHC3m and total costs GHC6m, the percentage is currently 50%. This will change when the
additional costs are included as follows:

Costs to date Total expected % Total expected Profit to


costs profit date
GHCm GHCm GHCm GHCm
Original 3 6 50% 2.4 1.20
Additional costs – 1.5 (1.5) –
Amended 3 7.5 40% 0.9 0.36

Recognised profit for the year to 31 March 2017 should therefore be restated as GHC360,000.
GHC840,000 (1.2m – 0.36m) will be written off in the income statement and adjusted in the
statement of financial position against amounts due to/from customers.

QUESTION 5
(a) The definition of a liability forms an important element of the International Accounting
Standards Board's Framework for the Preparation and Presentation of Financial Statements which, in
turn, forms the basis for IAS 37 Provisions, Contingent Liabilities and Contingent Assets.
Required
Define a liability and describe the circumstances under which provisions should be recognised.
Give two examples of how the definition of liabilities enhances the reliability of financial
statements. (5 marks)

(b) On 1 October 2007, Promoil acquired a newly constructed oil platform at a cost of GHC30
million together with the right to extract oil from an offshore oilfield under a government
licence. The terms of the licence are that Promoil will have to remove the platform (which will
then have no value) and restore the sea bed to an environmentally satisfactory condition in 10
years' time when the oil reserves have been exhausted. The estimated cost of this on 30
September 2017 will be GHC15 million. The present value of GHC1 receivable in 10 years at the
appropriate discount rate for Promoil of 8% is GHC0·46.
Required

250
(i) Explain and quantify how the oil platform should be treated in the financial statements of
Promoil for the year ended 30 September 2008; (7 marks)
(ii) Describe how your answer to (b)(i) would change if the government licence did not require
an environmental clean- up. (3 marks)
(Total = 15 marks)

Solution:
(a) The Framework defines a liability as a present obligation of an entity arising from past
events, the settlement of which is expected to result in an outflow from the entity of resources
embodying economic benefits. The obligation can be legal or constructive.
A provision is a liability of uncertain timing or amount. It can be recognised when the outflow
of resources is probable and when the amount concerned can be reliably estimated. Because it is
regarded as a liability, a provision must meet the definition of a liability. This regulates when a
provision should, or should not, be made. For instance, entities are not allowed to provide for
future operating losses, which used to be a means of 'profit smoothing', because the losses are in
the future, rather than arising from past events. At the same time, an entity which has a future
environmental liability because of past polluting activities, is required to make a provision as
soon as the liability becomes apparent.
(b) (i) Promoil must provide for dismantling and restoration costs at 30 September 2008, as the
liability came into existence with the granting of the licence and the cost has been reliably
estimated.
The provision at 30 September 2008 will be for the future cost discounted over 10 years. This
will be added to the carrying amount of the oil platform and depreciated over 10 years. The
discount will be 'unwound' each year and charged to finance costs. The credit entry will
increase the provision until at the end of 10 years it will stand at GHC15m.
At 30 September 2008:
INCOME STATEMENT
GHC'000
Depreciation (see SFP) 3,690
Finance costs (6,900 (see SFP) × 8%) 552
STATEMENT OF FINANCIAL POSITION
GHC'000
Non-current assets:
Oil platform 30,000
Dismantling (15m × .46) 6,900
36,900
Depreciation (36,900/10) (3,690)
Carrying value 33,210
Non-current liabilities:
Environmental provision at 1 October 2007 6,900
Discount unwound (6,900 × 8%) 552
7,452
(ii) If the government licence did not require an environmental clean up, Promoil would have
no legal obligation. It would then be necessary to determine whether or not Promoil had a
constructive obligation. This would apply if on past performance it had established a practice of
carrying out an environmental clean up where required, which would give rise to the
expectation that it would do so in this case.

251
If a constructive obligation existed, the accounting would be as per the above. If no obligation
were established, there would be no liability. No provision would be made for the clean-up. The
platform would be capitalised at GHC30m and depreciated over 10 years. There would be no
finance costs.
QUESTION 6
Blackburn is a local government organisation whose financial statements are prepared using
International Financial Reporting Standards.
Blackburn owns a warehouse. Chelsea has leased the warehouse from Blackburn and is using it
as a storage facility for chemicals. The national government has announced its intention to enact
environmental legislation requiring property owners to accept liability for environmental
pollution. As a result, Blackburn has introduced a hazardous chemical policy and has begun to
apply the policy to its properties. Blackburn has had a report that the chemicals have
contaminated the land surrounding the warehouse. Blackburn has no recourse against
Chelsea or its insurance company for the clean-up costs of the pollution. At 30 November 2012,
it is virtually certain that draft legislation requiring a clean up of land already contaminated will
be enacted shortly after the year end.
Required:
Discuss how the above events should be accounted for in the financial statements of Blackburn
(4 marks)
Solution:
A provision shall be recognised under IAS 37 Provisions, Contingent Liabilities and Contingent
Assets when there is a present obligation (legal or constructive) as a result of a past event, it is
probable that an outflow of resources embodying economic benefits or service potential will be
required to settle the obligation, and a reliable estimate can be made of the amount of the
obligation. If the above conditions are not met, no provision shall be recognised. In this case, the
obligating event is the contamination of the land because of the virtual certainty of legislation
requiring the clean up.
Additionally, there is probably going to be an outflow of resources embodying economic
benefits, because Blackcburn has no recourse against the entity or its insurance company.
Therefore a provision is recognised for the best estimate of the costs of the clean up. As
Blackburn has no recourse against Chelsea, recovery of the costs of clean up is not likely and
hence no corresponding receivable should be recorded.

QUESTION 7
A company spills chemicals onto Ghanaian land, causing damage that will cost GHC7m to
clean. There is no environmental legislation but the company has clear green policies on its
websites.
Required
Show the Financial Statement effects
Solution:
(R) Reasonably reliable estimate
There is already an estimate available in the sum of GHC7m.
(O) Obligation
There is a constructive obligation resultant from the public green policy.

Decision Obligation
Public Constructive

252
Secret None
(T) Transfer
Obviously money will flow out when the land is cleaned.
Conclusion
The costs must be provided.

Double entry
The following journal is appropriate:
Dr Cleaning costs (super exceptional) (P&L) GHC7m
Cr Provisions (B/S) GHC7m

QUESTION 8
A newspaper accuses a public figure of being mafia, even though they know this is not true.
The public figure sues and both sets of lawyers agree that it is likely that the public figure will
win the case and receive damages in the order of GHC1million.
There is no possibility of the case being resolved before the financial statements must be
finalised.
Required
State how the above litigation will be represented in the financial statements of both entities; the
newspaper and the public figure.
Solution:
Newspaper provides, public figure discloses

QUESTION 9
Sarkodie has two potential liabilities to assess. The first is an outstanding court case concerning a
customer claiming damages for losses due to faulty components supplied by Sarkodie. The
second is the provision required for product warranty claims against 200,000 units of retail goods
supplied with a one-year warranty.
The estimated outcomes of the two liabilities are:

Court case Product warranty claims


10% chance of no damages awarded 70% of sales will have no claim
65% chance of damages of GHS4m 20% of sales will require a GHS25 repair
25% chance of damages of GHS6m 10% of sales will require a GHS120 repair

Required:
Explain and quantify where possible, how the above items should be treated in Sarkodie's
financial statements for the year ended 31 March 2017. (3 marks)

Solution:
The two provisions must be calculated on different bases because IAS 37 Provisions, contingent
liabilities and contingent assets distinguishes between a single obligation (the court case) and a
large population of items (the product warranty claims).
For the court case the most probable single likely outcome is normally considered to be the best
estimate of the liability, ie GHS4m. This is particularly the case as the possible outcomes are
either side of this amount. The GHS4m will be an expense for the year ended 31 December 2017
and recognised as a provision.

253
The provision for the product warranty claims should be calculated on an expected value basis
at GHS3.4m (((70% × nil) + (20% × GHS25) + (10% × GHS120)) × 200,000 units). This will also be
an expense for the year ended 31 December 2017 and recognised as a current liability (it is a
one-year warranty scheme) in the statement of financial position as at 31 December 2017.
QUESTION 10
Nkoso Ltd (Nkoso) is an agro-processing company which reports under International Financial
Reporting Standards up to 31 December each year.

On 10 October 2019, Nkoso organised a Christmas party at the company‘s head office in Accra.
Unfortunately, there was an incident of food poisoning and the company has received 500 legal
claims from victims of the food poisoning, seeking compensation of an average of GH¢5,000
each. A letter from Nkoso‘s legal advisors, dated 10 December 2019, suggests 40% of these
claims are likely to be successful.

The Accountant of Nkoso does not want to make any provision for these claims on the grounds
that less than 50% of the cl aims are likely to be successful. The legal advisors have suggested
that an average of two years from the end of the current reporting period will elapse before the
claims are settled. The risk related discount rate is estimated to be 10%.

Required: In accordance with IAS 37: Provisions, Contingent Liabilities and Contingent Assets,
advise the directors of Nkoso on the appropriate accounting treatment of the above in the
financial statements for the year ended 31 December 2019. (6 marks)

SOLUTION
The food poisoning claims are covered by IAS 37 Provisions, Contingent Liabilities and
Contingent Assets. If there was a single claim, then it would be classified as a contingent
liability, and no provision should be recognised in the SFP. This is because the outcome is
possible not probable.

However, because there are 500 claims and each one has a 40% chance of succeeding, then
overall Nkoso would expect to lose 200 claims. Therefore, a provision should be recognised in
the SFP because:

Because the claims are not expected to be settled for another two years, the provision should be
discounted using the risk related time value of money. The provision should therefore be
carried in the SFP at 31 December at: 500 X 5,000 X 0.40 X 1/(1.10)2 = GH¢826,446
GH¢m GH¢m
Dr SPLOCI (retained earnings) 0.8
Cr Provisions (NCL) 0.8
Explanation of the treatment in IAS 37 or provisions and contingent liabilities, stating the
conditions: 2 marks
Amount of provision to be recognized taking into consideration the discount rate
2 marks Journal entries for the provision 2 marks

254
QUESTION 11
A newspaper accuses a public figure of being mafia, even though they know this is not true. The
public figure sues and both sets of lawyers agree that it is likely that the public figure will win
the case and receive damages in the order of GHC1million.
There is no possibility of the case being resolved before the financial statements must be
finalised.
Required
State how the above litigation will be represented in the financial statements of both entities; the
newspaper and the public figure.

Solution:
Newspaper provides, public figure discloses

INTANGIBLE ASSETS (IAS 38)


Objective
This Standard deals with the accounting treatment of Intangible Assets, which are not covered
by other accounting standards including the guidance for the main issues related to the
recognition & measurement of intangible assets, including relevant disclosure requirements.
Scope
The requirements of this standard are applicable for the accounting treatment of intangible
assets except for the following:
Financial assets, which are covered under IAS 32
Deferred tax assets, which are covered under IAS 12
Exploration and evaluation assets, which are covered under IFRS 6
Intangible assets which are held for sale and are covered under IAS 2
Goodwill acquired in a business combination which is under IFRS 3
Lease of intangible assets, which are covered under IAS 17
Long term intangible assets which are held for sale, and are covered under IFRS 5
Definitions
Cost
It is the amount of cash or cash equivalents paid or the fair value of the consideration
transferred to acquire, purchase or construct an asset.
Amortization
It is the systematic allocation of the depreciable amount of an intangible asset over its related
useful life.

Carrying Value
It is the value at which an intangible asset will be presented in the statement of financial
position, at the end of the reporting period, and it is determined as Cost less Accumulated
Amortization and Accumulated Impairment Loss.

255
Depreciable Amount
It is the amount of an asset, which will be depreciated over its useful and is determined as
the cost of an asset less its residual value.
Useful Life
It is the period of time for which asset will be used by the management.
Fair value
Itis amount that is expected to be received to sell an asset or required to be paid to transfer a liability, in
an orderly transaction between market participants at the date of measurement (IFRS 13).
Residual Value
It is the estimated net disposal proceeds that an entity would currently obtain from disposal of the
asset, if the asset were already in the condition and situation which is expected to be, at the end
of its useful life.
Development
It is the application of research findings or knowledge to produce new or significantly
improved material, device, product, process, system or service before the start of commercial
production or use.
Research
It is the original and planned investigation undertaken for the purpose of gaining new scientific
or technical information and understanding.
Intangible Assets
These are identifiable, non-monetary assets and do not have physical existence, controlled by
entity from which future economic benefits are expected.
Identifiable
As per the definition of intangible asset under this standard, an intangible asset must be
identifiable, to be distinguished from the goodwill, which is covered under IFRS 3. An asset will
be identifiable if it meets any one of the following:
(a) It is separable, i.e. it is capable of being separated from the business entity and sold,
transferred, licensed, rented or exchanged, either on individual basis or along with the related
contractor
(b) It arises from a contractual legal right, irrespective of the fact whether those rights are
transferable or separable from the business entity.
Control
The definition of intangible asset requires that the intangible asset must be controlled by the
entity, and an entity controls an intangible asset if it has ability to obtain economic benefits
related to the asset and can restrict others from such benefits. Normally control arises through a
legal contract or when an entity has absolute right of use of asset. For example:
Market share, customer loyalty and staff technical knowledge may give rise to future economic
benefits. An entity controls those benefits if, these are protected by having customer contracts,
trade agreements and legal contracts with employees. However, in the absence of legal contract
entity has insufficient control over related future economic benefits and in such a case these will
not qualify to be recognize as intangible assets.
Recognition and Measurement
An intangible asset may arise in following ways:
1. Intangible Assets Separately Acquired:
These are individually purchased from the external parties and these will be recognized, if
following criteria is satisfied:
(a) It should meet the definition of intangible asset and

256
(b) The recognition criteria given in IASB‘s framework i.e.
 The future economic benefits are probable to flow to the entity and
 The cost of the asset is reliably measurable.
These are initially measured at Cost, which comprises:
 Purchase Price
 Less any Trade Discount or Rebate,
 Plus any directly related cost which includes
Sales tax and Import duties (if non-refundable), Legal charges, Pre-production testing cost (Net
expense) and any other cost which is essential in bringing the asset into its operating or
intended use by the management.
The capitalization of cost will cease when the asset becomes available for operating or intended
use by the management.
Notes:
(a) Following elements of cost will not be added to the cost of asset rather these will be charged
to statement of profit or loss as an expense:
 Relocation cost
 Any general and administrative overheads
 Initial operating losses
(b) If an intangible asset is purchased on extended credit period or on deferred installment
basis, then the cost of such intangible asset will be its Cash Price Equivalent any excess paid over
the cash price will be treated as Interest expense which will be recognized over the period of
credit.
2. Intangible Assets acquired as part of Business Combination:
If there is an intangible asset related to subsidiary at the date of business combination, the
acquirer will recognize such intangible asset in the consolidated financial statements in
accordance with IFRS 3, separately from the goodwill at Fair Value at the date of business
combination, and it is irrelevant whether that asset has been recognized in the financial
statements of subsidiary such as brand name, trademarks, customer relationships and market
share.
3. Acquisition by way of Government Grant:
An entity may acquire an intangible asset free of cost, or for nominal consideration, as a result
of a government grant. It may be the case when government transfers to the entity intangible
assets such as airport landing rights, telecommunication license, or import license.
The entity will recognize such intangible assets either at Fair Value or at Nominal Cost
including directly attributable expenditure in accordance with IAS 20.
4. Intangible Asset acquired in Exchange:
An entity may acquire an intangible asset in exchange for a non-monetary asset or combination
of non-monetary and monetary assets, then the cost of the intangible asset acquired in exchange will
be determined as follows:
(a) If Transaction of Exchange has Commercial Substance:
The transaction of exchange will be deemed to have commercial substance if:
 The risk, timing and amount of cash flows related to the intangible asset acquired are
different from the asset transferred;
 The exchange has resulted in the change in the entity specific value of that operational
portion of the entity
 The change in (a) and (b) above is material.

257
In such circumstances the entity will determine the cost of the Intangible asset acquired in
exchange as:
i. The fair value of asset transferred ± cash,
ii. If the fair value of asset transferred is not determinable , then it will be recognized at the
fair value of asset acquired,
Any gain or loss on the exchange transaction will be charge to statement of profit or loss.
(b) If Transaction of Exchange does not have Commercial Substance:
If the transaction of exchange does not have commercial substance or the fair value of asset
transferred and the intangible asset acquired both are not determinable, then the intangible
asset acquired will be recognize at the carrying value of asset transferred, which will result in
no gain or loss on exchange.
5. Internally Generated intangible Assets:
These are generated by the entity using its own resources over the passage of time. These are
accounted for as follows:

i. Internally Generated Goodwill


Internally generated goodwill, brand name, customer loyalty, market share, labor skills or
advance knowledge, mastheads, trademarks and advertisement costs are not allowed to be
recognized as an intangible asset because:
a. These are not identifiable as per the definition of intangible asset i.e. (not separable from
the business) and
b. These do not have cost distinguished from the regular activities of the business.
ii. Other Internally Generated Intangible Asset
Other internally generated intangible assets which are generated by the entity as part of
research and development activities will be accounted for, by dividing these into following two
phases:
(a) Research Phase
(b) Development Phase

(a) Research Phase


Research phase is the initial or inception stage of the project and the entity cannot demonstrate
that an intangible asset exists which will generate economic benefits in future. Therefore, the
whole of the cost incurred during the research phase will be charged to statement of profit or
loss as an expense.
The research phase activities may include:
 Activities undertaken by the entity for the purpose of obtaining new knowledge;
 Activities undertaken to find out alternatives for material, device, product, or a
processes.
(b) Development Phase
 The development phase comes after the initial research phase and it may include the
following activities:
i. The design, preparation, finalization and testing of prototypes or models before the start
of commercial use
ii. The designing and preparation of tools, jigs, structures and dies containing advance or
new technology
iii. The designing, preparation and testing of a final selected alternative for new or
advanced material, device, product, process, or a systems.

258
ACCOUNTING TREATMENT OF DEVELOPMENT COSTS
Development costs must be recognised as an intangible asset, but only if all the following
conditions can be demonstrated.
 It is technically feasible to complete the development project.
 The entity intends to complete the development of the asset and then use or sell it.
 The asset that is being developed is capable of being used or sold.
 Future economic benefits can be generated. This might be proved by the existence of a
market for the asset‘s output or the usefulness of the asset within the entity itself.
 Resources are available to complete the development project.
 The development expenditure can be measured reliably (for example, via costing
records).

If any of these conditions is not met, the development expenditure should be treated in the
same way as research costs and recognised in full as an expense when it is incurred, and cannot
be treated as an intangible asset. Once such expenditure has been written off as an expense, it
cannot subsequently be
reinstated as an intangible asset. Expenditure on development may only be capitalised and
treated as an intangible asset if it is incurred after all the conditions for recognition of the
development costs as an intangible asset have been met.
 If the cost under development phase does not meet the above capitalization criteria, it
will be charged to the statement of profit or loss as an expense.
 The capitalization of development phase cost will commence right from the date, when
the project meets the capitalization criteria as above.
 The costs under development phase that may be capitalized as an intangible asset
include:
(a) Cost of material used for development
(b) Cost of employee services, which are engaged in the development of project
(c) Professional fee paid to get the legal title of asset (registration fee)
(d) Any depreciation or amortization of the assets that are used for the development of
intangible asset.
(e) Cost of preparing the prototype, before the start of commercial production
(f) Any other cost which incurred for the development of the intangible asset.
(g) Any interest cost incurred under development phase, if it satisfies the criteria given in IAS
23.
 The capitalization of cost will cease when the asset becomes available for operating or
intended use by the management.
 Following are the elements of cost under development phase which cannot be
capitalized, and are charged to the statement of profit or loss as an expense:
(a) Any cost of administrative and selling overheads
(b) Cost of initial operating losses
(c) Cost of staff training to operate the asset
 The development phase cost which is initially recognized as expense is allowed to be
reinstated as intangible asset later on, in future.
 If the entity is unable to differentiate between the research and development phase then
all the cost incurred on the project will be charge to the statement of profit or loss as an
expense.

259
 If the capitalized development cost as an intangible asset:
(a) Has limited life, then it will be amortized using appropriate method as per the pattern of
economic benefits, if pattern of economic benefits is not identifiable, then use straight line
method.
(b) Has unlimited useful life, then it will not be amortized and entity is required to apply annual
impairment test as per IAS 36.
 The amortization of intangible asset will start when it is available for commercial use by
the entity.
In-Process Research and Development Project acquired Separately or in a Business
Combination:
 Any in-process research and development project acquired separately or as part of a
business combination can be recognized as an intangible asset, if it meets the definition
of intangible asset.
 Any expenditure on the in-process and development project incurred after the
acquisition will be treated as follows:
(a) It will be recognized as an expense, if it is research expenditure
(b) It will be recognized as an expense, if it is development expenditure and does not satisfy the
capitalization criteria
(c) It will be capitalized in the carrying value of the project, if it is development expenditure and
satisfies the capitalization criteria
Example
Winners Ltd has undertaken the following activities during 2015.
(1) Training of sales staff at a cost of GHC50,000. Additional revenue as a result of training the
staff to a higher level of skill is expected to be in the region of GHC500,000.
(2) Development of a new product. Total expenditure on development of the product has been
GHC800,000. All of the conditions for recognising the development costs as an intangible asset
have now been met. However, GHC200,000 of the GHC800,000 was spent before it became clear
that the project was technically feasible, could be resourced and the developed product would
be saleable and profitable.
Required
Consider how the above amounts will be dealt with in the financial statements of Winners Ltd
for 2015.

Answer
Training costs. These must be written off as an expense in full when they are incurred. The
‗asset‘ (the training costs) is not controlled by Winners Ltd. The cost is controlled by the staff
who could leave at any time.
Development costs. The GHC200,000 incurred before all of the conditions for recognising the
development costs as an intangible asset were met must be written off as an expense. The
remaining GHC600,000 should be capitalised and recognised as an intangible asset
(development costs).

Example
Doug Co is developing a new production process. During 2013, expenditure incurred was
GHC100,000, of which GHC90,000 was incurred before 1 December 2013 and GHC10,000
between 1 December 2013 and 31 December 2013. Doug Co can demonstrate that, at 1 December

260
2013, the production process met the criteria for recognition as an intangible asset. The
recoverable amount of the know-how embodied in the process is estimated to be GHC50,000.
How should the expenditure be treated?

Solution
At the end of 2013, the production process is recognised as an intangible asset at a cost of
GHC10,000. This is the expenditure incurred since the date when the recognition criteria were
met, that is 1 December
2013. The GHC90,000 expenditure incurred before 1 December 2013 is expensed, because the
recognition criteria were not met. It will never form part of the cost of the production process
recognised in the statement of financial position.
Subsequent Measurement:
The entity has two options to account for the Intangible assets at reporting date as a choice of
accounting policy;
1. Cost Model
2. Revaluation Model
1. Cost Model:
If an entity chooses to measure the intangible asset under Cost model at reporting date, then such
intangible assets will be measured at Cost less accumulated amortization less accumulated
impairment loss.

Amortization:
(a) It is the systematic allocation of the depreciable amount of an intangible asset over its related
useful life.
(b)The amortization charge of the intangible asset for the accounting period will be charged to
the statement of profit or loss as an expense. However, if the intangible asset is being used in
the construction of another asset, then the amortization charge will be added to the cost of such
asset under construction or being produced.
(c) The residual value of the intangible asset will assumed to be zero unless an active market
exists, from which residual value can be determined or there is purchase commitment from the
third party at the end of its useful life.
(d) The entity will continue to amortize the intangible asset which has limited useful life, even if
the fair value of asset is higher than its carrying value. However, any amortization will not be
charged if the residual value of the intangible asset exceeds its carrying value.
(e) The amortization charge will commence, when the intangible asset is available for operating
use or intended use by the management.
(f) The entity will cease amortization charge when either, the intangible asset is classified as
held for sale under IFRS 5, or the intangible asset is de-recognized from the statement of
financial position.
(g) The intangible assets having indefinite life will not be amortized and will be tested for
impairment annually as per IAS 36.
Amortization Method:
(a) The amortization method should reflect the pattern of economic benefits from the intangible
asset. If the pattern of economic benefits is not determinable, then use straight line method.
(b) The entity should review the amortization method selected at each reporting date and if
there is any change in the pattern of consumption of economic benefits related to the intangible
asset, then the entity should change the amortization method in accordance with the new

261
pattern of consumption of economic benefits and such change will be accounted for as change
in accounting estimate, which will be applied prospectively from that date.
Useful Life:
 The entity should consider the following aspects in determination of the useful life of the
asset:
(a) The expected use of the intangible asset including its production capacity or output.
(b) The product life cycle related to the intangible asset.
(c) Any expected change in the demand of the product related to the intangible asset due
to commercial or technical changes in the market.
(d) Any legal restriction on the asset in terms of its use.
(e) Future actions of competitors in the market.
(f) Any subsequent expense required on the intangible asset to obtain future economic
benefits
 An intangible asset will be deemed to have an indefinite useful life if, based on an
analysis of all of the relevant factors, there is no determinable time period over which
the asset is expected to generate economic benefits for the entity.
 The entity should review the useful life the intangible asset at each reporting date, if it
has changed as of the original estimate the entity should also revise the useful life
accordingly following the change, it will be accounted for as change in accounting
estimate, and it will have Prospective Application in accordance with IAS 8.
 The useful life of the asset is a matter of judgment, according to the expected use of the
asset by management.
 The useful life of the asset which arises from a legal contract should not exceed its
contractual life.
Impairment:
Any impairment will be determined as per the requirements of IAS 36.
2. Revaluation Model:
If an entity chooses to measure the intangible asset under Revaluation model at reporting date,
then such assets will be measured at Revalued Amount less subsequent accumulated amortization less
subsequent accumulated impairment loss.
The entity should consider the following points in revaluation:
(a) Normally the revalued amount of intangible asset is taken as fair value from the active
market.
(b) The standard does not allow the revaluation for the intangible assets which are initially
measured at other than the cost.
(c) The frequency of revaluation depends upon the volatility of the market related to the
intangible asset.
(d) Revaluation should be performed regularly enough, so that the carrying value of intangible
asset should not be materially different from its revalued amount.
(e) When the intangible asset is revalued, its accumulated amortization charge to the date of
revaluation will be reset to zero, as it will be reflected in its revalued amount.
(f) Once an intangible asset is revalued, the whole class of such intangible asset has to be
revalued, to avoid the presentation of assets in the same category at different cost and values
with different valuation dates. However, if no active market exits for a particular intangible
asset it will be measured at cost less accumulated amortization less accumulated impairment
loss.

262
(g) Any increase in the carrying value of the intangible asset resulting from revaluation will be
recognized in other comprehensive income and will be accumulated in a separate column of the
statement of changes in equity until the disposal date. However first, it will reverse any loss
related to the same intangible asset up to the extent it is recognized in the previous years.
(h) Any decrease in the carrying value of the intangible asset resulting from the revaluation will
be recognized in statement of profit or loss as expense. However first, it will offset any
revaluation surplus related to the same intangible asset up to the extent it is recognized in the
previous years.
(i) If amortization charge on the basis of revalued amount of the intangible asset exceeds the
original depreciation charge, then the excess will be transferred out of the revaluation surplus to
the retained earnings as realization of the revaluation surplus. However, this transfer
is optional and if opted by the entity then it will be applicable annually till the disposal of related
intangible asset.
(j) Any remaining revaluation surplus in the statement of changes in equity will be transferred
as whole to the retained earnings when the related intangible asset is de-recognized from the
statement of financial position.
Example
An intangible asset is measured by a company at fair value. The asset was revalued by GHC400
in 2013, and there is a revaluation surplus of GHC400 in the statement of financial position. At
the end of 2014, the asset is valued again, and a downward valuation of GHC500 is required.

Required
State the accounting treatment for the downward revaluation.
Answer
In this example, the downward valuation of GHC500 can first be set against the revaluation
surplus of GHC400.
The revaluation surplus will be reduced to 0 and a charge of GHC100 made as an expense in
2014.
Disposal
An entity will de-recognize the intangible asset from statement of financial position when:
(a) The intangible asset is disposed off or
(b) No economic benefits are expected either from use or from sale of the intangible asset
 Any gain or loss on the disposal of intangible asset will be charged to the statement of
profit or loss which will be the difference between carrying value of the intangible asset
and its disposal proceeds.
 If the intangible asset is sold on extended credit period or on deferred installment basis,
then its disposal proceeds will be taken as cash price equivalent and any excess over the
cash price will be treated as Interest Income which will be recognized over the period of
credit.
Disclosures
The entity is required to disclose the following in respect of the intangible assets:
 Intangible assets having indefinite useful life
 Useful life of Intangible assets having limited useful life
 Amortization method used for the intangible assets with limited useful life
 Carrying value and accumulated amortization at the start of the year
 The line item in the statement of profit or loss, in which amortization charge is included.

263
 A statement reconciling the carrying value at the start of the period to the carrying value
at reporting date which includes:
(a) Any additions identifying the intangible assets separately purchase and internally
developed along with any disposals during the year
(b) Any assets acquired as part of a business combination
(c) Any revaluation increase or decrease or impairment loss recognized in the current year
(d) Depreciation charge for the year
(e) Assets classified as held for sale under IFRS 5
(f) Any exchange differences arising on translation of foreign currency assets.
 Any change in accounting estimate during the current year such as change in residual
value, useful life or amortization method
 For intangible assets having indefinite life, the factors supporting such assessment
 Intangible assets which are acquired by way of government grant and are recognized at
fair value, the entity will disclose:
(a) Initial fair value
(b) Carrying value at year end
(c) Measurement model at reporting date
 Intangible assets which are subject to pledge arrangements
 Class of intangible assets which are revalued
 Date of revaluation
 Carrying value of such intangible assets
 Carrying value that would have been if such assets have not been revalued
 Amount of expenditure under research phase in the current year.
 Intangible assets which are fully amortized but are still under use by the entity.
Worked Examples
Example 1
AB Ltd started a research and development project to develop a new production process on 1
January 2011. The research phase lasted on 30 April 2011 and incurred a cost of GHC500. The
development phase started from 1 May 2011 and incurred a total expenditure of GHC2,000 up
to 31 December 2011, of which GHC1800 was incurred up to 1 December 2011, and GHC200
was incurred between 1 December 2011 and 31 December 2011.
The director of AB Ltd stated that, at 1 December 2011, the production process met the criteria
for recognition as an intangible asset.
During 2012, further expenditure incurred was GHC4,000 and the project was completed
successfully. However, At the end of 2012, the recoverable amount of this new production
process is estimated to be GHC3800.
Required:
How this will be accounted for in the financial statements of AB Ltd for the year ended 31
December 2011 and 31 December 2012.
Solution
Year ended 31 December 2011:
 Research expense of GHC500 up to 30 April 2011 and development expenditure of
GHC1,800 from 1 May 2011 to 1 December 2011 will charge to statement of profit or loss
as an expense.
 As the capitalization criteria are met on 1 December 2011 therefore, development cost of
GHC200 from 1 December 2011 to 31 December 2011 will be capitalized as intangible
asset in the year ended 31 December 2011.

264
Year Ended 31December 2012:
 The development cost of GHC4,000 will be added capitalized development expenditure
recognized in the previous year. Hence total capitalized development expenditure as
intangible asset is GHC4,200.
 However, this has a recoverable value of GHC3,800 in comparison with carrying value
of GHC3800 which will result in impairment loss of GHC400 in the year ended 31
December 2012.
Example 2:
AB Ltd is a public listed company. It has asked for your opinion in respect of the accounting
treatment of the following matter for the year to 31 December 2011.
i. On 1 December 2011 AB Ltd acquired Darby, a small pharmaceutical drug company that
specializes in research and development. The purchase consideration was GHC70
million. The fair value of Darby's net assets was GHC30 million (excluding the items
referred to below). Darby owns a patent for an established successful drug that has a
remaining life of 8 years. A firm of experts has estimated the current value of this patent
to be GHC20 million. Also included in Darby's statement of financial position is GHC4
million for medical research that has been conducted on behalf of a client.
ii. Darby has developed and patented a new drug which has been approved for medical
use. The costs of developing the drug were GHC24 million. Based on early assessments
of future cash flows, it has an estimated market value at GHC40 million.
iii. Darby's manufacturing process has recently received a favorable inspection by
government officials. Consequently, the company has been allotted a license free of cost,
for a period of 10 years to manufacture a new drug. The firm of experts has placed a
value of GHC20 million on it.
iv. Darby has spent GHC6 million sending its staff on specialist training courses, in the
current year. These courses have been expensive, they have led to a marked
improvement in production quality, increase in revenue and cost reductions. The
directors of Darby believe these benefits will continue for at least two years and wish to
treat the training costs as an intangible asset.
Required
Explain how the above items will be treated in the financial statements of AB Ltd for the year to
31 December 2011.
Solution:
(i)
 Intangible assets may arise as a result of business combination and this standard
requires intangible assets related to subsidiary which arises as part of business
combination, to be recognized at fair value, separately from the goodwill. Therefore, the
patents related to Darby will be recognized at fair value of GHC20 million.
 As the research project is acquired as part of business combination and this is being
conducted on behalf of a client not for the Darby‘s own business, therefore, it will also be
recognized as intangible asset.
 Following assets will be recognized on the date of acquisition:

GHC‘million
Cost of Investment 70
- Fair value of Net Assets acquired:

265
Net Assets (30)
Patents (20)
In-Process Research for client (4)
Goodwill 16

(ii)
The development phase cost of the internally generated intangible asset is capitalized as
intangible asset if it satisfies the capitalization criteria given in IAS 38. As it is stated that the
drug has been approved for clinical use, therefore it will be recognized as an intangible asset at
its Cost of GHC24 million. The market value is irrelevant at initial recognition.
(iii)
IAS 38 requires, intangible assets which arises as a result of government grant are recognized
either at fair value or nominal cost. Therefore, the license received from government will be
recognized at a fair value of GHC20 million.
(iv)
IAS 38 does not allow the recognition of training cost as an intangible asset as the future actions
of employees are not in the control of the entity. Therefore, such cost will be charged to the
statement of profit or loss as expense.

REVIEW QUESTIONS
QUESTION 1
With specific reference to IAS 38-‗Intangible Assets‘,
Outline five criteria that must be satisfied before development costs of a project can be
capitalized and recognized as an asset.
QUESTION 2
During 2012 Henry has the following research and development projects in progress.
Project A was completed at the end of 2011. Development expenditure brought forward at the
beginning of 2012 was GHC412,500 on this project. Savings in production costs arising from this
project are first expected to arise in 2012. In 2012 savings are expected to be GHC100,000,
followed by savings of GHC300,000 in 2013 and GHC200,000 in 2013.
Project B commenced on 1 April 2012. Costs incurred during the year were GHC56,000. In
addition to these costs a machine was purchased on 1 April 2012 for GHC30,000 for use on the
project. This machine has a useful life of five years. At the end of 2012 there were still some
uncertainties surrounding the completion of the project.
Project C had been started in 2011. In 2011 the costs relating to this project of GHC36,700 had
been written off, as at the end of 2011 there were still some uncertainties surrounding the
completion of the project. Those uncertainties have now been resolved and a further GHC45,000
costs incurred during the year.
Required
Show how the above would appear in the financial statements (including notes to the financial
statements) of Henry as of 31 December 2012.

SOLUTION:

266
Property, plant and equipment
Plant and machinery
Cost GHC
On 1 January 2012 X
Additions 30,000
–––––––
On 31 December 2012 X
–––––––
Accumulated depreciation
On 1 January 2012 X
Charge for the year (30,000 × 9/12 ÷ 5) 4,500
–––––––
On 31 December 2012 X
–––––––
Carrying amount
On 31 December 2011 X
–––––––
On 31 December 2012 25,500
–––––––

Intangible assets
Internally generated research and
development expenditure
Cost GHC
On 1 January 2012 412,500
Additions 45,000
––––––––
On 31 December 2012 457,500
––––––––
Accumulated amortisation
On 1 January 2012 -
Charge for the year (W) 68,750
––––––––
On 31 December 2012 68,750
––––––––
Carrying amount
On 31 December 2011 412,500
––––––––
On 31 December 2012 388,750
––––––––
Workings
Amortisation charge (Project A) GHC
Total savings (100,000 + 300,000 + 200,000) 600,000
2012 amortisation charge (100,000/600,000 × 412,500) 68,750

267
Tutorial notes
The costs in respect of Project B cannot be capitalised as there are uncertainties surrounding the
successful outcome of the project – but the machine bought may be capitalised in accordance
with IAS16.
The 2012 costs in respect of Project C can be capitalised as the uncertainties have now been
resolved. However, the 2011 costs cannot be reinstated.

QUESTION 3
Tonaldo entered into the following transactions during the year ended 31 December 2011. The
directors of Tonaldo wish to capitalise all assets wherever possible.
(1) On 1 January Tonaldo acquired the net assets of George for GHC105,000. The assets acquired
had the following book and fair values.
Book value Fair value
GHC GHC
Goodwill 5,000 5,000
Patents 15,000 20,000
Non-current assets 40,000 50,000
Other sundry net assets 30,000 25,000
––––––– ––––––––
90,000 100,000
––––––– ––––––––
The patent expires at the end of 2018. The goodwill arising from the above had a recoverable
value at the end of 2011 of GHC7,000.
(2) On 1 April Tonaldo acquired a brand from a competitor for GHC50,000. The directors of
Tonaldo have assessed the useful life of the brand as five years.
(3) During the year Tonaldo spent GHC40,000 on developing a new brand name. The
development was completed on 30 June. The useful life of this brand has been assessed as eight
years.
(4) The directors of Tonaldo believe that there is total goodwill of GHC2 million within Tonaldo
and that this has an indefinite useful life.
Required
Prepare the note to the financial statements for intangible assets as at 31 December 2011.
Solution:
Intangible assets Goodwill Patents Brands Total
GHC GHC GHC GHC
Cost
On 1 January 2011 - - - -
Additions (W1) 10,000 20,000 50,000 80,000
––––––– ––––––– ––––––– –––––––
On 31 December 2011 10,000 20,000 50,000 80,000
––––––– ––––––– ––––––– –––––––

Goodwill Patents Brands Total


GHC GHC GHC GHC
Accumulated amortization
/impairment

268
On 1 January 2011 - - - -
Written off/amortised during
the year (W1 and W2) 3,000 2,500 7,500 13,000
––––––– ––––––– ––––––– –––––––
On 31 December 2011 3,000 2,500 7,500 13,000
––––––– ––––––– ––––––– –––––––
Carrying amount
On 31 December 2010 - - - -
––––––– ––––––– ––––––– –––––––
On 31 December 2011 7,000 17,500 42,500 67,000
––––––– ––––––– ––––––– –––––––
Workings
(1) Goodwill on acquisition of George
GHC
Cost of acquisition 105,000
Minus fair value of net assets acquired (100,000 – 5,000) (95,000)
––––––––
Goodwill 10,000
Recoverable value (7,000)
––––––––
Impairment write off 3,000
––––––––
(2) Amortisation of patent 20,000 ÷ 8 = GHC2,500
(3) Amortisation of brand 50,000 ÷ 5 × 9/12 = GHC7,500

Tutorial note
IAS38 Intangible assets prohibits the recognition of internally generated brands (3) or internally-
generated goodwill (4).
QUESTION 4
Product development costs are a material cost for many companies. They are either written off
as an expense or capitalised as an asset.
Required
(a) Discuss the conceptual issues involved and the definition of an asset that may be applied in
determining whether development expenditure should be treated as an expense or an asset.
(4 marks)

(b) Emerson has had a policy of writing off development expenditure to the income statement
as it was incurred. In preparing its financial statements for the year ended 30 September 2017 it
has become aware that, under IFRS rules, qualifying development expenditure should be
treated as an intangible asset. Below is the qualifying development expenditure for Emerson:
GHC'000
Year ended 30 September 2014 300
Year ended 30 September 2015 240
Year ended 30 September 2016 800
Year ended 30 September 2017 400

269
All capitalised development expenditure is deemed to have a four year life. Assume
amortisation commences at the beginning of the accounting period following capitalisation.
Emerson had no development expenditure before that for the year ended 30 September 2014.
Required
Treating the above as the correction of an error in applying an accounting policy, calculate the
amounts which should appear in the income statement and statement of financial position
(including comparative figures), and statement of changes in equity of Emerson in respect of the
development expenditure for the year ended 30 September 2007.
Note. Ignore taxation. (6 marks)
(Total = 10 marks)
Solution:
The IASB Framework defines an asset as a resource controlled by the entity as a result of past
events and from which future economic benefits are expected to flow to the entity. The
recognition criteria also require that the asset has a cost or value that can be measured reliably.
In the case of development expenditure it is not always possible to determine whether or not
economic benefits will result. IAS 38 deals with this issue by laying down the criteria for
recognition of an intangible asset arising from development expenditure. An entity must be able
to demonstrate that it is able to complete and use or sell the asset and has the intention to do so,
that the asset will generate probable future economic benefits and that the expenditure
attributable to the asset can be reliably measured. If these criteria are met, the asset is
recognised and will be amortised from the date when it is available for use.

GHC'000 GHC'000
Income statement
Amortisation of development expenditure (W) 335 135

Statement of financial position


Intangible asset: development expenditure (W) 1,195 1,130

Statement of changes in equity


Prior period adjustment
Added to retained earnings balance at 1/10/15 (W) 465

Working
Expenditure Amortisation Carrying amount
GHC'000 GHC'000 GHC'000
2014 300 300
2015 240 (75)* 165
Balance 2015 540 (75) 465
2016 800 (135)** 665
Balance 2016 1,340 (210) 1,130
2017 400 (335)*** 65
1,740 ( 545) 1,195

270
* 300 × 25%
** 540 × 25%
*** 1,340 × 25%

QUESTION 5
(a) During the last decade it has not been unusual for the premium paid to acquire control of a
business to be greater than the fair value of its tangible net assets. This increase in the relative
proportions of intangible assets has made the accounting practices for them all the more
important. During the same period many companies have spent a great deal of money
internally developing new intangible assets such as software and brands. IAS 38 'Intangible
assets' was issued in September 1998 and prescribes the accounting treatment for intangible
assets.
Required
In accordance with IAS 38, discuss whether intangible assets should be recognised, and if so
how they should be initially recorded and subsequently amortised in the following
circumstances:
(i) When they are purchased separately from other assets
(ii) When they are obtained as part of acquiring the whole of a business
(iii) When they are developed internally. (10 marks)
Note. Your answer should consider goodwill separately from other intangibles.

(b) Dauda is a public listed company. It has been considering the accounting treatment of its
intangible assets and has asked for your opinion on how the matters below should be treated in
its financial statements for the year to 31 March 2014.
(i) On 1 October 2013 Dauda acquired Tanko, a small company that specialises in
pharmaceutical drug research and development. The purchase consideration was by way of a
share exchange and valued at GHC35 million. The fair value of Tanko's net assets was GHC15
million (excluding any items referred to below). Tanko owns a patent for an established
successful drug that has a remaining life of 8 years. A firm of specialist advisors, Leadbrand, has
estimated the current value of this patent to be GHC10 million, however the company is
awaiting the outcome of clinical trials where the drug has been tested to treat a different illness.
If the trials are successful, the value of the drug is then estimated to be GHC15 million. Also
included in the company's statement of financial position is GHC2 million for medical research
that has been conducted on behalf of a client. (4 marks)
(ii) Dauda has developed and patented a new drug which has been approved for clinical use.
The costs
of developing the drug were GHC12 million. Based on early assessments of its sales success,
Leadbrand have estimated its market value at GHC20 million. (3 marks)
(iii) Dauda's manufacturing facilities have recently received a favourable inspection by
government
medical scientists. As a result of this the company has been granted an exclusive five-year
licence to manufacture and distribute a new vaccine. Although the licence had no direct cost to
Dauda, its directors feel its granting is a reflection of the company's standing and have asked
Leadbrand to value the licence. Accordingly they have placed a value of GHC10 million on it.
(3 marks)
(iv) In the current accounting period, Dauda has spent GHC3 million sending its staff on
specialist training courses. Whilst these courses have been expensive, they have led to a marked

271
improvement in production quality and staff now need less supervision. This in turn has led to
an increase in revenue and cost reductions. The directors of Dauda believe these benefits will
continue for at least three years and wish to treat the training costs as an asset.
(2 marks)
(v) In December 2013, Dauda paid GHC5 million for a television advertising campaign for its
products that will run for 6 months from 1 January 2014 to 30 June 2014. The directors believe
that increased sales as a result of the publicity will continue for two years from the start of the
advertisements.
(3 marks)
Required
Explain how the directors of Dauda should treat the above items in the financial statements for
the year to 31 March 2014. (15 marks as indicated)
Note. The values given by Leadbrand can be taken as being reliable measurements. You are not
required to consider depreciation aspects. (Total = 25 marks)

Solution:
(a) Recognition and amortisation
Goodwill
Only goodwill arising from a business combination is recognised. Under IFRS 3 goodwill is the
excess of the cost of a business combination over the acquirer's interest in the net fair value of
the assets, liabilities and contingent liabilities of the business acquired. Once recognised
goodwill is held indefinitely, without amortisation but subject to impairment reviews. One of
the key aspects of goodwill is that it cannot be separated from the business that it belongs to.
Therefore goodwill cannot be purchased separately from other assets. In addition, IAS 38 states
that internally generated goodwill must not be capitalised.

Other intangible assets


Other intangibles can be recognised if they can be distinguished from goodwill; typically this
means that they can be separated from the rest of the business, or that they arise from a legal or
contractual right.
Intangibles acquired as part of a business combination are recognised at fair value provided
that they can be valued separately from goodwill. The acquirer will recognise an intangible
even if the asset had not been recognised previously. If an intangible cannot be valued, then it
will be subsumed into goodwill.
Internally generated intangibles can be recognised if they are acquired as part of a business
combination. For example, a brand name acquired in a business combination is capitalised
whereas an internally generated brand isn't. Expenditure on research cannot be capitalised.
Development expenditure is capitalised if it meets the IAS 38 criteria. It is then amortised over
the life-cycle of the product.
Goodwill and intangibles with an indefinite useful life are not amortised but tested annually for
impairment.
(b) (i) The following assets will be recognised on acquisition:
GHCm
Fair value of sundry net assets 15
Patent at fair value 10
Research carried out for customer 2
Goodwill (balancing figure) 8

272
Total consideration 35
The patent is recognised at its fair value at the date of acquisition, even if it hadn't previously
been recognised by Tanko. It will be amortised over the remaining 8 years of its useful life with
an assumed nil residual value.
The higher value of GHC15m can't be used because it depends on the successful outcome of the
clinical trials. The extra GHC5m is a contingent asset, and contingent assets are not recognised
in a business combination. (Only assets, liabilities and contingent liabilities are recognised.)
Although research is not capitalised, this research has been carried out for a customer and
should be recognised as work-in-progress in current assets. It will be valued at the lower of cost
and net realisable value unless it meets the definition of a construction contract.
The goodwill is capitalised at cost. It is not amortised but it will be tested for impairment
annually.
(ii) New drug
Under IAS 38 the GHC12m costs of developing this new drug are capitalised and then
amortised over its commercial life. (The costs of researching a new drug are never capitalised.)
Although IAS 38 permits some intangibles to be held at valuation it specifically forbids
revaluing patents, therefore the GHC20m valuation is irrelevant.
(iii) Government licence
IAS 38 states that assets acquired as a result of a government grant may be capitalised at fair
value, along with a corresponding credit for the value of the grant. Therefore Dauda may
recognise an asset and grant of GHC10m which are then amortised/released over the five year
life of the license. The net effect on profits and on shareholders funds will be nil.
(iv) Training costs
Although well trained staff add value to a business IAS 38 prohibits the capitalisation of
training costs. This is because an entity has 'insufficient control over the expected future
economic benefits' arising from staff training; in other words trained staff are free to leave and
work for someone else. Training is part of the general cost of developing a business as a whole.
(v) Advertising costs
IAS 38 Para 69 states that advertising and promotional costs should be recognised as an expense
when incurred. This is because the expected future economic benefits are uncertain and they are
beyond the control of the entity.
However, because the year-end is half way through the campaign there is a GHC2.5m
prepayment to be recognised as a current asset.

QUESTION 6
Nyame Ltd incurred the following expenditure during the year:
GH¢'000 GH¢'000
Licence to operate in business sector for 10 years

from 1 January 2017
 400
Costs incurred in planning a website for a new product 40
(The website will be set up in 2018)

Purchase of 300 personal computers (PCs) on 1 July 2017



(Three-year useful life). Total cost:

300 PCs (excluding operating system) 480
Windows operating system for each unit (licence for 300 PCs) 60

273
Microsoft Office software for each unit (licence for 300 PCs) 48 588

Induction training for new staff 30

Nyame Ltd owns the rights to a popular range of books, which it purchased from another entity
for GH¢180,000 few years ago. The rights were not amortised as they have been attributed an
indefinite useful life. The books are still very popular so no impairment losses have been
necessary and it was valued by an independent valuer at GH¢280,000 at the year-ended 31
December 2017.
The company's policy is to use the revaluation model for its intangible assets where a market
valuation is available and permitted.
Required: Recommend with suitable calculations the carrying amount of intangible assets at the
end of the year 31 December 2017 according to the guidance given in IAS 38: Intangible assets.
(5 marks)
QUESTION 7
Oyarefa Ltd, acquired 80% ordinary shares in Abokobi Ltd on 1 January 2015. The intangible
assets of Abokobi Ltd include GH¢9 million of training and marketing expenditure incurred
during the year ended 31 December 2016. The Directors of Abokobi Ltd believe that these
should be capitalised as they relate to the startup period of a new business venture in Oyibi,
and they intend to amortise the balance over the five years commencing 1 January 2017.
On 1 July 2016, Oyarefa Ltd purchased a customer list from the liquidator of a competitor. The
price paid was GH¢4 million and was based on the list having a useful life of two years. At 31
December 2016, the Finance Director of Oyarefa Ltd commissioned a report on the value of the
customer list from a firm of independent valuers. The firm has valued the customer list at GH¢5
million and estimates a total useful life of five years. The customer list is currently included in
intangible assets at a carrying value of GH¢4 million but the Finance Director wants the list to
be revalued to the higher amount.
Required:
Recommend the treatment of the above in the consolidated financial statements for Oyarefa Ltd
Group for the year ended 31 December 2016 in accordance with IAS 38: Intangible Assets.
(6 marks)
SOLU
IAS 38 Intangible assets states that start-up, training and promotional costs should all be written
off as an expense as incurred as no intangible asset is created that can be recognised (the
benefits cannot be sufficiently distinguished from internally generated goodwill, which is not
recognised. Abokobi Ltd‘s retained earnings will be reduced by GH¢9 million (with result that
NCI share of profits are reduced) and the Intangible assets of Abokobi will also be reduced by
GH¢9 million. (2 marks)

The customer list would appear to satisfy the requirements of IAS 38, as it is identifiable, non-
monetary asset without physical substance. The list is identifiable as it was purchased from a
third party. The probability of future economic benefits is always assumed for such a separate
acquisition. Oyarefa Ltd has therefore adopted the correct accounting treatment by capitalising
the list at its cost of GH¢4 million. However, the list should be amortised over its original useful
life of two years. Therefore an amortisation charge of 4/2 x 6/12 = GH¢1.
Revaluation was done at the end of the year so no effect on the current year end.
A journal entry should be put through the books of Oyarefa Ltd

274
DR distribution and administration costs (SPLOCI) GH¢1
CR intangible assets (SFP) GH¢1
b) Intangible Assets

i) An intangible asset is an identifiable non-monetary asset without physical substance.
Intangible assets comprise expenditure on computer software, research and development,
advertising, brands, etc. It also includes rights under licensing agreements for items such as
motion picture films, video recordings, plays, manuscripts, patents and copyrights. (2
marks)
ii) Recognition criteria for intangible asset (8 marks)
1) The asset needs to be ‗identifiable‘
An asset is identifiable either if it is separable (can be
sold without disposing of the business as a whole) or if it arises from contractual or other legal
rights, irrespective of separability.
2) Control over the economic benefits derivable from the asset
Control involves the power to
obtain the future economic benefits flowing from the asset and to restrict the access of others to
those benefits. The capacity of an entity to control the future economic benefits would normally,
but not necessarily, stem from legal rights that are enforceable in a court of law.
3) Future economic benefits flowing to the entity
These benefits may include revenue from the
sale of products or services, but could also include cost savings or other benefits arising from
the use of the asset by the entity.
4) Cost can be measured reliably
‗Cost‘ will often be the cost of purchasing or developing the
asset. In the case of an asset acquired in a business combination, ‗cost‘ will be the fair value of
the asset at the date of acquisition, assuming this fair value can be reliably measured.

QUESTION 8
AT Group Ltd is engaged in a research and development project to produce a new product. In
the year to 30th June 2015, the company spent GH¢120,000 on research that concluded that there
were sufficient grounds to carry the project on to its development stage and a further
GH¢75,000 had been spent on development. At that date management having decided that they
were not sufficiently confident in the ultimate profitability of the project wrote off all the
expenditure to date to the income statement. In the current year further direct development
costs have been incurred of GH¢80,000 and the development work is now complete with only
an estimated GH¢10,000 of costs to be incurred in the future. Production is expected to
commence within the next few months. Unfortunately, the total trading profit from sales of the
new product is not expected to be as good as market research data originally forecast and is
estimated at only GH¢150,000. As the future benefits are greater than the remaining future
costs, the project will be completed, but due to the overall deficit expected, the directors have
again decided to write off all the development expenditure. (5 marks)
Required:
Advise, with numerical illustrations where possible, how the information above would affect
the preparation of AT Group Ltd.‘s consolidated financial statements to 30th June 2015.
SOLUTION
The treatment of the research and development costs in the year to 30th June 2015 was correct
due to the element of uncertainty at that date. The development costs of GH₵75,000 written off
in that same period should not be capitalized at a later date even if the uncertainties leading to
its original write off are favourably resolved. The treatment of the development costs in the year
to 30th June 2015 is incorrect. The directors‘ decision to continue the development is logical as at
the time of the decision the future costs are estimated at only GH₵10,000 and the future

275
revenues are expected to be GH₵150,000. It is also true that the project is now expected to lead
to an overall deficit of GH₵135,000 (120+75+80+10-150 in GH₵‘000). However at 30th June 2015,
the unexpensed development costs of GH₵80,000 are expected to be recovered. Provided the
criteria in IAS 38 Intangible Assets are met these costs of GH₵80,000 should be recognized as an
asset in the statement of financial position and matched to the future earnings of the new
product. Thus, the director‘s logic of writing off the GH₵80,000 development costs at 30th June
2015 because of an expected overall loss is flawed. The directors do not have the choice to write
off the development expenditure.

INVESTMENT PROPERTY (IAS 40)


Objective
This Standard deals with the accounting treatment of investment property and provides guidance
for the related disclosure requirements.
Scope
 The requirements of this Standard are applicable to deal with the accounting treatment
of Investment Property.
 This Standard also applies to:
 The books of lessee, for the accounting treatment of Property Interest held by lessee.
 The books of lessor, for the accounting treatment of Investment Property provided to
lessee under operating lease.
 The Standard does not cover the following aspects:
(a) For the classification of lease contracts,
(b) Treatment of Rental income related to investment property as covered in IAS 18.
Definitions
Investment Property
It is any:
 Land or Building, or
 Part of Land & Building (Owned or held under finance lease)
Held for the purpose of
 Rental earnings, or
 Capital appreciation, or both
Other than

276
 Property held for the purpose of use in production, supply of goods/services, or use in
administration i-e. Owner Occupied Property (IAS 16) and
 Property held for sale in normal course of business (IAS 2)

Examples of Properties which are treated as Investment Property:


(a) The property which is held for Capital Appreciation in long term.
(b) The Property which is held currently, for undetermined future use, i.e. (the owner has not
yet decided the future use of property)
(c) The property held to be leased out under one or more operating leases to lessee.
(d) The property under construction, which will be used as Investment Property in future.
Examples of Properties which are not treated as Investment Property:
(a) The property which is held for sale in the normal course of business (IAS 2 Inventory),
(b) The property being constructed on behalf of third parties (IAS 11 Construction Contracts).
(c) The property held for the purpose of use in production, supply of goods/services, or use in
administration i-e. Owner Occupied Property (IAS 16),
(d) The property which is occupied by employees, being provided by employer as part of
remuneration package (whether or not the employee pays rent)
(e) The property which is held to be leased out, under a finance lease.
WHY INVESTMENT PROPERTIES ARE TREATED DIFFERENTLY FROM OTHER
PROPERTIES.
Most properties are held to be used directly or indirectly in the entity‘s business. For example, a
factory houses plant and equipment which is used to produce goods for sale. The property is
being consumed and it appropriate to depreciate it over its useful life.
An investment property is held primarily because it is expected to increase in value.It generates
economic benefits for the entity because it will eventually be sold at a profit. An investment
property also differs from other properties because it generates revenue and cash flows largely
independently of other assets held by an entity.
The most relevant information about an investment property is its fair value (the amount for
which it could be sold). Depreciation is largely irrelevant. Therefore it is appropriate to re-
measure an investment property to fair value each year and to recognise gains and losses in
profit or loss for the period.
The IASB originally wanted to require entities to use the fair value model only.
However, at the time that IAS 40 was originally issued, this would have been a radical and
probably controversial step. Therefore the IASB decided to allow entities to continue to measure
investment properties at depreciated historical cost if they choose.
Owner Occupied Property:
The property (Owned or held under finance lease) for the purpose of:
 Use in the production,
 Supply of goods/services or
 Use in administration
Such type of property is covered under IAS 16
Property Interest:
The properly held by lessee under operating lease, and lessee has the right to:
 Sublet the property to other parties and
 Participate in Capital Appreciation in the value of such property.
The property with such rights is called Property Interest. This Standard provides an option for
such kind of Property interest, can be recognized as Investment Property in the financial

277
statements of lessee, under Fair Value model. This classification option is applicable upon each
Property Interest on property-by-property basis.
If this classification option is opted for one such Property Interest, all other Investment Properties
have to be accounted for under fair value model, and such classification option should be
included in the disclosures.
Dual-Purpose Properties
If a property is being under dual-use i.e. property contains a part of the property which is held for
rental earnings or capital appreciation and another part which is held for use in the production, supply of
goods/services, or for use in administration. Such property will be accounted for as:
 If both portions are separable i.e. (could be sold or leased out separately under finance
lease), then entity should account for each portion on individual basis under relevant
IAS
 If both portions are not separable i.e. (could not be sold or leased out separately under
finance lease), the property will be treated as Investment Property only, if an immaterial
part of such property is held for use in the production, supply of goods/services or for
use in administration.
Ancillary Services
Investment property should not include Ancillary Services (Meals, Cleaning, Security, Utilities, and
Maintenance services). If in case of a certain property, an entity provides ancillary services to the
occupants of a property, the entity shall apply the following:
 The property will be Investment Property, if quantum of the services is immaterial or
insignificant. For example security or maintenance services.
 The property will not be Investment Property, if quantum of the services is material or
significant. For example, owner-managed hotel. Therefore, such properties will be
covered in IAS 16

Investment Property in Consolidated Financial Statements


The property which is leased to, the Parent Co. by a Subsidiary Co. or vice versa, will not be
treated as Investment Property in the consolidated financial statements, instead it will be treated as
Owner-occupied Property under IAS 16, because the property is under owner-occupied use from
the Group perspective.
However, the property will remain Investment Property in the individual financial statements of
the entity who owns it.
Initial Recognition
A property will be recognized as Investment Property if it meets the following criteria:
 The definition of Investment Property
 If future economic benefits are probable to flow to the entity
 Its cost is reliably measurable.
Initial Measurement
The Investment Property is initially measured at Cost including Transaction Costs.
The cost of Investment Property includes:
 Purchase Price and
 Any directly related cost such as (professional or legal charges, property transfer taxes &
any other transaction costs).
Other Points related to Initial Measurement:
 Any other cost which relates to ongoing activities of Investment property will be
charged to Profit & Loss account as expense.

278
 If the Investment Property is purchased on extended credit period, the cost of the property
will be cash price equivalent and any excess over cash price will be treated as interest
expense and will be recognize over the period of credit.
 If lessee chooses to recognize the Property Interest as Investment Property as per
classification option available in IAS 40, then the initial cost of such a Property Interest
shall be prescribed, as for finance lease under IAS 17. Therefore, such a Property Interest
will be recognized at the lower off:
 The Present value of minimum lease payments and.
 Fair value of the Property Interest or
The entity will also recognize a liability with an equivalent amount.

Subsequent Recognition:
 Any expenditure upon Investment Property, during the life of Investment Property will
be recognize in the carrying amount of investment property, if such expense results in
increase in economic benefits of the investment property that would obtain otherwise.
 Any other expense to maintain the Investment Property will be treated as expense in the
statement of profit or loss.
Subsequent Measurement:
(1) The entity has two options to account for the Investment Property at reporting date;
 Cost Model
 Fair Value Model
(2) Whichever model is chosen, it should be applied for all the Investment Properties held by the
entity.
Cost Model:
The entity which chooses Cost model to account for its Investment Property after initial
recognition, will measure the investment Property as per Cost Model rules prescribed in IAS 16
i.e. Cost less Accumulated Depreciation less Accumulated impairment loss.
Fair Value Model:
The entity which chooses Fair Value model to account for its Investment Property after initial
recognition, will measure the investment Property at Fair Value.
 Under fair vale model, the investment property will be measured at fair value on
reporting date.
 Any change (increase or decrease) in the fair value of investment property at reporting
date, will be reported to the statement of profit or loss.
 Investment property under fair value model is not depreciated.
 Once the entity opts to use the fair value model, it should be used for all the investment
properties, except the Investment property for which fair value is not available under
specified circumstances.
 The entity which has opted to measure an investment property at fair value, it will
continue to measure the property at fair value, up to the date of disposal or until the
date of change in use of the property.
Fair Value Determination:
The fair value of the investment property is determined as per the requirements of IFRS 13;
however the entity should also consider the following points;
 The fair value should be determined as per the current condition of the investment
property, in the current market conditions.

279
 In determination of fair value of investment property, the entity should avoid the
double-counting, by not considering the different items separately, which are the part of
investment property, such items include lifts, air-conditioning, furniture & fixture and
integral parts of a building.
 If in exceptional circumstances, the fair value of a certain investment property is not
determinable and alternative reliable measurements (discounted cash flows) are also not
available, then entity should measure such investment property under cost model till the
date of disposal and residual value of such property will assumed to be zero.
 If the fair value of an investment property being constructed is not available,and entity
estimates that the fair value of such property will be determinable upon its completion,
then in such circumstances entity should account for the investment property being
constructed under cost model until
 Its fair value becomes available or
 Construction work is finished
(3) A property interest held by a lessee, which is classified as an investment property as per
classification option available in IAS 40, will be accounted for using the requirements of fair
value model.
(4 ) An entity has the option, that it may choose cost model or the fair value model for the
measurement of all the investment properties backing liabilities, whose return is directly linked to
the fair value of, or returns from, specified assets and pool of investment properties.

TRANSFERS
Transfers to or from investment property can only be made if there is a change of use.There are
several possible situations in which this might occur and the accounting treatment for each is
set out below;
1.TRANSFER FROM INVESTMENT PROPERTY TO OWNER –OCCUPIED PROPERTY:Use
the fair value at the date of the change for subsequent accounting under IAS16.
2.TRANSFER FROM INVESTMENT PROPERTY TO INVENTORY: Use the fair value at the
date of the change for subsequent accounting under IAS 2(Inventories).
3.TRANSFER FROM OWNER OCCUPIED PROPERTY TO INVESTMENT PROPERTY TO BE
CARRIED AT FAIR VALUE: Normal Accounting under IAS 16 (cost less depreciation) will
have been applied up to the date of the change.On adopting fair value,there is normally an
increase in value.This is recognized as other comprehensive income and credited to the
revaluation surplus in equity in accordance with IAS 16.If the fair valuation causes a decrease in
value,then it should be charged to profits.
4.TRANSFER FROM INVENTORIES TO INVESTMENT PROPERTY TO BE CARRIED AT
FAIR VALUE: Any change in the carrying amount caused by the transfer should be recognized
in profit and loss.
Expandable Notes:
The transfer of property will take place, if there is change in the use of property such
As:
(a) The development of investment property, to be sold in the normal course of business, will
result in transfer of property from IAS 40 to IAS 2.
(b) The use of investment property as owner-occupied property will result in transfer of
property from IAS 40 to IAS 16.

280
(c) Cease of use of owner-occupation in a property to be used as investment property, will result
in transfer of property from IAS 16 to IAS 40.
(d) If a property is let out under operating lease to other party, which was held for sale in
normal course of business, will result in transfer of property from IAS 2 to IAS 40.
In all such circumstances the entity will apply the following accounting treatment:
 If a property is transferred from inventory (IAS 2) to investment property (IAS 40), it
will be measured at fair value, any difference between the fair value of property and its
previous carrying value under IAS 2 will be reported in the statement of profit or loss on
the date of reclassification. Subsequently, the entity will apply fair value model under
IAS 40.
 If a property is transferred from owner-occupied (IAS 16) to investment property (IAS
40) which will be measured at fair value, the entity will apply IAS 16 rules up to the date
of reclassification. However, any difference between the fair value of property and its
carrying value under IAS 16 on the date of reclassification will be treated as Revaluation
Surplus/Loss,which will be accounted for as revaluation rules under IAS 16.
Subsequently, the entity will apply fair value model under IAS 40.
 If an investment property (IAS 40) is transferred to inventory (IAS 2) or owner-occupied
property (IAS 16), no gain/loss will arise on the date of reclassification and carrying
value under IAS 40 will become deemed cost for subsequent accounting.
 When the development of the investment property under construction is completed,
which will be measured under fair value model, any resulting difference between its fair
value and carrying value will be reported to the statement of profit or loss.

De-recognition of Investment Property


The investment property will be derecognized from the financial statements, under following
situations:
 Upon disposal of Investment property or
 When no economic benefits are available either by use of property or from its sale
 However, any gain or loss, resulting from the disposal of investment property will be
charged to statement of profit or loss in the related period.
 Any compensation recoverable from any third parties will be recognized in statement
profit or loss, in respect of investment property which was impaired or lost, in the
period in which it becomes receivable.
Disclosures
The entity should disclose the following:
(a) The measurement model used by the entity i.e. the cost or fair value model.
(b) The circumstance in which entity has opted the classification option for property interest.
(c) How entity has determined the fair value for investment property.
(d) Any amounts recognized in statement of profit or loss in respect of:
 Any rental earnings from investment property
 Any operating expense such as repair & maintenance
 Any movement in fair value of investment property.
(e) For the investment property under cost model, the entity should disclose:
 Depreciation Method
 Estimate of useful life
 Its gross carrying amount
 Any amount of impairment loss

281
(f) For the property for which fair value could not be determined and the entity has to measure
such property under cost model, the entity should disclose:
 Nature of the Investment Property
 Reason why the fair value is not determinable
(g) For the property which has been disposed off the entity should disclose:
 Its carrying amount on disposal date
 Any amount of gain or loss on disposal
Conclusion
The accounting rules contained in IAS 40 are very extensive and cover a wide scope of issues
relating to investment properties. Students should ensure that they study their respective
syllabuses carefully so as to gain a broad understanding of the issues that may crop up in an
exam situation. Professional accountants should ensure they are also familiar with the content
in IAS 40 to ensure correct accounting treatment. It is important that students fully understand
the initial recognition and subsequent measurement issues relating to investment properties
and how to account for such changes. Gaining a sound understanding of theoretical knowledge
is also important, particularly where there is scope for a discursive style questions to be
examined in a financial reporting examination.

REVIEW QUESTIONS
QUESTION 1
Sanchez Ltd is local government organization that owns a credible portfolio of properties which
includes;
 The properties which are surplus to the functional requirements and some other
properties which are held by the entity for different purposes.
 The portfolio includes several plots of land which are held for capital appreciation and
may be sold in the future.
 The entity has also some properties which have no current purpose, as the entity has not
yet determined whether it will use those properties to provide services such as those
provided by national parks or will put these on sales in the ordinary course of business.
 Sanchez Ltd also has a housing department to deal with regular sale and purchase of
some plots of land included within the portfolio of the properties, in the normal course
of business to supplement its income.
 Parts of the portfolio, some properties which are not held for sale, are provided as a
housing facility to low-income employees at nominal rentals, which are then used to
cover the cost of maintenance of the properties.
Required;
Explain how the properties included in the portfolio will be accounted for in the financial
statements of Sanchez Ltd.
Solution:
Investment Propertyis covered under IAS 40, as per IAS 40 Investment property is:
 Land or Building, or
 Part of Land & Building (Owned or held under finance lease)
Held for the purpose of
 Rental earnings, or
 Capital appreciation, or both
Other than

282
 Property held for the purpose of use in production, supply of goods/services, or use in
administration i-e. Owner Occupied Property (IAS 16) and
Property held for sale in normal course of business (IAS 2)

A property will be recognized as Investment Property if it meets the following criteria:


 The definition of Investment Property
 It is probable that future economic benefits will flow to the entity
 The cost is reliably measurable.
Therefore,
 the plots of land which are held by the entity for capital appreciation and the properties
that has no current purpose are both covered by IAS 40 as investment property as they
satisfy the definition criteria.
 And the plots of land which are held for sale in the normal course of business, by
housing department will be treated as inventory under IAS 2.
 Whereas, the properties which are not held for sale and are provided to low income
employees as a housing facility as part of the business will be accounted for as property
plan and equipment under IAS 16, as these are not primarily held for rental earnings,
which is reflected by the lower rentals.
QUESTION 2
(a) Define an Investment Property (In reference to IAS 40)
(b) An entity purchased an office building with a useful economic life of 50 years of GHC5.5
million on 1 January,2006.(The amount attributable to land was negligible).The entity used the
building as its head office for five years until 31 December,2010,when the entity moved its
business into a larger premises. The building was reclassified on that date as an investment
property and leased under a 40 year lease. The fair value of the head office at 31 December,2010
was GHC6 million.
Required:
Explain the treatment of the office building on the assumption that the entity uses the fair value
model for investment properties.
Solution:
i) Land or building, or part of a building, or both, held by the owner or the lessee under a
finance lease to earn rentals and/or for capital appreciation, rather than for use inproduction or
supply of goods and services or for administrative purposes or for sale in the ordinary course of
business.
ii) At 31 December 2010, the building has a carrying value of GHC4.95 million in accordance
with IAS 16. On 31 December 2010, the property should be recognized as an investment
property. The property should be fair valued at 31 December 2010 and any change in value
should be recognized in accordance with IAS 16. The property should therefore be recognized
at a carrying amount of GHS6 million and the difference of GHC1.05 million [GHC6 million –
GHC4.95 million] should be recognized in other comprehensive income as a revaluation
surplus. In subsequent period (unless there is further change in use) the building should be
measured at fair value with any gain or loss recognized directly in profit or loss.
QUESTION 3
Diego Ltd,purchased a property for GHC 100,000 on 1st January,2010 for its investment
potential. The Land element of the cost is believed to be GHC40, 000, and the building element
is expected to have a useful life of 50 years. At 31 December, 2010,local property indices suggest

283
that the fair value of the property has risen to GHC 110,000.Show how the property would be
presented in the financial statements as at 31 December,2010 if Diego Costa adopts:
(a) The cost model
(b) The fair value model
Solution:
(a)Cost Model
GHC
Investment Property at Cost 100,000
Less Accumulated depn (60,000/50) 1,200
Net Book Value 98,800
(b)Fair Value Model
GHC
Cost 100,000
Fair Value at End 110,000
Income Statement 10,000

QUESTION 4
On 1 January,2011 Paulino Ltd purchased a building for its investment potential. The building
cost GHC1 million with transaction costs of GHC10,000. Its depreciable amount at this date was
GHC300,000. The property has a useful life of 50 years. At the end of 2011 the property‘s fair
value had risen to GHC1.3 million.
Required
Show the amounts which would be included in the financial statements of Paulino Ltd at 31
December,2011, under:
(a) the cost model
(b) the fair value model

Solution:
(a) Cost model
In the statement of financial position the property will be included as follows.
GHC
Cost (1,000,000 + 10,000) 1,010,000
Accumulated depreciation (300,000 ÷ 50 years) (6,000)
––––––––––
Carrying amount 1,004,000
––––––––––
The income statement (statement of comprehensive income) will include depreciation of
GHC6,000.

(b) Fair value model


The property will be included in the statement of financial position at its fair value of
GHC1,300,000.
The statement of comprehensive income (income statement) will include a profit of GHC290,000
(= GHC1,300,000 – GHC1,010,000) in respect of the fair value adjustment. (Remember: this
treatment of revaluation of an investment property differs from the accounting treatment of a
revaluation gain for other non-current assets under IAS 16.)

284
QUESTION 5
Victoria owns several properties and has a year end of 31 December. Wherever possible,
Victoria carries investment properties under the fair value model.
Property 1 was acquired on 1 January,2011. It had a cost of GHC1 million, comprising
GHC500,000 for land and GHC500,000 for buildings. The buildings have a useful life of 40
years. Victoria uses this property as its head office.
Property 2 was acquired many years ago for GHC1.5 million for its investment potential. On 31
December,2017 it had a fair value of GHC2.3 million. By 31 December,2018 its fair value had
risen to GHC2.7 million. This property has a useful life of 40 years.
Property 3 was acquired on 30 June,2012 for GHC2 million for its investment potential. The
directors believe that the fair value of this property was GHC3 million on 31 December,2017
and GHC3.5 million on 31 December,2018. However, due to the specialised nature of this
property, these figures cannot be corroborated. This property has a useful life of 50 years.
Required
(a) For each of the above properties briefly state how it would be treated in the financial
statements of Victoria for the year ended 31 December,2018,identifying any impact on profit or
loss.
(b) Produce an analysis of property, plant and equipment for Victoria for the year ended 31
December 2018, showing each of the above properties separately.

Solution:
(a) Treatment in the financial statements for the year ended 31 December Year 8 (IAS16)
Property 1
This is used by Victoria as its head office and therefore cannot be treated as an investment
property. It will be stated at cost minus accumulated deprecation in the statement of financial
position. The depreciation for the year will be charged in the income statement.
Property 2
This is held for its investment potential and should be treated as an investment property. It will
be carried at fair value, Victoria‘s policy of choice for investment properties. It will be revalued
to fair value at each year end and any resultant gain or loss taken to the income statement
(GHC400,000 in 2018).
Property 3
This is held for its investment potential and should be treated as an investment property.
However, since its fair value cannot be arrived at reliably it will be held at cost minus
accumulated depreciation in the statement of financial position. The depreciation for the year
will be an expense in the income statement.
This situation provides the exception to the rule whereby all investment properties must be
held under either the fair value model, or the cost model.
(b) Analysis of property, plant and equipment for the year ended 31 December,2018
Other land (W1) Investment property Investment property Total
and buildings held at fair value held at cost (W2)
GHC GHC GHC GHC
Cost/valuation
On 1 January,2018 1,000,000 2,300,000 2,000,000 5,300,000
Revaluation - 400,000 - 400,000

285
––––––––– ––––––––– ––––––––– –––––––––
On 31 December,2018 1,000,000 2,700,000 2,000,000 5,700,000
––––––––– ––––––––– ––––––––– –––––––––

Accumulated depreciation
On 1 January,2018 87,500 - 220,000 307,500
Charge for the year (W1) 12,500 - 40,000 52,500
––––––––– ––––––––– ––––––––– –––––––––
On 31 December,2018 100,000 - 260,000 360,000
––––––––– ––––––––– ––––––––– –––––––––

Other land (W1) Investment property Investment property Total


and building held at fair value property held at cost (W2)
GHC GHC GHC GHC
Carrying amount
On 31 December,2017 912,500 2,300,000 1,780,000 4,992,500
––––––––– ––––––––– ––––––––– –––––––––
On 31 December,2018 900,000 2,700,000 1,740,000 5,340,000
––––––––– ––––––––– ––––––––– –––––––––
Tutorial note
In practice, with a more complex property, plant and equipment table the investment
properties would be included within the land and buildings column with the required
disclosures being given separately in a note to the table.
Workings
(1) Depreciation on Property 1 GHC
Brought forward (500,000 ÷ 40 × 7) 87,500
2018 (500,000 ÷ 40) 12,500
(2) Depreciation on Property 3 GHC
Brought forward (2,000,000 ÷ 50 × 5.5) 220,000
2018 (2,000,000 ÷ 50) 40,000
QUESTION 6
Messi owns a number of properties. An independent surveyor has assessed their market values
as:
Property Cost Valuation Valuation
1 July,2009 30 June,2010 30 June,2011
GHC GHC GHC
A 41,000 52,000 73,000
B 76,000 82,000 66,000
C 80,000 70,000 90,000
197,000 204,000 229,000
All the properties had an estimated life of 50 years when they were acquired. They are all let on
short leases on commercial terms,however property C is let to a fellow subsidiary of Messi
Ltd.The group policy (applied to all members of the group) is to adopt the fair value model in
IAS 40 for investment properties and to treat owner-occupied properties under the benchmark
treatment in IAS 16.
Required:

286
Prepare extracts of the group financial statements of Messi Ltd in respect of the above
properties for the years to 30 June,2010 and 2011.
Solution:
Income Statement
30th June 30th June
2010 2011
GHC GHC
Property C-Owner Occupied(80,000/50) (1,600) (1,600)
Investment Property – Surpluses : A 11,000 21,000
B 6,000
Deficit B (16,000)

Statement of Financial Position


30th June 30th June
2010 2011
PPE(80,000-1,600)/(80,000 – 3,200) 78,400 76,800
Investment Proerty (52,000+82,000) 134,000 139,000

QUESTION 7
The accounting treatment of investment properties is prescribed by IAS 40: Investment
property.
Required:
(i) Define investment property under IAS 40 and explain why its accounting treatment is
different from that of owner-occupied property.

(ii) Explain how the treatment of an investment property carried under the fair value model
differs from an owner-occupied property carried under the revaluation model.
(5 marks)

Solution:
(i)An Investment property Property held to earn rentals or for capital appreciation or both,
rather than for:
use in the production or supply of goods or services or for administration purposes; or
sale in the ordinary course of business

(ii)The accounting treatment for IP is different from that of owner occupied because Investment
properties are acquired to earn rentals or for capital appreciation or both and thus their cash
flows are largely independent of those from other assets held by the enterprise.

287
After initial recognition, if fair valuation model is adopted, an entity must measure all IPs at
that value with any gains/losses being included in net profit/loss for the period in which it
arises.
This differs from an owner occupied property carried under revaluation model because in the
case of the latter , the resultant gain/surplus is dealt with in OCI and held in equity.

QUESTION 8
Muller Ltd owns two properties at 1 January 2013:
Property X:
A headquarters building is held by the entity for administrative use. The property has a
carrying value of GHC4 million at 1 January 2013 with a remaining life of 20 years. At 1 July
2013, the entity under goes a reorganization and as a result, the property was let out to a third
party and reclassified as an investment property under fair value model as per IAS 40. An
independent expert valued the property at a fair value of GHC4.6 million at 1 July 2013, and
this has risen up to GHC4·68 million at 31 December 2013.
Property Y:
This is an office building, which is sub-let to a subsidiary of Muller Ltd. The property had a fair
value of GHC3 million at 1 January 2013, which was raised to GHC3·30 million at 31December
2013.
Required:
(i) Prepare extracts of financial statements of Muller Ltd for the year ended 31/12/2013.
(ii) The treatment of Property ‗Y‘, in the consolidated financial statements of Muller Ltd.
Solution:
(a) Extracts of Muller Ltd. Financial Statements:
Statement of Profit or loss:

For the year ended 31/12/2013


GHC‘000
Depreciation of Headquarter Building (GHC4,000/20years * 6/12) (100)
Fair Value Gain on Property X (GHC4,680-GHC4,600) 80
Property Y (GHC3,300-GHC3,000) 300

Other Comprehensive Income:


Revaluation Surplus on headquarter Building [GHC4,600-(GHC4,000-100)] 700
Statement of Financial Position:
As on 31/12/2013
Assets:
Non-Current Assets:
Investment Property(X & Y) (GHC4,680+GHC3,300) 7,980
Equity:
Revaluation Surplus 700

(b) Property ‘Y’ in Consolidated Financial Statements:


The property will be treated as owner occupied property under IAS 16 in consolidated financial
statements, from the group perspective.

288
QUESTION 9
You are the Financial Accountant of Adom Ltd. The assistant accountant responsible for
preparing the 2012 annual financial statements is considering the accounting treatment of the
following non-current assets and has approached you for guidance.

Adom Ltd acquired a property on 1 January 2012 at a cost of GHC400,000 and immediately
occupied it as office premise. On acquisition, it was estimated to have a useful life of 50 years.
Subsequent to its acquisition, the asset was measured at depreciated cost until 1 July 2012 when
management of Adom Ltd decided to convert the building into an investment property (mainly
for rentals). Following this decision, the property was fair valued at GHC373,800. Adom Ltd
adopted the fair value model for subsequent measurement of the investment property. At 31
December 2012, it was fair valued at GHC380,000.
Required:
Account for the treatment of this property in the 2012 financial statements of Adom Ltd.
(5 marks)
ICAG MAY,2013

Solution:
Income Statement for the year ended 31 December 2012:
Expenses GHC
Depreciation charge (400,000/50 years x 6/12) 4,000
Other income
Fair valuation surplus-Investment Property (380,000-373,800) 6,200

Statement of Financial Position as at 31 December 2012


Non-Current Assets GHC
Investment Property 380,000
Equity
Revaluation Surplus (373,800 – 356,000 17,800

QUESTION 10
Alavanyo Ltd owns the following properties at 1 January, 2013:

Property XYZ:
An office building used by Alavanyo Ltd for administrative purposes with a depreciated
historical cost of GHC4million. At 1 April 2012 it had a remaining life of 20 years. After a
reorganization on 1 July 2013, the property was let to a third party and reclassified as an
investment property to have a fair value of GHC4.6million at 1 July 2013, which had risen to
GHC4.68 million at 31 December 2013.

Property QRS:
Another office building sub-let to a subsidiary of Alavanyo Ltd. At 1 January 2013, it had a fair
value of GHC3million which had risen to GHC3.30million at 31 December 2013.
Required:

289
Prepare extracts from Alavanyo Ltd‘s entity statement of profit or loss and other comprehensive
income and statement of financial position for the year ended 31 December 2013 in respect of
the above properties. In the case of property QRS only, state how it would be classified in
Alavanyo Ltd‘s consolidated statement of financial position.
(5 marks)
ICAG NOV,2014

Solution:
b) Property XYZ
Statement of Profit or Loss and other comprehensive income for 2013 [Extracts]
Expenses: GHS
Depreciation of PPE 100,000
Other Income Fair valuation gain on IP 80,000
OCI Revaluation surplus 850,000

SOFP as at 31 December 2013[Extract]


Non-current asset Investment property 4,680,000
Equity Revaluation surplus 850,000

Explanation
At 1 January 2013, the carrying value was [GHS4,000,000 – GHS150,000] = GHS3,850,000
Depreciation from 1 January 2013 – 1 July 2013 = 20% of GHS4m x6/12 =GHS100,000
Carrying value as at 1 July 2013 [GHS3,850,000 –GHS100,000 = GHS3,750,000
Reclassified as Investment property at valuation of GH4,600,000 , creating a revaluation surplus
of GHS850,000. This is dealt with in equity in line with IAS 16.
At 31 December 2013 the fair value is GHS4, 680,000. The fair valuation gain of GHS80, 000
[GHS4,680,000 – GHS4,600,000] is dealt with in profit or loss in line with IAS 40 and the IP is
measured in the SOFP at GHS 4,680,000
Property QRS
Since it is occupied by a subsidiary [a related party] it is best described as owner occupied and
dealt with under IAS 16 - PPE. It will be recognised at GHS3,000,000 on 1 January 2013 and
depreciated as such. At 31 December 2013 it would be measured at GHS3,300,000 in the SOFP
and the difference between GHS3,300,000 and the depreciated historic cost [GHS3,000,000 –
Depreciation for 2013] will be dealt with as OCI and disclosed within equity .
In the consolidated statement of financial position, it will be classified under PPE .

QUESTION 11
Kumeri Ltd (Kumeri) is a real estate company which reports under International Financial
Reporting Standards (IFRSs). The Office Building of Kumeri had a net carrying amount of
GH¢18 million at the beginning of the financial year 1 January 2019. The property was held
under the cost model. As its residual value was estimated at more than its cost due to a buoyant
property market, no depreciation had been charged.

As part of a relocation of the company's business, the property became vacant and was leased
out to a third party on 1 April 2019 (under a six-month short lease). At the time the property
was leased out, its fair value was GH¢22 million.

290
At the end of the lease, the company decided to transfer the property to its inventories of
properties for sale in the ordinary course of its business. At that date the value of the property
was GH¢21 million. The property was sold in December 2019 for GH¢21.3 million.

The company uses the fair value model for its investment property.

Required: Determine the amounts to be recognised in profit or loss and in other comprehensive
income in respect of the property for the year ended 31 December 2019. (6 marks)

SOLUTION
P/L OCI
GH¢‘m GH¢‘m
Transfer from PPE to investment property (22 – 18) 4
Final fair value after rev‘n becomes cost of inventories (21-22) (1) -
Profit on sale of inventories (21.3 – 21) 0.3 -
(0.7) 4

Recognition of loss on transfer from Investment Property to inventory – 2 marks


Recognition of loss in transfer from Investment Property to Inventory - 2 marks

TRY
Kantanka Ltd adopts the revaluation model of IAS 16 Property, Plant & Equipment and the fair
value model of IAS 40 Investment Property.
Kantanka Ltd owns a piece of property it purchased on 1 April 2014 for GH¢3.7 million. The
land component of the property was estimated to be GH¢1.2 million at the date of purchase.
The useful economic life of the building on this land was estimated to be 25 years on 1 April
2014. The property was used as the corporate head office for two years from that date. On 1
April 2016, the company moved its head office to another building and leased the entire
property for five years to an unrelated tenant on an arm‘s length basis in order to benefit from
the rental income and future capital appreciation. The fair value of the property on 1 April 2016
was GH¢4.1 million (land component GH¢1.9 million), and on 31 March 2017, GH¢4.8 million
(land component GH¢2.1 million). The estimated useful economic life remained unchanged
throughout the period. Land and buildings are considered to be two separate assets by the
directors of Kantanka Ltd.
Required:
Advise Kantanka Ltd on how to account for the above transactions in accordance with relevant
accounting standards

291
AGRICULTURE (IAS 41)
OBJECTIVE
The objective of IAS 41 is to establish standards of accounting for agricultural activity

SCOPE
Within scope are Biological assets, Agricultural produce at the point of harvest and Government
grants related to biological assets.

Excluded from scope are Land and Intangible assets related to agricultural activity
The table below provides examples of biological assets, agricultural produce, and products that
are the result of processing after harvest:
Biological assets Agricultural produce Products that are the
result of processing
after harvest
Sheep Wool Yarn, carpet
Trees in a plantationLogs Lumber
forest
Plants Cotton Thread, clothing
Harvested cane Sugar
Dairy cattle Milk Cheese
Pigs Carcass Sausages, cured hams
Bushes Leaf Tea, cured tobacco
Vines Grapes Wine
Fruit trees Picked fruit Processed fruit

DEFINITIONS
ACTIVE MARKET: Exists when; the items traded are homogenous, willing buyers and sellers
can normally be found at any time and prices are available to the public.

AGRICULTURAL ACTIVITY: The management of the transformation of a biological asset for


sale into agricultural produce or another biological asset.
Agricultural activity covers a diverse range of activities; for example, raising livestock, forestry,
annual or perennial cropping, cultivating orchards and plantations, floriculture, and
aquaculture (including fish farming). Certain common features exist within this diversity:
(a) Capability to change. Living animals and plants are capable of biological transformation;
(b) Management of change. Management facilitates biological transformation by enhancing,
or at least stabilising, conditions necessary for the process to take place (for example, nutrient
levels, moisture, temperature, fertility, and light). Such management distinguishes agricultural
activity from other activities. For example, harvesting from unmanaged sources (such as ocean
fishing and deforestation) is not agricultural activity; and
(c) Measurement of change. The change in quality (for example, genetic merit, density,
ripeness, fat cover, protein content, and fibre strength) or quantity (for example, progeny,
weight, cubic metres, fibre length or diameter, and number of buds) brought about by biological
transformation is measured and monitored as a routine management function.

292
Biological asset: A living animal or plant.
A group of biological assets is an aggregation of similar living animals or plants.
Agricultural produce: The harvested produce of the entity‗s biological assets.

Carrying amount is the amount at which an asset is recognised in the balance sheet.
Fair value is the amount for which an asset could be exchanged, or a liability settled, between
knowledgeable, willing parties in an arm‘s length transaction. The fair value of an asset is based
on its present location and condition. As a result, for example, the fair value of cattle at a farm is
the price for the cattle in the relevant market less the transport and other costs of getting the
cattle to that market. The determination of fair value for a biological asset or agricultural
produce may be facilitated by grouping biological assets or agricultural produce according to
significant attributes; for example, by age or quality. An entity selects the attributes
corresponding to the attributes used in the market as a basis for pricing.
Biological transformation: The process of growth, degeneration, production, and procreation
that cause an increase in the value or quantity of the biological asset.
Biological transformation results in the following types of outcomes:
(a) asset changes through
(i) growth (an increase in quantity or improvement in quality of an animal or plant),
(ii) degeneration (a decrease in the quantity or deterioration in quality of an animal or plant), or
(iii) procreation (creation of additional living animals or plants); or
(b) production of agricultural produce such as latex, tea leaf, wool, and milk.
Harvest: The process of detaching produce from a biological asset or cessation of its life.

RECOGNITION
Biological assets or agricultural produce are recognised when:
Entity controls the asset as a result of a past event
Probable that future economic benefit will flow to the entity; and
Fair value or cost of the asset can be measurement reliably.

MEASUREMENT
Biological assets within the scope of IAS 41 are measured on initial recognition and at
subsequent reporting dates at fair value less estimated costs to sell, unless fair value cannot be
reliably measured.
If no reliable measurement of fair value, biological assets are stated at cost less accumulated
depreciation and accumulated impairment losses.
Agricultural produce is measured at fair value less estimated costs to sell at the point of
harvest.
The gain on initial recognition of biological assets at fair value less costs to sell, and changes in
fair value less costs to sell of biological assets during a period, are included in profit or loss.
A gain on initial recognition (e.g. as a result of harvesting) of agricultural produce at fair value
less costs to sell are included in profit or loss for the period in which it arises.
All costs related to biological assets that are measured at fair value are recognised as expenses
when incurred, other than costs to purchase biological assets.
An unconditional government grant related to a biological asset is measured at fair value less
estimated point-of-sale costs is recognised as income when, and only when, the government
grant becomes available

293
A conditional government grant, including where a government grant requires an entity not to
engage in specified agricultural activity, is recognised as income when and only when, the
conditions of the grant are met.
Gains and losses
A gain or loss arising on initial recognition of a biological asset at fair value less estimated
point-of-sale costs and from a change in fair value less estimated point-of-sale costs of a
biological asset shall be included in profit or loss for the period in which it arises.
A loss may arise on initial recognition of a biological asset, because estimated point-of-sale
costs are deducted in determining fair value less estimated point-of-sale costs of a biological
asset. A gain may arise on initial recognition of a biological asset, such as when a calf is born.
A gain or loss arising on initial recognition of agricultural produce at fair value less estimated
point-of-sale costs shall be included in profit or loss for the period in which it arises.
A gain or loss may arise on initial recognition of agricultural produce as a result of harvesting
Point-of-sale costs include commissions to brokers and dealers, levies by regulatory agencies
and commodity exchanges, and transfer taxes and duties. Point-of-sale costs exclude transport
and other costs necessary to get assets to a market.

Government grants
An unconditional government grant related to a biological asset measured at its fair value less
estimated point-of-sale costs shall be recognised as income when, and only when, the
government grant becomes receivable.
If a government grant related to a biological asset measured at its fair value less estimated
point-of-sale costs is conditional, including where a government grant requires an entity not to
engage in specified agricultural activity, an entity shall recognise the government grant as
income when, and only when, the conditions attaching to the government grant are met.
Terms and conditions of government grants vary. For example, a government grant may
require an entity to farm in a particular location for five years and require the entity to return all
of the government grant if it farms for less than five years. In this case, the government grant is
not recognised as income until the five years have passed. However, if the government grant
allows part of the government grant to be retained based on the passage of time, the entity
recognises the government grant as income on a time proportion basis.
If a government grant relates to a biological asset measured at its cost less any accumulated
depreciation and any accumulated impairment losses (see paragraph 30), IAS 20 Accounting for
Government Grants and Disclosure of Government Assistance is applied.
This Standard requires a different treatment from IAS 20, if a government grant relates to a
biological asset measured at its fair value less estimated point-of-sale costs or a government
grant requires an entity not to engage in specified agricultural activity. IAS 20 is applied only to
a government grant related to a biological asset measured at its cost less any accumulated
depreciation and any accumulated impairment losses.

294
NON-CURRENT ASSETS HELD FOR SALE AND
DISCONTINUED OPERATIONS (IFRS 5)
OBJECTIVE
The objective of this standard is to specify the accounting for Non-current assets held for sale,
and presentation and disclosure of discontinued operations.
Non-Current Assets Held For Sale
HELD FOR SALE
This term refers to a non-current asset whose carrying amount will be recovered principally
through a Sale transaction rather than through continuing use.
DISCONTINUED OPERATION
A discontinued operation includes the following criteria:
 is a separately identifiable components
 must represent a major line of the entity‗s business
 is part of a plan to dispose of a major line of business or a geographical area
 is a subsidiary acquired with a view to resell
DISPOSAL GROUP
This is a group of assets and possibly some liabilities that an entity intends to dispose of in a
single transaction. Non-current assets or disposal groups that meet the criteria to be classified as
held for sale are measured at the lower of:
 their carrying amount and
 fair value less costs to sell,
ACCOUNTING TREATMENT:
 Non-current assets that meet the criteria are presented separately on the Statement of
Financial Position within current assets.
 If the held for sale item is a disposal group then related liabilities are also reported
separately within current liabilities.
 Discontinued operations and operations held for sale must be disclosed separately in the
statement of financial position at the lower of their carrying value less costs to sell.

CLASSIFICATION OF NON-CURRENT ASSETS AS HELD FOR SALE


For an asset to be classified as held for sale:
a) It must be available for immediate sale in its present condition allowing for terms that are
usual or customary;
b) Its sale must be highly probable {expected within 1 year of reclassification);
c) It must be genuinely sold, not abandoned.
For a sale to be highly probable it must be significantly more likely than probable. In addition
the standard sets out the following criteria to be satisfied:
 Management, at a level that has the authority to sell the assets or disposal group, is
committed to a plan to sell;
 An active program to locate a buyer and complete the sale must have begun.
 The asset or disposal group must be actively marketed at a price that is reasonable
compared to its current fair value.
 The sale of the asset is expected to be recorded as completed within time, ear from the
date of classification.

295
 The actions required to complete the plan should indicate that it is not likely that there
will be significant changes made to the plan or that the plan will be withdrawn.
Example 1
An entity has agreed in a directors‘ meeting to sell a building, and has tentatively started
looking for a buyer for the building. The price of the building has been fixed at GHC4m and a
surveyor has valued the building based on market prices at GHC3.6m. The entity will continue
to use the building until another building has been found with equivalent facilities, and in a
suitable location for the office staff, who will not be relocated until the new building has been
found.
Additionally, the entity is planning to sell part of its business and has actively marketed the
business at a fair price but, before the business can be sold, government approval is required
and any sale requires government approval. This means that the sale time is difficult to
determine and it may take longer than one year to sell the disposal group.
Answer
The building will not be classified as held-for-sale as it is not available for immediate sale
because, until new premises have been found, the office staff will remain in the existing
building. Also, the directors have only tentatively started looking for a buyer which may
indicate that the entity is not committed to the sale. Additionally, the price being asked for the
building is above the market price, and is not reasonable compared to that price. It is unlikely
that the entity will sell the building for that price.
The disposal group, however, would be classified as held-for-sale because the delay is caused
by events or circumstances beyond the entity‘s control, and there is evidence that the entity is
committed to selling the disposal group.
Example 2
An entity has stopped using certain plants because of a downturn in orders. It is maintaining
the plant as the entity hopes that orders will pick up in future. Additionally, it intends to shut
down one-half of its manufacturing base. The units to be closed constitute a major segment of
its business and will close in the current financial year.
Answer
The equipment will not be treated as abandoned as it will subsequently be brought back into
usage, and the manufacturing units will be treated as discontinued operations.
MEASUREMENT
They are measured at the lower of:
 Fair value less costs to sell; and
 Carrying amount (in accordance with the relevant Standard).
Any impairment loss on initial or subsequent write-down of the asset or disposal group to
fair value less cost to sell is to be recognised in the statement of profit or loss.
Dr. Asset Held for Sale xxx
Dr. Impairment Loss xxx
Cr. Non Current Asset xxx
Any subsequent increase in fair value less cost to sell can be recognised in the statement of
profit or loss to the extent that it is not in excess of the cumulative impairment loss that has been
recognised in accordance with the IFRS 5 or previously in accordance with IAS 36.
SUBSEQUENT REMEASUREMENT
 Whilst a non-current asset/disposal group is classified as held for sale it should not be
depreciated or amortised.

296
 At each reporting date where a non-current asset or disposal group continues to be
classified as held for sale it should be re-measured at fair value less costs to sell at that
date.
 This may give rise to further impairments or a reversal of previous impairment losses. In
either case recognise in the statement of profit or loss.
 If subsequent criteria not met then following entry is passed
Dr. NCA xxx Dr./Cr. Loss/gain xxx
Cr. Asset held for sale xxx
The double entry for discontinued operation is:
Dr. Asset held for sale xxx
Dr. Non-current liabilities xxx
Dr./Cr. Loss/Profit xxx
Cr. Liability held for sale xxx
Cr. Non-current asset xxx
PRESENTATION
Current Assets: Disposal Group held for sale
Current Liabilities: Disposal Group held for sale
Equity: NCI held for sale
In statement of profit or loss:
• Profit from continued operations
• Profit/(loss) from discontinued operations

REVIEW QUESTIONS
QUESTION 1
On 1st January,2009 Messi and Ronaldo Ltd purchased a machine for GHC20,000.It has an
expected useful life of 10 years and a nil residual value.The company uses the straight line
method of depreciation.
On 31st December,2010 the company decides to sell the machine,Its current market value is
GHC15,000 and the company is confident they will find a buyer very quickly due to the short
supply in the market for this type of machinery.It will cost the company GHC500 to dismantle
the machine .
Required:
At what value should the machine be in Messi and Ronaldo Ltd‘s financial statement?
Solution:
GHC
Carrying Value(8/10 x 20,000) 16,000
Fair Value less Cost to Sell(15,000-500) 14,500
Impairment Loss 1,500
The Asset will be classified as held for sale at GHC14,500(lower)in the Statement of Financial
position as part of Current assets and an impairment loss of GHC1,500 charged to the Income
Statement.
QUESTION 2
Pinch plc owns a building which it has used for many years as a factory. On 1 January 2012 the
building had a carrying value of GHC15m with an estimated useful economic life of 15 years.
Pinch uses the cost model under IAS 16 to account for buildings. On 1 April 2012 Pinch plc
commenced operations in a new building, and the old one was placed on the market as it was

297
no longer being used. The estimated proceeds of sale were GHC13 million, less selling costs of
GHC0.2 million. It was seen as highly probable at that date that the building would sell at that
price. By year end, 31 December 2012, the building remained unsold, so Pinch plc reduced the
asking price to GHC11m. The estimate of selling costs remained the same. The directors of
Pinch plc believed at that date it was highly probable the sale would occur within 12 months at
the lower price.

Required:
Explain how the old building should be treated in the books of Pinch plc for year ended 31
December 2012.
Solution
The building qualifies for transfer to ―held for sale‖ on 1 April 2012 as the two conditions were
met at that date:
1. It was available for immediate sale in its present condition at the date classification to ―held
for sale‖ is made; and
2. The sale was considered highly probable.
The carrying value on 1 April 2012 was GHC15 million less 3 month‘s depreciation (15m * 1/15
* 3/12) of GHC0.25 million. Therefore the carrying value was GHC14.75 million. Always
assume depreciation is calculated on a time-apportioned basis unless otherwise instructed.
The ―fair value less costs to sell‖ on 1 April 2012 was GHC12.8 million (13m – 0.2m). Therefore
the initial value to be assigned to the non-current asset held for sale is GHC12.8 million (lower
of
(1) carrying value at date of transfer and
(2) ―fair value less costs to sell‖). The loss in value of GHC1.95 million (14.75m – 12.8m) is taken
to profit or loss for the year.
No depreciation is charged from 1 April 2012.
At 31 December 2012, the next reporting date, the asset has not been sold. The applicability of
the conditions is reviewed, and the fair value less costs to sell is also reviewed. Based on the
failure to sell the asset, the price was reduced. The conditions are still met in that:
1. It is still available for immediate sale in its present condition; and
2. The sale is still considered highly probable.
Therefore the classification continues to be ―held for sale‖, but the asset‘s carrying amount is
reduced to the revised ―fair value less costs to sell‖ of GHC10.8 million (11m – 0.2m). The
further reduction in value of GHC2 million (12.8m – 10.8m) is taken to profit or loss for year
ended 31 December 2012.
QUESTION 3
Sofoline Ltd has a plant which cost GH¢40,000 and was purchased on 1 January 2013 with a
useful life of 10 years. The plant was being used as part of its business operating capacity. On 30
June 2015, Sofoline Ltd made a decision to classify the plant as held for sale and an agent was
appointed for the sale of the plant that have started advertising the plant at a selling price of
GH¢29,000 which was considered to be its fair value. The selling expenses are estimated to be
GH¢1,500. The asset has not yet been sold by the year end of 31 December 2015 and it has a fair
value less cost to sell of GH¢24,000 on this date.
Required:
Discuss how this will be accounted for in the financial statements of Sofoline Ltd for the year
ended 31 December, 2015 in accordance with IFRS 5 Non-current Assets Held for Sale and
Discontinued Operations. (5 marks) ICAG NOV,2016

298
Solution:
As the plant appears to have met the criteria to be classified as held for sale on 30 June 2015, it
will be classified as held for sale on 30 June 2015at lower of:
*Carrying value on the date of classification
*Its fair value less cost to sell on the same date
Working (31/12/2015) GHS
Cost 40,000
Less Accumulated Dep. (GHS40,000/10 years) × 2 (8,000)
Carrying value at 1.1.2015 32,000
Less Current Yr. Dep. (6 months) (GHS4,000 × 6/12) (2,000)
Carrying value at 30.6.2015 30,000
Impairment loss at 30.6.2015 (2,500)
Fair value less cost to sell at 30.6.2015 (GHS29,000 - GHS1,500) 27,500
Further impairment loss at 31.12.2015 (3,500)
Fair value less cost to sell at 31.12.2015 24,000
If fair value less cost to sell is lower than the carrying value of asset on the date of classification
the difference will be impairment loss.The asset classified as held for sale is not depreciated
after being classified as held for sale.The asset will be presented separately from other assets, as
a separate line item in the statement of financial position under current assets at GHS24,000.
QUESTION 4
Supreme is a company that operates with three divisions A,B and C.During the year ended 31
March,2011 division B closed.
Extracts from the trial balance of Supreme Ltd as at 31 March 2011 are shown below:
Dr Cr
GHC’000 GHC’000
Sales revenue – division A&C 2,400
Sales revenue – division B 650
Operating expenses – division A& C 1,650
Operating expenses – division B 525
Finance costs (all relating to continuing activities) 70
Income tax 225
The income tax charge for the year is made up of a charge of GHC200,000 on continuing
activities and GHC25,000 for discontinuing activities.
A loss of GHC50,000 was also incurred on the disposal of assets belonging to division B and
GHC80,000 was spent on restructuring divisions A&C following the termination of division B.
Required:
Prepare the statement of comprehensive income for the year ended 31 March,2011 complying
with IAS 1(revised) Presentation of Financial Statements and IFRS 5 Discontinued Operations.
Solution:
Supreme Ltd
Statement of Comprehensive Income for the year ended 31 March,2011
Continuing Operations: GHC000
Sales Revenue 2,400
Operating Expenses 1,560
Operating Profit 750
Restructuring costs (Note 1) 80
670

299
Finance costs 70
Profit before tax 600
Income tax expenses 200
Profit for the period from Continuing operations 400
Profit from Discontinued Operations (Note 2) 50
Total Profit from Operations 450
Note 1
The Restructuring costs is material and not expected to recur and should be treated as an
exceptional item.
Note 2
Discontinued operations
GHC000
Sales 650
Operating Expenses 525
Profit from Operations 125
Loss on Disposal 50
Profit before tax 75
Income tax expenses 25
Profit on Discontinued Operations 50

QUESTION 5
(a) The objective of IFRS 5 Non-current Assets Held for Sale and Discontinued Operations specifies,
amongst other things, accounting for and presentation and disclosure of discontinued
operations.
Required:
Define a discontinued operation and explain why the disclosure of such information is
important to users of financial statements.
(5 marks)
(b) Rahman‘s sole activity is the operation of hotels all over the world. After a period of
declining profitability, Rahman‘s directors made the following decisions during the year ended
31 March 2013:
– it disposed of all of its hotels in country A;
– it refurbished all of its hotels in country B in order to target the holiday and tourism market.
The previous target market in country B had been aimed at business clients.
Required:
Treating the two decisions separately, explain whether they meet the criteria for being
classified as discontinued operations in the financial statements for the year ended 31 March
2013. (4 marks)

(c) At a board meeting on 1 July 2012, Emperor‘s directors made the decision to close down one
of its factories on 31 March 2013. The factory and its related plant would then be sold.
A formal plan was formulated and the factory‘s 250 employees were given three months‘ notice
of redundancy on 1 January 2013. Customers and suppliers were also informed of the closure at
this date.
The directors of Emperor have provided the following information:
Fifty of the employees would be retrained and deployed to other subsidiaries within the group
at a cost of

300
GHC125,000; the remainder will accept redundancy and be paid an average of GHC5,000 each.
Factory plant has a carrying amount of GHC2·2 million, but is only expected to sell for
GHC500,000 incurring GHC50,000 of selling costs; however, the factory itself is expected to sell
for a profit of GHC1·2 million.
The company rents a number of machines under operating leases which have an average of
three years to run after 31 March 2013. The present value of these future lease payments
(rentals) at 31 March 2013 was GHC1 million; however, the lessor has said they will accept
GHC850,000 which would be due for payment on 30 April 2013 for their cancellation as at 31
March 2013.
Penalty payments due to non-completion of supply contracts are estimated at GHC200,000.
Required:
Explain and quantify how the closure of the factory should be treated in Emperor’s financial
statements for the year ended 31 March 2013.
Note: The closure of the factory does not meet the criteria of a discontinued operation. (6 marks)
(15 marks)

Solution:
(a) A discontinued operation is a component (see below) of an entity that has either already
been disposed of or is classified as held for sale that represents a separate major line of business
or geographical area of business operations (or is part of a co ordinated plan to dispose of such).
It also applies to a subsidiary that is acquired specifically with a view to resale.
A component of an entity has operations and cash flows that are clearly distinguished for
reporting purposes from those of the rest of an entity. It would normally be a cash generating
unit (or a group of cash generating units) or a subsidiary.
This information is important to users of financial statements when they are forming an
assessment of the likely future performance of an entity. For example, if a group made a large
profit from one of its subsidiaries that it has recently sold (or will soon sell), this will have a
material effect on any forecast of the group‘s future profit. This is because the profits from the
subsidiary disposed of will no longer contribute to future group profit (though the re-
investment of any sale proceeds from the disposal could). Also, the converse would be true
where the disposal or closure of a loss-making subsidiary could improve future profitability.

(b) IFRS 5 Non-current Assets Held for Sale and Discontinued Operations has been criticised for
the use of the term ‗ a separate major line of business or geographical area of business
operations ‘ to identify a discontinued operation as it may mean different things to different
people and lead to inconsistency (and thus a lack of comparability). Despite this, the disposal of
hotels in country A would seem to represent a separate geographical location and should be
treated as a discontinued operation, even though the group will continue to operate hotels in
other countries. The example of country B is less conclusive.
Some might argue that a change in the target market (to holiday and tourism) does represent a
different ‗line of business operations‘ that has a different pricing structure, operating costs (such
as providing ‗all-inclusive‘ holidays) and profit margins than that of business clients. Also, the
refurbishment of the hotels would seem to indicate catering to a different market. Others may
argue that this is simply adapting a product (as all companies have to do) and does not
represent a change to a separate line of business.

301
(c)On its own, a board decision to close the factory is not sufficient to justify the creation of a
provision under IAS 37 Provisions,Contingent Liabilities and Contingent Assets. However, by
formulating a plan and informing interested parties (employees,customers and suppliers), this
is likely to constitute a constructive obligation for a restructuring provision by raising a valid
expectation of the closure.
The amounts that should be provided for at 31 March 2013 are:
(workings in brackets are in GHC‘000)
GHC’000
– redundancy (200 employees x 5) 1,000
– impairment loss on plant (2,200 – (500 – 50)) 1,750
(may be shown as a separate provision)
– onerous contract (lower amount) 850
– penalty payments 200
––––––
3,800
––––––
The GHC3·8 million should be charged to the statement of profit or loss for the year ended 31
March 2013 and the same amount reported in the statement of financial position as at 31 March
2013 as a current liability/plant impairment (assuming all parts of the factory closure will be
completed within the next 12 months).
The factory and the plant would be disclosed in the statement of financial position as non-
current assets held for sale at the lower of their carrying amount (the factory) or fair value less
cost to sell (the plant).
The GHC125,000 retraining costs cannot be provided for as they are part of future activities and
the anticipated GHC1·2 million profit on the disposal of the factory cannot be recognised until it
is realised.

QUESTION 6
In accordance with IFRS 5: Non-Current Assets held for Sale and Discontinued Operations
explain with reasons, whether each of the following could most likely be classified as a
discontinued operation in this year's financial statements:
i) A reportable operating segment that met the definition of held for sale after the year end, but
before the financial statements were authorised for issue. (2 marks)
ii) A division of a business, classified as held for sale, that was correctly treated as a
discontinued operation in last year's financial statements, but which has not been sold by this
year-end due to the sale being referred to the Securities and Exchange Commission (SEC). SEC
is not expected to report its findings until 6 months after this year end. (2 marks)
SOLUTION 6
IFRS 5: Non-Current Assets held for Sale and Discontinued Operations
Many candidates incorrectly misapplied the criteria for qualification to be classified as Non-
Current Assets held for Sale and Discontinued Operations under IFRS 5. In the first part of
the question even though the operating segment met the criteria, it was after the year end
and that was non-adjusting event (IAS 10). Many candidates failed to acknowledge after the
year end and therefore provided incorrect answers. The last part was not different as many
candidates equally failed to acknowledge that referring the sale to the Securities and
Exchange Commission was not under the control of the management of the company.

302
Although this question was supposed to be relatively straightforward for candidates as it
didn‘t involve any calculations, many candidates got it wrong.
QUESTION 7
IFRS 5: Non-current Assets Held for Sale and Discontinued Operations sets out the principles
governing the measurement and presentation of non-current assets that are expected to be realised
through sale rather than through continuing use. The standard also deals with reporting the results of
operations that qualify as discontinued operations.
Required:
Identify TWO (2) conditions which must be present in order to present the results of an operation as
“discontinued” and the accounting treatment that applies when such a classification is deemed
appropriate. (4 marks)

SOLUTION

classified as held for sale, and

-ordinated plan to dispose of a separate major line of business or


geographical area of operations or

(Any 2 points @ 1 mark each = 2 marks)


Accounting Treatment
ely at the end of profit or loss by including the
profit after tax generated by discontinued operations. This figure should include the post-tax
gain or loss on disposal of the assets of the operation or the gain or loss on remeasurement
following transfer to ―held for sale‖.

distinguished, operationally and for financial reporting purposes, from the rest of the entity. In
other words, a component of an entity will have been a cash-generating unit or a group of cash-
generating units while being held for use.
-current asset (or disposal group) that is to be
abandoned. This is because its carrying amount will be recovered principally through
continuing use.

QUESTION 8
IFRS 5: Non-current Assets Held for Sale and Discontinued Operations sets out the principles
governing the measurement and presentation of non-current assets that are expected to be
realised through sale rather than through continuing use. The standard also deals with reporting
the results of operations that qualify as discontinued operations.

Required: Identify TWO (2) conditions which must be present in order to present the results of
an operation as “discontinued” and the accounting treatment that applies when such a
classification is deemed appropriate.

SOLU
A discontinued operation is a component of an entity that either has been disposed of, or is
classified as held for sale, and
iness or geographical area of operations,

303
-ordinated plan to dispose of a separate major line of business or
geographical area of operations or

Accounting Treatment
Discontinued operations are presented separately at the end of profit or loss by including the
profit after tax generated by discontinued operations. This figure should include the post-tax gain
or loss on disposal of the assets of the operation or the gain or loss on remeasurement following
transfer to “held for sale”.

distinguished, operationally and for financial reporting purposes, from the rest of the entity. In
other words, a component of an entity will have been a cash generating unit or a group of cash-
generating units while being held for use.
-current asset (or disposal group) that is to be
abandoned. This is because its carrying amount will be recovered principally through continuing
use.

Question 9
Asawasi Ltd made a decision to sell a business unit on 15 June 2017, and the criteria to classify
the unit as held for sale were met on 1 July 2017. Asawasi Ltd‟s accounting year end is 31
December. At 1 July 2017, the carrying amount of the assets and liabilities of the business unit
(before any 2017 depreciation or revaluation adjustments) was as follows:
GH¢ million
Land and buildings 120
Equipment 50
Trade receivables 30
Inventories 20
Trade payables (26)

Additional Information:
i) The land and buildings are held under the revaluation model of IAS 16: Property, Plant and
Equipment and were last revalued on 31 December 2016 to GH¢120 million. Their market
valuation on 1 July 2017 was GH¢124 million and selling costs were estimated at GH¢2.5
million at that date. Residual value was, and continues to be, expected to be higher than cost.
ii) The equipment is held under the cost model of IAS 16. The equipment was purchased on 1
July 2015 for GH¢80 million and is being depreciated straight line over a four-year period to a
zero residual value. Its sale value at 1 July 2017 was GH¢55 million. Selling costs are
insignificant.
iii) The trade receivables are recorded at invoiced value, reduced by any allowances for credit
losses recognised at 31 December 2016. No adjustment to these allowances was necessary at 1
July 2017. The receivables, if factored, would realise approximately GH¢26 million, net of
transaction costs at 1 July 2017.
iv) The inventories are merchandise purchased for resale and are held at cost. Their market value
at 1 July 2017 was GH¢28 million. Associated selling costs would amount to GH¢1.4 million.
v) It was anticipated at 1 July 2017 that the business unit will be sold for GH¢200 million, net of
selling costs, to a rival company in a single transaction.

304
Required: In respect of Asawasi Ltd‟s year ended 31 December 2017, show, and briefly
explain, the amount recognised as Non-Current Assets Held for Sale under IFRS 5 at 1 July
2017 and the impairment charge (if any) for the business unit.
SOLU
Non-Current Assets Held for Sale (IFRS 5) Carrying amount at 1 July 2017, after applying IAS
16:
GH¢ million
Land and buildings 124
Equipment (50 – (80/4 years x 6/12)) 40
Trade receivables 30
Inventories 20
Trade payables (26)
188
Any test for impairment will be based on the disposal group as a whole. As a disposal group, fair
value less costs to sell (GH¢200 million) is higher than carrying amount (GH¢188 million) there
is no impairment charge.
The amount recognised as non-current assets held for sale is therefore:
GH¢ million
Land and buildings 124
Equipment 40
164
Trade receivables and inventories are outside the scope of IFRS 5.

305
IFRS 6

EXPLORATION FOR AND


EVALUATION OF MINERAL
RESOURCES(IFRS 6)

Purpose of the project


IFRS 6 confirms that the requirements of all IFRSs are applicable to entities involved in the
exploration for and evaluation of mineral resources, except where specific types of transaction
and activities are excluded from IFRS. The accounting for exploration of and evaluation of
mineral resources is explicitly excluded from the scope of IAS 16 Property, plant and equipment
and IAS 38 Intangible assets .
The IASB did not have time to develop comprehensive financial reporting in time for entities
converting to IFRS in 2005. IFRS 6 proposes an interim solution to help companies deal with the
IAS 16 and IAS 38 scope exclusions until the IASB develops a comprehensive solution.
Scope of the project
IFRS 6 exploration for and evaluation of mineral resources as:
"The search for mineral resources, including minerals, oil, natural gas and similar non-
regenerative resources, as well as the determination of the feasibility and commercial viability
of extracting the mineral resource before the decision to develop the mineral resource" .
IFRS 6 does not apply to:
(a) expenditure incurred in the development of mineral resources after commercial viability
has been determined.
(b) costs incurred before the right to explore has been established.

IAS 16 and IAS 38 rules continue to apply to the development and production of property,
plant and equipment and intangibles to be utilised in the exploration and evaluation. Property,
plant and equipment used by an entity to develop or maintain exploration and evaluation assets
continue to be accounted for in accordance with IAS 16.

Recognition and initial measurement of exploration and evaluation expenditures


IFRS 6 allows an entity to continue to apply its existing accounting policy in respect of
exploration for and evaluation of mineral resources. These policies may be applied even if they
are inconsistent with the treatment of similar expenditures that are addressed by existing IFRSs
or if they are inconsistent with the IFRS Framework.

306
An entity that chooses to make changes in its accounting policies can only do so if those changes
bring the policy closer in line with the treatment of similar expenditures set out in IFRSs and the
IFRS Framework (i.e. IAS 8 rules on accounting policy changes apply).
Recognition of exploration and evaluation assets
Exploration and evaluation assets should be classified as tangible or intangible according to
their nature. This classification should be applied consistently.
Acquired exploration and evaluation assets
IFRS 3 should apply to these assets.

Subsequent measurement
Entities should apply the IAS 16 and IAS 38 guidance to measure exploration and evaluation
assets at either:
 cost less accumulated depreciation/amortisation and accumulated impairment; or
 revalued amount less accumulated depreciation/amortisation and accumulated
impairment. Remeasurement of exploration and evaluation asset will be permitted when
the revaluation criteria are met, IAS 16 and 38‘s guidance on remeasurement will apply .

Impairment
Impairment testing is required where there is an indication of impairment. In such
circumstances IAS 36 requirements will apply to exploration and evaluation assets.
The following examples of possible indicators of impairment of exploration assets have been
provided:
(a) expiry of the right to explore where this right is not expected to be renewed
(b) additional expenditure on exploration in specific area has not been budgeted or planned
for
(c) failure to discover commercially viable resources and the intention to discontinue future
exploration
(d) carrying amount of assets unlikely to be recovered through successful resource finds or
disposal
An entity shall determine an accounting policy for allocating exploration and evaluation assets
to cash-generating units or groups of cash-generating units for the purpose of assessing such
assets for impairment.
Each cash-generating unit or group of units to which an exploration and evaluation asset is
allocated shall not be larger than an operating segment determined in accordance with IFRS 8
Operating Segments.

The IASB Board decided that its approach to the impairment of goodwill in the 2004 revisions to
IAS 36 offered the best model available within IFRSs to accomplish its objective. It noted that
entities might be able to monitor exploration and evaluation assets for internal management
purposes at the level of an oilfield or a contiguous ore body.

The IASB Board did not intend to require impairment to be assessed at such a low level.
Consequently, the IFRS permits CGUs to be aggregated.

However, the IASB Board decided to require the level at which impairment was assessed to be
no larger than an operating segment, in accordance with IFRS 8 Operating Segments.

307
The IASB Board concluded, consistently with the approach to goodwill in IAS 36, that this
approach was necessary to ensure that entities managed on a matrix basis could test exploration
and evaluation assets for impairment at the level of reporting that reflects the way they manage
their operations.

Comparative
Where it is impracticable to apply the proposals for impairment testing of exploration and
evaluation assets to prior period comparatives an entity should disclose that fact.

Overview
Entities should disclose information that identifies and explains the amounts recognised in the
financial statements relating to exploration and evaluation of mineral resources. The disclosures
should include the accounting policy followed and the level at which impairment of exploration
and evaluation assets is assessed (IFRS6.23-24 ).
References to IAS 16 and 38
Exploration and evaluation assets shall be treated as a separate class of assets.
Disclosures required by IAS 16 and IAS 38 should apply to the exploration and evaluation
assets.

308
FAIR VALUE MEASUREMENTS (IFRS 13)
NEED FOR FAIR VALUE
GUIDANCE:
IFRS 13 provides a single source of guidance for all fair value measurements, clarifying the
definition of fair value and enhancing disclosures requirements about reported fair value
estimates.

FAIR VALUE DEFINITION


Fair value is the price that would be received to sell an asset or paid to transfer a liability in an
orderly transaction between market participants at the measurement date.

From the above definition, it can be inferred that fair value is an exit price.

IFRS 13 provides a new framework to estimate fair value in a consistent manner across
standards.
For a fair value measurement, an entity has to determine:
The particular asset or liability that is the subject of the measurement
For an asset, the valuation premise that is appropriate for the measurement
The most advantageous market for the asset or liability and
The valuation technique appropriate for measurement.

THE MOST ADVANTAGEOUS MARKET


It is assumed that transactions take place in the most advantageous market to which the entity
has access. This means that the entity is in a position to receive the maximum amount on sale of
the asset or pay the minimum amount to transfer a liability after considering transaction and
transport costs.
While transaction and transport costs are relevant to identify the market, they are not
considered in determining the fair value.

MEASUREMENT ASSUMPTIONS
Fair value measurement of an asset or liability should use the assumptions that market
participants would use in pricing the asset or liability.
These assumptions include:
buyers and sellers are independent of each other

309
they have knowledge about the asset or liability
they are capable of entering into a transaction
They are willing to enter into a transaction, rather than being forced or otherwise compelled.

ASSET -SPECIFIC VALUATIONS


For a fair value measurement of an asset, it is assumed that the asset will be sold to a market
participant who will use it at its highest and best use.

LIABILITY-SPECIFIC VALUATION
Fair value measurement of a liability assumes that the liability is transferred to a market
participant at the measurement date. Where there is no observable market price for the transfer
of a liability, an entity would be required to measure the fair value of the liability using the
same methodology that the counterparty would use to measure the fair value of the
corresponding asset.

VALUATION TECHNIQUES
An entity uses valuation techniques appropriate in the circumstances and for which sufficient
data are available to measure fair value, maximising the use of relevant observable inputs and
minimising the use of unobservable inputs.Where fair value is determined using a valuation
technique, IFRS 13 prescribes that the technique should be one of the following.

i.Market approach: uses price and other relevant market information for identical or
comparable assets or liabilities

ii.Income approach: converts future amounts to a single discounted present value amount or

iii.Cost approach: amount that would currently be required to replace the service capacity of
the asset

iv.Fair value hierarchy


IFRS 13 seeks to increase consistency and comparability in fair value measurements and related
disclosures through a 'Fair Value Hierarchy'.
The hierarchy categorises the inputs used in valuation techniques into three levels.

Level 1 Inputs
Quoted prices (unadjusted) in active markets for identical assets or liabilities that the entity can
access at the measurement date.

Level 2 Inputs
Inputs other than quoted prices included in Level 1 that are directly or indirectly observable.

Level 3 Inputs
Inputs for the asset or liability that are not based on observable market data (unobservable
inputs).

RULES IN BUSINESS COMBINATION:

310
IFRS 3 sets out general principles for arriving at the fair values of a subsidiary's assets and
liabilities only if they satisfy the following criteria:
In the case of an asset other than an intangible asset, it is probable that any associated future
economic benefits will flow to the acquirer, and its fair value can be measured reliably. Vice
versa for liabilities

In the case of an intangible asset or a contingent liability, its fair value can be measured
reliably.

The acquiree's identifiable assets and liabilities might include assets and liabilities not
previously recognised in the acquiree's financial statements

An acquirer should not recognise liabilities for future losses or other costs expected to be
incurred as a result of the business combination.
The acquiree may have intangible assets which can only be recognised separately from
goodwill if they are identifiable. They must be able to be capable of being separated from the
entity.

The acquirer should measure the cost of a business combination as the total of the fair values at
the date of acquisition

If part of the consideration is payable at a later date, this deferred consideration is discounted
to present value at the date of exchange.
In case of equity instruments as cost of investment, the published price at the date of exchange
normally provides the best evidence of the instrument's fair value.
Costs attributable to the combination, for example professional fees and administrative costs,
should not be included: they are recognised as an expense when incurred.
If an asset or liability has been recognised at fair value at acquisition, it must be recorded in the
subsidiary‗s statement of financial position at fair value consequently also
Some fair value adjustments are made on depreciable assets such as buildings, the assets with
fair value adjustment must be depreciated at its fair value so there will be an adjustment, which
flows throughto profit or loss for this additional depreciation.

QUESTION 1
The International Accounting Standards Board has recently completed a joint project with the
Financial Accounting Standards Board (FASB) on fair value measurement by issuing IFRS 13
Fair Value Measurement.
IFRS 13 defines fair value, establishes a framework for measuring fair value and requires
significant disclosures relating to fair value measurement.
The IASB wanted to enhance the guidance available for assessing fair value in order that users
could better gauge the valuation techniques and inputs used to measure fair value. There are no
new requirements as to when fair value accounting is required, but the IFRS gives guidance
regarding fair value measurements in existing standards. Fair value measurements are
categorised into a three-level hierarchy, based on the type of inputs to the valuation techniques
used. However, the guidance in IFRS 13 does not apply to transactions dealt with by certain
specific standards.
Required:

311
(i) Discuss the main principles of fair value measurement as set out in IFRS 13. (7 marks)
(ii) Describe the three-level hierarchy for fair value measurements used in IFRS 13. (6 marks)

Solution:
(i)Fair value has had a different meaning depending on the context and usage. The IASB‘s
definition is the price that would be received to sell an asset or paid to transfer a liability in an
orderly transaction between market participants at the measurement date. Basically it is an exit
price. Fair value is focused on the assumptions of the market place and is not entity specific. It
therefore takes into account any assumptions about risk. Fair value is measured using the same
assumptions and taking into account the same characteristics of the asset or liability as market
participants would. Such conditions would include the condition and location of the asset and
any restrictions on its sale or use. Further, it is not relevant if the entity insists that prices are too
low relative to its own valuation of the asset and that it would be unwilling to sell at low prices.
Prices to be used are those in ‗an orderly transaction‘. An orderly transaction is one that
assumes exposure to the market for a period before the date of measurement to allow for
normal marketing activities and to ensure that it is not a forced transaction. If the transaction is
not ‗orderly‘, then there will not have been enough time to create competition and potential
buyers may reduce the price that they are willing to pay. Similarly, if a seller is forced to accept
a price in a short period of time, the price may not be representative. It does not follow that a
market in which there are few transactions is not orderly. If there has been competitive tension,
sufficient time and information about the asset,then this may result in a fair value for the asset.

IFRS 13 does not specify the unit of account for measuring fair value. This means that it is left to
the individual standard to determine the unit of account for fair value measurement. A unit of
account is the single asset or liability or group of assets or liabilities. The characteristic of an
asset or liability must be distinguished from a characteristic arising from the holding of an asset
or liability by an entity. An example of this is that if an entity sold a large block of shares, it may
have to do so at a discount to the market price. This is a characteristic of holding the asset rather
than of the asset itself and should not be taken into account when fair valuing the asset.

Fair value measurement assumes that the transaction to sell the asset or transfer the liability
takes place in the principal market for the asset or liability or, in the absence of a principal
market, in the most advantageous market for the asset or liability. The principal market is the
one with the greatest volume and level of activity for the asset or liability that can be accessed
by the entity.

The most advantageous market is the one which maximises the amount that would be received
for the asset or minimises the amount that would be paid to transfer the liability after transport
and transaction costs.

An entity does not have to carry out an exhaustive search to identify either market but should
take into account all available information. Although transaction costs are taken into account
when identifying the most advantageous market,the fair value is not after adjustment for
transaction costs because these costs are characteristics of the transaction and not the asset or
liability. If location is a factor, then the market price is adjusted for the costs incurred to

312
transport the asset to that market. Market participants must be independent of each other and
knowledgeable, and able and willing to enter into transactions.

IFRS 13 sets out a valuation approach, which refers to a broad range of techniques, which can be
used. These techniques are threefold. The market, income and cost approaches.

(ii)When measuring fair value, the entity is required to maximise the use of observable inputs
and minimise the use of unobservable inputs. To this end, the standard introduces a fair value
hierarchy, which prioritises the inputs into the fair value measurement process.
Level 1 inputs are quoted prices (unadjusted) in active markets for items identical to the asset
or liability being measured. As with current IFRS, if there is a quoted price in an active market,
an entity uses that price without adjustment when measuring fair value. An example of this
would be prices quoted on a stock exchange. The entity needs to be able to access the market at
the measurement date. Active markets are ones where transactions take place with sufficient
frequency and volume for pricing information to be provided. An alternative method may be
used where it is expedient. The standard sets out certain criteria where this may be applicable.
For example, where the price quoted in an active market does not represent fair value at the
measurement date. An example of this may be where a significant event takes place after the
close of the market such as a business reorganisation or combination.

The determination of whether a fair value measurement is level 2 or level 3 inputs depends on
whether the inputs are observable inputs or unobservable inputs and their significance.

Level 2 inputs are inputs other than the quoted prices in level 1 that are directly or indirectly
observable for that asset or liability. They are quoted assets or liabilities for similar items in
active markets or supported by market data. For example, interest rates, credit spreads or yield
curves. Adjustments may be needed to level 2 inputs and if this adjustment is significant, then it
may require the fair value to be classified as level 3.

Level 3 inputs are unobservable inputs. The use of these inputs should be kept to a minimum.
However, situations may occur where relevant inputs are not observable and therefore these
inputs must be developed to reflect the assumptions that market participants would use when
determining an appropriate price for the asset or liability. The entity should maximise the use of
relevant observable inputs and minimise the use of unobservable inputs. The general principle
of using an exit price remains and IFRS 13 does not preclude an entity from using its own data.
For example, cash flow forecasts may be used to value an entity that is not listed. Each fair value
measurement is categorised based on the lowest level input that is significant to it.

QUESTION 2
Jayach, a public limited company, is reviewing the fair valuation of certain assets and liabilities
in light of the introduction of IFRS 13.
It carries an asset that is traded in different markets and is uncertain as to which valuation to
use. The asset has to be valued at fair value under International Financial Reporting Standards.
Jayach currently only buys and sells the asset in the Australasian market. The data relating to
the asset are set out below:
Year to 30 November 2012 Asian European Australasian
Market Market Market

313
Volume of market – units 4 million 2 million 1 million
Price GHC19 GHC16 GHC22
Costs of entering the market GHC2 GHC2 GHC3
Transaction costs GHC1 GHC2 GHC2
Additionally, Jayach had acquired an entity on 30 November 2012 and is required to fair value a
decommissioning liability. The entity has to decommission a mine at the end of its useful life,
which is in three years‘ time. Jayach has determined that it will use a valuation technique to
measure the fair value of the liability.
If Jayach were allowed to transfer the liability to another market participant, then the following
data would be used.
Input Amount
Labour and material cost GHC2 million
Overhead 30% of labour and material cost
Third party mark-up – industry average 20%
Annual inflation rate 5%
Risk adjustment – uncertainty relating to cash flows 6%
Risk-free rate of government bonds 4%
Entity‘s non-performance risk 2%
Jayach needs advice on how to fair value the liability.
Required:
Discuss, with relevant computations, how Jayach should fair value the above asset and
liability under IFRS 13. (10 marks)

Solution:
Year to 31 December 2012 Asian Market European Market Australasian Market
Volume of market – units 4 million 2 million 1 million
Price GHC19 GHC16 GHC22
Costs of entering the market (GHC2) (GHC2) (n/a) see note
Potential fair value GHC17 GHC14 GHC22
Transaction costs (GHC1) (GHC2) (GHC2)
Net profit GHC16 GHC12 GHC20
Note:As Jayach buys and sells in Australasia, the costs of entering the market are not relevant as
these would not be incurred.Further transaction costs are not considered as these are not
included as part of the valuation.
The principal market for the asset is the Asian market because of the fact that it has the highest
level of activity due to the highest volume of units sold. The most advantageous market is the
Australasian market because it returns the best profit per unit. If the information about the
markets is reasonably available, then Jayach should base its fair value on prices in the Asian
market due to it being the principal market, assuming that Jayach can access the market. The
pricing is taken from this market even though the entity does not currently transact in the
market and is not the most advantageous. The fair value would be $17, as transport costs would
be taken into account but not transaction costs.
If the entity cannot access the Asian or European market, or reliable information about the
markets is not available, Jayach would use the data from the Australasian market and the fair
value would be GHC22. The principal market is not always the market in which the entity
transacts. Market participants must be independent of each other and knowledgeable, and able
and willing to enter into transactions.

314
Input Amount (GHC 000)
Labour and material cost 2,000
Overhead (30%) 600
Third party mark-up – industry average (20% of 2,600) 520
––––––
Total 3,120
Annual inflation rate (3,120 x 5% compounded for three years) 492
––––––
Total 3,612
Risk adjustment – 6% 217
––––––
Total 3,829
––––––
Discounted at risk free rate of government bonds plus entity‘s non-performance risk – 6% 3,215
The fair value of a liability assumes that it is transferred to a market participant at the
measurement date. In many cases there is no observable market to provide pricing information.
In this case, the fair value is based on the perspective of a market participant who holds the
identical instrument as an asset. If there is no corresponding asset, then a valuation technique is
used. This would be the case with the decommissioning activity. The fair value of a liability
reflects any compensation for risk and profit margin that a market participant might require to
undertake the activity plus the non-performance risk based on the entity‘s own credit standing.
Thus the fair value of the decommissioning liability would be GHC3,215,000.

DISCUSSION QUESTION
Gonja Ltd is an investment company which holds a portfolio of securities linked to the real
estatemarket in Ghana. The following information is available at 31 July 2018 regarding this
portfolio:
i) The portfolio cost GH¢13 million 2 years ago.
ii) Real-estate prices in Ghana are generally accepted to have dropped by 20-30% in the past 2
years.
iii) The portfolio of securities held by Gonja is difficult to value, as there is no active market.
However, the company has received an offer of GH¢2.6 million for this portfolio from an
investor. It has no intention of accepting this offer although similar companies have accepted
offers from this investor due to financial difficulties.
iv) A normal sale in the present climate could be reasonably expected to yield GH¢6 million,
based on an analysis of transactions in similar assets.
v) Gonja‘s valuation models suggest that the real estate market in Ghana will recover, and it
expects that the portfolio will generate GH¢12 million (at present value) over the next three
years.
Required:
In accordance with IFRS 13: Fair Value Measurement, advise Gonja Ltd on the amount it should
state its investment portfolio in its financial statements to 31 July 2018, assuming it
wishes to use fair value as measured. (5 marks)

315
REVENUE FROM CONTRACTS WITH
CUSTOMERS (IFRS 15)
IFRS 15 establishes the principles that an entity applies when reporting information about the
nature, amount, timing and uncertainty of revenue and cash flows from a contract with a
customer.
SCOPE
IFRS 15 - Revenue from Contracts with Customers applies to all contracts with customers,
except for:
–Lease contracts within the scope of IAS 17/IFRS 16 Leases;
–Insurance contracts within the scope of IFRS 4 Insurance Contracts;
–Financial instruments and other contractual rights and obligations within the scope of IFRS 9
Financial Instruments, IFRS 10 Consolidated Financial Statements, IFRS 11 Joint Arrangements,
IAS 27 Separate Financial Statements and IAS 28 Investments in Associates and Joint Ventures.
–Non-monetary exchanges between entities in the same line of business to facilitate sales to
customers or potential customers (such as a contract between two oil customers to exchange oil
to fulfil demand from their customers in different specified locations).
Definitions:
The following terms are used in this Standard with the meanings specified:
Contract: An agreement between two or more parties that creates enforceable rights and
obligations.

Contract asset: An entity‘s right to consideration in exchange for goods or services that the
entity has transferred to a customer when that right is conditioned on something other than the
passage of time (for example, the entity‘s future performance).

Contract liability: An entity‘s obligation to transfer goods or services to a customer for which
the entity has received consideration (or the amount is due) from the customer.

Customer: A party that has contracted with an entity to obtain goods or services that are an
output of the entity‘s ordinary activities in exchange for consideration.

Income: Increases in economic benefits during the accounting period in the form of inflows or
enhancements of assets or decreases of liabilities that result in an increase in equity, other than
those relating to contributions from equity participants.

316
Performance obligation: A promise in a contract with a customer to transfer to the customer
either:
(a) a good or service (or a bundle of goods or services) that is distinct; or
(b) a series of distinct goods or services that are substantially the same and that have the same
pattern of transfer to the customer.

Revenue: Income arising in the course of an entity‘s ordinary activities.


Stand- alone selling price (of a good or service) : The price at which an entity would sell a
promised good or service separately to a customer.

Transaction price (for a contract with a customer): The amount of consideration to which an
entity expects to be entitled in exchange for transferring promised goods or services to a
customer, excluding amounts collected on behalf of third parties
CORE PRINCIPLE
Applying IFRS 15, an entity recognises revenue to depict the transfer of promised goods or
services to the customer in an amount that reflects the consideration to which the entity expects
to be entitled in exchange for those goods or services.
To recognise revenue under IFRS 15, an entity applies the following five steps:
THE FIVE-STEP MODEL FRAMEWORK
The standard provides a single, principles based five-step model to be applied to all contracts
with customers.
STEP 1: IDENTIFY THE CONTRACT WITH THE CUSTOMER
A contract with a customer will be within the scope of IFRS 15 if all the following conditions are
met:
 The contract has been approved by the parties to the contract;
 Each party‘s rights in relation to the goods or services to be transferred can be identified;
 the payment terms for the goods or services to be transferred can be identified;
 the contract has commercial substance; and
 It is probable that the consideration to which the entity is entitled to in exchange for the
goods or services will be collected.
STEP 2: IDENTIFY THE PERFORMANCE OBLIGATIONS IN THE CONTRACT
At the inception of the contract, the entity should assess the goods or services that have been
promised to the customer, and identify as a performance obligation:
 a good or service (or bundle of goods or services) that is distinct; or
 A series of distinct goods or services that are substantially the same and that have the
same pattern of transfer to the customer.

A series of distinct goods or services is transferred to the customer in the same pattern if both of
the following criteria are met:
 each distinct good or service in the series that the entity promises to transfer
consecutively to the customer would be a performance obligation that is satisfied over
time; and
 A single method of measuring progress would be used to measure the entity‘s progress
towards complete satisfaction of the performance obligation to transfer each distinct
good or service in the series to the customer.
A good or service is distinct if both of the following criteria are met:

317
 The customer can benefit from the good or services on its own or in conjunction with
other readily available resources; and
 The entity‘s promise to transfer the good or service to the customer is separately
identifiable from other promises in the contract.
Factors for consideration as to whether a promise to transfer the good or service to the customer
is separately identifiable include, but are not limited to:
 The entity does not provide a significant service of integrating the good or service with
other goods or services promised in the contract.
 The good or service does not significantly modify or customise another good or service
promised in the contract.
 The good or service is not highly interrelated with or highly dependent on other goods
or services promised in the contract.
STEP 3: DETERMINE THE TRANSACTION PRICE
The transaction price is the amount to which an entity expects to be entitled in exchange for the
transfer of goods and services. When making this determination, an entity will consider past
customary business practices.
Where a contract contains elements of variable consideration, the entity will estimate the
amount of variable consideration to which it will be entitled under the contract.
However, a different, more restrictive approach is applied in respect of sales or usage-based
royalty revenue arising from licences of intellectual property. Such revenue is recognised only
when the underlying sales or usage occur.
STEP 4: ALLOCATE THE TRANSACTION PRICE TO THE PERFORMANCE OBLIGATIONS
IN THE CONTRACTS
Where a contract has multiple performance obligations, an entity will allocate the transaction
price to the performance obligations in the contract by reference to their relative standalone
selling prices. If a standalone selling price is not directly observable, the entity will need to
estimate it. IFRS 15 suggests various methods that might be used, including:
 Adjusted market assessment approach
 Expected cost plus a margin approach
 Residual approach (only permissible in limited circumstances).
STEP 5: RECOGNISE REVENUE WHEN (OR AS) THE ENTITY SATISFIES A PERFORMANCE
OBLIGATION
Revenue is recognised as control is passed, either over time or at a point in time.
Control of an asset is defined as the ability to direct the use of and obtain substantially all of the
remaining benefits from the asset. The benefits related to the asset are the potential cash flows
that may be obtained directly or indirectly. These include, but are not limited to:
 Using the asset to produce goods or provide services;
 Using the asset to enhance the value of other assets;
 Using the asset to settle liabilities or to reduce expenses;
 Selling or exchanging the asset;
 Pledging the asset to secure a loan; and
 Holding the asset.
An entity recognises revenue over time if one of the following criteria is met:
 The customer simultaneously receives and consumes all of the benefits provided by the
entity as the entity performs;

318
 The entity‘s performance creates or enhances an asset that the customer controls as the
asset is created; or
 The entity‘s performance does not create an asset with an alternative use to the entity
and the entity has an enforceable right to payment for performance completed to date.
If an entity does not satisfy its performance obligation over time, it satisfies it at a point in time.
Revenue will therefore be recognised when control is passed at a certain point in time. Factors
that may indicate the point in time at which control passes include, but are not limited to:
 The entity has a present right to payment for the asset;
 The customer has legal title to the asset;
 The entity has transferred physical possession of the asset;
 The customer has the significant risks and rewards related to the ownership of the asset;
and
 The customer has accepted the asset.

CONTRACT COSTS
The incremental costs of obtaining a contract must be recognised as an asset if the entity expects
to recover those costs. Costs incurred to fulfil a contract are recognised as an asset if and only if
all of the following criteria are met:
 The costs relate directly to a contract (or a specific anticipated contract);
 The costs generate or enhance resources of the entity that will be used in satisfying
performance obligations in the future; and
 The costs are expected to be recovered.
The asset recognised in respect of the costs to obtain or fulfil a contract is amortised on a
systematic basis that is consistent with the pattern of transfer of the goods or services to which
the asset relates.

OTHER ISSUES
Contract costs
Contract costs are initially recognised as an asset and expensed on a systematic basis that is
consistent with the transfer to the customer of the good or service to which those costs relate.
Contract costs comprise both incremental costs of obtaining a contract and costs to fulfil a
contract.

Incremental costs of obtaining a contract


Incremental costs incurred in obtaining a contract are those that would not have been incurred
had that individual contract not been obtained. This is restrictive and includes only costs such
as a sales commission that is paid only if the contract is obtained, unless the costs can be
explicitly recharged to a customer.
As a practical expedient, incremental costs of obtaining a contract can be recognised as an
immediate expense rather than capitalised if the period over which they would otherwise be
expensed (or amortised) is one year or less.

All other ongoing costs of running the business, including costs that are incurred with the
intention of obtaining a contract with a customer, are not incremental and will be expensed
unless they fall within the scope of another accounting standard (such as IAS 16 Property, Plant
and Equipment) and are required to be accounted for as an asset.

319
Costs to fulfil a contract
In contrast with the incremental costs of obtaining a contract, which fall wholly within its scope,
the requirements of IFRS 15 apply only to costs to fulfil a contract which do not fall within the
scope of another IFRS (for example, IAS 2 Inventories, IAS 16 Property, Plant and
Equipmentand IAS 38 Intangible Assets). For those costs which do fall within the scope of IFRS
15, the threshold for identifying costs to fulfil a contract is lower than the ‗incremental‘
threshold for costs in obtaining a contract. However, there are still restrictions and all of the
following criteria need to be met:
–The fulfilment costs relate directly to a contract or to an anticipated contract that can
specifically be identified;
–The costs generate or enhance resources of the vendor that will be used to satisfy performance
obligations in future; and
–The costs are expected to be recovered.

Sale with a right of return


A right to return enables a customer to receive:
–A full or partial refund of any consideration paid;
–A credit that can be applied against amounts owed or that will be owed to the vendor;
–Another product in exchange;
–Any combination of the above.

A right to return may be given for various reasons such as customer dissatisfaction with the
product or simply given if the customer changes their mind.

Although a right of return falls within the variable consideration guidance in IFRS 15, the price
per item sold (i.e. the contractual price) does not vary. Unlike other situations in which there is
variable consideration the entity will receive back the goods sold when a right or return is
exercised. However, the aggregate amount of revenue recognised is subject to variability.
Consequently, IFRS 15 requires that the variable consideration provisions in the standard
(including the requirements for constraining variable consideration) should apply when
measuring the amount of revenue to recognise for goods sold with a right of return.

For those items which are expected to be returned, the vendor does not recognise revenue.
Instead, it recognises a refund liability together with an asset representing item(s) expected to
be returned. Any refund liability is reassessed and updated at each reporting date. If the
realisable value of the item to be returned (including any adjustment for expected costs of
recovering the item and any potential decrease in value) is expected to be less than the cost of
the related inventory, an adjustment is made to cost of sales.

When a vendor transfers products with a right of return, revenue is recognised only to the
extent that the vendor expects to be entitled to it. To determine the amount of consideration to
which it expects to be entitled, a vendor:
–Applies the guidance regarding constraining estimates of variable consideration;
–Considers the nature of the products expected to be returned.

In subsequent periods the vendor updates:

320
–Its assessment(s) of amounts to which it expects to be entitled in exchange for the transferred
products;
–The measurement of the refund liability at the end of each reporting period for changes in
expectations about the amount of refunds;
–The measurement of the asset (i.e. so that it corresponds with changes in the measurement of
the refund liability and any impairment recognised).

Note that a vendor‘s obligation to accept a returned product during the return period is not
accounted for as a performance obligation in addition to the obligation to provide a refund
Warranties
IFRS15 distinguishes between two types of warranties:
–Warranties that provide a customer with the assurance that the product will function as
intended because it complies with agreed-upon specifications. These warranties may be
provided in accordance with the contract (and hence included in the purchase price of the
goods), required to be provided by law, or are provided in accordance with the vendor‘s
customary business practices. They are accounted for in accordance with the guidance on
product warranties included within IAS 37 Provisions, Contingent Liabilities and Contingent
Assets and not as a separate performance obligation.
Therefore, a vendor recognises a provision for the expected costs of meeting their obligation
under the warranty. If the warranty provides the customer with a service in addition to the
assurance that the product complies with agreed upon specifications, then only that incremental
service will be a performance obligation to which some of the transaction price should be
allocated;

–Optional warranties that provide the customer with assurance that the product complies with
agreed-upon specifications for an extended period. These ‗extended warranties‘ are accounted
for as a separate performance obligation from the related goods which have been sold. It is
therefore necessary to allocate a portion of the transaction price to the warranty in accordance
with the requirements of IFRS 15.

In assessing whether a contract contains a service in addition to the assurance that the product
complies with agreed-upon specifications, judgment may be needed. A vendor considers factors
such as:
–Whether the warranty is required by law – if required by law then this indicates the warranty
is not a separate performance obligation;
–The length of the warranty coverage period – the longer the coverage, the more likely it is that
the promised warranty is a performance obligation;
–The nature of the tasks that the vendor promises to perform – if the vendor must perform
specified tasks to provide the assurance that a product complies with agreed-upon
specifications (e.g. a return shipping service for a defective product), then those tasks are
unlikely to give rise to a performance obligation.

If a customer does not have an option to purchase a warranty separately, it is accounted for in
accordance with IAS 37 unless part or all of that warranty provides the customer with a service
in addition to an assurance that the good or services complies with agreed-upon specifications.

Principal vs. agent

321
When a third party is involved in providing goods or services to a customer, the vendor is
required to determine whether the nature of its promise is a performance obligation to:
–Provide the specified goods or services itself (principal); or
–Arrange for a third party to provide those goods or services (agent).

A vendor acting as principal controls a good or service before the vendor transfers the good or
service to the customer. It may satisfy a performance obligation by itself or engage another
party (for example, a subcontractor) to satisfy some or all of a performance obligation on its
behalf. When a vendor, in its role as a principal, satisfies a performance obligation, it recognises
revenue at the gross amount. However, the vendor is not necessarily acting as a principal if the
vendor obtains legal title of a product only shortly before legal title is transferred to a customer.
The obligation of an agent is to arrange for the provision of goods or services by another third
party. When a vendor represents an agent, and satisfies a performance obligation, it recognises
revenue as the amount of any fee or commission to which it expects to be entitled. A vendor‘s
fee or commission might be the net amount of consideration that the vendor retains after paying
the third party the consideration received in exchange for the goods or services to be provided
by that party. A vendor acting as agent does not control the good or service before it is
transferred to the principal‘s customer.

Indicators that an entity is acting as principal include:


–Having primary responsibility for fulfilling the promise to provide the specified good or
service;
–Assuming inventory risk before the specified good or service has been transferred to the
customer or after transfer of control to the customer (e.g. the customer has a right of return);
–Having discretion in establishing the price for the specified good or service.

The relevance of each of these indicators depend on the nature of the specified good or service,
and different indicators may provide more evidence in different contracts.

Customer options for additional goods or services


Customer options to acquire additional goods or services (either free of charge or at a discount)
come in many forms, including sales incentives, customer award credits (or points), contract
renewal options, or other discounts on future goods or services. Such customer options give rise
to a performance obligation in the contract when the option provides a material right to the
customer that it would not receive without entering into the contract. In those cases, the vendor
is required to defer the portion of payment received from its customer that relates to those
future goods or services and recognise that portion as revenue only when those future goods or
services are transferred to the customer (or when the option expires).
The allocation is based on the relative stand-alone selling prices of the goods or services and, if
the prices of the future potential goods or services are not observable, they are estimated. This
estimate takes into account any discount that the customer would receive without exercising the
option together with the likelihood that the option will be exercised.

Renewal options
A renewal option is different from customer loyalty programmes and many discount vouchers.
With loyalty programmes and vouchers, the underlying goods or services in the contract with
the customer will often have a different nature, and accordingly they would be considered as

322
separate deliverables rather than being similar to the original goods or services in the original
contract. A renewal option, in contrast, gives a customer the right to acquire additional goods or
services of the same type as those supplied under an existing contract.
If an entity grants a customer the option to acquire additional goods or services, that option
only gives rise to a performance obligation if it provides a material right to the customer that it
would not receive without entering into that contract (for example, a discount that is
incremental to the range of discounts typically given). Similarly, an option to acquire additional
goods or services at a price that would reflect their stand-alone selling price also does not
constitute a material right even if it can be exercised only be entering into an earlier contract.

Accounting for early renewal rights


It is common for entities to offer non-cancellable contracts that provide the customer with the
option to renew the contract prior to contract expiry. This would be common in
telecommunications industry and other industries where a product is sold on day 1 of the
contract with ongoing services to be provided over the contract period.

Breakage (unexercised rights)


Breakage refers to situations where customers do not exercise all of their contractual rights to
receive goods or services in the future. Common examples include:
–Forfeiting balances on gift cards;
–Not claiming loyalty points or air miles; and
–Non-refundable theatre and travel tickets, where the customer foregoes amounts paid in
advance if they do not turn up.
When a vendor expects to be entitled to a breakage amount, it recognises revenue in proportion
to the expected pattern of rights that customers will exercise (i.e. by comparing the goods or
services delivered to date with those expected to be delivered overall). This increases the
transaction price allocated to the individual goods or services transferred to include revenue
from the vendor‘s estimate of unexercised rights.
When the vendor does not expect to be entitled to a breakage amount, it is recognised as
revenue when the likelihood of the customer exercising its remaining rights becomes remote. As
with the variable consideration constraint, it would only recognise breakage as revenue to the
extent it was highly probable there would not be a significant reversal of that revenue

Non-refundable upfront fees


A vendor may charge a customer a non-refundable upfront fee at (or near) contract inception,
which may be related to an activity that the vendor is required to undertake at (or near) contract
inception in order to fulfil the contract. The vendor is required to determine whether the fee
relates to the transfer of a promised good or service, in order to identify the performance
obligations within the contracts.
When the non-refundable upfront fee is not related to a performance obligation but to setup
activities or other administrative tasks, the non-refundable upfront fee is accounted for as an
advance payment for future goods or services and is therefore only recognised as revenue when
those future goods or services are provided.
In practice, non-refundable upfront fees typically relate primarily to setup activities, and not to
a performance obligation.
Licensing

323
A license establishes a customer‘s rights over the intellectual property of a vendor, such as:
–Software and technology;
–Media and entertainment (e.g. motion pictures);
–Franchises;
–Patents, trademarks, and copyrights.

A contract to transfer (provide) a license to a customer may include obligations to provide other
goods and services in addition to the promised license. Those obligations may be specified in
the contract or implied by the vendor‘s customary business practices, published policies or
specific statements. The accounting treatment depends on whether or not the license is ‗distinct‘
from those other promised goods or services.
When the license is not distinct from those other goods or services , they are accounted for
together as a single performance obligation. This would be the case, for example, when the
license forms a component of a tangible good and is integral to the good‘s functionality (for
example, a software license which requires ongoing maintenance and upgrade services in order
for it to continue to operate), or it is a license that the customer can benefit from only in
conjunction with a related service (for
example, a software hosting agreement on an internet site).
When the license is distinct from the other promised goods or services, the license is accounted
for as a separate performance obligation. Revenue is then recognised either at a point in time, or
over time, depending on whether the nature of the vendor‘s promise in transferring the license
to the customer is to provide that customer with either:
–Access to the vendor‘s intellectual property as it exists at any given time throughout the license
period (i.e. the vendor continues to be involved with its intellectual property); or
–A right to use the vendor‘s intellectual property as it exists at a point in time the license is
granted.

A vendor continues to be involved with its intellectual property by undertaking activities that
do not transfer goods or services to the customer, but instead change its intellectual property to
which the customer has rights. This applies if all of the following criteria are met:
(i) The contract requires, or the customer reasonably expects, that the vendor will undertake
activities that significantly affect the intellectual property to which the customer has rights (that
is, the intellectual property to which the customer has rights is dynamic).

Factors that may indicate that a customer could reasonably expect that a vendor will undertake
activities that will significantly affect the intellectual property include:
–The vendor‘s customary business practices;
–Published policies;
–Specific statements;
–The existence of a shared economic interest (e.g. a sales-based royalty) between the vendor and
the customer related to the intellectual property licensed to the customer.

(ii)The rights granted by the license directly expose the customer to any positive or negative
effects of the vendor‘s activities that affect the intellectual property as and when the vendor
undertakes those activities.

324
(iii)The vendor‘s activities do not transfer a good or a service to the customer as those activities
occur (that is, the activities are not accounted for as performance obligations).
When all of the above criteria are met, a vendor accounts for the license as a performance
obligation satisfied over time because the customer will simultaneously receive and benefit
from the vendor‘s performance as the performance occurs. An appropriate method is selected to
measure the vendor‘s progress toward complete satisfaction of its performance obligation to
provide access to the intellectual property.
When one or more of the criteria above are not met, the nature of the license is to transfer a right
to access intellectual property as it exists at the point at which the license is granted. Because the
intellectual property to which the customer has rights to is ‗static‘ (i.e. is not affected by
continuing involvement by the vendor), the right granted enables the customer to direct the use
of and obtain substantially all of the remaining benefits from the intellectual property in its
form at the point at which the license is granted to the customer. Therefore, the promise of a
license that transfers a right is accounted for as a performance obligation satisfied at a point in
time. The point in time cannot be before control of the license is transferred to the customer.
This means that, if the vendor provides (or otherwise makes available) to the customer an access
code that is necessary to enable the customer to access or use licensed software, the vendor
would not recognise revenue until the access code has been made available, even though the
license period could have started at an earlier date.
When determining the type of license that has been granted (intellectual property as it exists at
any point during the license period or as it exists at the point at which the license is granted),
the following factors are disregarded:
–Restrictions of time, geography, or use. This is because these restrictions define the attributes
of the promised license, rather than define whether the vendor satisfies its performance
obligation at a point in time or over time;
–Guarantees provided by the vendor that it has a valid patent to intellectual property and that it
will defend that patent from unauthorised use. A promise to defend a patent right is not a
performance obligation because it protects the value of the vendor‘s intellectual property asset
and provides the customer with assurance that the license transferred meets the related
contractual specifications.

Customer acceptance
If a customer accepts an asset, this may indicate that control over the asset has passed to the
customer. However, contractual arrangements typically include clauses which enable the
customer to require the vendor to take action if the asset does not meet its contractually agreed
upon specifications, and might allow the customer to cancel the contract.

If a vendor can demonstrate that an asset that has been transferred to a customer meets the
contractually agreed upon specifications, then customer acceptance is considered to be a
formality that is not taken into account when determining whether control over the asset has
passed to the customer. For example, if the sale of an asset is subject to it meeting certain size
and weight specifications, the vendor would typically be able to confirm whether these had
been met when the asset is delivered to the customer. However, if the vendor recognises
revenue in advance of receiving customer acceptance, the vendor is required to consider
whether there are any other performance obligations that have not yet been fulfilled.

325
If the vendor is not able to determine that the asset that has meets the contractually agreed upon
specifications, then control over the asset does not transfer to the customer until the vendor has
received the customer‘s acceptance. In addition, if products are delivered to a customer for trial
purposes, and the customer has no commitment to pay any consideration until the trial period
has ended, control of the asset does not pass to the customer until the earlier of the point at
which the customer accepts the asset or the trial period ends.

Treatment of onerous contracts


IFRS 15 Revenue from Contracts with Customers does not contain specific guidance for onerous
contracts, and instead refers to IAS 37 Provisions, Contingent Liabilities and Contingent Assets
IAS 37 does not provide any guidance on how to combine or segment contracts, while IFRS 15
provides significant guidance on these topics.
In addition IAS37 specifies that onerous contracts need to be assessed taking into consideration
economic benefits to be received. IAS37 does not define these economic benefits, including
contracts which contain variable consideration, while for those contracts IFRS 15 constrain the
amount of variable consideration that can be recognised.
When assessing onerous contracts in accordance to IAS 37, an initial question is whether that
might arise is whether the unit of account is the contract as a whole or the performance
obligations identified through the application of IFRS 15.
Because IAS 37 refers to contracts, the unit of account is the contract as a whole and not the
individual performance obligations. Consequently, a provision would not be recognised for a
contract with multiple performance obligations, in which one performance obligation was
onerous but the overall contract was not. However, it is possible that two or more contracts
would need to be combined into a single unit of account.
The contract level unit of account is consistent with IFRS 15, which clearly distinguishes
between a contract and performance obligations in a contract. However if two or more contracts
are entered into at or near the same time with the same customer (or related parties of the
customer) and they meet one of the following criteria from IFRS 15, the contracts should be
combined for the purposes of both IFRS 15 and IAS 37:

–The contracts are negotiated as a package with a single commercial objective;


–The amount of consideration to be paid in one contract depends on the price or performance of
the other contract; or
–The goods or services promised in the contracts are a single performance obligation.
An additional question, for the purposes of assessing onerous contracts in accordance with IAS
37 is whether variable payments should be subject to the variable consideration constraint in
accordance with the requirements of IFRS 15.
For this issue, there is no requirement in IAS 37 that requires the revenue recognition constraint
in IFRS 15 to be applied when a contract gives rise to variable consideration. IFRS 15 does not
apply to an assessment of onerous contracts (instead referring to IAS 37), meaning that for the
purposes of IAS 37 the definition of ‗economic benefits to be received‘ is interpreted more
widely. This means that for the purposes of accounting for an onerous contract in accordance
with IAS 37, all expected revenues are included. This is in contrast to the constraint in IFRS 15,
which permits revenue from a contract which gives rise to variable consideration to be
recognised only when it is highly probable that there will not be a subsequent reversal in the
amount of revenue which has been recognised to date.

326
DISCLOSURES
IFRS 15 Revenue from Contracts with Customers includes an overall disclosure objective, which
is for the disclosures to include sufficient information to enable users of financial statements to
understand the nature, amount, timing and uncertainty of revenue and cash flows arising from
contracts with customers.

This is accompanied by comprehensive disclosure requirements about a vendor‘s:


–Contracts with customers;
–Significant judgements, and changes in the judgements, made in applying IFRS 15 to those
contracts;
–Assets recognised in respect of costs of obtaining contracts, and in fulfilling contracts.

Consistent with the IASB‘s current Disclosure Initiative project, IFRS 15 notes specifically that
consideration is to be given to the level of detail that is necessary to satisfy the disclosure
objective, and to the emphasis to be placed on each disclosure requirement. The purpose is to
ensure that the information that users will find useful is not obscured by a large amount of
insignificant detail, with items with sufficiently different characteristics being disaggregated
and presented separately.

QUESTIONS FOR REVIEW


QUESTION 1
There has been significant divergence in practice over recognition of revenue mainly because
International Financial Reporting Standards (IFRS) have contained limited guidance in certain
areas. The International Accounting Standards Board (IASB) as a result of the joint project
with the US Financial Accounting Standards Board (FASB) has issued IFRS 15 Revenue from
Contracts with Customers. IFRS 15 sets out a five-step model, which applies to revenue earned
from a contract with a customer with limited exceptions, regardless of the type of revenue
transaction or the industry. Step one in the five-step model requires the identification of the
contract with the customer and is critical for the purpose of applying the standard. The
remaining four steps in the standard‘s revenue recognition model are irrelevant if the contract
does not fall within the scope of IFRS 15.

Required:
(i) Discuss the criteria which must be met for a contract with a customer to fall within the scope
of IFRS 15. (5 marks)
(ii) Discuss the four remaining steps which lead to revenue recognition after a contract has been
identifiedas falling within the scope of IFRS 15. (8 marks)

Solution:
(i) The definition of what constitutes a contract for the purpose of applying the standard is
critical. The definition of contract is based on the definition of a contract in the USA and is
similar to that in IAS 32
Financial Instruments: Presentation.
A contract exists when an agreement between two or more parties creates enforceable rights
and obligations between those parties. The agreement does not need to be in writing to be a
contract but the decision as to whether a contractual right or obligation is enforceable is

327
considered within the context of the relevant legal framework of a jurisdiction. Thus,whether a
contract is enforceable will vary across jurisdictions. The performance obligation could include
promises which result in a valid expectation that the entity will transfer goods or services to the
customer even though those promises are not legally enforceable.
The first criteria set out in IFRS 15 is that the parties should have approved the contract and are
committed to perform their respective obligations. It would be questionable whether that
contract is enforceable if this were not the case. In the case of oral or implied contracts, this may
be difficult but all relevant facts and circumstances should be considered in assessing the
parties‘ commitment. The parties need not always be committed to fulfilling all of the
obligations under a contract. IFRS 15 gives the example where a customer is required to
purchase a minimum quantity of goods but past experience shows that the customer does not
always do this and the other party does not enforce their contract rights.
However, there needs to be evidence that the parties are substantially committed to the
contract.

It is essential that each party‘s rights and the payment terms can be identified regarding the
goods or services to be transferred. This latter requirement is the key to determining the
transaction price.
The contract must have commercial substance before revenue can be recognised, as without this
requirement, entities might artificially inflate their revenue and it would be questionable
whether the transaction has economic consequences.
Further, it should be probable that the entity will collect the consideration due under the
contract. An assessment of a customer‘s credit risk is an important element in deciding whether
a contract has validity but customer credit risk does not affect the measurement or presentation
of revenue. The consideration may be different to the contract price because of discounts and
bonus offerings. The entity should assess the ability of the customer to pay and the customer‘s
intention to pay the consideration. If a contract with a customer does not meet these criteria, the
entity can continually re-assess the contract to determine whether it subsequently meets the
criteria.
Two or more contracts which are entered into around the same time with the same customer
may be combined and accounted for as a single contract, if they meet the specified criteria. The
standard provides detailed requirements for contract modifications. A modification may be
accounted for as a separate contract or a modification of the original contract, depending upon
the circumstances of the case.

(ii) Step one in the five-step model requires the identification of the contract with the customer.
After a contract has been determined to fall under IFRS 15, the following steps are required
before revenue can be recognised.
Step two requires the identification of the separate performance obligations in the contract. This
is often referred to as ‘unbundling‘, and is done at the beginning of a contract. The key factor in
identifying a separate performance obligation is the distinctiveness of the good or service, or a
bundle of goods or services. A good or service is distinct if the customer can benefit from the
good or service on its own or together with other readily available resources and is separately
identifiable from other elements of the contract. IFRS 15 requires a series of distinct goods or
services which are substantially the same with the same pattern of transfer, to be regarded as a
single performance obligation. A good or service, which has been delivered, may not be distinct
if it cannot be used without another good or service which has

328
not yet been delivered. Similarly, goods or services which are not distinct should be combined
with other goods or services until the entity identifies a bundle of goods or services which is
distinct. IFRS 15 provides indicators rather than criteria to determine when a good or service is
distinct within the context of the contract. This allows management to apply judgement to
determine the separate performance obligations which best reflect the economic substance of a
transaction.

Step three requires the entity to determine the transaction price, which is the amount of
consideration which an entity expects to be entitled to in exchange for the promised goods or
services. This amount excludes amounts collected on behalf of a third party, for example,
government taxes. An entity must determine the amount of consideration to which it expects to
be entitled in order to recognise revenue.
The transaction price might include variable or contingent consideration. Variable consideration
should be estimated as either the expected value or the most likely amount. Management
should use the approach which it expects will best predict the amount of consideration and
should be applied consistently throughout the contract. An entity can only include variable
consideration in the transaction price to the extent that it is highly probable that a subsequent
change in the estimated variable consideration will not result in a significant revenue reversal. If
it is not appropriate to include all of the variable consideration in the transaction price, the
entity should assess whether it should include part of the variable consideration. However, this
latter amount still has to pass the ‘revenue reversal‘ test.
Additionally, an entity should estimate the transaction price taking into account non-cash
consideration, consideration payable to the customer and the time value of money if a
significant financing component is present. The latter is not required if the time period between
the transfer of goods or services and payment is less than one year. If an entity anticipates that it
may ultimately accept an amount lower than that initially promised in the contract due to, for
example, past experience of discounts given, then revenue would be estimated at the lower
amount with the collectability of that lower amount being assessed. Subsequently, if revenue
already recognised is not collectable, impairment losses should be taken to profit or loss.

Step four requires the allocation of the transaction price to the separate performance
obligations. The allocation is based on the relative standalone selling prices of the goods or
services promised and is made at inception of the contract. It is not adjusted to reflect
subsequent changes in the standalone selling prices of those goods or services. The best
evidence of standalone selling price is the observable price of a good or service when the entity
sells that good or service separately. If that is not available, an estimate is made by using an
approach which maximises the use of observable inputs. For example, expected cost plus an
appropriate margin or the assessment of market prices for similar goods or services adjusted for
entity-specific costs and margins or in limited circumstances a residual approach. When a
contract contains more than one distinct performance obligation, an entity allocates the
transaction price to each distinct
performance obligation on the basis of the standalone selling price.

Where the transaction price includes a variable amount and discounts, consideration needs to
be given as to whether these amounts relate to all or only some of the performance obligations
in the contract. Discounts and variable consideration will typically be allocated proportionately

329
to all of the performance obligations in the contract. However, if certain conditions are met, they
can be allocated to one or more separate performance obligations.

Step five requires revenue to be recognised as each performance obligation is satisfied. An


entity satisfies a performance obligation by transferring control of a promised good or service to
the customer, which could occur over time or at a point in time. The definition of control
includes the ability to prevent others from directing the use of and obtaining the benefits from
the asset. A performance obligation is satisfied at a point in time unless it meets one of three
criteria set out in IFRS 15. Revenue is recognised in line with the pattern of transfer.

If an entity does not satisfy its performance obligation over time, it satisfies it at a point in time
and revenue will be recognised when control is passed at that point in time. Factors which may
indicate the passing of control include the present right to payment for the asset or the customer
has legal title to the asset or the entity has transferred physical possession of the asset.

QUESTION 2
Neymar enters into a 12 –month telecom plan with the local mobile operator Kasapa Plc.The
terms of the plan are as follows:
- Neymar‘s monthly fixed fee is GHC100.
- Neymar recieves a free handset at the inception of the plan.
Kasapa Plc sells the same handset for GHC300 and the same monthly prepayment plans
without handset for GHC80/month.
Required:
How should Kasapa Plc recognise the revenues from this plan in line with IFRS 15-Revenue
from Contracts with Customers ?

Solution:
Step 1: Contrcat Indentification
Neymar 1.Receives a free handset
2.Monthly payment of GHC100

Step 2: Obligation Identification


Kasapa Plc 1.Deliver the free handset
2. Provide the network service to Neymar

Step 3: Price for the transaction


Fee (GHC100 x 12) = GHC1,200
Free handset = GHC 0
Total = GHC1,200

Step 4 : Allocation of transaction price over the performance obligations.


Stand-alone selling price % Actual
GHC GHC
Handset 300 300/1,260 23.8 286
Service (GHC80 x 12) 960 960/1,260 76.2 914
1,260 100 1,200

330
Step 5: Recognise Revenue
1.When the handset is delivered to the customer
Dr. Contract Asset GHC286
Cr. Revenue GHC286
2.Recognise Service Income
Dr. Receivables GHC100
Cr. Contract Asset (286/12) GHC24
Cr. Revenue (914/12) GHC76
When Cash is received
Dr. Cash GHC100
Cr. Receivables GHC100
QUESTION 3
Thomas enters into a contract with a customer to sell an existing printing machine such that
control of the printing machine vests with the customer in two years‘ time. The contract has two
payment options. The customer can pay GHC240,000 when the contract is signed or
GHC300,000 in two years‘ time when the customer gains control of the printing machine. The
interest rate implicit in the contract is 11·8% in order to adjust for the risk involved in the delay
in payment. However, Thomas‘ incremental borrowing rate is 5%.The customer paid
GHC240,000 on 1 December 2014 when the contract was signed. (4 marks)
Required:
Discuss how the above contract should be accounted for under IFRS 15.
(The discussion should include the accounting treatment up to 30 November 2016)

Solution:
The contract contains a significant financing component because of the length of time between
when the customer pays for the asset and when Thomas transfers the asset to the customer, as
well as the prevailing interest rates in the market. A contract with a customer which has a
significant financing component should be separated into a revenue component (for the
notional cash sales price) and a loan component. Consequently, the accounting for a sale arising
from a contract which has a significant financing component should be comparable to the
accounting for a loan with the same features.
An entity should use the discount rate which would be reflected in a separate financing
transaction between the entity and its customer at contract inception. The interest rate implicit
in the transaction may be different from the rate to be used to discount the cash flows, which
should be the entity‘s incremental borrowing rate. IFRS 15 would therefore dictate that the rate
which should be used in adjusting the promised consideration is 5%, which is the entity‘s
incremental borrowing rate, and not 11·8%.

Thomas would account for the significant financing component as follows:


Recognise a contract liability for the GHC240,000 payment received on 1 December 2014 at the
contract inception:
Dr Cash GHC240,000
Cr Contract liability GHC240,000

331
During the two years from contract inception (1 December 2014) until the transfer of the
printing machine, Thomas adjusts the amount of consideration and accretes the contract
liability by recognising interest on GHC240,000 at 5% for two years.

Year to 30 November 2015


Dr Interest expense GHC12,000
Cr Contract liability GHC12,000

Contract liability would stand at GHC252,000 at 30 November 2015.

Year to 30 November 2016


Dr Interest expense GHC12,600
Cr Contract liability GHC12,600

Recognition of contract revenue on transfer of printing machine at 30 November 2016 of


GHC264,600 by debiting contract liability and crediting revenue with this amount.

QUESTION 4
Tonaldo enters into a contract on 1 December 2014 to construct a printing machine on a
customer‘s premises for a promised consideration of GHC1,500,000 with a bonus of
GHC100,000 if the machine is completed within 24 months. At the inception of the contract,
Tonaldo correctly accounts for the promised bundle of goods and services as a single
performance obligation in accordance with IFRS 15. At the inception of the contract,Tonaldo
expects the costs to be GHC800,000 and concludes that it is highly probable that a significant
reversal in the amount of cumulative revenue recognised will occur. Completion of the printing
machine is highly susceptible to factors outside of Tonaldo‘s influence, mainly issues with the
supply of components.
At 30 November 2015, Tonaldo has satisfied 65% of its performance obligation on the basis of
costs incurred to date and concludes that the variable consideration is still constrained in
accordance with IFRS 15.
However, on 4 December 2015, the contract is modified with the result that the fixed
consideration and expected costs increase by GHC110,000 and GHC60,000 respectively. The
time allowable for achieving the bonus is extended by six months with the result that Tonaldo
concludes that it is highly probable that the bonus will be achieved and that the contract still
remains a single performance obligation. Tonaldo has an accounting year end of 30 November.
(6 marks)
Required:
Discuss how the above the contract should be accounted for under IFRS 15.
(The discussion should include the accounting treatment up to 4 December 2015.
Solution:

Tonaldo accounts for the promised bundle of goods and services as a single performance
obligation satisfied over time in accordance with IFRS 15. At the inception of the contract, Tang
expects the following:
Transaction price GHC1,500,000
Expected costs GHC800,000
Expected profit (46·7%) GHC700,000

332
At contract inception, Tonaldo excludes the GHC100,000 bonus from the transaction price
because it cannot conclude that it is highly probable that a significant reversal in the amount of
cumulative revenue recognised will not occur. Completion of the printing machine is highly
susceptible to factors outside the entity‘s influence. By the end of the first year, the entity has
satisfied 65% of its performance obligation on the basis of costs incurred to date. Costs incurred
to date are therefore GHC520,000 and Tonaldo reassesses the variable consideration and
concludes that the amount is still constrained.
Therefore at 30 November 2015, the following would be recognised:

Revenue GHC975,000
Costs GHC520,000
Gross profit GHC455,000

However, on 4 December 2015, the contract is modified. As a result, the fixed consideration and
expected costs increase by GHC110,000 and GHC60,000, respectively. The total potential
consideration after the modification is GHC1,710,000 which is GHC1,610,000 fixed
consideration + GHC100,000 completion bonus. In addition, the allowable time for achieving
the bonus is extended by six months with the result that Tonaldo concludes that it is highly
probable that including the bonus in the transaction price will not result in a significant reversal
in the amount of cumulative revenue recognised in accordance with IFRS 15. Therefore the
bonus of GHC100,000 can be included in the transaction price. Tonaldo also concludes that the
contract remains a single performance obligation. Thus,Tonaldo accounts for the contract
modification as if it were part of the original contract. Therefore, Tonaldo updates its estimates
of costs and revenue as follows:
Tonaldo has satisfied 60·5% of its performance obligation (GHC520,000 actual costs incurred
compared to GHC860,000 total expected costs). The entity recognises additional revenue of
GHC59,550 [(60·5% of GHC1,710,000) – GHC975,000 revenue recognised to date] at the date of
the modification as a cumulative catch-up adjustment. As the contract amendment took place
after the year end, the additional revenue would not be treated as an adjusting event

QUESTION 5
Carsoon constructs retail vehicle outlets and enters into contracts with customers to construct
buildings on their land. The contracts have standard terms, which include penalties payable by
Carsoon if the contract is delayed, or payable by the customer, if Carsoon cannot gain access to
the construction site.

Due to poor weather, one of the projects was delayed. As a result, Carsoon faced additional
costs and contractual penalties. As Carsoon could not gain access to the construction site, the
directors decided to make a counter-claim against the customer for the penalties and additional
costs which Carsoon faced. Carsoon felt that because claims had been made against the
customer, the additional costs and penalties should not be included in contract costs but shown
as a contingent liability. Carsoon has assessed the legal basis of the claim and feels it has
enforceable rights.

In the year ended 28 February 2017, Carsoon incurred general and administrative costs of
GHC10 million, and costs relating to wasted materials of GHC5 million.

333
Additionally, during the year, Carsoon agreed to construct a storage facility on the same
customer‘s land for GHC7 million at a cost of GHC5 million. The parties agreed to modify the
contract to include the construction of the storage facility, which was completed during the
current financial year. All of the additional costs relating to the above were capitalised as assets
in the financial statements.
The directors of Carsoon wish to know how to account for the penalties, counter claim and
additional costs in accordance with IFRS 15 Revenue from Contracts with Customers.
(7 marks)

Solution:
IFRS 15 Revenue from Contracts with Customers specifies how to account for costs incurred in
fulfilling a contract which are not in the scope of another standard. Costs to fulfil a contract
which is accounted for under IFRS 15 are divided into those which give rise to an asset and
those which are expensed as incurred. Entities will recognise an asset when costs incurred to
fulfil a contract meet certain criteria, one of which is that the costs are expected to be recovered.

For costs to meet the ‗expected to be recovered‘ criterion, they need to be either explicitly
reimbursable under the contract or reflected through the pricing of the contract and recoverable
through the margin.

The penalty and additional costs attributable to the contract should be considered when they
occur and Carsoon should have included them in the total costs of the contract in the period in
which they had been notified.

As regards the counter claim for compensation, Carsoon accounts for the claim as a contract
modification in accordance with IFRS 15. The modification does not result in any additional
goods and services being provided to the customer. In addition,all of the remaining goods and
services after the modification are not distinct and form part of a single performance obligation.
Consequently, Carsoon should account for the modification by updating the transaction price
and the measure of progress towards complete satisfaction of the performance obligation.

A contract modification may exist even though the parties to the contract have a dispute about
the scope or price (or both) of the modification or the parties have approved a change in the
scope of the contract but have not yet determined the corresponding change in price. In
determining whether the rights and obligations which are created or changed by a modification
are enforceable, an entity should consider all relevant facts and circumstances including the
terms of the contract and other evidence. On the basis of information available, it is possible to
feel that the counter claim had not reached an advanced stage, so that claims submitted to the
client could not be included in total revenues.

When the contract is modified for the construction of the storage facility, an additional GHC7
million is added to the consideration which Carsoon will receive. The additional GHC7 million
reflects the stand-alone selling price of the contract modification. The construction of the
separate storage facility is a distinct performance obligation; the contract modification for the
additional storage facility would be, in effect, a new contract which does not affect the
accounting for the existing contract. Therefore the contract is a performance obligation which

334
has been satisfied as assets are only recognised in relation to satisfying future performance
obligations. General and administrative costs cannot be capitalised unless these costs are
specifically chargeable to the customer under the contract. Similarly, wasted material costs are
expensed where they are not chargeable to the customer. Therefore a total expense of GHC15
million will be charged to profit or loss and not shown as assets.

QUESTION 6
Darlatt is a public limited company with a year end of 31 August 2017. It sells wind turbines as
part of a combined contract which includes a standard two-year warranty term and
maintenance services for a ten-year period. In addition, Darlatt offers the option of a ten-year
extension to the warranty for an additional fee which is paid at the time of the initial sale. The
sales price for the combined contract is GHC3·6 million and the customer will pay an additional
fee of GHC0·8 million for the extended warranty. If sold separately, the selling price of the wind
turbine would be GHC3·2 million and the selling price of the two-year warranty and ten-year
maintenance service contract would be GHC0·9 million. The extended warranty has a separate
selling price of GHC1 million.
The directors of Darlatt would like to know how the above transactions should be accounted for
under IFRS 15 Revenue from Contracts with Customers. (8 marks)

Solution:
IFRS 15 Revenue from Contracts with Customers sets out the core principle that an entity will
recognise revenue to depict the transfer of promised goods or services to customers in an
amount which reflects the consideration to which the entity expects to be entitled in exchange
for those goods or services. This principle is delivered through a five-step model. Once the
contract with the customer has been identified, step 2 of the model identifies those elements of
the contract which should be accounted for separately. The performance obligations should be
identified at the beginning of the contract by identifying distinct goods or services in the
contract. To do so, the entity should identify all the goods and services which have been
promised. The distinct performance obligations are the units of account which determine when
and how revenue is recognised. A good or service is distinct only if the customer can benefit
from the good or service either on its own or together with other resources available to the
customer and the good or service is separately identifiable from other promises in the contract.

A customer can benefit from a good or service on its own if it can be used, consumed, or sold to
generate economic benefits.
Determining whether a good or service is distinct within the context of the contract requires
assessment of the contract terms and the intent of the parties.

Thus in the case of Darlatt, the entity is required to assess whether the deliverables it has
promised to the customer give rise to separate performance obligations. The purchase of the
wind turbine and the maintenance contract are obviously separate performance obligations.
However, the two warranties require further consideration. The nature of the warranty will
determine the accounting impact. IFRS 15 states that an entity accounts for a warranty as a
separate performance obligation if the customer has the option to purchase the warranty
separately. An entity accounts for a warranty as a cost accrual if it is not sold separately, unless
the warranty is to provide the customer with a service in addition to assurance that the product
complies with agreed specifications. The free warranty simply provides the customer with the

335
assurance that the wind turbine meets the agreed specification and thus is not a separate
performance obligation. Where the warranty provides an additional service as is the case with
the ten-year warranty, then the income will be treated as deferred revenue.

Once the separate performance obligations have been identified, then the transaction price is
allocated to them based on the relative stand-alone selling prices of the goods or services
promised. This allocation is made at contract inception and not adjusted to reflect subsequent
changes in the stand-alone selling prices of those goods or services. The best evidence of stand-
alone selling price is the observable price of a good or service when the entity sells that good or
service separately.

Therefore, the wind turbine will be allocated with (GHC3·2m/GHC4·1m x GHC3·6m), i.e.
GHC2·8 million and the maintenance contract with (GHC0·9m/GHC4·1m x GHC3·6m), i.e.
GHC0·8 million of the total revenue. Thus the maintenance contract and additional warranty
will be recognised over time and the sale of the wind turbine and free warranty will be
recognised at a point in time. Where revenue is recognised over time, a method should be used
which best reflects the pattern of transfer of goods or services to the customer. In this case, it
would appear that both of the above elements would be recognised over 10 years.

QUESTION 7
The following trial balance relates to Dramani Co as at 31 March 2016:
GHC’000 GHC’000
Revenue (note (i)) 267,900
Cost of sales 166,600
Contract asset (note (ii)) 5,000

The following notes are relevant:


(i) Revenue includes an amount of GHC16 million for a sale made on 1 April 2015. The sale
relates to a single product and includes ongoing servicing from Dramani Co for four years. The
normal selling price of the product and the servicing would be GHC18 million and GHC500,000
per annum (GHC2 million in total) respectively.

(ii) The contract asset is comprised of contract costs incurred at 31 March 2016 of GHC15
million less a payment of GHC10 million from the customer. The agreed transaction price for
the total contract is GHC30 million and the total expected costs are GHC24 million. Dramani Co
uses an input method based on costs incurred to date relative to the total expected costs to
determine the progress towards completion of its contracts.
Required:
(a) Prepare the statement of profit or loss extract for Dramani Co for the year ended 31 March
2016.
(b) Prepare the statement of financial position extract of Dramani Co as at 31 March 2016.

Solution:
a) Dramani Co – Statement of profit or loss and other comprehensive income for the year ended 31 March
2016
GHC‘000

336
Revenue (267,900 – 1,200 (w (i)) + 18,750 (w (ii))) 285,450
Cost of sales (w (iii)) (181,600)
––––––––
Gross profit 103,850

(b) Dramani Co – Statement of financial position as at 31 March 2016


GHC‘000 GHC‘000
Current assets
Contract asset (w (ii)) 8,750

Non-current liabilities
Deferred revenue (w (i)) 800
Current liabilities
Deferred revenue (w (i)) 400

Workings (figures in brackets in GHC’000)


(i) Product and servicing sale
Under IFRS 15 Revenue from Contracts with Customers, sales made which include revenue for
on-going servicing work must have part of the revenue deferred and any discount offered to
stand-alone selling prices must (normally) be allocated to each component pro rata to the stand-
alone selling prices.
The stand-alone selling price of the product and the servicing work would be GHC20 million
(GHC18 million and GHC2 million (500 x 4 years) respectively). The actual combined selling
price of GHC16 million represents a 20% discount on the stand-alone selling prices ((20,000 –
16,000)/(18,000 + 2,000)). Thus the sales revenue of GHC16 million would be allocated GHC14·4
million (18,000 x 80%) to the product and GHC1·6 million (2,000 x 80%) to the servicing. At 31
March 2016 there are three more years of servicing work, thus GHC1·2 million ((1,600 x 3
years/4 years) must be treated as deferred revenue, split GHC400,000 as a current liability and
GHC800,000 as a non-current liability.

(ii) Contract with customer


GHC‘000
Total transaction price 30,000
Total estimated costs 24,000
–––––––
Estimated total profit 6,000
–––––––
Based on an input method (cost) basis to determine the completion of the contract, it is 62·5%
complete (15,000/24,000 x 100). Thus the profit for the year to 31 March 2016 is GHC3·75 million
(6,000 x 62·5%); GHC18·75 million (15,000 + 3,750) will be recognised as revenue; and GHC15
million as cost of sales.
The contract asset will be GHC8·75 million (18,750 – 10,000) received.

(iii) Cost of sales


GHC‘000
Per trial balance 166,600
Contract costs (w (ii)) 15,000

337
181,600
QUESTION 8
IFRS 15: Revenue from Contracts with Customers specifies how and when an IFRS reporter will
recognize revenue as well as requiring such entities to provide users of financial statements
with more informative,relevant disclosures.The standard provides a single,principles based five
–step model to be applied to all contracts with customers.
Mankranso Ltd,a hotel had the following transactions during the year:
(i) On 31 March 2019,Mankranso Ltd signed a contract to supply 50,000 units of food packs
at an agreed price of GHC10 per unit.On the same day,30,000 units were delivered at
that date,with the remainder delivered on 1 June 2019.It was agreed that the
customer would have extended credit terms of 12 months from the date of
delivery.Mankranso Ltd‘s cost of capital is 10%.
(ii) During the year ended 31March 2019,Mankranso Ltd received payment in advance for
the supply of 2,000 hotel room-nights to customers at GHC100 per room per
night.Only 400 of these had been occupied by 31 March 2019.The amounts paid by
the customers are non-refundable unless the company fails to provide the agreed
accommodation.
Assume Mankranso Ltd has decided to adopt IFRS 15 for the year ended 31 March 2019.
Required:
In each scenario above,calculate the amount of revenue to be recognized in the financial
statements of Mankranso Ltd for the year ended 31 March 2019.Justify the correct accounting
treatment for each transaction. (6 marks)

SOLUTION 8
The contract to supply is not sufficient to recognise revenue. It is necessary that control of
the goods have actually transferred to the customer. This is the case for 30,000 units.
The deferred payment does not prevent revenue from being recognised, but the
consideration needs to be measured at the fair value, on the transaction date, of the amount
receivable. The fair value needs to reflect a discount allowing for the time value of money,
as a result of the extended credit period.
The discount rate will be 10%, Mankranso‘s cost of capital. Hence revenue will be
recognised as follows: 30,000 units * GH¢10 * 1/1.10 = GH¢272,727.
The discount will be recognised as finance income as time passes, on a time-apportioned
basis. As the sale took place on 31 March 2019, no time has yet passed to trigger the
recognition of finance income.
Journal: GH¢ GH¢
Dr trade receivables 272,727
Cr Revenue 272,727
(recognition of revenue and trade receivables at fair value of consideration receivable)
Initial revenue recognition: 2 marks
Treatment of initial recognition : 1 mark
(ii) Again, the same principles apply. Revenue is recognised when control of the goods or
services are transferred to the customer.

Here, cash was received in advance. Nevertheless, revenue is only recognised when the
service is delivered to the customer. Any excess cash retained is recognised as deferred

338
income, a liability. If the cash is non-refundable, this does not change the timing of
recognition of revenue. However, if the customer‘s right to the service expires, and the
customer has no right to a refund, the revenue should then be recognised.
Total cash received in year ended 31 March 2019: 2000 * GH¢100 = GH¢200,000
Total room nights provided 400
Revenue recognised = 400 * GH¢100 = GH¢40,000
Deferred revenue = 200,000 – 40,000 = 160,000
Journal: GH¢ 000 GH¢ 000
Dr Cash 200
Cr Revenue 40
Cr Deferred revenue 160
(recognition of revenue, deferred revenue and cash received)

QUESTION 9
Ejura Ltd (Ejura) is a manufacturing and retail company which prepares financial statements in
accordance with International Financial Reporting Standards (IFRS) up to 31 December each
year.
In order to generate or improve sales on one of its older products,Ejura offered a promotion
named ‗something for free‘.The promotion included free maintenance services for the first two
years.On 1 October 2019,under the promotional offer,Ejura sold goods to a supermarket chain
for GHC4.4million.A two-year maintenance contract would normally be sold for
GHC0.5million,and the list price of the product would normally be GHC5 million.The
transaction has been included in revenue at GHC4.4million.
Required:
In accordance with IFRS 15:Revenue from Contracts with Customers,justify the appropriate
accounting treatment for the above transaction in the financial statements of Ejura for the year
ended 31 December 2019. (7 marks)
ICAG May,2020
SOLUTION 9
Five (5) point criteria for recognition of revenue
Step 1: Identify contract with the customer
Step 2: Identify the performance obligations within the contract

Step 3: Determine the transaction price – the transaction price is GH¢4.4 million
Step 4: Allocate the transaction price among the performance obligations within the contract

ilable, use expected cost plus %

In this case, the scenario provides the standalone selling prices and hence, these shall be used to
allocate the price. Allocation of transaction price to: Page 15 of 25
Sale of products (GH¢5 million / GH¢5 million + GH¢0.5 million x GH¢4.4 million = GH¢4
Maintenance: GH¢0.5 million/ GH¢5 million + GH¢0.5 million x GH¢4.4 million =
Step 5: Recognise the revenue over time or at point in time

339
the product would be recognize during the contract period as control over the
product is transferred to the customer.

Therefore, for the current period, 3/24 months would be recognised as revenue, and the
remainder would be deferred.

QUESTION 10
IFRS 15: Revenue from Contracts with Customers specifies how and when an IFRS reporter will
recognise revenue as well as requiring such entities to provide users of financial statements with
more informative, relevant disclosures. The standard provides a single, principles based five-step
model to be applied to all contracts with customers. Mankranso Ltd, a hotel had the following
transactions during the year:

i) On 31 March 2019, Mankranso Ltd signed a contract to supply 50,000 units of food packs at
an agreed price of GH¢10 per unit. On the same day, 30,000 units were delivered at that date,
with the remainder delivered on 1 June 2019. It was agreed that the customer would have
extended credit terms of 12 months from the date of delivery. Mankranso Ltd‟s cost of capital is
10%. (3 marks)

ii) During the year ended 31 March 2019, Mankranso Ltd received payment in advance for the
supply of 2,000 hotel room-nights to customers at GH¢100 per room per night. Only 400 of these
had been occupied by 31 March 2019. The amounts paid by the customers are non-refundable
unless the company fails to provide the agreed accommodation. (3 marks) Assume
Mankranso Ltd has decided to adopt IFRS 15 for year ended 31 March 2019.

Required: In each scenario above, calculate the amount of revenue to be recognised in the
financial statements of Mankranso Ltd for year ended 31 March 2019. Justify the correct
accounting treatment for each transaction.

SOLUTION
(i) The contract to supply is not sufficient to recognise revenue. It is necessary that control of the
goods have actually transferred to the customer. This is the case for 30,000 units. The deferred
payment does not prevent revenue from being recognised, but the consideration needs to be
measured at the fair value, on the transaction date, of the amount receivable. The fair value needs
to reflect a discount allowing for the time value of money, as a result of the extended credit
period. The discount rate will be 10%, Mankranso‟s cost of capital. Hence revenue will be
recognised as follows: 30,000 units * GH¢10 * 1/1.10 = GH¢272,727.

The discount will be recognised as finance income as time passes, on a timeapportioned basis.
As the sale took place on 31 March 2019, no time has yet passed to trigger the recognition of
finance income. Journal: GH¢ GH¢
Dr trade receivables 272,727
Cr Revenue 272,727
(recognition of revenue and trade receivables at fair value of consideration receivable)
(ii) Again, the same principles apply. Revenue is recognised when control of the goods or
services are transferred to the customer.

340
Here, cash was received in advance. Nevertheless, revenue is only recognised when the service is
delivered to the customer. Any excess cash retained is recognised as deferred income, a liability.
If the cash is non-refundable, this does not change the timing of recognition of revenue.
However, if the customer‟s right to the service expires, and the customer has no right to a refund,
the revenue should then be recognised.

Total cash received in year ended 31 March 2019: 2000 * GH¢100 = GH¢200,000 Total room
nights provided 400 Revenue recognised = 400 * GH¢100 = GH¢40,000
Deferred revenue = 200,000 – 40,000 = 160,000

Journal: GH¢ 000 GH¢ 000


Dr Cash 200
Cr Revenue 40
Cr Deferred revenue 160
(recognition of revenue, deferred revenue and cash received)

LEASE (IFRS 16)


IFRS 16 defines a lease as 'A contract, or part of a contract, that conveys the right to use an asset
for a period of time in exchange for consideration'. In order for such a contract to exist the user
of the asset needs to have the right to:
 Obtain substantially all of the economic benefits from the use of the asset.
 The right to direct the use of the asset.

AN ‘IDENTIFIED ASSET’
One essential feature of a lease is that there is an ‗identified asset‘. This normally takes place
through the asset being specified in a contract, or part of a contract. For the asset to be
‗identified‘ the supplier of the asset must not have the right to substitute the asset for an
alternative asset throughout its period of use. The fact that the supplier of the asset has the right
or the obligation to substitute the asset when a repair is necessary does not preclude the asset
from being an ‗identified asset‘.
Example – identified assets
Under a contract between a local government authority (L) and a private sector provider (P), P
provides L with 20 trucks to be used for refuse collection on behalf of L for a six-year period.
The trucks, which are owned by P, are specified in the contract. L determines how they are used
in the refuse collection process. When the trucks are not in use, they are kept at L‘s premises. L
can use the trucks for another purposes if it so chooses. If a particular truck needs to be serviced
or repaired, P is required to substitute a truck of the same type. Otherwise, and other than on
default by L, P cannot retrieve the trucks during the six-year period.

341
Conclusion: The contract is a lease. L has the right to use the 20 trucks for six years which are
identified and explicitly specified in the contract. Once delivered to L, the trucks can be
substituted only when they need to be serviced or repaired.
THE RIGHT TO DIRECT THE USE OF THE ASSET
IFRS 16 states that a customer has the right to direct the use of an identified asset if either:
 The customer has the right to direct how and for what purpose the asset is used
throughout its period of use, or
 The relevant decisions about use are pre-determined and the customer has the right to
operate the asset throughout the period of use without the supplier having the right to
change these operating instructions.
Example – the right to direct the use of an asset
A customer (C) enters into a contract with a road haulier (H) for the transportation of goods
from London to Edinburgh on a specified truck. The truck is explicitly specified in the contract
and H does not have substitution rights. The goods will occupy substantially all of the capacity
of the truck. The contract specifies the goods to be transported on the truck and the dates of
pickup and delivery.
H operates and maintains the truck and is responsible for the safe delivery of the goods. C is
prohibited from hiring another haulier to transport the goods or operating the truck itself.
Conclusion: This contract does not contain a lease.
There is an identified asset. The truck is explicitly specified in the contract and H does not have
the right to substitute that specified truck.
C does have the right to obtain substantially all of the economic benefits from use of the truck
over the contract period. Its goods will occupy substantially all of the capacity of the truck,
thereby preventing other parties from obtaining economic benefits from use of the truck.
However, C does not have the right to control the use of the truck because C does not have the
right to direct its use. C does not have the right to direct how and for what purpose the truck is
used. How and for what purpose the truck will be used (i.e. the transportation of specified
goods from London to Edinburgh within a specified timeframe) is predetermined in the
contract. C has the same rights regarding the use of the truck as if it were one of many
customers transporting goods using the truck.
ACCOUNTING FOR LEASES
With a very few exceptions IFRS 16 basically abolishes the distinction between an operating
lease and a finance lease in the financial statements of lessees. Lessees will recognise a right of
use asset and an associated liability at the inception of the lease.
IFRS 16 basically requires that the ‗right of use asset‘ and the lease liability should initially be
measured at the present value of the minimum lease payments. The discount rate used to
determine present value should be the rate of interest implicit in the lease.
Recording the asset
The ‗right of use asset‘ would include the following amounts, where relevant:
 Any payments made to the lessor at, or before, the commencement date of the lease, less
any lease incentives received.
 Any initial direct costs incurred by the lessee.
 An estimate of any costs to be incurred by the lessee in dismantling and removing the
underlying asset, or restoring the site on which it is located (unless the costs are incurred
to produce inventories, in which case they would be accounted for in accordance with
IAS 2 – Inventories). Costs of this nature are recognised only when an entity incurs an

342
obligation for them. IAS 37 – Provisions, Contingent Liabilities and Contingent Assets would
be applied to ascertain if an obligation existed.

Depreciation
The right of use asset is subsequently depreciated. Depreciation is over the shorter of the useful
life of the asset and the lease term, unless the title to the asset transfers at the end of the lease
term, in which case depreciation is over the useful life.
Lease liability
The lease liability is effectively treated as a financial liability which is measured at amortised
cost, using the rate of interest implicit in the lease as the effective interest rate.
Example 1 – accounting for leases
A lessee enters into a 20-year lease of one floor of a building, with an option to extend for a
further five years. Lease payments are GHC80,000 per year during the initial term and
GHC100,000 per year during the optional period, all payable at the end of each year. To obtain
the lease, the lessee incurred initial direct costs of GHC25,000.
At the commencement date, the lessee concluded that it is not reasonably certain to exercise the
option to extend the lease and, therefore, determined that the lease term is 20 years. The interest
rate implicit in the lease is 6% per annum. The present value of the lease payments is
GHC917,600.
At the commencement date, the lessee incurs the initial direct costs and measures the lease
liability GHC917,600.
Required:
Show how this should be accounted for

Solution:
The carrying amount of the right of use asset after these entries is GHC942,600 (GHC917,600 +
GHC25,000) and consequently the annual depreciation charge will be GHC47,130 (GHC942,600
x 1/20).
The lease liability will be measured using amortised cost principles. In order to help us with the
example in the following section, we will measure the lease liability up to and including the end
of year two. This is done in the following table:
Balance Finance Balance
b/fwd cost (6%) Rental c/fwd
Year GHC GHC GHC GHC
1 917,600 55,056 (80,000) 892,656
2 892,656 53,559 (80,000) 866,215

At the end of year one, the carrying amount of the right of use asset will be GHC895,470
(GHC942,600 less GHC47,130 depreciation).
The interest cost of GHC55,056 will be taken to the statement of profit or loss as a finance cost.
The total lease liability at the end of year one will be GHC892,656. As the lease is being paid off
over 20 years, some of this liability will be paid off within a year and should therefore be
classed as a current liability.
To find this figure, we look at the remaining balance following the payment in year two. Here,
we can see that the remaining balance is GHC866,215. This will represent the non-current

343
liability, being the amount of the GHC892,656 which will still be outstanding in over a year. The
current liability element is therefore GHC26,441. This represents the GHC80,000 paid in year
two less year two‘s finance costs of GHC53,559 (or GHC892,656-GHC866,215).
Example 2 (where rentals are paid in advance)
Nacee Records leased a set of musical instrument from Lynx Entertainment on 1 January 2010.
The cash price of the instruments was GHC10,000 and the annual rental payment is GHC3,000
payable in advance.
The initial period of the lease is four years and the implicit interest rate in the lease is 13.7%.
The set of instruments are expected to have an economic useful life of four years, and
depreciation is to be charged on this basis on cost (with nil residual value).
Nacee Records‘s accounting period ends on 31 December and finance charge is accrued at 31
December.
Required:
(i) Prepare a lease schedule showing the allocation of the finance charge over the four year
period.

(ii) Show the extracts of the Income Statement and the Statement of Financial Position in respect
of the lease over all relevant years.

Solution
Schedule of Lease payment and finance charge
Year Liability Lease Payment Liability Finance charge Liability
at start during @ 13.7% at end @ end
the year
GHC GHC GHC GHC GHC
2010 10,000 3,000 7,000 959 7,959
2011 7,959 3,000 4,959 679 5,638
2012 5,638 3,000 2,638 362 3,000
2013 3,000 3,000

Income Statement Extracts


2010 2011 2012 2013
Finance charge 959 679 361 -
Annual depreciation (10,000/4) 2,500 2,500 2,500 2,500

Statement of financial Position (Extracts)


2010 2011 2012 2013
Current Liability
Obligation under finance lease: 3,000 3,000 3,000 -
Non-Current Liability
Obligation under finance lease: 4,959 2,638 - -
Non-Current Assets
Leasehold Musical Instruments 10,000 10,000 10,000 10,000
Depreciation (2,500) (5,000) (7,500) (10,000)
7,500 5,000 2,500 -
SUM OF THE DIGITS METHOD

344
This method approximates the interest charge for each period by weighting the period in
reverse order so that the most interest is charged in earlier periods.

Example:
Smith Limited entered into a six-year finance lease for an item of plant on 1 August 2007. The
agreement requires Smith Limited to pay a deposit of GHC80,000 to be followed by five equal
annual installments of GHC120,000 on 1 August in each subsequent year. The purchase price of
the asset if purchased outright would be GHC620,000. Smith Limited has just recently paid the
insurance bill for the machine.
Smith Limited uses the sum of digits to allocate the finance charge for this lease.
Required:
Account for the above transaction in the books of Smith.

Solution:
GHC
Total payments (GHC80,000 + (5 X GHC120,000) 680,000
Less fair value of asset (620,000)
Finance charge 60,000
Sum of digits allocation = 5+4+3+2+1 = 15 (lease is payable in advance)
Year ended 31 July 2008 interest 5/15 * 60,000 = 20,000
Year ended B/f Payment Capital Interest c/f
GHC GHC GHC GHC GHC
31 July 2008 620,000 (80,000) 540,000 20,000 (5/15) 560,000
31 July 2009 560,000 (120,000) 440,000 16,000 (4/15) 456,000
31 July 2010 456,000 (120,000) 336,000 12,000 (3/15) 348,000
31 July 2011 348,000 (120,000) 228,000 8,000 (2/15) 236,000
31 July 2012 236,000 (120,000) 116,000 4,000 (1/15) 120,000
31 July 2013 120,000 (120,000)
Total liability at 31 July 2008 = GHC560,000
Non-current liability = GHC440,000
Current liability = GHC120,000 (balance)

A simplified approach for short-term or low-value leases


A short-term lease is a lease that, at the date of commencement, has a term of 12 months or less.
A lease that contains a purchase option cannot be a short-term lease. Lessees can elect to treat
short-term leases by recognising the lease rentals as an expense over the lease term rather than
recognising a ‗right of use asset‘ and a lease liability. The election needs to be made for relevant
leased assets on a ‗class-by-class‘ basis. A similar election – on a lease-by-lease basis – can be
made in respect of ‗low value assets‘.
The assessment of whether an underlying asset is of low value is performed on an absolute
basis. Leases of low-value assets qualify for the simplified accounting treatment explained
above regardless of whether those leases are material to the lessee. The assessment is not
affected by the size, nature or circumstances of the lessee. Accordingly, different lessees are
expected to reach the same conclusions about whether a particular underlying asset is of low
value.
An underlying asset can be of low value only if:

345
(a) The lessee can benefit from use of the underlying asset on its own or together with other
resources that are readily available to the lessee; and
(b) The underlying asset is not highly dependent on, or highly interrelated with, other assets.
A lease of an underlying asset does not qualify as a lease of a low-value asset if the nature of the
asset is such that, when new, the asset is typically not of low value. For example, leases of cars
would not qualify as leases of low-value assets because a new car would typically not be of low
value.
Examples of low-value underlying assets can include tablet and personal computers, small
items of office furniture and telephones.
SALE AND LEASEBACK TRANSACTIONS
Introduction
The treatment of sale and leaseback transactions depends on whether or not the ‗sale‘
constitutes the satisfaction of a relevant performance obligation under IFRS 15 – Revenue from
Contracts with Customers. The relevant performance obligation would be the effective ‗transfer‘
of the asset to the lessor by the previous owner (now the lessee).
Transaction constituting a sale
If the transaction does constitute a ‗sale‘ under IFRS 15 then the treatment is as follows:
 the seller-lessee shall recognise only the amount of any gain or loss that relates to the
rights transferred to the buyer-lessor.
 The buyer-lessor shall account for the purchase of the asset applying applicable
Standards, and for the lease applying the lessor accounting requirements in IFRS 16
(these being essentially unchanged from the predecessor standard).

If the fair value of the consideration for the sale of an asset does not equal the fair value of the
asset, or if the payments for the lease are not at market rates, an entity shall make the following
adjustments to measure the sale proceeds at fair value:
 Any below-market terms shall be accounted for as a prepayment of lease payments; and
 Any above-market terms shall be accounted for as additional financing provided by the
buyer-lessor to the seller-lessee.

Example – sale and leaseback


Entity X sells a building to entity Y for cash of GHC5 million. Immediately before the
transaction, the carrying amount of the building in the financial statements of entity X was
GHC3.5 million. At the same time, X enters into a contract with Y for the right to use the
building for 20 years, with annual payments of GHC200,000 payable at the end of each year.
The terms and conditions of the transaction are such that the transfer of the building by X
satisfies the requirements for determining when a performance obligation is satisfied in IFRS
15, Revenue from Contracts with Customers. Accordingly, X and Y account for the transaction as a
sale and leaseback.
The fair value of the building at the date of sale is GHC4.5 million. Because the consideration for
the sale of the building is not at fair value, X and Y make adjustments to measure the sale
proceeds at fair value. The amount of the excess sale price of GHC500,000 (GHC5 million –
GHC4.5 million) is recognised as additional financing provided by Y to X.
The annual interest rate implicit in the lease is 5%. The present value of the annual payments
(20 payments of GHC200,000, discounted at 5%) amounts to GHC2,492,400, of which
GHC500,000 relates to the additional financing and GHC1,992,400 (GHC2,492,200 –

346
GHC500,000) relates to the lease (as adjusted for the fair value difference already identified).
The annual payment that would be required to be made 20 times in arrears to repay additional
financing of GHC500,000 when the rate of interest is 5% per annum would be GHC40,122
(GHC500,000/12.462 (the cumulative discount factor for 5% for 20 years)). Therefore the
residual would be regarded as a ‗lease rental‘ at an amount of GHC159,878 (GHC200,000 –
GHC40,122).
Given the IFRS 15 treatment as a ‗sale‘ B would almost certainly regard the lease of the building
as an operating lease. This means that B would recognise the ‗lease rentals‘ of GHC159,878 as
income.
TRANSACTION NOT CONSTITUTING A ‘SALE’
In these circumstances the seller does not ‗transfer‘ the asset and continues to recognise it,
without adjustment. The ‗sales proceeds‘ are recognised as a financial liability and accounted
for by applying IFRS 9, Financial Instruments. In the same circumstances, the buyer recognizes a
financial asset equal to the ‗sales proceeds‘.
A company sells an item of plant on 1 January,2014 for GHC50 million,its fair value.The plant
had a book value of GHC 40 million at the date of the sale.The company has entered into an
agreement lo leaseback at a cost of GHC14 million per annum for the next five years,payable
annually in arrears.Depreciation is charged on plant at a rate of 20% per annum,on the reducing
balance basis.No depreciation has yet been recorded for the current year.
Required:
Prepare extracts from the income statement for the year ended 31 December,2014-2018 and a
statement of financial position as at that date assuming(the interest rate implicit in the lease is
12% per annum).

Solution:
Profit on Disposal = Proceeds – Net Book Value
= 50 million – 40 million
= GH10 million (deferred income)
LEASE SCHEDULE
Year Obligation Finance Charge Obligation Rental (Installment) Outstanding
@ start @ 12% during year payment @ year end
GHC000 GHC000 GHC000 GHC000 GHC000
2014 50,000 6,000 56,000 (14,000) 42,000
2015 42,000 5,040 47,040 (14,000) 33,040
2016 33,040 3,965 37,005 (14,000) 23,005
2017 23,005 2,761 25,766 (14,000) 11,766
2018 11,766 2,234 14,000 (14,000) -

Income Statement for the year ended 31st December…


2014 2015 2016 2017 2018
GHC GHC GHC GHC GHC
Depreciation (20%x50m) 10,000 8,000 6,400 5,120 2,048
Finance Charge 6,000 5,040 3,965 2,761 2,234

Statement of Financial Position as at 31st December….


2014 2015 2016 2017 2018
GHC GHC GHC GHC GHC

347
Non-Current Assets:
Leased Plant 50,000 50,000 50,000 50,000 50,000
Less Accum. Depn. 10,000 18,000 24,400 29,520 31,568
NBV 40,000 32,000 25,600 20,480 18,432
Non-Current Liabilities:
Deferred Income 6,000 4,000 2,000
Finance Lease Obligation 33,040 23,005 11,766 - -
Current Liabilities:
Deferred Income 2,000 2,000 2,000 2,000
Finance Lease Obligation 8.960 10,035 11,239 11,766 -
Summary
The requirements of IFRS 16 will have significant impacts on key accounting ratios of lessees.
The greater recognition of leased assets and lease liabilities on the statement of financial
position will reduce return on capital employed and increase gearing. Initial measures of profit
are likely to be reduced, as in the early years of a lease the combination of depreciation of the
right of use asset and the finance charge associated with the lease liability will exceed the lease
rentals (normally charged on a straight-line basis).

QUESTION 2
On 1 August 2018, Asawase Ltd entered into an agreement to acquire a motor vehicle. The terms of
the agreement were that the vehicle would be leased for 5 years from the date of inception, subject to
a deposit of GH¢19,972 and 5 annual payments of GH¢6,500 in advance, commencing on 1 August
2018. The fair value of the vehicle and the present value of the lease payments were GH¢48,000 at
inception. The interest rate implicit in the lease is 8%.
Required:
In accordance with IFRS 16: Leases, show with appropriate calculations, the accounting entries
required to record the transaction above in the financial statements for the year ended 31 July 2019.
(7 marks)
SOLUTION 2
Workings
Initial recognition & measurement:
The asset is recognized at: GH¢48,000
The lease obligation is initially recognized at GH¢48,000 – 19,972 – 6,500) GH¢21,528
Journal:
Dr Vehicles GH¢48,000
Cr Lease obligation GH¢21,528
Cr Cash (upfront payments: 19,972 + 6,500) GH¢26,472
Subsequent measurement:
Finance cost for year ended 31 July 2019 (21,528 * 8%) GH¢1,722
Depreciation of leased asset (48,000 / 5 years) GH¢9,600
Journal:
Dr Profit or loss (finance costs) GH¢1,722
Cr Lease obligation GH¢1,722
Dr Profit or loss (depreciation) GH¢9,600
Cr Leasehold asset accumulated depreciation) GH¢9,600

Closing balance on lease obligation (21,528 + 1,722) GH¢23,250


Presented as current liability (full payment as it is in advance, due 1 August 2019) GH¢6,500

348
Presented as non-current liability GH¢16,750

Extracts from financial statements for year ended 31 July 2019:


Statement of Profit or Loss for year ended 31 July 2019: GH¢
Operating costs (depreciation) 9,600
Finance costs 1,722

Statement of Financial Position as at 31 July 2019: GH¢


Non-current assets:
Leasehold building (48,000 – 9,600) 38,400
Non-current liabilities:
Lease obligation 16,750
Current liabilities:
Lease obligation 6,500

Example: Sale and leaseback (at fair value)


X plc sells an asset and leases it back. The transfer qualifies as a sale according to IFRS 15
criteria. Details of the asset: Carrying amount = GH¢1,000,000 Sale proceeds = GH¢1,300,000
Fair value = GH¢1,300,000 Terms of the lease: 20 annual payments of GH¢83,951.48 Interest
rate implicit in the lease = 5% (The 20 period, 5% annuity factor is 12.16)
Example: Sale (not at fair value) and leaseback
X plc sells an asset and leases it back. The transfer qualifies as a sale according to IFRS 15
criteria. Details of the asset: Carrying amount of = GH¢1,000,000 Sale proceeds =
GH¢1,500,000 Fair value = GH¢1,300,000 Terms of the lease: 20 annual payments of
GH¢100,000 Interest rate implicit in the lease = 5% (The 20 period, 5% annuity factor is
12.4622) Notes: The sale proceeds of GH¢1,500,000 comprise GH¢1,300,000 for the asset and
additional finance of GH¢200,000.
Example: Sale (not at fair value) and leaseback
X plc sells an asset and leases it back. The transfer qualifies as a sale according to IFRS 15
criteria. Details of the asset: Carrying amount of = GH¢1,000,000 Sale proceeds =
GH¢1,200,000 (GH¢100,000 less than fair value) Fair value = GH¢1,300,000 Terms of the lease:
20 annual payments of GH¢75,927 Interest rate implicit in the lease = 5% (The 20 period, 5%
annuity factor is 12.4622) The present value of lease payments is GH¢946,221 Notes: The sale
proceeds of GH¢1,200,000 are GH¢100,000 less than the fair value of the asset. In effect, this is
an extra amount being paid by X Plc in addition to the annual payments in order to obtain the
right-of-use asset. The lease liability proper is made up of the present value of the lease
payments plus the shortfall.

Q5
On 1 August 2018, Asawase Ltd entered into an agreement to acquire a motor vehicle. The terms
of the agreement were that the vehicle would be leased for 5 years from the date of inception,
subject to a deposit of GH¢19,972 and 5 annual payments of GH¢6,500 in advance, commencing

349
on 1 August 2018. The fair value of the vehicle and the present value of the lease payments were
GH¢48,000 at inception. The interest rate implicit in the lease is 8%.

Required: In accordance with IFRS 16: Leases, show with appropriate calculations, the
accounting entries required to record the transaction above in the financial statements for the
year ended 31 July 2019. (7 marks)
SOLUTION
Workings Initial recognition & measurement: The asset is recognized at: GH¢48,000
The lease obligation is initially recognized at GH¢48,000 – 19,972 – 6,500) GH¢21,528

Journal: Dr Vehicles GH¢48,000


Cr Lease obligation GH¢21,528 Cr Cash (upfront payments: 19,972 + 6,500) GH¢26,472

Subsequent measurement: Finance cost for year ended 31 July 2019 (21,528 * 8%) GH¢1,722
Depreciation of leased asset (48,000 / 5 years) GH¢9,600

Journal: Dr Profit or loss (finance costs) GH¢1,722


Cr Lease obligation GH¢1,722

Dr Profit or loss (depreciation) GH¢9,600


Cr Leasehold asset accumulated depreciation) GH¢9,600

350
GHANA STOCK EXCHANGE REQUIREMENTS
Stock Exchange Introduction
This is where a company which already has shares in issue, wants to obtain a listing (quotation)
in a recognized stock exchange. In a stock exchange introduction the company will not issue
new shares but will obtain a facility to have the existing shares traded in the stock exchange.
This can be used by the existing shareholders as an ‗exit rate‘ where the shareholders can
convert their paper wealth (share certificate) in to cash.
GUIDELINES AND REQUIREMENTS FOR LISTING ON THE GHANA STOCK
EXCHANGE
The Ghana Stock Exchange (GSE) was established in July 1989 as a private company limited by
guarantee under the Companies Code of 1963. The GSE provides a facility for the buying and
selling of stocks, bonds, shares, securities, etc. A company is listed when its securities are
approved by the GSE to be bought and sold on the stock exchange. The Council of Ghana Stock
Exchange decides whether or not to approve an application for the listing of securities on the
exchange.
Steps Toward Listing
 To become listed on the GSE, the shareholders of the company must first meet and agree
to change their private company status to a public liability company. The company, with
the help of lawyers or other legal counsel, then prepares new company regulations that
conform to both the Companies Code of 1963 and the requirements of the GSE.
 The company discusses the new regulations with the GSE. When found satisfactory, the
regulations are forwarded to the registrar to be filed. The company prepares its offer
documents through a team of advisers consisting of brokers, lawyers, accountants and
other professionals.
 The advisers discuss the draft reports from the meetings. The shareholders make key
decisions such as share prices and the number of shares to be sold. The company then
submits an application to list on the GSE, and the draft offer prospectus is also
submitted to the Securities and Exchange Commission for approval.
Once the application is approved, a date is set for the launch of the public offer, in which the
GSE discusses details of doing business with the company. If the offer is agreed to, the company
agrees on a date for the commencement of trading its shares on the floor of the GSE.

REQUIRED INFORMATION
 The GSE requires the following information from a company applying to be listed:
company background, capitalization and share distribution (authorized and issued
capital, distribution of shareholding, etc.), names of directors and key management, list
of debts and related details (financing arrangements, interest rates, maturity dates, etc.),
company investments and properties, name(s) of competition and industry, profit and
loss record for up to three years, any pending legal actions, and financial records such as
dividend records and fiscal year end reports.
Capital Requirements
 The GSE has three listing classes: The First Official List (FOL), Second Official List (SOL)
and Third Official List (TOL). What list a company is on depends on its capital. A
company must have a minimum stated capital of at least 1 million Ghana cedis to be on
the FOL, and 500,000 Ghana cedis for the SOL.

351
Other Information
 Minimum public float, or shares in the hands of public investors, cannot be less that 25
percent of shares issued by the company. Also, payment of shares must be fully paid.
The cost of listing on the GSE varies, depending on market capitalization and whether
the company is a member. The fees included are application, listing and annual fees.
ADVANTAGES FOR A COMPANY HAVING LISTED STOCK
In addition to the prestige a company gets when their stock is listed on a stock exchange, other
advantages for the company include:
 Being able to raise additional funds through the issuance of more stock
 Companies can offer securities in the acquisition of other companies
 Stock and stock options programs can be offered to potential employees, making the
company attractive to top talent
 Companies have additional leverage when obtaining loans from financial institutions
 Market exposure – having a company‘s stock listed on an exchange could attract the
attention of mutual and hedge funds, market makers and institutional traders
 Indirect advertising – the filing and registration fee for most major exchanges includes a
form of complimentary advertising. The company‘s stock will be associated with the
exchange their stock is traded on
 Brand equity – having a listing on a stock exchange also affords the company increased
credibility with the public, having the company indirectly endorsed through having
their stock traded on the exchange.

DISADVANTAGES OF LISTING
Accountability and scrutiny: Public companies are public property. As such they are expected
to comply with the rules of the markets they populate. Companies on AIM have to use the
services of a nominated advisor , a firm or company which has been approved by the Ghana
Stock Exchange, who effectively acts as the regulator of the business, managing its listing and
ensuring its ongoing compliance.
Undervaluation risk: Issuing shares is not only dilutive but shares can also lack liquidity. This
can undermine fundraising and acquisition activity, because there is a lack of demand for the
shares. In addition, a lack of demand normally translates into a low share price, so the use of
shares as an acquisition currency may also lose its appeal. On the public markets, companies‘
share prices are not only affected by their own performance, but by the performance of the
market and the economy as a whole.
Cost:The amount of management time and the significant costs associated with a flotation and
ongoing listing should never be underestimated. From the process of flotation itself, which can
take many months, to the time-consuming administration of regular and constant
announcements (interim and final financial results, director dealings in shares, trading updates
etc.) there is a lot of activity to manage.

352
CONCEPTUAL AND REGULATORY
FRAMEWORK OF FINANCIAL REPORTING.
The International Accounting Standards Board (IASB)
The IASB is responsible for developing international accounting standards.
The IASB consists of 14 members, all with technical expertise in accounting, who are appointed
by the Trustees. Each IASB member is appointed for a five-year term, which might be renewed
once for a further five years. Each IASB member has one vote, and approval of eight members is
required for the publication of: „
 an exposure draft „
 a revised International Accounting Standard (IAS) „
 an International Financial Reporting Standard (IFRS) „
 a final Interpretation of the International Financial Reporting Interpretations Committee
(IFRIC).
The IASB has full responsibility for all IASB technical matters, including the issue of IFRSs and
revised IASs, and has full discretion over the technical agenda of the IASB.
The International Financial Reporting Interpretations Committee (IFRIC)
The role of IFRIC is to issue rapid guidance where there are differing possible interpretations of
an international accounting standard.
Its role is therefore to: „
 interpret international accounting standards (IASs and IFRSs) „
 issue timely guidance on issues not covered by an IAS or IFRS, within the context of the
IASB Framework „
 publish draft Interpretations for public comment. After studying responses to the draft
Interpretation, it will obtain IASB approval for a final (published) Interpretation.
Standards Advisory Council (SAC)
The Standards Advisory Council (SAC) provides a forum through which the IASB is able to
gather opinions and advice from different countries and industries. The SAC consists of experts
from different countries and different business sectors, who offer advice to the IASB

THE IFRS INTERPRETATIONS COMMITTEE (IFRSIC)


The role of IFRSIC is to issue rapid guidance where there are differing possible interpretations
of an international accounting standard. Its role is therefore to:
 interpret international accounting standards (IASs and IFRSs);
 issue timely guidance on issues not covered by an IAS or IFRS, within the
context of the IASB Conceptual Framework; and
 publish draft Interpretations for public comment. After studying responses to
the draft Interpretation, it will obtain IASB approval for a final (published)
Interpretation (an IFRIC)

THE IFRS ADVISORY COUNCIL (IFRSAC)


The IFRS Advisory Council (IFRSAC) provides a forum through which the IASB is able to
gather opinions and advice from different countries and industries. The IFRSAC consists of
experts from different countries and different business sectors, who offer advice to the IASB.
Other influential bodies

353
GLOBAL PREPARERS FORUM (GPF)
The GPF is an independent body set up with the aim of providing the IASB with regular input
from the international community of preparers of financial statements.
The GPF consists of members with considerable practical experience of financial reporting and
are established commentators on accounting matters in their own right or through working
with representative bodies in which they are involved. Members are drawn from a variety of
industry and geographical backgrounds and are selected by the GPF on the merits of their
professional competence and practical preparer experience in order to contribute to the
development of high quality, global accounting standards.
The GPF
 provides input into concepts and proposals that the IASB is developing; and
 offers advice to the IASB on the practical implications of its intended proposals
for preparers of financial statements.
FINANCIAL CRISIS ADVISORY GROUP (FCAG)
FCAG was formed at the request of the IASB and the US Financial Accounting
Standards Board (FASB) to consider financial reporting issues arising from the crisis in
2008.

FCAG was comprised of 15-20 senior leaders with broad experience of international
financial markets and an interest in the transparency of financial reporting
information.

FCAG was asked to suggest how improvements in financial reporting could help enhance
investor confidence in financial markets. The group also helped identify significant
accounting issues that required the urgent and immediate attention of the boards, as well as
issues for longer-term consideration.

Issues that FCAG was invited to discuss included the following:

 Areas where financial reporting helped identify issues of concern, or created


unnecessary concerns, during the credit crisis.
 Areas where financial reporting standards could have provided more
transparency to help either anticipate the crisis or respond to the crisis more
quickly.
 Potential areas that require the attention of the IASB and FASB in order to avoid
future market disruption.
The need for a due process for accounting standard-setters and its implications for
resolving emergency issues on a timely and inclusive basis.

INTERNATIONAL ACCOUNTING HARMONIZATION


Convergence of accounting standards
Convergence of accounting standards refers to the goal of establishing a single set of
accounting standards that will be used internationally. This is also described as the
international harmonisation of accounting standards.

354
Harmonisation of accounting would result in all companies anywhere in the world
reporting financial position and financial performance in the same way with the belief
being that this would lead to greater market efficiency through the quality of the
information and should make raising finance cheaper and easier.
There are two candidate GAAPs to become the basis for an international GAAP. These are:
 US GAAP; and
 International Accounting Standards (International Financial Reporting Standards.
 International accounting standards have found widespread support.

Major International Organizations Promoting Harmonization of Accounting


Six organizations have become a major player in the determination of the international
accounting standards and in promoting international harmonization of accounting:
A. International Accounting Standards Board (IASB)
2. Commission of the European Union (EU)
3. International Organization of the Capital Market Commission (IOSCO)
4. International Federation of Accountants (IFAC)
5. Intergovernmental Working Group of Experts on the United Nations International Standards
of Accounting and Reporting (International Standards of Accounting and Reporting – Isar), part
of the United Nations Conference in Trade and Development (United Nations Conference on
Trade and Development-UNCTAD)
6. Accounting Standards Working Group in the Organization of Economic Cooperation and
Development OECD Working

ADVANTAGES OF HARMONIZATION
The first and most important advantage of harmonization of reporting standards is to achieve
comparability in financial statements. Due to different sets of financial reporting standards, the
way financial statements prepared and presented are different from each other which make it
complicated to compare them. This is even more noticeable in multinational companies when
they operate in more than just one country. If international harmonization is achieved, the level
of international comparability also increases making it easier for companies to prepare the
financial statements under one set of rules; investors who understand the financial statements
due to the nature of IFRS and make well thought investment decisions.

International Expansion
Moving to a single set of global financial standards would also ease barriers to expansion for
companies. If companies wish to expand overseas today, they need to consider international
costs of compliance, which could mean adopting a completely new set of accounting records to
meet statutory requirements in the new country. In some cases, this would nearly double the
company's accounting costs. For many small businesses, even the large rewards of moving
overseas are dwarfed by these expansion costs.

Central Authoritative Body


From a policy-making standpoint, moving to a single set of global standards puts rule making
into the hands of one body. Currently, accounting standards are set within each country by each
standard-setting body, as well as by an international group. One set of standards would reduce
disagreement between countries and international regulators, and it might also cut costs. In
some countries, businesses are required to pay reporting fees that go to fund these standard-

355
setting bodies. While the costs may not affect large companies, they can have a huge impact on
a small business. Moving to a central authoritative body could reduce these costs drastically.

There will be increased auditing efficiency and money saving as companies has to use only one
set of reporting standards. This also serves to reduce trade barriers among countries allowing
more access to international capital markets.

Another advantage worth noting is the consistency to be achieved under IFRS as it was one of
the objectives of IFRS as a single reporting standard. The consistency also contributes to better
understanding between investors, lenders and other businesses as there will be the nature of
predictability in place. Moreover, companies operating in different countries also can use their
expertise and systems in all countries they are operating due to consistency of the reporting
standards. Another benefit that derives from consistency is the time scale needed to implement
in new countries as there will be no need to learn and adapt to new country specific rules except
minor adjustments.

DISADVANTAGES OF HARMONIZATION
Cultural Differences
One of the criticisms of harmonized accounting standards is that the IASB has failed to fully
take into account the cultural, political and social differences between countries. This is
particularly relevant to their implementation in developing countries,where language barriers,
attitudes toward accounting and other socio-cultural aspects may affect their interpretation and
application. For example, when the harmonized standards were implemented in Jordan, they
were first translated into Arabic. Even though technical accounting terms have been well-
defined in Arabic, challenges arose when the English terminology was hard to interpret or used
inconsistently and, therefore, difficult to translate accurately.

Worldwide Acceptance
National accounting standards are highly politicized and there is often a natural tendency to
place the interests of the national economy ahead of those of the global economy. Private sector
businesses and professional accounting bodies also have a vested interest in accounting
practices and financial reporting. Pressure from these groups to change or reject certain
standards can carry a lot of weight with political decision makers. Adopting international
financial standards is met with additional challenges in developing countries. They often lack
the resources and infrastructure to adapt national legal and legislative frameworks in which to
house the standards, making proper implementation difficult.

International Enforcement
The success of harmonized financial reporting depends on individual governments enforcing
adherence to the international standards once they have been implemented. In 2008, the French
authorities allowed the bank Société Générale to transfer some of its losses from 2008 to 2007,
meaning its financial statement for 2008 looked much better than the reality. This provoked an
international outcry, not the least from the IASB. When exceptions are made, it undermines the
integrity of the whole system and renders it ineffective.

356
Training and Retraining
When a country decides to harmonize with the international standards, its companies,
accountants and auditors need to be retrained in the new standards and reporting procedures
for financial statements. College and university programs in this field also have to undergo
significant changes in order to educate new people entering the profession. Before any of this
can happen, trainers and professors will require training so they can instruct professionals and
students. This will require the development of new learning materials and curricula, new
examinations for professional licensing and new accounting software and reporting systems. To
further complicate matters, the adoption of harmonized standards has to be phased in, so for a
number of years,two different systems are in operation. Such a complex transition requires a lot
of safety mechanisms to ensure it achieves uniform results.

Another disadvantage of harmonization is when there exists different economic environment


as harmonization could be considered useless. If a particular country has its own practice in
place, and even though they adapt to use one of the international reporting standards, it could
be more harmful to the country rather than make anything good. This is because the irrelevant
element of the new reporting standard may be of no use and therefore may even introduce
ambiguity and complication to that country‘s reporting standards.

FINANCIAL REPORTING AND CORPORATE GOVERNANCE


Introduction to Corporate Governance
Corporate governance has been defined (in the Cadbury Report, 1992) as follows: 'Corporate
governance is the system by which companies are directed and controlled.'
Governance is about giving a lead to the company and monitoring and controlling management
decisions, so as to ensure that the company achieves its intended purpose and aims and is
concerned with matters such as:
 In whose interests is a company governed?
 Who has the power to make decisions for a company?
For what aims or purposes are those powers used?
In what manner are those powers used?
Who else might influence the governance of a company?
Are the governors of a company held accountable for the way in which they use their powers?

How are risks managed?


Financial reporting is a central issue in corporate governance. It is the means by which those
charged with governance report the results of that governance to the owners (and other users)
of the organisation.

The Enron collapse


The collapse of US energy corporation Enron is one of the most notorious examples of
misleading financial reporting. Enron was the seventh biggest company in the US and was
valued at nearly $70bn.
Enron used accounting policies that increased its reported income and asset values and took
liabilities off its statement of financial position. For example:
In one transaction it recorded about $200 million of project expenses as assets, even though the
projects had effectively been cancelled. This was justified on the grounds that the projects had
not yet been cancelled officially-

357
Another technique used extensively by the company was to set up special purpose entities in
order to take liabilities off the statement of financial position and avoid having to report losses.
Allegations of misleading financial reporting were made by a whistle-blower but were initially
rejected by the company, However, the company had to restate its financial statements for the
previous four years. This reduced the reported value
Regulatory framework of the company's equity by 10%. The company collapsed towards the
end of 2001.
An outsider would have judged Enron's corporate governance processes to be quite
robust before the collapse. The company was able to recruit highly qualified individuals
to run the company, the company had an audit committee comprising non-executive
directors who were had held very senior positions in commerce
Jog sand government and the company was audited one of the most respected firms of
financial advisors in the world.
However, the senior management abused their positions to hide adverse commercial
decisions and produced financial information that presented a false impression of
financial position and performance over many years. Several managers were subsequently
indicted and sentences to prison terms.
Furthermore, the audit committee and the auditors failed to do their jobs properly.
Enron's auditors were the Houston office of Arthur Andersen, at the time one of the five
largest audit firms in the world. The auditors failed to express any concerns about the
company's accounting policies.

It subsequently emerged that the Houston office of the firm derived 25% of its annual
revenue from Enron, casting serious doubt on the auditor's independence from Enron.
When the Securities and Exchange Commission announced an investigation into the
collapse of Enron, the Houston office of Andersen tried to cover up evidence of
inadequate audit work by shredding several tons of documents and deleting thousands of
e-mails and computer files.

The scandal eventually led to the dissolution of Arthur Andersen, until then one of the
most respected professional firms in the world.
Emergence of codes of corporate governance
Corporate governance emerged as an issue in the 1980s, initially in the UK. During the
1980s there were several financial scandals affecting major UK companies. In each case
the collapse was unexpected, and was subsequently attributed to misleading financial
reporting and dominant chief executives.

It was apparent that some company CEOs and chairmen were dominating companies,
and running them as personal empires. Others were simply running companies in
their own personal interests rather than in the interests of the shareholders.
There was also concern that some companies might be 'window dressing' their
published financial statements. In spite of the fact that company accounts were audited
and the auditors stated that they gave a 'true and fair view', the financial picture
presented by the accounts could not necessarily be relied on. There were suspicions that
external auditors might not be sufficiently independent of senior management in their
client companies, and that they might be allowing companies to get away with
questionable financial reporting practices.

358
Codes of corporate governance
Codes of corporate governance apply to listed companies. Laws and regulations on
corporate governance issues are also mainly applied to listed companies. This is not
surprising, as the main reason for codes of governance is to help and protect investors.
SUBSTANCE OVER FORM
IMPORTANCE OF SUBSTANCE OVER FORM
 Economic Reality is more important:
In substance over form principle the economic reality of transaction is more important than
legal form of the transaction. Thus the transaction are not recorded merely on the bases of legal
reality and economic reality is more relevant for accounting for the transaction. This is an
important characteristic of this principle.
 Portray Real Nature of Transaction:
Another important characteristic of Substance over form principle is portraying the real picture
of the transactions. This approach discourages the hiding the actual nature of transaction. This
concept has been explained with few examples.
 Achieve Faithful Representation:
Application of prudence concept helps in achieving the faithful representation of financial
statements. Faithful representation of financial statements is an important accounting principle.
 It has become clear that any hard rule can be creatively circumvented by contriving
transactions appropriately. This is because transactions are necessarily structured between
parties as they wish, and to suit their business needs. Regulators cannot anticipate the needs
of transacting parties, and rules by their nature always lag behind the transactions
themselves. In other words, as regulators see a transaction that is not covered by the existing
rules, they seek to ―plug the gap‖. This is not a satisfactory way to ensure that financial
statements reflect truth and fairness.

Features indicating that the substance of a transaction may differ from its legal form:
There are some key tell-tale indicators that should alert the accountant to the possibility that
substance and form issues may exist. Some of these are:
 If two or more transactions are executed together, and the combined effect of both taken
together is different from the effect of each individually, this is worth investigating. For
example a sale transaction selling an asset (possibly at a price in excess of market) and an
agreement to lease back that same asset (possibly at an inflated rental).
 Business arrangements entered into which seem to disproportionately advantage one party
over another. There is nothing legally wrong with entering into an arrangement that is not
in your best interests, but rational businesspeople tend not to do so unless there is a benefit
elsewhere.
 Contrived option arrangements that serve to divert risk from where it might otherwise lie.
 Unusual terms in business agreements that might affect our assessment of the timing of
revenues and costs.
 Transactions that may be structured to manipulate the reported results, for example by
misreporting expenses as assets, or liabilities as revenue.
THE NEED FOR A CONCEPTUAL FRAMEWORK
A conceptual framework can be defined as a system of ideas and objectives that lead to the
creation of a consistent set of rules and standards. Specifically in accounting, the rule and
standards set the nature, function and limits of financial accounting and financial statements.
The main reasons for developing an agreed conceptual framework are that it provides:

359
 a framework for setting accounting standards;
 a basis for resolving accounting disputes;
 fundamental principles which then do not have to be repeated in accounting standards

The purpose of this framework is to :

 Assist the IASB in the development of future accounting standards and in its review of
existing accounting standards
 Assist the IASB by providing a basis for reducing the number of alternative accounting
treatments
 Assist national standard-setting bodies in developing national standards
 Assist accountants to apply relevant accounting standards in preparing financial
statements and in dealing with topics that do not form the subject of International
accounting standards;
 Assist auditors in forming an opinion as to whether financial statements conform with
relevant accounting standards;
 Assist users of financial statements in interpreting the information contained in financial
statements prepared in conformity with International Accounting Standards

NON-COMPLIANCE WITH IFRS


IAS 1 allows non-compliance with a standard (or interpretation) only where management
concludes that compliance would be so misleading as to conflict with objectives of financial
statements set out in the IASB Framework. However this is only where the relevant regulatory
framework requires, or does not prohibit, such a departure.
The standard uses the phrase ‗where management concludes‘ which may indicate that there a
margin for those preparing the financial statements to use this exception where they believe it is
appropriate. However, IAS 1 talks about this coming about ‗in extremely rare circumstances‘.
To all intents and purposes, these circumstances will never occur.
Advantages of Conceptual Framework
(a) The situation is avoided whereby standards are developed on a patchwork basis, where a
particular accounting problem is recognised as having emerged, and resources were then
channeled into standardising accounting practice in that area, without regard to whether that
particular issue was necessarily the most important issue remaining at that time without
standardisation.
(b) As stated above, the development of certain standards (particularly national standards) have
been subject to considerable political interference from interested parties. Where there is a
conflict of interest between user groups on which policies to choose, policies deriving from a
conceptual framework will be less open to criticism that the standard-setter buckled to external
pressure.
(c) Some standards may concentrate on the income statement whereas some may concentrate
on the valuation of net assets (statement of financial position).

Disadvantages of Conceptual Framework


(a) Financial statements are intended for a variety of users, and it is not certain that a single
conceptual framework can be devised which will suit all users.
(b) Given the diversity of user requirements, there may be a need for a variety of accounting
standards, each produced for a different purpose (and with different concepts as a basis).

360
(c) It is not clear that a conceptual framework makes the task of preparing and then
implementing standards any easier than without a framework.

THE REGULATORY FRAMEWORK


The Institute of Chartered Accountants (Ghana) was established by an Act of Parliament, Act
170, in 1963. It is charged with the regulation of the accountancy profession in Ghana, including
setting professional standards and is empowered by law to adopt accounting standards.
The Institute adopted IFRS Standards as Ghana National Accounting Standards in 2007 and
adopted the IFRS for SMEs Standard in 2010.
The Ghana Stock Exchange listing rules require all listed companies to prepare financial
statements in accordance with the Ghana National Accounting Standards. Therefore, all
companies whose securities trade in a public market must use IFRS.
Furthermore, IFRS is required for the financial statements of all government business
enterprises, banks, insurance companies, securities brokers, pension funds, and public utilities,
whether or not their securities trade in a public market.
All other business entities (i.e. whose securities do not trade in a public market) can choose to
follow either full IFRS or the IFRS for SMEs.4Standard Examinable at
this level?
IFRSs and IASs
Standard
IAS 1 – Presentation of Financial Statements Yes
IAS 2 – Inventories Yes
IAS 7 – Cash Flow Statements Yes
IAS 8 – Accounting Policies, Changes in Accounting
Estimates and Errors Yes
IAS 10 – Events occurring after the reporting period Yes
IAS 12 – Income Taxes Yes
IAS 16 – Property, Plant and Equipment Yes
IAS 19 – Employee Benefits Yes
IAS 20 – Accounting for Government Grants and
Disclosure of Government Assistance Yes
IAS 21 – The Effects of Changes in Foreign Exchange
Rates Yes
IAS 23 – Borrowing Costs Yes
IAS 24 – Related Party Disclosures Yes
IAS 26 – Accounting and Reporting by Retirement
Benefit Plans Yes
IAS 27 – Separate Financial Statements Yes
IAS 28 – Accounting for Investments in Associates and
Joint ventures Yes
IAS 29 – Financial Reporting in Hyperinflationary
Economies Yes
IAS 32 – Financial Instruments: Presentation Yes
IAS 33 – Earnings Per Share Yes
IAS 34 – Interim Financial Reporting Yes
IAS 36 – Impairment of Assets Yes

361
IAS 37 – Provisions, Contingent Liabilities and
Contingent Assets Yes
IAS 38 – Intangible Assets Yes
IAS 40 – Investment Property Yes
IAS 41 – Agriculture Yes
IFRS 1 – First time adoption of IFRS Yes
IFRS 2 – Share-based payment Yes
IFRS 3 – Business combinations Yes
IFRS 5 – Non-current assets held for sale and
discontinued operations Yes
IFRS 6 – Exploration for and evaluation of mineral
resources Yes
IFRS 7 – Financial Instruments: Disclosures Yes
IFRS 8 – Operating segments Yes
IFRS 9 – Financial Instruments Yes
IFRS 10 – Consolidated financial statements Yes
IFRS 11 – Joint arrangements Yes
IFRS 12 – Disclosure of interests in other entities Yes
IFRS 13 – Fair value measurement Yes
IFRS 14 – Regulatory deferral accounts No
IFRS 15 – Revenue from contracts with customers Yes
IFRS 16 – Leases Yes
IFRS 17 – Insurance contracts Yes
IFRS for SMEs Yes

THE IFAC CODE OF ETHICS


Introduction
Ethics can be difficult to define but it is principally concerned with human character and
conduct. Ethical behaviour is more than obeying laws, rules and regulations. It is about doing
„the right thing‟. The accountancy profession is committed to acting ethically and in the public
interest.
Professional accountants may find themselves in situations where values are in conflict with one
another due to responsibilities to employers, clients and the public.
Accountants who are responsible for the preparation of financial information must ensure that
the information they prepare is technically correct, reports the substance of the transaction and is
adequately disclosed. The danger is that they are put under pressure from senior managers to
present figures that inflate profit or assets or understate liabilities. This puts the accountant in a
difficult position.
On one hand, they wish to prepare proper information and on the other hand,
there is a possibility they might lose their job if they do not comply with their managers wishes.
In this case, ethics starts with the individual preparing the information. They have a difficult
decision to make; whether to keep quiet or take the matter further. If they keep quiet, they will
certainly be aware that they are not complying with the ethics of the accounting body they
belong to. If they speak out, they may be bullied at work into changing the information or
sacked. Many accounting bodies have ethical „help lines‟ where an individual can ring for
advice.

362
ICAG has adopted IFAC‟s Code of Ethics for Professional Accountants published by the IESBA
(hereafter referred to as „the Code‟) and states that it adopts new and revised amendments to the
Code on an ongoing basis without modifications.
The most recent version of the Code was published in April 2018 and is effective from June
2019. This book has been updated for the updated Code.
The code provides guidance in situations where ethical issues arise.
Comment
Most people are honest and have integrity and will always try to behave in the right way in a
given set of circumstances. However, accountants might face situations where it is not easy to
see the most ethical course of action. One of the main roles of the code is to provide guidance in
these situations.
Acting in the public interest
An aspect of professional bodies, which separates a profession from a trade, is that members of
the profession are expected to act in the public interest. It is therefore a responsibility of the
accountancy profession „not to act exclusively to satisfy the needs of a particular client or
employer‟. When the demands or needs of a client or employer appear to be contrary to the
public interest, accountants should consider the public interest.
So what is the public interest? Professional codes of ethics do not provide a clear definition, but
it is usual to associate the public interest with matters such as:
 detecting and reporting any serious misdemeanour or crime;
 protecting health and public safety;
 preventing the public from being misled by a statement or action by an individual or an
organisation;
 exposing the misuse of public funds and corruption in government;
 revealing the existence of any conflict of interests of those individuals who are in a
position of power or influence.
Impacting on members in practice
All members and student members of ICAG are required to comply with the code of ethics and it
applies to both accountants in practice and in business.
This chapter explains ethical issues surrounding the preparation of financial statements and other
financial information. The Code contains the following material:
 Part 1: Complying with the Code, which includes the fundamental
principles and the conceptual framework applicable to all professional
accountants.
 Part 2: Professional Accountants in Business
 Part 3: Professional Accountants in Public Practice
Basic approach in the code
The Code requires professional accountants to comply with the fundamental principles of ethics.
They must apply the conceptual framework to identify, evaluate and address threats to
compliance with the fundamental principles.
A professional accountant must comply with each of the following fundamental principles which
establish the standard of behaviour expected of a professional accountant.
The Code expresses its guidance in terms of five fundamental principles. .
These are:
 integrity;
 objectivity;

363
 professional competence and due care;
 confidentiality; and
 professional behavior
Integrity
Professional accountants should be straightforward and honest in all professional
and business relationships. Integrity implies not just honesty but also fair dealing and
truthfulness.
A professional accountant should not be associated with reports, returns,
communications or other information where they believe that the information:
 contains a materially false or misleading statement;
 contains statements or information furnished recklessly; or
 omits or obscures information required to be included where such omission or obscurity
would be misleading.
Objectivity
Professional accountants should not allow bias, conflicts of interest or undue influence of others
to override their professional or business judgements.
A professional accountant may be exposed to situations that may impair objectivity. It is
impracticable to define and prescribe all such situations.
Relationships that bias or unduly influence the professional judgment of the professional
accountant should be avoided.
Professional competence and due care
Practising as a professional accountant involves a commitment to learning over one‟s entire
working life.
Professional accountants have a duty to attain and maintain their professional knowledge and
skill at such a level that a client or employer receives a competent service, based on current
developments in practice, legislation and techniques. Professional accountants should act
diligently and in accordance with applicable technical and professional standards.
Continuing professional development develops and maintains the capabilities that enable a
professional accountant to perform competently within the professional environments.
Confidentiality
Professional accountants must respect the confidentiality of information acquired as a result of
professional and business relationships and should not disclose such information to third parties
without authority or unless there is a legal or professional right or duty to disclose.
Confidential information acquired as a result of professional and business relationships should
not be used for the personal advantage of professional accountants or third parties.
Professional behavior
Professional accountants must comply with relevant laws and regulations and should avoid any
action which discredits the profession.
Threats to the fundamental principles
Compliance with the fundamental principles may potentially be threatened by a broad range of
circumstances.
The Code also requires professional accountants to apply the conceptual framework to identify,
evaluate and address threats to compliance with the fundamental principles.
Many threats fall into the following categories:
 self-interest;
 self-review;

364
 advocacy;
 familiarity; and
 intimidation.
Professional accountants must identify, evaluate and respond to such threats.
Unless any threat is clearly insignificant, professional accountants must implement safeguards to
eliminate the threats or reduce them to an acceptable level so that compliance with the
fundamental principles is not compromised.
Self-interest threats
Self-interest threats may occur as a result of the financial or other interests of professional
accountants or their immediate or close family members.
Such financial interests might cause professional accountants to be reluctant to take actions that
would be against their own interests.
Examples of circumstances that may create self-interest threats include, but are not limited to:
 financial interests, loans or guarantees;
 incentive compensation arrangements;
 inappropriate personal use of corporate assets;
 concern over employment security; or
 commercial pressure from outside the employing organisation.
Self-review threats
Self-review threats occur when a previous judgement needs to be re-evaluated by professional
accountants responsible for that judgement. For example, where a professional accountant has
been involved in maintaining the accounting records of a client he may be unwilling to find fault
with the financial statements derived from those records. Again, this would threaten the
fundamental principle of objectivity.
Circumstances that may create self-review threats include, but are not limited to, business
decisions or data being subject to review and justification by the same professional accountant in
business responsible for making those decisions or preparing that data.
Advocacy threats
A professional accountant in business may often need to promote the organisations position by
providing financial information. As long as information provided is neither false nor misleading
such actions would not create an advocacy threat.
Familiarity threats
Familiarity threats occur when, because of a close relationship, professional accountants become
too sympathetic to the interests of others. Examples of circumstances that may create familiarity
threats include:
 A professional accountant in business in a position to influence financial or non-financial
reporting or business decisions having an immediate or close family member who is in a
position to benefit from that influence.
 Long association with business contacts influencing business decisions.
 Acceptance of a gift or preferential treatment, unless the value is clearly insignificant.
Intimidation threats
Intimidation threats occur when a professional accountant‟s conduct is influenced by fear or
threats (for example, when he encounters an aggressive and dominating individual at a client or
at his employer).
Examples of circumstances that may create intimidation threats include:

365
 Threat of dismissal or replacement over a disagreement about the application of an
accounting principle or the way in which financial information is to be reported.
 A dominant personality attempting to influence decisions of the professional accountant
Safeguards
Safeguards which may remove or reduce threats to members fall into three categories:
 safeguards created by the profession, legislation or regulation
 safeguards in the work environment
 safeguards created by the individual.
Each of these categories is considered below.
Safeguards created by the profession, legislation or regulation include:
 educational, training and experience requirements for entry into the profession;
 continuing professional development requirements;
 corporate governance regulations;
 professional or regulatory monitoring and disciplinary procedures (such as the ICAG‟s
own disciplinary procedures).
Safeguards in the work environment include:
 the employer‟s own systems of monitoring and ethics and conduct programmes (such as
an internal training or a mentoring programme);
 recruitment procedures, ensuring that only high-calibre, competent staff are recruited;
 appropriate disciplinary processes;
 strong internal controls;
 leadership that stresses the importance of ethical behaviour and which expects employees
to behave ethically;
 policies and procedures to implement and monitor the quality of employee performance;
 policies and procedures to give employees the power to report ethica issues to senior staff
without fear of retribution from those about whom they are making the report.
Safeguards created by the individual include:
 complying with continuing professional development requirements;
 keeping records of contentious issues and approach to decision-making;
 having a broader perspective on how other organisations operate by
forming business relationships with other professionals;
 using an independent mentor; and
 keeping in contact with legal advisors and professional bodies

Resolving ethical conflicts


An accountant may be required to resolve a conflict in the application of the fundamental
principles.
When trying to resolve a conflict the accountant must consider the following:
 the relevant facts;
 the fundamental principles related to the matter in question;
 the ethical issues involved;
 whether there are any internal procedures to resolve ethical issues; and
 the alternative courses of action that may be taken.
Having considered these issues, an accountant should determine the appropriate course of
action that is consistent with the fundamental principles identified whilst bearing in mind the
consequences of each possible course of action.

366
If the matter remains unresolved, the accountant should consult with other appropriate
persons within the employing organisation for help in obtaining resolution.
Where a matter involves a conflict within, an organisation, an accountant should also
consider consulting with those charged with governance of the organisation, such as the
board of directors or the audit committee.
Issues and relevant discussions should be documented.
Non-resolution
If a significant conflict cannot be resolved, an accountant may wish to obtain professional
advice from the appropriate committee of the Institute or legal advisors, and thereby obtain
guidance on ethical issues without breaching confidentiality.
If, after exhausting all relevant possibilities, the ethical conflict remains unresolved, an
accountant should, where possible, refuse to remain associated with the matter creating the
conflict. The accountant may determine that, in the circumstances, it is appropriate to resign
from the employing organisation.

ETHICS AND ACCOUNTANTS IN BUSINESS


 Accountants in business
 Preparation and reporting of information
 Potential conflicts
3.1 Accountants in business
Accountants in business are often responsible for the preparation of accounting information.

Accountants in business need to ensure that they do not prepare financial information in a
way that is misleading or that does not show a true and fair view of the entity‟s operations.

Accountants who are responsible for the preparation of financial information must ensure
that the information they prepare is technically correct, reports the substance of the
transaction and is adequately disclosed.

There is a danger of influence from senior managers to present figures that inflate profit or
assets or understate liabilities. This puts the accountant in a difficult position. On one hand,
they wish to prepare proper information and on the other hand, there is a possibility they
might lose their job if they do not comply with their managers wishes.

In this case, ethics starts with the individual preparing the information. They have a difficult
decision to make; whether to keep quiet or take the matter further. If they keep quiet, they
will certainly be aware that they are not complying with the ethics of the accounting body
they belong to. If they speak out, they may be bullied at work into changing the information
or sacked.

Preparation and reporting of information


Professional accountants in business are often involved in the preparation and reporting of
information that may either be made public or used by others inside or outside the employing
organisation. Such information may include financial of management information, for example:
 forecasts and budgets;
 financial statements;

367
 management discussion and analysis; and
 the management letter of representation provided to the auditors as part of an audit of
financial statements.
Information must be prepared and presented fairly, honestly and in accordance with relevant
professional standards. In particular financial statements must be prepared and presented in
accordance with the applicable financial reporting standards.
A professional accountant in business must maintain information for which he is responsible in a
manner that:
 describes clearly the true nature of business transactions, assets or liabilities;
 classifies and records information in a timely and proper manner; and
 represents the facts accurately and completely in all material respects.

Threats to compliance with the fundamental principles, for example self-interest or intimidation
threats to objectivity or professional competence and due care, may be created where a
professional accountant in business may be pressured (either externally or by the possibility of
personal gain) to become associated with misleading information or to become associated with
misleading information through the actions of others.
The significance of such threats will depend on factors such as the source of the pressure and the
degree to which the information is, or may be, misleading.
The significance of the threats should be evaluated and unless they are clearly insignificant,
safeguards should be considered and applied as necessary to eliminate them or reduce them to an
acceptable level. Such safeguards may include consultation with superiors within the employing
organisation, for
example, the audit committee or other body responsible for governance, or with a relevant
professional body.
Where it is not possible to reduce the threat to an acceptable level, a professional accountant
should refuse to remain associated with information they consider is or may be misleading.
If the professional accountant is aware that the issuance of misleading information is either
significant or persistent, he should consider informing appropriate authorities in line with the
guidance in this code. The professional accountant in business may also wish to seek legal advice
or resign.

Potential conflicts
There may be times when the responsibilities of a professional accountant to an employing
organisation come into conflict with their professional obligations to comply with the
fundamental principles in the Code. Where compliance with the fundamental principles is
threatened, a professional accountant in business must consider a response to the circumstances.
Responsibilities to an employer may put a professional accountant under pressure to act or
behave in ways that could directly or indirectly threaten compliance with the fundamental
principles. Such pressure may be explicit or implicit; it may come from a supervisor, manager,
director or another individual within the employing organisation.
A professional accountant in business may face pressure to:
 act contrary to law or regulation;
 act contrary to technical or professional standards;
 lie to, or otherwise intentionally mislead (including misleading by remaining silent)
others, in particular:

368
 the auditors of the employing organisation; or
 regulators;
 issue, or otherwise be associated with, a financial or non-financial report
that materially misrepresents the facts, including statements in connection with, for
example:
 the financial statements;
 tax compliance;
 legal compliance; or
 reports required by securities regulators.
The significance of threats must be evaluated and unless they are clearly insignificant, safeguards
should be considered and applied to eliminate them or reduce them to an acceptable level.
Such safeguards may include:
 obtaining advice where appropriate from within the employing organisation, or an
independent professional advisor or a relevant professional body;
 the existence of a formal dispute resolution process within the employing organisation;
 seeking legal advice.

Practice question
Abal is an ICAG Chartered Accountant who works at the head office of a group of companies
with investments across West Africa.
She has recently transferred to the investor relations team where she reports to the FD and will
be involved in drafting financial statements. She previously spent two years in the mergers and
acquisitions department (M&A) of the group.
Abal was in charge of the due diligence exercise on the acquisition of X Limited when she
worked in M&A.
Recent indications suggest that the acquisition of X limited has not been as successful as hoped
and there are indications that several of its capital assets might be impaired.
Abel is now responsible for the team that carries out impairment reviews for the group.
Describe the ethical issues in this scenario explain how Abal should address them.

Practice question
Kwadwo is member of ICAG working as a unit accountant.
He is a member of a bonus scheme under which, staff receive a bonus of 10% of their annual
salary if profit for the year exceeds a trigger level.
Kwadwo has been reviewing working papers prepared to support this year‟s financial statements.
He has found a logic error in a spreadsheet used as a measurement tool for provisions.
Correction of this error would lead to an increase in provisions. This would decrease profit below
the trigger level for the bonus.
What are the ethical issues in this scenario and how should they be resolved?

Practice question
Nyamekye is a chartered accountant recruited on a short-term contract to assist the finance
director, Owusuwao (who is not a chartered accountant) in finalising the draft financial
statements.
Nyamekye hope to secure a permanent position with the company. The decision on whether to
employ Nyamekye on a permanent basis rests with Owusuwao.

369
The company‟s results in the financial year just ended (20X8) were not as strong as expected. In
fact, the lower figure reported in the statement of profit or loss would lead to the company
defaulting on debt covenant.
The company has made a material loss on a derivative position during that year (20X8). The
derivative position was taken in 20X8 in order to hedge a future transaction that is expected to
occur in the current year (20X9).
Nyamekye has been instructed to prepare hedge accounting documentation to comply with IFRS
9 and backdate it so that hedge accounting can be applied in the most recent period ended. The
effect of this would be to remove the loss from the statement of profit or loss into the statement
of other comprehensive income.
Nyamekye has explained to Owusuwao that hedge accounting documentation cannot be
backdated and that hedge accounting would only be possible from the date that the
documentation was drafted.
Owusuwao responded that Nyamekye should ignore this rule in order to avoid breaching the debt
covenant.
Describe the ethical issues in this scenario explain how Nyamekye should address them.
Practice question
Afram is an ICAG Chartered Accountant. He plans to retire shortly and sell his shares in the
company.
Afram has produced draft financial statements for the year and, in doing so, processed several
late adjustments which materially improved the profit for the year.
The managing director was surprised to find that the profit for the year was materially larger than
expected and he is worried that Afram's treatment of certain matters has been influenced by his
desire to improve the profit figure (whether consciously or unconsciously) so that he may get a
better price for his shares.
Nanyamka is an ICAG Chartered Accountant who is on the short list to replace Afram as FD.
The MD has asked her to examine the late adjustments and to redraft the financial statements if
necessary.
Nanyamka has examined the adjustments, and in her opinion believes that most should be
reversed, thus reducing the profit for the year back to the level originally expected.
Nanyamka has had an initial meeting with Afram who promised her that he would recommend
her as his replacement if she makes as few adjustments to the financial statements as possible.
Describe the ethical issues in this scenario explain how Nanyamka should address them.
Practice question
Ekow is a chartered accountant who works as in a team that reports to Adika, the finance director
of Tema Holdings.
Adika Is also a chartered accountant. He has a domineering personality.
Tema Holdings revalues commercial properties as allowed by IAS 16.
Valuation information received last year showed that the fair value of the property portfolio was
2% less than the carrying amount of the properties (with no single property being more than 4%
different). A downward revaluation was not recognised on the grounds that the carrying amount
was not materially different from the fair value.
This year‟s valuation shows a continued decline in the fair value of the property portfolio. It is
now 5% less than the carrying amount of the properties with some properties now being 15%
below the carrying amount.
Ekow submitted workings to Adika in which he had recognised the downward revaluations in

370
accordance with IAS 16.
Adika has sent him an email in response in which he wrote “Stop bothering me with this rubbish.
There is no need to write the properties down. The fair value of the portfolio is only 5% different
from its carrying amount. Restate the numbers immediately”.
Describe the ethical issues in this scenario explain how Ekow should address them.
CHAPTER REVIEW
 Before moving on to the next chapter check that you now know how to:
 Explain the roles of the IASB, IFRSIC, GPF and FCAG
 Describe the standard setting process
 Discuss the international harmonisation of accounting standards
 Explain the role of financial reporting in corporate governance
 Explain and apply the requirements of the IFAC Code of Ethics in financial reporting

Solution
Abal faces a self-interest threat because impairments appear to have arisen on an acquisition in
which she was involved. The self-interest threat might compromise both her integrity and
professional behaviour.
As an ICAG Chartered Accountant, Abal needs to ensure that she acts with integrity and
demonstrate high standards of both professional behaviour and conduct.
Her judgement must not be influenced by the fact that her competence may be questioned if
impairments arise on an investment decision with which she was involved.
She must fulfil her professional responsibilities and ensure that the impairment review is
properly performed. This might involve her explaining her earlier involvement in the initial
valuation to the FD and perhaps passing the detailed review on to a colleague.
Solution
Kwadwo faces a self-interest threat which might distort his objectivity.
Kwadwo has a professional responsibility to ensure that financial information is prepared and
presented fairly, honestly and in accordance with relevant professional standards. He has further
obligations to ensure that financial information is prepared in accordance with applicable
accounting standards and that records maintained represent the facts accurately and completely
in all material respects.
Kwadwo must make the necessary adjustment even though it would lead to a personal loss.
Solution
Nyamekye faces a self-interest threat which might distort his objectivity.
The proposed accounting treatment is incorrect. Nyamekye has a professional responsibility to
ensure that financial information is prepared and presented fairly, honestly and in accordance
with relevant professional standards. He has further obligations to ensure that financial
information is prepared in accordance with applicable accounting standards and that records
maintained represent the facts accurately and completely in all material respects.
Possible course of action.
Nyamekye must explain his professional obligations to Owusuwao in particular that he cannot be
party to the preparation and presentation of knowingly misleading information.
Nyamekye should refuse to remain associated with information that is misleading.
If Owusuwao persists in requiring the backdated hedge accounting documentation in order to
change the financial statements Nyamekye should report the matter to the audit committee or the

371
other directors.
As a last resort Nyamekye should resign from his post.
Solution
Afram faces a self-interest threat that calls his integrity into question.
If Nanyamka‟s view of the adjustments is correct then Afram should have taken a similar view.
This calls into question whether Afram has made these adjustments as he is influenced by the
fact that he may get a better price for his shares if the company's profit is higher.
It could be that Afram has made the adjustments in the mistaken belief that they were correct.
That calls his professional competence into question. ICAG Chartered Accountants have an
obligation to maintain their continuing professional development and they should ensure that
their technical knowledge and professional skills are kept up to date.
Nanyamka also faces a self-interest threat as she may be offered a permanent position at the
company. She should ignore the possibility of self-interest and discuss the adjustments with
Afram and remind him of his professional responsibilities to ensure that accounting standards are
correctly followed.
If Afram refuses to make the necessary amendments Nanyamka must discuss the matter with the
managing director.
Solution
Ekow faces an intimidation threat which might distort his objectivity.
The current accounting treatment might be incorrect. The value of the properties as a group is
irrelevant in applying IAS 16‟s revalution model. IAS 16 allows the use of a revalution model
but requires that the carrying amount of a property should not be materially different from its fair
value. This applies to individual properties not the whole class taken together.
(It could be that Adika is correct because there is insufficient information to judge materiality in
this circumstance. However, a 15% discrepancy does sound significant).
Ekow has a professional responsibility to ensure that financial information is prepared and
presented fairly, honestly and in accordance with relevant professional standards. He has further
obligations to ensure that financial information is prepared in accordance with applicable
accounting standards and that records maintained represent the facts accurately and completely
in all material respects.
Possible course of action
Ekow should arrange a meeting with Adika to try to explain Adika‟s misapplication of the IAS
16 guidance and to try to persuade Adika that a change might be necessary.
Adika should be reminded that he too is bound by the same guidance that applies to Ekow.
Indeed he has a greater responsibility as the more senior person to show leadership in this area.
Ekow cannot be party to the preparation and presentation of knowingly misleading information.
He should explain that he cannot remain associated with information that is misleading. If Adika
refuses to allow the necessary changes to the information Ekow should report the matter to the
audit committee or the other directors.
As a last resort if the company refuses to change the information Ekow should resign from his
post.
Ekow may need to consider informing the appropriate authorities in line with the Code‟s
guidance on confidentiality.

372
Accounting and reporting concepts

Contents
1 A conceptual framework for financial reporting
2 The IASB Conceptual Framework
3 Qualitative characteristics of useful financial
information
4 The elements of financial statements
5 Recognition and derecognition
6 Accounting concepts
7 Measurement and capital maintenance
8 Fair presentation
9 Chapter review

1 A CONCEPTUAL FRAMEWORK FOR FINANCIAL REPORTING


Section Overview
The meaning of GAAP
The meaning of a conceptual framework
The purpose of a conceptual framework
The alternative to a conceptual framework

The meaning of GAAP


The preparation and presentation of financial statements is based on a large number of concepts,
principles and detailed rules. Some of these are contained in law, and others are in financial
reporting standards. Many of the most fundamental concepts are not contained in any law or
regulation or standard, but are simply accepted accounting principles and conventions.
All the concepts, principles, conventions, laws, rules and regulations that are used to prepare and
present financial statements are known as Generally Accepted Accounting Principles or GAAP.
„Generally accepted accounting principles‟ vary from country to country, because each country
has its own legal and regulatory system. The way in which businesses operate also differs from
country to country. (For example, there is US GAAP, UK GAAP and Ghanaian GAAP).
Many countries have now adopted International Financial Reporting Standards or IFRSs,
sometimes called international accounting standards. It is now fairly common to refer to the
totality of the rules as IFRS or IAS.

The meaning of conceptual framework


A conceptual framework is a system of concepts and principles that underpin the preparation of
financial statements. These concepts and principles should be consistent with one another.
The International Accounting Standards Committee (the predecessor of the IASB) issued a
conceptual framework document in 1989. This was called the Framework for the Preparation
and Presentation of Financial Statements and was adopted by the IASB.
The IASB has been working closely with FASB (the US standard setter) on a wide range of
projects with the aim of converging IFRS and US GAAP. One of the projects has had the aim of
producing a conceptual framework common to each GAAP.
The new conceptual framework was developed on a chapter by chapter basis.
The complete new conceptual framework was published in March 2018 and is called ”The

373
conceptual framework for financial reporting”.
Note that the changes are not fundamental in terms of their impact on IFRS.
The new document is made up of the following sections:
Chapter 1 – The objective of general purpose financial reporting.
Chapter 2 – Qualitative characteristics of useful financial information.
Chapter 3 – Financial statements and the reporting entity.
Chapter 4 – The elements of financial statements.
Chapter 5 – Recognition and derecognition
Chapter 6 – Measurement
Chapter 7 – Presentation and disclosure
Chapter 8 – Concepts of capital and capital maintenance.
The purpose of a conceptual framework
Most preparers and users of financial statements recognise that there is a need for a formal
conceptual framework and that this can be useful in a number of ways.
Where there is a formal conceptual framework for accounting, accounting practice and
accounting standards are based on this framework.
Lack of a formal framework often means that standards are developed randomly or only to deal
with particular problems. The result is that standards are inconsistent with each other or with
legislation.
Lack of a conceptual framework may also mean that accounting standards fail to address
important issues. For example, until the IASB developed its Framework,
there was no proper definition of terms such as „asset‟, „liability‟, „income‟ and „expenses‟.
The business environment is becoming increasingly complex. It is unlikely that accounting
standards can cover all possible transactions. Where an entity enters into an unusual transaction
and there is no relevant accounting standard, it can refer to the framework and apply the
principles in it.
It can also be argued that a conceptual framework strengthens the credibility of financial
reporting and the accounting profession in general.

The alternative to a conceptual framework


The alternative to a system based on a conceptual framework is a system based on detailed rules.
Accounting standards based on detailed rules are open to abuse. „Creative accounting‟ is the
name given to techniques which enable management to give a biased impression (usually
favourable) of the company‟s performance while still complying with accounting standards and
other regulations. During the 1980s there were a number of scandals in which investors were
misled by the financial statements of apparently healthy companies which then collapsed. This
was one of the original reasons why the IASB and other standard setters developed their
conceptual frameworks. Principles are normally much harder to evade than rules Another
disadvantage of a rule-based system is that standard setters are more likely to be influenced by
„vested interests‟ such as large companies or a particular business sector. The existence of a
conceptual framework is an important safeguard against this kind of political pressure.
Despite these problems, some preparers and regulators still appear to favour rule based
standards. Standards based on principles may require management to use its judgement (and to
risk making a mistake), while rules simply need to be followed. This can be important where
management can face legal action if an investor makes a poor decision based on the financial
statements. The use of a conceptual framework can lead to standards that are theoretical and

374
complex. They may give the „right answer‟ but be very difficult for the ordinary preparer to
understand and apply. However, a system of extremely detailed rules can also be very difficult to
apply.

THE IASB CONCEPTUAL FRAMEWORK


Section overview
Introduction
Users and their information needs
Chapter 1: Objective of general purpose financial statements
Chapter 4: Financial statements and the reporting entity
Introduction
Financial reports are based on estimates, judgements and models rather than exact depictions.
The Conceptual Framework establishes the concepts that underlie those estimates, judgements
and models.
The Conceptual Framework deals with:
 the objective of financial reporting;
 the qualitative characteristics of useful financial information;
 the definition, recognition and measurement of the elements from which financial statements
are constructed; and
 concepts of capital and capital maintenance.
The Conceptual Framework sets out the concepts that underlie the preparation
and presentation of financial statements for external users. Its purpose is to:
 assist the IASB to develop IFRS Standards (Standards) that are based on
consistent concepts;
 assist preparers to develop consistent accounting policies when no standard applies to a
particular transaction or other event, or when IFRS allows a choice of accounting policy; and
 assist all parties to understand and interpret IFRS.
This Conceptual Framework is not an IFRS and nothing in the Conceptual Framework overrides
any specific IFRS. On very rare occasions there may be a conflict between the Conceptual
Framework and an IFRS. In those cases, the requirements of the IFRS prevail over those of the
Conceptual Framework.

Users and their information needs


Many existing and potential investors, lenders and other creditors cannot require reporting
entities to provide information directly to them and must rely on general purpose financial
reports for much of the financial information they need. These are the primary users to whom
general purpose financial reports are directed.
 General purpose financial reports cannot provide all the information needed and users also
need to consider pertinent information from other sources.
 General purpose financial reports do not show the value of a reporting entity; but they
provide information to help users estimate a value.
 Individual primary users have different information needs. The aim of IFRSs is to provide
information that will meet the needs of the maximum number of primary users.
Other users

375
 Regulators and members of the public other than investors, lenders and other creditors, may
also find general purpose financial reports useful but these reports are not primarily directed
to these groups.
 A company‟s management is often interested in financial information but the management do
not need to rely on general purpose financial reports.

Chapter 1: Objectives of general purpose financial statements


The objective of general purpose financial reporting forms the foundation of the Conceptual
Framework. Other aspects of the Conceptual Framework flow logically from the objective.
The objective
The objective of general purpose financial reporting is to provide financial information about the
reporting entity that is useful to existing and potential investors, lenders and other creditors in
making decisions about providing resources to the entity.
Those decisions involve buying, selling or holding equity and debt instruments, and providing or
settling loans and other forms of credit.
 In order to make these decisions the users need information to help them assess the prospects
for future net cash inflows to an entity.
 In order to assess an entity‟s prospects for future net cash inflows, users need information
about:
 the resources of the entity;
 claims against the entity; and
 how efficiently and effectively the entity‟s management have discharged their
responsibilities to use the entity‟s resources. (This information is also useful for
decisions by those who have the right to vote on or otherwise influence management
performance).
Information provided
General purpose financial statements provide information about:
 the financial position of the entity – information about economic resources and the claims
against them; and
 changes in its financial position which could be due to:
 financial performance; and/or
 other events or transactions (e.g share issues).
Economic resources and claims
Information about the nature and amounts of economic resources and claims can help users to:
 identify the financial strengths and weaknesses of a reporting entity;
 to assess a reporting entity‟s liquidity and solvency and its needs for additional financing;
Information about priorities and payment requirements of existing claims helps users to
predict how future cash flows will be distributed among those with a claim against the
reporting entity.
Changes in economic resources and claims – Financial performance
Accrual accounting depicts the effects of transactions and other events and circumstances on a
reporting entity‟s economic resources and claims in the periods in which those effects occur,
even if the resulting cash receipts and payments occur in a different period.
This is important because such information provides a better basis for assessing the entity‟s past
and future performance than information solely about cash receipts and payments during that

376
period.
Importance of information about a reporting entity‟s financial performance:
 It helps users to understand the return generated from its economic
resources. This in turn provides an indication of how well management has
discharged its responsibilities to make efficient and effective use of these resources.
 It shows the capacity of a reporting entity to generate net cash inflows through its
operations rather than by obtaining additional resources directly from investors and
creditors.
 It gives an indication of the extent to which events such as changes in market prices
or interest rates affect its ability to generate net cash inflows.
Information about the variability and components of return is also important,
especially in assessing the uncertainty of future cash flows.
 Information about past financial performance is helpful in predicting the entity‟s
future returns on its economic resources.
Another aspect of performance is management of cash flow. Information about a
reporting entity‟s cash flows during a period helps users to assess the entity‟s ability to
generate future net cash inflows. It indicates how the reporting entity obtains and spends
cash, including information about its borrowing and repayment of debt, cash dividends or
other cash distributions to investors, and other factors that may affect the entity‟s
liquidity or solvency. Information about cash flows helps users understand a reporting
entity‟s operations, evaluate its financing and investing activities, assess its liquidity or
solvency and interpret other information about financial performance.
Changes in economic resources and claims – Other events and transactions
Information about this type of change is necessary to give users a complete understanding of
why the reporting entity‟s economic resources and claims changed and the implications of those
changes for its future financial performance.
Objectives if financial statements: summary
The objectives of financial statements are met by:
 the main financial statements (statement of financial position, statement of
profit or loss and other comprehensive income (or statement of profit or
loss and statement of other comprehensive income), statement of cash
flows, and statement of changes in equity), and
 supporting notes to the accounts, which provide additional details.
Chapter 4: Financial statements and the reporting entity
Objective and scope of financial statements
The objective of financial statements is to provide financial information about the
reporting entity‟s assets, liabilities, equity, income and expenses that is useful to
users of financial statements in assessing the prospects for future net cash
inflows to the reporting entity and in assessing management‟s stewardship of the
entity‟s economic resources.
That information is provided:
 in the statement of financial position, by recognising assets, liabilities and
equity;
 in the statement(s) of financial performance, by recognising income and
expenses; and
 in other statements and notes, by presenting and disclosing information about:

377
 recognised and unrecognised assets and liabilities, equity, income and expenses;
 cash flows;
 contributions from holders of equity claims and distributions to them; and
 the methods, assumptions and judgements used in estimating the
amounts presented or disclosed, and changes in those methods, assumptions and judgements.
Reporting period
Financial statements are prepared for a specified period of time (reporting period) and provide
information about:
 assets and liabilities (including unrecognised assets and liabilities) and equity that existed at
the end of the reporting period, or during the reporting period; and
 income and expenses for the reporting period.

Perspective
Financial statements provide information viewed from the perspective of the reporting entity as a
whole.
Going concern assumption
Financial statements are normally prepared on the assumption that the reporting entity is a going
concern and will continue in operation for the foreseeable future.
It is assumed that the entity does not intend or need to enter liquidation or to cease trading. If that
is not the case, the financial statements may have to be prepared on a different basis and the basis
used must be described.
The reporting entity
A reporting entity is one that prepares financial statements.
A reporting entity can be a single entity or a portion of an entity or can comprise more than one
entity (e.g. a group).
A reporting entity is not necessarily a legal entity.
It can be difficult to determine the boundary of a reporting entity that is not a legal entity. In this
case the boundary is determined by taking into account the
information needs of the primary users.

QUALITATIVE CHARACTERISTICS OF USEFUL FINANCIAL INFORMATION


Section overview
Introduction
Relevance
Faithful representation
Cost constraint on useful information
Enhancing qualitative characteristics

Introduction
This is covered by chapter 2 of The IASB Conceptual Framework.
Information must have certain characteristics in order for it to be useful for decision making. The
IASB Conceptual Framework describes:
 fundamental qualitative characteristics; and
 enhancing qualitative characteristics
Fundamental qualitative characteristics:
 relevance; and

378
 faithful representation
The qualitative characteristics that enhance the usefulness of information relevant and a
faithful representation are:
 comparability;
 verifiability
 timeliness; and
 understandability
“If financial information is to be useful, it must be relevant and faithfully represent what it
purports to represent. The usefulness of financial information is enhanced if it is comparable,
verifiable, timely and understandable”.
Emphasis
Information must be both relevant and faithfully represented if it is to be useful.
The enhancing qualitative characteristics cannot make information useful if that information is
irrelevant or not faithfully represented.
Relevance
Information must be relevant to the decision-making needs of users. Information is relevant if it
can be used for predictive and/or confirmatory purposes.
 It has predictive value if it helps users to predict what might happen in future.
 It has confirmatory value if it helps users to confirm the assessments a predictions they
have made in the past.
The relevance of information is affected by its materiality.
Information is material if omitting, misstating or obscuring it could reasonably be expected to
influence decisions of the primary users based on financial statements.
 Materiality is an entity-specific aspect of relevance based on the nature magnitude (or
both) of the items to which the information relates in the context of an individual entity‟s
financial report.
 Therefore, it is not possible for the IASB to specify a uniform quantitative threshold for
materiality or predetermine what could be material in a particular situation.
Faithful representation
Financial reports represent economic phenomena (economic resources, claims against the
reporting entity and the effects of transactions and other events and conditions that change those
resources and claims) by depicting them in words and numbers.
To be useful, financial information must not only represent relevant phenomena, but it must also
faithfully represent the phenomena that it purports to represent.
A perfectly faithful representation would have three characteristics. It would be:
 complete – the depiction includes all information necessary for a user to understand the
phenomenon being depicted, including all necessary
descriptions and explanations.
 neutral – the depiction is without bias in the selection or presentation of financial
information; and
 free from error – where there are no errors or omissions in the description of the
phenomenon, and the process used to produce the reported information has been selected
and applied with no errors in the process.

379
Enhancing qualitative characteristics
Comparability
Comparability is the qualitative characteristic that enables users to identify and understand
similarities in, and differences among, items Information about a reporting entity is more useful
if it can be compared with
similar information about other entities and with similar information about the same entity for
another period or another date.
Consistency is related to comparability but is not the same. Consistency refers to the use of the
same methods for the same items, either from period to period.
within a reporting entity or in a single period across entities. Consistency helps to achieve the
goal of comparability.

Verifiability
This quality helps assure users that information faithfully represents the economic phenomena it
purports to represent.
 Verifiability means that different knowledgeable and independent observers could reach
consensus that a particular depiction is a faithful representation.
 Quantified information need not be a single point estimate to be verifiable.
A range of possible amounts and the related probabilities can also be verified.
 Verification can be direct or indirect.
 Direct verification means verification through direct observation, e.g. by counting cash.
 Indirect verification means checking the inputs to a model, formula or other technique
and recalculating the outputs using the same methodology. For example, the carrying
amount of inventory might be verified by checking the inputs (quantities and costs) and
recalculating the closing inventory using the same assumption (e.g. FIFO).

Timeliness
This means having information available to decision-makers in time to be capable of influencing
their decisions.
Understandability
Information is made understandable by classifying, characterising and presenting it in a clear and
concise manner.
Financial reports are prepared for users who have a reasonable knowledge of business and
economic activities and who review and analyse the information diligently.

Cost constraint on useful information


Reporting financial information that is relevant and faithfully represents what it purports to
represent helps users to make decisions with more confidence. This results in more efficient
functioning of capital markets and a lower cost of capital for the economy as a whole. An
individual investor, lender or other creditor also receives benefits by making more informed
decisions. However, it is not possible for general purpose financial reports to provide all the
information that every user finds relevant.
The benefits obtained from financial information should exceed the cost of obtaining and
providing it. Information should not be provided if the cost is not worth the benefit.
Since it is difficult to measure the benefits of financial information, the setters of accounting

380
standards must use their judgement in deciding whether certain items of information should be
provided in the financial statements (and if so, in how much detail).

THE ELEMENTS OF FINANCIAL STATEMENTS


Section overview
Introduction
Assets
Liabilities
Other definitions
Introduction
This is covered by chapter 4 of The IASB Conceptual Framework.
The IASB Framework discusses the five elements of financial statements:
 for reporting financial position: assets, liabilities and equity; and
 for reporting financial performance: income and expenses.
Assets
An asset is a present economic resource controlled by the entity as a result of past events.
An economic resource is a right that has the potential to produce economic benefits.
Rights
Rights can take many forms including the right to receive cash, exchange resources on
favourable terms, rights over physical objects and rights to use intellectual property.
Many rights are established by contract, legislation or similar means.
However, rights might be obtained in other ways (e.g. developing know-how that is not in the
public domain).
Some goods or services are received and immediately consumed (e.g. employee services). The
right to obtain the economic benefits produced by such goods or services exists momentarily
until the entity consumes the goods or services.
In order to be an asset, rights must both have the potential to produce economic benefits for the
entity beyond those available to all other parties and be controlled by the entity. Therefore, not
all rights are assets (e.g. right to use public infrastructure is not an asset).
Potential to produce economic benefits
An economic resource is a right that has the potential to produce economic benefits.
A right can be an asset, even if the probability that it will produce economic benefits is low.
However, low probability might affect decisions about what information to provide about the
asset and how to provide that information, including decisions about whether the asset is
recognised and how it is measured.
Control
Control links an economic resource to an entity
Control is the ability to obtain economic benefits from the asset, and to restrict the ability of
others to obtain the same benefits from the same item.
Liabilities
A liability is a present obligation of the entity to transfer an economic resource as a result of past
events.
For a liability to exist, three criteria must all be satisfied:
 the entity has an obligation;
 the obligation is to transfer an economic resource; and
 the obligation is a present obligation that exists as a result of past events

381
Obligation
An obligation is a duty or responsibility that an entity has no practical ability to avoid.
An obligation is always owed to another party (or parties) but it is not necessary to know the
identity of the party (or parties) to whom the obligation is owed.
Obligations might be established by contract or other action of law or they might be constructive.
A constructive obligation arises from an entity‟s customary
practices, published policies or specific statements when the entity has no practical ability to act
in a manner inconsistent with those practices, policies or statements.
Transfer of economic resource
An obligation must have the potential to require the entity to transfer an economic resource to
another party (or parties).
An obligation can meet the definition of a liability even if the probability of a transfer of an
economic resource is low. However, low probability might affect decisions about what
information to provide about the liability and how to provide that information, including
decisions about whether the liability is recognised and how it is measured.
Present obligation as a result of past events
A liability is an obligation that already exists. An obligation may be legally enforceable as a
result of a binding contract or a statutory requirement, such as a legal obligation to pay a supplier
for goods purchased.
Obligations may also arise from normal business practice, or a desire to maintain
good customer relations or the desire to act in a fair way. For example, an entity might undertake
to rectify faulty goods for customers, even if these are now outside their warranty period. This
undertaking creates an obligation, even though it is not legally enforceable by the customers of
the entity.
Past transactions or events
A liability arises out of a past transaction or event.
A present obligation exists as a result of past events only if:
 the entity has already obtained economic benefits or taken an action; and
 as a consequence, the entity will or may have to transfer an economic resource that it
would not otherwise have had to transfer.
For example, a trade payable arises out of the past purchase of goods or services, and an
obligation to repay a bank loan arises out of past borrowing.

Other definitions
Equity
Equity is the residual interest in an entity after the value of all its liabilities has been deducted
from the value of all its assets.
Income
Income is increases in assets, or decreases in liabilities, that result in increases in equity, other
than those relating to contributions from holders of equity claims.
The concept of income includes both revenue and gains.
 Revenue is income arising in the course of the ordinary activities of the
entity. It includes sales revenue, fee income, royalties income, income and
income from investments (interest and dividends). Revenue is recognized in the statement
of profit or loss.

382
 Gains represent other items that meet the definition of income. Gains may be recognised
in the statements of profit or loss or in the statement of other comprehensive income. For
example:
 Income includes gains on the disposal of non-current assets. These are recognised
in the statement of profit or loss.
 Income also includes unrealised gains which occur whenever an asset is revalued
upwards, but is not disposed of. For example, an unrealised gain occurs when a
property owned by the entity is revalued upwards. Unrealised gains might be
recognised in the statement of profit or loss (e.g. revaluation gains on property
accounted for under IAS 16) or in the statement of other
comprehensive income (e.g. revaluation gains on property accounted for under the
IAS 40 fair value model).
Expenses
Expenses are decreases in assets, or increases in liabilities, that result in decreases in equity,
other than those relating to distributions to holders of equity claims.
Expenses include:
Expenses arising in the normal course of activities, such as the cost of sales and other operating
costs, including depreciation of non-current assets. Expenses result in the outflow of assets (such
as cash or finished goods inventory) or the depletion of assets (for example, the depreciation of
non-current assets).
Losses include for example, the loss on disposal of a non-current asset, and losses arising from
damage due to fire or flooding. Losses are usually reported as net of related income.

Financial performance is measured by profit or loss and gains or losses recognised in other
comprehensive income. Profit is measured as income less
expenses.

RECOGNITION AND DERECOGNITION

Recognition
This is covered by chapter 5 of The IASB Conceptual Framework.
Recognition is the process of capturing for inclusion in the statement of financial position or the
statement(s) of financial performance an item that meets the definition of one of the elements of
financial statements.
Recognition involves depicting the item in words and by a monetary amount.
The amount at which an asset, a liability or equity is recognised in the statement of financial
position is referred to as its carrying amount.
Recognition links the elements as the recognition of one item (or a change in its carrying
amount) requires the recognition or derecognition of another item. For example, revenue is
recognised at the same time as the corresponding receivable.

Recognition criteria
Only items that:
 meet the definition of an asset, a liability or equity are recognised in the statement of
financial position; or

383
 meet the definition of income or expenses are recognised in the statement(s) of financial
performance.
However, not all items that meet the definition of one of those elements are recognised.
An asset or liability is recognised only if recognition of that asset or liability and of any
resulting income, expenses or changes in equity provides users of financial statements with
information that is useful, i.e. with:
 relevant information about the asset or liability and about any resulting income,
expenses or changes in equity; and
 a faithful representation of the asset or liability and of any resulting income,
expenses or changes in equity.
Information about an asset or liability may not be relevant when there is uncertainty about its
existence or when there is only a low probability of an inflow or outflow of economic benefits in
respect of that asset or liability.
Whether a faithful representation can be provided may be affected by the level of measurement
uncertainty associated with the asset or liability or by other factors.

Commentary on the new recognition criteria


Under the previous framework, an asset or liability would be recognised when:
 it meets the definition of an element: and
 satisfies the following two criteria:
 it must be probable that the future economic benefit associated with the item will
flow either into or out of the entity; and
 The item should have a cost or value that can be measured reliably.
The IASB‟s deliberations on this and other projects have led them to the conclusion that the
probability of an inflow or outflow is not a recognition attribute but a measurement attribute.
The practical impact of the change in focus of the criteria will be negligible but is believed to
provide a stronger conceptual foundation to the recognition process.

Derecognition
Derecognition is the removal of all or part of a recognised asset or liability from an entity‟s
statement of financial position.
This normally occurs when that item no longer meets the definition of an asset or of a liability:
 for an asset, derecognition normally occurs when the entity loses control of all or part of
the recognised asset; and
 for a liability, derecognition normally occurs when the entity no longer has a present
obligation for all or part of the recognised liability.

ACCOUNTING CONCEPTS
Consistency of presentation
Materiality and aggregation
Offsetting
Consistency of presentation
Other accounting concepts are used in financial reporting in addition to those explained in the

384
IASB Framework.
Consistency of presentation is needed if financial information is to be comparable. IAS 1 states
that there should be consistency in the presentation and classification of items in the financial
statements from one year to the next.
There are just two exceptions to the requirement for consistency
 Consistency is not required when it is apparent, following a significant change in the
entity‟s operations or a review of its financial statements, that a different presentation or
classification would be more appropriate.
 Consistency is not appropriate if a new accounting standard (or the interpretation of a
Standard by IFRIC) requires a change in the presentation of information.
Materiality and aggregation
IAS 1 also states that each material class of similar items should be presented separately in the
financial statements.
In addition, items of a dissimilar nature should not be aggregated together in the financial
statements (combined as a single item and in a single total), unless their value is immaterial.

Offsetting
IAS 1 states that:
 Assets and liabilities should not be offset against each other.
 Similarly incomes and expenses should not be offset against each other.
Instead they should be reported separately.
The exceptions to this rule are when:
 offsetting is required or permitted by an accounting standard or the Interpretation of a
standard
 offsetting reflects the economic substance of a transaction. An example specified in IAS
1 is reporting of a gain or loss on disposal of a non-current asset at sale value minus the
carrying value of the asset and the related selling expenses.

MEASUREMENT AND CAPITAL MAINTENANCE


Section overview
 Measurement of elements of financial statements
 Fair value
 Capital maintenance concepts

Measurements of elements of financial statements


The Conceptual Framework allows that several measurement bases are used for the elements in
financial statements. These include:
 Historical cost. Assets are measured at the amount of cash paid, or at the fair value of the
consideration given to acquire them. Liabilities are measured at:
 the amount of proceeds received in exchange for the obligation (for example,
bank loan or a bank overdraft), or
 the amount of cash that will be paid to satisfy the liability.
 Current cost or current value is the basis used in current value accounting/current cost
accounting. Assets are measured at the amount that would be paid to purchase the same
or a similar asset currently.

385
Liabilities are measured at the amount that would be required to settle the obligation
currently.
 Realisable value (or settlement value). This method of measurement is relevant when
an entity is not a going concern, and is faced with liquidation (and a forced sale of its
assets). Assets are measured at the amount that could be obtained by selling them.
Liabilities are measured at the amount that would be required to settle them currently.
 Present value. Assets might be measured at the value of the future net cash inflows that
the item is expected to generate, discounted to a present value. Similarly, a liability
might be measured at the discounted present value of the expected cash outflows that
will be made to settle the liability.
Historical cost is the most commonly used measurement basis. However, the other bases of
measurement are often used to modify historical cost. For example, inventories are measured
at the lower of cost and net realisable value.
Deferred income is measured at present value. Some non-current assets may be valued at
current value.
The Framework does not favour one measurement base over the others.
Fair value
Fair value is a possible basis for the valuation of assets in the financial statements. Although it is
not described in the IASB Conceptual Framework, many IASs and IFRSs require it to be used
instead of historical cost or as an alternative to historical cost. For example, IFRS 9 requires
many types of investment to be measured at fair value.
Fair value may be used in financial statements in the following circumstances:
 After its initial recognition at acquisition, a non-current asset may be revalued to its
fair value.

 Inventory is measured in the statement of financial position at the lower of cost or


net realisable value. Net realisable value (NRV) is the selling price of the inventory
item in the ordinary course of business, less the estimated further costs to completion
and the expected selling costs. NRV may or may not be the same as fair value.
Fair value is often approximately the same as current value, but sometimes fair value and current
value can be very different.

Problems with the use of fair value


Fair value is easy to understand and less complicated to apply than value to the business/current
value. Arguably, it is also more reliable than value to the business, because market value is more
easily verified than (for example) economic value. However, it has some serious disadvantages:
 There may not be an active market for some kinds of asset. Where there is no active
market, estimates have to be used and these may not be reliable.
 It anticipates sales and profits which may never happen (the entity may have no plans to
sell the asset).
 Market values can move up and down quite rapidly. This may distort trends
in the financial statements and make it difficult for users to assess an entity‟s
performance over time.
A notable example of this problem occurred during 2007 and 2008 with the collapse of the
market for certain types of asset-backed securities (mortgagerelated securities known as
CDOs). Many banks, particularly in the US and Europe, announced huge losses, due largely

386
to the requirement to write down their investments in these financial instruments to fair
value, even though fair value was difficult to assess.
Despite these problems, it looks increasingly likely that the IASB will require greater use of
fair value in future.
The IASB Conceptual Framework states that there are two concepts of capital
 a financial concept of capital;
 a physical concept of capital.
Different systems of accounts used different capital maintenance concepts. The choice of
capital maintenance has a profound effect on the measurement of profit.
Consider the basic accounting equation.
Formula Accounting equation
Assets = Liabilities + Equity or Assets - Liabilities = Equity
A = L + E A - L = E
Net assets
The accounting equation is an equation. Therefore, changes in one side are matched by
changes in the other side.
Profit or loss for a period can be calculated from the difference between the
opening and closing net assets after adjusting for any distributions during the period.

Formula: Profit
Change in equity = Closing equity - Opening equity
Increase in equity = Profit + capital introduced - distributions
Profit = Increase in equity - capital introduced + distributions
This shows that the value ascribed to opening equity is crucial in the measurement of profit.

Financial capital maintenance


With the financial concept of capital maintenance, a profit is not earned during a period
unless the financial value of equity at the end of the period exceeds the financial value of
equity at the beginning of the period (after adjusting for equity capital raised or distributed).
Historical cost accounting is based on the concept of money financial capital maintenance.
Under this concept, an entity makes a profit when its closing equity exceeds its opening
equity measured as the number of units of currency at the start of the period. Note that this is
a separate issue from asset valuation.
Assets could be revalued during the period but this would have no effect on the opening
capital position.
An alternative view of financial capital maintenance is used in constant purchasing power
accounting. This system is based on the concept of real financial capital maintenance.
Under this concept, an entity makes a profit when its closing equity exceeds opening equity
remeasured to maintain its purchasing power.
This requires the opening equity to be uplifted by the general inflation rate. This is achieved
by a simple double entry.
Illustration: Adjustment to maintain opening equity
Debit Credit
Statement of profit or loss X
Inflation reserve X

387
Physical capital maintenance
A physical concept of capital is that the capital of an entity is represented by its productive
capacity or operating capability. Where a physical concept of capital is used, the main concern of
users of the financial statements is with the maintenance of the operating capability of the entity.
With a physical concept of capital maintenance, a profit is not earned during a period unless
(excluding new equity capital raised during the period and adding back any distribution of
dividends to shareholders) the operating capability of the business is greater at the end of the
period than at the beginning of the period.
This requires the opening equity to be uplifted by the specific rates of inflation that apply to the
individual components of the net assets of the company. Again, this is achieved by the same
simple double entry.

THE INTERNATIONAL ACCOUNTING STANDARDS BOARD (IASB)


The organizational structure consists of:
(a) The IASC Foundation
(b) The IASB
(c) The Standards Advisory Council (SAC)
(d) The International Financial Reporting Interpretations Committee
The International Accounting Standards Board is an independent, privately-funded accounting
standard setter based in London.In March 2001 the IASC Foundation was formed as a
not-for-profit corporation incorporated in the USA. The IASC Foundation is the parent
entity of the IASB.From April 2001 the IASB assumed accounting standard setting
responsibilities from its predecessor body, the International Accounting Standards
Committee (IASC). This restructuring was based upon the recommendations made in the
Recommendations on Shaping IASC for the Future.The 14 members of the IASB come
from nine countries and have a variety of backgrounds with a mix of auditors, preparers
of financial statements, users of financial statements and an academic. The Board consists
of 12 full-time members and two part-time members.
Objectives of the IASB
The formal objectives of the IASB, formulated in its mission statement are:
(a) To develop, in the public interest, a single set of high quality, understandable and
enforceable global accounting standards that require high quality, transparent and
comparable information in general purpose financial statements
(b) To provide the use and vigorous application of those standards
(c) To work actively with national accounting standard setters to bring about
convergence of national accounting standards and IFRS to high quality solutions.
Structure of the IASB
The structure of the IASB has the following main features:
(a) The IASC Foundation is an independent corporation having two main bodies –
the Trustees and the IASB. The IASC Foundation holds the copyright of IFRSs and
all other IASB publications.
(b) The IASC Foundation trustees appoint the IASB members, exercise oversight and
raise the funds needed.
(c) The IASB has sole responsibility for setting accounting standards.
(d) There are also two further bodies, the Standards Advisory Council and the
International Financial Reporting Interpretations Committee(see below)

388
Trustees – The Trustees comprise a group of twenty two individuals, with diverse
geographic and functional backgrounds. The Trustees appoint the Members of the
Board, the International Financial Reporting Interpretations Committee and the
Standards Advisory Council. In addition to monitoring IASC's effectiveness and
raising its funds, the Trustees will approve IASC's budget and have responsibility
for constitutional changes.
Trustees were appointed so that initially there were six from North America, six from Europe,
four from Asia Pacific, and three others from any area, as long as geographic balance is
maintained.
(a) The International Federation of Accountants (IFAC) suggested candidates to fill
five of the nineteen Trustee seats and international organisations of preparers,
users and academics each suggested one candidate.
(b) The remaining eleven Trustees are 'at-large' in that they were not selected through
the constituency nomination process.
Standards Advisory Council – The Standards Advisory Council provides a formal vehicle for
further groups and individuals with diverse geographic and functional backgrounds to
give advice to the Board and, at times, to advise the Trustees. It comprises about fifty
members and meets at least three times a year. It is consulted by the IASB on all major
projects and its meetings are open to the public. It advises the IASB on prioritisation of its
work and on the implications of proposed standards for users and preparers of financial
statements.
International Financial Reporting Interpretations Committee –The IFRIC provides timely
guidance on the application and interpretation of International Financial Reporting
Standards. It deals with newly identified financial reporting issues not specifically

389
addressed in IFRSs, or issues where unsatisfactory or conflicting interpretations have
developed, or seem likely to develop.
Setting of IFRSs
IFRSs are developed through a formal system of due process and broad international
consultation involving accountants, financial analysts and other users and regulatory
bodies from around the world.
The Standard Setting process
The overall agenda of the IASB will initially be set by discussion with the Standards Advisory
Council. The process for developing an individual standard would involve the following
steps.
Step 1 During the early stages of a project, IASB may establish an
Advisory Committee to give advice on issues arising in the
project. Consultation with the Advisory Committee and the
Standards Advisory Council occurs throughout the project.
Step 2 IASB may develop and publish Discussion Documents (DD)
for public comment.
Step 3 Following the receipt and review of comments, IASB would
develop and publish an Exposure Draft (ED) for public
comment.
Step 4 Following the receipt and review of comments, the IASB would
issue a final International Financial Reporting Standard.

The period of exposure for public comment is normally 90 days. However, in exceptional
circumstances, proposals may be issued with a comment period of 60 days. Draft IFRIC
Interpretations are exposed for a 60 day comment period.

Developing a new standard


The development of a new or revised accounting standard involves widespread consultation
and discussion.
 A subject is identified as being appropriate for a new or revised standard.
 An advisory group is established to give advice to the IASB.
 A discussion document is issued by the IASB for public comment.
 After receiving comments on the discussion document, the IASB issues an
Exposure Draft (provided it is approved by at least 8 IASB members.) The Exposure Draft also
includes the opinions of any dissenting IASB members, and the basis for the IASB‘s conclusions.
 All comments on the Exposure Draft and discussion documents are considered.
 An approved IFRS is published (provided it is approved by at least 8 IASB members.)
This will also include the opinions of any dissenting IASB members, and the basis for
the IASB‘s conclusions. In some cases, a revised Exposure Draft has been issued, when
the deliberations following the original ED indicate that there will be substantial
changes from what was originally envisaged
 Each new or revised standard has a date for implementation.

Interpretations of IFRSs
An Interpretation of an IFRS might be developed by IFRSIC. The purpose of an Interpretation is
to give guidance and clarification on issues where an IFRS is not clear, and so the interpretation
is uncertain.

390
 The need for an Interpretation might become apparent after a new IFRS has been issued.
 IFRIC publishes a draft Interpretation for comment.
 After considering the comments received, IFRIC approves an Interpretation.
This interpretation is submitted to the IASB for approval. If approved by at least 8 IASB
members, the Interpretation is published.

Specialised, not-for-profit and public sector entities


Types of entity
Most of our discussion is about the financial statements of profit-making entities, such as
limited liability companies.
Other types of entity also prepare and publish financial statements. These entities include:
 Not-for-profit entities: such as charities, clubs and societies. Each of these organisations
is set up for a specific purpose. For example, a charity might be set up to campaign for
the protection of the natural environment or to help the poor.
 Public sector entities: these include central government bodies; local government
bodies; and other organisations that operate for the benefit of the general public, such as
state schools and hospitals. A public sector entity is owned by the state or by the general
public. Many different types of entity could be described under these headings. These
entities are different from limited liability companies, partnerships and sole traders in
one vital respect. They do not primarily exist to make a profit.In practice, the terms
‗specialised entity’, ‗not-for-profit entity’ and ‗public benefit entity’ are often used
interchangeably.

Objectives of specialised entities


The main objective of large commercial entities is to maximise their profits in order to provide a
return to their owners (investors) in the form of a dividend. This may not be their only objective
(for example, they may provide employment to the local community, or aim to operate in a
socially responsible way), but it is their main objective.
The objective of owner-managed businesses (small privately owned entities) is also to make a
profit.
In contrast, the main objective of a specialised entity is to carry out the activities for which it has
been created. Again, this may not be the only objective, because all entities need some form of
income. Many large charities, for example, carry out trading activities. However, making a
profit is not the main aim. In fact, most notfor- profit entities will aim to break even, rather than
to generate a surplus of income
over expenditure.
Accounting standards and specialised entities
International accounting standards are designed for profit-making entities.
Whether they are relevant to not-for-profit entities will depend on the way in which these
entities have to report and the information that they have to provide. There is a great deal of
variation from organisation to organisation and from country to country.
In some countries, charities and public sector bodies are required to follow accounting
standards specifically designed for the purpose (in the UK these are called Statements of
Recommended Practice.) Alternatively, the form and content of financial statements and the
accounting treatments to be followed may be prescribed by law. IASs and IFRSs are probably
largely irrelevant for these entities.

391
Some not-for-profit entities may be able to draw up financial statements in any form that its
members or officers wish. Many not-for-profit entities prepare financial statements on a cash
basis, rather than on an accruals basis. Public sector bodies may also use ‗cash accounting‘. (This
was the case in the UK until fairly recently.)
IASs and IFRSs require accruals accounting.
In some countries, public sector bodies and many charities are increasingly expected to apply
commercial-style accounting practices. Even for those entities that are not formally required to
adopt them, IASs and IFRSs are a useful source of information on current best practice. There is
an International body under the auspices of IFAC that develops and publishes International
Public Sector Accounting Standards (IPSASs), which are broadly consistent with the related
IFRSs

THE NEED FOR A REGULATORY FRAMEWORK


The regulatory framework is the most important element in ensuring relevant and reliable
financial reporting and thus meeting the needs of shareholders and other users.
Without a single body overall responsible for producing financial standards (the IASB) and a
framework of general principles within which they can be produced (the Framework),
there would be no means of enforcing compliance with GAAP. Also, GAAP would be
unable to evolve in any structured way in response to changes in economic conditions.
The IASB – International Accounting Standards Board, (Formerly IASC), issued its Framework
for the Preparation and Presentation of Financial Statements in 1989. This is referred to as its
conceptual framework. The framework sets out the concepts that shape the preparation and
presentation of financial statements for external users. The framework does not have the status
of an accounting standard, as also is the case with the ASB‘s Statement of Principles.

The IASB framework assists the IASB:


• In the development of future International Accounting Standards and in its review of existing
International Accounting Standards; and
• In promoting the harmonisation of regulations, accounting standards and procedures relating
to presentation of financial statements by providing a basis for reducing the number of
alternative accounting treatments permitted by International Accounting Standards.

In addition, the framework may assist:


• Preparers of financial statements in applying International Accounting Standards and in
dealing with topics that have yet to form the subject of an International Accounting
Standard;
• Auditors in forming an opinion as to whether financial statements conform with International
Accounting Standards;
• Users of financial statements in interpreting the information contained in financial statements
prepared in conformity with International Accounting Standards; and
• Those who are interested in the work of IASB, providing them with information about its
approach to the formulation of accounting standards.‖

To ensure the framework provides useful information it identifies a range of user groups, which
include:
• Investors
• Lenders

392
• Employees
• Suppliers
• Other trade creditors
• Customers
• Government agencies; and
• The public

The framework comprises seven sections which cover areas as:


(1) The objective of financial statements;
(2) Underlying assumptions;
(3) Qualitative characteristics of financial information;
(4) The elements of financial statements;
(5) Recognition of the elements of financial statements;
(6) Measurement of the elements of financial statements;
(7) Concepts of capital maintenance.

Objectives of financial statements

Published financial statements should provide information to a wide range of users informing
a:
• Financial position
• Financial performance; and
• Changes in financial position.

Financial statements should also portray the results of stewardship of management and the user
should be able to assess the accountability of management, which may influence the users‘
decision making process.
Information on financial position is primarily provided in a balance sheet, that of performance
in the income statement and changes in financial position through the presentation of a cash
flow statement (IAS7). IAS1 also requires a statement of changes in equity.
The statements should be viewed as a whole, for example an income statement provides an
overview of performance that is incomplete without its link with the balance sheet.
It is important that financial statements should show corresponding information for preceding
periods.
The statements should also be supplemented with notes and schedules to provide additional
information relevant to the user.

UNDERLYING ASSUMPTIONS
The framework identifies two underlying assumptions:
• Accruals basis
• Going concern

In order to meet their objectives financial statements are prepared on an accruals basis.
Transactions and events are simply recognised at the point when they occur, rather than when
cash or its equivalent is received or paid, and are reported in the financial statements of the
periods to which they relate.

393
Financial statements are usually prepared on the assumption that the reporting entity is a going
concern and is likely to operate for the foreseeable future. The assumption here is that the
reporting entity has no intention to liquidate or to adversely curtail its scale of activities. If this
is the case then a different basis of reporting may be necessary and the basis disclosed.

ALTERNATIVE MODELS & PRACTICES


It is possible to have an ‗alternative-bases‘ approach to financial reporting measurement, ie, to
choose one basis of measurement in some cases and a different one in others. The particular
approach that we consider employs current market prices for some items in accounts, but
historical cost (or recoverable historical cost) for most. This approach is perhaps of particular
interest, partly because it resembles the IASB ‘s current approach to the measurement of
financial instruments, a highly controversial issue.

The alternative-bases approach can be summarised as follows:


The use of market prices to measure assets and liabilities in financial reporting should be
restricted to those cases in which the firm‘s business model adds no value to assets. It is based
on the two key points for financial reporting that seem to arise from the theory of the firm:
market prices are unlikely to be either available or relevant for assets that are being used or
developed within the business, and to
which the firm is, therefore, adding value. Where the firm is not adding value, market prices are
likely to be both available and highly relevant.

QUESTIONS 1
A). Mrs. Stella Amoah (Stella), a Chartered Accountant and Head of Internal Audit in Ningo
Communications Authority (NCA) is about to tender for a contract in Internal Audit Service. A new
member of her team, Mr. Stephen Appiah Coker (Stephen) has been recruited from the Internal Audit
Service, who previously worked in the department responsible for devising the tender contract.
Although Stephen was not involved with the tender process, his former colleague and friend is
responsible for the tender specification document and the evaluation process.
Stephen had sight of some of the requirements and has offered to share with Stella information that
may be of use when preparing the tender. However, this information is confidential and should not be
seen by any of the tendering parties.
It will be an open tender process for both external and internal providers. Bids from external
providers are being encouraged. The evaluation process has been designed with this in mind. If the
contract is awarded externally, Stella will be unsure of her personal position in the organisation. She
understands the use of any insider knowledge of the tendering process would be inappropriate when
preparing the tender proposal, but she feels she would have a better chance of success if she used this
confidential information.
Required:
Advise Mrs. Stella Amoah on THREE (3) courses of action she should take in order to act ethically
in the tendering process given in the above scenario. (6 marks)

b) Amankwatia Ltd (Amankwatia) is a local construction company. The regulation in the


construction sector requires employers to provide personal protective equipment for every employee.
The company failed to do that, and a Plumber got involved in an accident in the course of work
resulting in a serious and costly injury. The Plumber has sued the company. The Solicitors of the
company have prepared to vigorously defend the company in the lawsuit. They estimated that the

394
company would have to make a compensation of GH¢17,000 to cover the injured party‟s costs. A
court decision, however, is not expected for at least a year.
Required:
What aspects of the conceptual framework might help you in determining the appropriate accountin
treatment for this situation? (4 marks)
SOLUTION 1
a) Suggested course of action

her tender document. Although there is potential personal gain from using the evaluation
information, she should not refer to it in the proposal.
to the member of staff who is making the information available that
his offer of assistance cannot be acted upon as this would be in breach of IFAC‘s code of
ethics.

should explain to the tendering department if she has been made aware of any additional
information so that they can make it available to the other bidders.

information and that he should inform his former colleague of this. She does not want to be
subject to any rumour that she had sight of any evaluation documentation as this could
jeopardize her tender proposal.
onstrating a level of expected
ethical behaviour to the department.
(3 points @ 2 marks each = 6 marks)

b) The definition of liability can help decide the accounting treatment of the situation.
Under the conceptual framework a liability is a present obligation of the entity arising from
past events, the settlement of which is expected to result in an outflow from the entity of
resources embodying economic benefits. In this case, the past event is the fall and injury to
the pedestrian.

Present obligation depends on the probability of payment. The solicitors has advised that a
GH¢17,000 loss is probable. Therefore appropriate accounting involves recognizing a
liability for the probable payment. An expense would also be recognized.

QUESTION 2
a)Define „equity’, and explain why the conceptual framework does not prescribe any recognition
criteria for equity. (4 marks)

b) Mr. Charles Agyekum is a qualified ICAG member who prepares accounts on behalf of a small
independent trader. An annual practicing certificate is not required.
This is the first year the member has prepared these accounts. When compiling the most recent
accounts, he noticed that some errors were noted in the previous accounts. It appeared that the
accounts were based on incomplete records as certain costs were excluded, either intentionally or
because records were not maintained.
The client has also requested some additional work to be completed on a complex tax issue.
However, the member has no prior experience and does not feel competent to do the work. The client

395
would also like him to provide an audit opinion as they are planning to apply for a bank loan and the
bank would like some additional assurance.
Required:
In accordance with IFAC‟s code of ethics, explain which ethical principles apply and comment on
their relevance to the above scenario.
SOLUTION 2
a) The conceptual framework defines equity as ‘the residual interest in the assets of the entity
after deducting all its liabilities’.
Equity cannot be identified independently of the other elements in the statement of financial
position/balance sheet. The characteristics of equity are that equity is a residual, i.e. something left
over after the entity has determined its assets and liabilities. In other words: Equity = Assets –
Liabilities.
There is no need for recognition criteria for equity as it is a residual, determined after recognition
criteria are applied to the other elements. In other words, the recognition of assets and liabilities
will lead to recognition of equity.
Definition of equity 2 marks
Explanation of reason for equity recognition 2 marks
b) Integrity – this is about being truthful, straightforward and honest, dealing fairly with people and
situations; it rules out making misleading or false statements, whether by omission or inclusion of
information, either knowingly or without taking care to find out. Mr Charles Agyekum has
highlighted his concerns to his client and explained his views with a clear rationale.

Professional competence and due care – this is about acquiring and maintaining appropriate
technical and other relevant skills and competence to perform our work, doing it thoroughly and
correctly, on a timely basis, and ensuring that users of our output understand its context and
limitations. Mr Charles Agyekum has acknowledged his professional ability and has identified a
situation where he may not be the most appropriate accountant to complete a specific piece of work.
Professional behaviour – this is about complying with standards and laws, and avoiding actions that
might bring the profession into disrepute, such as making unsubstantiated criticisms of a fellow
professional, or exaggerating one‟s experience. Throughout this scenario, Mr Charles Agyekum has
behaved professionally as he has explained his rationale to his client and not completed any work
that he was unable to finish to an appropriate standard.

QUESTION 3
Mr. Julius Opuni, a Chartered Accountant has been appointed as Chairman of a Senior High School
governing body on a voluntary basis. He has also been appointed to the Finance and Buildings
Committee that awards building contracts. The membership of this committee includes a number of
individuals with private sector experience and local businessmen. One is a local contractor (Mrs.
Esther Asamoah-Frimpong) who has been a governing member for a number of years and is well
respected by the governing body.

At his first meeting, the committee considers a report from the Head Teacher about the condition of
the school hall and sets out a scheme of remedial building works with estimated costs. After
discussion of the scheme, and recognising the need to move quickly if the work is to be carried out
during the first quarter, Esther offers to do the work at a competitive price and the other governing
members on the committee are minded to accept the offer.

However, although the offer has been made, the governors are not considering the use of a formal
tender process or making any reference to governance arrangements that could exist for tenders.

396
Julius is concerned that the committee is unable to demonstrate reasonable decision making,
stewardship of public money and its subsequent reputational risk.

Required: Advise Julius on THREE (3) courses of action he should take in order to act ethically in
awarding the contract. (6 marks)

b) Tenet Bank Ltd places a non-cancellable order for a new Automated Teller Machine (ATM) with
one of the major commercial ATM manufacturers at a fixed price, with delivery in 30 months and
payment in full to be made on delivery.

Required: Under the conceptual framework, advise whether Tenet Bank should recognise any asset
or liability at the time it places the order. (3 marks)

SOLUTION
Mr. Julius Opuni should make his concerns known and explain to the committee members why he feels
acceptance of the builder governor‟s offer could be inappropriate.

process. This would ensure that any tenders awarded would be subject to a proper process with integrity.
to demonstrate a proper decisionmaking process that
would support any contracts awarded. This would also protect the governors from any potential
reputational risk that the school did not properly award contracts, especially as it is funded by public
money. (3 points @ 2 marks each = 6 marks)

b) Under the IASB Conceptual Framework, Network Bank Ltd should not recognise any asset or liability
at the time it places the order, because the transaction has not taken place. There is no obligation arising
from past events. The Framework recognises purchase transactions when delivery takes place, and title
passes. At this point the bank, and not the manufacturer, has assumed the risks and rewards of owning the
ATM machine.
Nonetheless, the bank has made an important and irrevocable commitment. Generally, major
capital spending commitments are disclosed in the notes to the financial statements

EXAMS QUESTIONS

QUESTION 1
Goodman recently qualified as accountant with the Institute of Chartered Accountants
(Ghana). He works with a manufacturing company in Tamale, Ghana, and he has been
asked, by his line manager, to complete a costing exercise and given a very short
deadline as well as limited resources for the exercise. Goodman thinks that the
President of the company is planning to use this information to restructure the company,
including making some of Goodman‟s close colleagues redundant. Goodman is very
worried that the outcome of his work cannot be robust enough to be used for such a big
business decision by the company, but his line manager is putting him under a lot of
pressure to complete the work pretty much quickly.
Required:
Evaluate FOUR (4) ethical issues facing Goodman and recommend FOUR (4) possible courses
of action Goodman should consider taking. (10 marks)

SOLUTION

397
Fundamental principles
Integrity
Goodman could be honest and straightforward with the line manager and the president of the
company about the short time and limited resource to do a very robust work capable of meeting
the big business decision it is intended to be used for. Can Goodman realistically produce costing
information with the time and resources available, without compromising the standard of his
work?
Objectivity
Goodman could maintain an unbiased stance throughout the assignment given him, in view of
his close relationship with other colleagues in the company. Goodman must not allow conflict of
interest, bias and undue influence or pressure to influence his professional judgement. For
example, his line manager is putting him under a lot of pressure to complete the work pretty
much quickly.
Professional competence and due care
The company may be restructuring, and the president needs to have the most up to date and
complete financial information to inform any big business decisions. As a professional
accountant, you must ensure that any financial information you provide is robust. Can Goodman
realistically produce a costing information, with the time and resources available, without
compromising the standard of his work?
Confidentiality
Given the sensitivity of the situation, Goodman should maintain discretion and not share your
concerns with other staff, who may not be aware of the president‟s intentions. Is there anyone
else in the company with whom Goodman can raise his concerns? Is there a senior finance
officer who could advise Goodman, or another member of the board with whom Goodman can
discuss his dilemma?
(4 ethical issues well explained @ 1.5 marks each = 6 marks)
Possible courses of action

asked to produce to restructure the company. As a professional accountant, Goodman has a duty
to make his line manager and other users of the information aware of the limitations in the scope
of his work.

manager and the president of the company. He should arrange a meeting with his line manager
and explain that he is unwilling to do the work to the deadline requested, with the resources
available, because the work could not be relied upon. The process of clarifying the intended use
of the information and expressing his concerns regarding its reliability is likely to enhance your
credibility.

the work to be outsourced altogether. This would have the added benefit of enhanced objectivity.
sue with the president or
other members of the board, as appropriate. If his head of department is unsympathetic to his
concerns, he should not allow himself to be associated with information that may be misleading.
opriate way in which to make his concerns known to
the board. This may be through the president or the company secretary.

398
time pressure, he may have to make clear his refusal to conduct the work, and possibly resign
from the company.

case his ethical judgement is challenged in future period.


(1 mark for each action taken up to a maximum of 4 marks)

QUESTION 2
You have just obtained your full membership with the Institute of Chartered Accountants
(Ghana). Following this successful achievement, you have been appointed as the Head of
Finance at Asasiyemedeh Company Limited, a Ghanaian company, which provides catering
services. Your former employer Akwaba Limited is a large public sector organization operating
in Accra, where, as the Financial Accountant, you had the opportunity to work on areas relating
to financial accounting, procurement, contracts and bids. One of Asasiyemedeh Company
Limited‟s major contracts is with Akwaba Limited, your former employer. The contract is now
due for renewal, and Asasiyemedeh Company Limited is preparing a competitive bid for this
contract.
You have been tasked to lead the team responsible for bidding for this contract, but you are
concerned as a professional that you might breach confidentiality if you accept this role. You
also suspect that your knowledge and experience of Akwaba Limited were seen as good reasons
for appointing you to the position of Head of Finance at Asasiyemedeh Company Limited. You
do not in any way want to let your new employer down as you are aware that the loss of such a
major contract would have a significant effect on the financial performance of Asasiyemedeh
Company Limited, and its performance-related bonus scheme for management members.
Required:
i) Discuss the ethical issues raised in the above scenario. (5 marks)

ii) Recommend the possible courses of action that you will take in order to be ethically
responsible as expected from a Professional Accountant. (5 marks)

SOLUTIONS
i) Fundamental principles
Objectivity
Clearly there is a self-interest threat that arises because of the impact that losing Akwaba
Limited‟s contract would have on Asasiyemedeh Company Limited‟s financial performance and
reward policy. There is also intimidation threat as other employees company may be affected due
to the financial implications of the contract not being renewed. You may also be feeling that you
would like to impress your new employer and help to make a successful bid for the renewal of
the contract, which may be normal. However, the most in important question here is that “can
you safeguard against the significant self-interest threat which arises from Asasiyemedeh
Company Limited‟s performance-related bonus scheme? Confidentiality
Clearly there is a confidentiality threat here as you have worked with Akwaba Ltd in the past.
Your previous employment with Akwaba Ltd has provided you with information which may be
of value to Asasiyemedeh Company Limited.

399
The principle of confidentiality prohibits the use of confidential information acquired as a result
of your previous employment for your advantage or that of your current employer. While you
have a responsibility to advance the legitimate aims of your employing organization, this should
not extend to a breach of confidentiality. In this case, you (because of Asasiyemedeh Company
Limited‟s performance-related bonus) and Asasiyemedeh Company Limited stand to benefit
from the confidential information about how bids are assessed at Akwaba Ltd. The principle
would not be breached if you were in possession of information that was in the public domain, or
if you were simply to use experience gained in your previous employment, so long as you do not
use confidential knowledge that you acquired as a result of that employment.
If you accept this role, can you ensure that you do not use confidential information relating to
your former employer to your advantage or to the advantage of your current employer? You
must be careful and professional as winning that contracts may leads to confidential breaches
against you or your current employers perhaps from those bidders of the same contracts who
might lose the bids.
Professional behavior
You must demonstrate professionalism here. For example, what can you do to safeguard your
reputation as a professional, the reputation of your employer, and the accountancy profession to
which you belong? You must consider the Institute of Chartered Accountants (Ghana) code of
ethics, applicable laws (procurement Act 914) and regulations, your current and previous
contracts of employment, and your employer‟s policies and procedures.
(Any 2 ethical issues discussed @ 2.5 marks each = 5 marks)
ii) Possible courses of action

place, and ask for your involvement in the preparation of the contract bid to be limited. For
example, you may be able to contribute to aspects of the bid that do not in any way require you
to refer to confidential knowledge about your previous employment with Akwaba Ltd.

in your profession, you should request that you both discuss the matter with the board chairman
or other member of staff. During these discussions, you should refer to the company‟s ethical
code, if it has one, as well as that of the Institute of Chartered Accountants (Ghana).
other formal channels available, you should make the entire board aware of
your dilemma by writing formally to them. If necessary, you must refuse to take part in the bid
without necessary safeguards being implemented.
from Asasiyemedeh Company Limited may be the only
solution. However, before taking such a step, you should seek legal advice on your employment

Accountants, (Ghana)).

your ethical judgement is challenged in future period.

appropriate to suggest to managing director or the board of your employer that a policy on
conflicts of interest be developed and that the remuneration and bonus policy be reviewed in
light of this contract bid with Akwaba Ltd.
(1 mark for each action taken up to a maximum of 5 marks)

QUESTION 3

400
Fiagja Ltd is a retail trading company in Ghana. Nana Yaw Kawula (member of ICAG) is
the finance director and has been in this role for many years. Fiagja Ltd has a year-end
of 30 June each year. Nana Yaw Kawula is finalizing the financial statements for the
year ended June 30 2019.
On one hand, the warehouse manager of Fiagja Ltd has recently advised Nana Yaw
Kawula of a significant level of slow moving inventory, and that the inventory in question
is now more than seven months old and per the company policy would usually have
been written down some months previously.
On the other hand, the shareholders of Fiagja Ltd are trying to sell the company, and
the Chief Executive Officer (CEO) who happens to be the majority shareholder of Fiagja
Ltd has told Nana Yaw Kawula that it is not necessary to write down the inventory
values in the yearend financial statements.
Nana Yaw Kawula is sure that the CEO wants the financial statements to carry an
inflated inventory valuation because he has found a prospective buyer for the company.
The CEO has indicated to Nana Yaw Kawula that, if the proposed deal is indeed
successful, all employees will keep their jobs (including Nana Yaw Kawula) and the
finance director (Nana Yaw Kawula) will receive a pay rise.
Required:
i) Explain how the finance director could potentially act in order not to breach the
fundamental principles of the IFAC‟s code of ethics. (5 marks)

ii) Recommend the possible actions that the finance director should take as a member
of the Institute of Chartered Accountant (Ghana) in dealing with this ethical dilemma. (5
marks)
SOLUTION

(i). Fundamental principles


Integrity: In the light of the information Nana Yaw Kawula have, he must ensure that he act
honestly, and that he is open and straightforward towards those with whom he comes into contact
with. The CEO proposes misrepresenting information about the company in the financial
statements, which would be contrary to the fundamental principle of integrity as defined for all
ICAG members.

Nana Yaw Kawula cannot simply do what has been asked of by the CEO because the principle
of integrity requires a professional accountant not to be associated with information that they
believe to be false or misleading. Relying on the potential buyer‟s due diligence to identify the
overvaluation is not appropriate. You are responsible for the honest presentation of the accounts,
and you should not transfer that responsibility to either the buyer or the auditors.
Objectivity: Can Nana Yaw Kawula act without bias, despite the significant threats in the
form of self-interest and intimidation from others like in this case. In this case, there is self-
interest threat to the finance director‟s objectivity, which arises from the financial benefit that
Nana Yaw Kawula is likely to receive if the company is sold under the proposed deal. Nana Yaw
is also feeling intimidated by the CEO. The CEO appears to be suggesting that the future
employment of other employees depends upon the proposed deal being successful and, therefore,
upon the results shown by the financial statements. The CEO is also putting the finance director
under pressure to account for inventory at a higher value than that with which he feels
comfortable.

401
Professional competence and due care: The finance director must act diligently. Does Nana
Yaw Kawula have sufficient information to be able to determine the appropriate value of the
inventory to be included in the financial statements in order not to misstate it? In this case, the
first step is to ensure that the finance director have sufficient information. This would include the
establishment of the basis of valuation of the company‟s inventory, investigating the system for
counting and evaluating inventory, and discussing with the warehouse manager the reason why
the inventory is slow moving. The finance director may also need to discuss the realisable value
with someone else, such as the sales director.
Professional behaviour: All members of ICAG including Nana Yaw Kawula are required to
account for the inventory in accordance with relevant accounting standards. In this case IAS 2
inventory. Inventory is valued at the lower of cost and net realizable value under IAS 2, which
the finance director should be comfortable with. The finance director must ensure that any of the
actions he considers taking should not discredit the profession in the opinion of an informed third
party.

Confidentiality: The finance director has to be mindful that actions he takes both internal and
external including consulting others does not amount to given information of the company
without obtaining proper permission

Suggested marking scheme: (1 mark for each brief discussion up to a maximum of 5


marks)
ii) Possible course of action that the finance director should take

include establishing the basis of valuation of the company‟s inventory, investigating the system
for counting and evaluating inventory, and discussing with the warehouse manager the reason
why the inventory is slow moving. You may also need to discuss the realisable value with
someone else, such as the sales director.

continues to insist on an inflated inventory valuation being incorporated into the financial
statements
issue with the other board members.
Initially, he could suggest that both Nana Yaw Kawula and the CEO raise the matter with the
other board members. If he feels it appropriate to discuss the matter with anyone else within the
company, however, the finance director must bear in mind the need for appropriate
confidentiality and be clear about his reasons for raising the matter.

ethics, if it has one. If it does not, the finance director should make the CEO aware of the ethical
requirements of ICAG as a professional body of which Nana Yaw Kawula is member.

to value the inventory.

other written correspondence to record his points of view. This would be particularly appropriate
if you are of the opinion that the CEO or the board has not been sympathetic to the finance
director‟s concerns.

402
ly, for example alerting
the auditors to the existence of the slow-moving inventory, or seeking advice from ICAG as
professional body, which the finance director belongs.
om the
conflict. The clearest way is to disassociate himself from misleading financial statements would
be to resign. However, this would only be an option to be exercised, as a last resort, in the most
extreme circumstances. Resignation alone would not help to resolve this kind of situation. It
would be advisable to take legal advice before considering resignation.

issue, in case his ethical judgement is challenged in future periods.

(1 mark for each action taken up to a maximum of 5 marks)

403
STATEMENT OF CASH FLOWS (IAS 7).
Objective
The users of financial statements also take into account the entity‘s cash generating ability and
cash needs to evaluate its liquidity position in order to take economic decisions as the entity
needs cash to carry on its operations, for payment of its liabilities and distributions of returns to
its investors.
This standard prescribe the guide lines, which require an entity to present information about its
historic cash flows and changes in those cash flows during the accounting period, to intimate
the users of financial statements about the cash generating ability and cash needs of the entity,
in the form of statement of cash flows by classifying such cash flows into operating, investing
and financing functions or activities.
Scope
The requirements of this standard are applicable for the preparation and presentation of
statement of cash flows which is presented as an essential component of the financial statements
in each accounting period.
Definitions
Cash
It encompasses currency notes, coins used as currency and short term deposits accessible on
demand.
Cash Equivalents
The short term investments which are highly liquid and are convertible in to identifiable
amount of cash within a period of three months or less, these have least chances of variation in
value, are termed as cash equivalents
 These are normally held by entity in order to meet its short term cash needs or
commitments rather than held for investment purposes
 These also include bank overdrafts which are held by the entity for the purpose of cash
management.
Cash Flows
The inflows and outflows in the normal conduct of the business, of cash and cash equivalents
are termed as cash flows.

Operating Activities
The principal business activities of the entity, which generate revenues for the entity are termed
as operating activities
Investing Activities

404
The activities which are undertaken by the entity, for the purchase of long term assets and
investments (which are not the part of cash equivalents), including the disposal of such long
term assets and investments are termed as investing activities.
Financing Activities
The activities which are undertaken by the entity to raise capital or long term funds for the
business, and which results in change in the equity and borrowed funds of the entity are termed
as financing activities
Presentation of Statement of Cash Flows
The entity is required prepare the statement of cash flows by classifying such cash flows into
operating, investing and financing activities. Classification of cash flows of the entity by activity
will enable the users of financial statements to understand the effect of each category of cash
flows upon the financial position of the business.
Operating Activities
The cash flows which are generated by the principal business activities of the entity are termed
as cash flows from operating activities. Cash flows from the operating activities reflects the cash
generating ability of the operations and the extent to which such cash flows can be used to carry
on operations, for the payment of liabilities, distribution to shareholders and for the acquisition
of new investments.
The following are the examples of cash flows from operating activities:
 Cash received related the sale of goods
 Cash received related to rendering of service
 Cash received related to royalty or commissions income
 Cash received related to the sale of investments, which are held for trading
 Cash paid or received by a financial institute for the grant and receipt of loan amount
 Cash received or paid by the insurance company in respect of for premiums and claims
 Cash paid to suppliers for purchase of goods or services
 Salaries Paid to employees;
 Any cash paid or received as a refund of income tax
Any cash received from disposal of a non-current asset is not the part of cash flows from
operating activities, instead it is included in cash flows from investing activities.
Investing Activities
The activities which are undertaken by the entity, for the purchase of long term assets and
investments (which are not the part of cash equivalents), including the disposal of such long
term assets and investments are termed as investing activities. Cash flow from investing
activities reflects the amount of expenditure made by the entity for the purchase of long term
assets to generate economic benefits for a long time period.
The following are the examples of cash flows from investing activities:
 Cash paid to purchase non-current assets (tangible and intangible both)
 Cash paid to purchase long term investments other those held for trading
 Cash paid for the capitalized development expenditure
 Cash paid for the self constructed asset
 Cash received from disposal of non-current assets (tangible and intangible both) and
long term investments
 Loans granted to other party (except loans granted by the financial institution)
 Cash received in respect of loan receivables
 Cash received as a result of government grant
 Interest and dividend income received on long term investments

405
Financing Activities
The activities which are undertaken by the entity to raise capital or long term funds for the
business, and which results in change in the equity and borrowed funds of the entity are termed
as financing activities. Cash flows from financing activities enable the users to evaluate the
finance structure of the entity.
The following are the examples of cash flows from investing activities:
 Proceeds received on issue of equity instruments such as ordinary shares
 Proceeds received on issue of loan notes, debentures or bonds
 Bank loan borrowed
 Repayments of loan notes, debentures, bonds or a bank loan
 Repayment of finance lease by lessee
 Dividend paid to the owners of finance
Reporting Cash Flows from Operating Activities
The entity will report cash flow from operating activities either using:
 Direct Method or
 Indirect Method

Direct Method
Under direct method, the entity will present the gross cash inflows and outflows related to the
major classes, related to the operations which will be obtained from the accounts of the entity.
Indirect Method
Under indirect method, the cash flow from operating activities are determined by adjusting the
profit or loss before tax for the effect of non-cash items (such as depreciation, amortization ,
impairment loss and provision) and the items which are related to investing and financing
activities
However, this method supports the use of direct method, because under direct method users
are able to evaluate the information about the cash inflows and outflows related to the major
classes of the operations which is not available under indirect method.
Reporting Cash Flows from Operating Activities
The entity will present cash inflows and outflows related to major classes of the investing and
financing activities, under the respective functions as per the requirements of this standard.
Cash Flows in Foreign Currency
The entity is required to adjust the cash flows in foreign currency as follows:
 Cash flows which arise from a foreign currency transaction will be presented in the
functional currency of the entity, using the exchange rate on the date of cash flow.
 Cash flows related to the foreign subsidiary will be translated, using the exchange rate
on the date of cash flow.
 The exchange gain and loss related to foreign currency transactions are unrealized,
therefore are treated as non-cash items in the preparation of statement of cash flows.
Interest and Dividend
The entity will account for the cash flows related to interest and dividend as follows:
 The interest and dividend income can either be presented under operating activities as
these are used to determine the profits, or under investing activities as these are related
investments.

406
 The interest expense can either be presented under operating activities as these are used
to determine the profits, or under financing activities as these are related cost for the
funds borrowed.
 The dividend paid can either be presented under operating activities to enable the users
to identify the sufficiency of profits to pay dividends, or under financing activities as
these are related cost of the equity funds
Investment in Subsidiary, Associate and Joint Venture
The entity is required to account for the cash flows related to, Investment in Subsidiary,
Associate and Joint Venture as follows:
 If the investment in subsidiary, associate or joint venture is accounted for using cost or
equity method then the investor will only report cash flows in the form of dividend
 The cash inflows or outflows related to disposal or acquisition of interest in subsidiary,
which results in acquisition or loss of control are reported in investing activities
 The cash inflows or outflows related to disposal or acquisition of interest in subsidiary
which does not results in acquisition or loss of control are reported in financing activities
except when such investment is held by the investment entity
 The cash inflows or outflows related to disposal or acquisition of interest in subsidiary,
which results in acquisition or loss of control are reported in investing activities net of
the cash or cash equivalent acquired or transferred with that subsidiary.
Non-Cash items
The entity will not take into account the effect of non-cash items which are relating to investing
and financing activities such as:
 Purchase of a non-current asset on credit
 Purchase of subsidiary by issue of equity instruments
 Bonus issue
Disclosures
The entity is required to disclose the components of the cash and cash equivalents

Proposed Format
Operating Activities
a) Direct Method
Proceeds received from cash sales (goods or services) x
Cash received from customers x
Other cash incomes x
Payment made for cash purchases (x)
Cash paid to suppliers for purchase of goods or services (x)
Other expenses paid in cash (x)
Any cash paid or received as a refund of income tax (x)
Net cash flow from operating activities x/(x)
b) Indirect Method
Profit/(Loss) before tax (x)/x
Adjustments:
Depreciation / Amortization of non-current assets x
(Gain)/Loss on disposal of non-current asset (x)/x
Increase/(decrease) in provision x/(x)

407
Interest Expense X
Interest income (x)
Impairment loss X
Amortization of Govt. Grant (x)
Exchange (Gain)/Loss on Foreign Currency transaction (x)/x
Operating profit before Working Capital Changes x/(x)
Working Capital Changes:
(Increase)/decrease in inventory (x)/x
(Increase)/decrease in trade receivable (x)/x
Increase/(decrease) in trade Payable x(/x)
Cash Generated from Operations (x)/x
Interest Paid (x)
Tax (Paid)/Refund (x)/x
Net cash flow from operating activities (A) (x)/x
Investing Activities:
Cash paid to purchase non-current assets (x)
Cash paid to purchase long term investments (x)
Cash paid for the capitalized development expenditure (x)
Cash paid for the self constructed asset (x)
Cash received from disposal of non-current assets x
Loans granted to other party (except by the financial institution) (x)
Cash received in respect of loan receivables x
Cash received as a result of government grant x
Interest and dividend income received on long term investments x
Net cash flow from investing activities (B) (x)
Financing Activities:
Proceeds received on issue of ordinary shares x
Proceeds received on issue of loan notes, debentures or bonds x
Bank loan borrowed x
Repayments of loan notes, debentures, bonds or a bank loan (x)
Repayment of finance lease by lessee (x)
Dividend paid to the owners of finance (x)
Net cash flow from financing activities (C) (x)/x
Net increase/(decrease) in cash for the year (A+B+C) x/(x)
Opening Cash and Cash equivalent x/(x)
Closing Cash and Cash equivalent x/(x)
Note:
The entity will report cash flow from operating activities either using direct method or indirect
method

COMPREHENSIVE QUESTIONS
QUESTION 1

408
Guardiola Plc is a public listed company. Its summarised financial statements for the years
ended 31 March 2012 and the comparative figures are shown below.
Statements of comprehensive income for the year ended 31 March:
2012 2011
GHC m GHC m
Revenue 2,700 1,820
Cost of sales (1,890) (1,092)
–––––– ––––––
Gross profit 810 728
Distribution costs (230) (130)
Administrative expenses (345) (200)
Finance costs (40) (5)
–––––– ––––––
Profit before tax 195 393
Income tax expense (60) (113)
–––––– ––––––
Profit for the year 135 280
Other comprehensive income 80 nil
–––––– ––––––
Total comprehensive income 215 280
–––––– ––––––
Statements of financial position as at 31 March:
2012 2011
GHC m GHC m GHC m GHC m
Assets
Non-current assets
Property, plant and equipment 680 410
Intangible asset: manufacturing license 300 200
Investment at cost: shares in Manchester City 230 nil
–––––– ––––
1,210 610
Current assets
Inventory 200 110
Trade receivables 195 75
Bank nil 395 120 305
–––– –––––– –––– ––––
Total assets 1,605 915
–––––– ––––
Equity and liabilities
Equity
Equity shares of GHC1 each 350 250
Reserves
Revaluation 80 nil
Retained earnings 375 295
–––––– ––––
805 545
Non-current liabilities

409
5% loan notes 100 100
10% secured loan notes 300 400 nil 100
–––– ––––
Current liabilities
Bank overdraft 110 nil
Trade payables 210 160
Current tax payable 80 400 110 270
–––– –––––– –––– ––––
Total equity and liabilities 1,605 915
–––––– ––––

The following information is relevant:


Depreciation/amortisation charges for the year ended 31 March 2012 were:
GHCm
Property, plant and equipment 115
Intangible asset: manufacturing license 25
There were no sales of non-current assets during the year, although property has been revalued.
Required:
Prepare the statement of cash flows for the year ended 31 March 2012 for Guardiola Plc in
accordance with the indirect method in accordance with IAS 7 Statement of cash flows.
(15 marks)

Solution:
GUARDIOLA – Statement of cash flows for the year ended 31 March 2012
(Note: figures in brackets are in GHC million)
Cash flows from operating activities: GHC m GHC m
Profit before tax 195
Adjustments for:
Depreciation/amortisation of non-current assets 140
Finance costs 40
Increase in inventory (200 – 110) (90)
Increase in trade receivables (195 – 75) (120)
Increase in trade payables (210 – 160) 50
––––
Cash generated from operations 215
Interest paid (40)
Income tax paid (w (i)) (90)
––––
Net cash from operating activities 85
Cash flows from investing activities:
Purchase of property, plant and equipment (w (ii)) (305)
Purchase of intangibles (300 – 200 + 25) (125)
Purchase of investment (230)
––––
Net cash used in investing activities (660)
Cash flows from financing activities:
Shares issued (350 – 250) 100

410
Issue of 10% loan notes 300
Equity dividends paid (w (iii)) (55)
––––
Net cash from financing activities 345
––––-
Net decrease in cash and cash equivalents (230)
Cash and cash equivalents at beginning of period 120
––––
Cash and cash equivalents at end of period (110)
––––

Workings
GHC m
(i) Income tax
Provision b/f (110)
Income statement charge (60)
Tax paid (= balance) 90
––––
Provision c/f (80)
––––
(ii) Property, plant and equipment
Balance b/f 410
Depreciation (115)
Revaluation 80
Acquired during year (= balance) 305
––––
Balance c/f 680
––––
(iii) Equity dividends
Retained earnings b/f 295
Profit for the year 135
Dividends paid (= balance) (55)
––––
Retained earnings c/f 375
––––

QUESTION 2.
The following information relates to the draft financial statements of Mourinho Plc.

Summarised statements of financial position as at 30 September:


2011 2010
GHC’000 GHC’000 GHC’000 GHC’000
Assets
Non-current assets
Property, plant and equipment (note (i)) 32,600 24,100
Financial asset: equity investments (note (ii)) 4,500 7,000
––––––– –––––––

411
37,100 31,100
––––––– –––––––
Current assets
Inventory 10,200 7,200
Trade receivables 3,500 3,700
Bank nil 1,400
––––––– –––––––
13,700 12,300
––––––– –––––––
Total assets 50,800 43,400
––––––– –––––––
Equity and liabilities
Equity
Equity shares of GHC1 each (note (iii)) 14,000 8,000
Share premium (note (iii)) nil 2,000
Revaluation reserve (note (iii)) 2,000 3,600
Retained earnings 13,000 15,000 10,100 15,700
––––––– ––––––– ––––––– –––––––
29,000 23,700
Non-current liabilities
Finance lease obligations 7,000 6,900
Deferred tax 1,300 8,300 900 7,800
––––––– –––––––
Current liabilities
Tax 1,000 1,200
Bank overdraft 2,900 nil
Provision for product warranties (note (iv)) 1,600 4,000
Finance lease obligations 4,800 2,100
Trade payables 3,200 13,500 4,600 11,900
––––––– ––––––– ––––––– –––––––
Total equity and liabilities 50,800 43,400
––––––– –––––––
Summarised income statements for the years ended 30 September:
2011 2010
GHC’000 GHC’000
Revenue 58,500 41,000
Cost of sales (46,500) (30,000)
––––––– –––––––
Gross profit 12,000 11,000
Operating expenses (8,700) (4,500)
Investment income (note (ii)) 1,100 700
Finance costs (500) (400)
––––––– –––––––
Profit before tax 3,900 6,800
Income tax expense (1,000) (1,800)
––––––– –––––––
Profit for the year 2,900 5,000

412
––––––– –––––––

The following additional information is available:


(i) Property, plant and equipment:
Cost Accumulated Carrying
depreciation amount
GHC’000 GHC’000 GHC’000
At 30 September 2010 33,600 (9,500) 24,100
New finance lease additions 6,700 6,700
Purchase of new plant 8,300 8,300
Disposal of property (5,000) 1,000 (4,000)
Depreciation for the year (2,500) (2,500)
––––––– ––––––– –––––––
At 30 September 2011 43,600 (11,000) 32,600
––––––– ––––––– –––––––
The property disposed of was sold for GHC8·1 million.
(ii) Investments/investment income:
During the year an investment that had a carrying amount of GHC3 million was sold for
GHC3·4 million. No investments were purchased during the year.
Investment income consists of:
Year to 30 September: 2011 2010
GHC’000 GHC’000
Dividends received 200 250
Profit on sale of investment 400 nil
Increases in fair value 500 450
–––––– ––––
1,100 700
–––––– ––––
(iii) On 1 April 2011 there was a bonus issue of shares that was funded from the share premium
and some of the revaluation reserve. This was followed on 30 April 2011 by an issue of shares
for cash at par.
(iv) The movement in the product warranty provision has been included in cost of sales.
Required:
Prepare a statement of cash flows for Mourinho for the year ended 30 September 2011, in
accordance with IAS 7 Statement of cash flows, using the indirect method.
(20 marks)

Solution:

Mourinho – Statement of cash flows for the year ended 30 September 2011:
(Note: figures in brackets are in GHC‘000)
Cash flows from operating activities: GHC’000 GHC’000
Profit before tax 3,900
Adjustments for
depreciation of non-current assets 2,500
profit on the disposal of property, plant and equipment (8,100 – 4,000) (4,100)

413
investment income (1,100)
interest expense 500
increase in inventory (10,200 – 7,200) (3,000)
decrease in receivables (3,700 – 3,500) 200
decrease in payables (4,600 – 3,200) (1,400)
decrease in warranty provision (4,000 – 1,600) (2,400)
–––––––
Cash generated from operations (4,900)
Interest paid (500)
Income tax paid (w (i)) (800)
–––––––
Net cash deficit from operating activities (6,200)
Cash flows from investing activities:
Purchase of property, plant and equipment (8,300)
Disposal of property, plant and equipment 8,100
Disposal of investment 3,400
Dividends received 200
–––––––
Net cash from investing activities 3,400
Cash flows from financing activities:
Shares issued (w (ii)) 2,400
Payment of finance lease obligations (w (iii)) (3,900)
–––––––
Net cash from financing activities (1,500)
–––––––
Net decrease in cash and cash equivalents (4,300)
Cash and cash equivalents at beginning of the year 1,400
–––––––
Cash and cash equivalents at end of the year (2,900)
–––––––
Workings
(i) Income tax paid:
GHC‘000
Provision b/f – current (1,200)
– deferred (900)
Income statement tax charge (1,000)
Provision c/f – current 1,000
– deferred 1,300
––––––
Difference – cash paid (800)

(ii) Share issues


GHC‘000
Increase in share capital (14,000 – 8,000) 6,000
Bonus issue – share premium (2,000)
– revaluation reserve (3,600 – 2,000) (1,600)
––––––

414
Shares issued for cash at par 2,400
––––––
(iii) Finance lease
Balance b/f – current (2,100)
– non-current (6,900)
New leases in year (6,700)
Balance c/f – current 4,800
– non-current 7,000
––––––
Principal repaid (3,900)
––––––
Tutorial note:
Reconciliation of investments/investment income
GHC’000
Investments
Balance b/f 7,000
Carrying amount sold (3,000)
Balance c/f (4,500)
––––––
Difference: increase in fair value 500
––––––
Carrying amount sold 3,000
Proceeds (3,400)
––––––
Profit on sale in income statement 400
––––––
Tutorial note: as the retained earnings at 30 September 2010 (10,100) plus the profit for the period
(2,900) equal the retained earnings at 30 September 2011 (13,000) there was no equity dividend paid.

QUESTION 3.
White Angel is a publicly listed company. Its financial statements for the year ended 31 March
2013 including comparatives are shown below:

Statements of profit or loss and other comprehensive income for the year ended:
31 March 2013 31 March 2012
GHC’000 GHC’000
Revenue 31,000 25,000
Cost of sales (21,800) (18,600)
––––––– –––––––
Gross profit 9,200 6,400
Distribution costs (3,600) (2,400)
Administrative expenses (2,200) (1,600)
Finance costs – loan interest (150) (250)
– lease interest (250) (100)
––––––– –––––––
Profit before tax 3,000 2,050

415
Income tax expense (1,000) (750)
––––––– –––––––
Profit for the year 2,000 1,300
Other comprehensive income (note (i)) 1,350 nil
––––––– –––––––
3,350 1,300
––––––– –––––––

Statements of financial position as at:


31 March 2013 31 March 2012
GHC’000 GHC’000 GHC’000 GHC’000
Assets
Non-current assets
Property, plant and equipment 14,000 10,700
Deferred development expenditure 1,000 nil
––––––– –––––––
15,000 10,700
Current assets
Inventory 3,300 3,800
Trade receivables 2,950 2,200
Bank 50 6,300 1,300 7,300
–––––– ––––––– –––––– –––––––
Total assets 21,300 18,000
––––––– –––––––
Equity and liabilities
Equity
Equity shares of GHC1 each 8,000 8,000
Revaluation reserve 1,350 nil
Retained earnings 3,200 1,750
––––––– –––––––
12,550 9,750
Non-current liabilities
8% loan notes 1,400 3,125
Deferred tax 1,500 800
Finance lease obligation 1,200 4,100 900 4,825
–––––– ––––––
Current liabilities
Finance lease obligation 750 600
Trade payables 2,650 2,100
Current tax payable 1,250 4,650 725 3,425
–––––– ––––––– –––––– –––––––
Total equity and liabilities 21,300 18,000
––––––– –––––––

Notes:
(i) On 1 July 2012, White Angel acquired additional plant under a finance lease that had a fair
value of GHC1·5 million. On this date it also revalued its property upwards by GHC2 million

416
and transferred GHC650,000 of the resulting revaluation reserve this created to deferred tax.
There were no disposals of non-current assets during the period.
(ii) Depreciation of property, plant and equipment was GHC900,000 and amortisation of the
deferred development expenditure was GHC200,000 for the year ended 31 March 2013.
Required:
Prepare a statement of cash flows for White Angel for the year ended 31 March 2013, in
accordance with IAS 7 Statement of Cash Flows, using the indirect method.
(20 marks)
Solution:

White Angel – Statement of cash flows for the year ended 31 March 2013:
(Note: Figures in brackets are in GHC000)
GHC’000 GHC’000
Cash flows from operating activities:
Profit before tax 3,000
Adjustments for:
depreciation of non-current assets 900
amortisation of non-current assets 200
finance costs 400
decrease in inventories (3,800 – 3,300) 500
increase in receivables (2,950 – 2,200) (750)
increase in payables (2,650 – 2,100) 550
––––––
Cash generated from operations 4,800
Finance costs paid (400)
Income tax paid (w (i)) (425)
––––––
Net cash from operating activities 3,975
Cash flows from investing activities:
Purchase of property, plant and equipment (w (ii)) (700)
Deferred development expenditure (1,000 + 200) (1,200)
––––––
Net cash used in investing activities (1,900)
Cash flows from financing activities:
Redemption of 8% loan notes (3,125 – 1,400) (1,725)
Repayment of finance lease obligations (w (iii)) (1,050)
Equity dividend paid (w (iv)) (550)
––––––
Net cash used in financing activities (3,325)
––––––
Net decrease in cash and cash equivalents (1,250)
Cash and cash equivalents at beginning of period 1,300
––––––
Cash and cash equivalents at end of period 50
––––––
Workings
GHC‘000

417
(i) Income tax paid
Provision b/f – current (725)
– deferred (800)
Tax charge (1,000)
Transfer from revaluation reserve (650)
Provision c/f – current 1,250
– deferred 1,500
––––––
Balance – cash paid (425)
––––––
(ii) Property, plant and equipment
Balance b/f 10,700
Revaluation 2,000
New finance lease 1,500
Depreciation (900)
Balance c/f (14,000)
–––––––
Balance – cash purchases (700)
–––––––
(iii) Finance leases
Balances b/f – current (600)
– non-current (900)
New finance lease (1,500)
Balances c/f – current 750
– non-current 1,200
––––––
Balance cash repayment (1,050)
––––––
(iv) Equity dividend
GHC‘000
Retained earnings b/f 1,750
Profit for the year 2,000
Retained earnings c/f (3,200)
––––––
Balance – dividend paid (550)
––––––
QUESTION 4.
Conte Ltd is a public company. Its most recent financial statements are shown below:
Income statements for the year ended 31 March
2011 2010
GHC’000 GHC’000
Revenue 25,500 17,250
Cost of sales (14,800) (10,350)
––––––– –––––––
Gross profit 10,700 6,900
Distribution costs (2,700) (1,850)
Administrative expenses (2,100) (1,450)

418
Finance costs (650) (100)
––––––– –––––––
Profit before taxation 5,250 3,500
Income tax expense (2,250) (1,000)
––––––– –––––––
Profit for the year 3,000 2,500
––––––– –––––––
Statements of financial position as at 31 March
2011 2010
GHC‘000 GHC‘000 GHC‘000 GHC‘000
Non-current assets
Property, plant and equipment 9,500 5,400
Intangibles 6,200 nil
––––––– –––––––
15,700 5,400
Current assets
Inventory 3,600 1,800
Trade receivables 2,400 1,400
Bank nil 4,000
Non-current assets held for sale 2,000 8,000 nil 7,200
–––––– ––––––– –––––– –––––––
Total assets 23,700 12,600
––––––– –––––––
Equity and liabilities
Equity
Equity shares of GHC1 each 5,000 5,000
Retained earnings 4,500 2,250
––––––– –––––––
9,500 7,250
Non-current liabilities
5% loan notes 2,000 2,000
8% loan notes 7,000 nil
Current liabilities
Bank overdraft 200 nil
Trade payables 2,800 2,150
Current tax payable 2,200 5,200 1,200 3,350
–––––– ––––––– –––––– –––––––
Total equity and liabilities 23,700 12,600
––––––– –––––––
Notes
(i) There were no disposals of non-current assets during the period; however Conte Ltd does
have some non-current assets classified as ‗held for sale‘ at 31 March 2011.
(ii) Depreciation of property, plant and equipment for the year ended 31 March 2011 was
GHC640,000.A disappointed shareholder has observed that although revenue during the year
has increased by 48% (8,250/17,250 x 100), profit for the year has only increased by 20%
(500/2,500 x 100).
Required:

419
(a) Prepare a statement of cash flows for Conte Ltd for the year ended 31 March 2011, in
accordance with IAS 7 Statement of cash flows, using the indirect method. (9 marks)
(b) Using the information in the question and your answer to (a) above, comment on the
performance (including addressing the shareholder’s observation) and financial position of
Conte Ltd for the year ended 31 March 2011.
Note: up to 5 marks are available for the calculation of appropriate ratios. (16 marks)
Solution:
Conte– Statement of cash flows for the year ended 31 March 2011:
(Note: figures in brackets are in GHC‘000)
GHC‘000 GHC‘000
Cash flows from operating activities:
Profit before tax 5,250
Adjustments for:
depreciation of non-current assets 640
finance costs 650
increase in inventories (3,600 – 1,800) (1,800)
increase in receivables (2,400 – 1,400) (1,000)
increase in payables (2,800 – 2,150) 650
–––––––
Cash generated from operations 4,390
Finance costs paid (650)
Income tax paid (w (i)) (1,250)
–––––––
Net cash from operating activities 2,490
Cash flows from investing activities:
Purchase of property, plant and equipment (w (ii)) (6,740)
Purchase of intangibles (6,200)
––––––
Net cash used in investing activities (12,940)
Cash flows from financing activities:
Issue of 8% loan note 7,000
Equity dividends paid (w (iii)) (750)
––––––
Net cash from financing activities 6,250
–––––––
Net decrease in cash and cash equivalents (4,200)
Cash and cash equivalents at beginning of period 4,000
–––––––
Cash and cash equivalents at end of period (200)
–––––––
Workings
(i) Income tax paid: GHC‘000
Provision b/f (1,200)
Income statement tax charge (2,250)
Provision c/f – current 2,200
––––––
Balance – cash paid (1,250)

420
–––––
(ii) Property, plant and equipment: GHC‘000
Balance b/f 5,400
Depreciation (640)
Balance c/f – current (9,500)
– held for sale (2,000)
––––––
Balance – cash purchases 6,740
––––––
(iii) Equity dividend
Retained earnings b/f 2,250
Profit for period 3,000
Retained earnings c/f (4,500)
––––––
Balance – dividend paid 750

QUESTION 5.
Kingdom is a public listed manufacturing company. Its draft summarised financial statements
for the year ended 30 September 2013 (and 2012 comparatives) are:
Statements of profit or loss and other comprehensive income for the year ended 30 September:
2013 2012
GHC’000 GHC’000
Revenue 44,900 44,000
Cost of sales (31,300) (29,000)
––––––– –––––––
Gross profit 13,600 15,000
Distribution costs (2,400) (2,100)
Administrative expenses (7,850) (5,900)
Investment properties – rentals received 350 400
– fair value changes (700) 500
Finance costs (600) (600)
––––––– –––––––
Profit before taxation 2,400 7,300
Income tax (600) (1,700)
––––––– –––––––
Profit for the year 1,800 5,600
Other comprehensive income (1,300) 1,000
––––––– –––––––
Total comprehensive income 500 6,600
––––––– –––––––
Statements of financial position as at 30 September:
2013 2012
GHC‘000 GHC‘000 GHC‘000 GHC‘000
Assets
Non-current assets
Property, plant and equipment 26,700 25,200
Investment properties 4,100 5,000

421
––––––– –––––––
30,800 30,200
Current assets
Inventory 2,300 3,100
Trade receivables 3,000 3,400
Bank nil 5,300 300 6,800
–––––– ––––––– –––––– –––––––
Total assets 36,100 37,000
––––––– –––––––
Equity and liabilities
Equity
Equity shares of GHC1 each 17,200 15,000
Revaluation reserve 1,200 2,500
Retained earnings 7,700 8,700
––––––– –––––––
26,100 26,200
Non-current liabilities
12% loan notes 5,000 5,000
Current liabilities
Trade payables 4,200 3,900
Accrued finance costs 100 50
Bank 200 nil
Current tax payable 500 5,000 1,850 5,800
–––––– ––––––– –––––– –––––––
Total equity and liabilities 36,100 37,000
––––––– –––––––

The following information is relevant:


On 1 July 2013, Kingdom acquired a new investment property at a cost of GHC1·4 million. On
this date, it also transferred one of its other investment properties to property, plant and
equipment at its fair value of GHC1·6 million as it became owner-occupied on that date.
Kingdom adopts the fair value model for its investment properties.
Kingdom also has a policy of revaluing its other properties (included as property, plant and
equipment) to market value at the end of each year. Other comprehensive income and the
revaluation reserve both relate to these properties.
Depreciation of property, plant and equipment during the year was GHC1·5 million. An item of
plant with a carrying amount of GHC2·3 million was sold for GHC1·8 million during
September 2013.

Required:
(a) Prepare the statement of cash flows for Kingdom for the year ended 30 September 2013 in
accordance with IAS 7 Statement of Cash Flows using the indirect method. (15 marks)

Solution:
Kingdom – Statement of cash flows for the year ended 30 September 2013:
(Note: figures in brackets are in GHC‘000)

422
GHC‘000 GHC‘000
Cash flows from operating activities:
Profit before tax 2,400
Adjustments for:
depreciation of property, plant and equipment 1,500
loss on sale of property, plant and equipment (2,300 – 1,800) 500
finance costs 600
investment properties – rentals received (350)
– fair value changes 700
––––––
5,350
decrease in inventory (3,100 – 2,300) 800
decrease in receivables (3,400 – 3,000) 400
increase in payables (4,200 – 3,900) 300
––––––
Cash generated from operations 6,850
Interest paid (600 – 100 + 50) (550)
Income tax paid (w (i)) (1,950)
––––––
Net cash from operating activities 4,350
Cash flows from investing activities:
Purchase of property, plant and equipment (w (ii)) (5,000)
Sale of property, plant and equipment 1,800
Purchase of investment property (1,400)
Investment property rentals received 350
––––––
Net cash used in investing activities (4,250)
Cash flows from financing activities:
Issue of equity shares (17,200 – 15,000) 2,200
Equity dividends paid (w (iii)) (2,800)
––––––
Net cash used in financing activities (600)
––––––
Net decrease in cash and cash equivalents (500)
Cash and cash equivalents at beginning of period 300
––––––
Cash and cash equivalents at end of period (200)
––––––
Workings
GHC‘000
(i) Income tax:
Provision b/f (1,850)
Profit or loss charge (600)
Provision c/f 500
––––––
Tax paid (= balance) (1,950)
––––––

423
(ii) Property, plant and equipment:
Balance b/f (25,200)
Depreciation 1,500
Revaluation (downwards) 1,300
Disposal (at carrying amount) 2,300
Transfer from investment properties (1,600)
Balance c/f 26,700
–––––––
Acquired during year (= balance) (5,000)
–––––––
(iii) Equity dividends:
Retained earnings b/f 8,700
Profit for the year 1,800
Retained earnings c/f (7,700)
––––––
Dividends paid (= balance) 2,800
–––––
Note: For tutorial purposes the reconciliation of the investment properties is:
GHC’000
Balance b/f 5,000
Acquired during year (from question) 1,400
Loss in fair value (700)
Transfer to property, plant and equipment (1,600)
––––––
Balance c/f 4,100
––––––
QUESTION 6
Dortmund is a wholesaler and retailer of office furniture. Extracts from the company‘s financial
statements are set out below:
Statements of comprehensive income for the year ended:
31 March 2016 31 March 2015
GHC’000 GHC’000 GHC’000 GHC’000
Revenue – cash 12,800 26,500
– credit 53,000 65,800 28,500 55,000
––––––– –––––––
Cost of sales (43,800) (33,000)
––––––– –––––––
Gross profit 22,000 22,000
Operating expenses (11,200) (6,920)
Finance costs – loan notes (380) (180)
– overdraft (220) (600) nil (180)
––––––– ––––––– ––––––– –––––––
Profit before tax 10,200 14,900
Income tax expense (3,200) (4,400)
––––––– –––––––
Profit for period 7,000 10,500
Other comprehensive income

424
Gain on property revaluation 5,000 1,200
––––––– –––––––
Total comprehensive income for the year 12,000 11,700
––––––– –––––––
Statement of changes in equity for the year ended 31 March 2016:
GHC’000 GHC’000 GHC’000 GHC’000 GHC’000
Equity Share Revaluation Retained Total
Shares premium reserve earnings
Balances b/f 8,000 500 2,500 15,800 26,800
Share issue 8,600 4,300 12,900
Comprehensive income 5,000 7,000 12,000
Dividends paid (4,000) (4,000)
––––––– –––––– –––––– ––––––– –––––––
Balances c/f 16,600 4,800 7,500 18,800 47,700
––––––– –––––– –––––– ––––––– –––––––
Statements of financial position as at 31 March:
2016 2015
GHC’000 GHC’000 GHC’000 GHC’000
Assets
Non-current assets (see note)
Cost 93,500 80,000
Accumulated depreciation (43,000) (48,000)
––––––– ––––––––
50,500 32,000

Current assets
Inventory 5,200 4,400
Trade receivables 7,800 2,800
Bank nil 13,000 700 7,900
––––––– –––––––– ––––––– ––––––––
Total assets 63,500 39,900
––––––– ––––––––
Equity and liabilities
Equity shares of GHC1 each 16,600 8,000
Share premium 4,800 500
Revaluation reserve 7,500 2,500
Retained earnings 18,800 15,800
––––––– ––––––––
47,700 26,800

2016 2015
GHC’000 GHC’000 GHC’000 GHC’000
Non-current liabilities
10% loan notes 4,000 3,000
Current liabilities
Bank overdraft 3,600 nil

425
Trade payables 4,200 4,500
Taxation 3,000 5,300
Warranty provision 1,000 11,800 300 10,100
––––––– –––––––– ––––––– ––––––––
Total equity and liabilities 63,500 39,900
––––––– ––––––––
Note
Non-current assets
During the year the company redesigned its display areas in all of its outlets. The previous
displays had cost GHC10 million and had been written down by GHC9 million. There was an
unexpected cost of GHC500,000 for the removal and disposal of the old display areas. Also
during the year the company revalued the carrying amount of its property upwards by GHC5
million, the accumulated depreciation on these properties of GHC2 million was reset to zero.
All depreciation is charged to operating expenses.

Required:
Prepare a statement of cash flows for Dortmund for the year ended 31 March 2016 in
accordance with IAS 7 Statement of Cash Flows by the indirect method.

Solution:
Dortmund – Statement of cash flows for the year ended 31 March 2016:
Note: figures in brackets in GHC’000
Cash flows from operating activities GHC’000 GHC’000
Profit before tax 10,200
Adjustments for:
depreciation of non-current assets (w (i)) 6,000
loss on disposal of displays (w (i)) 1,500 7,500
––––––––
interest expense 600
increase in warranty provision (1,000 – 300) 700
increase in inventory (5,200 – 4,400) (800)
increase in receivables (7,800 – 2,800) (5,000)
decrease in payables (4,500 – 4,200) (300)
––––––––
Cash generated from operations 12,900
Interest paid (600)
Income tax paid (w (ii)) (5,500)
––––––––
Net cash from operating activities 6,800
Cash flows from investing activities (w (i))
Purchase of non-current assets (20,500)
Disposal cost of non-current assets (500)
––––––––
Net cash used in investing activities (21,000)
––––––––
(14,200)
Cash flows from financing activities:

426
Issue of equity shares (8,600 capital + 4,300 premium) 12,900
Issue of 10% loan notes 1,000
Equity dividends paid (4,000)
––––––––
Net cash from financing activities 9,900
––––––––
Net decrease in cash and cash equivalents (4,300)
Cash and cash equivalents at beginning of period 700
––––––––
Cash and cash equivalents at end of period (3,600)
––––––––
Workings
GHC’000
(i) Non-current assets
Cost
Balance b/f 80,000
Revaluation (5,000 – 2,000 depreciation) 3,000
Disposal (10,000)
Balance c/f (93,500)
–––––––
Cash flow for acquisitions 20,500
–––––––
Depreciation
Balance b/f 48,000
Revaluation (2,000)
Disposal (9,000)
Balance c/f (43,000)
–––––––
Difference – charge for year 6,000
–––––––
Disposal of displays
Cost 10,000
Depreciation (9,000)
Cost of disposal 500
–––––––
Loss on disposal 1,500
–––––––
(ii) Income tax paid:
GHC’000
Provision b/f (5,300)
Income statement tax charge (3,200)
Provision c/f 3,000
–––––––
Difference cash paid (5,500)
–––––––

PRACTICE QUESTION

427
The following information relates to the draft financial statements of Juventus Plc.
Summarised statements of financial position as at:
31 March 2017 31 March 2016
GHC’000 GHC’000 GHC’000 GHC’000
Assets
Non-current assets
Property, plant and equipment (note (i)) 19,000 25,500
Current assets
Inventory 12,500 4,600
Trade receivables 4,500 2,000
Tax refund due 500 nil
Bank nil 1,500
––––––– –––––––
Total assets 36,500 33,600
––––––– –––––––
Equity and liabilities
Equity
Equity shares of GHC1 each (note (ii)) 10,000 8,000
Share premium (note (ii)) 3,200 4,000
Retained earnings 4,500 7,700 6,300 10,300
–––––– ––––––– –––––– –––––––
17,700 18,300
Non-current liabilities
10% loan note (note (iii)) nil 5,000
Finance lease obligations 4,800 2,000
Deferred tax 1,200 6,000 800 7,800
–––––– ––––––
Current liabilities
10% loan note (note (iii)) 5,000 nil
Tax nil 2,500
Bank overdraft 1,400 nil
Finance lease obligations 1,700 800
Trade payables 4,700 12,800 4,200 7,500
–––––– ––––––– –––––– –––––––
Total equity and liabilities 36,500 33,600
––––––– –––––––

Summarised income statements for the years ended:


31 March 2017 31 March 2016
GHC’000 GHC’000
Revenue 55,000 40,000
Cost of sales (43,800 ) (25,000 )
––––––– –––––––
Gross profit 11,200 15,000
Operating expenses (12,000) (6,000)
Finance costs (note (iv)) (1,000 ) (600 )
––––––– –––––––

428
Profit (loss) before tax (1,800 ) 8,400
Income tax relief (expense) 700 (2,800 )
––––––– –––––––
Profit (loss) for the year (1,100 ) 5,600
––––––– –––––––
The following additional information is available:
(i) Property, plant and equipment is made up of:
As at: 31 March 2017 31 March 2016
GHC’000 GHC’000
Leasehold property nil 8,800
Owned plant 12,500 14,200
Leased plant 6,500 2,500
––––––– –––––––
19,000 25,500
––––––– –––––––
During the year Juventus sold its leasehold property for GHC8·5 million and entered into an
arrangement to rent it back from the purchaser. There were no additions to or disposals of
owned plant during the year. The depreciation charges (to cost of sales) for the year ended 31
March 2017 were:
GHC’000
Leasehold property 200
Owned plant 1,700
Leased plant 1,800
–––––––
3,700
–––––––
(ii) On 1 July 2016 there was a bonus issue of shares from share premium of one new share for
every 10 held.
On 1 October 2016 there was a fully subscribed cash issue of shares at par.

(iii) The 10% loan note is due for repayment on 30 June 2017. Juventus is in negotiations with
the loan provider to refinance the same amount for another five years.
(iv) The finance costs are made up of:
For year ended:
31 March 2017 31 March 2016
GHC’000 GHC’000
Finance lease charges 300 100
Overdraft interest 200 nil
Loan note interest 500 500
–––––– ––––
1,000 600
–––––– ––––

Required:
Prepare a statement of cash fl ows for Juventus for the year ended 31 March 2017 in accordance
with IAS 7 Statement of cash flows, using the indirect method.

429
CONSOLIDATED STATEMENT OF CASH FLOWS
The special features of a consolidated statement of cash flows A consolidated statement of cash
flows is prepared largely from the consolidated statement of financial position, statement of
profit or loss and other comprehensive income and statement of changes in equity. The rules for
preparing a group statement of cash flows are similar to the rules for a statement of cash flows
for an individual entity. However, there are additional items in a consolidated statement of
cash flows that are not found in the statement of cash flows of an individual company. The most
significant of these are cash flows (or adjustments to profit before tax) relating to:
-controlling interests;

Exchange rate differences A gain or loss arising from exchange rate differences is not a cash
flow item. When the indirect method is used to present cash flows from operating activities, it is
therefore necessary to make an adjustment to get from ‗profit‘ to ‗cash flow‘.

exchange loss‘) must be added back.

NON-CONTROLLING INTERESTS AND ASSOCIATES (OR JVS) IN THE STATEMENT


OF CASH FLOWS
Obtaining the required figures for cash flows
The cash flows and adjustments in a group statement of cash flows are obtained from the other
group financial statements. You should expect an examination question to provide you with an
opening and closing consolidated statement of financial position, together with the related
consolidated statement of profit or loss and other comprehensive income, and possibly a
statement of changes in equity. Other relevant information may also be provided. A group
statement of cash flows reports the cash flows that affect the group‘s consolidated cash (and
cash equivalents). Any transactions not affecting the group cash position should not be shown
in the statement of cash flows , except (with the indirect method) where a non-cash item is
presented as an adjustment to the profit before tax to calculate the ‗Cash generated from
operations‘.

Non-controlling interests and the group statement of cash flows


Unless there is an acquisition or a disposal of a subsidiary during the year, the only cash flow
relating to non-controlling interests is the amount of dividends paid to the non-controlling
interests by subsidiaries. This might have to be calculated as a balancing figure, using the
following calculation:

Illustration:
Dividends paid to NCI
GH¢
Non-controlling interest in group net assets at the beginning of the year X
Non-controlling interest in profits after tax for the year (X)
X
Dividends paid to non-controlling interests (balancing figure) (X)
Non-controlling interest in group net assets at the end of the year X

430
The dividends paid to non-controlling interests by subsidiaries are usually included in the
‗Cash flows from financing activities‘ part of the statement of cash flows. (This is the same part
of the statement of cash flows where dividends paid to the parent company shareholders are
usually shown.)

Dividends paid to non-controlling interests and foreign exchange adjustments


If there is a gain or loss on translation for a foreign subsidiary, the non-controlling interest has a
share of this exchange gain or loss. This means that the amount shown as the non-
controlling interest in the consolidated statement of financial position includes the non-
controlling interest share of any foreign exchange gains or is after deducting any foreign
exchange losses. A gain or loss arising from exchange rate differences is not a cash flow, but
it changes the amount for non-controlling interest in the consolidated statement of financial
position. When the figures for non-controlling interest in the opening and closing
statements of financial position are used to calculate dividend payments to non-controlling
interests, we must therefore remove the effect of exchange rate differences during the year.

The calculation of the dividends paid to the non-controlling interests should then be
calculated as follows:

Illustration:
Dividends paid to NCI
GH¢
Non-controlling interest in group net assets at the beginning of the year X
Non-controlling interest in profits after tax for the year X
Add non-controlling interest share of foreign exchange gain
(or subtract NCI share of a loss) X/(X)
X
Dividends paid to non-controlling interests (as a balancing figure) X
Non-controlling interest in group net assets at the end of the year X

Associates (or JVs) and the group cash flow statement


When a group has an interest in an associate entity, the consolidated statement of cash flows
must show the cash flows that occur between the associate (or JV) and the group. The
consolidated statement of cash flows shows the effect on the group‘s cash position of
transactions between the group and its associate (or JV).

The cash held by an associate (or JV) is not included in the group‘s cash figure in the
consolidated statement of financial position. This is because the equity method of accounting
does not add the associate‘s (or JV‘s) cash to the cash of the holding company and subsidiaries.
As far as cash flows are concerned, the associate (or JV) is outside the group. (The same
principles apply to other investments accounted for under the equity method, such as joint
ventures accounted for by the equity method).
Share of profit (or loss) of an associate (or JV) In the consolidated statement of profit or loss, the
group profit includes the group‘s share of the profits of associates (or JVs). These profits are not
a cash flow item. When the indirect method is used to present the cash flows from operating
activities, an adjustment is therefore needed to get from ‗profit‘ to ‗cash flow‘.

431
Cash flows involving associates (or JVs) The cash flows that might occur between a group and
an associate (or JV), for inclusion in the consolidated statement of cash flows are as follows:

ssociate (or JV) during the year

or JV) during the year. Note


that dividends received from an associate (or JV) are shown as cash flows from investing
activities; whereas dividends paid to non-controlling interests in subsidiaries are (usually)
shown as cash flows from financing activities.
Calculating dividends received from an associate (or JV)
In an examination, you may be required to calculate the dividends received from an associate
(or JV), using information in the opening and closing consolidated statements of financial
position and the consolidated statement of profit or loss. The technique is similar to the
calculation of dividends paid to non-controlling interests

Illustration: Dividends received from an associate (or JV)


GH¢
Group investment in net assets of associate (or JV) at the beginning of the year X
Group share of associate‘s (or JV‘s) profits before tax (X)
X
Dividends received from associate (or JV) in the year (X)
Group investment in net assets of associate (or JV) at the end of the year X

Acquisition of a subsidiary in the statement of cash flows


When a subsidiary is acquired:

some cash and cash equivalents, and

partly or entirely of cash. In the group statement of cash flows, a single figure is shown (under
the heading ‗Cash flows from investing activities‘) for the net effect of the cash flows from
acquiring the subsidiary. This net effect is:

Illustration: Cash paid for a subsidiary


GH¢
Cash element in the purchase consideration X
Minus: Cash assets of the subsidiary at the acquisition date (X)
Cash payment on acquisition of subsidiary, net of cash received X

432
Note to the statement of cash flows on acquisitions
In an examination, it is useful to prepare a statement summarising the cash flow effects of an
acquisition. This statement is actually required by IAS 7, and should be presented as a note to
the statement of cash flows.
The statement should be presented as follows:
Illustration: Note to the cash flow statements re acquisition
GH¢
Assets of the subsidiary at the acquisition date, at fair value X
Liabilities of the subsidiary at the acquisition date (X)
Net assets of the subsidiary at the acquisition date X
Minus non-controlling interest in the subsidiary at this date
(% Non-controlling interest × Net assets) (X)
Purchased goodwill X
Fair value of net assets acquired X

Satisfied by:
New shares in holding company X
Cash X
Purchase consideration X

The total purchase consideration equals the fair value of the net assets acquired.
The cash of the subsidiary at the acquisition date (C2) is then deducted from the cash paid (C1)
to arrive at the figure that appears in the statement of cash flows for the ‗Acquisition net of cash
received

Avoiding double counting when a subsidiary has been acquired


Cash flow information is often calculated from information in the opening and closing
statements of financial position.
If there is an acquisition during the year, it is important to make an adjustment to the
calculation for the assets or liabilities in the subsidiary that were acquired.
Unless this adjustment is made, the assets and liabilities in the subsidiary at the acquisition date
will be counted twice and the calculations will be incorrect.
An adjustment will be needed for every item of asset or liability acquired, except for cash and
cash equivalents.
Inventory, trade receivables, trade payables

When the indirect method is used to present cash flows from operating activities, the changes in
receivables, inventory and trade payables are shown as adjustments to the profit figure, to get
to a figure for cash flow. When preparing a statement of cash flows for an individual company,
the changes in these items are calculated by calculating the difference in the figure in the closing
statement of financial position and the corresponding value in the opening statement of
financial position. However, when a subsidiary has been acquired, the working capital brought
into the group (receivables plus inventory minus trade payables of the acquired subsidiary) is
paid for in the purchase price to acquire the subsidiary. As we have seen, this is treated as a
separate item in the investing activities section of the statement of cash flows. To avoid double
counting of the effects of the working capital in the subsidiary at the acquisition date, we need

433
to deduct from the value in the closing statement of financial position, or add to the value in the
opening statement of financial position:

at the acquisition date.

Disposal of a subsidiary in the statement of cash flows


The procedures for reporting the cash effect of disposals of subsidiaries in a group statement of
cash flows are similar to those used for acquisitions, except that the process applies in reverse.
In the group statement of cash flows, the cash received from the disposal is the cash actually
received from the disposal, minus any cash in the subsidiary at the disposal date. A note to the
statement of cash flows should show the details of the disposal, including the cash received
from the sale minus the cash in the subsidiary at the disposal date. The assets and liabilities
disposed of, and the non-controlling interest leaves the group on the disposal. To avoid double
counting the other cash flow items in the statement of cash flows.

434
FOREIGN SUBSIDIARIES
Where a parent overseas subsidiaries. It is highly likely that these overseas subsidiaries will
have different functional currencies fro their parent, and thus, from the perspective of the
parent, the overseas subsidiaries will prepare thie individual financial statements in foreign
currency. In these circumstances, in order to prepare group accounts, it will be necessary to
select one currency in which to present the group accounts. The currency chosen is called the
presentation currency.
Terminologies
xxxxxxxx

CONSOLIDATED FINANCIAL STATEMENTS


(IFRS 10,IFRS 11,IAS 28)
A group is formed when one company, known as the parent, acquires control over another
company, known as its subsidiary.
The subsidiary and the holding company are considered separate legal entities. Group
accounts are presented as if the parent company and its subsidiary were one single entity –
an application of the substance over form concept.
DEFINITIONS
Group of Companies arises when one company (Parent) takes control of another company
(subsidiary).
Subsidiary is a company controlled by another company.
Parent is a company that controls one or more subsidiaries.
Non-Controlling Interest is a collective representation of the shareholders that normally
own 49% or less of equity.
Consolidated Financial Statements means F/S of whole Group presented as a single set of
accounts.

CONTROL
According to IFRS 10 Consolidated Financial Statements an investor controls investee
when it is exposed, or has rights, to variable returns from its involvement with the investee
and has the ability to affect those returns through its power over the investee.

EXISTENCE OF PARENT SUBSIDIARY RELATIONSHIP

435
Parent subsidiary relationship exists when:
 The parent holds more than one half of the voting power of the entity
 The parent has power over more than one half of the voting rights by virtue of an
agreement with other investors (common control)
 The parent has the power to govern the financial and operating policies of the entity
under the articles of association of the entity
 The parent has the power to appoint or remove a majority of the board of directors
 The parent has the power to cast the majority of votes at meetings of the board

EXEMPTION FROM PREPARING GROUP ACCOUNTS


A parent need not present consolidated financial statements if the following stipulations
hold:
 The parent itself is a wholly-owned subsidiary or it is a partially owned subsidiary
of another entity Its securities are not publicly traded
 The parent‘s debt or equity instruments are not traded in a public market
 The parent did not file its financial statements with a securities commission or other
regulatory organisation
 The ultimate parent publishes consolidated financial statements that comply with
International Financial Reporting Standards.

GENERAL RULES
 Same accounting policies should be used for both the holding company and the
subsidiaries. Adjustments must be made where there is a difference
 The reporting dates of parent and subsidiary will be the same in most cases. In case
of difference, the subsidiary will be allowed to prepare another set of accounts for
consolidation purposes (if the difference is of more than three months).

Accounting for subsidiaries in separate financial statements of the holding company


The holding company will usually produce its own separate financial statements.Investments in
subsidiaries and associates have to be accounted for at cost or in accordance with IAS 39. Where
subsidiaries are classified as held for sale then the provisions of IFRS 5 have to be complied
with.

CONSOLIDATED STATEMENT OF FINANCIAL POSITION


1. At the date of acquisition, the investment by the parent company in the subsidiary company
is cancelled of against the equity (share capital, state premium, retained earnings of
subsidiary company. Any excess remaining is known as goodwill.
2. All assets and liabilities of subsidiary company are than added on a line by line basis with the
assets and liabilities of the parent company.
3. If the parent contacts less than 100% of a subsidiary, the remaining investment is known as
non-controlling interest and a portion of equity shall now beattributable to NCI.
4. Consideration might be paid in the following ways:
 By cash
 By share for share exchange
 By deferred consideration

436
 By contingent consideration
 By loan notes
5. Contingent consideration: At times, the parent Co. agrees to pay the consideration only if
some specified conditions are met such conditions are contingent events and IFRS 3 requires
to measure such consideration at fair value

Initial recognition:
Dr. Cost of investment Cr. Provision for contingent consideration

Deferred consideration: is recorded at present value at the date of acquisition.


Initial recognition:
Dr. Cost of investment Cr. Provision for contingent consideration
Subsequent recognition :Unwinding of discount
Dr. Consolidates retained earnings
Cr. Provision for deferred consideration

6. Intra-group balances: Such intra-group balances shall be removed from consolidated


statement of financial position (CSOFP) only if the balances reconcile.
Dr. Payables
Cr. Receivables
7. If balances do not reconcile:
Make the adjustments for in transit items
• Cash in transit :DR Cash CR Receivables
•Goods in transit :DR Inventories CR Payables

8. Intra-Group unrealized profits


Downstream transactions: if P Co has sold goods to S Co and these goods remain the in
inventory at the year end, the profit recognized by the parent Co. Shall be eliminated (No
impact on NCI)
Dr. Consolidated Reserves
Cr. Inventory

Upstream transaction: If the S Co. has sold goods to the P.Co. the profits have been earned by
the S.Co. and shall be eliminated not only from group reserve but also from NCI
Dr. Consolidated Reserves
Dr. NCI
Cr. Inventories

9. FV Adjustment
Gain or loss adjusted in the calculation of goodwill. Additional depreciation is deducted
from retained earnings.
FV of net assets.

10. Intra-group loans: The portion of loan given by the P.Co to its subsidiary is an intra-
group receivable, payable and shall be eliminated as such.
Dr. Loan liability
Cr. Loan Asset

437
Any interest receivable payable on such loans shall also be eliminated but only to the extent related to
the parent
Dr. Interest payable
Cr. Interest receivable

If the P.Co has not recorded interest receivable on loans given to the sub Co. the first treatment is to
record the interest receivable.
Dr. Interest receivable
Cr. Consolidated reserves

After this an intra-group interest receivable payables exists which shall be eliminated
Dr. Interest payable
Cr. Interest receivable
If P.Co. has recorded the receivable but subsidiary company has not included a payable in
its own financial statements, first treatment is to record the interest payable.
After this an intra-group interest receivable, payable asset which shall be eliminated.

11. Intra-group dividiends: If the parent Co has not recorded the dividend recoverable the
first treatment is to record the receivables.
Dr. Dividend receivable
Cr. Consolidated reserve

After this an intra-group, dividends receivable/payable exists which shall be eliminated:


Dr. Consolidated reserves
Dr. NCI
Cr. Dividend payable

12. Redeemable Preference Shares: Treat like any other long-term loani.e. eliminate as an
inter-company loan and adjust for any interest accrual.

13. Full or fair value of NCI: IFRS-3, allows/requires goodwill to be stated at full value i.e. a
part of goodwill shall now be attributable to NCI.
Now goodwill impairment shall be charged not only to be parent company but also to NCI
Dr. NCI
Dr. Consolidated Reserves
Cr. Goodwill
Goodwill in consolidated Statement of Financial Position:
Acquisition-date fair value of consideration transferred by parent X
Plus:Fair (or full) value of the N-CI at date of acquisition X
Less:Fair value of subsidiary's identifiable net assets
at date of acquisition (X)
Equals: Total Goodwill X

Impairment Of Goodwill
Under this approach the goodwill appearing in the consolidated Statement of Financial
Position is the total goodwill. The accounting treatment will be:

438
Dr Group retained Earnings X
Dr Non-controlling interest X
Cr Goodwill X

CONSOLIDATED STATEMENT PROFIT OR LOSS AND OTHER COMPREHENSIVE


INCOME
 The basic idea is to show the results of the group as if it were a single entity.
 The majority of figures are simple aggregations of the results of the parent and all the
subsidiaries (line by line) down to profit after tax.
 In aggregating the results of the parent and subsidiaries, intra-group transactions such
as dividend income, interest income and unrealised profits are eliminated.
 Any non-controlling interest is ignored until profit after tax. Their interest in profits after
tax is then subtracted as a one-liner to leave profits attributable to members of the
parent.

P group plc - Pro-forma Consolidated statement of profit or loss For year ended 30 November
2016
GHC'm
Sales revenue (P+S less intra-group sales) X
Cost of Sales (P+S less intra-group purchases plus unrealised profit in inventory) (X)
Gross Profit X
Distribution Costs (P+S) (X)
Administrative Expenses (P+S) (X)
Group operating Profit X
Interest and similar income receivable (P+S less intra group interest income) X
Interest expenses (P+S less intra-group interest expense) (X)
X
Share of Profits of Associate (PAT) X
Profit before tax X
Income tax expense (P+S) (X)
Profit for the period X
Profit attributable to :
Owners of the parent X
Non-controlling interest X
X

OTHER ADJUSTMENTS
 If the subsidiary is acquired during the current accounting period it is necessary to
apportion the profit for the period between its pre-acquisition and post-acquisition
elements. This is dealt with by determining on a line-by- line basis the post acquisition
figures of the subsidiary.
 After profit after tax in consolidated statement of profit or loss, total profits are divided
between profits attributable to group and profit attributable to NCI
 Any dividends receivable by the parent must be cancelled against dividends paid from
the subsidiary undertaking.

439
 Where group companies trade with each other one will record a sale and the other an
equal amount as a purchase. These items must be removed from the consolidated
statement of profit or loss by cancelling from both sales and cost of sales.
 The unrealized profit adjustment is to increase cost of sales. In case of upstream
transaction, the unrealized profit is deducted from profit attributable to NCI also.
 Investment in loans means an intra-group finance cost as well as inter-group dividends.
 These will cancel out in basically the same way as for dividends.
 Impairment of goodwill is treated as an administration expense unless otherwise stated
 There is no impact of fair value adjustment on acquisition at the statement of profit or
loss. However, any additional depreciation related to such fair value adjustment must be
charged by adding to cost of sales and deducting from profit after tax of subsidiary
while calculating profit attributable to NCI

REVIEW QUESTIONS
IFRS10: Consolidated Financial Statements outlines the requirements for the preparation and
presentation of consolidated financial statements, requiring entities to consolidate other entities
it controls.
Required:
i) Define control (1 mark) ii) Indicate
FOUR (4) circumstances that an entity may not have gained control in another entity but may be
allowed to prepare consolidated financial statements. (4 marks)

SOLUTION
Control is identified by IFRS10 Consolidated Financial Statements as the sole basis for
consolidation and comprises the following three elements: a. Power over the investee, where the
investor has current ability to direct activities that significantly affect the investee‘s returns; b.
Exposure, or right to, variable returns from involvement in the investee; and c. The ability to use
the power over the investee to affect the amount of the investor‘s returns. Control is the power
to govern the financial and operating policies of an entity so as to obtain benefits from its
activities. Control is presumed where the acquirer acquires more than one-half of that other
entity‘s voting rights (unless it can be demonstrated that such ownership does not constitute
control). (1 mark)

ii) Control may also have been obtained, even when one of the combining entities does not
acquire more than one-half of the voting rights of another, if, as a result of the business
-half of the voting rights of the other entity
by virtue of an agreement with other investors; or
y under a statue or an
agreement; or

equivalent governing body of the other entity; or


uivalent
governing body of the other entity.

QUESTION 1
On 1 October 2015, Palermo Co acquired 60% of Sunderland Co‘s equity shares by means of a
share exchange of one new share in Palermo Co for every two acquired shares in Sunderland

440
Co. In addition, Palermo Co will pay a further GHC0·54 per acquired share on 30 September
2016.
Palermo Co has not recorded any of the purchase consideration and its cost of capital is 8% per
annum.
The market value of Palermo Co‘s shares at 1 October 2015 was GHC3·00 each.
The summarised statements of financial position of the two companies as at 31 March 2016 are:
Palermo Co Sunderland Co
GHC’000 GHC’000
Assets
Non-current assets
Property, plant and equipment (note (i)) 25,400 13,500
Financial asset: equity investments (note (iv)) 5,500 2,000
––––––– –––––––
30,900 15,500
––––––– –––––––

Current assets
Inventory (note (iii)) 12,700 5,300
Other current assets 9,700 4,000
––––––– –––––––
22,400 9,300
––––––– –––––––
Total assets 53,300 24,800
––––––– –––––––
Equity and liabilities
Equity
Equity shares of GHC1 each 20,000 9,000
Retained earnings:
Brought forward at 1 April 2015 12,200 8,600
Profit/(loss) for the year ended 31 March 2016 5,000 (3,000)
––––––– –––––––
37,200 14,600
Non-current liabilities
Deferred tax (note (i)) 5,000 nil
Current liabilities 11,100 10,200
––––––– –––––––
Total equity and liabilities 53,300 24,800
––––––– –––––––
The following information is relevant:
(i) At the date of acquisition, Palermo Co conducted a fair value exercise on Sunderland Co‘s
net assets which were equal to their carrying amounts (including Sunderland Co‘s financial
asset equity investments) with the exception of an item of plant which had a fair value of
GHC2·5 million below its carrying amount. The plant had a remaining useful life of 30 months
at 1 October 2015.
The directors of Palermo Co are of the opinion that an unrecorded deferred tax asset of GHC1·2
million at 1 October 2015, relating to Sunderland Co‘s losses, can be relieved in the near future

441
as a result of the acquisition. At 31 March 2016, the directors‘ opinion has not changed, nor has
the value of the deferred tax asset.

(ii) Palermo Co‘s policy is to value the non-controlling interest at fair value at the date of
acquisition. For this purpose, a share price for Sunderland Co of GHC1·50 each is representative
of the fair value of the shares held by the non- controlling interest.

(iii) At 31 March 2016, Sunderland Co held goods in inventory which had been supplied by
Palermo Co at a mark-up on cost of 35%. These goods had cost Sunderland Co GHC2·43
million.

(iv) The financial asset equity investments of Palermo Co and Sunderland Co are carried at their
fair values at 1 April 2015.At 31 March 2016, these had fair values of GHC6·1 million and
GHC1·8 million respectively, with the change in Sunderland Co‘s investments all occurring
since the acquisition on 1 October 2015.

(v) There is no impairment to goodwill at 31 March 2016.

Required:
Prepare the following extracts from the consolidated statement of financial position of Palermo
Co as at 31 March 2016:

(i) Goodwill; (6 marks)


(ii) Retained earnings; (7 marks)
(iii) Non-controlling interest. (2 marks)
(15 marks)
Solution:
Workings:
Net Assets of Sunderland Co
At Acquisition At Reporting Post Acquisition
GHC’000 GHC’000 GHC’000

Equity shares 9,000 9,000


Retained Earnings (8600+(6/12x-3000) 7,100 (8600-3000) 5,600 (1,500)
Fair value adjustments: plant (2,500) (2,500)
Fair Value Depn.(6/30x2,500) 500 500
Deferred tax asset 1,200 1,200
Loss on Financial Asset Equity Investment (200) (200)
––––––– ––––––– –––––––

14,800 13,600 (1,200)


––––––– ––––––– –––––––

(i) Goodwill in Sunderland Co


GHC’000
Controlling interest
Share exchange (9,000 x 60% x 1/2 x GHC3·00) 8,100

442
Deferred consideration (9,000 x 60% x 0·54 x 1/1·08) 2,700
Non-controlling interest (9,000 x 40% x GHC1·50) 5,400
–––––––
16,200
Net Asset at Acquisition (see above) 14,800
–––––––
Goodwill arising on acquisition 1,400
–––––––
(ii) Consolidated retained earnings
GHC’000
Palermo Co‘s retained earnings (12,200 + 5,000) 17,200
Sunderland Co‘s post-acquisition losses (1,200 (see above) x 60%) (720)
URP in inventory (see below) (630)
Finance cost of deferred consideration (2,700 (w (i)) x 8% x 6/12) (108)
Profit on equity investments (6,100 – 5,500) 600
–––––––
16,342
–––––––

Sunderland Co‘s inventory at 31 March 2016 is GHC2·43 million, at a mark-up on cost of 35%
there would be GHC630,000 of unrealised profit (URP) (2,430 x 35/135) in inventory.

(iii) Non-controlling interest


GHC’000
Fair value on acquisition (w (i)) 5,400
Post-acquisition losses (1,200 (w (ii) x 40%) (480)
––––––
4,920
–––––

QUESTION 2
Pedro, a public listed company, acquired 600 million equity shares in Simeone on 1 April 2016.
The purchase consideration was made up of:

a share exchange of one share in Pedro for two shares in Simeone


the issue of GHC100 10% loan note for every 500 shares acquired; and
a deferred cash payment of 11 pesewas per share acquired payable on 1 April 2017.

Pedro has only recorded the issue of the loan notes. The value of each Pedro share at the date of
acquisition was75 pesewas and Pedro has a cost of capital of 10% per annum.

The Stetement of Financial Position of the two companies at 31 March 2017 are shown below:
Pedro Simeone
GHC million GHC million GHC million GHC million
Assets
Property, plant and equipment (note (i)) 640 340
Investments 120 nil

443
Intellectual property (note (ii)) nil 30
–––– ––––
760 370
Current assets
Inventory (note (iii)) 76 22
Trade receivables (note (iii)) 84 44
Bank nil 160 4 70
–––– –––– ––– –––––
Total assets 920 440
–––– ––––
Equity and liabilities
Equity shares of 25 pesewas each 300 200
Retained earnings – 1 April 2016 210 120
– year ended 31 March 2017 90 300 20 140
––– ––– –––– ––––––
600 340
Non-current liabilities
10% loan notes 120 20
Current liabilities
Trade payables (note (iii)) 130 57
Current tax payable 45 23
Overdraft 25 200 nil 80
––– ––– –––– ––––––
Total equity and liabilities 920 440
––– –––––
The following information is relevant:
(i) At the date of acquisition the fair values of Simeone‘s net assets were approximately equal to
their carrying amounts with the exception of its properties. These properties had a fair value of
GHC40 million in excess of their carrying amounts which would create additional depreciation
of GHC2 million in the post acquisition period to 31 March 2017. The fair values have not been
reflected in Simeone‘s balance sheet.

(ii) The intellectual property is a system of encryption designed for internet use. Simeone has
been advised that government legislation (passed since acquisition) has now made this type of
encryption illegal. Simeone will receive GHC10 million in compensation from the government.

(iii) Simeone sold to Pedro goods for GHC15 million in the post acquisition period. GHC5
million of these goods are included in the inventory of Pedro at 31 March 2017. The profit made
by Simeone on these sales was GHC6 million.Simeone‘s trade payable account (in the records of
Pedro) of GHC7 million does not agree with Pedro‘s trade receivable account (in the records of
Simeone) due to cash in transit of GHC4 million paid by Pedro.

(iv) Due to the impact of the above legislation, Pedro has concluded that the consolidated
goodwill has been impaired by GHC27 million.

Required:

444
Prepare the consolidated statement of financial position of Pedro as at 31 March 2017.
(25 marks)
Solution:
Consolidated Statement of financial position Pedro as at 31 March 2017:
GHC million GHC million
Assets
Non-current assets
Property, plant and equipment (640 + 340 + 40 – 2) 1,018
Intangible : Consolidated goodwill (135 (w (i)) – 27 impairment) 108
––––––
1,126
Current assets
Inventory (76 + 22 – 2 URP) 96
Trade receivables (84 + 44 – 11 intra-group) 117
Receivable re intellectual property 10
Bank 4 227
–––– ––––––
Total assets 1,353
––––––
Equity and liabilities
Equity attributable to equity holders of the parent
Equity shares 25p each (w (i)) 375
Reserves:Share Premium (w (i)) 150
Retained earnings (w (ii)) 264 414
–––– ––––––
789
Non Controlling interest (w (iii)) 89
––––––
Total equity 878
Non-current liabilities
10% loan notes (120 + 20) 140
Current liabilities
Trade payables (130 + 57 – 7 intra-group) 180
Cash consideration due 1 April 2017 (60 + 6 interest) 66
Overdraft (25 – 4 CIT) 21
Taxation (45 + 23) 68 335
––––– –––––
Total equity and liabilities 1,353
––––––

Workings (Note: all figures in GHC million)


(i)Goodwill: The acquisition of 600 million shares represents 75% of Simeone‘s 800 million
shares (GHC200m/25p). The share exchange of 300 million (i.e. 1 for 2) at GHC0·75 each will
result in an increase in equity share capital of GHC75 million (the nominal value)and create a
share premium balance of GHC150 million (i.e. GHC0·50 premium on 300 million shares).

Consideration:

445
Equity shares (600/2 x GHC0·75) 225
10% loan notes (see below) 120
Cash (600 x GHC0·11/1·1 i.e. discounted at 10%) 60
––––
405
Net Assets at Acquisition (270)
––––––––
135
Impairment (27)
––––
Goodwill 108

The issue of the 10% loan notes is calculated as 600 million/500 x GHC100 = GHC120 million.

(ii) Retained earnings:


Pedro 300
Interest on deferred consideration (60 x 10%) (6)
Goodwill impairment (from question) (27)
Simeone (75% x -4) (3)
––––
264
––––
The unrealised profit in inventory (URP) is GHC5m/GHC15m of the profit of GHC6 million
made by Simeone.

(iii) Non Controlling interest


Net Assets at reporting (356x 25%) 89
–––––––
(iv) Net Assets of Simeone Co
At Acquisition At Reporting Post Acquisition
GHC’m GHC’m GHC’m

Equity shares 200 200


Retained Earnings 120 140 20
Fair value adjustments: Property 40 40
FV Depreciation –Property (2) (2)
Intellectual Property written down - (20) (20)
Development Expenditure (18) (40) (22)
URP (5/15 x 6) - (2) (2)
––––––– ––––––– –––––––

360 356 (4)


––––––– ––––––– –––––––

446
IAS 28 – INVESTMENTS IN ASSOCIATES
SCOPE
This Standard shall be applied in accounting for investments in associates.

DEFINITIONS
The following terms are used in this Standard with the meanings specified:-
 An associate is an entity, including an unincorporated entity such as a partnership, over
which the investor has significant influence and that is neither a subsidiary nor an
interest in a joint venture.
 The equity method is a method of accounting whereby the investment is initially
recognized at cost and adjusted thereafter for the post-acquisition change in the
investor‘s share of net assets of the investee. The profit or loss of the investor includes
the investor‘s share of the profit or loss of the investee.
 Significant influence is the power to participate in the financial and operating policy
decisions of the investee but is not control or joint control over those policies.
Investments of 20% to 50% in voting power of companies lead to existence of significant
influence.

Significant influence by an investor is usually evidenced in one or more of the


following ways:-
a. Representation on the board of directors or equivalent governing body of the investee.
b. Participating in policy making process, including participation in decisions about dividends
or other distributions.
c. Material transactions between the investor and the investee
d. Interchange of managerial personnel; or e. Provision of essential technical information.

ACCOUNTING OF ASSOCIATE
Associate should be accounted for in consolidated financial statement using equity
method; i.e. investment is
 Initially recorded at cost;
 Adjusted for post acquisition change in net assets (investor share); Or post acquisition
profits/losses (investor share);
 The profit or loss of the investor includes the investor‘s share of the profit or loss of the
investee.

447
 Dividend paid or distributions made will reduce the investment.
 On acquisition any difference between the cost of investment and investor‘s share of net
fair value of associate‘s identifiable assets, liabilities and contingent liabilities is
accounted for in accordance with IFRS-3.
 Goodwill relating to an associate is included in the carrying value of investment
 Any excess of the investor‘s share of net fair value of the associate‘s assets,
liabilities and contingent liabilities over the cost of investment is excluded from
the carrying value of investment and is included in the income statement of the
year of acquisition.
 Adjustments in investor‘s share of profit and loss after acquisition are made in respect of
depreciation based on Fair Value.
 If different reporting dates, adjust the effect of significant events between reporting
dates;
 The investor‘s financial statements shall be prepared using uniform accounting policies
for like transactions and events in similar circumstances.
 If the investor‘s share of losses exceeds or equals its interest in associate, the investor
will discontinue the recognition of further losses. Additional losses can only be
recognized if there exist any legal or constructive obligation
 Impairment test will be applied on the entire amount of investment under IAS -36 and
the impairment loss will be recognized.
 In case of trading between group and associate, the profits or losses resulting from these
transactions are recognized in the investor‘s financial statements only to the extent of
un-related investor‘s interest in associate.
 No netting-off is done between receivables and payables

EXCEPTIONS TO THE EQUITY METHOD


An investment in an associate shall be accounted for using the equity method except
when:
1) There is an evidence that the investment is acquired and held exclusively with a view to its
disposal within twelve months from acquisition date (Then apply IFRS -5).
2) All of the following apply:
a. The investor is a wholly-owned subsidiary its other owners do not object if the investor does
not apply the equity method;
b. The investor‘s debt or equity instruments are not traded in a public market
c. The investor did not file its financial statements with securities commission, and
d. The ultimate parent of the investor produces consolidated financial statements.

Some noteworthy points include:


Investment described in 1 above shall be classified as held for trading and accounted
for in accordance with IFRS-5.

EQUITY METHOD
Statement of profit or loss
Dividend income from associates (reported in the investor's books) is replaced by the
profit after tax of the associate.

STATEMENT OF FINANCIAL POSITION

448
Initially the Investments in Associates is shown at cost (same as in the individual
accounts), identifying any goodwill included in the cost.
In subsequent years the Investor's accounts will show:
 the investment at cost
 Plus group share of associate's post acquisition reserves.
 Less any impairment of investment to date.

On the bottom of the balance sheet consolidated reserves will reflect the other side of
these adjustments.
Method GHC
Cost of Investment X
Plus group share of post acquisition reserves X
Less impairment of investment (X)
INVESTMENT IN ASSOCIATES X

TRY
QUESTION 1
Below are the separate financial statements of Broad Ltd and two investee companies
which also operate in the same industry as the investor entity.

Statements of profit or loss and other comprehensive income for the year ended 30
June 2016
Broad Ltd Narrow Ltd Shadow Ltd
GH¢’000 GH¢’000 GH¢’000
Revenue 92,500 48,000 30,000
Cost of sales (70,500) (36,000) (18,000)
Gross profit 22,000 12,000 12,000
Distribution costs (2,500) (1,200) (1,000)
Administrative expenses (5,500) (2,400) (2,000)
Finance costs (100) - -
Profit before tax 13,900 8,400 9,000
Income tax (3,900) (1,600) (2,200)
Profit for the year 10,000 6,800 6,800
Other comprehensive income:
Gain on revaluation of land 500 - -
Total comprehensive income 10,500 6,800 6,800

Statements of financial position as at 30 June 2016


Assets
Non-current assets: GH¢’000 GH¢’000 GH¢’000
Property, plant and equipment 18,300 18,900 15,000
Investments 12,600 - -
30,900 18,900
15,000
Current assets 12,500 2,400 3,000
Total assets 43,400 21,300 18,000
Equity and liabilities
Equity:

449
Equity shares of GH¢1 each 15,000 5,000 6,000
Land revaluation reserve – 30 June 2016 2,000 - -
Other equity reserve – 30 June 2015 500 - -
Retained earnings 12,900 9,500 5,000
30,400 14,500 11,000
Non-current liabilities:
6% loan note 3,000 - -
Current liabilities 10,000 6,800 7,000
Total equity and liabilities 43,400 21,300 18,000

The following information is relevant:


i) On 1 March 2016, Broad Ltd acquired 80% of the equity share capital of Narrow Ltd. The
consideration consisted of two elements: a share exchange of three shares in Broad Ltd for
every five acquired shares in Narrow Ltd and the issue of a GH¢100 6% loan note for every
500 shares acquired in Narrow Ltd. The share issue has not yet been recorded by Broad
Ltd, but the issue of the loan notes has been recorded. At the date of acquisition, shares in
Broad Ltd had a market value of GH¢5 each and the shares of Narrow Ltd had a stock
market price of GH¢3.50 each.
Broad Ltd had earlier on acquired 2.4 million shares in Shadow Ltd on the stock market at a
price of GH¢1.50 per share on 1 January 2016.
ii) At the date of acquisition, the fair values of Narrow Ltd.‟s assets were equal to their
carrying amounts with the exception of its property. This had a fair value of GH¢1.2 million
below its carrying amount. This would lead to a reduction of the depreciation charge (in cost
of sales) of GH¢50,000 in the post-acquisition period. Narrow Ltd has not incorporated this
value change into its separate financial statements.

iii) Broad„s group policy is to revalue all properties to current value at each year end. On 30
June 2016, the value of Narrow„s property was unchanged from its value at acquisition, but
the land element of Broad Ltd.‟s property had increased in value by GH¢500,000 as shown
in other comprehensive income.

iv) Sales from Narrow Ltd to Broad Ltd throughout the year ended 30 June 2016 was
GH¢12 million. Narrow made a mark-up on cost of 25% on these sales. Broad Ltd had
GH¢2 million (at cost to Broad Ltd) of inventory that had been supplied in the post-
acquisition period by Narrow Ltd as at 30 June 2016.

v) In June 2016, Broad Ltd sold goods to Shadow Ltd for GH¢2,000,000, thus achieving a
profit mark-up of 25%. The entire consignment remained unsold and was included in the
inventory of Shadow Ltd as at 30 June 2016.

vi) Broad‟s investments include some available-for-sale investments that have increased in
value by GH¢300,000 during the year. The other equity reserve relates to these
investments and is based on their value as at 30 June 2015. There were no acquisitions or
disposals of any of these investments during the year ended 30 June 2016.

vii) Broad‟s policy is to value the non-controlling interest at fair value at the date of
acquisition. For this purpose, Narrow‟s share price at that date can be deemed to be
representative of the fair value of the shares held by the non-controlling interest.

450
viii) It was determined at the year-end that 10% of the goodwill relating to the acquisition of
Narrow was impaired.

Required:
a) Prepare the consolidated statement of profit or loss and other comprehensive income for
Broad Ltd. Group for the year ended 30 June 2016. (10 marks)
b) Prepare the consolidated statement of financial position for Broad Ltd. Group as at 30
June 2016. (10 marks)
(Total: 20 marks)

REVIEW QUESTIONS
QUESTION 1
The statements of financial position of Precious Co and its investee companies,Super Co and
Austin Co,at 31 December,2009 are shown below.
STATEMENTS OF FINANCIAL POSITION AS AT 31 DECEMBER,2009
Precious Co Super Co Austin Co
GHC000 GHC000 GHC000
Non Current Assets
Freehold Property 1,950 1,250 500
Plant & Machinery 795 375 285

Investments 1,500 - -
4,245 1,625 785
Current Assets
Inventory 575 300 265
Trade Receivables 330 290 370
Cash 50 120 20
955 710 655
Total Assets 5,200 2,335 1,440
Equity & Liabilities
Equity
Share Capital-GHC1 shares 2,000 1,000 750
Retained Earnings 1,460 885 390
3,460 1,885 1,140
Non Current Liabilities
12% loan stock 500 100
Current Liabilities
Trade Payables 680 350 300
Bank Overdraft 560 - -
1,240 350 300
Total Equity & Liabilities 5,200 2,335 1,440
Additional Information:
(a) Precious Co acquired 600,000 ordinary shares in Super Co on 1st January,2004 for GHC
1,000,000 when the income surplus balance of Super Co was GHC 200,000.

451
(b) At the date of acquisition of Super Co,the fair value of its freehold property was
considered to be GHC400,000 greater that its value in Super Co‘s balance sheet.Super
Co had acquired the property in January 1994 and the building element(comprising 50%
of the total value) is depreciated on cost over 50 years.
(c) Precious Co acquired 225,000 ordinary shares in Austin Co on 1 January,2008 for GHC
500,000 when the retained earnings of Austin Co were GHC150,000.
(d) Super Co manufactures a component used by both Precious Co and Austin Co.Transfers
are made by Super CO at cost plus 25%.Precious Co held GHC100,000 inventory of these
components at 31 December,2009 and Austin Co held GHC80,000 at the same date.
(e) The goodwill in Super Co is impaired and should be fully written off.An impairment
loss of GHC92,000 is to be recognized on the investment in Austin Co.
(f) Non-controlling interest is valued at proportionate share of net assets.

Required:
(a)Prepare the consolidated Statement of Financial Position of Precious Group at 31
Decemeber,2009.
(b) Under IFRS 10, Consolidated Financial Statements, a parent entity is allowed not to present
or prepare consolidated financial statements if certain conditions prevail.
Required:
State any four (4) conditions (4 marks)

Solution:
PRECIOUS GROUP
CONSOLIDATED STATEMENT OF FINANCIAL POSITION AS AT 31 DECEMBER 2009
GHC'000
Non-current assets
Freehold property (W2) 3,570.00
Plant and machinery (795 + 375) 1,170.00
Investment in associate (W7) 475.20
5,215.20
Current assets
Inventory (W3) 855.00
Receivables (330 + 290) 620.00
Cash (50 + 120) 170.00 1,645.00
Total assets 6,860.20
Equity and liabilities
Equity
Share capital 2,000.00
Retained earnings (W8) 1,792.20
3,792.20
Non-controlling interest (W9) 878.00
4,670.20
Non-current liabilities
12% loan stock (500 + 100) 600.00
Current liabilities (680 + 560 + 350) 1,590.00
Total equity and liabilities 6,860.20

452
Workings:
1 Group structure

2 Freehold property GHC'000


P Co 1,950
S Co 1,250
Fair value adjustment 400
Additional depreciation (400 x 50% / 40) x 6 years (2004-2009) (30)
3,570

3. Inventory GHC'000
P Co 575
S Co 300
PUP (100 x 25/125) (W4) (20)
855

4. Unrealised profit (PUP) GHC'000


On sales by S to P (parent co) 100 x 25/125 20.0
On sales by P to A (associate) 80 x 25/125 x 30% 4.8

5 Fair value adjustments


Difference at acquisition Difference now
GHC'000 HC'000
Property 400 400
Additional depreciation: 200 x 6/40 – (30)
400 370
: Charge GHC30,000 to retained earnings

6 Goodwill GHC'000 GHC'000


S Co
Consideration transferred 1,000
Non-controlling interest (400 x GHC1.60) 640
Net assets acquired
Share capital 1,000
Retained earnings 200
Fair value adjustment 400 (1,600)
Goodwill at acquisition 40
Impairment loss (40)
0

7 Investment in associate GHC'000


Cost of investment 500.00
Share of post-acquisition profit (390 – 150) x 30% 72.00
Less PUP (4.80)
Less impairment loss (92.00)

453
475.20

8 Retained earnings
P S A
GHC'000 GHC'000 GHC'000
Retained earnings per question 1,460.0 885.0 390.0
Adjustments:
Unrealised profit (W4) (4.8) (20.0)
Fair value adjustments (W5) (30.0)
Impairment loss (P) (40.0)
795.0 390.0
Less pre-acquisition reserves (200.0) (150.0)
1,455.20 595.0 240.0
P: 60% x 595 357.00
S: 30% x 240 72.00
Impairment loss S (92.00)
1,792.20

9 Non-controlling interest at reporting date GHC'000


NCI at acquisition (W6) 640.00
Share of post-acquisition retained earnings (595 x 40%) 238.00
878.00

QUESTION 2
On 1 October 2010, Paladin secured a majority equity shareholding in Saracen on the following
terms:
an immediate payment of GHC4 per share on 1 October 2010; and
a further amount deferred until 1 October 2011 of GHC5·4 million.
The immediate payment has been recorded in Paladin‘s financial statements, but the deferred
payment has not been recorded. Paladin‘s cost of capital is 8% per annum.
On 1 February 2011, Paladin also acquired 25% of the equity shares of Augusta paying GHC10
million in cash.
The summarised statements of financial position of the three companies at 30 September
2011 are:
Paladin Saracen Augusta
Assets GHC’000 GHC’000 GHC’000
Non-current assets
Property, plant and equipment 40,000 31,000 30,000
Intangible assets 7,500
Investments – Saracen (8 million shares at GHC4 each)32,000
– Augusta 10,000 nil nil
–––––––– ––––––– –––––––
89,500 31,000 30,000

Current assets
Inventory 11,200 8,400 10,000
Trade receivables 7,400 5,300 5,000

454
Bank 3,400 nil 2,000
–––––––– ––––––– –––––––
Total assets 111,500 44,700 47,000
–––––––– ––––––– –––––––
Equity and liabilities
Equity
Equity shares of GHC1 each 50,000 10,000 10,000
Retained earnings – at 1 October 2010 25,700 12,000 31,800
– for year ended 30 September 2011 9,200 6,000 1,200
–––––––– ––––––– –––––––
84,900 28,000 43,000
Non-current liabilities
Deferred tax 15,000 8,000 1,000
Current liabilities
Bank nil 2,500 nil
Trade payables 11,600 6,200 3,000
–––––––– ––––––– –––––––
Total equity and liabilities 111,500 44,700 47,000
–––––––– ––––––– –––––––

The following information is relevant:


(i) Paladin‘s policy is to value the non-controlling interest at fair value at the date of acquisition.
For this purpose the directors of Paladin considered a share price for Saracen of GHC3·50 per
share to be appropriate.
(ii) At the date of acquisition, the fair values of Saracen‘s property, plant and equipment was
equal to its carrying amount with the exception of Saracen‘s plant which had a fair value of
GHC4 million above its carrying amount. At that date the plant had a remaining life of four
years. Saracen uses straight-line depreciation for plant assuming a nil residual value.
Also at the date of acquisition, Paladin valued Saracen‘s customer relationships as a customer
base intangible asset at fair value of GHC3 million. Saracen has not accounted for this asset.
Trading relationships with Saracen‘s customers last on average for six years.
(iii) At 30 September 2011, Saracen‘s inventory included goods bought from Paladin (at cost to
Saracen) of GHC2·6 million. Paladin had marked up these goods by 30% on cost. Paladin‘s
agreed current account balance owed by Saracen at 30 September 2011 was GHC1·3 million.
(iv) Impairment tests were carried out on 30 September 2011 which concluded that consolidated
goodwill was not impaired, but, due to disappointing earnings, the value of the investment in
Augusta was impaired by GHC2·5 million.
(v) Assume all profits accrue evenly through the year.
Required:
Prepare the consolidated statement of financial position for Paladin as at 30 September 2011.
(25 marks)

Solution:
Consolidated statement of financial position of Paladin as at 30 September 2011
GHC‘000 GHC‘000
Assets
Non-current assets:

455
Property, plant and equipment (40,000 + 31,000 + 4,000 – 1,000) 74,000
Intangible assets (w (i))
– goodwill 15,000
– other intangibles (7,500 + 3,000 – 500) 10,000
Investment in associate (w (ii)) 7,700
––––––––
106,700
Current assets
Inventory (11,200 + 8,400 – 600 URP (w (iii))) 19,000
Trade receivables (7,400 + 5,300 – 1,300 intra-group (w (iii))) 11,400
Bank 3,400 33,800
––––––– ––––––––
Total assets 140,500
––––––––

Equity and liabilities


Equity attributable to owners of the parent
Equity shares of GHC1 each 50,000
Retained earnings (w (iv)) 35,200
––––––––
85,200
Non-controlling interest (w (vi)) 7,900
––––––––
Total equity 93,100
Non-current liabilities
Deferred tax (15,000 + 8,000) 23,000
Current liabilities
Bank overdraft 2,500
Deferred consideration 5,400
Trade payables (11,600 + 6,200 – 1,300 intra-group (w (iii))) 16,500 24,400
––––––– ––––––––
Total equity and liabilities 140,500
––––––––
Workings:
(i) Goodwill in Saracen
GHC‘000 GHC‘000
Controlling interest (see below)
Immediate cash 32,000
Deferred consideration (5,400 x 100/108) 5,000
Non-controlling interest (10,000 x 20% (see below) x GHC3·50) 7,000
–––––––
44,000
Equity shares 10,000
Pre-acquisition reserves:
At 1 October 2010 12,000
Fair value adjustments – plant 4,000
– intangible 3,000 (29,000)

456
––––––– –––––––
Goodwill arising on acquisition 15,000
–––––––
The cost of the majority shareholding in Saracen was GHC32 million. Paladin acquired eight
million shares and Saracen has 10 million GHC1 shares, this gives a controlling interest of 80%
and a non-controlling interest of 20%.
The customer relationship asset is recognised as an intangible asset in the consolidated financial
statements under IFRS 3 Business combinations.
(ii) Carrying amount of Augusta at 30 September 2011
GHC‘000
Cash consideration 10,000
Share of post-acquisition profits (1,200 x 8/12 x 25%) 200
Impairment loss (2,500)
––––––
7,700
––––––
(iii) Unrealised profit (URP) in inventory/intra-group current accounts
The URP in Saracen‘s inventory (supplied by Paladin) of GHC2·6 million is GHC600,000
(2,600 x 30/130). The current account balances of Paladin and Saracen should be eliminated
from trade receivables and payables at the agreed amount of GHC1·3 million.
(iv) Consolidated retained earnings:
GHC‘000
Paladin‘s retained earnings (25,700 + 9,200) 34,900
Saracen‘s post-acquisition profits (4,500 (w (v)) x 80%) 3,600
Augusta‘s post-acquisition profits (w (ii)) 200
Augusta‘s impairment loss (2,500)
URP in inventory (w (iii)) (600)
Finance cost of deferred consideration (5,000 x 8%) (400)
–––––––
35,200
–––––––
(v) Post-acquisition adjusted profit of Saracen is:
GHC‘000
Profit as reported 6,000
Additional depreciation of plant (4,000/4 years) (1,000)
Additional amortisation of customer relationship asset (3,000/6 years) (500)
––––––
4,500
––––––
(vi) Non-controlling interest
GHC‘000
Fair value on acquisition (w (i)) 7,000
Post-acquisition profits (4,500 (w (v)) x 20%) 900
––––––
7,900
–––––
QUESTION 3

457
Below are the summarised statements of financial position for three companies as at 31 March
2017:
Pochetino Sarri Alonso
Assets
GHC m GHCm GHC m GHC m GHC m GHC m
Non-current assets
Property, plant and equipment 520 280 240
Investments 345 40 nil
–––––– –––– ––––
865 320 240
Current assets
Inventory 142 160 120
Trade receivables 95 88 50
Cash and bank 8 245 22 270 10 180
–––– –––––– –––– –––– –––– ––––
Total assets 1,110 590 420
–––––– –––– ––––
Equity and liabilities
Equity shares of GHC1each 500 145 100
Share premium 100 nil nil
Retained earnings 130 230 260 260 240 240
–––– –––––– –––– –––– –––– ––––
730 405 340
Non-current liabilities
10% loan notes 180 20 nil
Current liabilities 200 165 80
–––––– –––– ––––
Total equity and liabilities 1,110 590 420
–––––– –––– ––––
Notes:
Pochetino is a public listed company that acquired the following investments:
(i) Investment in Sarri
On 1 April 2015 Pochetino acquired 116 million shares in Sarri for an immediate cash payment
of GHC210 million and issued at par one 10% GHC100 loan note for every 200 shares acquired.
Sarri‘s retained earnings at the date of acquisition were GHC120 million.

(ii) Investment in Alonso


On 1 October 2016 Pochetino acquired 30 million shares in Alonso in exchange for 75 million of
its own shares. The stock market value of Pochetino‘s shares at the date of this share exchange
was GHC1·60 each.Pochetino has not yet recorded the investment in Alonso.

(iii) Pochetino‘s other investments, and those of Sarri, are available-for-sale investments which
are carried at their fair values as at 31 March 2016. The fair value of these investments at 31
March 2017 is GHC82 million and GHC37 million respectively.

Other relevant information:

458
(iv) Pochetino‘s policy is to value non-controlling interests at their fair values. The directors of
Pochetino assessed the fair value of the non-controlling interest in Sarri at the date of
acquisition to be GHC65 million.
There has been no impairment to goodwill or the value of the investment in Alonso.

(v) At the date of acquisition of Sarri owned a recently built property that was carried at its
(depreciated) construction cost of GHC62 million. The fair value of this property at the date of
acquisition was GHC82 million and it had an estimated remaining life of 20 years.
For many years Sarri has been selling some of its products under the brand name of ‗Esquisito‘.
At the date of acquisition the directors of Pochetino valued this brand at GHC25 million with a
remaining life of 10 years. The brand is not included in Sarri‘s statement of financial position.
The fair value of all other identifiable assets and liabilities of Sarri were equal to their carrying
values at the date of its acquisition.

(vi) The inventory of Sarri at 31 March 2017 includes goods supplied by Pochetino for GHC56
million (at selling price from Pochetino). Pochetino adds a mark-up of 40% on cost when selling
goods to Sarri. There are no intra-group receivables or payables at 31 March 2017.

(vii) Alonso‘s profit is subject to seasonal variation. Its profit for the year ended 31 March 2017
was GHC100 million.GHC20 million of this profit was made from 1 April 2016 to 30 September
2016.

(viii) None of the companies have paid any dividends for many years.

Required:
Prepare the consolidated statement of financial position of Pochetino as at 31 March 2017.
(25 marks)
Solution
Consolidated statement of financial position of Pochetino as at 31 March 2017:
GHCmillion GHCmillion
Non-current assets
Property, plant and equipment (w (i))
818
Goodwill (w (ii)) 23
Brand (25 – 5) 20
Investment in associate (w (iii)) 144
Other available-for-sale investments (82 + 37) 119
––––––
1,124
Current assets
Inventory (142 + 160 – 16 URP (w (iv))) 286
Trade receivables (95 + 88) 183
Cash and bank (8 + 22) 30 499
––––– ––––––
Total assets 1,623
––––––
Equity and liabilities

459
Equity attributable to the parent
Equity shares (500 + 75 (w (iii))) 575
Share premium (100 + 45 (w (iii)) 145
Retained earnings (w (iv)) 247 392
––––– ––––––
967
Non-controlling interest (w (v)) 91
––––––
Total equity 1,058
Non-current liabilities
10% loan notes (180 + 20) 200
Current liabilities (200 + 165) 365
––––––
Total equity and liabilities 1,623
––––––
Workings:
(all figures in GHC million)
The investment in Sarri represents 80% (116/145) of its equity and is likely to give Pochetino
control thus Sarri should be consolidated as a subsidiary. The investment in Alonso represents
30% (30/100) of its equity and is normally treated as an associate that should be equity
accounted.

Control Structure
Pochetino
Date of Acquisition: 1 April,2015 Group 80% 30% Date of Acq: 1 Oct,2016
Date of Reporting: 31 March,2017 NCI 20% Date of Rep: 31 March,2017
Post Acq. Period : 2 years Post Acq period: 6 months
Sarri Alonso

(i) Property, plant and equipment


Pochetino 520
Sarri 280
Fair value property (82 – 62) 20
Post-acquisition depreciation (2 years) (20 x 2/20 years) (2)
––––
818
––––
(ii) Goodwill in Sarri:
Investment at cost – cash 210
– loan note (116 x GHC100/200) 58
––––
Cost of the controlling interest 268
Fair value of non-controlling interest (from question) 65
Fair value of net assets at acquisition (see below) (310)
––––

460
Goodwill 23
––––
Net Assets of Sarri At Acqusition At Reporting Post Acquisition
GHCm GHCm GHCm
Equity Shares 145 145
Retained earnings 120 260 140
FV Adjustment-Property (82-62) 20 20
FV Depreciation (20/20yrs x 2 years) - (2) (2)
- Brand 25 25
Amortisation-Brand (25/10yrsx2yrs) - (5) (5)
Loss on AFS Investment (37- 40) - (3) (3)
310 440 130

(iii) Investment in associate: GHCmillion


Investment at cost (75 x GHC1·60) 120
Share of post-acquisition profit (100 – 20) x 30% 24
––––
144
––––
The purchase consideration by way of a share exchange (75 million shares in Pochetino for 30
million shares in Alonso) would be recorded as an increase in share capital of GHC75 million
(GHC1 nominal value) and an increase in share premium of GHC45 million (75 million x
GHC0·60).

(iv) Consolidated retained earnings:


Pochetino‘s retained earnings 130
Sarri‘s post-acquisition profits (130 x 80% see above) 104
Gain on investments – Pochetino (see below) 5
Alonso‘s post-acquisition profits (w (iii)) 24
URP in Inventories (56 x 40/140) (16)
––––
247
––––
Gain on the value of Pochetino‘s available-for-sale investments:
Carrying amount at 31 March 2008 (345 – 210 cash – 58 loan note) 77
Carrying amount at 31 March 2009 82
––––
Gain to retained earnings (or other components of equity) 5
––––
(v) Non-controlling interest
Fair value on acquisition (from question) 65
Share of adjusted post acquisition profit (130 x 20% (w (iv))) 26
–––
91
–––

461
QUESTION 4
On 1 May 2014, Pomaa acquired 8 million of the 10 million shares issued by Serebour.
The consideration was as follows:
An initial cash payment of GHS1.00 per share acquired.
A share for share exchange on the basis of 1 share in Pomaa for every 2 shares acquired in
Serebour. The share for share exchange has not yet been recorded by Pomaa, but the cash
payment was recorded.
At the date of acquisition the market price of shares in Pomaa was GHS9.60 per share and the
market price of the shares in Serebour was GHS5.80 per share. Serebour's income surplus and
capital surplus stood at GHS32m and GHS5.3m respectively.
On 1 July 2014, Pomaa acquired 4 million of the 10million shares issued by Asonaba for a cash
payment of GHS21.6m.
Below are the summarised draft financial statements of the three companies.

STATEMENTS OF FINANCIAL POSITION AS AT 31 DECEMBER 2014


Pomaa Serebour Asonaba
GHS'000 GHS'000 GHS'000
Non-current assets
Property, plant and equipment 122,000 54,000 43,000
Investments in equity instruments 29,600
151,600 54,000 43,000
Current assets 44,000 18,000 16,000
Total assets 195,600 72,000 59,000
Equity
Ordinary shares 15,000 10,000 5,000
Income surplus 116,600 40,400 39,000
Capital surplus 12,000 6,500 3,000
143,600 56,900 47,000
Non-current liabilities
5% loan notes 16,000 – –
Current liabilities 36,000 15,100 12,000
Total equity and liabilities 195,600 72,000 59,000

STATEMENTS OF PROFIT OR LOSS AND OTHER COMPREHENSIVE INCOME YEAR


ENDED 31 DECEMBER 2014
Pomaa Serebour Asonaba
GHS'000 GHS'000 GHS'000
Revenue 180,000 78,000 62,000
Cost of sales (108,000) (46,800) (37,200)
Gross profit 72,000 31,200 24,800
Distribution costs (14,700) (6,600) (4,600)
Administrative expenses (18,500) (7,800) (6,200)
Finance costs (800) - -
Profit before tax 38,000 16,800 14,000
Income tax expense (9,500) (4,200) (3,500)

462
PROFIT FOR THE YEAR 28,500 12,600 10,500

Other comprehensive income:


Gains on property revaluation 5,000 1,800 1,200
TOTAL COMPREHENSIVE INCOME FOR THE YEAR 33,500 14,400 11,700

The following information is relevant:


(i) At the date of acquisition, the fair values of Serebour's assets were equal to their carrying
amounts with the exception of the following:
Equipment (remaining useful life of 5 years) which had a fair value of GHS22,400,000 and a
carrying value of GHS20,000,000. Depreciation on equipment is included in cost of sales.
 Inventories which had a fair value of GHS800,000 more than their carrying amount.
The inventories were sold shortly after the acquisition. In addition, Serebour was the
defendant in a court case. It was estimated that the fair value of the potential liability at 1 May
2014 was GHS500,000. The case was eventually settled out of court for GHS800,000 in December
2014.
(ii) Serebour sold raw materials to Pomaa throughout the year ended 31 December 2014 for
GHS1m per month. Serebour made a profit margin of 20% on the sales. GHS2m (at transfer
price) of the raw materials supplied in the post-acquisition period by Serebour remained in
Pomaa's inventories at 31 December 2014.
(iii) Pomaa sold inventories to Asonaba during the final quarter of the year for GHS1,200,000.
These were sold at a mark-up on cost of 20%. Half of these goods were still in Asonaba's
inventories at the year end.
(iv) At 31 December 2014, Pomaa's trade payables included a balance of GHS1,000,000 owed to
Serebour. This agreed with the corresponding trade receivable in Serebour's books.
(v) Pomaa elected to measure the non-controlling interests in Serebour at fair value. Serebour's
share price at the date of acquisition can be deemed to be representative of the fair value of the
shares held by the non-controlling interest shareholders.
(vi) All items of income and expense in the above statements of profit or loss and other
comprehensive income are deemed to accrue evenly over the year unless otherwise stated.
There has been no impairment of consolidated goodwill.
Required
(a) Prepare the consolidated statement of financial position for Pomaa as at 31 December 2014.
(20 marks)
(b) Pomaa is considering acquiring 60% of the ordinary shares in an entity over which they will
have joint control. Explain how such an acquisition will be accounted for in the consolidated
statement of financial position. (5 marks)
(Total = 25 marks)
Solution:
(a) POMAA
CONSOLIDATED STATEMENT OF FINANCIAL POSITION AS AT 31 DECEMBER 2014

GHS'000
Non-current assets
Property, plant and equipment (122,000 + 54,000 + (W8) 2,080) 178,080
Goodwill (W2) 8,000
Investment in associate (W3) 23,900

463
209,980
Current assets (44,000 + 18,000 – (W9) 400 – (W9) 1,000) 60,600
Total assets 270,580
Equity attributable to owners of the parent
Share capital (15,000 + (W7) 38,400) 53,400
Income surplus (W4) 124,564
Capital surplus (W5) 13,200
191,164
Non-controlling interests (W6) 13,316
204,480
Non-current liabilities
5% loan notes 16,000
Current liabilities (36,000 + 15,100 – (W9) 1,000) 50,100
Total equity and liabilities 270,580

Workings
(2)Goodwill GHS'000 GHS'000
Consideration transferred:
– Cash (10,000 × 80% × GHC1.00) 8,000
– Share for share exchange (W7) 38,400
46,400

Fair Value of NCI (2,000 x GHC5.80) 11,600


58,000
Net Assets at reporting (50,000)
Goodwill at acquisition 8,000
Impairment losses to date (0)
Goodwill at year end 8,000

3 Investment in associate
GHS'000
Cost of associate 21,600
Share of post acquisition profits (40% x 5,850) 2,100
Share of post acquisition capital surplus (40% x 600) 240
URP (1/2 x 1,200 x 20/120) 40% (40)
23,900

4 Retained Earnings
Pomaa 116,600
Share of Post Acquisition Profit (80% x 7,380) 5,904
URP (1/2 x 1,200 x 20/120) 40% (40)
Share of post acquisition profit – Asonaba (40% x 5,250) 2,100
124,564
5 Net Assets of Serebour Co
At Acquisition At Reporting Post Acquisition
GHC000 GHC000 GHC000

464
Equity shares 10,000 10,000
Capital Surplus 5,300 6,500 1,200
Retained earnings 32,000 40,400 8,400
Fair value adjustments – Equipment 2,400 2,400
FV Depreciation : Equipment (2,400/5 x 8/12) - (320) (320)
Inventories 800 - (800)
Provision (500) - 500
URP (2,000 x 20%) - (400) (400)
––––––– ––––––– –––––––
50,000 58,580 8,580
––––––– ––––––– –––––––

Net Assets of Asonaba Co


At Acquisition At Reporting Post Acquisition
GHCm GHCm GHCm
Equity shares 5,000 5,000
Retained earnings 33,750 39,000 (6/12 x 10,500) 5,250
Capital Surplus 2,400 3,000 (6/12 x 1,200) 600
––––––– ––––––– –––––––
5,850
––––––– ––––––– –––––––

6 Non Controlling Interest


FV of NCI 11,600
Share of post acquisition profit (20% x 8,580) 1,716
13,316
(b) In the consolidated statement of financial position the joint venture will be recognised, using
equity accounting, in consolidated non current assets and in consolidated reserves.
Consolidated non current assets will include Investments in Joint Ventures at the cost of the
shares acquired plus 60% of the post acquisition change in net assets, less any impairments.
Consolidated reserves will similarly include 60% of the joint venture‘s post acquisition change
in net assets less any impairments.
QUESTION 5
On 1 October 2016 Portia acquired the following non-current investments:
– 3 million equity shares in Selina by an exchange of one share in Portia for every two
shares in Selina plus GHC1 per acquired Selina share in cash. The market price of each Portia
share at the date of acquisition was GHC6.
– 30% of the equity shares of Agnes at a cost of GHC7·50 per share in cash.
Only the cash consideration of the above investments has been recorded by Portia.

The summarised draft statement of financial position of the three companies at 30 September
2017 are:
Portia Selina Agnes
GHC’000 GHC’000 GHC’000
Assets
Non-current assets
Property, plant and equipment 18,400 10,400 18,000

465
Investments in Selina and Agnes 12,000 nil nil
Available-for-sale investments 6,500 nil nil
––––– ––––––– –––––––––
36,900 10,400 18,000
Current assets
Inventory 6,900 6,200 3,600
Trade receivables 3,200 1,500 2,400
–––––– ––––––– ––––––––
Total assets 47,000 18,100 24,000
–––––– ––––––– ––––––––
Equity and liabilities
Equity shares of GHC1 each 10,000 4,000 4,000
Retained earnings – at 30 September 2016 16,000 6,500 11,000
– for year ended 30 September 2017 8,000 2,400 5,000
–––––– ––––––– ––––––––
34,000 12,900 20,000
Non-current liabilities
7% Loan notes 5,000 1,000 1,000
Current liabilities 8,000 4,200 3,000
–––––– ––––––– ––––––––
Total equity and liabilities 47,000 18,100 24,000
–––––– ––––––– ––––––––
The following information is relevant:
(i) At the date of acquisition the fair values of Selina‘s assets were equal to their carrying
amounts with the exception of Selina‘s land which had a fair value of GHC500,000 below its
carrying amount; it was written down by this amount shortly after acquisition and has not
changed in value since then.

(ii) On 1 October 2016, Portia sold an item of plant to Selina at its agreed fair value of GHC2·5
million. Its carrying amount prior to the sale was GHC2 million. The estimated remaining life of
the plant at the date of sale was five years (straight-line depreciation).

(iii) During the year ended 30 September 2017 Selina sold goods to Portia for GHC2·7 million.
Selina had marked up these goods by 50% on cost. Portia had a third of the goods still in its
inventory at 30 September2017. There were no intra-group payables/receivables at 30
September 2017.

(iv) Impairment tests on 30 September 2017 concluded that the value of the investment in Agnes
was not impaired,but consolidated goodwill was impaired by GHC900,000.

(v) The available-for-sale investments are included in Portia‘s balance sheet (above) at their fair
value on 1 October 2016, but they have a fair value of GHC9 million at 30 September 2017

(vi) No dividends were paid during the year by any of the companies.

Required:

466
(a) Prepare the consolidated statement of financial position for Portia as at 30 September 2017.
(20 marks)
(b)A financial assistant has observed that the fair value exercise means that a subsidiary‘s net
assets are included at acquisition at their fair (current) values in the consolidated balance sheet.
The assistant believes that it is inconsistent to aggregate the subsidiary‘s net assets with those of
the parent because most of the parent‘s assets are carried at historical cost.
Required:
Comment on the assistant‘s observation and explain why the net assets of acquired subsidiaries
are consolidated at acquisition at their fair values. (5 marks)
(25 marks)

Solution:
a) Consolidated balance sheet of Portia as at 30 September 2017
GHC’000 GHC’000
Assets
Non-current assets:
Property, plant and equipment (18,400 + 10,400 – 400 (w (i))) 28,400
Goodwill (w (ii)) 3,600
Investments – associate (w (iii)) 10,500
– other available for sale 9,000
–––––––
51,500
Current assets
Inventory (6,900 + 6,200 – 300 URP (w (iv))) 12,800
Trade receivables (3,200 + 1,500) 4,700 17,500
–––––– ––––––––
Total assets 69,000
–––––––
Equity and liabilities
Equity shares of GHC1 each (w (v)) 11,500
Reserves:
Share premium (w (v)) 7,500
Retained earnings (w (vi)) 28,650 36,150
––––––– –––––––
47,650
Non Controlling interest (w (vii)) 3,150
–––––––
Total equity 50,800
Non-current liabilities
7% Loan notes (5,000 + 1,000) 6,000
Current liabilities (8,000 + 4,200) 12,200
–––––––
Total equity and liabilities 69,000
–––––––
Workings (figures in brackets are in GHC’000)

467
(i)Property, plant and equipment
The transfer of the plant creates an initial unrealised profit (URP) of GHC500,000. This is
reduced by GHC100,000 for each year (straight-line depreciation over five years) of depreciation
in the post-acquisition period. Thus at 30 September2007 the net unrealised profit is
GHC400,000. This should be eliminated from Portia‘s retained profits and from the carrying
amount of the plant. The fall in the fair value of the land has already been taken into account in
Selina‘sbalance sheet.
(ii) Goodwill in Savannah:
Investment at cost: GHC‘000 GHC‘000
Shares issued (3,000/2 x GHC6) 9,000
Cash (3,000 x GHC1) 3,000
–––––––
12,000
Share of Net Assets at Acquisition(75% x 10,000) (7,500)
––––––––––––––
Goodwill on consolidation 4,500
Impairment (900)
–––––––
3,600
Goodwill is impaired by GHC900,000 thus has a carrying amount at 30 September 2017 of
GHC3·6 million.
Selina‘s pre-acquisition reserves of GHC6·5 million require an adjustment for a write down of
GHC500,000 in respect of the fair value of its land being below its carrying amount. Thus the
adjusted pre-acquisition reserves of Selina are GHC6 million. A consequent effect is that the
post-acquisition reserves which are reported as GHC2·4 million in Selina‘s balance sheet will
become GHC2·9 million. This is because the fall in the value of the land has effectively been
treated by Selina as a post-acquisition loss.

(iii) Carrying amount of Agnes at 30 September 2017


GHC‘000
Cost (4,000 x 30% x GHC7·50) 9,000
Share post-acquisition profit (5,000 x 30%) 1,500
–––––––
10,500
–––––––
(iv) The unrealised profit (URP) in inventory is calculated as:
Intra-group sales are GHC2·7 million on which Selina made a profit of GHC900,000 (2,700 x
50/150). One third of these are still in the inventory of Portia, thus there is an unrealised profit
of GHC300,000.
(v) The 1·5 million shares issued by Portia in the share exchange at a value of GHC6 each would
be recorded as GHC1 per share as capital and GHC5 per share as share premium giving an
increase in share capital of GHC1·5 million and a share premium of GHC7·5 million.
(vi) Consolidated retained earnings:
GHC‘000
Portia‘s retained earnings 24,000
Selina‘s post-acquisition ((2,900 – 300 URP) x 75%) 1,950

468
Agnes‘s post-acquisition profits (5,000 x 30%) 1,500
URP in plant (see (i)) (400)
Gain on available-for-sale investment (9,000 – 6,500) see below 2,500
Impairment of goodwill (900)
–––––––
28,650
–––––––
The gain on available-for-sale investments must be recognised directly in equity.

(vii) Non Controlling interest


Adjusted equity at 30 September 2017 (12,600 x 25%) 3,150
(viii) Net Assets of Selina At Acquisition At Reporting Post Acquisition
GHC000 GHC000 GHC000
Equity Shares 4,000 4,000
Retained Earnings 6,500 8,900 2,400
FV Adjustments – Land (500) - 500
URP(50/150 x 2,700 x 1/3) - (300) (300)
10,000 12,600 2,600
(b)IFRS 3 Business Combinations requires the purchase consideration for an acquired entity to
be allocated to the fair value of the assets, liabilities and contingent liabilities acquired
(henceforth referred to as net assets and ignoring contingent liabilities) with any residue being
allocated to goodwill. This also means that those net assets will be recorded at fair value in the
consolidated balance sheet. This is entirely consistent with the way other net assets are recorded
when first transacted (i.e.the initial cost of an asset is normally its fair value).
The purpose of this process is that it ensures that individual assets and liabilities are correctly
classified (and valued) in the consolidated balance sheet. Whilst this may sound obvious,
consider what would happen if say a property had a carrying amount of GHC5 million, but a
fair value of GHC7 million at the date it was acquired.If the carrying amount rather than the fair
value was used in the consolidation it would mean that tangible assets (property,plant and
equipment) would be understated by GHC2 million and intangible assets (goodwill) would be
overstated by the same amount (note: in the consolidated balance sheet of Portia the opposite
effect would occur as the fair value of Selina‘s land is below its carrying amount at the date of
acquisition).
There could also be a ‗knock on‘ effect with incorrect depreciation charges in the years
following an acquisition and incorrect calculation of any goodwill impairment. Thus the use of
carrying amounts rather than fair values would not give a ‗faithful representation‘ as required
by the Framework. The assistant‘s comment regarding the inconsistency of value models in the
consolidated balance sheet is a fair point, but itis really a deficiency of the historical cost concept
rather than a flawed consolidation technique. Indeed the fair values of the subsidiary‘s net
assets are the historical costs to the parent. To overcome much of the inconsistency, there would
be nothing to prevent the parent company from applying the revaluation model to its property,
plant and equipment.
QUESTION 6
Hazard purchased the following equity investments:
On 1 October 2015: 80% of the issued share capital of Sarri. The acquisition was through a share
exchange of three shares in Hazard for every five shares in Sarri. The market price of Hazard‘s
shares at 1 October 2015 was GHC5 per share.

469
On 1 July 2016: 6 million shares in Alexis paying GHC3 per share in cash and issuing to Alexis‘s
shareholders 6% (actual and effective rate) loan notes on the basis of GHC100 loan note for
every 100 shares acquired.

The summarised income statements for the three companies for the year ended 30 September
2016 are:
Hazard Sarri Alexis
GHC’000 GHC’000 GHC’000
Revenue 105,000 62,000 50,000
Cost of sales (68,000) (36,500) (61,000)
––––––– –––––––– ––––––––
Gross profit/(loss) 37,000 25,500 (11,000)
Other income (note (i)) 400 nil nil
Distribution costs (4,000) (2,000) (4,500)
Administrative expenses (7,500) (7,000) (8,500)
Finance costs (1,200) (900) nil
––––––– –––––––– –––––––––
Profit/(loss) before tax 24,700 15,600 (24,000)
Income tax (expense)/credit (8,700) (2,600) 4,000
––––––– –––––––– –––––––––
Profit/(loss) for the period 16,000 13,000 (20,000)
––––––– –––––––– –––––––––
The following information is relevant:
(i) The other income is a dividend received from Sarri on 31 March 2016.

(ii) The details of Sarri‘s and Alexis‘s share capital and reserves at 1 October 2015 were:
Sarri Alexis
GHC‘000 GHC‘000
Equity shares of GHC1 each 20,000 15,000
Retained earnings 18,000 35,000

(iii) A fair value exercise was carried out at the date of acquisition of Sarri with the following
results:
Carrying fair value remaining life (straight line)
amount
GHC’000 GHC’000
Intellectual property 18,000 22,000 still in development
Land 17,000 20,000 not applicable
Plant 30,000 35,000 five years
The fair values have not been reflected in Sarri‘s financial statements.
Plant depreciation is included in cost of sales.
No fair value adjustments were required on the acquisition of Alexis.

(iv) In the year ended 30 September 2016 Hazard sold goods to Sarri at a selling price of GHC18
million.Hazard made a profit of cost plus 25% on these sales. GHC7·5 million (at cost to Sarri) of
these goods were still in the inventories of Sarri at 30 September 2016.

470
(v) Impairment tests for both Sarri and Alexis were conducted on 30 September 2016. They
concluded that the goodwill of Sarri should be written down by GHC1·6 million and, due to its
losses since acquisition, the investment in Alexis was worth GHC21·5 million.

(vi) All trading profits and losses are deemed to accrue evenly throughout the year.

Required:
(a) Calculate the goodwill arising on the acquisition of Sarri at 1 October 2015. (5 marks)
(b) Calculate the carrying amount of the investment in Alexis at 30 September 2016 under the
equity method prior to the impairment test. (4 marks)
(c) Prepare the consolidated income statement for the Hazard Group for the year ended 30
September 2016. (11 marks)
(20 marks)
Solution:
(a)Cost of control/Goodwill in Sarri:
Consideration GHC‘000
Shares (20,000 x 80% x 3/5 x GHC5) 48,000
Share of Net Assets at Acquisition (50,000 x 80%) (40,000)
–––––––
8,000
Impairment (1,600)
–––––––
Goodwill 6,400

(b)Carrying amount of Alexis 30 September 2016 (prior to impairment loss):


At cost GHC000
Cash (6,000 x GHC3) 18,000
6% loan notes (6,000 x GHC100/100) 6,000
–––––––
24,000
Less Post acquisition losses (20,000 x 40% x 3/12) (2,000)
–––––––
22,000
Impairment (500)
–––––––
21,500

(a) Hazard Group


Consolidated income statement for the year ended 30 September 2016
GHC’000
Revenue (105,000 + 62,000 – 18,000 intra group) 149,000
Cost of sales (see working) (89,000)
––––––––
Gross profit 60,000
Distribution costs (4,000 + 2,000) (6,000)
Administrative expenses (7,500 + 7,000) (14,500)

471
Finance costs (1,200 + 900) (2,100)
Impairment losses:
Goodwill (1,600)
Investment in associate (22,000 – 21,500) (500)
Share of loss from associate (20,000 x 40% x 3/12) (2,000)
––––––––
Profit before tax 33,300
Income tax expense (8,700 + 2,600) (11,300)
––––––––
Profit for the period 22,000
––––––––
Attributable to:
Equity holders of the parent 19,600
Non Controlling Interest ((12,000x 20%) 2,400
––––––––
22,000
––––––––
Note: the dividend from Sarri is eliminated on consolidation.
Working GHC’000

(i)Cost of sales
Hazard 68,000
Sarri 36,500
Intra group purchases (18,000)
Additional depreciation of plant (5,000/5 years) 1,000
Unrealised profit in inventories (7,500 x 25%/125%) 1,500
––––––––
89,000
––––––––
(ii) Net Assets of Sarri At Acqusition At Reporting Post Acquisition
GHC000 GHC000 GHC000
Equity Shares 20,000 20,000
Retained earnings 18,000 (18,000+13,000) 31,000 13,000
FV Adjustment- Intellectual Property 4,000 4,000
Land 3,000 3,000
Plant 5,000 5,000
Depreciation : Plant - (1,000) (1,000)
50,000 62,000 12,000
–––––––
QUESTION 8
(a)IFRS 3 Business combinations permits a non-controlling interest at the date of acquisition to
be valued by one of two methods:
(i) at its proportionate share of the subsidiary’s identifiable net assets; or
(ii) at its fair value (usually determined by the directors of the parent company).
Required:
Explain the difference that the accounting treatment of these alternative methods could have on
the consolidated financial statements, including where consolidated goodwill may be impaired.

472
(b) On 1 April 2011, Police acquired 80% of Soldier‘s equity shares by means of an immediate
share exchange and a cash payment of 88 pesewas per acquired share, deferred until 1 April
2012. Police has recorded the share exchange, but not the cash consideration. Police‘s cost of
capital is 10% per annum.
The summarised statements of financial position of the two companies as at 31 March 2012 are:
Police Soldier
ASSETS
GHC’000 GHC’000
Non-current assets
Property, plant and equipment 38,100 28,500
Investments – Soldier 24,000
– Captain at cost (note (iv)) 6,000
– Loan notes (note (ii)) 2,500
– Other equity (note (v)) 2,000 nil
––––––– –––––––
72,600 28,500
Current assets
Inventory (note (iii)) 13,900 10,400
Trade receivables (note (iii)) 11,400 5,500
Bank (note (iii)) 900 600
––––––– –––––––
Total assets 98,800 45,000
––––––– –––––––
Equity and liabilities
Equity
Equity shares of GHC1 each 25,000 10,000
Share premium 17,600 nil
Retained earnings – at 1 April 2011 16,200 18,000
– for year ended 31 March 2012 14,000 8,000
––––––– –––––––
72,800 36,000
Non-current liabilities
11% loan notes (note (ii)) 12,000 4,000
Deferred tax 4,500 nil
Current liabilities (note (iii)) 9,500 5,000
––––––– –––––––
Total equity and liabilities 98,800 45,000
––––––– –––––––
The following information is relevant:
(i) At the date of acquisition, Police conducted a fair value exercise on Soldier‘s net assets which
were equal to their carrying amounts with the following exceptions:
– An item of plant had a fair value of GHC3 million above its carrying amount. At the date of
acquisition it had a remaining life of five years. Ignore deferred tax relating to this fair value.
– Soldier had an unrecorded deferred tax liability of GHC1 million, which was unchanged as at
31 March 2012.

473
Police‘s policy is to value the non-controlling interest at fair value at the date of acquisition. For
this purpose a share price for Soldier of GHC3·50 each is representative of the fair value of the
shares held by the non-controlling interest.
(ii) Immediately after the acquisition, Soldier issued GHC4 million of 11% loan notes, GHC2·5
million of which were bought by Police. All interest due on the loan notes as at 31 March 2012
has been paid and received.
(iii) Police sells goods to Soldier at cost plus 50%. Below is a summary of the recorded activities
for the year ended 31 March 2012 and balances as at 31 March 2012:
Police Soldier
GHC‘000 GHC‘000
Sales to Soldier 16,000
Purchases from Police 14,500
Included in Police‘s receivables 4,400
Included in Soldier‘s payables 1,700
On 26 March 2012, Police sold and dispatched goods to Soldier, which Soldier did not record
until they were received on 2 April 2012. Soldier‘s inventory was counted on 31 March 2012 and
does not include any goods purchased from Police.
On 27 March 2012, Soldier remitted to Police a cash payment which was not received by Police
until 4 April 2012. This payment accounted for the remaining difference on the current
accounts.
(iv) Police bought 1·5 million shares in Captain on 1 October 2011; this represents a holding of
30% of Captain‘s equity. At 31 March 2012, Captain‘s retained profits had increased by GHC2
million over their value at 1 October 2011. Police uses equity accounting in its consolidated
financial statements for its investment in Captain.
(v) The other equity investments of Police are carried at their fair values on 1 April 2011. At 31
March 2012, these had increased to GHC2·8 million.
(vi) There were no impairment losses within the group during the year ended 31 March 2012.
Required:
Prepare the consolidated statement of financial position for Police as at 31 March 2012.

Solution:
(a) IFRS 3 allows (as an option) a non-controlling interest to be valued at its proportionate share
of the acquired subsidiary‘s identifiable net assets; this carries forward the only allowed method
in the previous version of this Standard. Its effect on the statement of financial position is that
the resulting carrying value of purchased goodwill only relates to the parent‘s element of such
goodwill and as a consequence the non-controlling interest does not reflect its share of the
subsidiary‘s goodwill. Some commentators feel this is an anomaly as the principle of a
consolidated statement of financial position is that it should disclose the whole of the
subsidiary‘s assets that are under the control of the parent (not just the parent‘s share). This
principle is applied to all of a subsidiary‘s other identifiable assets, so why not goodwill?
Any impairment of goodwill under this method would only be charged against the parent‘s
interest, as the non-controlling interest‘s share of goodwill is not included in the consolidated
financial statements.
The second (new) method of valuing the non-controlling interest at its fair value would
(normally) increase the value of the goodwill calculated on acquisition. This increase reflects the
non-controlling interest‘s ownership of the subsidiary‘s goodwill and has the effect of ‗grossing
up‘ the goodwill and the non-controlling interests in the statement of financial position (by the

474
same amount). It is argued that this method reflects the whole of the subsidiary‘s
goodwill/premium on acquisition and is thus consistent with the principles of consolidation.
Under this method any impairment of the subsidiary‘s goodwill is charged to both the
controlling (parent‘s share) and non-controlling interests in proportion to their holding of
shares in the subsidiary.
(b) POLICE
Consolidated statement of financial position as at 31 March 2012
GHC’000 GHC'000
Assets
Non-current assets:
Property, plant and equipment
(38,100 + 28,500 + 3,000 fair value – 600 depreciation) 69,000
Goodwill (w (i)) 7,400
Investments
– associate (w (ii)) 6,600
– fair value equity investments 2,800 9,400
––––––– ––––––––
85,800
Current assets
Inventory (13,900 + 10,400 + 1,500 GIT – 500 URP (w (iii))) 25,300
Trade receivables
(11,400 + 5,500 – 4400 intra group ) 12,500
Bank (900 + 600 + 1,200 CIT (w (iii))) 2,700 40,500
––––––– ––––––––
Total assets 126,300
––––––––
Equity and liabilities
Equity attributable to owners of the parent
Equity shares of GHC1 each 25,000

Reserves:
Share premium 17,600
Retained earnings (w (iv)) 36,325 53,925
––––––– ––––––––
78,925
Non-controlling interest (w (v)) 8,535
––––––––
Total equity 87,460
Non-current liabilities
11% loan notes (12,000 + 4,000 – 2,500 intra-group) 13,500
Deferred tax (4,500 + 1,000) 5,500 19,000
–––––––
Current liabilities
Deferred consideration (6,400 + 640 unwinding of discount (w (iv))) 7,040

Other current liabilities


(9,500 + 5,000 -1700) 12,800 19,840

475
––––––– ––––––––
Total equity and liabilities 126,300

Group Structure
Police
Date of Acq(1/4/11) Group = 80% 30%(Associate) Date of Acq (1/10/11)
Date of Rep(31/3/12) NCI =20% Date of Rep (31/3/12)
PAP = 1 year PAP = 6 months

Soldier Captain

Workings (figures in brackets are in GHC’000)


(i) Goodwill in Square
GHC‘000 GHC‘000
Controlling interest Share exchange 24,000
Deferred consideration (10,000 x 80% x 0·88/1·1) 6,400
Non-controlling interest (10,000 x 20% x GHC3·50) 7,000
–––––––
37,400
Net Assets at Acquisition (30,000)
Goodwill 7,400

Net Assets of Soldier At Acquisition At Reporting Post Acq.


GHC000 GHC000 GHC000
Equity shares 10,000 10,000
Pre-acquisition reserves 18,000 26,000 8,000
Fair value adjustments – plant 3,000 3,000
Fair value depreciation Adjustment(3000/5) (600) (600)
– unrecorded deferred tax (1,000) (1,000)
**Interest Paid to Parent by Subsidiary(11%x2500) 275 275
30,000 37,675 7675

(ii) Investment in Associate (Captain)


GHC‘000
Cost 6,000
Share post-acquisition profit (2,000 x 30%) 600
––––––
6,600
––––––
(iii) Reconciliation of current accounts
POLICE SOLDIER
GHC‘000 GHC‘000
Current account balances per question to eliminate 4,400 1,700
Goods-in-transit (GIT) (16,000 – 14,500) 1,500
Cash-in-transit (CIT) (balance required to reconcile) (1,200)
–––––– ––––––
3,200 3,200

476
–––––– ––––––
The goods-in-transit sale of GHC1·5 million includes unrealised profit (URP) of GHC500,000
(1,500 x 50/150).
(iv) Consolidated retained earnings:
GHC‘000
Police‘s retained earnings (16,200 + 14,000) 30,200
Interest received from subsidiary (intra-group) (275)
Soldier‘s post-acquisition profit (7,675 see above x 80%) 6,140
Captain‘s post-acquisition profit (2,000 x 30%) 600
Interest on deferred consideration (6,400 x 10%) (640)
URP in inventory (w (iii)) (500)

Gain on equity investments (2,800 – 2,000) 800


–––––––
36,325
–––––––
(v) Non-controlling interest
GHC‘000
Fair value on acquisition (w (i)) 7,000
Post-acquisition profit (7,675 x 20% (w (iv))) 1,535
––––––
8,535
––––––
QUESTION 9
On 1 April 2009 Patasi acquired 75% of Santasi‘s equity shares in a share exchange of three
shares in Patasi for every two shares in Santasi. The market prices of Patasi‘s and Santasi‘s
shares at the date of acquisition were GHC3·20 and GHC4·50 respectively.
In addition to this Patasi agreed to pay a further amount on 1 April 2010 that was contingent
upon the post-acquisition performance of Santasi. At the date of acquisition Patasi assessed the
fair value of this contingent consideration at GHC4·2 million, but by 31 March 2010 it was clear
that the actual amount to be paid would be only GHC2·7 million (ignore discounting). Patasi
has recorded the share exchange and provided for the initial estimate of GHC4·2 million for the
contingent consideration.
On 1 October 2009 Patasi also acquired 40% of the equity shares of Adiembra paying GHC4 in
cash per acquired share and issuing at par one GHC100 7% loan note for every 50 shares
acquired in Adiembra. This consideration has also been recorded by Patasi.
Patasi has no other investments.
The summarised statements of financial position of the three companies at 31 March 2010 are:
Patasi Santasi Adiembra
Assets GHC’000 GHC’000 GHC’000
Non-current assets
Property, plant and equipment 37,500 24,500 21,000
Investments 45,000 nil nil
––––––– ––––––– –––––––
82,500 24,500 21,000
Current assets
Inventory 10,000 9,000 5,000

477
Trade receivables 6,500 1,500 3,000
––––––– ––––––– –––––––
Total assets 99,000 35,000 29,000
––––––– ––––––– –––––––
Equity and liabilities
Equity
Equity shares of GHC1 each 25,000 8,000 5,000
Share premium 19,800 nil nil
Retained earnings – at 1 April 2009 16,200 16,500 15,000
– for the year ended 31 March 2010 11,000 1,000 6,000
––––––– ––––––– –––––––
72,000 25,500 26,000
Non-current liabilities
7% loan notes 14,500 2,000 nil

Current liabilities
Contingent consideration 4,200 nil nil
Other current liabilities 8,300 7,500 3,000
––––––– ––––––– –––––––
Total equity and liabilities 99,000 35,000 29,000
––––––– ––––––– –––––––
The following information is relevant:
(i) At the date of acquisition the fair values of Santasi‘s property, plant and equipment was
equal to its carrying amount with the exception of Santasi‘s factory which had a fair value of
GHC2 million above its carrying amount. Santasi has not adjusted the carrying amount of the
factory as a result of the fair value exercise. This requires additional annual depreciation of
GHC100,000 in the consolidated financial statements in the post-acquisition period.
Also at the date of acquisition, Santasi had an intangible asset of GHC500,000 for software in
its statement of financial position. Patasi‘s directors believed the software to have no
recoverable value at the date of acquisition and Santasi wrote it off shortly after its acquisition.
(ii) At 31 March 2010 Patasi‘s current account with Santasi was GHC3·4 million (debit). This
did not agree with the equivalent balance in Santasi‘s books due to some goods-in-transit
invoiced at GHC1·8 million that were sent by Patasi on 28 March 2010, but had not been
received by Santasi until after the year end. Patasi sold all these goods at cost plus 50%.
(iii) Patasi‘s policy is to value the non-controlling interest at fair value at the date of acquisition.
For this purpose Santasi‘s share price at that date can be deemed to be representative of the fair
value of the shares held by the non-controlling interest.
(iv) Impairment tests were carried out on 31 March 2010 which concluded that the value of the
investment in Adiembra was not impaired but, due to poor trading performance, consolidated
goodwill was impaired by GHC3·8 million.
(v) Assume all profits accrue evenly through the year.
Required:
Prepare the consolidated statement of financial position for Patasi as at 31 March 2010.
(20 marks)

Solution:
Consolidated statement of financial position of Patasi as at 31 March 2010

478
GHC’000 GHC’000
Assets
Non-current assets:
Property, plant and equipment (37,500 + 24,500 + 2,000 – 100) 63,900
Goodwill (16,000 – 3,800 (w (i))) 12,200
Investment in associate (w (ii)) 13,200
––––––––
89,300
Current assets
Inventory (10,000 + 9,000 + 1,800 GIT – 600 URP (w (iii))) 20,200
Trade receivables (6,500 + 1,500 – 3,400 intra-group (w (iii))) 4,600 24,800
––––––– ––––––––
Total assets 114,100
––––––––
Equity and liabilities
Equity attributable to owners of the parent
Equity shares of GHC1 each 25,000
Share premium 19,800
Retained earnings (w (iv)) 27,500 47,300
––––––– ––––––––
72,300
Non-controlling interest (w (v)) 8,400
––––––––
Total equity 80,700
Non-current liabilities
7% loan notes (14,500 + 2,000) 16,500
Current liabilities
Contingent consideration 2,700
Other current liabilities (8,300 + 7,500 – 1,600 intra-group (w (iii))) 14,200 16,900
––––––– ––––––––
Total equity and liabilities 114,100
––––––––
Workings (figures in brackets are in GHC’000)
(i) Group Structure
Patasi
Date of Acq. : 1/4/09 Date of Acq.: 1/10/09
Date of Rep: 31/03/10 Group=75% 40% Date of Rep.: 31/03/10
Post Acq: 1 year NCI=25% Post Acq.: 6 months

Santasi Adiembra
(ii) Net Assets of Santasi At Acquisition At Reporting Post Acquisition
GHC000 GHC000 GHC000
Equity Shares 8,000 8,000 -
Retained Earnings 16,500 17,500 1,000
FV Adjustment – factory 2,000 2,000 -
FV Depreciation – factory (100) (100)
FV Adjustment – Intangible Assets (500) - 500
26,000 27,400 1,400

479
(iii)
Net Assets of Adiembra At Acquision At Reporting Post Acquisition
GHC000 GHC000 GHC000
Equity Shares 5,000 5,000 -
Retained Earnings 18,000 21,000 (6/12x6,000) 3,000
23,000 26,000 3,000

(iv) Goodwill in Santasi


GHC’000 GHC’000
Controlling interest(Cost of Investment) :
Share exchange (8,000 x 75% x 3/2 x GHC3·20) 28,800
Contingent consideration 4,200
Non-controlling interest (8,000 x 25% x GHC4·50) 9,000
–––––––
42,000
Net Assets at Acquisition (26,000)
–––––––
16,000
Impairment Loss (3,800)
Goodwill 12,200

Goodwill is impaired by GHC3·8 million and therefore has a carrying amount at 31 March 2010
of GHC12·2 million. The goodwill impairment is charged against Santasi‘s retained earnings
(see working (vi)), thus ensuring it is allocated between the controlling and non-controlling
interests in proportion to their share ownership in Santasi.
The effect of the software having no recoverable amount is that its write-off in the post-
acquisition period should be treated as a fair value adjustment at the date of acquisition for
consolidation purposes. The consequent effect is that this will increase the post-acquisition
profit for consolidation purposes by GHC500,000.
(v) Carrying amount of Adiembra at 31 March 2010 (Investment in Associate)
GHC’000
Cash consideration (5,000 x 40% x GHC4) 8,000
7% loan notes (5,000 x 40% x GHC100/50) 4,000
Share of post-acquisition profits (3,000 x 40%) 1,200
–––––––
13,200
–––––––
(vi) Goods in transit and unrealised profit (URP)
The intra-group current accounts differ by the goods-in-transit sales of GHC1·8 million on
which Patasi made a profit of GHC600,000 (1,800 x 50/150). Thus inventory must be increased
by GHC1·2 million (its cost), GHC600,000 is eliminated from Patasi‘s profit, GHC3·4 million is
deducted from trade receivables and GHC1·6 million (3,400 – 1,800) is deducted from trade
payables (other current liabilities).
(vii) Consolidated retained earnings
GHC‘000
Patasi‘s retained earnings 27,200

480
Santasi‘s post-acquisition losses (1,400 x 75% see below) 1,050
Gain from reduction of contingent consideration (4,200 – 2,700 see below) 1,500
URP in inventory (w (iii)) (600)
Adiembra‘s post-acquisition profits (3,000 x 40%) 1,200
Impairment Loss (3,800 x 75%) (2,850)
–––––––
27,500
–––––––
The adjustment to the provision for contingent consideration due to events occurring after the
acquisition is reported in income (goodwill is not recalculated).
(viii) Non-controlling interest
Fair value on acquisition (w (i)) 9,000
Post-acquisition losses (1,400 x 25% (w (iv))) 350
Impairment Loss (3,800 x 25%) (950)
–––––––
8,400
–––––––
QUESTION 11
Hedra, a public listed company, acquired the following investments:

(i) On 1 October 2014, 72 million shares in Salvador for an immediate cash payment of GHC195
million. Hedra agreed to pay further consideration on 30 September 2015 of GHC49 million if
the post acquisition profits of Salvador exceeded an agreed figure at that date. Hedra has not
accounted for this deferred payment as it did not believe it would be payable, however
Salvador‘s profits have now exceeded the agreed amount (ignore discounting).Salvador also
accepted a GHC50 million 8% loan from Hedra at the date of its acquisition.

(ii) On 1 April 2015, 40 million shares in Aragon by way of a share exchange of two shares in
Hedra for each acquired share in Aragon. The stock market value of Hedra‘s shares at the date
of this share exchange was GHC2·50. Hedra has not yet recorded the acquisition of the
investment in Aragon.

The summarised statement of financial position of the three companies as at 30 September 2015
are:
Hedra Salvador Aragon
Non-current Assets GHCm GHCm GHCm GHCm GHCm GHCm
Property, plant and equipment 358 240 270
Investments – in Salvador 245 nil nil
– other 45 nil nil
––– –––– –––––
648 240 270
Current Assets
Inventories 130 80 110
Trade receivables 142 97 70
Cash and bank nil 272 4 181 20 200
––– –––– ––– –––– ––––– –––––
Total assets 920 421 470

481
–––– –––– ––––
Equity and liabilities
Ordinary share capital (GHC1 each) 400 120 100
Reserves:
Share premium 40 50 nil
Revaluation 15 nil nil
Retained earnings 240 295 60 110 300 300
–––– –––– –––– –––– –––– ––––
695 230 400

Non-current liabilities
8% loan note nil 50 nil
Deferred tax 45 45 nil 50 nil nil
–––– –––– ––––
Current liabilities
Trade payables 118 141 40
Bank overdraft 12 nil nil
Current tax payable 50 180 nil 141 30 70
––– ––––– –––– ––– –––– ––––
Total equity and liabilities 920 421 470
––––– ––– ––––
The following information is relevant:
(a) Fair value adjustments and revaluations:
(i) Hedra‘s accounting policy for land and buildings is that they should be carried at their fair
values. The fair value of Salvador‘s land at the date of acquisition was GHC20 million in excess
of its carrying value. By 30 September 2015 this excess had increased by a further GHC5 million.
Salvador‘s buildings did not require any fair value adjustments. The fair value of Hedra‘s own
land and buildings at 30 September 2015 was GHC12 million in excess of its carrying value in
the above balance sheet.

(ii) The fair value of some of Salvador‘s plant at the date of acquisition was GHC20 million in
excess of its carrying value and had a remaining life of four years (straight-line depreciation is
used).

(iii) At the date of acquisition Salvador had unrelieved tax losses of GHC40 million from
previous years. Salvador had not accounted for these as a deferred tax asset as its directors did
not believe the company would be sufficiently profitable in the near future. However, the
directors of Hedra were confident that these losses would be utilised and accordingly they
should be recognised as a deferred tax asset. By 30 September 2015 the group had not yet
utilised any of these losses. The income tax rate is 25%.
(b) The retained earnings of Salvador and Aragon at 1 October 2014, as reported in their
separate financial statements, were GHC20 million and GHC200 million respectively. All profits
are deemed to accrue evenly throughout the year.
(c) An impairment test on 30 September 2015 showed that consolidated goodwill should be
written down by GHC20 million. Hedra has applied IFRS 3 Business combinations since the
acquisition of Salvador.
(d) The investment in Aragon has not suffered any impairment.

482
Required:
Prepare the consolidated balance sheet of Hedra as at 30 September 2015. (25 marks)
Solution:
Consolidated balance sheet of Hedra as at 30 September 2015:
GHCm GHCm
Non-current assets
Property, plant and equipment (358 + 240 + 12 + 20 + 5 +15 (w (iv))) 650
Goodwill (w (i))) 80
Investment in associate (w (v)) 220
Other investments 45
––––––
995
Current Assets
Inventories (130 + 80) 210
Trade receivables (142 + 97) 239
Cash and bank 4 453
–––– ––––––
Total assets 1,448
––––––
Equity and liabilities
Equity attributable to the parent
Ordinary share capital (400 + 80 (w (v))) 480
Reserves: Share premium (40 + 120 (w viii) 160
Revaluation (15 + 12 + (5 x 60%) (w ix)) 30
Retained earnings (w (ii)) 261 451
––––– –––––
931
Non Controlling interest (w (iii)) 112
––––––
1,043
Non-current liabilities
Deferred tax (45 – 10) 35
Current liabilities
Bank overdraft 12
Trade payables (118 + 141) 259
Deferred consideration (w (i)) 49
Current tax payable 50 370
––––– –––––
Total equity and liabilities 1,448
––––––

Workings – Note: all working figures in GHCmillion.


The investment in Salvador represents 60% (72/120) of its equity and is likely to give Hedra
control thus Salvador should be consolidated as a subsidiary. The investment in Aragon
represents 40% (40/100) of its equity. Normally this would give Hedra significant influence and
Aragon would be classed as an associate that should be equity accounted.

483
(i)Cost of control / Goodwill
Investment at cost :
Immediate 195
Deferred 49
244
Share of Net Assets at Acquisition (60% x 240) (144)
100
Impairment (20)
Goodwill 80
The deferred contingent consideration has now become payable and has to be accounted for.

(ii)Retained earnings
Hedra 240
Share of post acquisition profit – Salvador (60% x 35) 21
Share of post acquisition profit –Aragon (40% x 50) 20
Goodwill Impairment (20)
261

(iii)Non Controlling Interest


NCI at Reporting (40% x 280) 112
The increase in the fair value of the land at the date of acquisition is accounted for as a fair value
adjustment. The increase of a further GHC5 million in the year ended 30 September 2015 is a
revaluation increase (accounted for as 60% to the group revaluation reserve and 40% to
minority interest).The fair value adjustment of GHC20 million to plant will be realised evenly
over the next four years in the form of additional depreciation at GHC5 million per annum. In
the year ended 30 September 2015 the effect on the consolidated financial statements is that
GHC5 million will be charged to Salvador‘s profit (as additional depreciation); and a net
GHC15 million added to the carrying value of the plant.
(v) Investment in associate:
Investment at cost (100 x 40% x 2 x GHC2·50) 200
Share of post acquisition profit (50x 40%) 20
––––
220
––––
The purchase consideration by way of a share exchange (80 million in Hedra for 40 million in
Aragon) would be recorded as an increase in share capital of GHC80 million (GHC1 nominal
value) and an increase in share premium of GHC120 million (80 xGHC1·50).
(vi) Net Assets of Salvador Co
At Acquisition At Reporting Post Acquisition
GHCm GHCm GHCm
Equity shares 120 120
Share Premium 50 50
Retained earnings 20 60 40
Fair value adjustments – Land 20 25 5
– Plant 20 20
FV Depreciation : Plant (20/4) - (5) (5)
Deferred Tax Asset 10 10

484
––––––– ––––––– –––––––
240 280 40
––––––– ––––––– –––––––

(vii) Net Assets of Aragon Co


At Acquisition At Reporting Post Acquisition
GHCm GHCm GHCm
Equity shares 100 100
Retained earnings 250 300 (6/12 x 100) 50
––––––– ––––––– –––––––
350 400 50
––––––– ––––––– –––––––
(viii) Share Premium
Hedra 40
Share Exchange (2/1 x 40 x GHC1.5) 120
160
(ix) Revaluation Reserve
Balance bld 15
Salvador (60% x 5) 3
Hedra (land & building) 12
30
QUESTION 12
Portugal, a public listed company, acquired the following investments:

–On 1 April 2013, 24 million shares in Spain. This was by way of an immediate share exchange
of two shares in Portugal for every three shares in Spain plus a cash payment of GHC1 per
Spain share payable on 1 April 2016. The market price of Portugal‘s shares on 1 April 2013 was
GHC2 each.

–On 1 October 2013, 6 million shares in Austria paying an immediate GHC2·50 in cash for each
share.
Based on Portugal‘s cost of capital (taken as 10% per annum), GHC1 receivable in three years‘
time can be taken to have a present value of GHC0·75.

Portugal has not yet recorded the acquisition of Spain but it has recorded the investment in
Austria.
The summarized statement of financial position at 31 March 2014 are:
Portugal Spain Austria
Non-current assets GHC000 GHC000 GHC000 GHC000 GHC000 GHC000
Property, plant and equipment 41,000 34,800 37,700
Investments 15,000 3,000 nil
——— –—— —–———–
56,000 37,800 37,700
Current Assets
Inventory 9,900 4,800 7,900
Trade and other receivables 13,600 8,600 14,400
Cash 1,200 24,700 3,800 17,200 nil 22,300

485
——— –——— –——– ———– ———– ———–
Total assets 80,700 55,000 60,000
Equity and liabilities
Capital and reserves
Ordinary shares GHC1 each 20,000 30,000 20,000
Reserves:
Share premium 8,000 2,000 nil
Accumulated profits 10,600 18,600 8,500 10,500 8,000 8,000
——— –——— –——— –——–– ——— –———–
38,600 40,500 28,000
Non-current liabilities
10% loan note 16,000 4,200 12,000
Current liabilities
Trade and other payables 16,500 6,900 13,600
Bank overdraft nil nil 4,500
Taxation 9,600 26,100 3,400 10,300 1,900 20,000
——— –——— –——— –——— –——— –———–
Total equity and liabilities 80,700 55,000 60,000
———– ———– ———–
The following information is relevant:
(i) Below is a summary of the results of a fair value exercise for Spain carried out at the date of
acquisition:
Asset Carrying value Fair value
at acquisition at acquisition Notes
GHC000 GHC000
Plant 10,000 15,000 remaining life at acquisition four years
Investments 3,000 4,500 no change in value since acquisition
The book values of the net assets of Austria at the date of acquisition were considered to be a
reasonable approximation to their fair values
(ii) The profits of Spain and Austria for the year to 31 March 2014, as reported in their entity
financial statements, were GHC4·5 million and GHC6 million respectively. No dividends have
been paid by any of the companies during the year. All profits are deemed to accrue evenly
throughout the year.
(iii) In January 2014 Austria sold goods to Portugal at a selling price of GHC4 million. These
goods had cost Austria GHC2·4 million. Portugal had GHC2·5 million (at cost to Portugal) of
these goods still in inventory at 31 March2014.
(iv) Goodwill is to be written off over a five-year life with a proportionate charge in the year of
acquisition.
(v) All depreciation/amortisation is charged on a straight-line basis.
(vi) Portugal uses the Allowed Alternative Treatment in IFRS 3 ‘Business Combinations’ to
record the fair value of assets and liabilities.
Required:
Prepare the Consolidated Statement of Financial Position of Portugal as at 31 March 2014.
(25 marks)
Solution:
Consolidated Balance Sheet of Portugal as at 31 March 2014:
GHC000 GHC000

486
Non current assets
Goodwill (w (i)) 12,800
Property, plant and equipment (41,000 + 34,800 + 3,750 (w (i))) 79,550
Investments:
– in associate (w (iv)) 15,600
– ordinary (3,000 + 1,500 (fair value increase) 4,500 20,100
——— ————
112,450
Current Assets
Inventory (9,900 + 4,800 – 300 (w (v))) 14,400
Trade receivables (13,600 + 8,600) 22,200
Cash (1,200 + 3,800) 5,000 41,600
——— ————
Total assets 154,350
———
Equity and liabilities
Ordinary share capital (20,000 + 16,000 (w (i))) 36,000
Reserves:
Share premium (8,000 + 16,000 (w (i))) 24,000
Retained Earnings (w (ii)) 8,800 32,800
———– ———–
68,800
Non Controlling interests (w (iii)) 9,150
Non-current liabilities
10% Loan note (16,000 + 4,200) 20,200
Deferred consideration (18,000 + 1,800 (w (vi))) 19,800
———–
40,000
Current liabilities:
Trade payables (16,500 + 6,900) 23,400
Taxation (9,600 + 3,400) 13,000 36,400
———– ————
Total equity and liabilities 154,350
————

Workings – Note: all working figures in GHC000.


The 80% (24m/30m shares) holding in Spain is likely to give Portugal control and means it is a
subsidiary and should be consolidated. The 30% (6m/20m shares) holding in Austria is likely to
give Portugal influence rather than control and thus it should be equity accounted.

(i)Cost of control/Goodwill
Investments at cost (see below) 50,000
Share of Net Assets (80% x GHC42,500) (34,000)
16,000
Amortisation(16,000/5) (3,200)
———
Goodwill 12,800

487
The purchase consideration for Spain is GHC50 million. This is made up of an issue of 16
million shares (24/3 ×2) at GHC2 each totalling GHC32 million and deferred consideration of
GHC24 million (GHC1 per share) which should be discounted to GHC18 million (24 million
×GHC0·75). The share issue should be recorded as GHC16 million share capital and GHC16
million share premium.The goodwill will be depreciated over five years at GHC3·2 million per
annum.

(ii) Retained Earnings


Portugal 10,600
Share of Post Acquisition Profit - Spain (80% x 3,250) 2,600
Share of Post acquisition profits – Austria (30% x 2,000) 600
Amortisation (3,200)
Unwound Interest (10% x 18,000) (1,800)
8,800

(iii)Non Controlling interest


NCI at Reporting (20% x 45,750) 9,150

(iv) Investment in associate:


Investment at cost(6m x GHC2.5) 15,000
Share of post acquisition profit (30% x 2,000) 600
———–
15,150
———–
(v) Unrealised profit in inventory
As the transaction is with an associate, only the group share of unrealised profits must be
eliminated:GHC1·6 million ×2·5 million/4 million ×30% = GHC300,000
(vi) The deferred consideration of GHC24 million has been discounted to GHC18 million. The
discounted amount will be‗unwound‘ at 10% per annum. In the year to 31 March 2014 this will
give an accrued finance cost of 10% (added to the carrying value of the deferred consideration)
(vii) Net Assets of Spain Co
At Acquisition At Reporting Post Acquisition
GHC000 GHC000 GHC000
Equity shares 30,000 30,000
Share Premium 2,000 2,000
Retained earnings 4,000 8,500 4,500
Fair value adjustments – Plant 5,000 5,000
FV Depreciation : Plant (5,000/4) - (1,250) (1,250)
Investments 1,500 1,500
––––––– ––––––– –––––––
42,500 45,750 3,250
––––––– ––––––– –––––––

(viii) Net Assets of Austria Co


At Acquisition At Reporting Post Acquisition
GHCm GHCm GHCm

488
Equity shares 20,000 20,000
Retained earnings 5,000 8,000 (6/12 x 6,000) 3,000
URP (2.5/4 x 1.6) - (1,000) (1,000)
––––––– ––––––– –––––––
25,000 27,000 2,000
––––––– ––––––– –––––––
QUESTION 13
a) Explain the accounting treatment for ‘deferred consideration’ and ‘contingent consideration’
in the context of the acquisition of a subsidiary by a parent entity. (5 marks)

b) You are the Financial Accountant of Faisal Ltd (Faisal), a Ghanaian listed company,
involved in food retailing. During 2017, Faisal acquired interests in Zaytuna Ltd (Zaytuna)
and Medeama Ltd (Medeama).The Statement of profit or loss for Faisal, Zaytuna and
Medeama for the year ended 31 December 2017 are as follows:

Statement of profit or loss account for the year ended 31 December 2017
Faisal Zaytuna Medeama
GH¢’million GH¢’million GH¢’million
Revenue 450 150 75
Cost of sales (300) (90) (45)
Gross profit 150 60 30
Operating expenses (25) (15) (5)
Operating profit 125 45 25
Interest and similar charges (15) (5) (1)
Profit on ordinary activities before 110 40 24
taxation
Income tax expense (27.5) (10) (6)
Profit on ordinary activities after 82.5 30 18
taxation
Retained earnings at start of year 117.5 45 7
Retained earnings at end of year 200 75 25

Additional information:
i) On 1 April 2017, Faisal purchased 12 million of the 15 million GH¢1 ordinary shares in
Zaytuna at a cost of GH¢8 per ordinary share. At the date of acquisition the fair values of
Zaytuna‘s net assets were equal to their book value with the exception of property, the
details of which are as follows:
GH¢’million
Cost 75
Accumulated depreciation at 1 January 2017 (6)
Net book value at 1 January 2017 69

The property, which had a useful economic life of 25 years on 1 January 2015, is in a prime
commercial location and has increased dramatically in value since it was purchased by
Zaytuna on 1 January 2015. The replacement cost of a similar building, with a similar
remaining useful economic life at 1 April 2017, is GH¢100 million. The fair value at
acquisition has not been reflected in the records of Zaytuna.

489
ii) On 1 July 2017, Faisal purchased 4 million of the 10 million GH¢1 ordinary shares in
Medeama at a cost of GH¢6 per ordinary share. At the date of acquisition the fair values of
Medeama‘s net assets were equal to their book value with the exception of property that had
a fair value of GH¢9 million in excess of its book value and a remaining useful life of four
years.

iii) In August 2017, Faisal sold goods to Zaytuna for GH¢7.5 million and 20% of these goods
remained unsold at 31 December 2017. Faisal prices its sales at cost plus 50%.

iv) On 23 January 2018, Faisal sold its former head office administrative building for GH¢1.25
million. At 31 December 2017, the building was for sale and unoccupied, with staff having
moved to a new premises. The book value of the building in the statement of financial
position of Faisal as at 31 December 2017 was GH¢2 million.

v) Each company charges depreciation on a time apportionment basis to operating expenses.

vi) The directors of Faisal believe that any goodwill arising on the acquisition of Zaytuna and
Medeama has been impaired by 25% as at 31 December 2017. The directors have a policy of
measuring non-controlling interests at the proportionate share of identifiable net assets.

(Note: All calculations may be taken to the nearest GH¢0.01 million and assume all
expenses and gains accrue evenly throughout the year unless otherwise instructed.)

Required:
Prepare the consolidated statement of profit or loss account of Faisal Group for the year
ended 31 December 2017in accordance with International Financial Reporting Standards
(IFRS).
(20 marks)

(Total: 25 marks)
Solution:

a) Accounting treatment of deferred consideration:


 Deferred consideration must be recognised by the purchaser at the acquisition date at its fair
value.
 This is usually recognised as part of the cost of investment and a liability in the
Consolidated Statement of Financial Position.
 This is normally the agreed cash amount discounted to the acquisition date at the
purchaser‘s cost of capital.
 The discount is unwound over time by the purchaser, by recognising it as a finance cost in
the post-acquisition period as time passes.
 The liability at any reporting date will be the initial fair value plus the amount of the
discount that has unwound to date.
Accounting treatment of contingent consideration:

490
 Any contingent consideration must also be recognised at acquisition at its fair value. This
will normally include a discount to reflect the time delay before payment, plus a discount to
reflect the probability that the amount will be paid in part or in full.
 Goodwill is calculated based on this estimate. At each reporting date after acquisition, the
fair value is re-estimated, with any change being taken to profit or loss (of the purchaser) in
the year of re-estimation.
 Any change due to the unwinding of the time value of money discount is recognised as a
finance cost.
 The revised amount is carried as a liability until further re-estimated, or paid.

b)
Faisal Group
Consolidated statement of profit or loss account for the year ended 31 December 2017
GH¢’million

Revenue (450 + (150*9/12) – 7.5 (W4) 555


Cost of sales (300 + (90*9/12) -7.5 (W4) + 0.5(W4) 360.5
Gross profit 194.5
Operating expenses (25+ (15*9/12)+4.15(W1)+1.05(W3)+0.75 (W7) (42.2)
Operating profit 152.3
Interest payable and similar charges (15 + 5*(9/12) (18.75)
Share of profit of associate (Medeama) W6 3.15
Profit on ordinary activities before tax 136.7
Tax on profit on ordinary activities (27.5 + 10*9/12 (35)
Profit on ordinary activities after tax 101.7
Non-controlling interest (W5) (4.29)

97.41
Retained earnings at start of year 117.5
Retained earnings at end of the year 212.41

1) Goodwill
GH¢million GH¢million
Investment at cost 96
Non-controlling interest (20% x99.25) 19.85
115.85
Net assets acquired
GH¢1 ordinary shares 15
Pre-acquisition profits (01/01/2017) 45

491
Pre-acquisition profits (GH¢30million *3/12) 7.5
Fair value adjustment (W2) 31.75
99.25
16.60
Impairment @ (25% x16.60) 4.15
12.45

Alternative approach
Goodwill
GH¢million GH¢million
Investment at cost 96

Net assets acquired


GH¢1 ordinary shares 15
Pre-acquisition profits (01/01/2017) 45
Pre-acquisition profits (GH¢30million *3/12) 7.5
Fair value adjustment (W2) 31.75
99.25
Group Share (80% x 99.25) 79.40
16.60
Impairment @ (25% x16.60) 4.15
12.45

2) Fair value adjustment GH¢million GH¢million


Replacement cost @ 01/04/2017 100
Net book value @ 01/01/2017 69
Depreciation 01/01/2017 to 01/04/2017:
(GH¢75 million/25) * 3/12 (0.75) 68.25
31.75

3) Additional depreciation on fair value adjustment GH¢million


(GH¢31.75 million/22.75) * 9/12 1.05

4) Intercompany transactions
Eliminate intercompany sales and cost of sales: GH¢7.5 million

Eliminate inventory profit (GH¢7.5 million/5) * 50/150 GH¢0.5 million

5) Non-controlling interest charge GH¢million


Net profit for the year (9/12 x30m) 22.5
Less depreciation on fair value adjustment 1.05
21.45
X 20% 4.29

492
6) Share of profit of associate GH¢million
Profit after tax (GH¢18 million * 6/12 * 0.4) 3.6
Depreciation adjustment (GH¢9 million/4 * 0.4 *6/12) (0.45)
3.15
Faisal
7) Sale of former head office
IAS 10 Events after the Reporting Period states that sales of non-current assets are non-adjusting
events and accounts would not normally be amended to reflect this (although it would be
disclosed in the notes). However, in this case, it could be argued that the sale price is evidence
of impairment at the year-end and unless it can be shown that the conditions causing the
impairment arose after the year-end then the accounts should be adjusted. Given that the sale
took place on 23 January 2018, it is likely that the conditions existed at the date of statement of
financial position.
Therefore the impairment should be charged in the 2017 financial statements in accordance with
IAS 36 Impairment of Assets.
Dr Income statement – operating expenses GH¢0.75 million
Cr Property GH¢0.75 million
The property would be classified as a ‗non-current asset held for sale‘ in accordance with IFRS 5
Non-Current Assets Held for Sale and Discontinued Operations.

STEP ACQUISITION
An investment in an entity will, in practice, be bought in stages over a period of time.
1. No control to control
The accounting treatment is to treat the original investment as being disposed of at fair value and
reacquired at fair value. The fair value on re-acquisition plus the extra consideration paid for the
additional new shares bought becomes the cost of the increased investment.
1. Re-measure original investment to fair value and gain to profit and loss
2. Calculate goodwill
(W) Goodwill
GHS
Cost of additional investment xxx
Fair value of existing interest xxx
NCI at acquisition xxx
Less fair value of S net assets at acquisition (xxx)
Goodwill xxx

Example 1
Kwame acquired 40% of the equity interest of Nkrumah for GHS40million several years ago. On
the 1 January 2019 Kwame acquired an additional 35% for GHS45million when the fair value of
the identifiable net assets were GHS105million.
The fair value of the non-controlling interest on 1 January 2019 was GHS32million the fair value
of the 40% holding was GHS52million.
Required
Calculate the goodwill to appear in the Kwame group statement of financial position as at 31st
December 2019.

493
2. Control to Control – Reduction in NCI
We are buying the appropriate part of the NCI entitlement to the subsidiary net assets, including
goodwill.
An increase in ownership resulting in a reduction in non-controlling interest. Transfer from the
non-controlling interest.
DR. NCI xxx
CR Other components of equity (balancing figure) xxx
CR Bank xxx
As we have not acquired a subsidiary, there is no gain or loss to be calculated it just a transfer
between owners

Example 2
Continuing from example 1
On 31st December, Kwame acquired a further 5% of Nkrumah for GHS8million. Nkrumah had
made profits since being acquired by Kwame of GHS10million. There has been no impairment
of Goodwill.
Required
Prepare the journal entry to record the changes in ownership from a 75% holding to 80%
holdings
3. Control to Control (Increase in NCI)
A decrease in ownership resulting in an increase in non-controlling interest. Transfer to the non-
controlling interest.
DR Bank xxx
CR Non-controlling interest xxx
CR other components of equity (balancing figure) xxx

Example 3
Rose owned 90% of the equity shares of Mat before it then sold 20% of the subsidiary on 31
December 2019 for GHS90million.
The net asset at the date of disposal of the shares was GHS350million and the goodwill on
acquisition of the original 90% was GHS50million.
Required
Prepare the journal entry to record the change in ownership from a 90% holding to a 70%
holding.

4. Control to no control (Disposal of subsidiary)


Calculate a group profit or loss on disposal of subsidiary

(W) Group profit/loss on disposal


GHS
Proceeds xxx
Add: Investment still held (fair value) xxx
Add: non-controlling interest xxx
Less: net assets at disposal (xxx)
Less: goodwill (xxx)

494
Group profit or loss on Disposal xxx

Example: Profit on disposal of a subsidiary


On 1 January Year 9, H plc acquired 90% of the equity shares of S Ltd for GH¢120 million.
The fair value of the identifiable net assets in S Ltd at that date was GH¢111 million. The fair
value of the NCI at 1 January Year 9 was GH¢9 million. H plc uses the full goodwill method for
consolidation leading to the recognition of good will of GH¢18m ((GH¢120m + GH¢9m -
GH¢111m).
H plc subsequently sold the shares on 31 December Year 9 for GH¢197 million. The carrying
value of the net assets of S Ltd at 31 December Year 9 was GH¢124 million.

The gain on disposal recognised in profit or loss should be calculated as follows:


GH¢ m
Consideration received for shares in S Ltd on 31 December 197.0
Net assets de-recognised in consolidated accounts
Net assets de-recognised 124.0
Value of NCI at 31 December (10% x 124 million) (12.4)
111.6
Goodwill de-recognised: (120 + 9 – 111) 18.0
Net assets sold (129.6)
Gain on disposal, reported in profit or loss 67.4

TRY
Nana owned 90% of Sister before it decided to sell a 50% stake of its investment on 31st
December 2019 for GHS120million. The non-controlling interest at that date was GHS53million
and the fair value of the remaining 40% is GHS96million.
The goodwill on acquisition of the original 90% was GHS38milion and the net assets at the date
of disposal were GHS201million.
Required
Calculate the group profit on disposal that will appear in the group financial statements of Nana
group for the year ended 31st December 2019.

495
VERTICAL AND MIXED GROUPS
A subsidiary is an entity controlled by another entity (its parent). It follows that a parent will also
control its subsidiary‟s subsidiary. A group structure in which a parent has a subsidiary which in
turn is itself a parent of another subsidiary is known as a vertical group. In the following
explanation we shall refer to the parent as H, its subsidiary as S and the subsidiary‟s subsidiary
as T.
Illustration: Vertical group
H

75%

60%

H Ltd owns 75% of S Ltd. S Ltd is thus a subsidiary of H Ltd.


S Ltd owns 60% of T Ltd.
T Ltd is a subsidiary of S Ltd.
T Ltd is also a subsidiary of H Ltd.

Explanation H Ltd has a direct interest in S Ltd and an indirect interest in T Ltd (exercised via S
Ltd‟s holding in T Ltd). T Ltd is a subsidiary of H Ltd because H Ltd has a controlling interest
in S Ltd and S Ltd has a controlling interest in T Ltd. This is a „vertical group‟ consisting of H
Ltd, S Ltd and T Ltd. T Ltd is said to be a sub-subsidiary of H Ltd.

In practice, structures can be much more complex than this with groups comprised of many
layers of companies. However, they will be consolidated by applying the same principles as
explained in this chapter for the relatively straightforward vertical group structure shown above.
Another group structure that could be examined is a mixed group (also known as a D-shaped
group).

496
Illustration: Mixed group

60%

S 40%

20%

T
H Ltd owns 60% of S Ltd and 40% of T Ltd S Ltd owns 20% of W Ltd. S Ltd is a subsidiary of
H Ltd. T Ltd is also a subsidiary of H Ltd.

Explanation
H Ltd and S Ltd between them own more than 50% of T Ltd (the fact that S Ltd is not a wholly-
owned subsidiary of H Ltd is irrelevant). H Ltd controls T Ltd because it owns 40% directly and
because it controls another 20% through its control of S Ltd.

You may also come across examples where there is a sub-associate. This will be covered in a
later section of this chapter.
Status of the investment The starting point in any question involving a complex structure is to
draw a diagram of the group and then decide on the status of the bottom company in relation to
the ultimate parent. The bottom company will either be a sub-subsidiary as shown above or a
subassociate. The status of the bottom investment is always decided in terms of whether H can
exercise control or significant influence either directly or indirectly.

Date of acquisition of the sub-subsidiary A subsidiary must be consolidated from the date of
acquisition. The issue is the date of acquisition of the sub-subsidiary. It is not always obvious as
to what that date should be. The acquisition date for a sub-subsidiary in a vertical group depends
on whether:

sub-subsidiary T; or

sub-subsidiary T.

The date of acquisition of the sub-subsidiary is the later of the date on which the main subsidiary
(S) was purchased by the parent and the date on which the subsubsidiary (T) was purchased by
the main subsidiary (S). In other words:
: If the holding company H acquired its shares in subsidiary S before S acquired its
shares in the sub-subsidiary T, the date that T becomes a member of the H Group is the date that
S acquired the shares in T.

497
: If the holding company H acquired its shares in subsidiary S after S acquired its
shares in the sub-subsidiary T, the date that T becomes a member of the H Group is the date that
H acquired its shares in S

Illustration: Date of acquisition – Situation 1

75% 1 January 20X1

60% 1 January 20X3

Explanation
H acquired 75% of S on 1 January 20X1. S acquired 60% of T two years later on 1 January
20X3. H acquired control of T when S bought its interest on 1 January 20X3. Therefore, the date
of acquisition of T Ltd from H Ltd‟s viewpoint is 1 January 20X3.

Illustration: Date of acquisition – Situation 2

75% 1 January 20X3

60% 1 January 20X2

Explanation
H acquired 75% of S on 1 January 20X3. S already held 60% of T. H therefore acquired control
of S and T at the same date. Therefore, the date of acquisition of T Ltd from H Ltd‟s viewpoint is
1 January 20X3.

498
MIXED GROUPS
In a mixed group, the parent has both direct and indirect interests in the subsubsidiary. The
following illustration was shown earlier in section 1 of this chapter. It is repeated here for your
convenience.

Illustration: Mixed group

60%

S 40%

20%

H Ltd owns 60% of S Ltd and 40% of T Ltd


S Ltd owns 20% of W Ltd. S Ltd is a subsidiary of H Ltd.
T Ltd is also a subsidiary of H Ltd.

Effective interest in mixed groups


In the above example of a mixed group, H has two interests in T:

hrough its control of S, which also owns shares in T.

The group interest in the sub-subsidiary T is the sum of the direct interest and the indirect
interest in T as follows:

499
Illustration: Mixed group – Effective interest in sub-subsidiary
H‟s direct holding in T 40%
H‟s indirect holding in T (60% x 20%) 12%
H‟s effective holding in T 52%
Non-controlling interest in T (balance) 48%
100%

Date of acquisition of the sub-subsidiary in a mixed group


The same rules apply to mixed groups as they do to vertical groups.
Illustration: Mixed group

Illustration: Date of acquisition – Situation 1


H

80%

1 May 20X1 S 30% 1 May 20X1

45%

1 May 20X3 T

Commentary
H obtains control of S on 1 May 20X1. H obtains control of T on 1 May 20X3 when S acquires
its stake in T. From 1 May 20X2 to 1 May 20X3, T is an associate of H. From 1 May 20X3
onwards T is a subsidiary of H and H has an effective holding of 66% (30% + (80% × 45%)) in
T.

Illustration: Date of acquisition – Situation 2

80%

1 May 20X4 S 30% 1 May 20X2

500
45%

1 May 20X1 T

Commentary
H achieves significant influence over T on 1 May 20X2. H obtains control of S on 1 May 20X4.
Thus H also obtains control of T. due to gaining indirect control over S‟s holding in T. From 1
May 20X2 to 1 May 20X4, T is an associate of H. From 1 May 20X4 onwards T is a subsidiary
of H.
The above illustrations show that there is a further complication that must be taken into account
when consolidating mixed groups. If H‟s direct and indirect interests in the sub-subsidiary arise
on different dates the step acquisition rules apply.

ANALYSING AND INTERPRETING FINANCIAL


STATEMENTS
USERS OF FINANCIAL STATEMENTS
Users and their information needs

501
The IASB Framework outlines seven different groups of users of financial statements. Each user
group has different information needs, but as a general rule financial statements prepared in
accordance with IFRSs should provide all user groups with most of their needs.
The table below lists the user groups, indicates the information that they require from published
reports and accounts, and suggests which items in the financial statements will be of most
interest to each group.
User Information needs Items of interest
Investors/potential investors � Risks and returns relating � Trend analysis: changes in
to revenue, costs and profits
their investment over the past few years
� Security of dividend � Dividend cover
payments � Events and
� Information to make announcements after the
decisions about buying, reporting period
selling or holding shares � Share price
� Future growth prospects. � Corporate governance
reports. Narrative business
review.
Employees � Stability of the company � Profitability and cash
(job position
security and job prospects) � Increases in salaries (%)
� Information about the relative to increases in
company‘s ability to pay profit and dividends
bonuses or higher salaries. � Directors‘ remuneration
Lenders � Whether the entity has � Cash flow
(banks, sufficient cash flow to repay � Total borrowing by the
bondholders) loans entity: financial gearing
� The entity‘s ability to pay � Interest cover
interest � New charges created over
� The adequacy of collateral/ the entity‘s assets
security for loans and bonds
Suppliers � The entity‘s ability to settle � Net current assets
its liabilities � Growth record
� The entity‘s ability to
survive and continue as a
customer
Customers � The entity‘s ability to � Growth record
survive and continue as a � Cash flow
supplier
Government � The entity‘s contribution to � Revenue and profit
the economy � Market share
� Regulation of activities
� Taxation
� Obtaining government
statistics
General public � Environmental and social � Environmental and social
awareness reports

502
� Contributions to the local � Directors‘ report
economy � Narrative business review
Note
Management are not included as a user group because they should have access to much more
detailed information about the company‘s financial position and performance, from internal
reports and budgets.

FINANCIAL RATIOS
A ratio is a mathematical relationship between two variables in the form of a fraction or
percentage. Ratio analysis is primarily concerned with the calculation of relationships which
after proper identification and interpretation may provide information about the operations and
state of affairs of a business enterprise.

Scope: With a single financial ratio, no conclusion is to be arrived at. The ratios are to be studied
in relation to each other, in comparison of the past ratios of the firm as well as ratios of the
industry, better with its immediate competitors to understand their relative significance and
impact.
Ratios are the symptoms of health of an organisation like blood pressure, pulse or
temperature of an individual. Ratios are the indicators for further investigation.

A single financial ratio on its own is not useful. It only becomes useful when it is compared with
other yardsticks. These are three (3) yardsticks which a ratio can be compared with and these
are as follows:
 Comparison with the ratio of the same company in previous periods
 Comparison with ratios of other companies in the same industry
 Comparison with a target ratio, an ideal ratio or industry average

ADVANTAGES OR SIGNIFICANCE OF FINANCIAL RATIOS


The main significance of financial ratios are as follows:
 Financial ratios provide management information on operating efficiency
 Financial ratios are very useful for inter-company comparison to help management
assess how it is fairing in the industry that it operates
 Financial ratios can be used to predict the failure or success of a company
 It can also help management to take decision on whether to continue with a line of
business, product or to discontinue
 It helps management to identify the relative contribution of an asset. This aids
management to decide whether to outsource the asset or dispose of it.

LIMITATIONS OF RATIO ANALYSIS


The main limitations of the usefulness of ratio analysis are as follows:
 Unless ratios are calculated in a uniform manner, from uniform data, comparisons can
be very misleading
 The accounting periods covered by the financial statements may not reflect
representative financial positions. Many businesses produce financial statements to a
date on which there are relatively low amounts of trading activities. If trade is seasonal,
the items in the Statement of Financial Position may not be representative of values
throughout the accounting period.

503
 Financial statements themselves have limitations, e.g., they contain arbitrary estimates
and figures which are based on personal decisions.
 Definition of financial ratios is sometimes not uniform across companies and industries
 Financial ratios alone do not mean anything unless compared with a yardstick
 Financial ratios are historical and do not tell management the health of the company
now.
 Differences in accounting policies, financing structures and financing policies, level of
diversification, government incentive, production technologies and buying policies
make comparisons of companies using ratios flawed.

Limitations of interpretation techniques


There are several limitations or weaknesses in the use of interpretation techniques for analysing
the financial position and financial performance of companies. Some of these are limitations of
ratio analysis (the method of interpretation most often used) and some are limitations of
financial statements and financial information.

Differences in accounting policy


One of the uses of financial ratios is to compare the financial position and performance of one
company with those of similar companies for the same period.
Comparisons between companies might not be reliable, however, when companies use different
accounting policies, or have different judgements in applying accounting policies or making
accounting estimates. For example:
 Entities might have different policies about the revaluation of non-current assets.
 Entities might use different methods of depreciation.
 Entities might use different judgements in estimating the expected profitability on
incomplete construction contracts.
 Entities might use different judgements in assessing whether a liability should be treated
as a provision or a contingent liability.
IAS 8 states that an entity should not change its accounting policies unless the change is
required by an accounting standard or it will result in more relevant and reliable information.
Therefore changes should not happen often.
Where there has been a change in an accounting policy, IAS 8 also requires comparative figures
to be restated and information to be disclosed. However,changes in accounting policies and
accounting estimates can still make it difficult to compare the financial statements of an entity
over time, particularly if analysis is based on extracts rather than the full published financial
statements.

Other limitations in the use of financial ratios


There are other problems with the use of financial ratios, particularly where these are used to
compare the performance and position of different entities or of an entity with an industry
average.
 It is possible to calculate the same ratio in different ways. For example, there are several
variations of return on capital employed (ROCE) and gearing.Comparisons can be
misleading if different calculations are used.

504
 Even where two entities operate in the same industry, comparisons can be misleading.
Entities can operate in different markets (for example, high volume/low margin sales
and low volume/high margin sales). The size of an entity can affect the way it operates
and therefore its ratios. For example, large entities can often negotiate more favourable
terms with suppliers than small ones.
 Financial statements are published infrequently. If ratios are used to study trends and
developments over time, they are only useful for trends or changes over one year or
longer, and not changes in the short term.
 Ratios can only indicate possible strengths or weaknesses in financial position and
financial performance. They might raise questions about performance, but do not
provide answers. They are not easy to interpret, and changes in financial ratios over
time might not be easy to explain.
It can be argued that financial position and financial performance should be analysed using
market values rather than accounting values. For example, it can be argued that investment
yield is more relevant for the assessment of financial
performance than return on capital employed.

Using historical information


Financial statements are often used to predict the future performance of an entity.Where
comparative figures are available for several years it may be possible to extrapolate trends and
to base forecasts on these. If comparative figures are only available for one or two years,
predictions may be unreliable.
There may be some limited information about future transactions in the notes to the financial
statements. For example, details of contingent liabilities and non-adjusting events after the
reporting period must be disclosed. However, published financial statements present historical
information.
Generally, financial statements do not reflect future transactions or events. They do not
anticipate the effect of significant changes to the entity after the financial statements have been
authorised for issue. These may include events beyond the control of management (for example,
the liquidation of a major customer) or events that could not possibly have been foreseen at the
time the most recent financial statements were issued.
It should also be remembered that financial statements are not normally published until several
months after the year end. The financial statements are often out of date by the time that they
become available.

Creative accounting
Management may use various forms of creative accounting to manipulate the view given by
the financial statements while complying with all applicable accounting standards and
regulations.
Some of the techniques that can be used have been discussed in earlier chapters.
They include:
 Window dressing: an entity enters into a transaction just before the year end and
reverses the transaction just after the year end. For example, goods are sold on the
understanding that they will be returned immediately after the year end;this appears to
improve profits and liquidity. The only reason for the transaction is to artificially
improve the view given by the financial statements.

505
 ‘Off balance sheet’ finance: transactions are deliberately arranged so as to enable an
entity to keep significant assets and particularly liabilities out of the statement of
financial position ( ‗off balance sheet‘). This improves gearing and return on capital
employed. Examples include sale and repurchase agreements and some forms of
leasing.
 Changes to accounting policies or accounting estimates: for example, an entity can
revalue assets (change from the cost model to the revaluation model) to improve
gearing or change the way in which it depreciates assets to improve profits.
 Profit smoothing: manipulating reported profits by recognising (usually) artificial assets
or liabilities and releasing them to profit or loss as required.
 Aggressive earnings management: artificially improving earnings and profits by
recognising sales revenue before it has been earned.
 Capitalising expenses: recognising ‗assets‘ which do not meet the definition in the IASB
Conceptual Framework or the recognition criteria. Examples include: human resources,
advertising expenditure and internally generated brand names.Most of these are now
effectively prevented by accounting standards. However,management may still attempt
‗creative accounting‘, especially if the entity is suffering falling profits or poor cash
flow.If directors‘ salaries or bonuses are based on profits or on particular measures,
(such as earnings per share), they may try to manipulate that particular measure so that
it is as favourable to them as possible.

Related party relationships and transactions


A user of financial statements will normally expect the financial statements to reflect
transactions that have taken place on normal commercial terms (‗at arm‘s length‘).The user of
the financial statements would want to be informed if:
 transactions have taken place that were not at ‗arm‘s length‘, or
 there are parties that could enforce transactions on the entity that are not on an ‗arm‘s
length‘ basis.
For example, an entity might sell an asset such as a property to another company owned by one
of its directors on more favourable terms than it would sell to a third party.
In this situation, the financial performance or financial position reported by the financial
statements would be misleading. There is a special relationship between the parties to the
business transactions. This is referred to as a ‗related party relationship‘.

Related parties of an entity can include:


 parents, subsidiaries and fellow subsidiaries
 associates
 key management personnel (such as directors)
 close family members of any of the above.

A related party transaction is:


 a transfer of resources, services, or obligations between related parties
 whether or not a price is charged.
Examples of related party transactions include:
 purchases or sales of goods

506
 purchases or sales of property and other assets
 rendering or receiving of services
 leases
 finance arrangements (such as loans or contributions to equity).
Related party relationships and transactions are a normal part of business and there is nothing
wrong with entering into them. However, a related party relationship can have an effect on the
profit or loss, or on the financial position of an entity, because related parties might enter into
transactions with each other on terms that other entities or individuals (unrelated parties)
would not. For example, where an entity sells goods to a related party, its profits may not be
comparable with those of a similar entity that only trades with third parties on normal
commercial terms.

Using figures in the statement of financial position


In practice, ratio calculations are often based on figures in the year-end statement of financial
position. These may be very similar to average values for the period, but this is not always the
case.
Some businesses are seasonal and make a high proportion of their sales at a specific time of year
(for example, in the few months before a national holiday period).Seasonal businesses often
arrange their year-ends so that they fall when inventories and receivables are at their lowest
(probably just after the main period for sales).
Where this happens, ratios such as inventory turnover will be lower than they would be if they
were based on the average figure for the year. This means that ratios may not be strictly
comparable with those of other businesses or with
industry averages.
Major purchases of assets can have a significant effect on figures in the statement of financial
position and on ratios if they take place near the end of the accounting period.
 The carrying value of non-current assets is unusually high, because cost has increased,
but a full year‘s depreciation has not been charged.
 Return on capital employed and asset turnover are reduced, because assets have
increased but revenue and profits have not. New assets should generate increased
profits, but they have not yet been owned for long enough to do so.
Non-financial information
One of the most serious limitations of traditional financial statements is that they only reflect
the financial effects of transactions. Items are not recognised unless they can be measured
reliably in money terms.
There are two problems here:
 Businesses and the transactions that they enter into are becoming increasingly complex.
Much information that is relevant to users cannot be expressed easily in monetary terms
or in numbers.
 Businesses increasingly accept that they are not only accountable to investors and
lenders, but to a much wider group of people, or ‗stakeholders‘.
Stakeholders can include customers, suppliers, employees, the local community as a whole and
(for some large public entities) society as a whole. These groups are often more interested in the
non-financial effects of an entity‘s activities, (for example, its effect on the natural environment),
than in its financial performance.
At present entities reporting under IFRSs do not have to publish any non-financial information
of this kind. Recently the IASB issued a non-mandatory ―Practice Statement on Management

507
Commentary‖. It is up to companies or individual legal jurisdictions to decide whether to
follow this guidance. (The practice statement is not in the Level 2 Financial Reporting syllabus).

Useful non-financial information


Useful non-financial information could include the following:
 a description of the business, objectives and strategies of the entity
 a narrative review of the performance of the business during the period
 a description of the main risks and uncertainties facing the entity and the ways in which
these risks are managed
 details of any significant factors or events that may have an impact on the entity‘s
performance in future details of any significant factors or events that may have an
impact on the entity‘s cash flows in future
 information about key relationships with other entities and transactions with related
parties, including management
 a description of the entity‘s research and development activities (if any) and of any
material intangible assets, including internally generated intangible assets that have not
been recognised in the balance sheet
 additional explanations of amounts included in the financial statements, where
appropriate (for example, where these are based on estimates)
 information about the entity‘s policies in relation to environmental matters, in relation to
its employees and on social and community issues.

RATIO ANALYSIS
Ratio analysis is one of the most widely used techniques used for analyzing financial
statements.
a) Choice of Ratios
The variety of ratios that can be calculated is vast; so it is important to restrict the calculations
by being selective. The ratios chosen should be the key ones relevant to the requirements of the
situation. The main point to emerge is that there is no single group of ratios suitable for all
purposes; specific ratios are required for specific purposes and the analysis must be developed
accordingly.
b) Commenting on the ratios
Ratios are meaningless on their own unless they are interpreted to bring out their intended
meaning and to tell the intended stories to aid management decisions. If you doubt that you
have anything to say, the following points should serve as a check list:
 What does the ratio literally mean?
 What does a change in the ratio mean?
 What is the norm?
 What are the limitations of the ratio?

TYPES OF FINANCIAL RATIOS


Generally, ratios are classified into the following categories:
 Profitability ratios
 Liquidity ratios
 Asset Management/Activity ratios
 Shareholders‘ (investors) investment ratio

508
 Financial Leverage (debt) or Gearing ratios.

PROFITABILITY RATIOS
Profitability is the ability of a business to earn profit over the period of time. These ratios
measure a company‘s operating efficiency. It measures the company‘s ability to generate profit
to enable it grow, survive and meet the demands of shareholders and other stakeholders.
Profitability ratio shows the combined effects of liquidity, assets management (activity) and
debt management (gearing) on operating results. The overall measure of success of a business is
the profitability which results from the effective use of its resources.
The main profitability ratios include:
 Return on Capital Employed (ROCE)
This ratio is the most important profitability ratio since it relates the profit to the amounts of
investment used in the production of the profit. It is the relationship between a company‘s
profit before interest and the amount of investment. Return on capital employed (ROCE) is
composed of operating profit margin and assets turnover.
The ROCE is expressed as:
Return on Capital Employed (ROCE) = Profit before interest and tax (PBIT) × 100%
Capital Employed

Or
ROCE = PBIT × Sales
Sales Capital Employed

Note: Profit before interest and tax usually seen to be better figure to use than profit after tax,
because there might be unusual variation in the tax charge from year to year and from company
to company.
Capital Employed is defined as total equity (shareholders fund) and long term debt.

 Gross Profit Margin


This ratio explains the relationship between a company‘s gross profit and sales. Normally the
gross profit has to rise proportionally with sales. It can also be useful to compute the gross
profit margin across similar businesses although there will often be good reasons for any
disparity.
It can be expressed as:
Gross Margin = Sales-Cost of sales × 100%
Sales

A change in this ratio over various accounting periods may be traced to a change in:
i) Selling prices-normally deliberate though sometimes unavoidable, for example, because
of increased competition
ii) Sales mix (often deliberate)
iii) Purchase cost including carriage and discounts
iv) Inventory (errors in counting, valuing or cut off, inventory shortages)
Low margins usually suggest poor performance but may be due to expansion cost (e.g.
launching a new product) or trying to increase market share.
Above average margins are usually a sign of good management although unusually high
margins may make the competition keen to join in and enjoy the rich pickings.

509
 Net Profit Margin
It shows the relationship between net profit after tax and sales. It shows the company‘s ability
to control its operating expenses. It is expressed as:
Net Profit Margin = Net profit after tax × 100%
Sales

This ratio is affected by items such as depreciation of non-current assets, selling and
administration expenses etc.
 Return on Equity (ROE)
This ratio shows the profit attributable to the amount invested by the owners of the business. It
also shows potential investors into the business what they might hope to receive as a return.
This ratio explains the relationship between the amount available for distribution to equity
holders and total equity. The shareholders equity includes share capital, share premium,
distributable and non-distributable reserves. The ratio is calculated as follows:
Return on Equity (ROE) = After tax earnings × 100%
Shareholders‘ Fund
LIQUIDITY RATIOS
Liquidity refers to the ability of a firm to meet its short-term financial obligation when and as
they fall due. The main concern of liquidity ratio is to measure the ability of the firms to meet
their short-term maturing obligations. Thus, the ratios aid management to assess whether the
company can pay its short term pressing obligations. Failure to do this will result in the total
failure of the business, as it would be forced into liquidation.
The current ratio is given as follows:

 Current Ratio
Current ratio expresses the relationship between the firm‘s current assets and its current
liabilities. Current assets are assets that are of short term in nature, for example cash, stock,
short term investment and other marketable instruments. Current liabilities consist of accounts
payable, short term notes payable, short term loans, accrued income tax and accrued expenses.
It is expressed as:
Current Ratio = Current Assets
Current Liabilities
The rule of thumb says that the current ratio should be at least 2, that is, the current assets
should meet current liabilities at least twice. (In practice, what is an ideal ratio depends
on the industry and the particular circumstances of the entity involved.)
A higher figure should be perceived with some suspicion as it may be due to high level of
inventories and receivables or high cash levels which could be put to better use.

 Quick Ratio
This is represented by current assets less stock divided by current liabilities. This ratio realizes
that some of current assets are not easily convertible to cash e.g. inventories. The quick ratio,
also referred to as acid test ratio, examines the ability of the business to cover its short term
obligations from its ‗quick‘ assets only (i.e. It ignores stock). The quick ratio is calculated as
follows:

510
Quick ratio = Current Assets- inventory
Current Liabilities
Clearly this ratio will be lower than the current ratio but the difference between the two (the
gap) will indicate the extent to which current assets consist of stock.

Cash Ratio or Absolute Liquid Ratio


Cash is the most liquid asset. Although receivables (debtors) and bills receivable are,
generally,better realisable than inventories, still, there are doubts regarding their realisation,
more so, in time. So, they are not considered, immediately, available for making payments and
so excluded for the calculation of cash ratio.
Cash Ratio = Cash & Bank + Short-term Securities
Current Liabilities
Cash ratio of 1:2 is considered acceptable. It means Rs. 1 liquid assets are considered adequate
to pay Rs. 2 of current liabilities as all the creditors are not expected to demand cash,at the same
time, and cash may be realised, at least something, from debtors and inventories too. More so,
sanctioned working capital limits of the bank are not always, fully, utilised and the balance
drawing power is available to the firm for immediate withdrawal of cash.
Illustration :
Current Ratio = 2.8
Acid-test Ratio = 1.5
Working Capital = GHC 162,000

Calculate:
(A) Current Assets
(B) Current Liabilities
(C) Liquid Assets

Solution:
Current Ratio = 2.8
It means if current assets are 280, current liabilities are 100.
Current Ratio = Current Assets
Current Liabilities
= 280
100
= 2.8
Working Capital = Current Assets – Current Liabilities
= 2.8 – 1
= 1.8
Working capital = 1,62,000
In such a case, if 1.8 is GHC 162,000 current Assets (2.8) would be 252,000 and current liabilities
(1) would be GHC 90,000.
The correctness can be checked with the current ratio.
Current Ratio =252,000
90,000
= 2.8
Acid Test Ratio or Liquid Ratio = Liquid assets
Current liabilities

511
Liquid assets = current assets – stock
Liquid ratio is 1.5. It means if current liabilities are 1, liquid assets are 1.5.
Current liabilities are GHC 90,000, so liquid assets are GHC135,000.
Acid Test Ratio or Liquid Ratio =135,000
90,000
= 1.5
(A) Current Assets = GHC252,000
(B) Current Liabilities = GHC90,000
(C) Liquid Assets =GHC135,000

ASSET MANAGEMENT/ACTIVITY RATIOS


If a business does not use its assets effectively, investors in the business would rather take their
money and place it somewhere else. In order for the assets to be used effectively, the business
needs a high turnover. Unless the business continues to generate a high turnover, assets will be
idle as it is impossible to buy and sell fixed assets continuously as turnover changes. Activity
ratios are therefore used to assess how active various assets are in the business.
Basically, this ratio attempts to measure how effectively management has used its assets.
Note: Increased turnover can be just as dangerous as reduced turnover if the business does not have the
working capital to support the turnover increase. As turnover increases, more working capital and cash
are required and if not, overtrading occurs.
Examples of asset management/activity ratios include:

 Average collection period


The average collection period measures the quality of debtors since it indicates the speed of
their collection. The ratio measures the rate at which the company is collecting its credit sales or
sales that have not been paid for and it also informs management of its credit strategies. The
ratio is expressed as follows:
Average Collection Period = Account Receivable ×365days/12months/52 weeks
Annual credit sales
Note:
An increasing collection period usually is a bad sign as it suggests lack of proper credit control
leading to the impairment of the firm‘s liquidity. However it may be due to:
 A deliberate policy to extend the stated credit period to attract more trade
 One major new customer being allowed different terms.
A falling collection period is usually a good sign, though it could indicate that the company is
facing a cash shortage, hence forcing its customers to pay earlier than initially agreed.

 Inventory Turnover
This ratio measures the stock in relation to turnover in order to determine how often the stock
turns over in the business. It indicates the efficiency of the firm in selling its product. It is
calculated as:
Inventory turnover = Sales or cost of goods sold
Average Inventory
Average inventory is calculated as:
Opening inventory + Closing Inventory
2

512
Note that if the cost of goods sold figure is available they are more preferably used than to use
the sales figures.

 Stock Turnover Period


The stock turnover period shows the average number of days that items are held in inventory. It
shows vigorous a company is trading. It is given as:
Stock Turnover Period = Average Inventory × 365 days/12 months/52 weeks
Cost of Sales

A lengthening stock turnover period indicates slow down in trading and/or stock build up to
prevent stock run outs or to meet unexpected increase in future demand.
A high stock turnover ratio would also tend to indicate that there was little chance of the firm
holding damaged or obsolete stock.

 Total Assets Turnover


Assets Turnover is the relationship between a company‘s sales and assets. The assets turnover
indicates the efficiency with which the firm uses all its assets to generate sales revenue.
Generally, the higher the firm‘s total assets turnover, the more efficiently its assets have been
utilized.
It is calculated as:
Assets Turnover = Sales
Total Assets

 Fixed Assets Turnover


The fixed assets turnover ratio measures the efficiency with which the firm has been using its
fixed assets to generate sales. Generally, high fixed assets turnovers are preferred since they
indicate a better efficiency in fixed assets utilization.
It is calculated as:
Fixed Assets Turnover = Sales
Net Fixed Assets
 Creditors Payment Period
Creditors‘ payment period gives an indication of the length of credit allowed to the company by
suppliers. It is measured as:
Creditors Payment Period = Trade Creditors × 365 days/12 months/52 weeks
Annual credit Purchases

A long payment period may be good. It represents a source of free finance and how well the
company is able to negotiate credit terms. It may however indicate that the company is unable
to pay more quickly because of liquidity problems.
Note that if the payment period is long,
 The company may develop a poor reputation as slow payer and not be able to attract
new suppliers,
 Existing suppliers may decide to discontinue suppliers
 The company may be losing out if worthless cash discount

SHAREHOLDERS’ (INVESTOR) INVESTMENT RATIO

513
These ratios are regarded as very important in the context of reaching a decision about whether
to buy or sell shares. Investors are concerned about the amount of cash discounted to its present
value which they will receive from their investments in shares. This cash is the result of
dividends received and the proceeds from the shares when they are sold.
These are ratios that measure potential and actual growth of a shareholder‘s investment. These
ratios include:

 EARNINGS PER SHARE (EPS)


This ratio shows how much a company earns on each share. The EPS is used primarily as a
measure of profitability and so an increasing EPS is a good sign. It is calculated as:

EPS = Net profit after tax – Preference share dividend


Number of equity shares in issue
The limitations of EPS are as follows:
 In times of rising prices, EPS will increase as profit increases. Thus any improvement in
EPS should be viewed in the context of the effect of the price level changes on the
company‘s profit.
 EPS is dependent on an earnings figure, which is a subjective measure. Some elements of
the earnings figure are particularly subjective such as depreciation charges.
 EPS is a historical figure based on historical accounts.
 EPS cannot be used as a basis of comparison between companies, as the number of
shares in issue in any particular company is not related to the amount of capital
employed.

 DIVIDEND PER SHARE (DPS)


This indicates the amount payable as dividend per an ordinary share. It is usually
recommended by the directors and approved by the shareholders. This is the dividend
expressed as an annual rate of return on the share price. The ratio is therefore computed as
follows:

Dividend Per Share = Ordinary Dividend Payable /Paid


Number of Equity Shares

 EARNINGS YIELD
This measure relates EPS to the market price per share. It shows how much will the price paid
for an equity share yield in the form of earnings. It is a very important market performance
indicator.
It is computed as:

Earnings Yield = Earnings per share × 100% or Net profit after tax – Pref. Dividend
Market price per share Market Capitalisation

 DIVIDEND YIELD
The dividend yield ratio indicates the returns that investors are obtaining on their investment in
the form of dividends. This yield is usually fairly low as the investors are also receiving capital

514
growth on their investment in the form of an increased share price. The dividend yield ratio
compares the dividend per share against the price of the share and is calculated as:

Dividend Yield = Dividend per share ×100%


Market price per share
Normally a very high dividend yield signals potential financial difficulties and possible
dividend payout cut.

 PRICE-EARNING RATIO (P/E RATIO)


This ratio is the reciprocal of the Earnings yield and it indicates the payback period of the
investment made in the shares. This ratio is generally used to evaluate a company‘s share price.
The P/E ratio shows the premium or the discount investors are ready to pay or receive for their
investment. It is commonly referred to as Earnings Multiple.
It is computed as:

P/E ratio = Market price per share or Market capitalisation


Earnings Per share Total Earnings

The higher the price in relation to earnings, the higher the P/E ratio which indicates the higher
the premium an investor is prepared to pay for the share. This occurs because the investor is
extremely confident of the potential growth and earnings of the share.
High P/E ratio generally reflects lower risk and/or higher growth prospects for earnings.
Correspondingly, the lower the P/E ratio, the lower the expected future growth in earnings.

 DIVIDEND COVER
This ratio measures the amount of earnings that are being paid out in the form of dividends, i.e.
how many times the dividend paid out to equity shareholders are covered by earnings.
It shows the relationship between available profits and ordinary dividends payable out of those
profits, reflecting how sustainable the dividend level is likely to be in future.
It is computed as:

Dividend Cover = Earnings for equity shareholders or Earnings per share


Equity share dividend payable Dividend per share

A higher cover would indicate that a larger percentage of earnings are being retained and re-
invested in the business while a lower dividend cover would indicate the converse.

 DIVIDEND PAYOUT RATIO


This ratio is the reciprocal of the dividend cover. It indicates what percentage of the earnings for
the year (available to equity shareholders) was actually declared as dividend. This ratio looks at
the dividend payment in relation to net income or earnings.
It is computed as:
Dividend Payout ratio = Equity share dividend payable × 100% or Dividend per share × 100%
Earnings available to equity shareholders Earnings per share
GEARING OR FINANCIAL LEVERAGE RATIOS

515
Gearing is an expression of the relationship between the amount of finance provided by equity
shareholders and the amount provided by lenders. Gearing affects the riskiness of the company
and the potential return to ordinary shareholders.
Gearing ratios measure the relationship between equity capital and debt capital of the company
and they also indicate long term financial risk and stability of the company i.e. it shows the
degree to which the activities to which the activities of a firm are supported by creditor‘s funds
as opposed to owners.
Note, the greater the proportion of equity funds, the greater the degree of financial strength.
Financial leverage will be to the advantage of the ordinary shareholders as long as the rate of
earnings on capital employed is greater than the rate payable on borrowed funds.
A company has high gearing when a high proportion of its capital is in the form of preference
shares, debentures or a loan
The following ratios can be used to identify the financial strength and risk of the business.

 DEBT TO EQUITY RATIO


This ratio indicates the extent to which debt is covered by shareholders‘ funds. It reflects the
relative position of the equity holders and the lenders and indicates the company‘s policy on the
mix of capital funds. Within this context, preference shares are considered as debt capital
(probably because they normally attract fixed rate of dividends and also they are cumulative).
The debt to equity ratio is calculated as follows:

Debt-to-equity ratio = Long term loan + Preference Shares or Total Debts


Equity shares + Surpluses Shareholders‘ fund

An alternative calculation is
Debt –to-equity ratio = Long term loans + Preference shares
Shareholders‘ fund + Debt capital
A figure of more than 100% or more than 1 means that debt capital is higher than equity fund
and that indicates a highly geared entity.

 EQUITY TO ASSETS RATIO


This ratio tells us what proportion of total assets is financed by equity and hence what
proportion is financed by loans. It is also referred to as proprietary ratio. It is calculated as:

Equity ratio = Equity share capital + Surplus ×100%


Total Assets
A high equity to assets ratio means that most or all of the long term capital is equity and that
much of the business is financed by equity fund. A low equity to assets ratio means that much
of the business is financed by loans.
 DEBIT RATIO
This is the measure of financial strength that reflects the proportion of capital which has been
funded by debt, including preference shares. Debt ratio is complementary to the equity ratio as
total debt plus equity gives 100% of the assets.
This ratio is calculated as follows:
Debt ratio = Long term loan + Preference shares
Total Assets
The higher the debt ratio the more difficult it becomes for the firm to raise debt.

516
 TIMES INTEREST EARNED RATIO OR FIXED INTEREST COVER RATIO
This ratio measure the extent to which earnings can decline without causing financial losses to
the firm and creating an inability to meet the interest cost. The times interest earned shows how
many times the business can pay its interest bills from profit earned.
Times interest earned ratio is calculated as:

Interest Cover ratio = Earnings before interest tax (EBIT)


Interest (on long term loans) charges

Present and prospective loan creditors such as bondholders are vitally interested to know how
adequate the interest payments on their loans are covered by the earnings available for such
payments.
Owners, Managers and Directors are also interested in the ability of the business to service the
fixed interest charges on outstanding debt.

 FIXED DIVIDEND COVER


This is the ratio of earnings after tax to preference share dividend liability. i.e. it shows the
number of times that the earnings after tax of the company can meet (cover) preference
dividend liability.
It is computed as:
Fixed dividend cover = Net Profit after tax
Preference Share Dividend

REVIEW QUESTIONS
QUESTION 1
Quartile sells jewellery through stores in retail shopping centres throughout the country. Over
the last two years it has experienced declining profitability and is wondering if this is related to
the sector as whole. It has recently subscribed to an agency that produces average ratios across
many businesses. Below are the ratios that have been provided by the agency for Quartile’s
business sector based on a year end of 30 June 2012.
Return on year-end capital employed (ROCE) 16·8%
Net asset (total assets less current liabilities) turnover 1·4 times
Gross profit margin 35%
Operating profit margin 12%
Current ratio 1·25:1
Average inventory turnover 3 times
Trade payables‘ payment period 64 days
Debt to equity 38%

The financial statements of Quartile for the year ended 30 September 2012 are:

517
Income statement
GHC’000 GHC’000
Revenue 56,000
Opening inventory 8,300
Purchases 43,900
–––––––
52,200
Closing inventory (10,200) (42,000)
––––––– –––––––
Gross profit 14,000
Operating costs (9,800)
Finance costs (800)
–––––––
Profit before tax 3,400
Income tax expense (1,000)
–––––––
Profit for the year 2,400
–––––––

Statement of financial position


GHC’000 GHC’000
Assets
Non-current assets
Property and shop fittings 25,600
Deferred development expenditure 5,000
–––––––
30,600
Current assets
Inventory 10,200
Bank 1,000 11,200
––––––– –––––––
Total assets 41,800
–––––––
Equity and liabilities
Equity
Equity shares of GHC1 each 15,000
Property revaluation reserve 3,000
Retained earnings 8,600
–––––––
26,600
Non-current liabilities
10% loan notes 8,000
Current liabilities
Trade payables 5,400
Current tax payable 1,800 7,200
––––––– –––––––
Total equity and liabilities 41,800

518
–––––––
Note: The deferred development expenditure relates to an investment in a process to
manufacture artificial precious gems for future sale by Quartile in the retail jewellery market.
Required:
(a) Prepare for Quartile the equivalent ratios that have been provided by the agency.
(9 marks)
(b) Assess the financial and operating performance of Quartile in comparison to its sector
averages.
(12 marks)
(c) Explain four possible limitations of the usefulness of the above comparison. (4 marks)
(25 marks)

Solution:
(a)Below are the specified ratios for Quartile and (for comparison) those of the business sector
average:

Quartile sector average


Return on year-end capital employed
((3,400 + 800)/(26,600 + 8,000) x 100) 12·1% 16·8%
Net asset turnover (56,000/34,600) 1·6 times 1·4 times
Gross profit margin (14,000/56,000 x 100) 25% 35%
Operating profit margin (4,200/56,000 x 100) 7·5% 12%
Current ratio (11,200:7,200) 1·6:1 1·25:1
Average inventory (8,300 + 10,200/2) = 9,250)
turnover (42,000/9,250) 4·5 times 3 times
Trade payables‘ payment period (5,400/43,900 x 365) 45 days 64 days
Debt to equity (8,000/26,600 x 100) 30% 38%

(b) Assessment of comparative performance


Profitability
The primary measure of profitability is the return on capital employed (ROCE) and this shows
that Quartile‘s 12·1% is considerably underperforming the sector average of 16·8%. Measured as
a percentage, this underperformance is 28% ((16·8 – 12·1)/16·8). The main cause of this seems to
be a much lower gross profit margin (25% compared to 35%). A possible explanation for this is
that Quartile is deliberately charging a lower mark-up in order to increase its sales by
undercutting the market. There is supporting evidence for this in that Quartile‘s average
inventory turnover at 4·5 times is 50% better than the sector average of three times. An
alternative explanation could be that Quartile has had to cut its margins due to poor sales which
have had a knock-on effect of having to write down closing inventory.

Quartile‘s lower gross profit percentage has fed through to contribute to a lower operating
profit margin at 7·5% compared to the sector average of 12%. However, from the above figures,
it can be deduced that Quartile‘s operating costs at 17·5% (25% – 7·5%) of revenue appear to be
better controlled than the sector average operating costs of 23% (35% – 12%) of revenue. This
may indicate that Quartile has a different classification of costs between cost of sales and

519
operating costs than the companies in the sector average or that other companies may be
spending more on advertising/selling commissions in order to support their higher margins.

The other component of ROCE is asset utilisation (measured by net asset turnover). If Quartile‘s
business strategy is indeed to generate more sales to compensate for lower profit margins, a
higher net asset turnover would be expected. At 1·6 times, Quartile‘s net asset turnover is only
marginally better than the sector average of 1·4 times. Whilst this may indicate that Quartile‘s
strategy was a poor choice, the ratio could be partly distorted by the property revaluation and
also by whether the deferred development expenditure should be included within net assets for
this purpose, as the net revenues expected from the development have yet to come on stream. If
these two aspects were adjusted for, Quartile‘s net asset turnover would be 2·1 times
(56,000/(34,600 – 5,000 – 3,000)) which is 50% better than the sector average.

In summary, Quartile‘s overall profitability is below that of its rival companies due to
considerably lower profit margins, although this has been partly offset by generating
proportionately more sales from its assets.

Liquidity
As measured by the current ratio, Quartile has a higher level of cover for its current liabilities
than the sector average (1·6:1 compared to 1·25:1). Quartile‘s figure is nearer the ‗norm‘ of
expected liquidity ratios, often quoted as between 1·5 and 2:1, with the sector average (at 1·25:1)
appearing worryingly low. The problem of this ‗norm‘ is that it is generally accepted that it
relates to manufacturing companies rather than retail companies, as applies to Quartile (and
presumably also to the sector average). In particular, retail companies have very little, if any,
trade receivables as is the case with Quartile. This makes a big difference to the current ratio
and makes the calculation of a quick ratio largely irrelevant.

Consequently, retail companies operate comfortably with much lower current ratios as their
inventory is turned directly into cash. Thus, if anything, Quartile has a higher current ratio than
might be expected. As Quartile has relatively low inventory levels (deduced from high
inventory turnover figures), this means it must also have relatively low levels of trade payables
(which can be confirmed from the calculated ratios). The low payables period of 45 days may be
an indication of suppliers being cautious with the credit period they extend to Quartile, but
there is no real evidence of this (e.g. the company is not struggling with an overdraft). In short,
Quartile does not appear to have any liquidity issues.

Gearing
Quartile‘s debt to equity at 30% is lower than the sector average of 38%. Although the loan note
interest rate of 10% might appear quite high, it is lower than the ROCE of 12·1% (which means
shareholders are benefiting from the borrowings) and the interest cover of 5·25 times ((3,400 +
800)/800) is acceptable. Quartile also has sufficient tangible assets to give more than adequate
security on the borrowings, therefore there appear to be no adverse issues in relation to gearing.

Conclusion
Quartile may be right to be concerned about its declining profitability. From the above analysis,
it seems that Quartile may be addressing the wrong market (low margins with high volumes).
The information provided about its rival companies would appear to suggest that the current

520
market appears to favour a strategy of higher margins (probably associated with better quality
and more expensive goods) as being more profitable. In other aspects of the appraisal, Quartile
is doing well compared to other companies in its sector.

(c) Factors which may limit the usefulness of the comparison with business sector averages:
It is unlikely that all the companies that have been included in the sector averages will use the
same accounting policies. In the example of Quartile, it is apparent that it has revalued its
property; this will increase its capital employed and (probably) lower its ROCE (compared to if
it did not revalue). Other companies in the sector may carry their property at historical cost.

The accounting dates may not be the same for all the companies. In this example the sector
averages are for the year ended 30 June 2012, whereas Quartile‘s are for the year ended 30
September 2012. If the sector is exposed to seasonal trading (although this may be unlikely for
jewellery), this could have a significant impact on many ratios, in particular working capital
based ratios. To allow for this, perhaps Quartile could prepare a form of adjusted financial
statements to 30 June 2012.

It may be that the definitions of the ratios have not been consistent across all the companies
included in the sector averages (and for Quartile). This may be a particular problem with ratios
like ROCE as there is no universally accepted definition. Often agencies issue guidance on how
the ratios should be calculated to minimise these possible inconsistencies. Of particular
relevance in this example is that it is unlikely that other jewellery retailers will have an
intangible asset of deferred development expenditure.

Sector averages are just that: averages. Many of the companies included in the sector may not be
a good match to the type of business and strategy of Quartile. ‗Jewellery‘ is a broad category
and some companies may adopt a strategy of high-end (expensive) goods which have high
mark-ups, but usually lower inventory turnover, whereas other companies may adopt a
strategy of selling more affordable jewellery with lower margins in the expectation of higher
volumes.

Note: Other relevant examples may be acceptable, but they must relate to issues of inter-company
comparison and not general issues of interpretation such as inflation distorting profit trends.

QUESTION 2
Kantanka Co is a manufacturer of domestic appliances. Its chairman is pleased with the results
for the year ended 31 December 2015 as they show a continuing improvement over recent past
performance. However, the finance director says that a better assessment of the company‘s
performance would be made by a comparison to other companies in the same sector. The
finance director has obtained some ratios for Kantanka Co‘s business sector, based on a year
end of 31 December 2015, which are:

Return on capital employed (ROCE) 18·5%


Net asset (total assets less current liabilities) turnover 1·8 times
Gross profit margin 21%
Operating profit margin 10·3%
Current ratio 1·6:1

521
Gearing (debt/equity) 36%

The summarised financial statements of Kantanka Co are:

Statement of profit or loss for the year ended 31 December 2015


GHC‘000
Revenue 62,500
Cost of sales (51,800)
–––––––
Gross profit 10,700
Operating costs (5,800)
Finance costs (1,800)
–––––––
Profit before tax 3,100
Income tax expense (1,000)
–––––––
Profit for the year 2,100
–––––––
Statement of financial position as at 31 December 2015
GHC’000 GHC’000
Assets
Non-current assets
Property 8,100
Owned plant 12,600
Leased plant 12,200
–––––––
32,900
Current assets 16,400
–––––––
Total assets 49,300
–––––––

Equity and liabilities


Equity
Equity shares of GHC1 each 9,000
Property revaluation surplus 4,000
Retained earnings 10,600
–––––––
23,600
Non-current liabilities
10% loan notes 10,000
Finance lease obligations 6,400 16,400
–––––––
Current liabilities
Finance lease obligations 2,100
Other current liabilities 7,200 9,300
––––––– –––––––

522
Total equity and liabilities 49,300
–––––––
Required:
(a) Prepare for Kantanka Co the equivalent ratios to those of its sector.
Note: The finance lease obligations should be treated as debt in the ROCE and gearing
calculations. (6 marks)

(b) Analyse the financial performance and position of Kantanka Co for the year to 31 December
2015 in comparison to the sector averages. (9 marks)
(TOTAL : 15 MARKS)

Solution:
(a) Equivalent ratios for Kantanka Co
Kantanka Sector
average
Return on capital employed ((3,100 + 1,800)/(23,600 + 16,400 + 2,100) x 100)11·6% 18·5%
Net asset turnover (62,500/40,000) 1·6 times 1·8 times
Gross profit margin (10,700/62,500 x 100) 17·1% 21%
Operating profit margin (4,900/62,500 x 100) 7·8% 10·3%
Current ratio (16,400:9,300) 1·8:1 1·6:1
Gearing (debt/equity) ((16,400 + 2,100)/23,600) 78% 36%

(b) Analysis of comparative financial performance and position


Financial performance
As measured by the return on capital employed (ROCE), Kantanka Co‘s overall profitability
does not compare well with its competitors, underperforming the sector average profitability by
over 37% ((18·5% – 11·6%)18·5%). The component parts of overall profitability are asset
turnover and profit margins and, on both of these, Kanatanka Co considerably underperforms
the sector average. The underperformance is worse for profit margins than for asset utilisation
and indeed it is the gross margin which is the main cause of the unfavourable comparison. This
may be due to a deliberate policy of underpricing competitors (to increase sales) or it may be
due to inefficient manufacturing. Kanatanka Co‘s control of operating expenses, as shown by
the difference between gross and operating profit margins, is relatively good (at 9·3% of
revenue compared to 10·7% for the sector) which confirms that it is gross profit margin which is
the problem area, assuming there are no differences in cost classification.
Kantanka Co is generating approximately 11% ((1·8 – 1·6)/1·8) less revenue from its assets
compared to the sector average which (as already noted) is also contributing to overall lower
profitability (ROCE). Apart from the obvious implication that Kanatanka Co may be a less
efficient manufacturer, there could also be a number of other reasons for the lower asset
utilisation. Kantanka Co has revalued its property, whereas it is not known if its competitors
have (without the revaluation Kantanka Co‘s ROCE would be 12·9% ignoring additional
depreciation). Some of Kanatanka Co‘s plant may have been recently acquired and therefore
may not be up to full production capacity, meaning the current year‘s revenue does not contain
sales for a full year in respect of production from this plant. The leasing of plant is usually more
expensive than outright purchase (although the finance costs would not affect ROCE). Of course

523
other competitors may also experience some of these issues, the effects of which would be
included in the sector average figures.
Financial position
The current ratio shows that the liquidity of Kantanka Co is within expected norms and
compares well with its competitors. There may be an argument to exclude the current finance
lease liability from the current ratio which would then put it at 2·3:1 (16,400:7,200) which is
perhaps a little higher than expected norms (usually an upper limit of 2).
Kantanka Co‘s gearing is quite high at more than double that of its competitors. This obviously
increases finance costs and with an interest cover of only 2·7 times (4,900/1,800), any downturn
in profit may place Kantanka Co in a difficult position. That said, a finance cost of 10% on the
loan notes (plus the finance costs of the lease obligations) is a lower percentage than the ROCE
so shareholders are getting a (slight) benefit from the debt, but this is at considerable risk.
It is tempting to say that if Kantanka Co had not leased some of its plant its gearing would be
more in line with the sector average, but this begs the question of how else would it have
financed the plant. Issuing a further loan note would leave Kantanka Co in a similar debt
position as now; only a cash injection from a new share issue would reduce the gearing.
Another possibility is that Kantanka Co could structure its plant leases such that they qualified
as operating rather than finance leases. Indeed, it may be that Kantanka Co already has some
operating leased plant, but this cannot be determined from the information provided.
Conclusion
Kantanka Co is considerably underperforming its sector averages and the finance director is
correct to say that a comparison with its competitors is a better indication of Kantanka Co‘s
current performance than looking at the past trend of its own performance, subject to the
definitions and accounting policies used by other companies in the sector.
The analysis indicates Kantanka Co may need to look at its pricing policy or manufacturing
efficiency and also needs to investigate a strategy of reducing its gearing.

QUESTION 3
Paris Saint Germain (PSG) is a public company that would like to acquire (100% of) a suitable
private company. It has obtained the following draft financial statements for two companies,
Guardiola and Mourinho. They operate in the same industry and their managements have
indicated that they would be receptive to a takeover.

INCOME STATEMENTS FOR THE YEAR ENDED 30 SEPTEMBER 2018


Guardiola Mourinho
GHC’000 GHC’000 GHC’000 GHC’000
Revenue 12,000 20,500
Cost of sales (10,500) (18,000)
–––––––– ––––––––
Gross profit 1,500 2,500
Operating expenses (240) (500)
Finance costs – loan (210) (300)
– overdraft nil (10)
– lease nil (290)
–––––––– ––––––––
Profit before tax 1,050 1,400

524
Income tax expense (150) (400)
–––––––– ––––––––
Profit for the year 900 1,000
–––––––– ––––––––
Note: dividends paid during the year 250 700
–––––––– ––––––––

STATEMENTS OF FINANCIAL POSITION AS AT 30 SEPTEMBER 2018


ASSETS
Non-current assets
Freehold factory (note (i)) 4,400 nil
Owned plant (note (ii)) 5,000 2,200
Leased plant (note (ii)) nil 5,300
–––––––– ––––––––
9,400 7,500
Current assets
Inventory 2,000 3,600
Trade receivables 2,400 3,700
Bank 600 5,000 nil 7,300
–––––––– –––––––– –––––––– ––––––––
Total assets 14,400 14,800
–––––––– ––––––––
EQUITY AND LIABILITIES
Equity shares of GHC1 each 2,000 2,000
Property revaluation reserve 900 nil
Retained earnings 2,600 3,500 800 800
–––––––– –––––––– –––––––– ––––––––
5,500 2,800
Non-current liabilities
Finance lease obligations (note (iii)) nil 3,200
7% loan notes 3,000 nil
10% loan notes nil 3,000
Deferred tax 600 100
Government grants 1,200 4,800 nil 6,300
–––––––– ––––––––
Current liabilities
Bank overdraft nil 1,200
Trade payables 3,100 3,800
Government grants 400 nil
Finance lease obligations (note (iii)) nil 500
Taxation 600 4,100 200 5,700
–––––––– –––––––– –––––––– ––––––––
Total equity and liabilities 14,400 14,800
–––––––– ––––––––
Notes
(i) Both companies operate from similar premises.

525
(ii) Additional details of the two companies‘ plant are:
Guardiola Mourinho
GHC’000 GHC’000
Owned plant – cost 8,000 10,000
Leased plant – original fair value nil 7,500
There were no disposals of plant during the year by either company.

(iii) The interest rate implicit within Mourinho‘s finance leases is 7·5% per annum. For the
purpose of calculating ROCE and gearing, all finance lease obligations are treated as long-term
interest bearing borrowings.

(iv) The following ratios have been calculated for Guardiola and can be taken to be correct:
Return on year end capital employed (ROCE) 14·8%
(capital employed taken as shareholders’ funds plus long-term interest bearing borrowings –
see note (iii) above)
Pre-tax return on equity (ROE) 19·1%
Net asset (total assets less current liabilities) turnover 1·2 times
Gross profit margin 12·5%
Operating profit margin 10·5%
Current ratio 1·2:1
Closing inventory holding period 70 days
Trade receivables‘ collection period 73 days
Trade payables‘ payment period (using cost of sales) 108 days
Gearing (see note (iii) above) 35·3%
Interest cover 6 times
Dividend cover (Earnings/Dividend) 3·6 times
Required:
(a) Calculate for Mourinho the ratios equivalent to all those given for Guardiola above.
(7 marks)

(b) Assess the relative performance and financial position of Guardiola and Mourinho for the
year ended 30 September 2018 to inform the directors of Paris Saint Germain (PSG) in their
acquisition decision. (6 marks)

(c) Explain any TWO limitations of ratio analysis and any further information that may be
useful to the directors of Paris Saint Germain (PSG) when making an acquisition decision.
(2 marks)
(TOTAL: 15 MARKS)

Solution:
(a) Equivalent ratios from the financial statements of Mourinho (workings in GHC’000)
Return on year end capital employed (ROCE) 20·9%
(1,400 + 590)/(2,800 + 3,200 + 500 + 3,000) x 100
Pre tax return on equity (ROE) 50%
1,400/2,800 x 100
Net asset turnover 2·3 times
20,500/(14,800 – 5,700)

526
Gross profit margin 12·2%
2,500/20,500 x 100
Operating profit margin 9·8%
2,000/20,500 x 100
Current ratio 1·3:1
7,300/5,700
Closing inventory holding period 73 days
3,600/18,000 x 365
Trade receivables‘ collection period 66 days
3,700/20,500 x 365
Trade payables‘ payment period 77 days
3,800/18,000 x 365
Gearing 71%
(3,200 + 500 + 3,000)/9,500 x 100
Interest cover 3·3 times
2,000/600
Dividend cover 1·4 times
1,000/700
As per the question, Mourinho‘s obligations under finance leases (3,200 + 500) have been
treated as debt when calculating the ROCE and gearing ratios.

(b) Assessment of the relative performance and financial position of Guardiola and
Mourinho for the year ended 30 September 2018
Introduction
This report is based on the draft financial statements supplied and the ratios shown in (a) above.
Although covering many aspects of performance and financial position, the report has been
approached from the point of view of a prospective acquisition of the entire equity of one of the
two companies.
Profitability
The ROCE of 20·9% of Mourinho is far superior to the 14·8% return achieved by Guardiola.
ROCE is traditionally seen as a measure of management‘s overall efficiency in the use of the
finance/assets at its disposal. More detailed analysis reveals that Mourinho‘s superior
performance is due to its efficiency in the use of its net assets; it achieved a net asset turnover of
2·3 times compared to only 1·2 times for Guardiola. Put another way, Mourinho makes sales of
GHC2·30 per GHC1 invested in net assets compared to sales of only GHC1·20 per GHC1
invested for Guardiola. The other element contributing to the ROCE is profit margins. In this
area Mourinho‘s overall performance is slightly inferior to that of Guardiola, gross profit
margins are almost identical, but Guardiola‘s operating profit margin is 10·5% compared to
Mourinho‘s 9·8%. In this situation, where one company‘s ROCE is superior to another‘s it is
useful to look behind the figures and consider possible reasons for the superiority other than
the obvious one of greater efficiency on Mournho‘s part.
A major component of the ROCE is normally the carrying amount of the non-current assets.
Consideration of these in this case reveals some interesting issues. Mourinho does not own its
premises whereas Guardiola does. Such a situation would not necessarily give a ROCE
advantage to either company as the increase in capital employed of a company owning its
factory would be compensated by a higher return due to not having a rental expense (and vice
versa). If Mourinho‘s rental cost, as a percentage of the value of the related factory, was less

527
than its overall ROCE, then it would be contributing to its higher ROCE. There is insufficient
information to determine this. Another relevant point may be that Mourinho‘s owned plant is
nearing the end of its useful life (carrying amount is only 22% of its cost) and the company
seems to be replacing owned plant with leased plant. Again this does not necessarily give
Mourinho an advantage, but the finance cost of the leased assets at only 7·5% is much lower
than the overall ROCE (of either company) and therefore this does help to improve Mourinho‘s
ROCE. The other important issue within the composition of the ROCE is the valuation basis of
the companies‘ non-current assets. From the question, it appears that Guardiola‘s factory is at
current value (there is a property revaluation reserve) and note (ii) of the question indicates the
use of historical cost for plant. The use of current value for the factory (as opposed to historical
cost) will be adversely impacting on Guardiola‘s ROCE. Mourinho does not suffer this
deterioration as it does not own its factory.
The ROCE measures the overall efficiency of management; however, as PSG is considering
buying the equity of one of the two companies, it would be useful to consider the return on
equity (ROE) – as this is what PSG is buying. The ratios calculated are based on pre-tax profits;
this takes into account finance costs, but does not cause taxation issues to distort the
comparison. Clearly Mourinho‘s ROE at 50% is far superior to Guardiola‘s 19·1%. Again the
issue of the revaluation of Guardiola‘s factory is making this ratio appear comparatively worse
(than it would be if there had not been a revaluation). In these circumstances it would be more
meaningful if the ROE was calculated based on the asking price of each company (which has
not been disclosed) as this would effectively be the carrying amount of the relevant equity for
PSG.
Gearing
From the gearing ratio it can be seen that 71% of Mourinho‘s assets are financed by borrowings
(39% is attributable to Mourinho‘s policy of leasing its plant). This is very high in absolute terms
and double Guardiola‘s level of gearing. The effect of gearing means that all of the profit after
finance costs is attributable to the equity even though (in Mourinho‘s case) the equity represents
only 29% of the financing of the net assets. Whilst this may seem advantageous to the equity
shareholders of Mourinho, it does not come without risk. The interest cover of Mourinho is only
3·3 times whereas that of Guardiola is 6 times. Mourinho‘s low interest cover is a direct
consequence of its high gearing and it makes profits vulnerable to relatively small changes in
operating activity. For example, small reductions in sales, profit margins or small increases in
operating expenses could result in losses and mean that interest charges would not be covered.
Another observation is that Guardiola has been able to take advantage of the receipt of
government grants; Mourinho has not. This may be due to Guardiola purchasing its plant
(which may then be eligible for grants) whereas Mourinho leases its plant. It may be that the
lessor has received any grants available on the purchase of the plant and passed some of this
benefit on to Mourinho via lower lease finance costs (at 7·5% per annum, this is considerably
lower than Mourinho has to pay on its 10% loan notes).
Liquidity
Both companies have relatively low liquid ratios of 1·2 and 1·3 for Guardiola and Mourinho
respectively, although at least Guardiola has GHC600,000 in the bank whereas Mourinho has a
GHC1·2 million overdraft. In this respect Mourinho‘s policy of high dividend payouts (leading
to a low dividend cover and low retained earnings) is very questionable. Looking in more
depth, both companies have similar inventory days; Mourinho collects its receivables one week
earlier than Guardiola (perhaps its credit control procedures are more active due to its large
overdraft), and of notable difference is that Guardiola receives (or takes) a lot longer credit

528
period from its suppliers (108 days compared to 77 days). This may be a reflection of Guardiola
being able to negotiate better credit terms because it has a higher credit rating.
Summary
Although both companies may operate in a similar industry and have similar profits after tax,
they would represent very different purchases. Mourinho‘s sales revenues are over 70% more
than those of Guardiola, it is financed by high levels of debt, it rents rather than owns property
and it chooses to lease rather than buy its replacement plant. Also its remaining owned plant is
nearing the end of its life. Its replacement will either require a cash injection if it is to be
purchased (Mourinho‘s overdraft of GHC1·2 million already requires serious attention) or
create even higher levels of gearing if it continues its policy of leasing. In short although
Mourinho‘s overall return seems more attractive than that of Guardiola, it would represent a
much more risky investment. Ultimately the investment decision may be determined by PSG‘s
attitude to risk, possible synergies with its existing business activities, and not least, by the
asking price for each investment (which has not been disclosed to us).

(c) The generally recognised potential problems of using ratios for comparison purposes are:
– inconsistent definitions of ratios
– financial statements may have been deliberately manipulated (creative accounting)
– different companies may adopt different accounting policies (e.g. use of historical costs
compared to current values)
– different managerial policies (e.g. different companies offer customers different payment
terms)
– statement of financial position figures may not be representative of average values throughout
the year (this can be caused by seasonal trading or a large acquisition of non-current assets near
the year end)
– the impact of price changes over time/distortion caused by inflation

When deciding whether to purchase a company, PSG should consider the following
additional useful information:
– in this case the analysis has been made on the draft financial statements; these may be
unreliable or change when being finalised. Audited financial statements would add credibility
and reliance to the analysis (assuming they receive an unmodified Auditors‘ Report).
– forward looking information such as profit and financial position forecasts, capital
expenditure and cash budgets and the level of orders on the books.
– the current (fair) values of assets being acquired.
– the level of risk within a business. Highly profitable companies may also be highly risky,
whereas a less profitable company may have more stable ‗quality‘ earnings
– not least would be the expected price to acquire a company. It may be that a poorer
performing business may be a more attractive purchase because it is relatively cheaper and may
offer more opportunity for improving efficiencies and profit growth.

QUESTION 4
Comparator assembles computer equipment from bought in components and distributes them
to various wholesalers and retailers. It has recently subscribed to an interfirm comparison
service. Members submit accounting ratios as specified by the operator of the service, and in
return, members receive the average figures for each of the specified ratios taken from all of the

529
companies in the same sector that subscribe to the service. The specified ratios and the average
figures for Comparator‘s sector are shown below.

Ratios of companies reporting a full year’s results for periods ending between 1 July 2013 and
30 September 2013

Return on capital employed 22·1%


Net assets turnover 1·8 times
Gross profit margin 30%
Net profit (before tax) margin 12·5%
Current ratio 1·6:1
Quick ratio 0·9:1
Inventory holding period 46 days
Accounts receivable collection period 45 days
Accounts payable payment period 55 days
Debt to equity 40%
Dividend yield 6%
Dividend cover 3 times

Comparator‘s financial statements for the year to 30 September 2013 are set out below:

Income statement
GHC000
Sales revenue 2,425
Cost of sales (1,870)
––––––
Gross profit 555
Other operating expenses (215)
–––––
Operating profit 340
Interest payable (34)
Exceptional item (note (ii)) (120)
–––––
Profit before taxation 186
Income tax (90)
–––––
Profit after taxation 96
–––––

Extracts of changes in equity


Accumulated profits – 1 October 2012 179
Net profit for the period 96
Dividends paid (interim GHC60,000; final GHC30,000) (90)
–––––
Accumulated profits – 30 September 2013 185
–––––
Statement of Financial Position

530
GHC000 GHC000
Non-current assets (note (i)) 540
Current Assets
Inventory 275
Accounts receivable 320
Bank nil 595
–––– –––––
1,135
–––––
Share Capital and Reserves
Ordinary shares (25 pesewas each) 150
Accumulated profits 185
–––––
335
Non-current liabilities
8% loan notes 300
Current liabilities
Bank overdraft 65
Trade accounts payable 350
Taxation 85 500
–––– –––––
1,135
–––––
Notes
(i) The details of the non-current assets are:
Cost Accumulated depreciation Net book value
GHC000 GHC000 GHC000
At 30 September 2013 3,600 3,060 540

(ii) The exceptional item relates to losses on the sale of a batch of computers that had become
worthless due to improvements in microchip design.

(iii) The market price of Comparator‘s shares throughout the year averaged GHC6·00 each.

Required:
(a) Explain the problems that are inherent when ratios are used to assess a company‘s financial
performance.
Your answer should consider any additional problems that may be encountered when using
interfirm comparison services such as that used by Comparator.

(b) Calculate the ratios for Comparator equivalent to those provided by the interfirm
comparison service.

(c) Write a report analysing the financial performance of Comparator based on a comparison
with the sector averages.

531
Solution:
(a) Ratios are used to assess the financial performance of a company by comparing the
calculated figures to various other sources. This may be to previous years‘ ratios of the same
company, it may be to the ratios of a similar rival company, to accepted norms (say of liquidity
ratios) or, as in this example, to industry averages. The problems inherent in these processes
are several. Probably the most important aspect of using ratios is to realise that they do not give
the answers to the assessment of how well a company has performed, they merely raise the
questions and direct the analyst into trying to determine what has caused favourable or
unfavourable indicators. In many ways it can be said that ratios are only as useful as the skills
of the person using them. It is also true that any assessment should also consider other
information that may be available including non-financial information.

More specific problem areas are:


– Accounting policies: if two companies have different accounting policies, it can invalidate any
comparison between their ratios. For example return on capital employed is materially affected
by revaluations of assets. Comparing this ratio for two companies where one has revalued its
assets and the other carries them at depreciated historic cost would not be very meaningful.
Similar examples may involve depreciation methods, inventory valuation policies etc.

– Accounting practices: this is similar to differing accounting policies in its effects. An example
of this would be the use of debtor factoring. If one company collects its accounts receivable in
the normal way, then the calculation of the accounts receivable collection period would be a
reasonable indication of the efficiency of its credit control department.
However if a company chose to factor its accounts receivable (i.e. ‗sell‘ them to a finance
company) then the calculation of its collection period would be meaningless. A more
controversial example would be the engineering of a lease such that it fell to be treated as an
operating lease rather than a finance lease.

– Balance sheet averages: many ratios are based on comparing income statement items with
balance sheet items. The ratio of accounts receivable collection period is a good example of this.
For such ratios to have meaning, there is an assumption that the year-end balance sheet figures
are representative of annual norms. Seasonal trading and other factors may invalidate this
assumption. For example the level of accounts receivable and inventory of a toy manufacturer
could vary largely due to the nature of its seasonal trading.

– Inflation can distort comparisons over time.

– The definition of an accounting ratio: If a ratio is calculated by two companies using different
definitions, then there is an obvious problem. Common examples of this are gearing ratios
(some use debt/equity, others may use debt/debt + equity). Also where a ratio is partly based
on a profit figure, there can be differences as to what is included and what is excluded from the
profit figure. Problems of this type include the treatment of extraordinary items and finance
costs.

– The use of norms can be misleading. A desirable range for the current ratio may be say
between 1·5 and 2 : 1, but all businesses are different. This would be a very high ratio for a

532
supermarket (with few accounts receivable), but a low figure for a construction company (with
high levels of work in progress).

–Looking at a single ratio in isolation is rarely useful. It is necessary to form a view when
considering ratios in combination with other ratios.
A more controversial aspect of ratio analysis is that management have sometimes indulged in
creative accounting techniques in order that the ratios calculated from published financial
statements will show a more favourable picture than the true underlying position. Examples of
this are sale and repurchase agreements, which manipulate liquidity figures, and off balance
sheet finance which distorts return on capital employed.

Inter firm comparisons:


Of particular concern with this method of using ratios is:
–they are themselves averages and may incorporate large variations in their composition. Some
inter firm comparison agencies produce the ratios analysed into quartiles to attempt to
overcome this problem.
– it may be that the sector in which a company is included may not be sufficiently similar to the
exact type of trade of the specific company. The type of products or markets may be different.
–companies of different sizes operate under different economies of scale, this may not be
reflected in the industry average figures.
– the year end accounting dates of the companies included in the averages are not going to be
all the same. This highlights
issues of balance sheet averages and seasonal trading referred to above. Some companies try to
minimise this by grouping companies with approximately similar year-ends together as in the
example of this question, but this is not a complete solution.

(b)
Calculation of specified ratios:
Comparator Sector average
Return on capital employed (186 +34 loan interest/635) 34·6% 22·1%
Net assets turnover (2,425/635) 3·8 times 1·8 times
Gross profit margin (555/2,425 x 100) 22·9% 30%
Net profit (excluding exceptionals) margin (306/2,425 x 100) 12·6% not available
Net profit (before tax) margin (186/2,425 x 100) 7·7% 12·5%
Current ratio (595/500) 1·19 : 1 1·6 : 1
Quick ratio (320/500) 0·64 : 1 0·9 : 1
Inventory holding period (275/1,870 x 365) 54 days 46 days
Accounts receivable collection period (320/2,425 x 365) 48 days 45 days
Creditor payment period (350/1,870 x 365) (based on cost of sales) 68 days 55 days
Debt to equity (300/335 x 100) 90% 40%
Dividend yield (see below) 2·5% 6%
Dividend cover (96/90) 1·07 times 3 times

The workings are in GHC000 (unless otherwise stated) and are for Comparator‘s ratios.
The dividend yield is calculated from a dividend per share figure of 15pesewas
(GHC90,000/150,000 x 4) and a share price of GHC6·00.
Thus the yield is 2·5% (15p/GHC6·00 x 100%).

533
(c)
REPORT
Analysis of Comparator’s financial performance compared to sector average for the year to 30
September 2013:
To:
From:
Date:
Operating performance
The return on capital employed of Comparator is impressive being more than 50% higher than
the sector average. The components of the return on capital employed are the asset turnover
and profit margins. In these areas Comparator‘s asset turnover is much higher (nearly double)
than the average, but the net profit margin after exceptionals is considerably below the sector
average. However, if the exceptionals are treated as one off costs and excluded, Comparator‘s
margins are very similar to the sector average. This short analysis seems to imply that
Comparator‘s superior return on capital employed is due entirely to an efficient asset turnover
i.e. Comparator is making its assets work twice as efficiently as its competitors. A closer
inspection of the underlying figures may explain why its asset turnover is so high. It can be seen
from the note to the balance sheet that Comparator‘s non-current assets appear quite old. Their
net book value is only 15% of their original cost. This has at least two implications; they will
need replacing in the near future and the company is already struggling for funding; and
their low net book value gives a high figure for asset turnover. Unless Comparator has
underestimated the life of its assets in its depreciation calculations, its non-current assets will
need replacing in the near future. When this occurs its asset turnover and return on capital
employed figures will be much lower. This aspect of ratio analysis often causes problems and to
counter this anomaly some companies calculate the asset turnover using the cost of non-current
assets rather than their net book value as this gives a more reliable trend. It is also possible that
Comparator is using assets that are not on its balance sheet.
It may be leasing assets that do not meet the definition of finance leases and thus the assets and
corresponding obligations are not recognised on the balance sheet.

A further issue is which of the two calculated margins should be compared to the sector
average (i.e. including or excluding the effects of the exceptionals). The gross profit margin of
Comparator is much lower than the sector average. If the exceptional losses were taken in at
trading account level, which they should be as they relate to obsolete inventory, Comparator‘s
gross margin would be even worse. As Comparator‘s net margin is similar to the sector average,
it would appear that Comparator has better control over its operating costs. This is especially
true as the other element of the net profit calculation is finance costs and as Comparator has
much higher gearing than the sector average, one would expect Comparator‘s interest to be
higher than the sector average.

Liquidity
Here Comparator shows real cause for concern. Its current and quick ratios are much worse
than the sector average, and indeed far below expected norms. Current liquidity problems
appear due to high levels of accounts payable and a high bank overdraft. The high levels of
inventory contribute to the poor quick ratio and may be indicative of further obsolete inventory
(the exceptional item is due to obsolete inventory). The accounts receivable collection figure is
reasonable, but at 68 days, Comparator takes longer to pay its accounts payable than do its

534
competitors. Whilst this is a source of ‗free‘ finance, it can damage relations with suppliers and
may lead to a curtailment of further credit.

Gearing
As referred to above, gearing (as measured by debt/equity) is more than twice the level of the
sector average. Whilst this may be an uncomfortable level, it is currently beneficial for
shareholders. The company is making an overall return of 34·6%, but only paying 8% interest
on its loan notes. The gearing level may become a serious issue if Comparator becomes unable
to maintain the finance costs. The company already has an overdraft and the ability to make
further interest payments could be in doubt.

Investment ratios
Despite reasonable profitability figures, Comparator‘s dividend yield is poor compared to the
sector average. From the extracts of the changes in equity it can be seen that total dividends are
GHC90,000 out of available profit for the year of only GHC96,000 (hence the very low dividend
cover). It is worthy of note that the interim dividend was GHC60,000 and the final dividend
only GHC30,000. Perhaps this indicates a worsening performance during the year, as normally
final dividends are higher than interim dividends. Considering these factors it is surprising the
company‘s share price is holding up so well.

Summary
The company compares favourably with the sector average figures for profitability, however
the company‘s liquidity and gearing position is quite poor and gives cause for concern. If it is to
replace its old assets in the near future, it will need to raise further finance. With already high
levels of borrowing and poor dividend yields, this may be a serious problem for Comparator.
Yours faithfully

QUESTION 5
Pink Sheet Ltd is listed on the Ghana Stock Exchange. It assembles domestic electrical goods
which it then sells to both wholesale and retail customers. Pink Sheet Ltd‘s management were
disappointed in the company‘s results for the year ended 31st March, 2012. In an attempt to
improve performance the following measures were taken early in the year ended 31 March
2009.
- A national advertising campaign was undertaken

- Rebates to all wholesale customers purchasing goods above set quantity levels were
introduced

- The assembly of certain lines ceased and was replaced by bought in completed products. This
allowed Pink Sheet Ltd to dispose of surplus plant.

Pink Sheet Ltd‘s summarized financial statement for the year ended 31 March 2013 are set out
below:
Statement of profit or loss and other comprehensive income
GH₵ million
Revenue (25% cash sales) 8,000

535
Cost of sales (6,900)
Gross profit 1,100
Operating expenses (740)
360
Profit on disposal of plant (note (i)) 80
Finance charges (40)
Profit before tax 400
Income tax expense (100)
Profit for the period 300
Statement of Financial Position
GH₵ million GH₵ million
Non-current assets
Property, plant and equipment (note (i)) 1,100
Current assets
Inventory 500
Trade receivables 720
Bank - 1,220
Total assets 2,320
Equity and liabilities
Stated capital 200
Retained earnings 760
960

Non-current liabilities
8% loan notes 460
Current liabilities
Bank overdraft 20
Trade payables 860
Current tax payable 80 960
Total equity and liabilities 2,320

Below are ratios calculated for the year ended 31 March 2012
Return on year end capital employed
(Profit before interest and tax over total assets less current liabilities) 28.1%
Net- asset (equal to capital employed) turnover 4 times
Gross profit margin 17%
Current ratio 1.6:1
Closing inventory holding period 46days
Trade receivables collection period 45days
Trade payables payment period 55days
Dividend yield 3.75%

Notes
(i) Pink Sheet Ltd received GH₵240million from the sale of plant that had a carrying amount of
GH₵160million at the date of its sale.

(ii) The market price of Pink Sheet Ltd‘s shares throughout the year averaged GH₵3.75 each

536
(iii) There were no issues or redemption of shares or loans during the year

(iv) Dividends paid during the year ended 31 March 2013 amounted to GH₵180million,
maintaining the same dividend paid in the year 31 March 2012.

(v) The stated capital consists of equity shares issued at 25 pesewas per share.

Required:
(a) Calculate ratios for the year ended 31 March 2013 (equivalent to those provided above)
(8 marks)
(b) Analyze the financial performance and position of Pink Sheet Ltd for the year ended 31
March 2013 compared to the previous year.
(7 marks)

Solution:
RATIO FORMULA CALCULATION 2013 2012
ROCE PBIT/TA-TL GHS440m/GHS1,360 m 32.36% 28.1%
Net assets turnover Turnover/Net assets GHS8,000 m/GHS1,360 m 5.9 times 4 times
Current Ratio CA/CL: 1 GHS1,220 M/GHS960 m: 1 1.27: 1 1.6: 1
GP Margin GP/Sales GHS1,100m/GHS8,000 m x100% 13.75% 17 %
Closing Inventory
holding period Av Inventory/Cost of sales X 365 days
GHS500m/GHS6,900 m x365 days 26.4 days 46 days
Trade receivable collection period
Trade receivables/Credit sales x 365 days
GHS720m/GHS6,000m x 365 days 43.8 days 45 days
Trade payables settlement period
Trade payables/Credit purchases x 365days
GHS860m/GHS6,900m x 365 days 45.5 days 55 days
Dividend yield Dividend per share/Mkt price per share x 100%
GHS0.225/GHS3,75 X 100% 6% 3.75%
QUESTION 6
We Are Done Limited, manufactures a variety of consumer products. The company‟s founders
have managed the company for thirty years and are now interested in selling the company and
retiring. Seekers Limited is looking into the acquisition of We Are Done Limited and has
requested the company‟s latest financial statements and selected financial ratios in order to
evaluate We Are Done Limited‟s financial stability and operating efficiency. The summary of
information provided by We Are Done Limited is presented below:

537
We Are Done Limited
Income Statement for the year ended
31st December, 2009
GHS‟000 GHS‟000
Revenues 30,500
Expenses:
Cost of goods sold 17,600
Selling and
administrative expense 3,050

Depreciation and
amortization expense 1,890
Interest expense 9,000 23,440
Income before taxes 7,060
Income taxes 2,900
Net income 4,160

Statement of Financial Position as at


December 31, 2008 and 2009
2009
GHS‟000

2008
GHS‟000
Assets:
Current assets:
Cash 400 500
Marketable securities 500 200
Accounts receivable, net 3,200 2,900
Inventory 5,800 5,400
Total current assets 9,900 9,000
Property, plant and
equipment, net 7,100 7,000
Total assets 17,000 16,000

Liabilities and
Shareholders‟ Equity:
Current liabilities:
Accounts payable 3,700 3,400
Income taxes payable 900 800
Accrued expenses 1,700 1,400
Total current liabilities 6,300 5,600
Long-term debt 2,000 1,800
Total liabilities 8,300 7,400
Ordinary shares 2,700 2,700
Share deals 1,000 1,000

538
Income surplus 5,000 4,900
Total shareholders‟ 8,700 8,600
equity
Total liabilities and 17,000 16,000
shareholders‟ equity

Selected Financial Ratios


We Are Done Limited Current Industry
2008 Average
Current ratio 1.61 1.63
Acid-test ratio 0.64 0.68
Inventory turnover 3.17 3.18
Times interest earned 8.55 8.45
Debt-to-equity ratio 0.86 1.03
Required:
(a) Calculate the above ratios for the year 2009 for We Are Done Limited. (5 marks)

(b) What do these ratios tell you about the company‟s operations and ability to take on
additional debt?
(5 marks)

(c) Identify five (5) limitations of ratio analysis. (5 marks)


(Total: 15 marks)

Solution:
a) Calculations of the financial ratios:
Current ratio = current assets /current liabilities = GHS9,900 / GHS6,300 = 1.57

Acid-test ratio = (cash + marketable securities + net receivables) / current liabilities = (GHS400
+ GHS500 + GHS3,200) /GHS6,300 = 0.65

Inventory turnover = cost of goods sold / average inventory = GHS17,600 / [1/2 (GHS5,800 +
GHS5,400)] = 3.14 times

Times interest earned = income before interest and taxes / interest expenses = (GHS7,060 +
GHS900) +/GHS900 = 8.84 times

Debt-to-equity ratio = total liabilities /stockholders‘ equity = GHS8,300 + GHS8,700 = 0.95

539
b) The analytical use of each of these five ratios:
Current ratio:
- Measures ability to meet short-term obligations using short-term assets.
- We Are Done Limited‘s current ratio has declined slightly over the last three years from 1.62 to
1.57 and the level of the current ratio is a bit below the industry average. This may be cause for
some concern, although the magnitudes are not large.

Acid-test ratio:
- Measures ability to meet short-term obligations using the most liquid assets.
- We Are Done Limited has improved its acid-test ratio over the last three years, but it is still
below the industry average. Furthermore, an acid-test ratio below 1.0 indicates that We Are
Done Limited may have difficulty meeting its short-term obligations.

Inventory turnover:
- Measures how quickly inventory is sold
- We Are done Limited‘s ratio has been steadily declining and is below the industry average.
This may indicate a decline in operating efficiency, obsolete inventory, or a poor marketing
strategy.

Times interest earned:


- Measures the ability to meet interest commitments from current earnings. The higher the
ratio, the more safety there is for long-term creditors.
- We Are Done Limited‘s ratio has been improving over the last two years and is above the
industry average. This indicates that the company has additional capacity to borrow and repay
funds.

Debt-to-equity ratio:
- Measures the level of protection creditors have in the case of possible insolvency. It is also
used to help gauge the company‘s capacity to take on additional debt.
- We Are done Limited‘s debt-to-equity ratio has deteriorated slightly but has been below the
industry average over the last three years. We Are done Limited should be able to raise
additional funds though debt and still remain below the industry average.

c) The difficulties and limitations of ratio analysis include the following:


i. Although ratios are useful as a starting point in financial analysis, they are not an end in
themselves. Ratios can be used as indicators of what to pursue in a more detailed analysis.
ii. Different companies often use different accounting methods (e.g. FIFO versus LIFO inventory
valuation) and this can have an impact on the financial ratios that does not reflect real
differences in the operations and financial health of the companies.
iii. Making comparisons across industries can be difficult. Companies in different industries
tend to have different financial ratios.
iv. Since the ratios are based on accounting statements, they measure what has happened in the
past and not necessarily what will happen in the future.

QUESTION 7

Shown below are the financial statements of Woodbank for its most recent two years:

540
Statements of profit or loss for the year ended 31 March:
2014 2013
GHC’000 GHC’000
Revenue 150,000 110,000
Cost of sales (117,000) (85,800)
–––––––– ––––––––
Gross profit 33,000 24,200
Distribution costs (6,000) (5,000)
Administrative expenses (9,000) (9,200)
Finance costs – loan note interest (1,750) (500)
–––––––– ––––––––
Profit before tax 16,250 9,500
Income tax expense (5,750) (3,000)
–––––––– ––––––––
Profit for the year 10,500 6,500
–––––––– ––––––––

Statements of financial position as at 31 March:


2014 2013
GHC’000 GHC’000
Assets
Non-current assets
Property, plant and equipment 118,000 85,000
Goodwill 30,000 nil
–––––––– ––––––––
148,000 85,000
–––––––– ––––––––
Current assets
Inventory 15,500 12,000
Trade receivables 11,000 8,000
Bank 500 5,000
–––––––– ––––––––
27,000 25,000
–––––––– ––––––––
Total assets 175,000 110,000
–––––––– ––––––––
Equity and liabilities
Equity
Equity shares of GHC1 each 80,000 80,000
Retained earnings 15,000 10,000
–––––––– ––––––––
95,000 90,000
–––––––– ––––––––
Non-current liabilities
10% loan notes 55,000 5,000
–––––––– ––––––––
Current liabilities

541
Trade payables 21,000 13,000
Current tax payable 4,000 2,000
–––––––– ––––––––
25,000 15,000
–––––––– ––––––––
Total equity and liabilities 175,000 110,000
–––––––– ––––––––
The following information is available:
(i) On 1 January 2014, Woodbank purchased the trading assets and operations of Shaw for
GHC50 million and, on the same date, issued additional 10% loan notes to finance the purchase.
Shaw was an unincorporated entity and its results (for three months from 1 January 2014 to 31
March 2014) and net assets (including goodwill not subject to any impairment) are included in
Woodbank‘s financial statements for the year ended 31 March,2014 .There were no other
purchases or sales of non-current assets during the year ended 31 March 2014.
(ii) Extracts of the results (for three months) of the previously separate business of Shaw, which
are included inWoodbank‘s statement of profit or loss for the year ended 31 March 2014, are:

GHC’000
Revenue 30,000
Cost of sales (21,000)
–––––––
Gross profit 9,000
Distribution costs (2,000)
Administrative expenses (2,000)
(iii) The following six ratios have been correctly calculated for Woodbank for the year ended 31
March 2013:
Return on capital employed (ROCE) 10·5%
(profit before interest and tax/year-end total assets less current liabilities)
Net asset (equal to capital employed) turnover 1·16 times
Gross profit margin 22·0%
Profit before interest and tax margin 9·1%
Current ratio 1·7:1
Gearing (debt/(debt + equity)) 5·3%
Required:
(a) Calculate for the year ended 31 March 2014:
(i) equivalent ratios (all six) to the above for Woodbank based on its reported figures; and
(ii) equivalent ratios to the first FOUR only for Woodbank excluding the effects of the
purchase of Shaw.
Note: Assume the capital employed for Shaw is equal to its purchase price of GHC50 million.
(10 marks)
(b) Assess the comparative financial performance and position of Woodbank for the year
ended 31 March 2014.
Your answer should refer to the effects of the purchase of Shaw. (15 marks)
(25 marks)
Solution:
(a) Note: Figures in the calculations of the ratios are in GHCmillion
(i) 2014 (ii) 2014 2013

542
As reported Excluding Shaw From question
Return on (year-end) capital employed 12·0% 18/(175 – 25) 13·0% (18 – 5)/(150 – 50) 10·5%
Net asset turnover 1·0 times 150/150 1·2 times (150 – 30)/100 1·16 times
Gross profit margin 22·0% 33/150 20·0% (33 – 9)/(150 – 30) 22·0%
Profit before loan interest and tax margin 12·0% 18/150 10·8% (18 – 5)/(150 – 30) 9·1%
Current ratio 1·08:1 27/25 1·67:1
Gearing 36·7% 55/(95 + 55) 5·3%
(b) Analysis of the comparative financial performance and position of Woodbank for the
year ended 31 March 2014
Note: References to 2014 and 2013 should be taken as the years ended 31 March 2014 and 2013
respectively.
Introduction
When comparing a company‘s current performance and position with the previous year (or
years), using trend analysis, it is necessary to take into account the effect of any circumstances
which may create an inconsistency in the comparison. In the case of Woodbank, the purchase of
Shaw is an example of such an inconsistency. 2014‘s figures include, for a three-month period,
the operating results of Shaw, and Woodbank‘s statement of financial position includes all of
Shaw‘s net assets (including goodwill) together with the additional 10% loan notes used to
finance the purchase of Shaw. None of these items were included in the 2013 financial
statements. The net assets of Shaw when purchased were GHC50 million, which represents one
third of Woodbank‘s net assets (capital employed) as at 31 March 2014; thus it represents a
major investment for Woodbank and any analysis necessitates careful consideration of its
impact.
Profitability
ROCE is considered by many analysts to be the most important profitability ratio. A ROCE of
12·0% in 2014, compared to 10·5% in 2013, represents a creditable 14·3% (12·0 – 10·5)/10·5)
improvement in profitability. When ROCE is calculated excluding the contribution from Shaw,
at 13·0%, it shows an even more favourable performance. Although this comparison (13·0%
from 10·5%) is valid, it would seem to imply that the purchase of Shaw has had a detrimental
effect on Woodbank‘s ROCE. However, caution is needed when interpreting this information as
ROCE compares the return (profit for a period) to the capital employed (equivalent to net assets
at a single point in time). In the case of Woodbank, the statement of profit or loss only includes
three months‘ results from Shaw whereas the statement of financial position includes all of
Shaw‘s net assets; this is a form of inconsistency. It would be fair to speculate that in future
years, when a full year‘s results from Shaw are reported, the ROCE effect of Shaw will be
favourable. Indeed, assuming a continuation of Shaw‘s current level of performance, profit in a
full year could be GHC20 million. On an investment of GHC50 million, this represents a ROCE
of 40% (based on the initial capital employed) which is much higher than Woodbank‘s pre-
existing business.
The cause of the improvement in ROCE is revealed by consideration of the secondary
profitability ratios: asset turnover and profit margins. For Woodbank this reveals a complicated
picture. Woodbank‘s results, as reported, show that it is the increase in the profit before interest
and tax margin (12·0% from 9·1%) which is responsible for the improvement in ROCE, as the
asset turnover has actually decreased (1·0 times from 1·16 times) and gross profit is exactly the
same in both years (at 22·0%). When the effect of the purchase of Shaw is excluded the position
changes; the overall improvement in ROCE (13·0% from 10·5%) is caused by both an increase in
profit margin (at the before interest and tax level, at 10·8% from 9·1%), despite a fall in gross

543
profit (20·0% from 22·0%) and a very slight improvement in asset turnover (1·2 times from 1·16
times).
Summarising, this means that the purchase of Shaw has improved Woodbank‘s overall profit
margins, but caused a fall in asset turnover. Again, as with the ROCE, this is misleading
because the calculation of asset turnover only includes three months‘ revenue from Shaw, but
all of its net assets; when a full year of Shaw‘s results are reported, asset turnover will be much
improved (assuming its three-months performance is continued).
Liquidity
The company‘s liquidity position, as measured by the current ratio, has fallen considerably in
2014 and is a cause for concern. At 1·67:1 in 2013, it was within the acceptable range (normally
between 1·5:1 and 2·0:1); however, the 2014 ratio of 1·08:1 is very low, indeed it is more like
what would be expected for the quick ratio (acid test). Without needing to calculate the
component ratios of the current ratio (for inventory, receivables and payables), it can be seen
from the statements of financial position that the main causes of the deterioration in the
liquidity position are the reduction in the cash (bank) position and the dramatic increase in
trade payables. The bank balance has fallen by $4·5 million (5,000 – 500) and the trade payables
have increased by $8 million.
An analysis of the movement in the retained earnings shows that Woodbank paid a dividend of
$5·5 million (10,000 + 10,500 – 15,000) or 6·88 cents per share. It could be argued that during a
period of expansion, with demands on cash flow, dividends could be suspended or heavily
curtailed. Had no dividend been paid, the 2014 bank balance would be $6·0 million and the
current ratio would have been 1·3:1 ((27,000 + 5,500):25,000). This would be still on the low side,
but much more reassuring to credit suppliers than the reported ratio of 1·08:1.
Gearing
The company has gone from a position of very modest gearing at 5·3% in 2013 to 36·7% in 2014.
This has largely been caused by the issue of the additional 10% loan notes to finance the
purchase of Shaw. Arguably, it might have been better if some of the finance had been raised
from a share issue, but the level of gearing is still acceptable and the financing cost of 10%
should be more than covered by the prospect of future high returns from Shaw, thus benefiting
shareholders overall.
Conclusion
The overall operating performance of Woodbank has improved during the period (although the
gross profit margin on sales other than those made by Shaw has fallen) and this should be even
more marked next year when a full year‘s results from Shaw will be reported (assuming that
Shaw can maintain its current performance). The changes in the financial position, particularly
liquidity, are less favourable and call into question the current dividend policy. Gearing has
increased substantially, due to the financing of the purchase of Shaw; however, it is still
acceptable and has benefited shareholders. It is interesting to note that of the $50 million
purchase price, $30 million of this is represented by goodwill. Although this may seem high,
Shaw is certainly delivering in terms of generating revenue with good profit margins.

QUESTION 8
a) You are a private consultant for Ashtown Ltd, a listed company in Ghana operating in the
manufacturing sector. Below is a Statement of financial position and a summarized statement of
changes in equity with comparatives for the year ended 31 December 2015.
Statement of Financial Position as at 31 December 2015:
2015 2014

544
GH¢ GH¢
Non-Current Assets
Property, Plant & 200,000 250,000
Equipment
Research & Development 40,000 -
240,000 250,000
Current Assets
Available For Sale 12,000 20,000
Investments
Inventory 70,000 45,000
Debtors 30,000 23,000
Cash and Cash equivalents 18,000 17,000
130,000 105,000
370,000 355,000
Equity
Equity Share Capital (GH¢1 150,000 100,000
per Share)
10% Preference Share 50,000 50,000
Capital
Retained Earnings 60,000 55,000
260,000 205,000
Non-Current Liabilities
Interest-bearing borrowing 50,000 90,000
Deferred Tax 5,000 3,000
55,000 93,000

Current Liabilities
Trade & Other Payables 30,000 20,000
Bank Overdraft 25,000 37,000
55,000 57,000
370,000 355,000

Summarised Statement of Changes in Equity


2015 2014
GH¢ GH¢
Opening Balance 205,000 165,000
Profit for the period 65,000 45,000
Less Dividends 10,000 5,000
260,000 205,000

Additional Information:
2015 2014
GH¢ GH¢
Sales 365,000 219,000
Cost of Sales 219,000 156,000
Income Tax Expense 17,000 8,000
Finance Cost 9,000 6,000

545
Industry Ratios
Current Ratio: 2.0 times
Debt to Equity Ratio: 50%
Gross Profit Margin: 32%
Net Profit Margin: 20%
Return on Capital Employed: 22%
Required:
Prepare a report and address to the Chief Executive Officer, analyzing the financial
performance and financial position of Ashtown Ltd based on the industry ratios above for the
years 2014 and 2015.
(12 marks)
b) Krofrom Ltd is also a listed company operating in the manufacturing sector in Ghana. The
non-current asset turnover ratio of Krofrom Ltd for the year 2015 is 1.3. Compute the non-
current asset turnover ratio for Ashtown Ltd and explain TWO reasons, why these ratios may
not provide a good comparison of the efficiency of the entities.
(3 marks)
(Total: 15 marks)

Solution:
APPENDIX
i) Current Ratio= Currents Assets/Current Liability
ii) Debt to Equity= Debt/Equity*100
iii) Gross Profit Margin= Gross Profit/Sales * 100
iv) Net Profit Margin= Net Profit/Sales * 100
v) Return on Capital Employed= Profit before Interest & Tax/Capital Employed*100
2015 2014 Industry
Current Ratio GHS130,000/GHS55,000 GHS105,000/GHS57,000 2 times
2.36 times 1.84 times
Debt/Equity GHC50,000/GHC260,000x100 GHS90,000/GHS205,000 50%
Ratio x 100%
19.2% 43.9%
Gross Profit (GHS365,000-GHS219,000)/ (GHS219,000- 32%
Margin GHS365000 x 100% GHS156,000)/
GHS219000 x 100 %
40% 28.77%
Net Profit Margin GHS65,000/GHS365,000 x GHS45,000/GHS219,000 20%
100% x 100%
17.8% 20.54%
Return on Capital (GHS65,000+GHS17,000+ (GHS45,000+GHS8,000+ 22%
Employed GHS9,000)/(GHS370,000- GHS6,000)/(GHS355,000-
GHS55,000) GHS57,000)
28.88% 19.80%

a) REPORT
To: Chief Executive Officer
From: Consultant

546
Date: 31/12/2015
Subject: Financial Performance and Financial Position of Ashtown Ltd for the year ending
2015.
As requested, I have analyzed the financial performance and financial position of Ashtown Ltd.
My analysis is based on the Statement of financial Position, summarised statement of changes in
equity and the additional information given. A number of key measures have been calculated
and these are set out in the attached Appendix.
Financial Performance
Financial performance is a quantitative measure of how well a firm can use assets from its
primary mode of business and generate revenues using the financial statements. This can be
measured using Gross Profit Margin, Net Profit Margin and Return on Capital Employed
according to the Industry measures.
Gross profit margin measures Ashtown Ltd's manufacturing and distribution efficiency during
the production process. Gross profit margin has increased considerably (from 28.77% to 40%)
during the year and this is great increment compared to the industry average of 32%. This High
gross profit margin increment indicates that the company can make a reasonable profit, as long
as it keeps the overhead cost in control. The company can improve this by cutting down cost of
manufacturing and increasing sales.
Net profit margin measures company's profitability or how much of each Ghana Cedi earned by
the company is translated into net profits. Net profit margin is an indicator of how efficient a
company is and how well it controls its costs. The higher the margin is, the more effective the
company is in converting revenue into actual profit. The company did well in the previous year
(20.5% compared to the industry average of 20%) doing slightly better than the industry
average. Though Ashtown Ltd‘s Gross Profit increased considerably, there was a marginal fall
in net profits in the current year (17.8%). The company should adopt cost reduction techniques
to cut down the operational, administrative and selling expenses so as to increase profitability.
The Return on Capital Employed ratio helps assess how efficiently a company employs its
available capital and is particularly helpful when comparing profitability across companies in
the same business with similar amounts of working capital. From the ratios for every GHS1
employed, Ashtown gained GHS0.28 in 2015. Ashtown Ltd‘s ROCE ratio is relatively high, (an
improvement from 19.8% to 28.88%) that is commonly interpreted as an indication that the
company is making more efficient use of its capital. The Company can improve the ROCE ratio
in subsequent years by selling off of unprofitable or unnecessary assets this can be evidenced
from disposing part of the available for sale instrument in the financial statements.
Financial Position
The status of the assets, liabilities, and owners' equity (and their interrelationships) of an
organization, as reflected in its financial statements is the financial position. The financial
position helps users to understand the gearing, liquidity and value of the company. This can be
measured using Debt to Equity ratio for gearing, and Current Ratio for liquidity looking at the
industry averages given.
Gearing
The debt to equity ratio measures the ratio of a company‘s interest bearing debt finance to its
equity finance (shareholders‘ funds). The Debt to Equity ratio indicates how much debt a
company is using to finance its assets relative to the amount of value represented in
shareholders' equity. This decreased substantially from 43.9% to 19.2% and still lower than the
industry ratio. A high debt/equity ratio generally means that a company has been aggressive in
financing its growth with debt. Aggressive leveraging practices are often associated with high

547
levels of risk. This may result in volatile earnings as a result of the additional interest expense. It
can be evidenced that Ashtown Ltd is not highly geared and it can improve on this by retaining
substantial portions of the earnings generated internally.
Liquidity
Liquidity ratios are a class of financial metrics used to determine a company's ability to pay off
its short-terms debts obligations. Generally, the higher the value of the ratio, the larger the
margin of safety that the company possesses to cover short-term debts. The current ratio is a
liquidity ratio that measures whether or not a firm has enough resources to meet its short-term
and long-term obligations. Ashtown Ltd has improved from (1.84:1 to 2.36:1) more than the
industry average of 2:1. A high current ratio may not always be a green signal. Ashtown Ltd
with high current ratio of 2.36 may not always be able to pay its current liabilities as they
become due because of a large portion of its current assets consists of slow moving (available
for sale instruments and inventories) or obsolete inventories.
Conclusion
Ashtown Ltd‘s profit margins appear to be reasonable for a company in its industry sector.
Although it‘s net profit margin is below the industry average. It is possible at least some of this
improvement can be achieved by deliberately reducing the operating expenses for the year.
There are no apparent gearing or short-term liquidity problems but Ashtown Ltd should find
solution of reducing its slow moving short term assets.
b) Ashtown Ltd (2015)
Non-Current Asset Ratio = Sales/Non-Current Assets
= GHS365,000/GHS240,000 = 1.52 times.
Krofrom Ltd= 1.3times Ashtown Ltd =1.52 times
Reasons:
 The non-current assets of one entity could be nearing the end of their useful life and
therefore be unrealistically low giving a higher non-current asset turnover figure.
 Alternatively, the non-current assets of one entity could have been revalued which
would result in asset turnover being low but not necessarily because of low efficiency.

QUESTION 9
Glory Inc is a publicly listed company which has experienced rapid growth in recent years
through the acquisition and integration of other companies. Glory Inc is interested in acquiring
Rita Inc, a retailing company, which is one of several companies owned and managed by the
same family. The summarised financial statements of Rita Inc for the year ended 30 September
2014 are:

Statement of profit or loss


GHS‘000
Revenue 70,000
Cost of sales (45,000)
–––––––
Gross profit 25,000
Operating costs (7,000)
Directors‘ salaries (1,000)
–––––––
Profit before tax 17,000

548
Income tax expense (3,000)
–––––––
Profit for the year 14,000
–––––––
Statement of financial position
GHS’000 GHS’000
Assets
Non-current assets
Property, plant and equipment 32,400
Current assets
Inventory 7,500
Bank 100
7,600
–––––––
Total assets 40,000
–––––––
Equity and liabilities
Equity
Equity shares of GHS1 each 1,000
Retained earnings 18,700
–––––––
19,700
Non-current liabilities
Directors‘ loan accounts (interest free) 10,000
Current liabilities
Trade payables 7,500
Current tax payable 2,800
10,300
–––––––
Total equity and liabilities 40,000
–––––––
From the above financial statements, Glory Inc has calculated for Rita Inc the ratios below for
the year ended 30 September 2014. It has also obtained the equivalent ratios for the retail sector
average which can be taken to represent Rita Inc‘s sector.
Rita Inc Sector average
Return on equity (ROE) (including directors‘ loan accounts) 47·1% 22·0%
Net asset turnover 2·36 times 1·67 times
Gross profit margin 35·7% 30·0%
Net profit margin 20·0% 12·0%

From enquiries made, Glory Inc has learned the following information:
(i) Rita Inc buys all of its trading inventory from another of the family companies at a price
which is 10% less than the market price for such goods.
(ii) After the acquisition, Glory Inc would replace the existing board of directors and need to
pay remuneration of GHS2·5 million per annum.
(iii) The directors‘ loan accounts would be repaid by obtaining a loan of the same amount with
interest at 10% per annum.

549
(iv) Glory Inc expects the purchase price of Rita Inc to be GHS30 million.
Required:
(a) Recalculate the ratios for Rita Inc after making appropriate adjustments to the financial
statements for notes (i) to (iv) above. For this purpose, the expected purchase price of GHS30
million should be taken as Rita Inc’s equity and net assets are equal to this equity plus the
loan. You may assume the changes will have no effect on taxation. (6 marks)
(b) In relation to the ratios calculated in (a) above, and the ratios for Rita Inc given in the
question, comment on the performance of Rita Inc compared to its retail sector average.
(9 marks)
(15 marks)

Solution:
(a) For comparison
Rita Rita
adjusted as reported Sector average
Return on equity (ROE) 21·7% 47·1% 22·0%
Net asset turnover 1·75 times 2·36 times 1·67 times
Gross profit margin 28·6% 35·7% 30·0%
Net profit margin 9·3% 20·0% 12·0%
Rita‘s adjusted ratios:
On the assumption that after the purchase of Rita, the favourable effects of the transactions with
other companies owned by the family would not occur, the following adjustments to the
statement of profit or loss should be made:
GH’000
Cost of sales (45,000/0·9) 50,000
Directors‘ remuneration 2,500
Loan interest (10% x 10,000) 1,000

These adjustments would give a revised statement of profit or loss:


Revenue 70,000
Cost of sales (50,000)
–––––––
Gross profit 20,000
Operating costs (7,000)
Directors‘ remuneration (2,500)
Loan interest (1,000)
–––––––
Profit before tax 9,500
Income tax expense (3,000)
–––––––
Profit for the year 6,500
–––––––
In the statement of financial position:
Equity would be the purchase price of Rita (per question) 30,000
The commercial loan (replacing the directors‘ loan) would now be debt 10,000
From these figures the adjusted ratios above are calculated as:
Return on equity ((6,500 /30,000) x 100) 21·7%

550
Net asset turnover (70,000/(30,000 + 10,000)) 1·75 times
Gross profit margin ((20,000)/70,000) x 100) 28·6%
Net profit margin ((6,500/70,000) x 100) 9·30%

(b) An analysis of Rita‘s ratios based on the financial statements provided reveals a strong
position, particularly in relation to profitability when compared to other businesses in this retail
sector. Rita has a very high ROE which is a product of higher-than-average profit margins (at
both the gross and net profit level) and a significantly higher net asset turnover. Thus,on the
face of it, Rita is managing to achieve higher prices (or reduced cost of sales), has better control
of overheads and is using its net assets more efficiently in terms of generating revenue.
However, when adjustments are made for the effects of its favourable transactions with other
companies owned by the family,the position changes somewhat. The effect of purchasing its
inventory from another family owned supplier at favourable market prices means that its
reported gross profit percentage of 35·7% is flattered; had these purchases been made at market
prices, it would fall to 28·6% which is below the sector average of 30·0%. The effects of the
favourable inventory purchases carry through to net profit. Based on Glory‘s estimate of future
directors‘ remuneration, it would seem the existing directors
of Rita are not charging commercial rates for their remuneration. When Glory replaces the
board of Rita, it will have to increase directors‘ remuneration by GHC1·5 million. Additionally,
when the interest free directors‘ loans are replaced with a commercial loan, with interest at 10%
per annum, this would reduce net profit by a further GHC1 million. The accumulation of these
adjustments means that the ROE which Glory should expect would be 21·7% (rather than the
reported 47·1%) which is almost exactly in line with the sector average of 22·0%.

QUESTION 10
Shown below are the recently issued (summarized) financial statements of Boadu Ltd, a
company listed on the Ghana Stock Exchange, for the year ended 31 December 2015, together
with comparatives for 2014 and extracts from the Chief Executive‘s report that accompanied
their issue.
Statement of Profit or Loss for the year ended 31 December
2015 2014
GHS‘000 GHS‘000
Revenue 250,000 180,000
Cost of sales (200,000) (150,000)
Gross profit 50,000 30,000
Operating expenses (26,000) (22,000)
Finance costs (8,000) (nil)
–––––––– ––––––––
Profit before tax 16,000 8,000
Income tax expense (at 25%) (4,000) (2,000)
–––––––– ––––––––
Profit for the period 12,000 6,000
–––––––– ––––––––
Statement of Financial Position
Non-current assets
Property, plant and equipment 210,000 90,000

551
Goodwill 10,000 nil
–––––––– ––––––––
220,000 90,000
` –––––––– ––––––––
Current assets
Inventory 25,000 15,000
Trade receivables 13,000 8,000
Bank nil 14,000
38,000 37,000
Total assets 258,000 127,000
–––––––– ––––––––
Equity and liabilities
Equity shares of GHS1 each 100,000 100,000
Retained earnings 14,000 12,000
114,000 112,000
Non-current liabilities
8% loan notes 100,000 nil

Current liabilities
Bank overdraft 17,000 nil
Trade payables 23,000 13,000
Current tax payable 4,000 2,000
–––––––– ––––––––
44,000 15,000
–––––––– ––––––––
Total equity and liabilities 258,000 127,000

Extracts from the Chief Executive’s report:


‗Highlights of Boadu‘s performance for the year ended 31 December 2015:
 an increase in sales revenue of 39%
 gross profit margin up from 16·7% to 20%
 a doubling of the profit for the period.
In response to the improved position, the Board paid a dividend of 10 pesewas per share in
December 2015; an increase of 100% on the previous year.

You have also been provided with the following further information:
On 1 January 2015, Boadu purchased the whole of the net assets of Ayeboafo (previously a
privately owned entity) for GHS100 million. The contribution of the purchase to Boadu‘s results
for the year ended 31 December 2015 was
GHS‘000
Revenue 70,000
Cost of sales (40,000)
–––––––
Gross profit 30,000
Operating expenses (8,000)
–––––––

552
Profit before tax 22,000
–––––––
There were no disposals of non-current assets during the year.
The following ratios have been calculated for Boadu Ltd for the year ended 31 December 2014:

i) Return on year-end capital employed 7·1%


(profit before interest and tax over total assets less current liabilities)
ii) Net asset turnover 1·6
iii) Net profit (before tax) margin 4·4%
iv) Current ratio 2·5
v) Closing inventory holding period (in days) 37
vi) Trade receivables‘ collection period (in days) 16
vii) Trade payables‘ payment period (based on cost of sales) (in days) 32
viii) Earnings per share GHS0.06 (6 Gp)
ix) Dividend pay-out ratio 83.3%
x) Gearing (debt over debt plus equity) nil

Required:
(a) Calculate ratios for Boadu Ltd for the year ended 31 December 2015 equivalent to those
calculated for the year ended 31 December 2014 (showing your workings).
(Note: Ayeboafo’s capital employed is to taken as its profit after tax using the same tax rate of
25%)
(b) Assess the financial performance and position of Boadu for the year ended 31 December
2015 compared to the previous year. Your answer should refer to the information in the Chief
Executive‘s report and the impact of the purchase of the net assets of Ayeboafo on the
profitability of Boadu.

Solution:
(a) Ratios
Including Excluding
Ayeboafo Ayeboafo
ROCE 16,000 +8,000 x 100
114,000+ 100,000 11.2%

24,000- 22,000 x 100


114,000- (22,000 - 5,500) 2.05%

Net asset turnover 250,000


114,000 +100,000 1.17%

250,000- 70,000
114,000- (22,000- 5,500) 1.85

Net profit margin 16,000 x 100


250,000 6.4%
24,000- 22,000 x 100
250,000- 70,000 1.1%

553
Current ratio 38,000:44,000 0.86:1

Closing inventory holding period 25,000 x 365


200,000 46 days

Trade receivables collection period 13,000 x 365


250,000 19 days

Payables payment period 23,000 x 365


200,000 42 days
Gearing 100,000
100,000 +114,000 46.7%
(b) It is clear that the acquisition of Ayeboafo has had a very positive impact on Boadu's results
for the year ended 31 Dec 20015. For this reason it is instructive to look at the 2015 ratios which
have been affected by the acquisition and see what they would have been without the addition
of Ayeboafo's results.

Profitability
It is immediately apparent that without the purchase of Ayeboafo the Chief Executive's report
would have looked very different. The increase in sales revenue of 39% would have
disappeared. The sales revenue of Boadu is static. The increase in gross profit margin from
16.7% to 20% would have been a fall to 11.1%.
The profit for the period would not have doubled. It would have gone from GHC8m profit
before tax in 2014 to a GHC2m profit before tax in 2015, assuming that the loan note interest
would not have arisen. This would have given an ROCE of 2.05% for 2015 rather than the 11.2%
when Ayeboafo is included. If we break ROCE down into net profit% and asset turnover, we
can see that Ayeboafo's results have increased the net profit% by almost six times, while having
an adverse effect on the asset turnover due to the GHC100m funding through loan notes. There
is some distortion in the 2015 figures arising from interest charges which are not deducted in
calculating ROCE but have been deducted in arriving at net profit.

Liquidity
While it has greatly enhanced Boadu's profitability, the purchase of ayeboafo has done little for
liquidity, an aspect not touched on in the extract from the Chief Executive's report.Boadu
borrowed GHC100m to pay for Ayeboafo, so the purchase was not funded from working
capital. However, it has paid GHC8m loan note interest,increased its inventory holding by
GHC10m, invested in additional property, plant and equipment and paid a GHC10m dividend.
In this way it has, despite the increased profit, converted a positive cash balance of GHC14m to
an overdraft of GHC17m. The ratios show this very clearly. Boadu's current ratio has declined
from 2.5:1 to 0.86:1 and its quick ratio (not shown above) has declined from 1.47:1 to 0.30:1,
casting some doubt upon whether it will be able to continue to meet its commitments as they
fall due.
The increase in the inventory holding period is worrying, as it suggests that Boadu may have
inventory which is slow-moving, and the increase in the payables period by 10 days suggests
problems paying suppliers. Boadu has a GHC4m tax bill outstanding. If this is not paid on time
it will incur interest, which will further weaken the cash position.

554
Gearing
The cost of acquiring Ayebaofo is directly reflected in the gearing ratio, which has gone from nil
in 2014 to 46.7% in 2015, with the issue of the loan notes. This will reduce profits available for
distribution to shareholders in the future and if Boadu's cash position does not improve it may
be forced to seek further loans. In the light of this, the increase of 25% in the dividend is hard to
justify.

QUESTION 11
Below are the financial ratios for the year 2014 for Samaria Limited, a company engaged in the
buying and shipment of shea butter products. The ratios for the industry have also been
provided.
Samaria Limited Industry Average
Quick ratio 0.52:1 0.84:1
Current ratio 1.20:1 1.80:1
Debtors collection period 46 days 41 days
Creditors payment period 70 days 50 days
Inventory holding period 58 days 48 days
Dividend yield 3.6% 9%
Debt to equity 85% 45%
Dividend cover 1.4 times 3.4 times
Gross profit margin 18% 28%
Net profit margin 8% 12.8%
Return on capital employed 28% 14%
Net assets turnover 4.2 times 1.9 times
Required:
Provide an assessment of Samaria Limited in comparsion with the industry in respect of
profitability, liquidity, efficiency and shareholders‘ investment.
Solution:
Samaria Limited
i) Profitability
Both gross profit margin and net profit margin have fallen below the industry average. This
may be the result of uncontrolled overhead cost on the presence of large obsolete equipment.
Unless steps are taken quickly to improve the income account investors may shift their interest
into more profitable companies in the industry.

ii) Liquidity
Both quick and current ratios fall far below those of the industry as confirmed by the fact that it
takes longer to collect its debt than the industry. The problem may have been worsened by the
fact that inventory stay longer at Samaria Ltd than in the industry which is not the best use to
which resources should be put.
iii) Efficiency
Even though Samaria Ltd takes longer to collect its debts (5 days), this is compensated by an
even longer time to settle the debts (20 days). This means that creditors provide free finance for
its operations. However, the holding on to inventory for a long time (10 days) cannot be
justified. This should be turned over in like manner as that assets turnover far exceeds that of
the market.
iv) Shareholders’ Investment

555
Both dividend cover and dividend yield fall below that of the industry. Such a situation is not
likely to excite investors especially income – oriented investors who are seeking to recoup their
investment in the shortest time possible. This poor performance may be due to the fact that
high gearing ratio of Samaria Ltd has effectively put its future in the hands of debenture holders
who are reaping the bulk profits in the form of interest charges.

QUESTION 12
Hukpor Ltd (Hukpor) manufactures a variety of consumer products. The company‘s founders
have managed the company for thirty years and are now interested in selling the company and
retiring. Seekers Ltd is looking into the acquisition of Hukpor and has requested the company‘s
latest financial statements and selected financial ratios in order to evaluate Hukpor‘s financial
stability and operating efficiency. The summary of information provided by Hukpor is
presented below:
Hukpor Ltd
Statement of Profit or Loss for the year ended 31 December
2018 2017
GH¢'000 GH¢'000
Revenue 30,500 30,290
Cost of Sales (17,600) (18,900)
Gross Profit 12,900 11,390
Operating expenses (4,940) (4,550)
Profit before interest and taxes 7,960 6,840
Interest expense (900) (800)
Profit before tax 7,060 6,040
Taxation (2,900) (2,400)
Profit after tax 4,160 3,640

Statements of Financial Position as at 31 December 2018 2017


GH¢'000 GH¢'000
Assets
Non-Current assets
Property, plant and equipment 7,100 7,000
Current Assets
Inventories 5,800 5,400
Accounts receivable 3,200 2,900
Marketable securities 500 200
Cash 400 500
Total current assets 9,900 9,000
Total assets 17,000 16,000
Liabilities and Shareholders' Equity
Current liabilities:
Accounts payable 3,700 3,400
Income taxes payable 900 800
Accrued expenses 1,700 1,400
Total current liabilities 6,300 5,600
Long-term debt 2,000 1,800
Total liabilities 8,300 7,400

556
Stated Capital 3,700 3,700
Retained Earnings 5,000 4,900
Total shareholders' equity 8,700 8,600
Total liabilities and shareholders' equity 17,000 16,000

Selected Financial Ratios of Hukpor Ltd for 2017


Current ratio 1.61:1
Acid-test ratio 0.64:1
Inventory turnover 3.17 times
Times interest earned 8.55 times
Debt-to-equity ratio 86%
Required:
a) Calculate ratios for the years 2018 for Hukpor in comparison with ratios for 2017.
(5 marks)
b) For each of the ratios computed for 2018, analyse Hukpor‘s performance for 2018 based on
the results of the ratio computed, in comparison with the results for 2017. (10 marks)
c) Explain FIVE (5) limitations of accounting ratios. (5 marks)

solution
c) Limitations of Ratio Analysis

themselves. Ratios can be used as indicators of what to pursue in a more detailed analysis.
FIFO versus LIFO inventory
valuation) and this can have an impact on the financial ratios that does not reflect real
differences in the operations and financial health of the companies.
n different industries tend
to have different financial ratios.

past and not necessarily what will happen in the future.


ratio analysis in the context of the general business
environment. For example, 60 days of sales outstanding for receivables might be considered
poor in a period of rapidly growing sales, but might be excellent during an economic
contraction when customers are in severe financial condition and unable to pay their bills.

example, a current ratio of 2:1 might appear to be excellent, until you realize that the company
just sold a large amount of its stock to bolster its cash position. A more detailed analysis might
reveal that the current ratio will only temporarily be at that level, and will probably decline in
the near future.

firms that are pursuing different strategies. For example, one company may be following a low-
cost strategy, and so is willing to accept a lower gross margin in exchange for more market
share. Conversely, a company in the same industry is focusing on a high customer service
strategy where its prices are higher and gross margins are higher, but it will never attain the
revenue levels of the first company. (1 mark for each limitation well explained x 5 limitations =
5 marks)

557
QUESTION 13
Adenta Ltd (Adenta) assembles telecommunication equipment and sells to wholesalers and
retailers. The following ratios relate to the average figures for Adenta‘s industry for the year
ended 31 December, 2018:
Return on capital employed 20.10%
Gross profit margin 32%
Net profit (before tax) margin 12.50%
Current ratio 1.6:1
Acid-test ratio 0.9:1
Inventory turnover period 46 days
Trade receivable collection period 45 days
Debt-to-equity ratio 40%
Dividend yield 6%
Dividend cover 3 times

Adenta’s financial statements for the year to 31 December, 2018 are set out below:

Statement of Profit or Loss for the year ended 31 December, 2018


GH¢‘000
Sales 48,500
Cost of sales (37,400)
Gross profit 11,100
Operating expenses (6,880)
Operating profit 4,220
Finance cost (500)
Profit before taxation 3,720
Taxation (1,800)
Profit after taxation 1,920

Extracts of Statement of Changes in Equity for the year ended 31 December, 2018
GH¢‘000
Retained earnings as at 1 January 2018 3,580
Net profit after tax 1,920 5,500 Dividends (1,800)
Retained earnings as at 31 December 2018 3,700

Statement of Financial Position as at 31 December 2018


GH¢‘000
Non-current assets
Property, plant and equipment 10,800
Current Assets Inventories 5,500
Trade receivables 6,400
11,900
Total assets 22,700

Equity Stated capital (Ordinary shares issued @ GH¢0.25 per share) 3,000
Retained earnings 3,700
6,700

558
Non-current Liabilities
8% Debenture 6,000
Current liabilities
Bank overdraft 1,300
Trade payables 7,000
Taxation 1,700
10,000
Total equity and liabilities 22,700
(Note: The market price of Adenta‘s shares throughout the year averaged GH¢6.00 each.)

Required: a) Calculate the ratios for Adenta Ltd equivalent to the industry averages.
(10 marks)
b) As the Financial Controller of Adenta Ltd, write a report to the Board of Directors analysing
the financial performance of Adenta Ltd based on a comparison with the industry averages.
(10 marks)

SOLUTION
B).
b) Report to the Board of Directors
To: The Board
From: Financial Controller
Date: 1 November, 2019
Subject: Analysis of Adenta Limited‘s financial performance compared to industry average for
the year to 31 December, 2018.
Introduction
This report present an analysis of the financial performance of Adenta Ltd compared to the
industry averages.
Profitability
The return on capital employed of Adenta is impressive being higher than the industry average.
The company is employing its assets more efficiently and effectively in generating more
revenue and hence income. However, gross profit margin and net profit margin of Adenta are
comparatively lower than the industry averages. This implies that the company is incurring
more costs in generating its revenue.
Liquidity
Adenta Ltd‘s current and quick ratios are much worse than the industry average, and indeed
far below expected norms. Current liquidity problems appear due to high levels of trade
payables and a high bank overdraft. The high level of inventories constitutes to the poor acid
test ratio and may be indicative of further obsolete inventories. The trade receivables‘ collection
figure is reasonable compared to the industry average.
Gearing
Adenta Ltd‘s gearing is more than twice the level of the industry average. The company is
making an overall return of 33.23% but only paying 8% interest on its loans notes. The gearing
level may become a serious issue if Adenta becomes unable to maintain the finance costs. The
company already has an overdraft and the ability to make further interest payments could be in
doubt.

559
Investment Ratios
Despite reasonable profitability figures, Adenta‘s dividend yield is poor compared to the sector
average. From the extracts of the changes in equity it can be seen that total dividends are
GH¢1.8 million out of available profit for the year of only GH¢1.92 million, hence the very low
dividend cover, compared to the industry average.

Conclusion
The company compares favourably with the industry average figures for profitability, however,
the company‘s liquidity and gearing position is quite poor and gives cause for concern.
Signed
Financial Controll

QUESTION 14
As a newly employed Accountant of Spring Ltd, you have been presented with the following
financial statements:

Statement of Profit or Loss for the year ended 31, December


2018 2017
GH¢ GH¢
Net Turnover 456,500 420,000
Cost of Sales (295,000) (227,000)
Gross Profit 161,500 193,000
General, Administrative and Selling Expenses (109,500) (93,000)
Operating Profit 52,000 100,000
Interest Expenses (14,500) (3,000)
Investment Income 5,000 4,500
Profit before Tax 42,500 101,500
Corporate Tax (12,500) (15,000)
Profit for the year 30,000 86,500

Statement of Retained Earnings for the year ended 31 December


2018 2017
GH¢ GH¢
Balance b/f 149,500 89,500
Net Profit for the year 30,000 86,500
179,500 176,000
Dividends: Preference Shares (10,000) (9,000)
Ordinary Shares (12,000) (17,500)
Balance c/f 157,500 149,500

560
Statement of Financial Position as at 31 December
2018 2017
GH¢ GH¢
Assets
Goodwill 10,000 5,000
Tangible Non-current 106,000 132,000
Inventories 147,000 118,500
Accounts Receivable 80,000 24,000
Cash and Bank balances 26,000 28,500
369,000 308,000
Liabilities and Capital
Accounts Payable 37,500 26,500
Accruals 25,500 20,000
Debentures 58,500 37,000
Share Capital
Preference Shares 40,000 25,000
Ordinary Shares 50,000 50,000
Retained Earnings 157,500 149,500
369,000 308,000

Required: a) Compute, for the two years, the following ratios:


i) Return on Capital Employed
ii) Assets Turnover
iii) Current Ratio
iv) Quick Ratio
v) Debt/Equity Ratio
vi) Interest Cover (6 marks)

b) Write a report to the Finance Director commenting on the financial performance and position
of the company. (10 marks)
c) Earnings per share is a financial ratio, and it's usually the first ratio investors look at when
analyzing a company. Although earnings per share is a very popular performance
measurement tool, it is not without its drawback and limitations.

Required: Identify FOUR (4) limitations of Earnings per share. (4 marks)


(Total: 20 marks

SOLUTIONS
SPRING LTD Formula 2018 2017
i) ROCE =
𝐸𝐵𝐼𝑇/𝐶𝑎𝑝𝑖𝑡𝑎𝑙 𝐸𝑚𝑝𝑙𝑜𝑦𝑒𝑑 × 100 52,000/306,000 × 100= 17% 100,000/261,500 = 38.24%
ii) Assets Turnover =
𝑆𝑎𝑙𝑒/𝐶𝑎𝑝𝑖𝑡𝑎𝑙 𝐸𝑚𝑝𝑙𝑜𝑦𝑒𝑑
456,500/306,000=1.49 times 420,000/261,500 = 1.61 times

561
iii) Current Ratio
𝐶𝑢𝑟𝑟𝑒𝑛𝑡 𝐴𝑠𝑠𝑒𝑡 /𝐶𝑢𝑟𝑟𝑒𝑛𝑡 𝐿𝑖𝑎𝑏𝑖𝑙𝑖𝑡𝑦
253,000/63,000 = 4.02:1 171,000/46,500= 3.68:1
iv) Quick Ratio
𝐶𝑢𝑟𝑟𝑒𝑛𝑡 𝐴𝑠𝑠𝑒𝑡 − 𝐼𝑛𝑣𝑒𝑛𝑡𝑜𝑟𝑖𝑒𝑠
𝐶𝑢𝑟𝑟𝑒𝑛𝑡 𝐿𝑖𝑎𝑏𝑖𝑙𝑖𝑡𝑖𝑒𝑠
(253,000 − 147,000)/63,000= 1.68:1 (171,000 − 118,500)/46,500= 1.13:1

v) Debt/Equity Ratio
𝐷𝑒𝑏𝑡 /𝐸𝑞𝑢𝑖𝑡𝑦 × 100
98,500 x 100 /207,500 = 47.47% 62,000 /199,500 × 100 = 31%
vi) Interest Cover
𝐸𝐵𝐼𝑇 /𝐼𝑛𝑡𝑒𝑟𝑒𝑠𝑡
52,000/14,500= 3.59 times 100,000/ 3,00 = 33.33 times

Report
To: Financial Director
From: Accountant
Date: 1 November 2019
Subject: Financial Performance and Position of Peace Ltd

Following the discussions on the above subject, I wish to submit this report for your study and
consideration.

Financial Performance
The financial performance of the entity is measured in terms of its profitability in relation to
resources deployed during the period. Over the two years, the profit level has declined, using
return on capital as a measure. ROCE compares the returns earned for a period to the net assets
or capital employed used in generating that returns. There was a decline in ROCE from 38.24%
for 2017 to 17% in 2018. This shows that management was not able to utilize assets more
efficiently to generate higher returns. Assets turnover which is the revenue per cedi of long-
term capital used declined marginally from 1.61 times in 2017 to 1.49 times in 2018. This means
that management ability to regenerate revenue for every one cedi of capital used declined in
2018.

Financial Position
The financial position of the firm is measured in terms of the short-term liquidity and long-term
solvency position of the firm. In terms of liquidity, there has been an improvement in the
company‘s ability to settle its current liabilities from its current assets from 3.68:1 (2017) to
4.02:1 (2018). Over the period the company has more equity than prior charged capital. But the
proportion of capital to prior charged capital has increased from 31% (2017) to 47.47% (2018).

Conclusion
Generally, the company has experienced declining trends in its performance with the exception
of liquidity.

Signed Accountant (10 marks)

562
c) Limitations of Ratio Earnings Per Share

they‘ll naturally want the EPS figure to appear as high as possible in the short term. They may
make decisions to maximise the EPS figure in the short term, which may damage the entity‘s
prospects in the long term.
ncreasing the
earnings figure, they start selling to bad customers who don‘t pay or sell at lower margins. If
the company can‘t earn cash to pay its bills, no matter how large the earnings are, it may be
insolvent.
of goods and services generally may be increasing, so this
could be contributing to the good EPS figure, but this growth might be misleading if the
company can‘t buy as many goods this year as it could last year.
ing policies; this makes it harder to compare
individual companies on a like for like basis.
(1 mark for each limitation well explained x 4 limitations = 4 marks) (Total: 20 marks)

EXAMS QUESTIONS
Question 1
In 2012, the management team of Sawaleh Ltd, a manufacturer of motorcycle parts, acquired the
company from its parent company in a management buyout deal. The managers of the company
are considering the possibility of listing on the Ghana Stock Exchange. The following financial
statements relates to the company:
Sawaleh Ltd
Statement of financial position as at GH¢000 GH¢000
June 30 2016 Non-current assets
Land and buildings 3,600
Plant and machinery 9,900
13,500
Current assets:
Inventories 4,400
Trade receivables 4,700
Cash in hand and at bank 1,000 10,100
23,600
Equity and Liabilities
Ordinary share capital issued at Gh¢1 each
Voting 1,800
“A” Shares (Non-voting) 900
Retained earnings 9,700
Shareholders‟ funds 12,400
Non-Current Liabilities
12% Debenture loan (2018) 2,200
Current Liabilities
Bank Overdraft 2000
Trade payables 7,000

563
23,600

Statement of profit or loss for the year ended June 30 2016


Gh¢000
Revenue 36,500
Cost of sales (31,600)
Profit before interest and taxes 4,900
Interest (1,300)
Profit before taxes 3,600
Taxation expense (500)
Profit attributable to ordinary shareholders 3,100
Dividends (300)
Retained profit 2,800

The industry performance average ratios in which Sawaleh Ltd operates are stated below
Industry sector ratios
Price/earnings ratio 10.0
Interest cover 4.5
Dividend cover 4.0
Total debt: equity 24%
Quick (acid test) ratio 1.0:1
Current ratio 1.6:1
Operating profit as percentage of sales 11%
Return after tax on equity 16%
Return before interest and tax on long-term capital employed 24%

Required:
As a newly qualified accountant working with Sawaleh Limited, you are to write a memo to the
CEO of the company evaluating the financial position and performance of Sawaleh Limited by
comparing it with that of its industry sector given above.

Question 2
Below are the financial ratios for the year 2015 for Decimal Limited, a company engaged in the
buying and shipment of agricultural products. The ratios for the industry have also been provided.
Decimal Industry
Limited Average
Quick ratio 0.52:1 0.84:1
Current ratio 1.20:1 1.80:1
Debtors collection period 46 days 41 days
Creditors payment period 70 days 50 days
Inventory holding period 58 days 48 days
Dividend yield 3.6% 9%
Debt to equity 85% 45%
Dividend cover 1.4 times 3.4 times
Gross profit margin 18% 28%
Net profit margin 8% 12.8%

564
Return on capital employed 28% 14%
Net assets turnover 4.2 times 1.9 times

Required:
Write a report to the Shareholders of Decimal Ltd assessing its performance in comparison with
the industry in respect of profitability, liquidity, efficiency and shareholders‟ investment. (10
marks)

QUESTION 3
Mion Ltd is a listed company in Ghana and operates many super markets in Ghana. During the
year 2014, there was speculation in the financial press that the entity was likely to be a takeover
target for larger companies in Ghana. A recent newspaper publication has suggested that the
directors are unlikely to resist a takeover. The seven member board are all nearing retirement and
all own significant minority shareholdings in the business.
You have been approached by a private shareholder in Mion Ltd. She is concerned that the
directors have conflict of interests and that financial statements for 2014 may have been
manipulated. The income statement and summarized statement of changes in equity of Mion
together with comparatives for the year ended 31st December 2014 and a statement of financial
position as at that date are given below:

INCOME STATEMENT FOR THE YEAR ENDED 31ST DECEMBER 2014


2014 2013
GH¢m GH¢m
Revenue 1,255 1,220
Cost of sales (1,177) (1,145)
Gross profit 78 75
Operating expenses (21) (29)
Profit from operations 57 46
Finance cost (10) (10)
Profit before tax 47 36
Income tax (14) (13)
Net profit 33 23 SUMMARISED STATEMENT OF CHANGES IN EQUITY
2014 2013
GH¢m GH¢m
Opening balance 276 261
Profit for the period 33 23
Dividends 8 8
Closing balance 301 276
STATEMENT OF FINANCIAL POSITION AS AT 31ST DECEMBER 2014
2014 2013
NON-CURRENT ASSETS GH¢m GH¢m
Property, Plant& Equipment 580 575
Goodwill 100 100
680 675
CURRENT ASSET
Inventory 47 46

565
Receivables 12 13
Cash 46 12
105 71
TOTAL 785 746
EQUITY
Share Capital 150 150
Retained Earnings 151 126
301 276
NON-CURRENT LIABILITIES
Interest-bearing 142 140
borrowing
Deferred Tax 25 21
167 161
CURRENT LIABILITIES
Trade & other payables 297 273
short-term borrowing 20 36
317 309
785 746

The following additional information is relevant:


i) Non-current asset turnover (including both tangible and intangible non-current asset): 1.93
ii) Mion Ltd‟s directors have undertaken a reassessment of useful lives of non-current tangible
assets during the year. In most cases they estimate that the useful lives have increased and the
depreciation charges in 2014 have been adjusted accordingly.
iii) Six new stores have been opened during 2014, bringing the total to 42.
iv) Three key ratios for the supermarket sector (based on the latest available financial statement of
12 listed entities in the sector) are as follows:
¢27.6m

Required:
a) Prepare a report and address to the investor, analyzing the performance and position of Mion
Ltd based on the financial statements and supplementary information provided above. The report
should also include comparisons with key sector ratios, and it should address the investor‟s
concerns about the possible manipulation of the 2014 financial statements. (15 marks)

QUESTION 4
Ahomka Ltd is a public listed manufacturing company. Its summarised financial
statements for the year ended 30 April 2017 (and 2016 comparatives) are as follows:
Statements of Comprehensive Income for the year ended 30 April: 2016
2017
GH¢000
GH¢000
Revenue 310,000 360,000
Cost of sales (270,000) (260,000)

566
Gross profit 40,000 100,000
Distribution costs (10,000) (8,000)
Administrative expenses (49,000) (39,000)
Investment income - 2,000
Finance costs (6,000) (5,000)
Profit (loss) before taxation (25,000) 50,000
Income tax (expense) relief 4,000 (15,000)
Profit (loss) for the year (21,000) 35,000
Other comprehensive income (loss) for the year (45,000) -
Total comprehensive income (loss) for the year (66,000) 35,000

Statements of Financial Position as at 30 April: 2016


2017
Assets GH¢000 GH¢000
Non-current assets
Property, plant and equipment 176,000 245,000
Investments at fair value through profit or loss 24,000 40,000
285,000
200,000
Current assets
Inventory and work in progress 22,000 19,000
Trade receivables 22,000 28,000
Tax refund due 6,000 -
Bank 12,000 1,000
48,000
62,000
Total assets 262,000 333,000

Equity and liabilities


Equity
Equity shares of GH¢1 each 130,000 120,000
Share deals account 10,000 -
Revaluation reserve - 45,000
Retained earnings 36,000 65,000
230,000
176,000
Non-current liabilities
Bank loan 40,000 50,000
Long term provisions 12,000 7,000
57,000
52,000
Current liabilities
Trade payables 34,000
46,000
Total equity and 262,000
liabilities 333,000

567
The following additional information is available:
i) There were no additions to, or disposals of, non-current assets during the year.
ii) In order to help cash flows, the company made a rights issue of shares during the year
ending 30 April 2017, all of which ranked for dividend. No shares were issued during the
year ended 30 April 2016.
iii) The dividend per share has been reduced by 50% for year ended 30 April 2017.

Required
Analyse and discuss the financial performance and financial position of Ahomka Ltd
as portrayed in the financial statements and in the additional information provided. Your
analysis should be supported by relevant ratios.

Uses of accounting ratios (Any 2 points x 1 marks = 2 marks):


i. Ratios are used to express the logical relationships which exist between certain items in the
financial statements.
ii. Ratios are also used to assess the liquidity position and financial condition of an enterprise over
a period as well as with other similar enterprises.
iii. Ratios provide a better means of comparison than absolute values of financial or operational
measurement.
iv. Ratios are useful in assessing the efficiency in managing the entity‟s assets.
v. Ratios provide an indication of whether it is safe to invest in an entity.
vi. It is also useful in summarising the information contained in financial statements and in
directing the user‟s attention to certain key areas which may vary from firm to firm.
vii. Trend analysis, horizontal analysis and vertical analysis are all undertaken using ratios.

Limitations of the use of accounting ratios in appraising financial performance. (3 points x 1


mark = 3 marks)
1. Inconsistent definitions of ratios
2. Financial statements may have been deliberately manipulated (creative accounting)
3. Different companies may adopt different accounting policies (e.g. use of historical costs
compared to current values)
4. Different managerial policies (e.g. different companies offer customers different payment
terms)
5. Statement of financial position figures may not be representative of average values throughout
the year (this can be caused by seasonal trading or a large acquisition of non-current assets near
the year-end)
6. The impact of price changes over time/distortion caused by inflation.

Limitations of inter-period and inter-company comparisons


ght need to be adjusted to for inflation if valid comparisons are to be made over
time.

choice of depreciation and stock valuation policies


e up to a different date which can significantly affect ratios if the business
is seasonal

568
assessing liquidity and the capacity to adapt by reference to cash flow statement

possible to practice window dressing and this can affect ratios.


average. It would
be helpful to have quartile and decile figures.

QUESTION 5
It is often argued that historical cash flow is more useful in appraising a company than historical
profit, particularly because cash flows are factual and do not involve the exercise of judgement. 

Required:
Explain FOUR arguments against this view. . (4 marks) 


b) FordlandLtdandFiatlandLtdaretwocompaniesintheGarmentIndustry.Thefollowing are financial


ratios computed by the Research Department of ICAG as part of analyzing companies‟
performance industry by industry. 

Ratios of companies for the year ended 31 December 2017
Fordland Ltd Fiatland Ltd
Return on capital employed 24.10% 30%
Net assets turnover
 1.9times 2.5times
Gross profit margin
 35% 20%
Net profit (before tax) margin 10.50% 38%
Current ratio 1.0:1 2.0:1
Quick ratio
 0.8:1 1.0:1
Inventory holding period 60days 90days
Receivables collection period 58days 60days
Payables payment period 50days 50days
Debt to equity
 50% 30%
Dividend yield
 3% 2%
Dividend cover 2 times 1.5 times

Required
a) Explain Three problems that are inherent when ratios are used to compare the performance of two
companies even in the same industry.
b) Write a report analyzing and comparing the financial performance of Fordland Ltd and Fiatland Ltd. The
report should cover the Operating Performance, Liquidity, Gearing and Investment Ratio.
(8 marks)

SOUTIONS
a) Although it is true that cash flows are factual and not affected by accounting policies, there are
a number of areas in which it could be argued that profit is a better measure of performance:
Profit is accrual based. This has the following implications: – Management can 
manipulate
cash flows to appear better than they are by delaying payments and bringing forward receipts; this
is irrelevant to profits. – Certain items relating to the future are recognised in profit but not in cash
flows. For example a provision for a liability is recognised in profit or loss so providing indicators
of future cash flows. 

Non-cash items may be represented in profit or loss, but are obviously not represented in the
statement of cash flows. An example is share-based payments. Although these do not involve the

569
payment of cash, they are still relevant to performance appraisal. 

Due to the way in which cash flows are disclosed, normalisation for exceptional and one-off
items is difficult to achieve. In the statement of profit or loss these items are disclosed and so can
be excluded in order to assess sustainable performance more accurately. 

Management can time the purchase and sale of non-current assets in order to improve cash
flow performance in a given year. The equivalent expense is depreciation; as this is recognised
over the life of the underlying asset, there is limited opportunity to take a similar short-term
approach to improving performance. 
( Any 4 points for 4 marks) 


b) Of particular concern with this method of using ratios is: 



Many large firms operate different divisions in different industries. For these 
companies it is
difficult to find a meaningful set of industry-average ratios. 

Inflation may have badly distorted a company's balance sheet. In this case, profits will also be
affected. Thus a ratio analysis of one company over time or a comparative analysis of companies
of different ages must be interpreted with 
judgment. 

Seasonal factors can also distort ratio analysis. Understanding seasonal factors that affect a
business can reduce the chance of misinterpretation. For example, a retailer's inventory may be
high in the summer in preparation for the back-to- school season. As a result, the company's
accounts payable will be high and its ROA low. 

Different accounting policies and practices can distort comparisons even within the same
company (leasing versus buying equipment, LIFO versus FIFO, etc.). 

It is difficult to generalize about whether a ratio is good or not. A high cash ratio 
in a
historically classified growth company may be interpreted as a good sign, but could also be seen
as a sign that the company is no longer a growth company and should command lower valuations.
A company may have some good and some bad ratios, making it difficult to tell if it's a good or
weak company. 


c)
Report Format
From:

To: Research Department of ICAG

Date: 8 May 2018

Subject: Analysis of Fordland Ltd & Fiatland Ltd
Introduction
Operating Performance
Operating performance ratios are tools which measure the function of certain 
core operations
for an organization or business. Particularly, these ratios reveal information about how efficiently
that organization is using resources to generate sales and cash. A company with strong
performance ratios is able to utilize a minimum resource pool to generate high levels of sales, as
well as a significant cash inflow.
Using the Gross Profit Margin, Fordland is doing better than Fiatland. This means that
Fordland is able to control its cost of production and at the same time maximise its revenue or
sales. 

Using the Net profit margin Fiatland (38%) is doing extremely well compared to Fordland
(10.50%). This means that Fiatland is able to control its operating expenses compared to Fordland.

570
Fiatland is more efficient in controlling its expenses. 

Fiatland's ROCE of 30% is higher than Fordland's 24.10% ROCE. This means that Fiatland
does a better job of deploying its capital than Fordland. A higher ROCE indicates more efficient
use of capital. ROCE should be higher than the company‟s capital cost; otherwise it indicates that
the company is not employing its capital effectively and is not generating shareholder value.
Liquidity
Liquidity ratios analyse the ability of a company to pay off both its current liabilities as they
become due as well as their long-term liabilities as they become current. In other words, these
ratios show the cash levels of a company and the ability to turn other assets into cash to pay off
liabilities and other current obligations. 

The current ratio is mainly used to give an idea of a company's ability to pay back its liabilities
(debt and accounts payable) with its assets (cash, marketable securities, inventory, and accounts
receivable). As such, current ratio can be used to make a rough estimate of a company‟s financial
health. For every GH¢1 of current debt, Fordland has GH¢1 available to pay for the debt. Fiatland
has GH¢2 in current assets for each GH¢1 of current debt. Fiatland has more than enough to cover
its current liabilities if they come due. 

The quick ratio measures the cedi amount of liquid assets available for each GH¢1 of current
liabilities. Thus, a quick ratio of 0.8:1 for Fordland means that a company has GH¢0.80 of liquid
assets available to cover each GH¢1 of current liabilities and Fiatland has GH¢1.0 of liquid assets
available to cover each GH¢1 of current liabilities. 
 (2 marks) 

Gearing 

Gearing ratio refers to the fundamental analysis ratio of a company's level of long-term debt
compared to its equity capital/capital employed. The point when processing what amount of debt
an organization is undertaking as contrasted with its equity, the debt to equity ratio is generally
utilized. 

Generally, a lower gearing ratio means greater financial stability. However, not all debt is bad
debt. Loans and other fixed interest liabilities are a way for companies to leverage their value to
increase profits for shareholders, so the optimal gearing ratio is largely determined by the
individual company relative to others in its sector. Therefore Fordland with a debt to equity ratio
of 50% is riskier than Fiatland with 30%. 
 (2 marks) 

Investment Ratio 

Dividend yield is a way to measure how much cash flow you are getting for each cedi invested
in an equity position. In other words, it measures how much cedi you are getting from dividends.
In the absence of any capital gains, the dividend yield is effectively the return on investment for a
stock. Assuming all other factors are equivalent, then, an investor looking to use his or her
portfolio to supplement his or her income would likely prefer Fordland's stock over that of
Fiatland, as it has 3% the dividend yield compared to 2% of Fiatland. 

Dividend cover is the ratio of company's net income over the dividend paid to shareholders,
calculated as earnings per share divided by the dividend per share. 

It helps indicate how sustainable a dividend is. Dividend Cover of less than 1.5 may indicate a
danger of a dividend cut while more than 2 is viewed as healthy. Investors will prefer Fordland
with a dividend cover of 2 as compared to Fiatland with a dividend cover of 1.5
Signature

QUESTION 7

571
Adenta Ltd (Adenta) assembles telecommunication equipment and sells to wholesalers and
retailers. The following ratios relate to the average figures for Adenta‘s industry for the year
ended 31 December, 2018:
Return on capital employed 20.10%
Gross profit margin 32%
Net profit (before tax) margin 12.50%
Current ratio 1.6:1
Acid-test ratio 0.9:1
Inventory turnover period 46 days
Trade receivable collection period 45 days
Debt-to-equity ratio 40%
Dividend yield 6%
Dividend cover 3 times

Adenta’s financial statements for the year to 31 December, 2018 are set out below:

Statement of Profit or Loss for the year ended 31 December, 2018


GH¢‘000
Sales 48,500
Cost of sales (37,400)
Gross profit 11,100
Operating expenses (6,880)
Operating profit 4,220
Finance cost (500)
Profit before taxation 3,720
Taxation (1,800)
Profit after taxation 1,920

Extracts of Statement of Changes in Equity for the year ended 31 December, 2018
GH¢‘000
Retained earnings as at 1 January 2018 3,580
Net profit after tax 1,920 5,500 Dividends (1,800)
Retained earnings as at 31 December 2018 3,700

Statement of Financial Position as at 31 December 2018


GH¢‘000
Non-current assets
Property, plant and equipment 10,800
Current Assets Inventories 5,500
Trade receivables 6,400
11,900
Total assets 22,700

Equity Stated capital (Ordinary shares issued @ GH¢0.25 per share) 3,000
Retained earnings 3,700
6,700
Non-current Liabilities

572
8% Debenture 6,000
Current liabilities
Bank overdraft 1,300
Trade payables 7,000
Taxation 1,700
10,000
Total equity and liabilities 22,700
(Note: The market price of Adenta‘s shares throughout the year averaged GH¢6.00 each.)

Required: a) Calculate the ratios for Adenta Ltd equivalent to the industry averages.
(10 marks)
b) As the Financial Controller of Adenta Ltd, write a report to the Board of Directors analysing
the financial performance of Adenta Ltd based on a comparison with the industry averages.
(10 marks)

SOLUTION
B).
b) Report to the Board of Directors
To: The Board
From: Financial Controller
Date: 1 November, 2019
Subject: Analysis of Adenta Limited‘s financial performance compared to industry average for
the year to 31 December, 2018.
Introduction
This report present an analysis of the financial performance of Adenta Ltd compared to the
industry averages.
Profitability
The return on capital employed of Adenta is impressive being higher than the industry average.
The company is employing its assets more efficiently and effectively in generating more revenue
and hence income. However, gross profit margin and net profit margin of Adenta are
comparatively lower than the industry averages. This implies that the company is incurring more
costs in generating its revenue.
Liquidity
Adenta Ltd‘s current and quick ratios are much worse than the industry average, and indeed far
below expected norms. Current liquidity problems appear due to high levels of trade payables
and a high bank overdraft. The high level of inventories constitutes to the poor acid test ratio and
may be indicative of further obsolete inventories. The trade receivables‘ collection figure is
reasonable compared to the industry average.
Gearing
Adenta Ltd‘s gearing is more than twice the level of the industry average. The company is
making an overall return of 33.23% but only paying 8% interest on its loans notes. The gearing
level may become a serious issue if Adenta becomes unable to maintain the finance costs. The
company already has an overdraft and the ability to make further interest payments could be in
doubt.
Investment Ratios
Despite reasonable profitability figures, Adenta‘s dividend yield is poor compared to the sector
average. From the extracts of the changes in equity it can be seen that total dividends are GH¢1.8

573
million out of available profit for the year of only GH¢1.92 million, hence the very low dividend
cover, compared to the industry average.

Conclusion
The company compares favourably with the industry average figures for profitability, however,
the company‘s liquidity and gearing position is quite poor and gives cause for concern.
Signed
Financial Controll

QUESTION 7
The Gandi Group (TGG) operates in the farming industry and has operated a number of 100%
owned subsidiaries for many years. The Gandi group has its operations in the Brong-Ahafo
Region of Ghana. Its financial statements for the last two years are shown below.
Consolidated statements of profit or loss for the year ended 30 September.
2018 2019
GH¢000 GH¢000
Revenue 47,000 34,250
Cost of Sales
 (23,000) (14,000)
Gross profit
 24,000 20,250
Distributions costs (10,600) (9,650)
Administrative expenses (12,800) (7,700)
Profits from operations 600 2,900
Investment income - 300
Finance costs (60) -
Profit before tax
 540 3,200
Taxation (150) (960)
Profit for the year 390 2,240
Attributable to:
Shareholders of Gandi 790 2,240
Non-controlling interest (400) -
390 2,240

Consolidated statement of financial position (extract) as at 30 September


2018 2019
GH¢000 GH¢000
Current assets
Inventory 3,250 2,285
Trade receivables 8,500 7,800
Bank 305 3,000
Equity

Share capital
 12,500 3,000
Retained earnings
 36,750 36,250
Non – controlling interest
 255
Non-current liabilities

Loan 10,000

574
Additional information:
i) TGG has become increasingly worried about two major areas in its business environment. First of all, there
are concerns that reliance on large supermarkets are putting pressure on cash flow, as the supermarkets
demand long payment terms. Secondly, the consistent increases in fuel prices mean that delivering the
produce nationally is becoming extremely expensive.
In order to deal with the above worrying concerns, TGG purchased 80% of Asida Ltd on 1
October 2017. This was the first time TGG had purchased a subsidiary without owning 100% of it.
Asida Ltd operates two luxury hotels in the Ashanti Region of Ghana, and Gandi Ltd purchased
Asida Ltd with a view to diversification and to provide a long term solution to the cash flow
concerns raised above. 

ii) TGG raised finance from multiple sources to finance its activities as it did not have ready
finance. Part of this finance came from the disposal of GH¢5.5 million held in investments,
making a GH¢2.25 million gain on disposal, which is included in administrative expenses. 

iii) Asida Ltd opened a third hotel which is located in Accra in March 2018, its largest yet. After
poor initial reviews, Asida Ltd appointed a new marketing director in May 2018. As a result of an
extensive marketing campaign, online feedback had improved. 

iv) The following ratios have been calculated for the year ended 30 September 2017: 

Gross profit margin 59.1%

Operating margin
 8.5%

Return on capital employed 7.4%

Inventory turnover period 60days

Receivables collection period 83days

Required:
a) Prepare the equivalent ratios for the year ended 30 September 2018. 5 marks)
b) Analyse the financial performance and cash flow of TGG for the year ended 30 September
2018, making specific reference to any concerns or expectations regarding future periods.
(10 marks)

solu

b) Analysis of the performance and cash flow of the Gandi group Performance
Revenue and expenses have all increased during the year. Some of this will be due to the
acquisition of Asida Ltd in the year, but without Asida‟s individual financial statements it is
difficult to see how much this has contributed. Asida Ltd has been controlled since 1 October 2017
so has contributed full year‟s results for the year to September 2018 which would not have been
included in the year to 30th September 2017.
It is also important to recognize that the new Hotel was only opened in March 2018, so a full
year‟s revenue has not yet been generated by this. Whereas revenue has increased significantly by
37.2%, it can be seen that higher expenses have meant that the Gandi Ltd group has made less
profit than in the previous period. For example, cost of sales increased astronomically by 64.3%
from the previous year.
The gross profit margin has fallen in the year from 59.1% to 51.1%. This

575
could be attributable to the increased pressure on prices from supermarkets or difficult trading
conditions, but could also be as a result of the addition of Asida Ltd into the group.
It may be that the hotel industry generates much lower margins than the farming industry, or it
may be that Asida Ltd itself generates margins. It may also be that the poor reputation associated
with the new hotel resulted in the need to offer lower room rates to attract new customers or to
bring customers back. Following the improvement in feedback as indicated in the question, this
will hopefully improve profits in future periods. The improvement in feedback should lead to
increased future bookings, so it may be that the new hotel generates a significantly improved
return in future years.
The operating margin has also deteriorated in the year. It is also important to note that there is a
significant one-off GH¢ 2.25 million gain on disposal in relation to the sale of investments during
the year. Without this exceptional item, Gandi Ltd group would have made a worrying loss from
operations of GH¢1.65 million (2,250 – 600). Further analysis of this loss, the reason for this loss
is due to a significant increase in the administrative expenses, which would be GH¢ 30.1 million
excluding the GH¢ 2.25 million profit on disposal. Therefore, the Gandi Ltd group would have
incurred costs relating to the acquisition of Asida Ltd, which will not be incurred in future periods.

There will have been significant expenses associated with the set-up of the new hotel during the
year, which again are unlikely to be repeated in future periods unless Asida Ltd plans on opening
another hotel in the near future. The increased marketing expenses will also have had a significant
impact on the profit, though it is unclear whether this is something that may need to be repeated or
maintained in future periods.
There will be many costs associated with the new hotel which will be recurring, such as the
increase in staff numbers, and the running costs of the hotel. The new marketing director is likely
to have demanded a significant salary, as he or she was brought in specifically to address a
problem with the hotel‟s reputation.
The distribution costs have increased marginal by 9.8% during the year. This could suggest that
there has been a decline in the underlying farming business, as fuel costs have risen significantly.
It could also mean that the hotel business has extremely low distribution costs, which is likely.

Whereas the Gandi Ltd group has managed to reduce its exposure to fuel prices through the
purchase of Asida Ltd not dependent on distribution, it is questionable how wise this has been.
Asida Ltd appears to have lower margins and will still incur significant costs in terms of cooling
and lighting, which may rise in a similar manner to fuel costs.
The return on capital employed has also deteriorated in the year from 7.4% to 1%, which is
expected due to the reduced operating profits. Removing the gain on disposal of the investments
would make the Gandi group loss-making, meaning that it would make a negative return on
capital employed.
It thus appears that Asida Ltd is a loss-making entity, as the non-controlling share of the group‟s
profit is negative. As Asida Ltd is the only non-100% controlled subsidiary, the non-controlling
interest will relate solely to Asida‟s performance.
This may also suggest that there is goodwill
impairment following the purchase of Asida Ltd. This could have arisen from the poor reputation
associated with the new hotel. If the non-controlling interest is valued at fair value, 20% of the
impairment would be allocated to the non-controlling interest‟s share.
In addition to the reduced operating profits, the capital employed by the Gandi Ltd group has
increased significantly. A total of GH¢9.5 million of new shares have been issued during the year,

576
probably to fund the purchase of Asida Ltd. There has also been a GH¢10 million long-term loan
contracted, although it is possible that Asida Ltd already had this loan and this has resulted from
the consolidation of Asida‟s assets and liabilities.
The sale of investments and share issue and potentially the contracted loan suggest that the Gandi
Ltd group paid a very high price for Asida Ltd. This may not have been wise if Asida Ltd has low
margins, or is loss making. If Asida Ltd owns the hotel premises, then this may have explained the
high price as the land is likely to hold its value, even if Asida performs poorly as demonstrated.

Cash flow position


The cash balance has fallen to GH¢ 0.3 million as at 30 September 2018. It is important to note
that the Gandi Ltd group has raised significant funds during the year through a share issue, sale of
investments and the potential receipt of a long-term loan. This means that significant amount of
the cash raised is likely to have been spent on the purchase of Asida Ltd and the new hotel.
However, the decrease in receivable collection period will have a positive effect on the cash flow
of the Gandi Ltd group. As a hotel will largely be cash based rather than offering credit, this will
aid the cash flow of the Gandi Ltd group. This means that the purchase of Asida Ltd may help
offset its problems caused in the farming sector with the longer terms demanded by the
supermarkets.
Another positive sign on cash flow is the reduction in the inventory turnover period. This will be
as a result of the fact that Asida Ltd will not carry much inventory, as this is likely to relate to food
and drink served in the hotels.
Conclusion
The success of the Gandi Ltd group in the current year is difficult to judge; it has been a
transitional year. There are concerns over the performance of Asida Ltd, although there are
reasons to believe this may improve in the instant future periods. For a more meaningful
comparison, individual financial statements of companies within the Gandi Ltd group would need
to be accessed.

QUESTION 7
Below are the recently issued financial statements of Madina Ltd, a listed company, for the year
ended 30 September 2018, together with comparatives for 2017.

Statements of profit or loss for the year ended 30 September.


2018 2017
GH¢000 GH¢000
Revenue 125,000 90,000
Cost of Sales (100,000) (75,000)
Gross profit 25,000 15,000
Operating expenses (13,000) (11,000)
Finance costs (4,000) -
Profits before tax 8,000 4,000
Tax (at 25%) (2,000) (1,000)
Profit for the year 6,000 3,000

577
Statements of financial position 2018 2017
as at 30 September Assets
Non-Current Assets: GH¢000 GH¢000
Property, Plant and Equipment 105,000 45,000
Goodwill 5,000 -
110,000 45,000
Current assets
Inventory 12,500 7,500
Receivables 6,500 4,000
Bank - 7,000
19,000 18,500
129,000 63,500
Equity
Share Capital 50,000 50,000
Retained Earnings 7,000 6,000
57,000 56,000
Non-current liabilities
8% Loan Notes 50,000 -
Current liabilities
Bank Overdraft 8,500 -
Trade Payables 11,500 6,500
Current Tax Payable 2,000 1,000
22,000 7,500
129,000 63,500

Additional Information:
i) On 1 October 2017 Madina Ltd acquired 100% of the net assets of Aboabu Ltd for GH¢50
million. In order to finance this transaction, Madina Ltd issued GH¢50 million 8% loan notes on
the acquisition date.

ii) Aboabu Ltd‟s results for the year ended 30 September 2018 is shown below:
Statements of profit or loss for the year ended 30 September. GH¢000
Revenue 35,000
Cost of sales (20,000)
Gross profit 15,000
Operating expenses (4,000)
Profit before tax 11,000
Tax (at 25%) (2,750)
8,250

iii) The following ratios have been calculated for Madina Ltd for the year ended 30 September 2017:
Return on capital employed 7.1%
Gross profit margin 16.7%
Net profit (before tax) margin 4.4%

578
Required:
a) Calculate the equivalent ratios for Madina Ltd for 2018:
i) Including the results of Aboabu Ltd acquired during the year. (3 marks)
ii) Excluding all effects of the purchase of Aboabu Ltd. (3 marks)
b) Analyse the performance of Madina Ltd for the year ended 30 September 2018. (5
marks)
c) Analyse the cash position of Madina Ltd as at 30 September 2018. (4 marks)

Note: Capital employed will be made up of share capital and retained earnings, as no loan
notes will exist without the purchase of Aboabu Ltd. Retained earnings without Aboabu Ltd
will actually be GH¢1.75 million. This can be calculated in two ways:

Closing retained earnings of GH¢7 million less GH¢8.25 million from Aboabu Ltd‟s profit,
plus GH¢3 million increase in profit after tax relating to the interest on the loan notes (GH¢4
million interest saved less GH¢3 million tax relief at 25%).

An alternative calculation of retained earnings would be GH¢6 million in 2017 plus GH¢0.75
million from Madina Ltd‟s profit excluding Aboabu Ltd less GH¢5 million dividend (GH¢0.05
per share), which would also give GH¢1.75 million.

A final alternative calculation of retained earnings would be closing retained earnings of


GH¢7 million less the original profit of GH¢6 million plus the GH¢0.75 million profit
excluding Aboabu Ltd, to give GH¢1.75 million.

b) Performance of Madina Ltd


of the Madina Ltd group increased by GH¢35 million. However,
it can be seen that all of the GH¢35 million increase came from the acquisition of Aboabu Ltd,
meaning that the underlying business in Madina Ltd was poor during the year.

that Aboabu Ltd was a good acquisition in terms of generating revenue, it


also highlights a poor performance by Madina Ltd. It appears that Madina Ltd have increased
property, plant and equipment in addition to the purchase of Aboabu Ltd as this has increased in
the year by GH¢60 million. As Aboabu Ltd was purchased for GH¢50 million, it is unlikely that
all of the increase in these asset comes from Aboabu Ltd.

to Aboabu Ltd‟s acquisition. It can be seen that Aboabu Ltd makes a gross margin of 42.9%
(GH¢15 million/GH¢35 million) which has generated 11.2%. This is further indication of both
the strength of Aboabu Ltd and the poor underlying performance of Madina Ltd.

Ltd‟s profit before tax is GH¢11 million and the group profit before tax is only GH¢8 million.
This suggests that Madina Ltd would not have incurred the GH¢ 4 million finance costs without

579
the Aboabu Ltd purchase, meaning that Madina has made a small profit before tax of GH¢ 1
million.

t profit margin and the return on capital employed, the


performance of the Madina group is an improvement on the prior year. Further investigation
shows that without Aboabu Ltd this would again be below the previous year, highlighting a
decline.

nstant improvement in the measures relating to performance shows that Aboabu Ltd has
been a successful acquisition and that Madina Ltd‟s profits without it have significantly declined.
It can also be seen that there appears to be some cost savings within operating expenses. These
have only increased by GH¢2.

million in the year, but Aboabu Ltd‟s operating expenses are GH¢4 million. This suggests that
the group may have benefited from being able to share administrative or head office functions
and reduce expenses.
(Any 5 points for 5 marks)

c) Cash Position of Madina Ltd

2018, going from a cash position of GH¢7 million to being overdrawn by GH¢8.5 million. While
Aboubu Ltd has been acquired for GH¢50 million, this has been funded through the issue of
GH¢ 50 million loan notes, so this cannot be the reason for the decline in cash during the year.
There may have been some additional costs linked to the acquisition, but this is unlikely to make
up a significant part of the cash movement.

There are two major reasons for this decrease:

shareholders happy, the prudence of this could be questioned, especially in the light of the
declining underlying performance in Madina Ltd.
lso been an increase in non-current assets beyond the acquisition of
Aboabu Ltd, as these have increased by GH¢60 million after depreciation. More information will
be required on where these additions have taken place. If the assets have been made within
Aboabu Ltd, this would seem to be a wise move due to the high level of Aboabu Ltd profits. If
assets have been acquired for the Madina Ltd business, it would raise concerns as it appears that
these assets have not been turned into profits during the year.

noted that Aboabu Ltd generates enough profits to cover this payment.

Madina Ltd and Aboabu Ltd do, it is difficult to make judgments on inventory or receivables
management, but both balances have increased in line with the increased performance from
Aboabu Ltd. It is possible that Aboabu Ltd may have had an overdraft when acquired, as
information on Aboabu‟s individual position has not been given.

580
profit generation, as this has generated a larger portion of profits for the year. This has disguised
the fact that the underlying Madina Ltd business seems to have struggled with declining profits.
Careful attention must be paid to the position, as the cash is a major concern. The idea of the
dividend could be called into question. It may be that Madina Ltd has finished the expansion in
terms of acquiring non-current assets which could relieve some pressure on the future cash flow.

QUESTION 8
You are the Financial Controller of Oxtom Ltd. Pep Ltd is a competitor in the same industry and
it has been operating for 20 years. Summaries of Pep Ltd‟s Statements of Profit or Loss and
Financial Position for the previous three years are given below.
Pep Ltd
Summarised Statement of Profit or Loss for the year ended 31 December
2016 2017 2018
GH¢‟m GH¢‟m GH¢‟m
Revenue 840 981 913
Cost of sales (554) (645) (590)
Gross profit 286 336 323
Selling, distribution and administration expenses (186) (214) (219)
Profit before interest and taxes 100 122 104
Finance cost (6) (15) (19)
Profit before taxation 94 107 85
Taxation (45) (52) (45)
Profit after taxation 49 55 40
Dividends 24 24 24

Pep Ltd
Summarised Statements of Financial Position as at 31 December
2016 2017 2018
GH¢‟m GH¢‟m GH¢‟m
Assets
Non-current assets
Intangible assets 36 40 48
Tangible assets at net book value 176 206 216
212 246 264
Current assets
Inventories 237 303 294
Receivables 105 141 160
Bank 52 58 52
394 502 506
Total Assets 606 748 770

Equity and Liabilities

581
Equity
Stated capital 100 100 100
Retained earnings 299 330 346
399 430 446
Non-current liabilities
Long-term loans 74 138 138

Current liabilities
Trade payables 53 75 75
Other payables 80 105 111
133 180 186
Total Equity and Liabilities 606 748 770

Required: Write a report to the Chief Executive Officer of Oxtom Ltd,


a) Analysing the performance of Pep Ltd and showing any calculations in an appendix to this
report.
(14
marks)

b) Summarising THREE (3) areas which require further investigation, including reference to
other pieces of information which would complement your analysis of the performance of
Pep Ltd.
(6
marks)

To: Chief Executive Officer


From: Financial Controller
Subject: Performance Analysis of Water Limited from 2016 to 2018
Date: 4th November 2019

Introduction
This performance report relates to the financial statements of Water Limited for the period 2016
to 2018. The report covers the profitability, operating efficiency, liquidity and solvency positions
of Water Limited over the period under review. An appendix is attached to this report which
shows the ratios calculated as part of the performance review.

Profitability
The gross profit margin has remained relatively static over the three year period, although it has
risen by approximately 1% in 2018. ROCE, while improving very slightly in 2017 to 21.5% has
dropped dramatically in 2018 to 17.8%. The net profit margin has also fallen in 2018, in spite of
the improvement in the gross profit margin. This marks a rise in expenses which suggests that
they are not being well controlled. The utilization of assets compared to the turnover generated
has also declined reflecting the drop in trading activity between 2017 and 2018.

Operating Efficiency

582
It is apparent that there was a dramatic increase in trading activity between 2017 and 2018, but
then a significant fall in 2018. Revenue rose by 17% in 2017 but fell by 7% in 2018. The reasons
for this fluctuation are unclear. It may be the effect of some kind of one-off event, or it may be
the effect of a change in product mix. Whatever the reason, it appears that improved credit terms
granted to customers (receivables payment period up from 46 to 64 days) has not stopped the
drop in sales.

Liquidity
Both the current ratio and quick ratio demonstrate an adequate working capital situation,
although the quick ratio has shown a slight decline. There has been an increased investment over
the period in inventories and receivables, which has been only partly financed by longer payment
periods to trade payables and a rise in other payables (mainly between 2016 and 2017).

Solvency/Gearing
The level of gearing of the company increased when a further GH¢64m was raised in long-term
loans in 2017 to add to the GH¢74m already in the statement of financial position. Although this
does not seem to be a particularly high level of gearing, the debt/equity ratio did rise from 18.5%
to 32.0% in 2017. The interest charge has risen to GH¢19m from GH¢6m in 2016. The 2017
charge was GH¢15m, suggesting that either the interest rate on the loan is flexible, or that the
full interest charge was not incurred in 2017. The new long-term loan appears to have funded the
expansion in both fixed and current assets in 2017.

Appendix
Ratio Working 2016 2017 2018
Gross profit margin (1) 34.0% 34.30% 35.40%
ROCE (2) 21.1% 21.50% 17.80%
Profit margin (3) 11.9% 12.40% 11.40%
Assets turnover (4) 1.78 1.73 1.56
Gearing ratio (5) 15.6% 24.30% 23.60%
Debt/equity ratio (6) 18.5% 32.00% 30.90%
Interest cover (7) 16.7 8.1 5.5
Current ratio (8) 3.0 2.8 2.7
Quick ratio (9) 1.2 1.1 1.2
Receivables collection period (days) (10) 46 52 64
Inventory turnover period (days) (11) 156 171 182
Payables payment period (12) 35 42 46

Workings (all in GH¢‟m) 2016 2017 2018


1 Gross profit margin 286 336 323
840 981 913

2 ROCE* 100/473 122/568 104/584

3 Profit margin 100/840 122/981 104/913

583
4 Assets turnover 840/473 981/568 913/584

5 Gearing ratio 74/74+399 138/138+430 138/138+446

6 Debt/equity ratio 74/399 138/430 138/446

7 Interest cover 100/6 122/15 104/19

8 Current ratio 394/133 502/180 506/186

9 Quick ratio 157/133 199/180 212/180

10 Receivables collection period 105/840×365 141/981×365 160/913×365


11 Inventory turnover period 237/554×365 303/645×365 294/590×365
12 Payables payment period 53/554×365 75/645×365 75/590× 365

ROCE has been calculated here as: 𝑃𝑟𝑜𝑓𝑖𝑡 𝑏𝑒𝑓𝑜𝑟𝑒 𝑖𝑛𝑡𝑒𝑟𝑒𝑠𝑡 𝑎𝑛𝑑 𝑡𝑎𝑥𝑎𝑡𝑖𝑜𝑛(𝑃𝐵𝐼𝑇)
𝐶𝑎𝑝𝑖𝑡𝑎𝑙 𝑒𝑚𝑝𝑙𝑜𝑦𝑒𝑑

b) Areas for further investigation include the following;


i) Long-term loan: There is no indication as to why this loan was raised and how it was used to
finance the business. Further details are needed of interest rate(s), security given and repayment
dates.

ii) Trading activity: The level of sales has fluctuated in quite a strange way and this requires
further investigation and explanation. Factors to consider would include pricing policies, product
mix, market share and any unique occurrence which would affect sales.

iii) Further analysis: It would be useful to break down some of the information in the financial
statements, perhaps into a management accounting format. Examples would include the

costs.

iv) Accounting policies: Accounting policies may have a significant effect on certain items. In
particular, it would be useful to know what the accounting policies are in relation to intangible
assets (and what these assets consist of), and whether there has been any change in accounting
policies.

v) Dividend policy: Water Limited has maintained the level of dividend paid to shareholders
during the three-year period. Presumably, Water Limited would have been able to reduce the
amount of long-term debt taken on if it had retained part or all of the dividend during this period.

584
It would be interesting to examine the share price movement during the period and calculate the
dividend cover.

QUESTION 9
Gamashie Ltd is considering purchasing an interest in its competitor Bossman Ltd. The
Managing Director of Gamashie Ltd has obtained the three most recent statements of profit or
loss and statements of financial position of Bossman Ltd as shown below.

Bossman Ltd
Statements of Profit or Loss for years ended 31 December
2016 2017 2018
GH¢'000 GH¢'000 GH¢'000
Revenue 18,000 18,900 19,845
Cost of sales (10,440) (10,340) (11,890)
Gross profit 7,560 8,560 7,955
Distribution costs (1,565) (1,670) (1,405)
Administrative expenses (1,409) (1,503) (1,591)
Profit before interest & tax 4,586 5,387 4,959
Finance cost (704) (815) (1,050)
Profit before tax 3,882 4,572 3,909
Income tax (1,380) (2,000) (1,838)
Profit after tax 2,502 2,572 2,071

Bossman Ltd
Statements of Financial Position as at 31 December
2016 2017 2018
Assets GH¢'000 GH¢'000 GH¢'000
Non-current assets
Land and buildings 11,460 12,121 11,081
Plant and machinery 8,896 9,020 9,130
20,356 21,141 20,211
Current assets
Inventory 1,775 2,663 3,995
Trade receivables 1,440 2,260 3,164
Cash 50 53 55
3,265 4,976 7,214
23,621 26,117 27,425
Equity and liabilities
Equity and reserves
Share capital 8,000 8,000 8,000
Retained earnings 6,434 7,313 7,584
14,434 15,313 15,584
Non-current liabilities
12% loan stock 5,000 5,000 5,000
Current liabilities
Trade payables 390 388 446

585
Bank 1,300 2,300 3,400
Taxation 897 1,420 1,195
Dividend payable 1,600 1,696 1,800
4,187 5,804 6,841
23,621 26,117 27,425
Required:
Prepare a report for the Managing Director of Gamashie Ltd commenting on the financial
performance and position of Bossman Ltd and highlighting any areas that require further
investigation.
SOLUTIONS
To: MD of Gamashie Ltd
From: An Accountant
Date: 01/01/19
Subject: The financial position of Bossman Ltd

Introduction
This report has been prepared on the basis of the three most recent statements of comprehensive
income and statement of financial position of Bossman Ltd covering the years 2016 to 2018
inclusive. Ratio analysis used in this report is based on the calculations shown in the appendix
attached.

Financial Performance
Sales have increased at a steady 5% per annum over the three-year period. In contrast, the gross
profit percentage has increased from 42% in 2016 to 45% in 2017 before dropping back to 40%
in 2018. Similarly, operating profit as a percentage of sales was 26% in 2016, 28.5% in 2017 and
25% in 2018. This may indicate some misallocation of costs between 2017 and 2018 and should
be investigated or it may be indicative of a longer downward trend in profitability. Return on
capital employed, as one would expect, has shown a similar pattern with an increase in 2017 with
a subsequent fall in 2018 to a level below that of 2016.

Financial Position
The debt ratio measures the ratio of a company‟s total debt to its total assets. Although we have
no information as to the norm for the industry as a whole, the debt ratios appear reasonable.
However, it should be noted that it has risen steadily over the three year period. When reviewing
Bossman Ltd liquidity the situation has improved over the period. The current ratio measures a
company‟s ability to meet its current liabilities out of current assets. A ratio of at least 1 should
therefore be expected. Bossman Ltd did not meet this expectation in 2016 and 2017. This ratio
can be misleading as inventory is included in current assets. Because inventory can take some
time to convert into liquid assets a second ratio, the quick ratio, is calculated which excludes
inventory. As can be seen, the quick ratio, although improving, is low and this shows that current
liabilities cannot be met from current assets if inventory is excluded. As a major part of current
liabilities is the bank overdraft, the company is obviously relying on the bank‟s continuing
support with short-term funding. It would be useful to find out the terms of the bank funding and
the projected cash flow requirements for future funding.

586
The efficiency ratios, receivables ratio and inventory turnover, give a useful indication of
how the company is managing its current assets. As can be seen from the appendix the
receivables collection period has increased over the three years from 29 days to 58 days. This
may indicate that the company is failing to follow up its debts efficiently or that it has given
increased credit terms to some or all of its customers. Looking at inventory turnover period, this
has also risen from 62 days to 122 days. This may be an indication of over-stocking, stocking up
on the expectation of a substantial sales increase or the holding of obsolete or slowmoving
inventory items which should be written down. More investigation needs to be done on both
receivables and inventory. The financing of additional receivables and inventory has been
achieved in the main through the bank overdraft as the trade payables figure has not increased
significantly.
Conclusion
The review of the three-year financial statements for Bossman Ltd has given rise to a number of
queries, which need to be resolved before a useful conclusion can be reached on the financial
performance and position of Bossman Ltd. It may also be useful to compare Bossman Ltd‟s
ratios to those of other companies in the same industry in order to obtain some idea of the
industry norms. Structure of the report = 2 marks Analysis of performance = 8
marks 10 marks

Appendix to Memorandum
2016 2017 2018
% sales increase 5% 5%
Gross profit % 42% 45% 40%
Operating profit % 25.5% 28.5% 25%
Return on capital employed=Profit before interest tax × 100%
Capital employed
4,586 5,387 4,959
14,434+5,000 =23.6% 15,313+5,000=27% 15,584+5,000 =24.1%
Debt ratio
= Total debt × 100%
Total asset
4,187 + 5,000 5,804 + 5,000 6,841 + 5,000
20,356 + 3,265 = 38.9% 21,114 + 4,976 = 41.4% 20,211 + 7,214 = 43.2%

Current ratio
= Current assets
Current liabilities
3,265/4,187= 0.78 4,976/5,804= 0.86 7,214/6,814 = 1.05

Quick ratio
= Current assets − inventory
Current liabilities
3,265 − 1,775 4,976 − 2,663 7,214 − 3,995
4,187 = 0.36 5,804 = 0.40 6,814 = 0.47

587
Receivables collection period
=
Trade receivables/sales × 365 days
1,440/ 18,000= 29.2days 2,260/18,900 = 43.6 days 3,164/19,845 = 58.2 days
Inventory turnover period
= Inventory/Cost of sales × 365 days
1,775/10,440= 62 days 2,260/10,340 = 94 days 3,995/11,890 = 122.6 days

The following are the accounts of Bounce Back Ltd, a company that manufactures playground
equipment, for the year ended 30 November, 2019.
Statement of comprehensive income for year ended 30 November
2019 2018
GH¢’000 GH¢’000
Profit before interest and tax 2,200 1,570
Interest expense (170) (150)
Profit before tax 2,030 1,420
Taxation (730) (520)
Profit after tax 1,300 900
Dividends paid (250) (250)
Retained profit 1,050 650

Statement of financial position as at 30 November 2019


2019 2018
GH¢’000 GH¢’000
Non-current assets (written-down value) 6,350 5,600

Current assets
Trade receivables 2,100 2,070
Inventories 1,710 1,540
Total current assets 3,810 3,610

Creditors: amounts due within one year


Trade payables 1,040 1,130
Taxation 550 450
Bank overdraft 370 480
Total current liabilities 1,960 2,060

Net current assets 1,850 1,550

Total net assets 8,200 7,150

Creditors: amounts due after more than one year


10% debentures 2020/ 2021 1,500 1,500

588
Equity:
Share Capital (ordinary shares of 50p fully paid up) 3,000 3,000
Retained earnings 3,700 2,650
6,700 5,650

Long term liabilities and Equity 8,200 7,150


Required:
a) Calculate, for both years, the return on equity and the return on capital employed.
b) Calculate, for both years, TWO (2) investment ratios of interest to a potential investor.
c) Calculate, for both years, TWO (2) ratios of interest to a potential long-term lender.
d) Report on the performance and state of the business from the view point of a potential
shareholder and lender using the ratios calculated above and explain any weaknesses in these
ratios. (8 marks)

SOLUTION
a. Computation of return on equity and return on capital employed
Return on Equity = 𝑃𝑟𝑜𝑓𝑖𝑡 𝑎𝑓𝑡𝑒𝑟 𝑡𝑎𝑥 𝑎𝑛𝑑 𝑝𝑟𝑒𝑓𝑒𝑟𝑒𝑛𝑐𝑒 𝑑𝑖𝑣𝑖𝑑𝑒𝑛𝑑 × 100%
𝐸𝑞𝑢𝑖𝑡𝑦 𝑠ℎ𝑎𝑟𝑒ℎ𝑜𝑙𝑑𝑒𝑟𝑠 𝑓𝑢𝑛𝑑

2019 𝑅𝑂𝐸 = 1,300/6,700 × 100 = 19.4%


2018 𝑅𝑂𝐸 = 900 / 5,650 × 100 = 15.93%
Return on Capital Employed = 𝑃𝑟𝑜𝑓𝑖𝑡 𝑏𝑒𝑓𝑜𝑟𝑒 𝑖𝑛𝑡𝑒𝑟𝑒𝑠𝑡 𝑎𝑛𝑑 𝑡𝑎𝑥 × 100%
𝑇𝑜𝑡𝑎𝑙 𝑎𝑠𝑠𝑒𝑡−𝐶𝑢𝑟𝑟𝑒𝑛𝑡 𝑙𝑖𝑎𝑏𝑖𝑙𝑖𝑡𝑖𝑒𝑠(𝑐𝑎𝑝𝑖𝑡𝑎𝑙 𝑒𝑚𝑝𝑙𝑜𝑦𝑒𝑑)

2019,𝑅𝑂𝐶𝐸 = 2,200 / 8,200 × 100 = 26.83%

2018,𝑅0𝐶𝐸 = 1,570 / 7,150 × 100% = 21.96%

b) Computation of investment ratios


Dividend per share = 𝐷𝑖𝑣𝑖𝑑𝑒𝑛𝑑 𝑝𝑎𝑖𝑑
𝑇𝑜𝑡𝑎𝑙 𝑛𝑢𝑚𝑏𝑒𝑟 𝑜𝑓 𝑜𝑟𝑑𝑖𝑛𝑎𝑟𝑦 𝑠ℎ𝑎𝑟𝑒𝑠

2019,𝐷𝑃𝑆 = 250 / 6,000 = ¢0.042 𝑝𝑒𝑟𝑠ℎ𝑎𝑟𝑒

2018,𝐷𝑃𝑆 = 250 / 6,000 = ¢0.042 𝑝𝑒𝑟𝑠ℎ𝑎𝑟𝑒

Earnings per share = 𝑃𝑟𝑜𝑓𝑖𝑡 𝑎𝑓𝑡𝑒𝑟 𝑡𝑎𝑥 𝑎𝑛𝑑 𝑝𝑟𝑒𝑓𝑒𝑟𝑒𝑛𝑐𝑒 𝑑𝑖𝑣𝑖𝑑𝑒𝑛𝑑


𝑇𝑜𝑡𝑎𝑙 𝑛𝑢𝑚𝑏𝑒𝑟 𝑜𝑓 𝑜𝑟𝑑𝑖𝑛𝑎𝑟𝑦 𝑠ℎ𝑎𝑟𝑒𝑠

2019,𝐸𝑃𝑆 = 1,300 / 6,000 = ¢0.22

2018,𝐸𝑃𝑆 = 900 / 6,000 = ¢0.15

Dividend Cover = 𝐸𝑃𝑆 𝐷𝑃𝑆


2019 = 0.22 / 0.042 = 5.23

589
2018 = 0.15 / 0.042 = 3.57

Dividend Pay-out ratio = 𝐷𝑃𝑆 𝐸𝑃𝑆


2019 = 0.042 / 0.22𝑥 100% = 19.23%

2018 = 0.042 / 0.15 𝑥 100% = 27.78%

c) Computation of ratios of interest to a potential long-term lender

𝐷𝑒𝑏𝑡 𝑟𝑎𝑡𝑖𝑜 = 𝑇𝑜𝑡𝑎𝑙 𝑑𝑒𝑏𝑡 / 𝑇𝑜𝑡𝑎𝑙 𝑎𝑠𝑠𝑒𝑡𝑠


2019 = 3,460 / 10,160 = 34.06%
2018 = 3,560 / 9,210 = 38.65%

Gearing = 𝐼𝑛𝑡𝑒𝑟𝑒𝑠𝑡 𝑏𝑒𝑎𝑟𝑖𝑛𝑔 𝑑𝑒𝑏𝑡 (𝐿𝑜𝑛𝑔−𝑡𝑒𝑟𝑚 𝑑𝑒𝑏𝑡) × 100%


𝑆ℎ𝑎𝑟𝑒ℎ𝑜𝑙𝑑𝑒𝑟′𝑠 𝑒𝑞𝑢𝑖𝑡𝑦+𝐼𝑛𝑡𝑒𝑟𝑒𝑠𝑡 𝑏𝑒𝑎𝑟𝑖𝑛𝑔 𝑑𝑒𝑏𝑡

2019 = 1500 / 6,700 + 1,500


= 1,500 8,200 = 18.29%

2018 = 1,500 5,650 + 1,500


= 1,500 7,150 = 20.98%

Debt to equity ratio = 𝐼𝑛𝑡𝑒𝑟𝑒𝑠𝑡 𝑏𝑒𝑎𝑟𝑖𝑛𝑔 𝑑𝑒𝑏𝑡 (𝐿𝑜𝑛𝑔−𝑡𝑒𝑟𝑚 𝑑𝑒𝑏𝑡) × 100%


𝑆ℎ𝑎𝑟𝑒ℎ𝑜𝑙𝑑𝑒𝑟′𝑠 𝑒𝑞𝑢𝑖𝑡𝑦

2019 = 1500/ 6,700 = 22%

2018 = 1,500 / 5,650 = 27%

Interest Cover: 𝐸𝐵𝐼𝑇/ 𝐼𝑛𝑡𝑒𝑟𝑒𝑠𝑡 𝑐ℎ𝑎𝑟𝑔𝑒𝑠


2019 = 2,200 / 170 = 12.9 𝑡𝑖𝑚𝑒𝑠

2018 = 1,570 / 150 = 10.5 𝑡𝑖𝑚𝑒𝑠


b).
Report on performance and state of the business from the view point of a potential shareholder
and weaknesses in the ratios
From the view point of a potential shareholder who would be interested in the returns on his
investment, ratios such as return on equity, return on capital employed, dividend per share,
earnings per share, dividend yield and dividend cover would be of essence to assessing the
performance of the business.
The return on equity tells us the returns from the use of the shareholders fund and this has
increased from 15.93% to 19.4% between 2018 and 2019. This means the business is performing
well. The return on capital employed, tells us how the total assets less current liabilities have
been utilised to generate returns. The business has an improved performance, because it has
increased its return on capital employed by 4.87% (26.83-21.96) between 2018 and 2019.

590
The earnings per share tells us about the earnings before interest and tax that is attributable to
ordinary shareholders and this has increased from ¢0.26 to ¢0.37. This could increase
shareholders confidence in the business from the viewpoint of the lender, who would be
interested in how the business can pay back it debt when they fall due.
Debt ratio talks about the ratio of debt to asset in the business. Usually, a debt ratio of less than
50% is considered acceptable. The business has managed to reduce this ratio within the period
under review. Gearing tells us about the ratio of long-term debt in the business. Again, the
business has performed well to reduce it between the two years. This will increase her chance of
getting more borrowings if needed. Further, a gearing ratio of less than 50% is said to be low
gearing and is acceptable since the company depend less on borrowing. There is a reduction in
risk that little will be available for distribution to ordinary shareholders.
Interest cover tells us whether the business has earned enough profit before interest and tax to
pay its interest cost. From the analysis, the business has made enough EBIT to pay it interest cost
by 10.5 times in 2018 and 12.9 times in 2019. It can be concluded that the business state of
affairs currently is good and acceptable. It has improved on her performance between 2018 and
2019.

the accounting statements, they measure what has happened in the past and not necessarily what

nfluenced the results of


these ratios. Different accounting methods (e.g. FIFO versus LIFO inventory valuation) can have
an impact on financial ratios. Ratios may therefore not reflect real differences in the operations
and financial health of companies.
Companies in different industries tend to have different financial ratios.

CORPORATE FAILURE PREDICTION MODELS


Altman’s Z-Scores
The Z-score formula for predicting bankruptcy was published in 1968 by Edward I. Altman,
who was, at the time, an Assistant Professor of Finance at New York University. The formula
may be used to predict the probability that a firm will go into bankruptcy within two years. Z-
scores are used to predict corporate defaults and an easy-to-calculate control measure for the
financial distress status of companies in academic studies. The Z-score uses multiple corporate
income and balance sheet values to measure the financial health of a company.
Estimation of the formula
The Z-score is a linear combination of four or five common business ratios, weighted by
coefficients. The coefficients were estimated by identifying a set of firms which had declared
bankruptcy and then collecting a matched sample of firms which had survived, with matching by
industry and approximate size (assets).
Altman applied the statistical method of discriminant analysis to a dataset of publicly held
manufacturers. The estimation was originally based on data from publicly held manufacturers,
but has since been re-estimated based on other datasets for private manufacturing, non-
manufacturing and service companies.
The original data sample consisted of 66 firms, half of which had filed for bankruptcy under.

591
The original Z-score formula was as follows:[1]
Z = 1.2X1 + 1.4X2 + 3.3X3 + 0.6X4 + 1.0X5.
X1 = Working Capital / Total Assets. (Measures liquid assets in relation to the size of the
company).
X2 = Retained Earnings / Total Assets. (Measures profitability that reflects the company's age
and earning power).
X3 = Earnings Before Interest and Taxes / Total Assets. (Measures operating efficiency apart
from tax and leveraging factors. It recognizes operating earnings as being important to long-term
viability).
X4 = Market Value of Equity / Book Value of Total Liabilities. (Adds market dimension that can
show up security price fluctuation as a possible red flag).
X5 = Sales / Total Assets. (Standard measure for total asset turnover)- (varies greatly from
industry to industry).
Zones of Discrimination:
Z > 2.99 -“Safe” Zone
1.81 < Z < 2.99 -“Gray” Zone
Z < 1.81 -“Distress” Zone
Z>3.00 – Unassociated with failure

TAFFLER’S Z SCORE
This on the other hand was developed by Professor Taffler on a research conducted on UK
enterprises in 1977.If a Z score model is correctly developed its components ratios typically
reflect certain key dimensions of corporate solvency and performance,such as
profitability,working capital adequacy,financial risk and liquidity.Taffler‟s Z score was
developed on variables below;
Z = 0.53X1 + 0.13X2 + 0.18X3 + 0.16X4
Where:
X1 = Profit before tax
Current Liabilities
X2 = Current Assets
Total Liabilities
X3 = Current Liabilities
Total Assets
X4 = No Credit Interval (How far you can rely on your internal generations without relying on
external liabilities)
No Credit interval = Immediate Assets – Current Liabilities
Operating Cost Depreciation
Note;
X1 = Measures Profitability
X2 = Measures Working Capital Position
X3 = Measures financial risk
X4 = Measures no credit interval
A negative score means the enterprise has a financial profile similar to previously failed
business. A negative score in this context means the ratio (Z Score) as measured is less than
100% or 1.

592
ROBERTSON’S Z-SCORE
Robertson‟s Z-Score lays much emphasis on the rate of change from year to year.The researcher
finds out that if a score falls 40% or more,the company should carry out immediate investigation
to identify the cause.If it further falls by 40% or more in a successive year,then the company is
likely to fail.To him annual review is required to estimate the success of the company for
successive years.
Z = 3.0X1 + 3.0X2 + 0.6X3 + 0.3 X4 + 0.3X5
Where:
X1 = Sales – Total Assets
Sales
X2 = Profit before tax
Total Assets
X3 = Current assets – Total Assets
Current Liabilities
X4 = Equity – Total Borrowings
Total Debt Borrowings
X5 = Liquid Assets – Overdraft
Creditors

It is vital that the above computations are done at yearly interval for comparative reasons. A
change in the respective years‟ figures is checked against 40% (0.4) which may either indicate a
fall or otherwise.

593
BUSINESS AND SHARE VALUATIONS
Valuation is an assessment of the worth of an enterprise which is subject to a merger or takeover
so that the value of the enterprise can be determined. Such valuation helps in determining the
value of shares of the acquired and acquiring company to safeguard the interest of the
shareholders of both companies. A share represents a bundle of rights, like, right to elect
directors, to vote on resolutions of the company, shares in the surplus, if any, on liquidation.
The above topic explores how various values can be placed on the assets of an enterprise.
Usually management, shareholders, and other stakeholders may wish to know the worth of a
business for various decisions. For instance, a shareholder may expect to know how much his
wealth is worth. This and many more other questions require the valuation of a business.

'Valuing a company is a part art and part science'


Placing value on a company is more of an art than a science, since there no had no fast scientific
rules to determining value of a business.
It should be recognized that the value of business is equal to the market or book value of the
aggregate of the number of shares of a company in issue.

MARKET CAPITALISATION
Market capitalisation is the market value of a company‟s shares multiplied by the number of
issued shares.
WHEN VALUATIONS ARE REQUIRED
A share valuation will be necessary:
a. For quoted companies, when there is a takeover bid. A takeover is the acquisition by a company
of a controlling interest in the voting share capital of another company, usually achieved by the
purchase of a majority of the voting shares.
b. For unquoted companies, when:

594
 The company wishes to go public and must fix a price for its shares
 There is a scheme of merger
 Shares are sold
 Shares need to be valued for the purpose of taxation
 Shares are pledged as collateral for a loan
c. For any company, where a shareholder wishes to dispose of his or her shareholding. Some of the
valuation methods we describe will be most appropriate if a large or controlling interest is being
sold
d. For any company, when the company is being broken up in a liquidation situation or the
company needs to obtain additional finance or re-finance current debt.
e. During the complete Sale of company
INFORMATION REQUIREMENT FOR VALUATION
1. Financial statements for at least past five (5) years
2. Summary of Non-current assets list and depreciation schedule
3. Aged accounts receivable summary
4. Aged accounts payable summary
5. List of marketable securities
6. Inventory summary
7. Organization chart and management responsibilities
8. Information regarding the industry and economic environment
9. Projections or forecast for five years
10. Forecast dividend and earnings per share

POSSIBLE PROBLEMS WITH VALUING ASSETS


Asset values can be difficult because values ought to be realistic and attaching a figure to an
individual asset may vary considerably depending on whether it is valued on a going concern or
a break-up basis. Assets values can be based on:

 Historic basis
 Replacement basis (going concern basis)
 Realisable basis (break-up basis)

BASIS OF BUSINESS VALUATION


Businesses are usually valued using various models, including the following:
1. Asset valuation basis
2. Earnings valuation model
3. Dividend basis
4. Cash flow models

Each of the above models could be used to ascertain the value of a business at a particular point
in time. Either of these models provides different value to the business; it therefore rests on
management, shareholders or the stakeholder to decide on the appropriate value of the business.

ASSETS VALUATION MODELS

595
This is a valuation model which is based on the value of the firm's assets. Upon using the asset
valuation basis, the value of the company's shares is represented by the value of the assets. In this
regard, the value of shares is equal to the company's net assets.

Thus, the firm's value = Total Assets – Total liabilities.

Using this method of valuation, the value of a share in a particular class is equal to the net
tangible assets divided by the number of shares in issue. Intangible assets (including goodwill)
should be excluded, unless they have a market value (for example patents and copyrights which
could be sold).
a) Goodwill, if shown in the accounts, is unlikely to be shown as a true figure for the purpose of
valuation, and the value of goodwill should be reflected in another method of valuation (for
example the earnings basis)
b) Development Expenditure, if shown in the accounts, would also have a value which is related to
future profits rather than to the worth of the company‟s physical assets.

Thus: Price per share = Net Tangible Assets


Number of shares in issue
Or
Total tangible assets – Total liabilities
Number of equity shares issued

It should be noted that preference shares are taken as loan capital, and thus, treated as such.
GH¢
Total Tangible Assets xxx
Total Firm's liabilities, including preference share capital xxx
Net Assets xxx
Net Assets per share: Net Assets/number of ordinary shares in issue.

CHOICE OF ASSETS VALUATION BASIS


The choice of values attached to assets should be realistic. The reliability of the assets values
depends on two major concerns that, the business is valued on a going concerns basis or on the
grounds of liquidation.

The following assets values may be adopted by a firm:


 Historic cost valuation
 Replacement cost valuation
 Net Realizable value basis

HISTORIC COST VALUATION BASIS


This is an asset –based business valuation model where the value of the business is determined
on the basis of the book values of the assets. In this case, the net book value of the assets
represents the net book value of the company's shares in issue. This gives a share price of the
average of the net book values to the number of ordinary shares in issue.

596
The use of the historical or book values usually appears unrealistic since the values of the assets
are affected by the accounting policies, such as depreciation, amortization, etc., of the enterprise.
Again, the book values may not reflect prevailing market conditions or even the circumstance of
the enterprise itself.

REPLACEMENT COST BASIS


This is an alternative to the historical-cost business valuation. It is unlikely that a firm would use
the historical cost to determine its value, especially on the going concern basis. For the purpose
of going concern, businesses are usually valued using their replacement values of the assets. This
is based on the notion that the assets would be replaced regularly. The replacement cost is the
cost at which the asset can be replaced to its current state. It does mean acquiring a brand new
asset, but that which has the same state and life as the current asset.

NET REALISABLE VALUE BASIS


The net realizable value of assets represents the value at which the assets can be sold separable.
In this circumstance, the business is said to be have been liquidated or wound up. When the
business is likely to be sold to the extent that it will no longer exists, the break up values are
appropriate to use than the going concern values, such as the replacement cost or the historical
values. The NRV represents the gross proceeds receivable from the sale of the business' assets in
an open market and all liabilities settled.

FACTORS AFFECTING ASSETS VALUATION MODEL


Factors to be considered before choosing asset values include the following:
 Whether there is need for a professional valuation and how much such a valuation is likely to
cost
 Whether or not all liabilities have been accurately valued e.g. deferred tax and any contingent
liabilities
 How have the current assets been valued? Are all debtors collectable and are all inventories
realizable?
 Can all hidden liabilities be accurately assessed? Would there be redundancy payments and
closure costs?
 Is there an available market in which the assets can be realized?
 Are there any prior charges on the assets?
 Does the business have a regular revaluation and replacement policy?

CIRCUMSTANCES UNDER WHICH THE ASSETS BASIS OF VALUATION SHOULD


BE USED
The net asset basis of valuation might be used in the following circumstances:
 As a measure of the 'security' in a share value. The assets‟ backing for shares provides measure
of the possible loss if the company fails to make the expected earnings or dividend payments.
The value of the assets provides the guarantee that at a worse position that the company finds
itself, purchasers would be compensated by the quality of the assets. Where earnings are low, the
assets must be good enough to provide alternative reward to the shareholders through sale.
The asset backing for shares thus provides a measure of the possible loss if the company fails to
make expected earnings or dividend payments. Valuable tangible assets may be a good reason

597
for acquiring a company, especially freehold property which might be expected to increase in
value over time.

 As a measure of comparison in as scheme of merger. Merging refers to the pool of resources by


two or more companies so as to increase the value of the combined business compared to the
aggregate of the individual companies‟ values(synergy). This is usually used when for the two
companies, one has a lower asset backing and the other has a higher assets backing. Thus, where
one firm has a low asset backing and the other having high asset backing, the weaker firm is
strengthened by the asset of the stronger asset.
 As a floor value- for business which is up for sale. Thus, with all the models that may be used,
the minimum price that will be acceptable to the shareholders is the asset values. Thus, at worse
value the asset value is appropriate. In effects, the asset value is the minimum acceptable price to
the shareholders.

Problems with the assets basis


There are two main problems with this valuation approach:
 In practice, realistic asset values are often difficult to identify because there is no necessarily an
active market in second-hand industrial assets.
 The technique only values the hard or tangible assets of the business (such as buildings,
machinery, and stocks) and ignores the company's intellectual assets.

The term 'intellectual assets' refers to such intangible assets as managerial know-how; work
force skills, education and training; network of suppliers; the distribution system; patents,
copyrights, trademarks, and brands, and general business contacts. These are amongst the most
valuable assets that any business uses, but this method ignores it since they do not appear on the
balance sheet.

Illustration:
Akosua,Oppong,Boadi and Isha are the directors of Dankwah and Sons Company Ltd.The
directors own 25% each of equity shares of the company.The directors would wish that the
company is sold as going concern to any interested entity.Akosua and Isha are of the view that
the values shown in the statement of financial position should be used to calculate the price at
which to offer to sell the company.Oppong and Boadi are of different opinion as they believe
that the replacement costs are more realistic.The following information is relevant:

Dankwah and Sons Ltd


Statement of Financial Position as at 2015

Non-Current Assets GH¢ GH¢


Freehold Premises 4,000
Plant and Equipment 8,000
12,000
Current Assets
Inventory 3,000
Trade Receivable 2,000
Cash 400 5,400

598
Total Assets 17,400

Capital and Liabilities


Ordinary shares of no par value 4,000
Income Surplus 6,800
10,800
Current Liabilities 2,600
Secured loan 4,000
17,400
The following information is relevant:
(i)Share Capital consist of 50,000 shares
(ii)The Replacement costs of the assets based on expert advice are:
GH¢
Freehold Premises 15,000
Plant and Equipment 20,000
Inventory 2,000

Required:
Calculate the value of the company as a whole and the value of individual‟s share value on the
following baisis:
(a)Book Values
(b)Replacement Costs
(c)Advice the directors which valuation basis are more appropriate
Solution:
(a)Book Value Method:
Value of Business = Total Assets – Total Liabilities
GH¢ GH¢
Total Assets 17,400
Total Liabilities:
Current 2,600
Loan 4,000 (6,600)
10,800
Individual‟s share (0.25 x 10,800) = 2,700
Price (Net Asset)/share = 10,800
50,000
= GH¢0.22

(b)Replacement Cost: GH¢


Assets:
Freehold Premises 15,000
Plant and Equipment 20,000
Inventory 2,000
Receivables 2,000
Cash 400
39,400
Liabilities (2,600 + 4,000) (6,600)

599
32,800
Price / share = 32,800
50,000
= GH¢ 0.66
Individual‟s share(0.25 x 32,800) = GH¢8,200
Advice:
The directors should accept the replacement cost basis since it is more realistic as compared to
the book value method.
INCOME VALUATION BASIS
This involves the use of the company's future earnings and/or past earnings as the basis of
valuing the company. The quality of the company's earnings is very important to every investor
since the basis of the investors' reward depends on the company‟s earnings. There are number of
models that corroborate the use of the earning basis:
 The Price/Earnings ratio (P/E)
 Dividend basis
 Earnings Yield(EY)
 Accounting Rate of Return(ARR)
 Present Value of Future Earnings / Cash Flows

a. THE PRICE/EARNINGS RATIO (P/E):


This is a common method of valuing a controlling interest in a company, where the owner can
decide on dividend and retention policy. An investor of a large concern is much concerned with
the earning potential of the selling business as he/she is influenced by the reward afterwards.
Investors of this nature have controlling interest in the financial and operating policies, such as
the dividend policy, of the enterprise; hence, the earnings are important to them.

This model could be used to value a block of company usually listed on the stock market. It may
also be used to ascertain the value of an unquoted company, having adjusted for risk preferences,
quality of management, and other relevant variables required by the market.
The P/E ratio = Market Capitalization/Earnings of the company.

The valuations is therefore = P/E * earnings

Earnings: This is the total after tax company profit and preference dividends. It is the residue of
profit after tax and interest that is available to the equity shareholders. This EAIT (Earnings
After Interest and Taxes), as given in the income statement may require adjustments for
exceptional items and other unusual events and transactions such as sale of fixed assets, loss
arising from disaster, unrepresentative payment to managements, etc.

NOTE
The P/E ratio is usually available for listed companies and used (borrowed) for valuing an
unlisted company. It should be noted that the shares in an unlisted company are less marketable
than those of similar listed companies. Where an unquoted company is concerned, the P/E ratio
of a similar quoted company of a comparable size in the same industry with the same operating

600
risk should be reduced to take into account, the non-marketability of the shares. The
reduction is technically called an abatement factor.

THE SIGNIFICANCE OF THE PRICE- EARNING RATIO


A high price/earnings ratio may indicate the following:
 Expectation that the EPS will grow rapidly

 It provides a security of earnings. A well-established company will be valued on a higher P/E


ratio than a similar company whose earnings are subject to greater uncertainty

 Status: A quoted company would usually expect a higher P/E ratio than an unquoted company
when the former company makes a bid to acquire the latter company.

PROBLEMS WITH THE P/E RATIO


 A single years earning per share may not be a good basis, if earnings are volatile
 The quoted company may have a different capital structure from the unquoted company
 It is difficult obtaining similar company that may have the same business structure or risk to act
as the pure-play comparison.

FACTORS THAT INFLUENCE THE SIZE OF THE P/E RATIO


The following are some of the factors that influence the price of a share of a company:
 General economic and financial conditions e.g. inflation
 The type of industry and the prospect of the industry
 The size of the undertaking and its status within the industry
 The marketability of the shares of the company
 The diversity of shareholdings and the financial status of the principal shareholders
 The reliability of profit estimates and past profit records
 The level of gearing of the company
 Assets backing and liquidity
 The nature of the assets, for example, whether some of the fixed assets are of highly specialized
nature and so have only a small break-up value.

Illustration 1:
The statement of financial position of Kante Ltd,a private company,is as follows:
GH¢
Non – Current Assets:
Goodwill 500,000
Tangible 1,800,000
Net Current Assets 1,700,000
4,000,000
Share Capital:
1,800,000 ordinary shares @ GH¢1 each 1,800,000
Income Surplus 1,200,000
3,000,000

601
1,000,000 7% preference shares of NPV 1,000,000
4,000,000
The following information is relevant:
The price earnings ratio of a quoted company in the same industry is 12
(i)The premises included in the fixed assets figure is GH¢1,400,000 which at current rate is now
valued at GH¢1,500,000
(ii)The net current asset are considered to be overvalued by GH¢200,000
(iii)The preference share capital is redeemable at book value
(iv)The trend profit (or loss) before taxation is as follows:

Years GH¢
1 500,000
2 (50,000)
3 450,000
4 650,000
5 700,000
Required:
How would you value the ordinary shares of Kante Ltd using the earning (P/E) basis?

Illustration 2:
Executive Group Ltd wishes to make a takeover bid for the shares of unquoted company,Special
Ltd,the earnings of Special Ltd over the past five years were as follows:
Years GH¢
2011 25,000
2012 36,000
2013 34,000
2014 35,000
2015 37,500

The average price earnings ratio of quoted companies in the industry in which Special Ltd
operates is 10.Quoted companies which are similar in many respects to Special Ltd are:
(a)True Vine Ltd which has a P/E ratio of 15,but is a company with a very good prospects
(b)Sweet Apple Ltd,which has had a poor profit record for several years,and has a P/E ratio of 7.

Required:
What would be a suitable range of valuations for shares in Special Ltd?

b. EARNING YIELD VALUATION MODEL


This model incorporates the volatility in the earnings than the use of the P/E ratio. It reciprocates
the P/E model in order to measure the growth rate or the earning yield as follows:

Earning Yield = Maintainable Earnings ×100%


Market valuation

602
Therefore: Market Valuation = Earnings
E/Y

Where there is expected growth in earnings:

Market Value = Earnings (1+g)


Ke- g

Earnings per share = Net Profit after tax- Preference Dividend


Number of ordinary shares in issue

Exactly the same guidelines apply to this method as for the P/E ratio method. Note that where
high growth is envisaged, the EY will be low, as current earnings will be low relative to a market
price that has built in future earnings growth. A stable earnings yield may suggest a company
with low risk characteristics.

Illustration 1:
Below is a summary income statement of Sanchez Ltd for 2014:
GH¢ GH¢
Trading Profit 360,000
Less:
Directors Salary 76,000
Depreciation 44,000 120,000
240,000
Income tax (30%) (72,000)
Profit after tax 168,000
Proposed dividend (16,800)
15,200
Retained Profit 320,800
472,000
Assume:
 An expressed fair return of this class of business is between 20% and 25%
 The Directors remunerative to excessive and would normally be expected to be GH¢40,000.
 Rent for the period of GH¢76,000 have not been taken into account
 The number of shares owned is 800,000
 Both the change for depreciation and stock values are considered reasonable for this type of
business
 Market Values of freehold land is estimated at GH¢800,000.All other assets have no market
values

Required:
To value the shareholdings of Sanchez Ltd on earning basis

Solution:

603
GH¢ GH¢
Current Earnings before tax 240,000
Adjustments for:
Remuneration 36,000
Rent (76,000) (40,000)
Earnings before tax 200,000
Tax (30%) 60,000
Maintainable Earnings 140,000

Value of the Business:


Assuming security on assets rate of 20% is appropriate.
Value of Business = Maintainable Profit
Capitalisation Rate
= 140,000
0.2
= GH¢700,000
Illustration 2:
A company is expected to generate future profits of GH¢30 million per annum. What is the value
of the company, based on earnings yield, if investment in the industry are expected to give
annual return of 12%.

Solution:
Profit after tax = GH¢30,000,000
Earnings Yield = 12%

Value of Business= GH¢30,000,000


0.12
=GH¢250,000,000

c. DIVIDEND VALUATION MODEL


This method is appropriate for valuing small shareholdings in unquoted companies. It is based
on the principle that small shareholders are mainly interested in dividends since they cannot
control decisions affecting the company‟s profits and earnings.
The dividend yield ratio relates to the cash return from dividends to the current market value per
share. This ratio measures the return to an investor in the form of dividends for a cedi investment
and it is calculated as:
Dividend Yield = Gross dividend per share × 100%
Market price per share
By mathematically re-arrangement, the market price per share can be obtained by:

Market price per share = Dividend per share


Dividend yield
NB:The value of a share found by using such dividend yield will be adjusted downward for risk
differentials or the similar listed company’s yield should be adjusted upwards by the risk
factor before using it as the appropriate or suitable dividend yield.

604
 Dividend for the year in both interim and final dividends for the year.
 Use gross dividend or per share and not net dividend or per share.

 Where dividend growth is expected


Market price per share (ex-div.) = dividends (1 + growth rate)
Dividend yield – Expected growth rate
NB:
Using the Capital Assets Pricing Model (CAPM) formula
Required Rate of Return (r) = RF + B (RM – RF)
Where:
Rf = risk free
B = beta
Rm= return on market

It is often preferable to use gross dividends. In examination students‟ may be required to


calculate the required return and this can be done using CAPM formula.

Price per share = Dividend (1+g)


ke - g
The cost of capital is the cost of equity (Ke) or the Weighted Average Cost of Capital (WACC)
of the buying company (the acquirer). The appropriate rate is the Ke.

Problems of using the dividend yield


 Forecasting dividend is very difficult
 Problem deciding on the suitable dividend yield
 The valuation is affected by management‟s dividend policy
 The valuation does not take into account the total performance of the company but only the
dividends declared.

REASONS WHY THE DIVIDEND YIELD OF A SIMILAR LISTED COMPANY NEED


TO BE ADJUSTED
 The shares of the listed company are more marketable compared to private unlisted companies
 Quoted companies often have more certain dividend policy than private unlisted companies
 Investments in quoted companies as less risky than in private unlisted companies since they are
more diversified
 Quoted companies are often bigger in size as compared to private unlisted companies
 The level of gearing may differ for listed and unlisted companies
 Other specific factors may need to be considered.

ASSUMPTIONS UNDERLYING THE DIVIDEND MODEL


 Investors are homogenous and rational
 The estimate of future dividends, prices, and cost of capital are reasonable.
 Dividends are used to signal the investors on the company's strength
 Growth in dividends is constant or nil
 There are no other influence on share price than dividends

605
 Discount rate exceed the growth in dividend

Illustration 1:
Dumelo Ltd is planning to obtain a stock market quotation by offering 45% of its shares to the
public.No new shares will be issued.It most recent summarized results are as follows:
GH¢
Turnover 250,631
Earnings 35,000
Number of shares in issue 60,000

The company is lowly geared and has a dividend policy of 50% retention.This retention policy is
expected to achieve 8% dividend growth each year.
Below is summarized detailed of two listed companies in the same industry as Dumelo Ltd:
Beautiful Ltd Gardner Ltd
Gearing (Total Debt/Total Equity) 48% 15%
Equity Beta 1.5 1.1
The risk free rate of return is 20% per annum and the average market return is 28%.The shares
will be offered to the public at a price of 20% lower than the estimated market price valuation in
order to increase the prospects of the public issue.
Required:
What is the price per share?

Solution:
Using the Dividend Growth Model:
MPS = Dividend per share (1+g)
Ke – g
DPS = Total Dividend
No. of shares
Earnings = GH¢35,000
Payout ratio (50%) = GH¢17,500
Dividend per share = GH¢17,500
60,000
= GH¢0.29
Growth = 8%
Ke = rf + B(rm-fr)
= 20+1.1(28-20)
= 20 + 1.128
= 28.8
MPS = 0.29(1+0.08)
0.288-0.08
= 0.29(1.08)
0.208
= GH¢1.51
Actual Price = 0.8 x 1.51
= GH¢1.21

606
d. SUPER PROFIT METHOD
The value of a business using super profit is simply equal to Goodwill plus net book value assets
of the target company less liabilities. Thus, by this method, a fair rate of return is applied to the
net tangible assets and the result is compared with the expected profits. Any excess of profit
(referred to as super profit) is used to calculate goodwill. The goodwill calculated at a specified
number of years‟ super profit is added to the value of target company‟s net assets to arrive at a
value for the business.
Demerits of Super Profit Method
 the expected rate of return is subjectively or arbitrary chosen without any scientific precision
 The number of years‟ purchase of super profit is arbitrary.
In effect,the value of the shares is determined as follows:
GH¢
Maintainable Earnings xxx
Less return on capital employed (Net Tangible Assets) xxx
Super profit xxx
Value of Assets (both Tangible and Intangible):
GH¢
Goodwill (n x super profit) xxx
Net tangible assets xxx
Value of Business xxx

Where:
N = Number of years purchase of super profit
Capital employed = total tangible assets less total liabilities
Illustration:
Joe Boy Ltd has net tangible assets of GH¢96,000 and earnings of GH¢50,000.Storng Ltd wishes
to acquire the former and considers that a fair return for this type of industry is 25% and decided
to value Joe Boy Ltd‟s goodwill at 4 years purchase of super profit.
Required:
Determine the value of Joe Boy Ltd.
Solution:
GH¢
Earnings 50,000
Return on Tangible Assets 24,000
Super Profit 26,000
Goodwill (26,000 x 4) 104,000
Value of Business:
GH¢
Net Tangible Assets 96,000
Goodwill 104,000
200,000

ACCOUNTING RATE OF RETURN (ARR) METHOD


This method considers the rates of return which will be required from the company whose shares
are to be valued. This valuation relates to the average annual profit of the company to the
accounting rate of return or the capital employed.

607
The ARR might be used in a takeover when the acquiring company is trying to assess the
maximum amount it can afford to pay.
If the valuation is for a takeover bid, it will be necessary to adjust the profit figure to allow for
expected changes after the takeover. Such adjustments may include new levels of directors‟
remuneration, new levels of interest charges, a charge for notional rent and the effects of product
rationalization and improved management.
It is given as:
ARR = Profit after tax or ARR= Average Annual Profit after tax
Capital employed Average investment
Therefore, by re-arrangement, capital employed, which is equivalent to the value of the business,
is given as:
Value (price) per share = Estimated Profit after Tax
ARR

FORECAST (DISCOUNTED) CASH FLOW BASIS


This is most appropriate when a company intends to buy the assets of another company and to
make further investments in order to improve on the cash flows in future. Thus, this method is
adopted when there is going to be additional capital investments to boost the current status of the
company acquired by the acquirer. The value of the company on this basis is the present value of
its future expected cash flows.

The only difference between the dividend model and the cash flow model is the fact that the
former uses the dividend policies of the enterprise whiles the latter is based on the genuine or
free cash flow of the company.

The free cash flow (FCF) is defined as the cash flow that is available to be distributed to the
suppliers of finance-both equity and debt capital-while maintain the company's continued
existence. It is equal to the firms operating cash flow after tax but before interest payments,
minus capital investments required to maintain the company' continued level of activity in the
future.

This free cash flow may grow constantly forever or irregularly.

FCF = operating cash flow – tax paid – capital maintenance charge + depreciation

Where the FCF grows irregularly:


Value of the company (Vo) = Present value of the discounted future expected FCF

Alternatively:
Constant growth: FCF (1+g)
WACC – g
g- Growth rate in FCF

Illustration :
Vigilance Ltd has been in existence since 2001, with steady results. Since 2007 it has generated
cash flow of GH¢80,000 and the directors expect this to continue for the next five (5) years.In

608
2007,at the AGM, the shareholders requested the directors of Vigilance Ltd to contact the
directors of Propaganda Ltd who were willing to sell the entire business of Propaganda Ltd for
GH¢130,000.The directors of Vigilance Ltd obtained the following results:
i. It was expected that Propaganda Ltd will make a profit of GH¢80,000 in each year for the
next five years, after charging annual depreciation of GH¢10,000.
ii. Propaganda Ltd has to repay a debenture of GH¢24,000 on 31st December,2009 will have
to replace a plant at a cost of GH¢60,000 on 31st December,2011
iii. If Propaganda Ltd is taken over, Vigilance Ltd will carry out a reorganization scheme and
st
by 31 December,2008 will be able to dispose of property to realize GH¢40,000 and to reduce
the combined working capital by GH¢10,000.It will also make an annual cost savings of
GH¢5,000.
iv. The appropriate discount rate to be applied to the acquisition is 25%.Assume that the
cash flow arises at the end of the year.

Year 0 1 2 3 4 5
Discounting factor 1.000 0.800 0.640 0.512 0.410 0.328

Required:
Advise the directors of Vigilance Ltd on whether the acquisition of Propaganda Ltd at the stated
price should be undertaken.

Solution:
Project Appraisal (Business Valuation)
Years 2008 2009 2010 2011 2012
Cash Flows: GH¢ GH¢ GH¢ GH¢ GH¢
Annual Profit 90,000 90,000 90,000 90,000 90,000
Redemption of Deb. (24,000)
Asset Replacement (60,000)
Disposal of Property 40,000
Releasing of WC 10,000
Cost Savings 5,000 5,000 5,000 5,000 5,000
Net Cash Flows 145,000 71,000 95,000 35,000 95,000
Discounting factor 0.800 0.640 0.512 0.410 0.328
116,000 45,440 48,640 14,350 31,160
Total Present Value GH¢255,590
P.V of Investment (GH¢130,000)
NPV GH¢125,590

DUAL CAPITALISATION METHOD


This method of valuation is used to determine acceptable yields for both tangible and intangible
assets.Infact,this is much closer,to the super profit approach.It blends features of super profit and
the super profit valuation models.An acceptable rate of return is applied on the value of tangible
assets.The figure obtained is known as the return on tangible assets.This return is deducted from
the gross earnings to obtain the return on the intangible assets.The earnings attributable to the
intangible assets is capitalized at an acceptable rate of return;and thus ,added to the total tangible
assets to ascertain the value of the business.The earnings capitalized give rise to goodwill.

609
Illustration:
Yeremiah Ltd has an average earnings of GH¢75,000 and this approximate its maintainable
earnings.The tangible assets of the business is GH¢250,000 and the total liabilities of
GH¢50,000.The required return on the net tangible assets is 20%.The required return on the
intangible assets is pegged at 30% due to high risk associated with the company.
Required:
Determine the value of Yeremiah Ltd using the Dual Capitalisation model
Solution: GH¢
Earnings 75,000
Return on Tangible Assets (0.2 x 200,000) (40,000)
Return on Intangible Assets 35,000
Value of Intangible Asset = GH¢35,000
0.3
=GH¢117,000
Value of Business:
GH¢
Tangible Assets 250,000
Intangible Assets 117,000
Liabilities (50,000)
317,000

SUGGESTED EXAMINATION APPROACH


Unless otherwise stated in the question in exams, the following may be followed:
 Briefly explain or define the method applied
 State the formula for calculating the value of the business under the method
 State any assumptions made and why
 Apply the formula to value the business and calculate the value per share
 Draw your conclusions as appropriate
 Briefly state the merits and flaws(demerits) in the formula applied.

QUESTION 1
Obuoba Ltd is a family controlled company that has grown over the years. The company is now
considering listing on the Stock Market. The MD believes that as the company has assets with a
book value of GH¢46 million and shareholders fund GH¢24 million, the company‟s value, when
listed should be at least GH¢70 million . He proposes that 500,000 new shares should be issued
to the public to raise approximately GH¢3.5 million for future expansion .
The company‟s financial performance for the last three years is summarized below:
Income statement for the years ended 31 December
2009 2010 2011
GH¢‟000 GH¢‟000 GH¢‟000
Sales 40,000 45,000 52,000
Cost of goods sold 25,000 26,000 29,000
Gross Profit 15,000 19,000 23,000
Admin expenses 3,000 4,000 4,500
Selling and general expenses 1,600 2,600 4,500

610
Interest payable 1,400 1,400 2,000
Net Profit before tax 9,000 11,000 12,000
Current Taxation @ 20% 1,800 2 200 2,400
Net profit after tax 7,200 8,800 9,600

Statement of Financial Position as at 31 December


2009 2010 2011
GH¢‟000 GH¢‟000 GH¢‟000
Non-current assets
Property, plant and equipment (Net):
Land and buildings 15,000 16,000 16,000
Plant 5,000 6,000 7,000
Investment (FVTPL) 2,000 2,000
Current Assets
Inventory 7,000 7,000 11,000
Trade Receivables 5,000 8,000 9,000
Cash 1,000 1,000 1000
33,000 40,000 46,000
Financed by
Stated Capital (Issued at GH¢0.10 per share) 1,000 1,000 1,000
Retained earnings 18,000 19,000 23,000
Shareholders fund 19,000 20,000 24,000
Long term loans 7,000 7,000 11,000
Current liabilities
Trade payables 5,000 10,800 8,600
Taxation 2,000 2,200 2,400
33,000 40,000 46,000
Additional notes relating to the Income Statement for the year ended 31 December 2011 and the
Statement of Financial Position as at 31 December 2011 is as follows:
(i) Obuoba‟s income statement includes GH¢ 8 million of revenue for credit sales made on a
„sale or return‟ basis. At 31 December 2011, customers who had not paid for the goods, had the
right to return GH¢ 2·6 million of them. Obuoba applied a mark-up on cost of 30% on all these
sales. In the past, Obuoba‟s customers have sometimes returned goods under this type of
agreement.
(ii) The property, plant and equipment have not been depreciated for the year ended 31
December 2011. Obuoba has a policy of revaluing its land and buildings at the end of each
accounting year. The values in the above statement of financial position are as at 1 January 2011
when the buildings had a remaining life of 20 years. A qualified surveyor has valued the land and
buildings at 31 December 2011 at GHS 18 million. Additional plant was installed in January
2011. Plant is depreciated at 20% on the reducing balance basis.
(iii) The investments at fair value through profit and loss [FVTPL] are held in a fund whose
value changes directly in proportion to a specified market index. At 1 January 2011 the relevant
index was 150 and at 31 December 2011, it was 162.
(iv) In late December 2011, the directors of Obuoba discovered a material fraud perpetrated by
the company‟s financial accountant that had been continuing for some time. Investigations
revealed that a total of GH¢ 2 million of the trade receivables as shown in the statement of

611
financial position at 31 December 2011 had in fact been paid and the money had been stolen by
the Financial Accountant. An analysis revealed that GH¢ 1·5 million had been stolen in the year
to 31 December 2010 with the rest being stolen in the current year. Obuoba is not insured for this
loss and it cannot be recovered from the Financial Accountant.
(v) During the year taxable temporary differences of GH¢ 500,000 arose. The applicable income
tax rate is 20%. This has not been included in the figures in the financial statements above.
vi) The long term loans consists of:
 20% GH¢ 7 million debenture stocks issued on 1 January 2009 and redeemable at par on 31
December 2013; and
 15% GHS 4 million loan notes issued on 1 January 2011 and redeemable on 31 December 2012
for GH¢4,262,000 resulting in an effective interest rate of 18% per annum. This Financial
liability is to be measured at amortised cost.

(vii) The only movements in the statement of changes in equity [retained earnings column] in
2011 were the reported draft profit and dividend payment.
Obuoba‟s management has obtained some financial information on a listed company in the same
industry, which has the same number of listed equity shares as Sankofa.

Dadeba Ltd
Market capitalization GH¢42 million
Number of shares 10 million
Earnings per share GH¢0.60
Dividend pay-out ratio 60%
Required
a) Assess the validity or otherwise of the Managing Director‟s statement (5marks)

b) Advise the directors of Obuoba Ltd the values to be placed on an ordinary share using the
following methods
i) Price Earnings ratio
ii) Dividend yield
iii) Net Assets [fair values] (15 marks)

Note
The following assumptions and bases may be relevant:
i) The revised profit after tax for the current year may be a good representation of the earnings of
the entity.
ii) Additional information (i) – (iv) above would necessitate a revision of the 2011 Income
statement and statement of financial position profit. Dividend payment will however not be
affected
iii) Investing in unlisted securities is about 25% more risky than investing in listed securities.

(Total: 20 marks)
Solution:
(a). The Managing Director‟s statement is incorrect for a number of reasons:
i. The figures are based on book values. It is more appropriate to use market values in arriving at
a valuation for listing purposes.

612
ii. The addition of total assets and surpluses is to double count since the surplus represents one of
the sources of finance use to acquire the existing assets.
iii. The liabilities should have been deducted from the total assets figure in order to arrive at the
net asset position of the company.
iv. The issue of 500,000 new shares would bring the total number of shares of the company to
10,500,000 . If only the new shares are offered on the exchange, this represents 4.7% of the total
number of shares. The minimum percentage that can be offered in the market is about 25%.
v. In view of the problems with the net assets valuation, and taken into account the valuation
estimates given below, Obuoba is unlikely to achieve the price of GH¢7 per share which would
be necessary to raise GH¢3.5 million from 500,000 shares

(b)
i. Revised Profit (for PE Ratio method)
GHS’000
Profit before Tax per draft accounts 12,000
URP on sale or return [30/130 x GHS2.6 m] (600)
Depreciation: Land and building (800)
Plant (1,400)
Fair valuation gain of FAFVTPL 160
Embezzlement: current year (500)
Additional interest [amortization] 720-600 (120)
-------
Profit before tax 8,740
Taxation: Current Tax @ 20% 1748
Deferred Tax [20% 500) 100
-------
(1,848)
---------
Profit after tax 6,892
=====

ii. Dividend Paid [For Dividend Yield method]


GHS’000
Retained earnings as at 31 December 2010 19,000
Reported profit 9,600
----------
28,600
Balance as at 31 December 2011 (23,000)
-----------
Dividend paid 5,600
=====
An alternative is to prepare a revised Income Statement as follows:
GHS‟000
Sales (52000-2,600] 49,400
Cost of sales [29,000 – 2,000 + 800+1400] (29,200)

613
-----------
Gross Profit 20,200
Selling expense (4,500)
Admin (4,500)
Interest [2000+120] (2,120)
Fair valuation gain 160
--------
Profit before tax 8,740
Taxation (1,848)
--------
Net Profit 6,892
====
iii. Revised Net Asset [ For Net Assets Method]
GHS‟000
Per the trial balance 24,000
Prior year adjustment [embezzlement] (1,500)
Adjustment to reported profit [9,600 – 6,892] (2,708)
Revaluation surplus – Land and buildings 2,800
--------
22,592
=====
An alternative is to prepare the revised SOFP as follows
GHS‟000
Land and buildings 18,000
Plant 5,600
FAFVTPL 2,160
-------
25760
-------
Current assets
Inventory 11,000+ 2000 13,000
Trade receivables [9,000-2,000 -2,600] 4,400
Cash 1,000
--------
18,400
--------
Total assets 44,160
====
Equity
Stated capital 1,000
Revaluation surplus 2,800
Retained earnings [23,000- 1500 - 2708] 18,792
------
22,592
Non-current- liabilities
Long term loans [7,000 + 4120] 11,120

614
Deferred tax provision 100
Current liabilities
Trade payables 8,600
Current tax 1,748
-------
44,160
i. Price Earnings Ratio
Value per share = EPS X PE ratio
EPS GHS6,892,000/10 million shares = GHS 0. 6892
PE ratio = That of Dadeba as adjusted
= (GHS4.20/GHS0.60) = 7 adjusted to 5.25
Value per share = GHS0.6892 x 5.25 = GHS3.6183

ii. Dividend Yield Method


Value per share = Do/Dividend yield
Do = GHS5,600,000 / 10m shares = GHS0.5600
Dividend yield = That of Dadeba as adjusted
= (GHS0.36/GHS4.20) = 8.57% adjusted to 10.71% or 11.42%
Value per share = GHS0.5600/0.1142 = GHS4.9037 or GHS0.5600/0.1071 = GHS5.228

iii. Net Assets Method


Value /share = Net Assets/No of ordinary shares
=GHS22,592,000/10 million shares
=GHS2.2592

QUESTION 2
Quality Handicraft Ltd [QHL] produces handicrafts for both local and foreign market. The
company was incorporated in the year 2008 and now employs about 150 craftsmen.
The shareholders of QHL mainly comprise the original founder and close family members who
would now like to realize their investment. In order to arrive at an estimate of what they believe
the business is worth, they have identified a long established quoted company, Suama Handicraft
Company [SHC] which has similar business, though it also produces for the European market.
Summarized financial statistics for the two companies for the most recent financial year are as
follows:
QHL SHC
Issued shares (million) 8 20
Net assets value [GH¢ ‟million] 14.4 30
Earnings per share (pesewas) 35 28
Dividend per share (pesewas) 20 24
Debt: Equity ratio 1:7 1:6.5
Share price (as quoted on the stock market) - pesewas - 160
Expected rate of growth in earnings/dividends 5% 5%
Additional Information:
i. The net assets of QHL stated above are the net book values of tangible non-current assets plus
working capital. However:

615
 A recent valuation of the buildings was GH¢1,500,000 above book value
 An investment held which is designated as available for sale with a carrying value of
GH¢1,000,000 is fair valued at GH¢1,100,000
 Due to a dispute with one of their clients, an additional allowance for bad debts of GH¢750,000
could prudently be made.
 An item of plant with a carrying value of GH¢800,000 is assessed to have value-in-use of
GH¢760,000 and fair value less cost to sell of GH¢780,000.

ii. Growth rate should be assumed to be constant per annum. QHL‟s earnings growth rate
estimate was provided by the marketing manager, based on expected growth in sales adjusted by
normal profit margins. SHC‟s growth rates are gleaned from press reports.

iii. The dividend yield of SHC approximates its cost of equity.

Required:
(a) Compute a range of valuations for the business of QHL, using the information available and
stating any assumptions made. Use the following methods for the valuation (Net Assets, Price-
earning method and dividend growth methods) (9 marks)

(b) Comment on the strengths and weaknesses of the methods used in (a) and their suitability for
valuing QHL
(6 marks)

Note: The additional information (i) may affect the net asset value and the earnings per share
stated above. Ignore tax implications.
(Total: 15 marks)
Solution:
Net Assets Method GH¢‟000
Net Assets as per the draft account 14,400
Adjustments:
Revaluation surplus –buildings 1,500
Fair valuation surplus –AFSFA 100
Allowance for doubtful debts (750)
Impairment loss (20)
Value of business 15,230
Price earnings Ratio Method
Value of business = Earnings x PE Ratio
Earnings GH¢‟000
Per draft accounts [GH¢0.35 X 8 million shares] 2,800
Adjustments
Allowance for doubtful debts (750)
Impairment loss (20)
2,030
PE Ratio

616
Taken that the PE Ratio of the unlisted entity must be adjusted for lack of marketability and
higher risk
PE Ratio of SHC = 160p/28 p =5.7
Adjusted to say 4
Value of business = GH¢2, 030,000 X 4 = GH¢8,120,000
Dividend Growth method
Value of business = Do(1+g)/(DY-g)
Do = GHS0.20 X 8,000,000 shares = GH¢1,600,000
DY = that of listed entity (appropriately adjusted)
= 24p/160p =15%
Adjusted to say 20%
Value of business = GH¢1,600,000X 1.05
0.20 – 0.05
= GH¢1,680,000/0.15
= GH¢11,200,000
Summary GH¢
PE Ratio 8,120,000
Dividend growth 11,200,000
Net Assets 15,230,000

b) Comment on relative merits of the methods used, and their suitability


Asset Based Valuation
Valuing a company on the basis of its asset values alone is rarely appropriate if it is to be sold on
a going concern basis. Exceptions would include property investment companies and investment
trusts, the market values of the assets of which will bear a close relationship to their earning
capacities.
Knowledge of the Net Asset Value (NAV) of a company will, however, be important as a floor
value for a company in financial difficulties or subject to a takeover bid. Shareholders will be
reluctant to sell for less than the net asset value even if future prospects are poor.
P/E Ratio Valuation
The P/E ratio measures the multiple of the current year‟s earnings that is reflected in the market
price of a share. It is thus a method that reflects the earnings potential of a company from a
market point of view. Provided the market is efficient, it is likely to give the most meaningful
basis for valuation.
One of the first things to say is that the market price of a share at any point in time is determined
by supply and demand forces prevalent during small transactions, and will be dependent upon a
lot of factors in addition to a realistic appraisal of future prospects. A downturn in the market,
economies and political changes can all affect the day-to-day price of a share, and thus its
prevailing P/E ratio. It is not known whether the share price given for SHC was taken on one
particular day, or was some sort of average over a period. The latter would perhaps give a
sounder basis from which to compute a applicable P/E ratio.
Even if the P/E ratio of SHC can be taken to be indicative of its true worth, using it as a basis to
value a smaller, unquoted company in the same industry can be problematic.
The status and marketability of shares in a quoted company have tangible effect on value but
these are difficult to measure.

617
The P/E ratio will also be affected by growth prospects – the higher the growth expected, the
higher the ratio. The growth rate incorporated by the shareholders of SHC is probably based on a
more rational approach than that used by QHL.
In the valuation in (a) a crude adjustment has been made to SHC‟s P/E ratio to arrive at a ratio to
use to value QHL‟s earnings. This can result in a very inaccurate result if account has not been
taken of all the differences involved.

Dividend Based Valuation


The dividend valuation model (DVM) is a cash flow based approach, which valued the dividends
that the shareholders expect to receive from the company by discounting them at their required
rate of return. It is perhaps more appropriate for valuing a non-controlling shareholding where
the holder has no influence over the level of dividends to be paid than for valuing a whole
company, where the total cash flows will be of greater relevance.
The practical problems with the dividend valuation model lie mainly in its assumptions. Even
accepting that the required „perfect capital market‟ assumptions may be satisfied to some extent,
in reality, the formula used in (a) assumes constant growth rates and constant required rates of
return in perpetuity.
Determination of an appropriate dividend yield/cost of equity is particularly difficult for an
unquoted company, and the use of an „equivalent‟ quoted company‟s data carries the same
drawbacks as discussed above. Similar problems arise in estimating future growth rates and the
results from the model are highly sensitive to changes in both these inputs.
It is also highly dependent upon the current year‟s dividend being a representative base from
which to start.
The dividend valuation model valuation provided in (a) results in a higher valuation than that
under the P/E ratio approach. Reasons for this may be:
 The share price of SHC may be currently depressed below its normal level, resulting in an
inappropriate low P/E ratio.
 The adjustment to get to an appropriate P/E ratio for QHL may have been too harsh, particularly
in light of its apparently better growth prospects.
 The dividend yield/cost of equity used in the dividend valuation model was that of SHC. The
validity of this will largely depend upon the relative levels of risk of the two companies.
Although they both operate the same type of business, the fact that SHC sells its material
externally means it is perhaps less reliant on a fixed customer base.
Even if business risks and gearing risk may be thought to be comparable, a prospective
buyer of QHL may consider investment in a younger, unquoted company to carry
greater personal risk. His required return may thus be higher than that envisaged in the
dividend valuation model, reducing the valuation.

BUSINESS VALUATION
Question 1(nov 2017)
Santader Limited intends to take over Agos Limited. The financial statements of Agos
Limited for the year ended 30 June 2016 are as follows:
Agos Limited
Income Statement for the year ended 30 June, 2016
GH¢

618
Profit before tax 450,000
Tax 125,000
Profit after tax 325,000

Statement of Retained Earnings for the year ended 30 June 2016.


GH¢
Balance at beginning 250,000
Profit after tax 325,000
Dividend paid (180,000)
Balance at end 395,000

Statement of Financial Position as at 30 June 2016 GH¢ GH¢


Non-current Assets:
Freehold land and building at cost 204,500
Plant and machinery (Net) 156,700
Motor vehicles (Net) 560,000
Patent 150,000

1,071,200
Current Assets:
Inventories 500,000
Trade receivable 680,000
Bank balance 120,000

1,300,000
Current Liabilities:
Trade payable 240,000
Accrued expenses 180,000

420,000

880,000
Net current assets
1,951,200
Financed By:
Stated capital ordinary shares issued
@GH¢1 1,000,000
Retained earnings
395,000

1,395,000
25% Debenture stock
556,200

1,951,200
Additional Information:

619
i) Turnover, profits before tax and dividend of Agos Limited over the past 5 years were
as follows:
Year Ending Sales Pre-Tax Profits Dividend
30 June
GH¢ GH¢
GH¢
2012 5,800,000 250,000 65,000
2013 6,900,000 320,000 80,000
2014 7,700,000 330,000 100,000
2015 8,500,000 410,000 120,000
2016 9,800,000 450,000 180,000

ii) The patent represents a license to produce and sell a special product. This product is
expected to generate a pre-tax profit of GH¢12,000 per annum in perpetuity.
iii) The discount rate of Agos Limited is 10% per annum.
iv) Nhyira Limited, a major competitor of Agos Limited, is listed on the Stock Exchange
and has a P/E ratio of 8 and a dividend yield of 10%.
v) Nhyira Limited expects a return of 11% of the net assets.
Required:
Estimate the value per share of Agos Limited as at 30 June, 2016 using the following
methods:
i) Net Assets (4 marks)
ii) Dividend valuation (4 marks)
iii) Price/earnings ratio (4 marks)

Note
1. You may assume that it is about 10% more risky investing in unlisted entity than
investing in listed entity.
2. Unless otherwise deemed inappropriate, the current year’s financial statements
(appropriately adjusted) may form the basis of the valuation.

(Total: 12 marks)
BUSINESS VALUATION: Question 2. (may 2016)
In 2016, the shareholders of Power Ltd decided to sell their equity stake in the company. The
company is not listed and the new shareholders plan to prepare the company for listing once the
acquisition was completed. The summarized financial statements of Power Ltd for the year
ended 30th June, 2016 are stated below:
Statement of Income for the year ended 30th June, 2016
GH¢
Profit before tax 24,800,000
Taxation (8,000,000)
Profit after tax 16,800,000
Dividends (3,200,000)
Retained Earnings 13,600,000

620
Statement of Financial Position as at 30th June, 2016
GH¢
Non-Current Assets 62,400,000
Current Assets 21,400,000
83,800,000

Current Liabilities 12,800,000


Long Term Liabilities 35,000,000
47,800,000

Net Assets 36,000,000

Stated Capital 20,000,000


Retained Earnings 16,000,000
36,000,000
The following additional information is provided;
1) The discounted present value of future cash payments in respect of the long-term loan is
GH¢48,800,000.
2) The stated capital of Depot Ltd is made up of 25,000,000 ordinary shares of no par
value.
3) Current Assets include inventory of GH¢6,600,000 representing goods received from a
major supplier on "not for sale but display only" basis.
4) The fair value of the tangible non-current assets was GH¢96,000,000.
5) The profit for the current year includes VAT of 17.5% on turnover of GH¢8,500,000
being invoice amount sold to a customer.
6) The discount rate of Depot Ltd is 10% per annum.
7) Warehouse Ltd, a major competitor of Depot Ltd is listed with a P/E ratio of 9 and
dividend yield of 5.2.
8) Profits after tax over the 4 years were as follows;
GH¢
30th June 2015 12,000,000
30th June 2014 14,400,000
30th June 2013 6,400,000
30th June 2012 11,200,000

Required:
Compute the value to be placed on the ordinary shares using three methods of valuation and
advise the Directors accordingly.

Question 3
In February 2016 the shareholders of Adu Ltd a private company, decided to list on the Ghana
stock exchange to make a public offer of its shares. The financial statements of the company for
the year 2016 are given below:

Statement of Comprehensive Income for the year-ended 28 February, 2016

621
GH¢
Turnover 212,600
Cost of sales 58,900
Gross profit 153,700
Selling, general & administrative expenses 86,400
Profit before taxation 67,300
Taxation 12,300
Profit after taxation 55,000
Proposed dividend 9,000
Retained profit 46,000

Statement of Financial Position at 28 February 2016


GH¢
Patent 40,000
Tangible Asset 236,000
Inventory 26,520
Receivables 25,800
Bank and cash 7,200
Trade payables (11,600)
Accred charges (1,600)
18% Debenture (13,000)
309,320
Equity
Stated capital 200,000
Share deals 25,820
Retained earnings 83,500
309,320
Additional information
1. The stated capital of Adu Ltd is made up of 2,500 ordinary shares of no par value
2. the fair value of the tangible assets has been estimated at GH¢440,000 by an independent valuer,
valuation charges of 2.5% have not been accrued for the above accounts .
3. The inventory include obsolete items worth GH¢2,200 being hold despite persistent advice by
the auditors to have them written off.
4. Receivables include an amount of GH¢15,000 resulting from the bankruptcy of a major
customer. Adu Ltd is not likely to realize any amount from this but the directors have refused to
make any provisions
5. The patents represent a right to sell a special product. This product is expected to generate cash
flows of GH¢1,500 per annum indefinitely.
6. The discounted present value of future cash payments in respect of the debenture is GH¢16,500.
7. Profits after tax of Adu Ltd over the past four years were as follows:
Year 2015 2014 2013 2012
Profits (GH¢) 36,000 30,000 28,000 34,000
8. A corporate plan prepared by the directors of Adu Ltd in 2015 included the following positions
Year to 30 June Profit after tax Depreciation
GH¢ GH¢
2015 38,000 5,600

622
2016 41,500 8,300
2017 40,000 10,000
2018 43,000 12,000
2019 46,000 15,000
2020 50,000 16,000
2021 55,000 18,000
9. The price-earning ratio and dividend yield of quoted companies in the same line in the same
industrial sector in which Adu Ltd operate are 8 and 4.5%
10. the net assets of Adu Ltd as at 28 February 2015 was GH¢251,100
11. The cost of capital of Adu Ltd is 20%

Required
Using any four methods of valuation advise the directors of Adu Ltd on the value to be
place on the ordinary shares.

QUESTION 4
POWER Limited operates a large-scale commercial farm. It now plans to off-load some of its
shareholding to the public in order to raise funds to expand its operations.
Financial statements of POWER Limited are as follows:
Statement of Comprehensive Income for the year-ended 31st December, 2016
GH¢
Turnover 245,800
Cost of sales 117,300
Gross profit 128,500
Selling, general & administrative expenses 87,140
Earnings before interest & tax 41,360
Interest 3,360
Profit before taxation 38,000
Taxation at 25% 9,500
Profit after tax 28,500

Income Surplus Account for the year ended 31st December 2016
GH¢
Balance at 1/1/2015 95,940
Profit for the year 28,500
Dividends paid (12,400)
Balance at 31/12/2016 112,040

Statement of Financial Position as at 31st


December, 2016 GH¢ GH¢

623
Property, plant & equipment 197,500
Patents 16,400
Development expenditure 26,100
240,000
Current Assets
Inventories 43,400
Receivables 25,002
Bank and cash 11,888
80,290
Current Liabilities
Payables 30,800
Net Current assets 49,490
289,490
14% Medium-term loan (24,000)
Net assets 265,490

Financed By:
Stated capital 100,000
Capital surplus 53,450
Income surplus 112,040
265,490
Additional Information:
(i) Stated Capital is made up as follows:
GH¢
Ordinary shares (issued @ GH¢0.20 each) 80,000
22% Irredeemable Preference Shares 20,000
100,000
(ii) A review of the development expenditure revealed that 60% of it is worthless.
(iii) An independent valuer has placed values on some of the assets, detailed as follows:

GH¢
Property, plant & equipment 222,000
Inventories 32,400
Receivables 20,000

(iv) Profit forecasts for the next five years are as follows:
Profit before tax Depreciation Charge
GH¢ GH¢
2017 29,800 2,200
2018 32,000 2,450
2019 38,500 3,100
2020 39,600 4,050
2021 43,100 4,260

The estimated profit before tax figures are arrived at after charging the estimated depreciation.

624
(v) Yam Limited is a competitor listed on the Ghana Stock Exchange and data extracted from its
recently published statements revealed the following:

 Market capitalisation = GH¢2,000,000


 Number of ordinary shares = 800,000
 Earnings per share = GH¢0.20 (20 pesewas)
 Dividend payout ratio = 80%

(vi) The patents represent a license to produce an improved variety of potatoes and is expected to
generate a pre-tax profit of GH¢20,000 per year for the next five years.

(vii) The cost of capital of POWER Limited is 18%.

Required:
(a) Determine the value to be placed on each share of POWER Limited using the following
methods of valuation:

(i) Net assets


(ii) Prices-earnings ratio
(iii) Dividend yield
(iv) Discounted cash flow
(12 marks)
The discount factors and annuity at 18% for the relevant years are as follows:
Year 1 2 3 4 5
Discount factor 0.847 0.718 0.609 0.516 0.437
Annuity 0.847 1.565 2.174 2.690 3.127
(b) Outline any three (3) reasons why the dividend yield of Yam Ltd should be adjusted before it
is used to value the shares of POWER Limited. (3 marks)
(Total: 15 marks)
Question 5 )
The shareholders of Wunam Bank (Ghana) Limited, have decided to sell the company to GCC
Bank (Ghana) Limited following their inability to recapitalize the company as being demanded
by the Bank of Ghana. The statement of financial positions of the two banks as at 31st March
2018 are given below.
GCC Bank Ltd Wunam Bank
GH¢000 GH¢000

Cash and balances with other banks (Note (vi)) 43,000 6,250
Investments (Note (ii))
 64,250 13000
Loan & advances (Note (i))
 115,100 16,700
Other assets (Note (iii)) 3,150 4,250
Property, Plant & Equipment (Note (v)) 14,300 8,650
239,800 48,850
Deposits and Current Accounts (Note (iv)) 191,100 37,750
Other liabilities (Note (vii)) 4,050 11,000
195,150 48,750

625
Stated Capital (Note (x)) 22,500 2,500
Statutory Reserve Fund 5,100 900
Retained Earnings 17,050 (3,300)
44,650 100
239,800 48,850

The following additional information relate to Wunam Bank Ltd;


i) Wunam Bank Ltd. carries a huge non-performing loan portfolio. It is estimated that only 40%
of the outstanding loans are recoverable. 

ii) Investments represent91-DayTreasuryBills held as secondary reserves. An audit has shown
that the investments were overstated in 2017 as interest on investments for that year amounts to
GH¢4.15 million. 

iii) Other assets include long outstanding debits amounting to GH¢3.6 million. These are not
represented by tangible assets. 

iv) Deposits amounting to GH¢3.75 million could not be accounted for. This phenomenon has
prevailed since 2014 but has not been provided for in the accounts. 

v) Property, plant and equipment include an old banking software amounting to GH¢1.25
million. This is considered worthless. The remaining tangible fixed assets have been revalued at
GH¢15.3 million. 

vi) Cash and balances with other banks include an amount of GH¢2.4 million due from Sakara
Rural Bank Ltd which was liquidated in 2016. 

vii) Other liabilities include interest earned on investments amounting to GH¢3.15 million. 

viii) Goodwill was assessed at 2.5% of adjusted deposits and current accounts.
. ix) Wunam Bank Ltd. has been investing each year in Government bonds to provide funds in
order 
to install Automatic Teller Machines and to open two ultra-modern branches in Takoradi
and Kumasi respectfully. This practice is only known to the Managing Director and the Finance
Manager, and the investments are worth GH¢12.6 million as at 31 March, 2018. 

. x) The stated capital of Wunam Bank Ltd. is made up of 100 million ordinary shares of no par
value. 

Required: 

. a) Identify FOUR (4) factors you would consider in determining the value to be placed on assets

when using the net assets approach to valuation of Wunam Bank Ltd. (4 marks) 

. b) Determine the value to be placed on the shares of Wunam Bank Limited using the net assets

approach to valuation. (5 marks) 

. c) Prepare the statement of financial position of GCC Bank (Ghana) Limited after the takeover
using your answer in (b) above. Assume the following: The purchase consideration was duly
settled; GCC Bank Ltd. took over all assets and liabilities; and Goodwill was written off. 

(6 marks) (Total: 15 marks) 


solu
QUESTION THREE
a) Factors to consider in determining the value to be placed on assets.
i) Consideration of whether or not the assets should be professionally valued. 


626
ii) The existence of hidden liabilities such as contingent liabilities, deferred taxes, 
redundancy
payments, among others 

iii) The realisability of receivables such as debtors and other bills receivables. Also consider the
recoverable amount of inventories. 

iv) Consideration of whether or not the assets can be separately disposed of in the open market.

v) Consideration of whether or not there are prior charges on any of the assets. 

vi) The specific valuation basis to use whether it should be going concern basis, replacement cost
basis, breakup basis. 

(b)
(Any four points for 1 mark each up to a maximum of 4 marks)

GH¢000
Loans and Advances (40% x 16,700) 6,680
Investment (13,000 – 4,150) 8,850
Other Assets (4,250 – 3,600) 650
Deposit and Current Assets (37,750 + 3,750) (41,500)

Property, Plant & Equipment 15,300

Cash and Balances (6,250 – 2,400) 3,850

Other Liabilities (11,000 – 3,150) (7,850)

Interest Receivable 3,150

Goodwill (25% of 41,500) 1,037.5

Investment in Govt. Bonds 12,600


2,767.5
Total value of company

Price of a share GH¢2,767,500/100,000,000 =GH¢0.0277


(10 ticks x 0.50 mark per tick = 5 marks)
(c) GCC BANK LIMITED
Statement of financial position as at 31/03/18 merger or
amalgamation

GH¢000

Cash balance with other banks (43,000 + 3,850-2,767.5) 44,082.5

Investments (64,250 + 8,850) 73,100

627
Investment in government bonds 12,600

Loans & advances (115,100 + 6,680)


121,780

Other assets (3,150 + 650) 3,800

29,600
Property plant & equipment (14,300+ 15,300)

284,962.5

Deposit and current accounts (191,100+ 41,500) 232,600

Other liabilities (4,050 + 7,850) 11,900

Total liabilities 244,500


Stated capital
22,500

Statutory reserve fund 5,100

12,862.5
Income Surplus (17,050 – 3,150 – 1,037.5)

40,462.5
Total equity

Total liabilities and equity 284,962.5

Question 6

The Board of Pogas Furniture Ltd (PFC) after few years of incorporation has decided to get the
company listed on the Ghana Stock Exchange. The Board has contacted you to assist in
determining the true value of the business as at 31 December 2018 and to provide a range of
possible issue price based on Net Assets and Earnings Yield. Oliso Ltd, a listed company and a
competitor of PFC, current results show price-earnings ratio of 5 and earnings yield of 20%. The
summarised unaudited financial statements of PFC are as follows:

Statement of profit or Loss for the year ended 31 December 2018


GH¢’000
Sales Revenue (note i) 150,000
Cost of Sales (72,000)
Gross Profit 78,000

628
Operational Expenses (34,800)
Finance Costs (Interest on debenture stocks) (1,200)
Net Profit 42,000
Taxation (@ 25%) (10,500)
Profit for the period 31,500

Statement of financial position as at 31 December 2018


Assets
GH¢’000
Non-current assets (note ii)
Property at Valuation (Land GH¢3 million; buildings GH¢ 27 million) (ii) 30,000
Plant and Equipment (ii) 24,000
Intangible Asset – Patent Right (ii) 3,000
Financial Asset (fair valued through profit or loss at 1/1/2018) (iii) 7,500
64,500
Current Assets (note iv) 30,000
Total Assets 94,500

Equity and Liabilities


Stated Capital (4 million shares issued at GH¢3.00 per share) 12,000
Retained Earnings 57,960
69,960

Non-current liabilities
20% Debenture Stocks (2018-2020) 6,000
Deferred Tax provision -1 January 2018 (note v) 4,500

Current liabilities
Trade Payables 3,540
Current Tax liability 10,500
Total Equity and Liabilities 94,500

Your examination of the financial statements and the underlying records revealed the
following additional information:
i) The sales revenue includes GH¢24 million of revenue for credit sales made on a 'sale
or return' basis. At 31 December 2018, customers who had not paid for the goods, had
the right to return GH¢7.8 million of them. PFC applied a markup on cost of 30% on all
these sales. In the past, PFC‟s customers have sometimes returned goods under this
type of agreement.

ii) The depreciable non-current assets have not been depreciated for the year ended 31
December 2018.

year. The values in the above statement of financial position are as at 1 January 2018
when the buildings had a remaining life of 18 years. A qualified surveyor has valued the
land and buildings at 31 December 2018 at GH¢33 million.

629
basis. As at 31 December 2018, the value in use and the fair value less cost to sell were
assessed at GH¢21.3 million and GH¢20.25 million respectively.

expected to be used for 5 years after which the right of usage would have to be
renewed in January 2023.

iii) The financial assets at fair value through profit or loss are held in a fund whose value
changes directly in proportion to a specified market index. At 1 January 2018 the
relevant index was 240.0 and at 31 December 2018 the index was 259.2

iv) In late December 2018, the directors of PFC discovered a material fraud perpetrated
by the company's credit controller. Investigations revealed that a total of GH¢9 million of
the trade receivables (included in current assets) as shown in the statement of financial
position at 31 December 2018 had in fact been paid and the money had been stolen by
the credit controller. An analysis revealed that GH¢3 million had been stolen in the year
to 31 December 2017 with the rest being stolen in the current year. PFC is not insured
for this loss and it cannot be recovered from the credit controller since his where about
is unknown.

v) As at 31 December 2018, the company‟s taxable temporary differences had


increased to GH¢24 million. The deferred tax relating to the increase in the temporary
differences should be taken to profit or loss. The applicable corporate tax rate is
25%.The above figures do not include the estimated provision for current income tax on
the profit for the year ended 31 December 2018. After allowing for any adjustments
required in items (i) to (iv), the directors have estimated the provision of current tax
liability for 2018 at 25% of adjusted profit. (This is in addition to the deferred tax effects
of item (v)).

(Note: Assume that it is about 20% riskier in investing in a non-listed entity (as compared
with a listed entity.)

Required
a) Redraft the financial statements above (taking into consideration the additional
information (i) – (v) above.
(11 marks)
b) Based on the revised financial statements, provide a range of possible issue prices per
share using Net Assets Method and Earnings Yield/Price Earnings Ratio Method.
(4 marks)

Solutions

(PFC) Revised Financial Statement


Statement of profit or Loss for the year ended 31 December 2018
GH¢‟000
Sales Revenue (150,000-7,800) 142,200

630
Cost of sales (72,000-6,000 (W1) (66,000)
Gross Profit 76,200
Operational Expenses (34,800)
Depreciation/Amortization (3,000+600+1,500) (W2) (5,100)
Bad debts (9,000-3,000) (6,000)
Fair valuation gains of FA (W3) 600
Finance costs (Interest on debenture stocks) (1,200)
Net Profit 29,700
Current Taxation (@ 25%) (7,425)
Deferred Tax (1,500) (8,925)
Net Profit for the period 20,775
OCI 4,500
Total Comprehensive Income 25,275

Statement of financial position as at 31 December 2018


Assets GH¢’000
Non-current assets (note ii)
Property at valuation 33,000
Plant and equipment (24,000 -3,000) 21,000
Intangible Asset – Patent Right (3,000 – 600) 2,400
Financial asset (fair valued through profit or loss) W3 8,100
64,500
Current Assets (30,000 – 7,800 + 6,000 – 9,000) 19,200
Total Assets 83,700

Equity and Liabilities


Stated Capital (4 million shares issued at GH¢3.00 per share) 12,000
Retained Earnings W4 44,235
Revaluation Surplus (30,000 – 25,500) 4,500
60,735
Non-current liabilities
20% Debenture Stocks (2018-2020) 6,000
Deferred Tax provision - 31 December 2018 (25% of 24,000) 6,000
Current liabilities
Trade Payables 3,540
Current Tax liability 7,425
Total Equity and Liabilities 83,700

WORKINGS
W1 Cost of Sales
Sale or return
Dr Revenue 7,800
Cr Current Assets 7,800
Mark up = 30%
130% = 7,800
100/130 * 7,800 = 6,000

Dr Current Assets 6,000


Cr Cost of Sales 6,000

631
W2 Depreciation/Amortisation
Property
Land 3,000
Building 27,000
27,000/18 years = 1,500 depreciation
Plant and equipment
12.5% on reducing balance
=0.125*24,000 =3,000
Amortisation of Intangible asset
=3,000/5 years = 600

W3 Fair value through profit or loss


Index, If 240.0 = 7,500
259.2 = ?
259.2/240 * 7,500 = 8,100
Fair value gain = 8,100-7,500 =600
W4 Retained Earnings
As per balance sheet = 57,960
Profit for the year as per profit or loss = (31,500)
Prior year retained earnings 26,460
Less prior year fraud 3,000
23,460
Add profit for the year 2018 20,775
44,235

b) Share Valuation

(i) Net Assets Basis


Value of a share = Net Assets/No. of shares
= GH¢60,735,000/4 million shares
= GH¢15.183
(2 marks)

(ii) Earnings Yield


Value of a share = EPS/Earnings Yield
EPS = GH¢20,775,000/4 million shares
= GH¢5.194
Earnings Yield = PFC (20%) discounted to say 24%
Value of a share = GH¢5.194/0.24
= GH¢21.63

WHEN VALUATIONS ARE REQUIRED


A share valuation will be necessary:

632
a. For quoted companies, when there is a takeover bid. A takeover is the acquisition by a
company of a controlling interest in the voting share capital of another company, usually
achieved by the purchase of a majority of the voting shares.
b. For unquoted companies, when:
 The company wishes to go public and must fix a price for its shares
 There is a scheme of merger
 Shares are sold
 Shares need to be valued for the purpose of taxation
 Shares are pledged as collateral for a loan
c. For any company, where a shareholder wishes to dispose of his or her shareholding. Some of the
valuation methods we describe will be most appropriate if a large or controlling interest is being
sold
d. For any company, when the company is being broken up in a liquidation situation or the
company needs to obtain additional finance or re-finance current debt.
e. During the complete Sale of company

REASONS WHY THE DIVIDEND YIELD/PE RATIO OF A SIMILAR LISTED


COMPANY NEED TO BE ADJUSTED
 The shares of the listed company are more marketable compared to private unlisted companies
 Quoted companies often have more certain dividend policy than private unlisted companies
 Investments in quoted companies as less risky than in private unlisted companies since they are
more diversified
 Quoted companies are often bigger in size as compared to private unlisted companies
 The level of gearing may differ for listed and unlisted companies
 Other specific factors may need to be considered.
 The insecurity of the earnings of the unquoted company

633
CAPITAL REORGANIZATION
Introduction
Running business activities and hence, corporate bodies with a convenient level of efficacy
seems to be any good management teams desire. However, it is very common for many
businesses to face challenges which often prevent them from achieving their goals. So many
businesses could only bounce back onto the winning ways after bold financial repairs had been
made, and of course reorganization and reconstruction of business activities have been embarked
upon in one form or other. Capital Readers will also learn of how to suggest a scheme when the
business is torn between intending to fold up and repackaging the capital structure to remain a
going concern.

THEORETICAL AND LEGAL FRAMEWORK


Definitions
The terms capital reduction, capital reorganization and reconstruction are often used
synonymously referring to a situation where a company reorganizes its capital structure usually
after persistent losses has resulted into Retained earnings deficit and liquidity problems.

Capital re-organization involves restructuring the activities, operations and or capital structure
of a company internally without the involvement of another company. The organization therefore
maintains its original identity but with different activities and structures. It may include any of
the following:
• Management restructuring which involves repackaging of the management
team to make it more competent and effective. This may consist of transfers of management staff
to different location, injection of fresh talents and dropping of non-performing members.

• Capital reduction may refer to a scheme of cancellation of capital unrepresented


by available assets or a repayment of capital to members. Thus it amounts to write off of capital
to a reduced amount. The company‟s code allows a company to reduce its capital in any way it
feels appropriate. Specific instances requiring capital reduction include:
o Share capital which is lost or unrepresented by available assets may be cancelled e.g. debit balance
on the retained earnings account.
o Share capital in excess of the needs of the company may be repaid e.g. over capitalization or
surplus cash.
o A company may extinguish or reduce liability on any of its shares in respect of share capital not
paid.

Capital reconstruction:
This usually involves a new organization with a completely new capital structure taking over,
absorbing or merging with the activities of the ailing company.

Theoretical Framework
Capital reorganization may be undertaken after a strategic document duly prepared by
management is submitted to stakeholders for deliberation. After shareholders are satisfied with
the strategic document then they may vote for scheme of capital reduction to be implemented.
After voting for the scheme, the scheme will go through certain legal formalities.

634
Conditions for successful capital reduction scheme
for shareholders to vote in support of a capital reduction scheme, certain conditions must be met.
■ Essential: The scheme must be essential, that is there is a real need for it and
without it the company cannot run.

■ Equitable: The scheme should treat all stakeholders equitably. The major loss
will be borne by equity shareholders.

• Sufficient working capital: There should be provision for sufficient working


capital. For this purpose, calls may be made, new shares may be issued, and overdraft
arrangement may be made.

• Existing control: Existing control should remain.

• Sufficient prospects: There should be sufficient prospects for each group that is called
upon to make a sacrifice.

• Onerous liabilities and pressing loans: There should be possibilities for paying off or
conversion of onerous liabilities and pressing loans, otherwise the company will soon find
itself meeting high charges as well as shortage of working capital.

• Turned the corner: The Company must have turned the corner. In other words
the difficult period is over and there should be fair profits.

Legal formalities for a capital reduction scheme


Before a company can reduce its share capital, the following legal formalities must be met.
• An extraordinary meeting of shareholders to pass a special resolution for the
reduction scheme
• The resolution must be approved by a high court
• Consent of other stakeholders.
• The scheme must be gazetted.
The companies code provision on resolution to reduce capital
Capital reduction is closely controlled by the code under paragraph of the Companies‟ Code Act
992, 2019 and requires confirmation by the court subject to the approval of the court, a company
limited by shares may pass a special resolution to:
• Reduce its stated capital in any way
• Extinguish or reduce the unpaid liability on any of its shares
• Resolve to pay or return to its stakeholders any of its shares which are in excess
of the wants of the company
• Alter the regulations by cancelling any of its shares.

Reorganization process
Reconstruction process may involve all or some of the following activities
 Writing off debit balance of Retained earnings

 Writing of all intangible and fictitious assets except where future values can be established.

635
 Creation of a new share capital of a different value from the existing share capital or cancellation
of existing share capital.
• The conversion of debt to equity and debt of one class to debt of another.
• Revaluation of assets to reflect their fair value
• Calling on other stakeholders to make some sacrifices where (necessary)
• Raising additional equity capital to improve upon the liquidity situation
• Closing down loss-making plant, delisting unprofitable products or introducing
potentially viable products.

Reasons for capital reduction


Reconstruction may be introduced to attain one or more of the following;

 To raise additional finance by issuing partly paid shares in a new company to exchange for fully
paid shares in the old. Any uncalled capital must be called up as required.
 To take new powers in the regulations.
 Write off over valued assets. Or a large debit income surplus account, or cancel assets not
represented by assets.
 To arrange the rights of the proprietors as to capital and income
 To pave way for the payment of dividends.
 To compromise with creditors
 To return the company to its original position.

ACCOUNTING ARRANGEMENTS FOR IMPLEMENTING A CAPITAL REDUCTION


SCHEMES

Introduction
When the scheme is approved by the court, it must then be implemented by the accountant.
Capital‟ reduction scheme could be effected through journal entry or ledger accounts. The
principal accounts to open include:
 Capital Reduction Account
 Bank Account
 Ordinary share Capital Account
 Preference share Capital Account
A new statement of financial position may also be prepared using the new values of assets and
liabilities.
The process may involve the following steps:
• Write off debit Retained earnings Balance
Debit Capital Reduction Account
Credit Retained earnings Account
With debit balance of Retained earnings written off
• Write off intangible assets unless their future values can be guaranteed. These
assets will include Goodwill, Development expenditure, deferred preliminary costs.
Debit Capital Reduction Account
Credit Intangible Assets Account With the amount on intangible assets written off
• Reorganization expenses

636
Debit Capital Reduction Account
Credit: Bank Account
• Dividend in Arrears not yet accounted for
Debit Capital Reduction Account
Credit Dividend in Arrears Account With any preference dividend in arrears
• Utilize capital surplus and any reserves to reduce amount of loss.
Debit Capital surplus or reserve account
Credit Capital Reduction Account With the amount of reserves utilized
• Reduction in Ordinary & Preference share capital, loans and other creditors
Debit Ordinary & Preference share capital, loans etc. account
Credit Capital Reduction Account
With the amount of sacrifices made by the various stakeholders
• Downward valuation of assets
Debit Capital Reduction Account
Credit Assets Account
With any under valuation of assets ■ Valuation of assets upwards
Debit Assets Account
Credit Capital Reduction Account
With any upwards valuation of assets
• Overvalued assets
Debit Capital Reduction Account
Credit Assets Account
With amount of over valuation (e.g. bad debts, stock write off etc)

A SAMPLE OF CAPITAL REDUCTION ACCOUNT


Capital Reduction Account
GH¢ GH¢
Revaluation loss on PPE XX Reduction in Ordinary shares XX
Intangible assets written off XX Reduction in Preference shares XX
Settlement of pref. dividend in arrears xx Revaluation surplus on PPE XX
Debit balance on retained earnings XX Discount received XX
Bad debts written off XX Capital surplus utilized XX
Obsolete inventory XX Credit balance on retained earnings XX
Interest on loans settled XX
Reorganisation cost XX XX

Note
This account is supposed to balance. Any difference is added to or deducted from equity.

Activity 1
Doma Ltd, which had experienced trading difficulties, decided to reorganize its finances. On 31
December 2014, a final trial balance extracted from the books showed the following position.
Debit Credit

637
GH¢ GH¢
Share capital, authorized and issued:
150,0000 6% cumulative preference
Share of GH¢1 each 150,000
200,000 ordinary shares of GH4 1 each. 200,000
Capital surplus account 40,000
Retained earnings account 114,375
Preliminary expenses 7,250
Goodwill (at cost) 55,000
Trade payables 43,500
Trade receivables 31,200
Bank overdraft 51,000
Leasehold property (at cost) 80,000 Provision 30,000
for depreciation
Plant and machinery (at cost) 210,000
Provision for depreciation 62,500
Inventory in hand 79,175
577,000 577.000
Additional information
Approval of the court was obtained for the following scheme for reduction of capital:
1. The preference shares to be reduced to GH¢0.75 per share
2. The ordinary shares to be reduced to GH¢0.125 per share
3. One GH¢0.125 ordinary share to be issued for each GH41 of gross preference dividend in
arrears; the preference dividend had not been paid for three years.
4. The balance on capital surplus account to be utilized
5. Plant and machinery to be written down to GH¢75,000
6. The Retained earnings balance and all intangible assets to be written off.
7. At the same time as the resolution to reduce capital was passed, another resolution was approved
restoring the total authorized capital to GH¢350,000 consisting of 150,000 6 per cent cumulative
preference share of GH¢0.75 each and the balance in ordinary shares of GH¢0.125 each. As soon
as the above resolution had been passed, 500,000 ordinary shares of no par value were issued, for
cash payable in full upon application
You are required:
a) To show the journal entries necessary to record the above transactions in the company‟s
books; and
b) To prepare a statement of financial of the company, after completion of the scheme.

Activity solution
Doma Ltd
Journal entries (narrations omitted)
DR CR
GH¢ GH¢
Preference share capital 37,500
Ordinary share capital 175,000
Capital reduction 212,500
Preference shares reduced by 25p each (0.25 x 150,000) &

638
Ordinary shares by 87.5p each (0.875 x 200,000).
Capital reduction 3,375
Ordinary share capital 3,375
Ordinary shares issued for preference dividend
in arrears, (27,000+ x 0.125)

639
Capital surplus 40,000
Capital reduction 40,000
Capital surplus balance utilized.

Provision for depreciation 62,500


Capital reduction 72,500
Plant and machinery 135,000
Plant and machinery written down to 75,000
Capital reduction 176,625
Retained earnings 114,375
Preliminary expenses 7,250
Goodwill 55,000
Retained earnings balance and intangible assets written off
Cash 62,500
Ordinary share applicants* 62,500
Applications for shares 500,000 x 0.125
Ordinary share applicants* 62,500
Ordinary share capital 62,500
500,000 ordinary shares issued
Notes
+Preference dividend in arrears for three years:
Annual preference dividends (GH¢150,000 x 6%) = GH¢9,000
For three years (GH¢150,000 x 6% x 3 years) = GH¢27,000,
*.
The “unwaived” portion of the total preference dividends in arrears amounts to the proportion for
which ordinary shares would be issued and which is GH¢3,375 (GH¢27,000 x 0.125). The rest
represents the portion of the dividends waived by the preference shareholders and which
simultaneously represents cost because of the dividends crystallizing and income because of the
waiver, hence they cancel off and do not show up in the records.
However, an alternative treatment that could be followed for the dividends is as follows
• First debit the capital reduction account and credit preference dividends in
arrears account with the full amount of GH¢27,000
• Next, debit preference dividends in arrears and credit capital reduction account
with the income portion of the dividends (which is the portion waived by the shareholders)
leaving GH¢3,375 at the debit side and credit-side of the capital reduction account and
preference dividends in arrears account respectively
■ The balance on the preference dividends in arrears account is transferred to ordinary share
capital account by debiting the former and crediting the latter.
*This account could be ignored since it serves as a transition so that the ordinary share capital is
directly credited.

Doma Ltd
Statement of financial position as at 31 December 2014
GH¢ GH¢
Non-current assets
Leasehold property at cost 80,000

Page | 640
Less Provision for depreciation 30,000 50,000
Plant and equipment 75,000
125,000
Current assets
Stock 79,175
Receivables 31,200
Bank 11,500
(62,500 cash issue - 51,000 O/D) 121,875
Less: Current liabilities:
Creditors 43,500
Working capital 78,375
203,375
Financed by
Preference share capital (150,000x0.75) 112,500
Ordinary shares (727,000 x 0.125 see below) 90,875
203,375
Working
Ordinary share capital
Old balance not written off (200,000 x 0.125) 25,000
Preference dividends converted into shares 3,375
(27,000x0.125)
Cash issue of shares (500,000 x 0.125) 62,500
90,875
Number of ordinary shares required to be in issue 727,000
(90,875/0.125)

QUESTION 2 (nov 2016) CAPITAL REDUCTION ACCOUNT


MM Ltd, producers of telecommunication equipment has been making losses in recent times.
The directors have proposed a scheme of reorganization, to take effect on 1 October 2013. The
statement of financial position of the company at 30 September 2013 is as follows:
Statement of Financial Position as at 30 September 2013
Non-current assets GH¢
Premises 80,000
Plant & Equipment 190,000
Vehicles 30,000
300,000
Current assets
Inventory 40,000
Trade receivables 30,000
Bank 10,000
80,000
Current liabilities
Trade payables (140,000)
(60,000)
Net assets 240,000

Page | 641
Financed by:
Stated capital
Issued and fully paid
100,000 7% Preference shares of GH¢1 Per share 100,000
400,000 ordinary shares of GH¢0.75 fully paid 300,000
400,000
Retained Earnings (160,000)
240,000
Additional information.
i) The ordinary shares are to be written down to GH¢0.25 per share and then to be converted into
new ordinary shares of GH¢1.0 per fully paid.
ii) The preference shareholders are to receive 40,000 ordinary shares of GH¢1 per share, fully
paid in exchange for their preference shares.
iii) Dividends of 7% preference shares are two years in arrears. In consideration of waiving their
rights to arrears of preference dividend, the Preference shareholders have agreed to accept
10,000 new ordinary shares of GH¢1.00 per share, fully paid in final settlement.
iv) The creditors have agreed to take 100,000 new ordinary shares of GH¢1 per share fully paid
in part settlement of the amounts due them.
v) The balance on the retain earnings account is to be written off.
vi) Some assets of the company have been revalued and are to be incorporated into the accounts
as follows:
GH¢
100,000
125,000
25,000
36,000
vii) An allowance of GH¢3,500 is to be made for doubtful debts
viii) The ordinary shareholders have agreed to inject additional GH¢90,000 cash by acquiring
120,000 ordinary shares at 0.75 per share fully paid.
ix) Reorganization costs amounted to GH¢7, 500.
Required:
a) Prepare a capital reduction account, stated capital account and bank account (9 marks)
b) Prepare a statement of financial position of MM Ltd as at 1 October 2016, after the
reorganization (6 marks)

Question 3
Steve Ltd, producers of telecommunication equipment has been making losses in recent times.
The directors have proposed a scheme of reorganization, to take effect on 1 October 2014. The
statement of financial position of the company as at 30 September 2014 is as follows:
Non-current assets Cost Accum. Net
Dept. Book Value
GH¢ GH¢ GH¢
Premises 100,000 20,000 80,000
Plant & Equipment 250,000 60,000 190,000
Vehicles 45,000 15,000 30,000
395,000 95,000 300,000

Page | 642
Current assets
Inventory 40,000
Trade receivables 30,000
Bank 10,000
80,000
Current liabilities
Trade payables (140,000)
(60,000)
Net Assets 240,000
Financed by:
Stated capital Issued and fully paid
100,000 7% preference share GH¢1 each 100,000
400,000 ordinary shares GH¢1each 300,000
400,000
Retained earnings (160,000)
240,000
Additional information
1. The ordinary shares are to be written down to GH¢0.25 per share and then to be converted into
new ordinary shares of GH¢1per fully paid.
2. The preference shareholders are to receive 40,000 ordinary shares of GH¢ 1 per share, fully paid
in exchange for their preference shares.
3. Dividends of 7%preference shares are two years in arrears. In consideration of waiving their
rights to arrears of preference dividend, the Preference shareholders have agreed to accept
10,000 new ordinary shares of GH¢l per share, fully paid in final settlement.
4. The creditors have agreed to take 100,000 new ordinary shares of GH¢1 per share fully paid in
part settlement of the amounts due them.
5. The balance on the Retained earnings account is to be written off.
6. Some assets of the company have been revalued and are to be incorporated into the accounts as
follows:
GH¢
Freehold premises 100,000
Plant and equipment 125,000
Vehicles 25,000
Inventories 36,000

7. An allowance of GH¢3,500 is to be made for doubtful debts


8. The ordinary shareholders have agreed to inject additional GH¢90.000 cash by acquiring
120,000 ordinary shares at GH¢0.75 per share fully paid.
9. Reorganization costs amounted to GH¢7,500.

Required:
a. Prepare a capital reduction account, stated capital account and bank account
b. Prepare a statement of financial position of Steve Ltd after the reorganization

Page | 643
QUESTION 4
Sasasila Ltd has been operating profitably for a number of years now. However, in recent times,
the company has been making losses. Find below the statement of financial position as at 30
June 2019.
Assets GH¢000
Non-Current Assets
Patents and copyrights 75,000
Land and buildings (net) 200,000
Plant and machinery (net) 150,000
425,000

Current Assets
Inventories 125,000
Trade receivables 125,000
Bank 37,500
Investments (cost) 100,000
387,500

Total Assets 812,500

Equity and liabilities:


Equity:
Ordinary share capital (issued at GH¢10 each) 375,000
20% cumulative preference shares (issued at GH¢10 each) 175,000
Retained earnings (75,000)
475,000

Non-current Liabilities
15% Debentures 125,000

Current Liabilities
Interest on debentures 18,750
Trade payables 93,750
Provision for business restructuring 50,000
Provision for legal damage & claims 12,500
Provision for warranties 37,500
212,500

Equity and liabilities 812,500

Additional relevant information:


The following scheme of reconstruction was approved by all parties as well as the High Court
with the exception of only one ordinary shareholder:
i) The ordinary shares were to be reduced to GH¢5 per share.
ii) The preference shares were to be reduced to GH¢7.5 per share and arrears in dividend thereof
for three years were to be cancelled from the company‟s books

Page | 644
iii)The fair values of the assets were agreed at the following values:
GH¢000
Patents and copyrights Nil
Land and buildings 225,000
Plant and machinery 75,000
Investments 75,000
Inventories 105,000
Trade receivables 70,000

iv) The balance on the retained earnings is to be eliminated in full


v) The liability for legal damages and claims was to be settled for GH¢10 million and the
provision for warranty reduced to GH¢27.5 million.
vi) The accrued debenture interest was to be paid in cash.
vii) Investment with a carrying amount of GH¢52.5 million were to be sold for cash at that value
in desperation to strengthen the working capital position of the business.
The amount set aside for business restructuring was to be eliminated also.
viii)The High Court directed a payment of GH¢0.2 million to a member who opposed the
scheme for 50 ordinary shares held by him. Mr. Salia a director of the company, has offered to
take over the shares
.ix)The following additional information is relevant: Sasasila Limited continued trading from 1
July 2019 to 31 December2019 which resulted in the following changes in assets and liabilities:

Increased by Decreased by
GH¢000 GH¢000
Bank and cash 10,000 -
Inventories 12,500 -
Trade and other receivables 20,000 -
Trade payables - 5,000

Plant and machinery is depreciated at 10% per annum and no depreciation is provided on land
and buildings. The half year debenture interest has not been paid.
Required:
a)Prepare the Capital Reduction Account as at 30 June2019. (6marks)
b)Prepare the statement of financial position as at 31 December2019. (14marks)

QUESTION 5 (may 2017)


Abusua Ltd. has been trading profitably for several years but for the past four years its operations
have resulted in losses. The board of directors has decided to restructure the company.
The Statement of Financial Position as at 30 September, 2016
GH¢000 GH¢000
Non- current assets
Freehold land and buildings 3,788
Plant and equipment 7,020
Furniture and fixtures 3,080
Investment 2,300
Deferred development expenditure 7,050

Page | 645
Patent rights 4,200
27,438
Current assets
Inventories 3,510
Trade receivables 1,600
Cash 428
5,538
Current liabilities
Bank loan 5,360
Trade payables 4,650
Sundry creditors 1,060
11,070 (5,532)
Net current liabilities
22% Debentures (7,875)
14,031
Financed by:
Share capital 17,625
Capital surplus 2,250
Income surplus (5,844)
14,031
You have been provided with the following additional information.
i) Abusua Ltd‟s share capital consists of GH¢000
Ordinary shares 13,500
20% Cumulative Preference shares 4,125
ii) No dividend was declared on the Preference shares for the year ended 30 September, 2016.
iii) The following assets have net realizable values as indicated below:
GH¢000
Freehold land and buildings 4,005
Plant and equipment 3,750
Furniture and fixtures 2,800
iv) The investment in Abusua Ltd. is 55% holding in Obi Ltd. An offer of GH¢1,350,000 has
been made for it and it has been accepted by the directors.
v) Following further feasibility study carried out on the project which gave rise to the deferred
development expenditure, the directors have decided to discontinue the project. The project is
not patented.
vi) The directors have decided to sell the patent rights for a net realizable value of
GH¢1,800,000.
vii) Inventories were written down by GH¢ 2,232,000.
viii) The 22% debentures are secured on the freehold land and buildings.
ix) The bank has a fixed and floating charge over the assets in respect of the loan.
x) It is considered that a proposed reconstruction of the company should result in net profit after
tax of GH¢1,500,000 in the year ending 30 September, 2017 and GH¢1,800,000 or more in each
of the years thereafter.
xi) The company will require a ratio of accounts receivable and cash to current liabilities of
0.80:1 in future to trade satisfactory.

Page | 646
Required:
As a Director of Finance of Abusua Ltd, recommend a scheme of reconstruction for
consideration by the board of directors of the company and prepare a summarized statement of a
financial position (15 marks)

Question 6
Stev Ltd was incorporated as a private company several years ago. Trading was profitability, but
in the last three years, trading losses have been incurred and the board is considering the
alternative of liquidation or capital reduction.

The statement of financial position of Stev Ltd as at 31st March 2018 is as follows:
Non current assets GH¢’m GH¢’m
Freehold property 392
Plant, machinery & Motor vehicle 105
Deferred development expenditure 349
846
Current assets
Inventories 202
Receivables 76
278
Current liabilities
Bank overdraft (floating charges) 210
Trade payables 320
Accrued expenses 60
590
Net current liabilities (312)
534
20% debenture ( secured on freehold property ) (380)
154
Financed by:
stated capital 888
Retained earnings (734)
154
Additional information
1. The company‟s freehold property was revalued on 31 January 2017. The directors believe that its
net disposal value at 30/4/2018 will be GH¢400 million.
2. It is estimated that the break-up value of plant, machinery and motor vehicle at 30/4/2018 will be
GH¢90 million.
3. The deferred development expenditure has been patented. it is estimated that if the company is
liquidated, the deferred development expenditure could be sold for GH¢200 million.
4. The inventory is estimated to realize its carrying amount in the event of liquidation.
5. One of the company customers become bankrupt in March 2018. Stev Ltd is likely to lose
GH¢10million of the amount due from that customer.
6. In the event of Stev Ltd. Going into liquidation, it is estimated that the liquidation cost will be
GH¢42 million.
7. Reorganization cost of the company is estimated at GH¢30 million.

Page | 647
8. The ordinary shareholders are prepared to bring in additional GH¢300 million capital if they
consider the proposed scheme satisfactory.
9. the bank overdraft limit is GH¢400 million
10. Preference dividends are two years in arrears
11. Stev Ltd‟s Stated Capital comprised of;
Ordinary share capital GH¢573 million
18% Cumulative Preference Share GH¢315 million
Required
Examine the two-alternative suggested above and advise the board on the alternative which is
likely to be acceptable to the ordinary and preference shareholders.

Question 6
Bank of Ghana (BoG) earlier last year announced an increase in the minimum capital
requirement for Micro Finance Institutions in the country from the current GH¢500,000 to GH¢2
million by June 2018. Capital Link, a Micro Finance Company has been affected by the increase
in players in the Micro Finance Industry which has seen a reduction of its loan portfolio and an
increase in loan default rate.
A statement of Financial Position recently prepared by Capital Link is as follows:
Statement of Financial Position as at 30th April, 2017
GH¢ GH¢
Non-current assets
Goodwill
 450,000
Leasehold building (Carrying Amount) 750,000
1,200,000
Current assets
Loan Portfolio
 825,000
Investment in Treasury bills 570,000
1,395,000
Current liabilities
Interest to depositors
 885,000
Bank overdraft (secured on Leasehold property) 975,000
1,860,000
Net current liabilities (465,000)
735,000
Financed by:
Share capital 600,000
General reserves 120,000
Retained Earnings 15,000
735,000
Additional information
Depositors have the information that Capital Link is considering funding options to ensure the
survival of the business. They are however not convinced that Management of Capital Link
would be able to raise the required capital to meet the minimum requirement.
In a Stakeholders meeting, Management of Capital Link proposed two possible options for the
company‟s future.

Page | 648
Scheme 1- Close Down Mission
Unity Capital Ltd has made a firm offer of GH¢600,000 for the company‟s leasehold property.
Total loan portfolio is conservatively valued at GH¢592,500 in a forced sale. Investments in
Treasury bills amounts to GH¢465,000. Liquidation expenses are estimated at GH¢30,000 and
interest to depositors is GH¢330,000.
Scheme 2- Rescue Mission
Management of Capital Link propose to implement a scheme which includes a cheap loan of
GH¢600,000 from a distress fund set up by an NGO, Business Rescuers. The following terms
have been arranged:
GH¢375,000 would be reserved to pay for the loan from the NGO when the first installment falls
due. The NGO has been asked to accept 20% debenture in exchange for the balance remaining
due. The debenture would be repayable in four annual installments of GH¢150,000, commencing
28 February 2017, with the interest due for the preceeding year, paid on the same date.
Assume that:
The current rate of interest on all borrowing is 14% 

The calculations are being made on 30th April, 2017 and either scheme could be put onto

effect immediately. 

he present value of GH¢1 recoverable at the end of each year is:
Year 1 Year 2 Year 3 Year 4
14% 0.88 0.77 0.67 0.59
20% 0.83 0.69 0.58 0.48
Required:
a) By means of numerical analysis of the two schemes, evaluate how much the bank would
recover from each scheme. (12 marks) 

b) Discuss TWO advantages of each scheme. (3 marks) (Total: 15 marks) 


Solu
Scheme 1

Immediate liquidation GH¢000 GH¢000
Sales proceeds: Leasehold
 600
Less: Bank overdraft – secured to the extent of 600

Sales proceeds: Loan Portfolio 592.50


Investments in T bills 465 1057.50

Less: Liquidation expenses 30


Interest to depositors 330 (360)
697.5
Unsecured creditors: Bank (975,000 – 600,000) 375
Other (885,000 – 330,000) 555
930
Unsecured creditors get for each GH¢ 1 owed: GH¢
GH¢1 × (697.5/930) = GH¢ 0.75

Page | 649
Bank therefore gets: Secured
 600,000
Unsecured 375,000 × 75p 281,250
881,250
Scheme 2 Rescue operation:
Received:

Amount received immediately
 375,000
Amount received at the end of each year;
1 150,000 + (600,000 × 20%) = 270.000 × 0.88 
 237,600
2 150,000 + (450,000 × 20%) = 240.000 × 0.77 
 184,800
3 150,000 + (300,000 × 20%) = 210.000 × 0.67 140,700 

4 150,000 + (150,000 × 20%) = 180.000 × 0.59 106,200
(12 marks evenly spread by way of ticks) 
 1044,300
b) Scheme 1
. The advantage of immediate liquidation is that the cash flows can be estimated with a reasonable
amount of accuracy, particularly as a firm offer has been received for the leasehold buildings. 

. On the face of it the valuations of the loan portfolio do not appear excessive, though the
possibility of them realizing as much as forecast needs to be considered. 

. The bank suffers a material loss on its investment, but nevertheless manages to recover 90% of
the amount owing. 
(Any 2 points for 1.5 marks) 

Scheme 2
. The advantage of this option is that the bank retains a customer, and avoids bad publicity
possibly associated with the closure of a local firm. 

. The computed financial advantage of this option is also a significant factor. However, there must
be considerable doubt concerning the achievement of these estimates. Is there any evidence that
the recent trading pattern has been reversed? 

. The nature of the arrangement with the Cheap Finance Company Ltd also needs to be explored.
If the bank is being offered 20% on its investment, the finance company is probably being
offered something higher. Also, what is the order of priority on repayment between the planned
loans from the finance company and the bank? 

. Can the company really expect to generate the cash flows necessary to service and finance
repayment of amounts due to both financial institutions? 

. Finally, what is the financial commitment of the directors in terms of further cash investment or
offering assets as security? 


NOV, 2019
Mahadi Ltd has operated profitably in Ghana for several years but now facing financial
difficulties after recording losses in its operations recently. The company‟s Statement of
Financial Position as at 30 September, 2019 is given below:
GH¢
Non-current Assets
Freehold property 68,000
Equipment 468,000
536,000

Page | 650
Current Assets
Inventories 120,000
656,000
Equity and Liabilities
Stated Capital (400,000 ordinary shares issued at 25 pesewas per share) 100,000
Capital surplus 68,000
Retained earnings (40,000)
128,000

10% debenture stocks 48,000


Sundry payables 412,000
Bank overdraft (from Northern Rock Bank) 68,000
656,000
i. Additional Information: i) Mahadi Ltd operates a number of retail outlets for snack bar; most
of these outlets are rented out. The company‟s largest supplier is Banda Ltd which holds all
of the debenture stocks and is also a trade creditor for GH¢240,000 included in sundry
payables above. Included in the sundry payables are preferential payables of GH¢44,000; an
amount due to the Ghana Revenue Authority.
ii. The bank overdraft from Northern Rock Bank is secured by a fixed charge over the freehold
property of Mahadi Ltd, and the debenture stock is secured by a floating charge over the
company‟s assets.
iii. On October 1, 2019, Mahadi Ltd has scheduled a meeting of all the stakeholders of the
company in order to consider the following alternative proposals:
Proposal Alternative 1
The management of Mahadi Ltd proposes for the immediate liquidation of the company, which
would result in the following estimated amounts for realized assets:
GH¢
Freehold property 56,000
Equipment 204,000
Inventories 40,000

Proposal Alternative 2:
Banda Ltd has made an offer of support which would allow the reconstruction of Mahadi Ltd to
continue to operate as a going concern. Under this reconstruction arrangement, the debentures
held by Banda Ltd would be converted into 48,000 ordinary shares (issued at GH¢1.00 per share)
and for every GH¢200.00 of GH¢240,000 trade debt owed to Banda Ltd, there would be issued
110 ordinary shares (issued at GH¢1.00 each) in the reconstructed company. The balance of the
trade debt owed to Banda Ltd would be written off against the equipment of Mahadi Ltd.
Mahadi Ltd existing shareholders would receive one ordinary share in the reconstructed
company (credited as to GH¢1.00 per share) in exchange for every five presently held. The
balance on the retained earnings is not absorbed by the existing shareholders (in the share
exchange above), and is to be written off against the capital surplus. The fair value costs of the
freehold property and the inventories approximate their carrying value.
An additional 140,000 ordinary shares in Mahadi Ltd (issued at GH¢1.00) would be issued to
Banda Ltd for cash consideration of GH¢140,000 in order to improve the liquidity position of the

Page | 651
reconstructed entity. It has been estimated by the Management of Banda Ltd that after
reconstruction, Mah
adi Ltd would earn regular net profit of GH¢54,000 per annum.
Other relevant information:

at Banda Ltd can currently earn 15% per annum on new investments involving
similar risks to those which would be incurred by investing in Mahadi Ltd.

Required:
a) Show the total amount Banda Ltd would receive in case of liquidation of Mahadi Ltd per
proposal 1. (10 marks)
b) Prepare the statement of financial position of Mahadi Ltd after the reconstruction (on
assumption that proposal 2 is adopted and implemented). (10 marks)
(Total: 20 marks)

Page | 652
SOLUTION
Sales Proceeds
Freehold property 56,000
Equipment 204,000
Inventories 40,000
300,000
Distribution:
Northern Rock Bank (Overdraft) [68,000 – 12,000] 56,000
Tax Authorities 44,000
Debentures stocks 48,000 148,000

Available for unsecured creditors 152,000


Analyzed as to:
Northern Rock Bank (O/D) [68,000 -56,000] 12,000
Sundry payables (412,000 – 44,000) 368,000 380,000

Dividend payout
152,000/380,000 = 40% or 40 pesewas in a GH¢1

Total amount receivable by Banda Ltd


Debenture 48,000
Unsecured credit (40% of GH¢240,000) 96,000
144,000

Statement of Financial Position after Reconstruction of Mahadi Ltd


Non-Current Assets
Freehold property 68,000
Equipment [468,000 – 108,000] 360,000
428,000

Inventory 120,000
Cash 140,000
Total Assets 688,000

Stated Capital 400,000


Capital surplus (68,000 – 20,000) 48,000
448,000
Sundry payables (412,000 – 240,000) 172,000
Bank overdraft 68,000
688,000

NB: THE EXISTING SHARE CAPITAL HAS REDUCE BY 20,000 (100,000-80,000) which
will reduce the capital surplus by 20,000

Page | 653
Workings
Shareholding in the reconstruction entity
Number GH¢
Banda Ltd
Conversion of Debenture stocks 48,000 48,000
Conversion of trade credit 132,000 132,000
New subscription 140,000 140,000
320,000 320,000
Existing Shareholders 1/5 x 400,000 80,000 80,000
400,000 400,000

Example: External reconstruction


X Ltd’s statement of financial position is as follows: GH¢000
Assets 2,200
Share capital 1,500
Accumulated loss (300)
Preference share capital 200
10% debenture 500
Bank overdraft 300
2,200
The company has proposed the following scheme of reconstruction.
1. Assets are to be revalued to GH¢1,620,000.
2. A new company is to be set up (X (Newco) Ltd) with initial share capital subscribed by the
existing shareholders of X Ltd for cash consideration of GH¢750,000.
3. X (Newco) Ltd to purchase the trade and assets of X Ltd and take responsibility for the bank
overdraft for a consideration of GH¢900,000 made up of new capital instruments in X
(Newco) Ltd as follows:
New capital in X (Newco) Ltd Amount
Ordinary share capital GH¢500,000
Preference share capital GH¢150,000
Debentures GH¢250,000
4. X Ltd will distribute the purchase consideration received from X (Newco) Ltd as follows:
Ordinary share capital (GH¢500,000) GH¢125,000 to the debenture holders
GH¢375,000 to the ordinary shareholders
Preference share capital (GH¢150,000) To preference shareholders
Debentures (GH¢250,000) To the debenture holders
Required
a) Prepare the statement of financial position for X (Newco) immediately after the
reconstruction.
b) Prepare T accounts to reflect the winding up of X Ltd.

Page | 654
REFERENCES
 ACCA. (2015). Study Text on corporate reporting (Paper P5). Kaplan Publishing Inc.
 Addo. A (2014). Advanced financial reporting. Black Mask Ltd
 Advanced Accounting,Institute of Chartered Acccounts(India)
 Corporate Reporting,Association of Chartered Certified Accountants(ACCA),Emile
Woolf Publishing International-September 2010-British Library (UK)
 Corporate Reporting,Certified Public Accountants(CPA)-Martin Kelly,Queens
University,Belfast
 Elliot B and Elliot J. (2008), Financial Reporting and Analysis, 12th edn, FT Prentice Hall.
 Financial Accounting & Reporting , Barry Elliott and Jamie Elliott-Pearson Education
 Financial Reporting Question Bank (ICAG)
 Financial Reporting,Association of Chartered Certified Accountants(ACCA),Emile
Woolf Publishing International-January 2013-British Library (UK)
 Financial Reporting,BPP Learning Media(Study text & Exam Kit)
 ICAG (2015). Study text on financial reporting. London: BPP Learning Media.
 ICAG (2019). Study text on Corporate reporting.Berkshire:Emile Woolf International
1. ICAG (2019). Study text on financial reporting. Berkshire: Emile Woolf International.

 International Accounting Standards Board. (2015). International financial reporting


standards (IFRSs). London: IASB.
 Internet sources
 Kirk, R. J. (2005). International financial reporting standards in depth: Theory and
practice (Vol. 1). London: Elsevier Butterworth Heinemann.
 Lewis, R.,& Pendrill, D. (2004). Advanced financial accounting. Pearson Education
 Milichamp, RA (2007), Foundation accountting. DP Publication, London
 Past Questions and Answers (Association of Chartered Certified Accountants-U.K)
 Past Questions and Answers (Institute of Chartered Accountants-Ghana)
 Rees, B (1995), Financial Analysis, 2nd edn, FT Prentice Hall.
 Wood, F and Slanger (2012). Business Accounting 2, 12th edn, Financial times

Page | 655

You might also like